You are on page 1of 778

PVL3703 20th May 2023 Exam

QUESTION 1

In legal contexts, conduct is generally understood as a voluntary human act or


omission. The term "voluntary" refers to the ability of an individual to control their
muscular movements through their will. This means that the person must have the
capacity to consciously direct their actions or choose to refrain from acting.

Voluntariness is an essential element in establishing delictual liability, which is


liability for wrongful acts or omissions that cause harm to another person. The
principle of voluntariness requires that the wrongdoer's act or omission be the result
of their own free will and intentional choice.

The concept of voluntariness is crucial in differentiating between actions that can be


attributed to an individual's liability and those that cannot. Acts that are involuntary,
such as those performed under compulsion, during unconsciousness, or due to
reflex movements, may not be considered voluntary for the purposes of establishing
delictual liability.

While there may not be a specific case or statute in South African law that directly
defines conduct as a voluntary human act or omission, the understanding of
voluntariness is a fundamental principle underlying the assessment of liability for
wrongful conduct.

John's liability for the collision of the two airplanes would depend on whether his
actions were voluntary or involuntary, and whether he intentionally placed himself in
a mechanical state or was negligent in not taking his prescribed blood pressure
medication.

According to the legal principle of conduct, a voluntary human act or omission is


required for delictual liability. Automatism, a defense that asserts the absence of
voluntary action, can be raised to argue that the defendant did not act according to
the law. Various conditions, such as unconsciousness, sleep, and mental disease,
can render a person incapable of controlling their bodily movements involuntarily.

In the case of Molefe v Mahaeng, the defendant was not required to prove "sane
automatism," and the burden of proof rested on the plaintiff to demonstrate voluntary
action. Similarly, in the Du Plessis case, the defendant was found not guilty of
negligent driving when a blackout caused by low blood pressure led to an accident.1

The case of Molefe v Mahaeng is an important case that clarifies the burden of proof
in establishing voluntary action and the defense of automatism.2 In Molefe v
Mahaeng, the court ruled that the defendant does not bear the onus to prove a state
of "sane automatism." Rather, it is the plaintiff's responsibility to demonstrate that the
defendant acted voluntarily. This case reinforces the principle that the plaintiff must
establish the voluntary nature of the defendant's conduct in delictual liability cases.

Applying these principles to the given facts, it can be concluded that John did not act
voluntarily when the collision occurred because he was unconscious and unable to
give the pilots the correct instructions. However, if John had been receiving medical
treatment for low blood pressure but failed to take his prescribed medication on that
particular occasion, he cannot rely on automatism. The defense of automatism fails
when the defendant intentionally creates a situation in which they act involuntarily to
harm another person. Therefore, John would be held liable for his culpable conduct
in creating the state of automatism that resulted in the collision.

Automatism does not imply the absence of any voluntary act whatsoever. It refers to
a state where the conduct in question is not within the control of the individual due to
factors such as sleep, unconsciousness, reflex movements, or certain medical
conditions. While the actions may not be consciously willed or controlled at the time,
there may have been prior voluntary acts or choices that led to the situation where
automatism arises.

Automatism recognizes that even though a person may not have voluntary control
over their actions in a particular moment, there may have been voluntary actions or
choices that contributed to their condition. The focus is on the voluntariness of the
specific act or omission that caused the harm, rather than considering prior voluntary
acts.

It's important to note that the defense of automatism may not be successful if the
person intentionally placed themselves in a state of automatism or if they were

1 Molefe v Mahaeng, [2014] ZAGPJHC 15; 2014 (5) SA 338 (GP).


2 Molefe v Mahaeng, [2014] ZAGPJHC 15; 2014 (5) SA 338 (GP).
negligent in their actions leading to that state. In such cases, the individual may still
be held liable for their culpable conduct.

If a person negligently places themselves in a mechanical state where automatism


arises, such as by failing to take prescribed medication, they cannot rely on
automatism as a defense. Negligently failing to take necessary medication
demonstrates a lack of reasonable care and may be considered a negligent act. In
such cases, the person may be held liable for their negligence and the resulting
consequences.

Furthermore, in certain circumstances, such as when there is a known medical


condition and prescribed medication, the failure to take the necessary medication
may even indicate intention in the form of dolus eventualis. Dolus eventualis refers to
a situation where a person foresees the potential harm that may result from their
actions or omissions but proceeds with the conduct regardless.

Moreover, if John negligently failed to take his medication, his reliance on


automatism would also fail. Negligently placing oneself in a mechanical state
precludes the defense of automatism. In this case, John's negligence in not taking
his prescribed blood pressure medication could potentially establish liability or even
indicate intention (dolus eventualis). Consequently, a defense of automatism would
not succeed in this scenario. Therefore, in the case of John, his failure to take his
prescribed blood pressure medication, if it can be established as negligent or
indicative of dolus eventualis, would undermine any reliance on automatism as a
defense. Automatism would not absolve John of liability if he negligently or
intentionally placed himself in a mechanical state where he could not control his
actions or omissions.

To summarize, John's liability in the collision of the airplanes hinges on the


voluntariness of his actions and his failure to take prescribed medication.
Automatism is not applicable if John intentionally created the involuntary state or was
negligent in his conduct. Hence, reliance on automatism would not be successful in
this case.

QUESTION 2
The question at hand pertains to the wrongfulness of an omission by the police and
prosecutors in failing to protect Anna against Chad's assault. Wrongfulness in the
context of an omission arises when the defendant has a legal duty to act positively to
prevent harm suffered by the plaintiff. The determination of wrongfulness revolves
around the existence and breach of a legal duty to act. Generally, a person is not
considered to have acted wrongfully in the law of delict when they fail to prevent
harm to another person. However, liability requires the presence of wrongful and
unreasonable prejudice caused by an act. The boni mores test, reflecting the legal
convictions of the community, serves as the fundamental standard for assessing
wrongfulness. Factors that indicate the presence of a legal duty include prior
conduct, control of a dangerous object, rules of law, a special relationship between
the parties, particular office, contractual undertakings, and creating an impression of
protecting the interests of a third person.

The principle of prior conduct plays a significant role in evaluating wrongfulness. A


person can be deemed prima facie wrongful when they create a new source of
danger and fail to eliminate it, thereby causing harm to another individual. While prior
conduct is not a prerequisite for the establishment of a legal duty, historically, liability
for omissions was primarily based on the prior-conduct rule. The cases of Halliwell v
Johannesburg Municipality3 and Silva's Fishing Corporation v Maweza4 exemplify
the application of the prior-conduct rule in holding defendants liable for omissions.

However, the case of Regal v African Superslate marked a shift in the prominence of
the prior-conduct rule as the sole basis for establishing liability for an omission.5 The
Appellate Division recognized that the prior-conduct rule and the rule related to the
control of dangerous property are both grounds for establishing the wrongfulness of
an omission. This principle was reinforced in the case of Minister of Forestry v
Quathlamba,6 where the failure to control or contain a fire that spread to a
neighboring property resulted in a finding of wrongfulness.

An extension of the rules governing wrongfulness in omissions is evident in the case


of Minister van Polisie v Ewels. In this case,7 the court held that the prior-conduct

3 Halliwell v Johannesburg Municipality 1912 AD 659.


4 Silva's Fishing Corporation (Pty) Ltd v Maweza 1957 (2) SA 256 (A).
5 Regal v African Superslate (Pty) Ltd 1963 (1) SA 102 (A).
6 Minister of Forestry v Quathlamba 1973 (3) SA 69 (A).
7 Minister van Polisie v Ewels 1975 3 (SA) 590 (A).
rule and the rule related to the control of dangerous property are merely two of
several factors to consider when determining wrongfulness in omissions. Factors
such as the legal duty placed on the police to protect citizens, the special
relationship between police officers and the public, and the authority or control that
passive police officers possess over their assaultive colleague were taken into
account in finding wrongfulness. In the case of Carmichele v Minister of Safety and
Security,8 the Constitutional Court emphasized that the boni mores should be
informed by the values enshrined in the Bill of Rights.9

Minister van Polisie v Ewels, is indeed a relevant case in discussing the extension of
the rules related to the wrongfulness of omissions. In this case, the court dealt with a
situation where police officers passively stood by and watched as a fellow police
officer brutally assaulted a civilian.

The court in Minister van Polisie v Ewels recognized that the prior-conduct rule and
the control of dangerous property rule, as shown in previous cases, are not the
exclusive criteria for determining wrongfulness in omissions. Instead, the court
emphasized that these rules are just two of several factors that can be considered
when assessing the wrongfulness of an omission.

In Minister van Polisie v Ewels,10 the court identified additional factors to consider in
determining wrongfulness, such as the legal duty placed on the police to protect
citizens from harm, the special relationship between police officers and the public,
and the fact that the passive police officers were in a position to exercise authority or
control over their assaultive colleague. These factors weighed in favor of a finding of
wrongfulness of the omission by the passive police officers.

The case highlights the importance of considering various factors beyond prior
conduct and control of dangerous property in assessing the wrongfulness of
omissions. It underscores the need to evaluate the specific circumstances of each
case and take into account the legal duties, relationships, and authority involved
when determining whether the omission was wrongful.

8 Carmichele v Minister of Safety and Security, (Centre for Applied Legal Studies Intervening) 2001
9 Constitution of South Africa, 1996
10 Minister van Polisie v Ewels, 1975 3 (SA) 590 (A).
Applying these principles to the given facts, it can be concluded that the omission by
the police and prosecutors was indeed wrongful. The police and prosecutors had a
legal duty, based on the legal convictions of the community, to prevent the assault
on Anna. Therefore, they can be deemed to have acted wrongfully.

In the South African law of delict, when assessing wrongfulness, a dual investigation
is necessary to determine whether there has been an infringement of a subjective
right. This investigation involves two key elements:

1. Factual infringement: The first aspect of the investigation is to establish


whether the subject-object relationship, referring to the relationship between
the holder of a right and the object of that right, has actually been disturbed.
This involves examining the factual circumstances to determine whether there
has been an actual interference with the plaintiff's subjective right.

2. Norm-violation: The second aspect of the investigation focuses on whether


the infringement, if it indeed occurred, was in a legally reprehensible or
wrongful manner. This element examines whether the defendant's actions or
omissions violated a legally recognized norm or principle. It assesses whether
the defendant's conduct fell short of the expected legal standards or
obligations, resulting in a breach of the plaintiff's rights.

By conducting this dual investigation, the court examines both the factual aspect of
the infringement and the legal aspect of the norm-violation. It considers whether the
subject-object relationship was actually disturbed and whether that disturbance was
legally reprehensible, according to the applicable legal standards and norms.

This dual investigation ensures a comprehensive analysis of the wrongfulness of the


defendant's actions or omissions, taking into account both the factual and normative
aspects of the infringement of a subjective right.

The following factors can serve as indications that a legal duty rested on the
defendant in a given situation:

1. Prior conduct (omissio per commissionem): If the defendant's prior actions or


omissions have created a new source of danger and they fail to eliminate that
danger, resulting in harm to another person, a legal duty may arise.
2. Control of a dangerous object: When the defendant has control over a
dangerous object or instrumentality, they may have a legal duty to prevent
harm that could result from its use or misuse.

3. Rules of law: The existence of specific legal rules or statutes that impose a
duty on the defendant to act or refrain from acting can establish a legal duty.

4. Special relationship between the parties: Certain relationships, such as those


of a parent and child, doctor and patient, or employer and employee, may give
rise to a legal duty to protect the other party from harm.

5. Particular office: If the defendant holds a specific office or position that comes
with legal responsibilities to act in certain ways to prevent harm, a legal duty
may be imposed.

6. Contractual undertaking for the safety of a third party: When the defendant
has voluntarily undertaken a contractual obligation to ensure the safety or
protection of a third party, they may be legally obligated to fulfill that duty.

7. Creating an impression of protecting a third person's interests: If the


defendant creates an impression, through their words, actions, or conduct,
that they will protect the interests or safety of a third person, they may be
under a legal duty to fulfill that expectation.

These factors are considered in determining whether a legal duty existed and
whether there was a breach of that duty, which is a crucial element in assessing the
wrongfulness of an omission and establishing liability in delictual claims.

QUESTION 3

The excerpt from Brand AJ's statement in Le Roux v Dey 201111 emphasizes the
importance of the reasonableness criterion and considerations of public and legal
policy in determining wrongfulness in the South African law of delict. This excerpt
highlights two key aspects:

Reasonableness and Judicial Determination:

11 Le Roux v Dey, 2011 3 SA 274 (CC) 315


The reasonableness of imposing liability on a defendant for damages resulting from
specific conduct is a crucial factor in determining wrongfulness. This determination is
made by the court, assuming all other elements of delictual liability are present. The
court evaluates whether it is reasonable, based on the legal convictions of the
community and considering all the circumstances, to hold the defendant liable for the
harm caused. It recognizes that wrongfulness is not an absolute standard but
depends on the reasonableness of imposing liability in a given case.

Considerations of Public and Legal Policy:

In assessing reasonableness and wrongfulness, the court takes into account


considerations of public and legal policy in accordance with constitutional norms.
This means that the court considers societal expectations, prevailing legal principles,
and constitutional values when evaluating the reasonableness of imposing liability.
The excerpt acknowledges that the determination of wrongfulness is not solely
based on traditional legal principles but also encompasses broader policy
considerations aligned with constitutional norms.

The criterion for the test of negligence in the South African law of delict involves
assessing the conduct of the defendant against the standard of a reasonable person
in their position. This criterion consists of two key elements:

(a) Foreseeability of Harm: The reasonable person, placed in the same position as
the defendant, is expected to foresee the reasonable possibility that their conduct
could cause injury to another person's body or property, resulting in financial loss
(patrimonial loss). The test focuses on whether a reasonable person would have
anticipated the potential harm arising from their actions or omissions.

(b) Reasonable Steps to Prevent Harm: The reasonable person, being aware of the
foreseeable risk, is further expected to take reasonable steps to guard against such
harm. This means that the defendant should exercise the level of care, caution, and
precaution that a hypothetical reasonable person would have taken in similar
circumstances to prevent the harm from occurring. The test evaluates whether the
defendant's actions or omissions fell short of what a reasonable person would have
done to avoid causing harm.
In a negligence claim, if the defendant's conduct fails to meet the standard of the
reasonable person test, meaning they did not foresee the reasonable possibility of
harm or did not take reasonable steps to prevent it, they may be held liable for
negligence.

The criterion for negligence serves as a yardstick for assessing the reasonableness
of the defendant's behavior, ensuring that individuals exercise a certain level of care
and responsibility towards others. By applying this standard, the law aims to provide
protection and compensation to those who suffer harm due to the negligent actions
or omissions of others.

The importance of this excerpt lies in its recognition of the dynamic nature of
wrongfulness and the need to balance legal principles with societal and constitutional
values. It underscores the evolving nature of the law of delict in South Africa, which
considers the context-specific reasonableness and policy considerations in
determining wrongfulness.

QUESTION 4

To determine whether there was fault on John's part in the given scenario, we need
to assess the wrongfulness of his actions and consider the defense of necessity. The
analysis can be presented as follows:

• Test for wrongfulness:

• Was there an infringement of a legally protected interest? The act must have
harmful consequences. Here, John's actions resulted in damage to Tom's car,
which infringed upon Tom's real right to his property.

• If the interest has been prejudiced, legal norms are used to determine if it
occurred in a legally reprehensible manner (unreasonable manner) based on the
boni mores test. In this case, John's conduct was wrongful as it infringed upon
Tom's property right and occurred in an unreasonable manner.

• Defense of necessity:

• John may raise the defense of necessity as a ground of justification. Necessity


arises when the defendant is compelled by superior force (vis major) to
reasonably violate the interests of an innocent third party in order to protect their
own interests or the interests of someone else.

• Guidelines to determine necessity include:

• The existence of an actual state of necessity.

• Objective determination of the possible existence of a state of necessity.

• The state of necessity must be present or imminent.

• The defendant can protect the interests of another, not just their own.

• Various interests, including property, can be protected out of necessity.

• Necessity cannot be relied upon if the person is legally compelled to endure


danger.

• The interests sacrificed must be more valuable than the interest being
protected.

• The act of necessity must be the only reasonable possible means of


escaping the danger.

In applying the above analysis to the given facts, it can be concluded that John's
actions were wrongful, as they resulted in harm to Tom's property in an
unreasonable manner. However, John may potentially succeed in raising the
defense of necessity if he can establish that his actions were necessary to protect
his own interests or the interests of another and met the requirements outlined for
the defense.

QUESTION 5

From the facts provided, it can be established that the defendant, Bright Future High
School, was negligent in their omission to guard against the poles injuring the
schoolboys. However, it is important to consider whether the plaintiff, Tim, was also
negligent and whether contributory negligence can be raised as a defense by the
school.

Contributory negligence is a defense available to the defendant when the plaintiff's


own negligence contributed to the harm suffered. In South Africa, the Apportionment
of Damages Act 34 of 1956 applies in cases of contributory negligence.12 This Act
provides for the apportionment of damages between the parties based on their
respective degrees of negligence.13 The court assesses the deviation from the
reasonable person standard by both the defendant and the plaintiff and expresses
this deviation as percentages for the purpose of apportionment.

The percentages of negligence attributed to the defendant and the plaintiff will
always add up to 100% according to the Ltd v. Smit 1962 3 SA 826 (A)14 and
Association Ltd v Nomeka 1976(3) SA 45 (A) 52E-G cases. In Jones NO v Santam
Bpk 1965, it was established that the percentages must be assessed independently,
allowing for scenarios where the defendant may be, for example, 80% negligent
while the plaintiff is 30% negligent.15 This approach, as preferred by Neethling and
Potgieter, can be reconciled with the approach in Jones.

However, it is worth noting that the defense of contributory negligence is applicable


to negligence in respect of the damage itself, rather than negligence in respect of the
event causing the damage. In Bowkers Park Komga Cooperative Ltd v SAR and H
1980, the court clarified that contributory negligence refers to negligence in respect
of the damage suffered by the plaintiff. This was further confirmed by the Appellate
Division in Union National South British Insurance Co Ltd v Vitoria 198216 and
General Accident Versekeringsmaatskappy SA Bpk v Uijs 1993.17 Therefore, if Tim's
failure to heed the warnings and sit on the pole contributed to his own injury, it could
constitute contributory negligence.

If the defense of contributory negligence is successfully raised by the school, the


effect would be the apportionment of damages between the parties based on their
respective degrees of negligence. The court will determine the percentages of
negligence for both the defendant and the plaintiff and use these percentages as a
basis for the apportionment of damages.

12 The Apportionment of Damages Act 34 of 1956.


13 Damages Act.
14 Ltd v. Smit 1962 3 SA 826 (A)
15 Jones NO v Santam Bpk 1965 (2) SA 542 (A)
16 Union National South British Insurance Co Ltd v Vitoria 1982.
17 General Accident Versekeringsmaatskappy SA Bpk v Uijs 1993.
QUESTION 6

The legal test for establishing causation is known as the flexible approach, as
outlined in the cases of S v Mokgethi and Others (16/1989) [1989] ZASCA 105;
[1990] 1 All SA 320 (A)18 and International Shipping Co (Pty) Ltd v Bentley 1990.19 In
the case of Mokgethi and others, the court examined a situation where a bank teller
was shot by a bank robber and later developed complications and died due to
pressure sores.20 The court emphasized that the key question in determining legal
causation is whether there is a sufficiently close relationship between the defendant's
conduct and the resulting harm, taking into account considerations of
reasonableness, fairness, and justice.

While there are other theories of legal causation, such as adequate causation, direct
consequences, foreseeability, and novus actus interveniens, none of these criteria
alone can be universally applied. Instead, they can serve as supplementary factors
to support the application of the flexible approach. In the Mokgethi case, the court
ultimately concluded that the shot fired by the bank robber was not the legal cause of
the teller's death.

Application to the Facts:

Applying the principles of legal causation to the facts presented in the question, it is
likely that Roy's pneumonia would be considered too remote to be imputed to the
wrongdoer, Rob. Additionally, the nurse's act of leaving the windows of the ward
open could be seen as a new intervening act, known as novus actus interveniens.
This further supports the conclusion that there is no legal causal link between Rob's
conduct of throwing the apple and Roy contracting pneumonia.

QUESTION 7.1

Mitigation of loss refers to a principle in the law of delict where a plaintiff is required
to take reasonable steps to prevent or minimize the damages resulting from the
defendant's conduct. It means that the plaintiff cannot claim damages for a loss that
could have been avoided or reduced if they had taken reasonable measures to
mitigate the harm caused by the defendant's actions. The plaintiff has a duty to act

18 S v Mokgethi and Others (16/1989) [1989] ZASCA 105; [1990] 1 All SA 320 (A).
19 International Shipping Co (Pty) Ltd v Bentley 1990.
20 S v Mokgethi and Others (16/1989) [1989] ZASCA 105; [1990] 1 All SA 320 (A).
reasonably and make efforts to mitigate their losses to the extent possible. Failure to
do so may impact the amount of damages they can recover in a legal claim.

QUESTION 7.2

1. Defamation: Defamation refers to the act of making false statements about


someone that harm their reputation. It involves the communication of false
information about a person to others, leading to damage to their character,
reputation, or standing in the community.

2. Invasion of privacy: Invasion of privacy occurs when an individual's personal


information, private affairs, or intimate details of their life are intruded upon
without their consent. This can include unauthorized surveillance, public
disclosure of private information, or intrusion into a person's private space.

3. Intentional infliction of emotional distress: This form of iniuria involves


intentionally causing severe emotional or mental distress to another person
through extreme and outrageous behavior. It includes actions or statements
that go beyond what is considered acceptable in society and causes
significant emotional harm to the victim.

QUESTION 7.3

To successfully rely on the actio de pauperie, the following four requirements must
be met:

1. The defendant must be the owner of the animal when the damage is inflicted.
This means that the person who has control and possession of the animal at
the time of the incident will be held liable.

2. The animal must be a domestic animal. Domestic animals are those typically
kept by humans for various purposes, such as pets or livestock. This excludes
wild animals, which are subject to different rules and liabilities.

3. The animal must act contrary to its own nature when inflicting damage. This
means that the animal's behavior must deviate from what is expected or
considered normal for its species. Generally, an animal is not acting contrary
to its nature if it is simply reacting to external stimuli. However, certain
defenses may apply, such as vis major (irresistible force), culpable conduct on
the part of the injured person, or provocation.

4. The prejudiced person or their property must be lawfully present at the


location when the damage is inflicted. This requirement ensures that the
person seeking recourse under the actio de pauperie was not trespassing or
unlawfully present in the area where the incident occurred.

By fulfilling these requirements, Cecile may be able to bring a successful delictual


action under the actio de pauperie against Brad for the injuries caused by his dog.
BIBLIOGRAPHY

Books:

Neethling, J., Potgieter, J. M., & Visser, P. J. (2015). Law of Delict. LexisNexis South
Africa.

Van der Walt, A. J. (2011). Introduction to the Law of South Africa. Kluwer Law
International.

Joubert, J. J. (2012). The Law of South Africa: Volume 9 (2nd ed.). LexisNexis South
Africa.

Van der Merwe, C. G., Rowland, J. M., & Van Niekerk, D. H. (2012). Van der Merwe
and Rowland: The Law of Damages (3rd ed.). Juta Law.

Legislations:

Constitution of South Africa, 1996

The Apportionment of Damages Act 34 of 1956.

Cases:

Molefe v Mahaeng, [2014] ZAGPJHC 15; 2014 (5) SA 338 (GP).

Minister van Polisie v Ewels, 1975 3 (SA) 590 (A).

Le Roux v Dey, 2011 3 SA 274 (CC) 315

Carmichele v Minister of Safety and Security, 2001 (1) SA 489 (SCA)

Jones NO v Santam Bpk 1965 (2) SA 542 (A)Ltd v.

Smit 1962 3 SA 826 (A)

Association Ltd v Nomeka 1976(3) SA 45 (A) 52E-G

King Insurance NO v Pearl Insurance Co Ltd 1970(1) SA 462 (W)

Bowkers Park Komga Cooperative Ltd v SAR and H 1980


Union National South British Insurance Co Ltd v Vitoria 1982

General Accident Versekeringsmaatskappy SA Bpk v Uijs 1993


QUESTION 01

Negligence is the blameworthy attitude of someone that had acted wrongfully and is
blamed for conduct which is careless; by giving insufficient attention to his actions and
failing to adhere to the standard of care legally required of him. The defendant is
negligent if the reasonable person would have acted differently, if the unlawful causing
of damage was reasonably foreseeable and reasonably preventable.
The test of negligence was formulated in Kruger v Coetzee and is also known as
reasonable person test. The test follows three steps. A reasonable in the same position
as defendant would have:
(a) Foreseen the possibility of his conduct injuring another and causing him patrimonial
loss.
(b) Would take reasonable steps to prevent such occurrence
(c) And the defendant failed to take such steps.
John has foreseen the possibility that not taking his medication may cause him to have
black outs while on duty, but nevertheless he took a risk not to take his medication to
prevent such occurrence of having black outs at work and to prevent harm from
occurring, therefore John’s conduct was negligent.

QUESTION 02

An act can usually be wrongful if it has some consequence. The determination of


wrongfulness entails a dual investigation. Firstly it has to be ascertained whether the
perpetrator’s act was in fact the cause of a harmful result to another person and
secondly it must be ascertained whether the causing of harm took place in an
unreasonable or legally reprehensible way.
The question deals with wrongfulness of an omission. An omission is wrongful if the
defendant is under the legal duty to act positively to prevent harm suffered by the
plaintiff. The basic question to determine whether an omission is wrongful is whether
legal duty to act was present and was breached. This is determined with reference to
the legal convictions of the community (boni mores). Factors which may serve as
indications that a legal duty rested on the defendant include: prior conduct (omissio per
2|Page
commissio); control of a dangerous object; rules of law; a special relationship between
the parties; particular office; contractual undertaking for the safety of a third party; and
creating of an impression that the interests of a third person will be protected. If a
person’s conduct is contra boni mores that is to say if its contrary to the legal
convictions of the community, then that person would have acted in a wrongful manner.
The court follows an objective test based on reasonableness, based on legal
convictions of community and on all circumstances. The question asked is: Did the
defendant infringe interest of plaintiff reasonably or unreasonably? Court must weigh
conflicting interests in light of circumstances.

In the so-called municipality cases, prior conduct was considered to a prerequisite for
the wrongfulness of a commission. Prior conduct refers to the creation of a new source
of danger and failing to eliminate it.
In Halliwell v JHB municipality a wagon slipped on cobblestones and court held that a
legal duty that arises from prior conduct was needed.
In Minister van Polisie v Ewels the court held that the existence of a legal duty is
determined by boni mores and prior conduct is only an indication thereof. Not a
prerequisite.
In the Carmichele case the court held that the State was delictually liable for damages
arising out of the unlawful omissions of its servants. The Constitutional court made it
clear that boni mores must be informed by values underpinning the Bill of Rights.
A defendant is under a legal duty to act positively to prevent harm to the plaintiff if it is
reasonable to expect of the defendant to have taken positive measures to prevent the
harm. In the scenario in question the Shop-till-you-drop shopping mall the management
did not take all the necessary measures required to warn third parties of the puddle of
sanitizers. They should have put a sign indicating there’s puddle of water or mop the
sanitizers. Their prior conduct had imputed a legal duty on them to notify any third
parties of the harm an omission to do such constituted an act of wrongfulness. If the
principles of the Carmichele case are followed then the conduct of the Shop-Till-You-
Drop management was wrongful.

QUESTION 03

From the given facts, we can conclude that the defendant (Steady Wheels) has been
negligent, but the plaintiff (Mike) appears to have been negligent too. Thus we must
consider whether contributory negligence was present.

3|Page
Contributory negligence is negligence on the part of the plaintiff, and it is a defense that
the defendant can raise. The Apportionment of Damages Act 34 of 1956 is applicable.
This Act provides that a contributorily negligent plaintiff’s damages be apportioned. The
court will determine the degree of deviation from the reasonable person standard shown
by the conduct of both the defendant and the plaintiff, express the deviation as
percentages, and use these percentages as a basis for the apportionment.
According to the Smit and Nomeka cases, the percentages of negligence attributed to
the defendant and plaintiff respectively will always add up to a hundred per cent.
According to Jones NO v Santam Bpk 1965 case, both percentages must be assessed
independently, which could mean that, for example, a defendant may be 80% negligent
while the plaintiff is 30% negligent.
According to King v Pearl Insurance case a defense of contributory negligence could
not succeed where the plaintiff had omitted to wear a crash-helmet while driving a
scooter, but had not been negligent in respect of causing the accident. However, in
Bowkers Park Komga Cooperative Ltd v SAR and H case,the court held that
contributory negligence did not refer to negligence in respect of the damage-causing
event, such as a motorcar accident, but to negligence in respect of the damage itself,
and this was confirmed by the Appellate Division in Union National South British
Insurance Co Ltd v Vitoria 1982 and General Accident Versekeringsmaatskappy SA
Bpk v Uijs 1993 cases. Therefore, failure to wear a safety helmet would constitute
contributory negligence if it contributed to the plaintiff’s damage.
Applying these principles to the facts, we can conclude that Mike was contributorily
negligent. Steady wheels will not be liable for damages incurred. Mike’s claim will be
instituted against Steady wheels instead of David.

QUESTION 04

(a) Factual causation is the conditio sine qua non test, or ‘‘but for test’. Mentally
eliminating, or thinking away, the conduct. If the damage then also
disappears, a factual causal link is present between the conduct and the
damage. The condition sine qua non test is used to determine whether there
was factual causal link between two parties.
If the test is applied to the facts, we must conclude that if Tom had not did
not knocked the ladder that William was standing on, William would not have
fell and injured his leg and needed to go to the hospital and broke his leg after
being discharged from hospital, therefore a factual causal link is present
between Bargain buy’s conduct and Alex’s damage.

4|Page
(b) The test for legal causation is the so-called flexible approach, as formulated in
S v Mokgethi and International Shipping Co (Pty) Ltd v Bentley case. In
Mokgethi a bank robber shot a teller. The teller was rendered a paraplegic
and was discharged from hospital in a wheelchair. Subsequently, the
paraplegic man failed to shift his body position in the chair frequently and
developed pressure sores, eventually dying from complications. The question
that arose was whether the shot fired by the robber was the legal cause of the
teller’s death.

According to the court, the main question in respect of legal causation is


whether there is a close enough relationship between the wrongdoer’s
conduct and its consequence for such consequence to be imputed to the
wrongdoer in view of policy considerations based on reasonableness,
fairness and justice. Several other legal causation theories exist, such as
adequate causation, direct consequences, foreseeability and novus actus
interveniens. None of these criteria is suitable to be applied to all situations.
They may, however, be used as subsidiary aids when employing the flexible
approach. In the Mokgethi case, the court held that the shot was not a legal
cause of the death.
Applying the principles to the facts in the question, the conclusion is probably
that William broken leg was too remote and should not be imputed to the
wrongdoer. It could also be argued that a so-called novus actus interveniens,
that is, a new intervening act, was constituted by the fall on William’s conduct.
QUESTION 05

Necesseity will be suitable ground of justification in the scenario


Ben will be able to raise necessity as a defence against Connie. Necessity must be
clearly distinguished from private defence. The distinction is that when acting in defence
the actor’s conduct is directed at an attack by the wrongdoer. When acting out of
necessity, his conduct violates the interests of an innocent party. The fact that Jenna
incited the dog means that the act was actually a human act by her using the dog as an
instrument, however, the dog is a legal object of Connie and therefore by killing the dog
Ben violated Connie’s (an innocent party) subjective right to his dog. Ben will still be
able to raise a defence of necessity.

QUESTION 06

Actio de pauperize action will be available to Jane. To be successful with


the actio de pauperie against the owner of a domestic animal that injured or
harmed a person, it has to be establish that the animal acted ferociously or
contrary to its nature and that the conduct of the animal caused the damages
suffered by the person.

To succeed in bringing the actio de pauperie, the following requirements must be


met:
(a) ) The defendant must be the owner of the animal when the damage is
inflicted.( Mere control over the animal is insufficient for the purpose of the
action)
(b) The animal must be domesticated animal.
(c) The animal must act contra naturam sui generis when inflicting the damage.
(d) ) The prejudiced person or his property must be lawfully present at the
location where the damage is inflicted
QUESTION 07

(a) Provocation
(b) Unjustified Enrichment
(c) Private Defence

QUESTION 08

(a) Assault
(b) Invasion of Privacy

Powered by
Page 1 of 50
1
LAW OF DELICT

1. Define a delict
The act of a person which in a wrongful and culpable way causes loss/damage

2. List the 5 elements of a delict


- Act
- Wrongfulness
- Fault
- Causation
- Damage

3. Name the most important delictual remedies available, and briefly indicate what are the differences
between them

I. Actio legis Aquiliae: Claim damages for wrongful and culpable causing of patrimonial damage
II.
III. Actio iniuriarum: Claim satisfaction for wrongful and intentional injury to personality.
IV.
V. Action for pain and suffering: Claim compensation for wrongful and culpable impairment of bodily or
physical-mental integrity.

za
ls s
4. Write brief notes on the differences/ similarities between a delict and a breach of contract

o.
ria al
.c
to ori

Seems the same, but a breach of contract is only constituted by the non-fulfilment by a contractual party of a
personal right or an obligation to perform.
rtu ut

Thus, the remedies are primarily directed at enforcement, fulfilment or execution of the contract.
.g T

Delictual remedies are directed at damages and not fulfilment.


w RG

Law of contract provides specific rules/remedies for breach of contract that are not applicable to a delict.
Delict=breach of duty imposed by law.

Breach of contract is the breach of a duty voluntarily assumed.


w

5. Write brief notes on the differences/ similarities between a delict and a crime
w

Distinction between private and public law.

Protection of individual interests (delict) vs protection of public interest (Crime)

Delictual remedies are compensationary, while criminal sanctions are of a penal nature, to punish
criminal for transgression against public interest.

Each delict is not necessarily a crime and vice versa.

Page | 1
Page 2 of 50
2
6. Explain in 4 or 5 sentences how Chpt. 2 of the Constitution may influence the law of delict

The Constitution is the supreme law of RSA. Chapter2 (BOR) is applicable to all law, incl Delict.
Vertical & horizontal application of Const can take place directly or indirectly.

Fundamental rights in terms of the Law of Delict:


• Right to property
• Right to life
• Right to freedom and security of person
• Right to privacy
• Right to human dignity
• Right to equality
• Right to freedom of expression
• Right to freedom of religion, belief, opinion
• Right to assembly, demonstration, picket, petition
• Right to freedom of association
• Right to freedom of trade, occupation, profession

Not every delict is necessarily a constitutional wrong.

Constitutional remedies are aimed at affirming/enforcing/protecting/vindicating fundamental rights and

za
deterring future violations of Ch2.
ls s
o.
Write brief notes on the indirect application of the BOR to the law of delict (5)
ria al
.c
to ori

Indirect application is implemented/applicable eg to open-ended/flexible delictual principles, namely:


rtu ut

• Boni mores test for wrongfulness


.g T

• Imputability test for legal causation


• Reasonable person test for negligence
w R

• Policy consideration eg reasonableness, fairness and justice


G

The act

7. Define an act
w
w

Conduct is prerequisite for delictual liability.


Conduct is a voluntary human act or omission.

8. Enumerate the requirements of an act and apply them to practical factual examples

- Where a human uses an animal as an instrument to a commission a delict, a human act is still present.

- A juristic person acts through its organs: an act performed by or at the order if or with the permission of a
director, official or servant of a juristic person in the exercise of his duties or functions in advancing or attempting
to advance the interests if the juristic person, is deemed to have been performed by the juristic person.

- Voluntariness implies that the person is question has the mental ability sufficiently to control his
muscular movements.

- Voluntariness does not mean that a person must have willed or desired his conduct.

Page | 2
Page 3 of 50
3

9. Explain the requirements of the defences of automatism and apply them to practical factual examples

- This defence suggests that the person acted mechanically.

- The following conditions may cause a person to act involuntary in hat they render him incapable of controlling his
bodily movements:

- absolute compulsion (vis absoluta- exerted by human agency or through forces of nature),
- sleep,
- unconsciousness,
- a fainting fit,
- a epileptic fit,
- serious intoxication,
- a blackout,
- reflex movements,
- strong emotional pressure,
- mental disease,
- hypnosis
- a heart attack and certain other conditions. Relative compulsion ( vis compulsive- no choice)

- The defence of automatism will not succeed if the defendant intentionally created the situation in which

za
he acts involuntarily in order to harm others.
ls s
o.
- The defendant will not be able to successfully rely of the defence of automatism where he was negligent
ria al
with regard to his automatic conduct- where the reasonable man would have foreseen the possibility of him
.c
to ori

causing harm while in a state if automatism.

- Where the automatism is not a consequence of mental illness the onus is on the plaintiff to prove that the
rtu ut

defendant acted voluntarily and therefore not mechanically


.g T

- If a defendant raises automatism resulting fro mental illness, the defendant will probably bear the onus to
w R

prove the absence of such conduct


G

- Automatism does not mean that there is no voluntary conduct whatsoever by the defendant which caused
the damage, but only that the conduct in question was not voluntary.

10. Briefly explain the difference between a commission and an omission


w
w

- Liability for an omission is generally more restricted than liability for a commission.

- Commission is an act performed while an omission is a failure to perform a certain act.

- The law is hesitant to find that there was a legal duty on someone to act positively and so to prevent
damage to another.

- Omission is a failure to take any positive step whatsoever to prevent damage to other people.

Wrongfulness

11. Describe the 2 steps involved in an inquiry into wrongfulness

Dual investigation:

Page | 3
Page 4 of 50
4
- Whether a legally recognised individual interest has in fact been encroached upon- the act
must have caused a harmful result.
- Legal norms must be used to determine whether such prejudice occurred in a legally
reprehensible or unreasonable manner.

Individual interest: an individual interest which is protected by law.


.

12. Explain the relationship between wrongfulness and a harmful result, and apply this knowledge to factual
examples

An act is only delictually wrongful if it has as its consequence the factual infringement of an individual
interest.

In delict, the wrongfulness of an act is always determined with reference to its consequence.

An act and its consequence are always separated by time and space.

If one keeps in mind that the act and its consequences are separated in time and space, it is unnecessary
to employ the nascituris fiction in order to grant a delictual action to a child who is born with defects
resulting from pre-natal injuries

za
13. Explain what is meant by the legal convictions of the community (boni mores)
ls s
o.
ria al
The bones mores test is an objective test based on the criterion of reasonableness.
.c
to ori

The basic question is whether according to the legal convictions of the community and in the light of all
the circumstances of the case, the defendant infringed the interests of the plaintiff in a reasonable or
rtu ut

unreasonable manner.
.g T

14. Name and explain 3 characteristics of the boni mores test for wrongfulness
w R

- The balancing of interests: entails the ex post facto weighing up of the interests which the
G

defendant promoted by his act and those which he infringed.

Factors influencing the balancing of interests-:

1) the nature and extent of the harm of the foreseeable or foreseen loss;
w

2) the possible value to the defendant or to society of the harmful conduct;


3) the cost and effort of steps which could have been taken to prevent the loss;
w

4) the degree of probability of the success of preventative measures;


5) the nature of the relationship between the parties;
6) the motive of the defendant and the knowledge on his part that his conduct would have caused
harm;
7) economic considerations;
8) the legal position in other countries;
9) ethical and moral issues; all well as other consideration of public interest or public policy.

The legal convictions of the community must now incorporate the constitutional values and norms
and give effect to them

- A delictual criterion: in applying the boni mores criterion in the law of delict, we are not concerned
with what the community regards as socially, morally, ethically or religiously right or wrong, but
whether the community regards a particular act or form of conduct as delictually wrongful.

- An objective criterion: “the legal convictions of the community “must be seen as the legal
convictions of the legal policy makers of the community, such as the legislature and judges.

Page | 4
Page 5 of 50
5

15. Write brief notes on the role of subjective factors in the determination of wrongfulness

Subjective factors DO NOT play a role in determining wrongfulness

In exceptional circumstances. Such as “malice”, subjective factors do play a part in determining


wrongfulness.

Malice is the improper motive of the defendant; it will render his apparently reasonable conduct wrongful.
The fact that the defendant actually knew, or subjectively foresaw that the plaintiff would suffer damages as a
result of his conduct, is taken into consideration in determining wrongfulness

Intent as a form of fault is a technical legal concept with particular requirements. Improper motive is a general
concept suggesting merely a reprehensible purpose or objective on the part of the defendant. Intent may be
present even in the absence of improper motive.

za
16. discuss, with reference to examples, the ways in which the boni mores can be applied in practice
ls s
o.
ria al
The practical application of the boni mores criterion
.c
to ori

- The fact of an actual infringement is already an indication of, or pointer to, the wrongfulness.
rtu ut

- Practical applications of the general criterion of reasonableness


.g T

- Conduct is in conflict with the legal convictions of the community- wrongful- if it infringes a subjective right or
violates a legal duty.
w RG

- There are 2 main ways in which the general boni mores or reasonableness criterion is applied as a supplementary
test for wrongfulness:

- in novel case where there is no clear legal norm or ground or justification involved; and
- for purposes of refinement, especially in assessing wrongfulness in boarder line cases
w

- If case law is silent of the liability or wrongfulness if a specific conduct, the general criterion is applied, and
w

there is room in practise for the application of general criteria, including the doctrine of subjective rights.

- The reasonable person therefore embodies or represents the legal convictions of the community.

- In cases of necessity the defendant infringes the interests of an innocent third party in order to protect his
own interest. The requirement of necessity, namely that the interest infringed should in general not be greater (or
more valuable) than the interest protected. S v Goliath

17. explain the concept “subjective right”

Universiteit van Pretoria v Tommie Meyer films (Edms) Bpk the court accepted the doctrine of subjective rights-

wrongfulness consists of the infringement of a subjective right.

The holder of a subjective right has a right to something enforceable against other people.

Page | 5
Page 6 of 50
6

18. describe how it is ascertained whether a subjective right has been infringed, and apply this knowledge to
practical examples

- There is a dual relationship that characterises every right-

- Subject-object relationship: there is a relationship between the holder of the right (the legal subject) and the
particular object of the right (the legal object);

- The subject-subject relationship: relationship between the holder of the right and all other persons (legal
subjects)

- The holding of a right confers power to use, enjoy and alienate the object of his right. The content and extent of
these powers are determined and regulated by the rules and norms of the law

za
- The nature of a subjective right: is determined by the nature of the particular object of the right. Rights are
ls s
o.
categorised and named with reference to the different types of legal objects to which the rights relate
ria al
-
.c
Because a subjective right has not yet been identified in every instance where damage is caused, it is
to ori

expedient to determine wrongfulness in those cases by inquiring whether a legal duty has not been complied with,
rather than trying to determine whether a right has been infringed
rtu ut

- Real rights- things e.g. a car, a pen


- Personality rights- aspects of personality e.g. physical integrity, hour
.g T

- Personal rights- acts and performances e.g. delivery of a thing


w R

- Immaterial property rights- immaterial e.g. poem, work of art


- Personal immaterial property rights- personal immaterial property e.g. Earning capacity,
G

creditworthiness
- Absolute rights can be enforced against all people. Whereas relative rights are enforceable against a
particular person or persons

- The existing subjective rights are not restricted.


w

Subjective rights arise when the law recognises existing individual interests as being worthy of protection
w

2 conditions must be met before the courts will recognise an individual interest as a legal object in terms
of the doctrine of subjective rights:
1) It must have value to the holder of that right

2) It must have a measure of independence that it is possible to dispose of it and to enjoy it

19. explain the relationship between legal duties and wrongfulness

In cases of liability for an omission or from the causing of pure economic loss, wrongfulness is normally
determined not by asking whether the plaintiffs subjective right has been infringed, but rather by asking
whether the defendant had a legal duty to prevent the loss.

20. explain the relationship between boni mores and the breach of legal duty

According to the boni mores criterion there is neither a general duty to prevent loss to others through
positive conduct, nor a general duty to prevent pure economic loss.

Page | 6
Page 7 of 50
7
The test for wrongfulness where breach of a legal duty is involved, is in principle clearly the same as
he question whether a subjective right has been infringed: it still involves a determination of the objective
reasonableness of the conduct of the person who acted in light if the prejudice he caused to another person

The enquiry into the existence of a legal duty and its breach is very different from the enquiries into the
so-called policy based and fact based notions of a duty of care. The question of a defendants fault or
negligence is not an issue.

21. explain the principles according to which it is determined whether an omission is wrongful or not, and
apply them to sets of facts

As a general rule a person does not act wrongfully for the purposes of delict if he omits to prevent harm to another person.

Liability follows only if the omission was in fact wrongful; and this will be the cases only if in the particular
circumstances a legal duty rested on the defendant to act positively to prevent harm from occurring, and he failed to comply
with that duty

The question whether such a duty existed is answered with reference to the flexible criterion of the legal convictions of the
community and legal policy ( Van Eeden case)

za
ls s
22. explain the factors which may be taken into account during the determination of the wrongfulness of an

o.
ria al
omission, and apply this knowledge to factual situations
.c
to ori

Objective test- all relevant circumstances of the particular case must be taken into account.
rtu ut

1) Prior conduct: a person acts prima facie wrongful when he created a new source of danger by means of a
.g T

positive conduct and consequently fails to eliminate that danger (omissio) , with the result that harm is caused
to another person.
w R

2) Prior conduct is not necessarily a prerequisite for the existence of such a legal duty. In 1912- 1957 it
G

was viewed that prior conduct was an indispensable requirement of liability for omission as a result of Halliwell
v Johannesburg Municipal Council case. It was however reject by Steyn JA in a minority decision in Silva’s
fishing Corporation (pty) Ltd v Maweza and later accepted the Appellate division in Regal v African Superslate
(pty) Ltd and eventual expressed in Minister van Polisie v Ewels
w

3) Control of a dangerous object: 2 questions- whether there was actual control and whether, in light of
such control, a legal duty rested on the defendant to take steps to prevent damage resulting from his or
w

her omission to exercise proper control.

4) Rules of law: in certain instances the law places an obligation upon a person to perform certain acts.
A flexible approach is followed- the question must still be asked whether it is equitable and reasonable to
award the plaintiff a claim for damages or not in the view of the non-compliance with a legal provision. The
conduct will ne wrongful, not due to the non-compliance with the statutory legal duty per se, but rather because
it is reasonable in the circumstances to compensate the plaintiff for the infringement of his right.

5) A special relationship between the parties: the existence of a contractual relationship may indicate such
a legal duty.. To determine whether a legal duty to prevent damage exists, each case must be measured
against the boni mores criterion in the light of all the circumstances, including the special relationship between
the parties.

6) A particular office: the office held by a person sometimes places a legal duty upon him to act in a certain
manner towards the public or specific persons

Page | 7
Page 8 of 50
8
7) A contractual undertaking for the safety of a third party: where A enters into a contract with B to take
steps to ensure the safety of C, and A then fails to take those steps and C suffers damages as a result. The
legal duty is violated and A acts wrongfully in relation to C.

8)Creation of the impression that the interests of a third party will be protected: where one party acts in
a reasonable reliance on the impression created by another party that the latter will protect the person or
property of the former, a legal duty rests upon the party creating the impression to prevent prejudice to the
party acting in reliance on that impression.

9) Interplay of factors: the duty of a policeman to prevent the assault on the plaintiff in Minister van Polisie v
Ewels may be deduced from the statutory duty to prevent crime, from the special relationship between the
policeman and citizen, as well as from the public office occupied by the policeman

10) The general wrongfulness criterion: the determination of the reasonableness of the defendant’s failure
to act in view of all the circumstances of the case. By means of the boni mores test a balancing process must
take place between, on the one hand, the interests of the defendant and, on the other hand, the interests of the
plaintiff.

23. write brief notes on the determination of the delictual wrongfulness of a non-compliance with a statutory
duty

- The causing of damage by means of conduct in breach a statutory duty is prima facie wrongful. The violation of a
norm does not in itself constitute wrongfulness; rather it is the infringement of the interests of the plaintiff in a

za
legally reprehensible manner that constitutes wrongfulness.

- ls s
In order to constitute wrongfulness in these circumstances the plaintiff must prove the following

o.
ria al
.c
1. that the relevant statutory measure provided the plaintiff with a private law remedy
to ori

2. that the plaintiff is a person for whose benefit and protection the statutory duty was imposed
rtu ut

3. that the nature if the harm and the manner in which it occurred are such as are contemplated by the
.g T

enactment
w R

4. that the defendant in fact transgressed the statutory provision


G

5. that there was a casual nexus between the transgression of the statutory provision and the harm

24. briefly describe the concept of a ground of justification with reference to an example
w

Grounds of Justification Is a special circumstance in which conduct that appears to be wrongful is


w

rendered lawful. The violation of interest is therefore not unreasonable or contra bones mores.

Practical expression of the boni mores or reasonableness criterion with reference to typical factual
circumstances that occur in practice.

Whether the actor’s violation of an individual interest was reasonable in the particular circumstances and
therefore lawful.

Eg Private defence

25. briefly indicate the connection between grounds of justification and the boni mores (legal convictions of
the community)

Because grounds of justification such as defence, consent and necessity are merely embodiments of the legal
convictions of the community, the existing grounds of justification do not constitute a numerus clausus.

Page | 8
Page 9 of 50
9
The defendant was exercising his own “right” or “power”; he acted within the confines of his own
right.

Onus to prove the existence of a ground of justification rests on the defendant

26. describe private defence with reference to an example

Private defence:

− Is present where the defendant directs his actions against another person’s actual or imminently
threatening wrongful act in order to protect his own legitimate interests or such interests of someone
else.

− Both the attack and the defensive conduct must meet certain requirements for defence to be applicable.

− Requirements for the attack:

1) The attack must consist of a human act


§ may consist of either commissio or omission

2) The attack must be wrongful; in other words it must threaten or violate a legally protected interest
without justification

za
§ The courts have recognised defence as aground of justification against attacks on the following interests;
life, bodily integrity, honour and property or possessions.
ls s
o.
ria al
The test is objective. .c
to ori

§ Putative or imagined defence does not constitute private defence; reasonable grounds for private defence
must exist objectively.
rtu ut

§ An objective test deals with the facts as they appear ex post facto
.g T

3) The attack must already have commenced or be imminently threatening, but must not yet have
w R

ceased-
§ where an attack is imminently threatening, one may act in defence even before the attack commences
G

with the intention of preventing the attack from taking place

− The following 2 considerations ARE NOT requirements for defence:



⇒ Fault on the part of the aggressor is not a requirement- one may act in defence even against someone
w

who is incapable of having a blameworthy state of mind (who can act wrongfully but not culpably)
w

rd
⇒ It is not a requirement that the attack must be directed at the defender- where the 3 party consents to
the attack one cannot legally act in his defence because in such circumstances the attack against him
is not wrongful

27. name the requirements for private defence and apply them to a given set of facts

1) The defence must be directed at the aggressor himself

2) The defence must be necessary to protect the threatened right


§ the act of defence must be the only reasonable alternative to protect the threatened interest

3) The act of defence must not be more harmful than is necessary to ward of the attack-
§ The threatened interest and the interest infringed in the process of defence need not be of equal value,
nor do the means employed by the defendant need to be similar to those of the attacker.
§ Extreme imbalance is unacceptable. In the case if extreme imbalance the fact that the attacker steps
outside the bounds of the law is therefore insufficient to render the defensive act lawful

Page | 9
Page 10 of 50
10
§ The following criterion may serve as points of departure for assessing the reasonableness or otherwise
of an act of defence:
⇒ The value of the interests may differ
⇒ The interests need not be similar in character
⇒ The means of defence employed by the defender need not be similar to those of the attacker
⇒ The surrounding circumstances will therefore be of decisive importance

28. define necessity

− A state of necessity exists when the defendant is placed in such a position by superior force that he is
able to protect his interests (or those of someone else’s ) only by reasonably violating the interests of an
rd
innocent 3 part

29. differentiate between defence and necessity

Private defence Is present where the defendant directs his actions against another person’s actual or
imminently threatening wrongful act in order to protect his own legitimate interests or such interests of
someone else.

− A state of necessity exists when the defendant is placed in such a position by superior force that he is
able to protect his interests (or those of someone else’s ) only by reasonably violating the interests of an

za
rd
innocent 3 part
ls s
o.
ria al
30. name, and apply to factual situations, the guidelines for a successful reliance on necessity
.c
to ori

a) The question at issue is whether a state of necessity really exists, not whether it has been caused by
rtu ut

human action, animals or forces of nature


.g T

⇒ Every act committed out of necessity is lawful, irrespective of whether it was caused by the defendant or
not, but damaged caused thereby may still be actionable because it is (factually and legally) casually
w R

connected to a preceding wrongful act.


G

b) A state of necessity must be determined objectively- should take into consideration the circumstances
which actually prevailed and the circumstance which actually ensued (the fact that the appellant subjectively
believed that a state of necessity existed, does not mean that it existed in fact)

c) The state of necessity must be present or imminent


w
w

d) The defendant need not only protect his own interests, but may also protect the interests of others
Not only physical integrity, but other interests such as property, may be protected out of necessity –
negotiorum gestio is present where a person attends to the interests of another without the latter’s
permission.

e) A person may not rely on necessity where he is legally compelled to endure the danger.

f) In general, the interest that is sacrificed must not be more valuable than the interest that is protected.
The defendant must not cause more harm than is necessary, the principle of commensurability
(proportionality) therefore applies in the case of necessity

g) Whether necessity may ever justify homicide


⇒ Following English law, our law initially answered in the negative (R v Dudley and Stephens)
⇒ S v Goliath the appellate division held by implication that homicide may be justified by necessity.
⇒ “it is generally accepted, even by moralists, that an ordinary human being regards his own life as more
important than that of another person”

Page | 10
Page 11 of 50
11
⇒ The law recognises the conviction of the community that a person’s own life is more important to him
than the life of another. Therefore compulsion may justify homicide
⇒ Minority judgement in Goliath case was that necessity could not justify the killing of an innocent person, it
could however exclude fault.

h) The act of necessity must be the only reasonably possible means of escaping the danger

− The concept of impossibility may play a role in excluding liability when applied to different elements of delict.
Therefore impossibility is not necessarily a ground of justification that excludes wrongfulness

31. discuss the importance of S v Goliath for the law regarding necessity

S v Goliath the appellate division held by implication that homicide may be justified by necessity.

Minority judgement in Goliath case was that necessity could not justify the killing of an innocent person, it
could however exclude fault.

32. define provocation

Provocation is present when a defendant is provoked or incited by words or actions to cause harm to the
plaintiff. The plaintiff who provoked the defendant, may have to forfeit any compensation for injury to personality

za
caused b the defendants conduct
ls s
o.
33. take a point of view on the correct legal basis for the defence of provocation
ria al
.c
to ori

There is no unanimity regarding the correct legal basis for the defence of provocation: whether provocation
constitutes a ground excluding fault or whether it merely serves to mitigate the damages recoverable by the plaintiff.
Preferred view is that provocation is a ground of justification that renders the defendants conduct lawful (assessed
rtu ut

objectively)
.g T
w R

34. distinguish between provocation and private defence


G

Provocation must be distinguished from private defence- one acts in defence in order to avert an attack that is
imminent or has already commenced. Conduct as a result of provocation, on the other hand, takes place immediately
after the provocative conduct as already terminated it is, in other words an act of revenge, in contrast with an act of
defence.
w
w

35. discuss the requirements for provocation in the case of physical assault, defamation and insult

− Provocation in the case of physical assault: where he provocation takes the form of a physical assault, such
provocation may very well serve as a ground of justification for a subsequent counter-assault of a physical nature

− provocation may serve as a complete defence against an action based on assault, provided that 2 requirements
are met:

1) The provocative conduct must be of such a nature that a reaction to it by means of a physical assault
is reasonable, and accordingly excusable. The question is whether, viewed objectively, the reasonable
person in the position of the defendant would have acted as the defendant did in light of the provocation
2) The conduct of the provoked defendant must be an immediate and reasonable retaliation against the
body of the plaintiff. It must also be reasonable- the physical assault by the second person is not out of
proportion in its nature and degree to the assault by the first aggressor

− Provocation is cases of defamation and insult: defamatory or insulting allegations made during an
argument in reaction to provocative conduct, may be justified in certain circumstances. The requirements
for provocation as a defence in the case of bodily integrity apply here.

Page | 11
Page 12 of 50
12

36. explain the principle of Compensation

⇒ The general compensatory function of the law of delict implies that there must be some loss or damage for which
the law makes compensation available.

There are two forms of compensation, namely

⇒ Compensation for damages: “damages” is a monetary equivalent of damage awarded to a person with the
object of elimination as fully as possible his past as well as future patrimonial and, where applicable, non-
patrimonial damage
⇒ Satisfaction: implies the reparation of damage in the form of injury to personality by inter alia effecting
retribution for the wrong suffered by the plaintiff and by satisfying the plaintiffs and/or community’s
sense of justice

37. distinguish briefly, with reference to examples, between the following concepts
- consent to injury
- consent to the risk of injury
- volenti non fit injury
- voluntary assumption of risk
- contributory negligence and contributory intent

za
− In the case of consent to injury, the injured party consents to specific harm
ls s
o.
− In the case of consent to the risk of injury, the injured party consents to the risk of harm caused by the
ria al
defendants conduct .c
to ori

− Terminology the maxim Volenti non fit iniuria, volenti, is used as a common concept to describe both forms of
consent. Volenti non fit iniuria may thus mean either consent to injury, or consent to the risk of injury.
rtu ut

− The expression “voluntary” assumption of risk” is sometimes used to imply consent to the risk of injury (a
.g T

ground of justification) and sometimes to refer to contributory intent (a ground excluding fault or culpability). A
distinction must be made between contributory negligence and consent and contributory intent.
w RG

− Voluntary assumption of risk” in both its forms constitutes a complete defence excluding delictual liability:
consent excludes wrongfulness; contributory intent cancels out the defendant’s negligence. Contributory
negligence on the other hand, is not a complete defence. The claim of a plaintiff who is guilt of contributory
negligence may, however, be reduced by the court in accordance with the degree of negligence
w

− Whether wrongfulness was excluded because of the consent of the injured, or whether the negligence of
a defendant was cancelled by the plaintiff’s intention (contributory intent) or whether, although the
w

plaintiff neither consented nor had contributory intent, he was in fact contributorily negligent in respect of
his damage because he acted in a manner different from that of the reasonable person

Page | 12
Page 13 of 50
13

38. name the characteristics and requirements for valid consent and apply them to given set of facts

− Characteristics of consent as a ground of justification:


⇒ Consent to injury is a unilateral act. The consent need not necessarily be made known to the defendant. It
may be unilaterally revoked by the consenting party at any stage preceding the defendants conduct. The
defendant acts wrongfully if he proceeds to cause harm despite revocation of consent.
⇒ Consent is a legal act that restricts the injured person’s rights. The consent must be apparent, or
manifest
⇒ Consent may be given either expressly or tacitly. Incitement, encouragement and invitation to injure
normally, but not necessarily indicate that consent is present. Mere acquiescence (submission) does not
necessarily amount to consent. Neither does the knowledge that prejudice will ensue in itself constitute
consent
⇒ Consent must be given before the prejudicial conduct; approval given after act is not consent but may
amount to an undertaking not to institute an action against the defendant (a pactum de non petendo)

⇒ The prejudiced person himself must consent.

− Whether consent is present, depends on the facts of the case

− Requirements for valid consent:

za

1) Consent must be given freely or voluntarily
ls s
o.
ria al
2) The person giving consent must be capable of violation- he must be mature enough to appreciate the
.c
to ori

implications of his acts

3) The consenting person must have full knowledge of the extent of the (possible) prejudice- the
rtu ut

consenting person must have full knowledge of the nature and extent of the risks in order to consent to such
risk. There must be informed consent
.g T

─ Castell v De Greef a reasonable doctor test: the court has to be led by medical evidence on what a
reasonable doctor would have told the patient in the circumstances. In an appeal to the full bench
w R

Ackermann J differed from this view- he preferred the reasonable patient test whereby the doctor’s duty to
G

inform is to be established with reference to the needs and expectations of the particular patient rather
than the insight of the medical profession. “ for the patients consent to constitute a justification that
excludes wrongfulness of medical treatment and its consequence, the doctor is obliged to warn a patient
so consenting if a material risk inherent in the proposed treatment; a risk being material if, in the
circumstances of the particular case a reasonable person in the patients position if warned of the risk,
would be likely to attach significance to it; or the medical practitioner is or should be reasonably aware
w

that the particular patient, if warned of the risk, would be likely to attach significance to it”
w

4) The consenting party must realise or appreciate fully what the nature and extent of the harm will be

5) The person consenting must in fact subjectively consent to the prejudicial act

6) The consent must be permitted by legal order; the consent must not be contra bonos mores. Consent
to bodily injury or consent to the risk of such injury is normally contra bonos mores unless the
contrary is evident

─ Boshoff v Boshoff the plaintiff was struck on his head by the plaintiff’s racket during a squash game
resulting in injury to his eye. The court rejected his claim for damages on the ground that he had
consented to the risk of injury and that the consent was not contra bonos mores

7) The impairment must fall within the lines of consent

Page | 13
Page 14 of 50
14

39. briefly discuss a pactum de non petendo

The pactum de non petendo in anticipando


Is a contractual undertaking not to institute an action against the actor

In the case of pactum non petendo in anticipando there is no doubt that the actor committed a delict, but the
prejudiced person undertakes not to hold the actor liable. Wrongfulness is thus not excluded in such cases, only
the resultant action is.

40. explain when a statute authorises an infringement of interests

A person does not act wrongfully if he performs an act ( which would otherwise have been wrongful) while
exercising a statutory authority

By authorising an infringement of interests, the statute limits the rights of the prejudiced person

Two principles apply: the statute must authorise the infringement of the particular interest concerned and the
conduct must not exceed the bounds of the authority conferred by the statute

Whether the statute authorises the infringement of the interest concerned depends on the intention of the
legislature. The intention of the legislature is determined in accordance with the principles regulating the
interpretation of statutes

za
ls s
o.
ria al
41. explain when act falls within the boundaries of statutory authorisation
.c
to ori

− Whether the permitted act fell within the boundaries of the authorisation , the following is taken into
account:
rtu ut

⇒ It must not have been possible for the defendant to exercise the powers without infringing the
.g T

interests of the plaintiff


⇒ The defendants conduct must have been reasonable
w RG

42. explain when official capacity will constitute a ground of justification

− Law enforcement officers and judicial officers are obliged or authorised by law to perform certain acts. Should they
cause damage in the process, their conduct will be justified and consequently they will not be liable
w

− Should such an official exceed his authority, he acts unreasonably and therefore wrongfully and may be held
w

liable

43. explain when execution of a wrongful command can constitute a defence

− When execution of a wrongful command can constitute a defence S v Banada the following requirement for such
a defence:
a) The order must issue from a person in a position of lawful authority over the accused
b) There must be a duty on the accused to obey the order given
c) The accused must have done no more harm than was necessary to carry out the order

− According to case law there is no absolute duty- that is, a duty of blind obedience- on subordinates to obey the
orders of their superiors

− Two main approaches are followed: there is a view that carrying out a wrongful order is always wrongful. On
the other hand it is argued that obeying a wrongful order is not necessarily wrongful, but that only the
execution of “manifestly” or “palpably” illegal orders is wrongful

− Evaluated with reference to the judgement of the reasonable person


Page | 14
Page 15 of 50
15
− In cases where there is a duty to obey wrongful orders, the applicable ground of justification is, in reality, necessity
in the form of compulsion and not official command

44. indicate when punishment will be lawful and which factors must be taken into consideration with
reference to case law and S10 of the South African Schools act

− Parents and persons in loco parentis have by virtue of their authority over children, the power to administer
punishment to them for the purpose of education and correction.

− The power of persons in loco parentis to discipline is an original authority and delegated parental authority. A
person to whom the power to discipline has been delegated, does not possess more or wider powers than those
delegated to him or her

− S10 South African Schools act prohibits corporal punishment. Objectively viewed punishment must be exercised
moderately and reasonably

− Malice and improper motive is indicative of unreasonable punishment which is wrongful

− The following factors must be considered in determining whether the punishment was moderate and reasonable:
R v Janke and Janke

za
⇒ The nature and seriousness of the transgression;
⇒ The degree of punishment or force inflicted;
ls s
o.
⇒ The physical and mental condition of the person punished;
ria al
⇒ The gender and age of the child; .c
to ori

⇒ The means of correction; and


⇒ The purpose and motive of the person inflicting the punishment
rtu ut
.g T

45. explaining the underlying notion of the doctrine of abuse of rights


w R

− Underlying the so-called doctrine of abuse of right is the notion that the exercise of a right of a power may
take place in a manner or in circumstances which render such exercise wrongful
G

− The doctrine entails the basic question of whether or not the defendant acted wrongfully

− In the event of a dispute between neighbours it must be determined whether the actor exceeded his
powers of ownership and, therefore, acted wrongfully in relation to his neighbour. Answered with reference to
w

considerations of reasonableness and fairness


w

− Reasonableness implies a weighing up of the benefits that the exercise of his rights has for the
defendant, against the prejudice suffered by the plaintiff as a result of such conduct

− The doctrine applies mostly where the property rights of neighbours are concerned

− “nuisance” in English law involves the repeated unreasonable use of land by one neighbour at the
expense of another

− As a general rule the owner of immovable property may use his property as he sees fit as long as he acts within
the bounds placed by the law on his powers of ownership Gien v gien

46. discuss the role an improper motive plays in the doctrine of the abuse of rights

Improper motive is a general concept suggesting merely a reprehensible purpose or objective on the part of
the defendant. Intent may be present even in the absence of improper motive.

Page | 15
Page 16 of 50
16
Improper motive renders an act, which would have been lawful but for such motive, wrongful if it
prejudices a neighbour without benefiting the actor in any way (Kirsh v Pincus)

47. describe the delict nuisance with reference to practical examples

− “nuisance” in English law involves the repeated unreasonable use of land by one neighbour at the
expense of another

− Nuisance usually involves repeated infringements of the plaintiff’s property rights. an objective weighing
up of the interests of the various parties, taking into account all the relevant circumstances, is involved

Fault

48. name the 2 forms of fault

The two main forms of fault are recognised: intention (dolus) and negligence (culpa) in the narrow sense

za
ls s
o.
49. define accountability and explain the influence of youth, mental disease or illness, intoxication and
ria al
provocation on accountability .c
to ori

- A person is accountable ( culpa capax) if that person has the necessary mental ability to distinguish
between right and wrong and if he can also act in accordance with such appreciation
rtu ut
.g T

- He must have the required mental ability at the time of the commission of the act for which the law wants
to blame him
w R

- According to our law a person may lack the necessary mental capacity ( and is thus not accountable-
G

culpa incapax) where one or more of the following factors are present:

⇒ Youth: a child younger than 7 year olds (an infans) is always regarded by the law as being culpa incapax.
There is an irrebuttable presumption that he is not accountable. There is a rebuttable presumption that a child
over the age of 7 but under the age of 14 (an impubes) lacks accountability
w

⇒ Mental disease or illness: where, because of mental disease or illness, a person cannot at a given moment
w

distinguish between right and wrong, or where he is able to make such a distinction but cannot act in
accordance with his appreciation of the distinction he is culpae incapax

⇒ Intoxication: the mere consumption of liquor or use of drugs may in a given situation be a negligent act for
which the defendant may be held responsible. They may however by culpae incapax. A consumption of liquor
may be seen as a negligent act performed at a time when the driver was accountable

⇒ Provocation: where a person loses his temper and becomes passionately angry, he may be said to lack
accountability and will thus not be blamed for his (intentional) conduct

50. explain the relationship between accountability and fault

Fault refers to the legal blameworthiness or the reprehensible state of mind or conduct of someone who has
acted wrongfully.
A person is accountable ( culpa capax) if that person has the necessary mental ability to distinguish between
right and wrong and if he can also act in accordance with such appreciation

Page | 16
Page 17 of 50
17
51. describe all 3 forms of intent and be able to apply them to practical examples

Direct intent (dolus directus) - present where the wrongdoer actually desires a particular consequence of his
conduct. It does not make any difference whether the wrongdoer is certain that the consequence would result
or whether it only appears to him to be probable or possible
Indirect intent (dolus indirectus) – present where a wrongdoer directly intends one consequence of his
conduct but at the same time has knowledge that another consequence will unavoidably or inevitably also
occur. The causing of the second consequence is accompanied by indirect intent.
Dolus eventualis- present where the wrongdoer, while not desiring a particular result, foresees the possibility
that he may cause the result and reconciles himself to this fact. The wrongdoer foresees a consequence but
recklessly carries on with his conduct

52. briefly distinguish between intent and motive

- Motive indicates the reason for someone’s conduct and must not be confused with intent

- Intent is a technical legal term that denotes willed conduct which the wrongdoer knows is wrongful;
motive on the other hand refers to the reason why a person acts in a particular way, that is, the object he
wishes to achieve, his desire, or the facts behind the formation of his will.

- Motive is clearly of evidentiary value to prove direct intent

- Motive may serve as proof of consciousness of wrongfulness.

za
- Bad motive (malice or mala fides) usually indicates knowledge of wrongfulness while a good motive
ls s
o.
usually indicate the opposite
ria al
.c
to ori
rtu ut

53. briefly explain the effect of mistake concerning the casual chain of events
.g T

- whether intent is present where the wrongdoer causes a result in a manner different from that foreseen
w R

by him. A distinction must be made between a material and an immaterial deviation from the planned
foreseen casual nexus.
G

- A material deviation intent is absent while intention is assumed to be present where the deviation is not
markedly different from the foreseen casual chain of events
w

54. state the test for negligence with reference to the formulation in Kruger v Coetzee and apply it to practical
sets of facts
w

Kruger v Coetzee “ for the purposes of liability culpa arises if- a diligens paterfamilias in the position of the
defendant would foresee the reasonable possibility of his conduct injuring another in his person or
property and causing patrimonial loss and; would take reasonable steps to guard against such
occurrence; and the defendant failed to take such steps”

55. form a reasoned opinion on whether negligence and intent can overlap

− There are a number of decisions which tend to support the view that a person cannot act intentionally and
negligently in respect of the same consequence. On the other hand there are decisions in which it was stated
that if intent is present, negligence is included in the intent

- S v Ngubane: intent and negligence may be present simultaneously “where dolus is present, so too is
culpa” to be logical and justifiable. One may argue here that the international causing of harm to another
person is contrary to the standard of care which the reasonable person would have exercised and that negligence
is simultaneously present

Page | 17
Page 18 of 50
18

56. explain whether it is necessary to differentiate between ordinary and gross negligence

- It Makes no difference for Aquilian liability whether the defendant acted with slight or gross negligence,
some statutory provisions limit liability to instances of gross negligence.

- In respect of malicious prosecution as iniuria. It has already been held that liability may be based on
gross negligence instead of intent and that it is therefore material to determine whether the wrongdoer
has acted with gross negligence

MV v Stella Tingas Transnet ltd t/a Portent v owners of the MV Stella Tingas: “to qualify as gross negligence
the conduct in question although falling short of dolus eventualis, must involve a departure from the standard
of the reasonable person to such an extent that it may properly be categorised as extreme; it must demonstrate,
where there is found to be conscious risk-taking, a complete obtuseness of mind or, where there is no conscious
risk-taking, a total failure to take care. If something less were required, the distinction between ordinary and gross
negligence would lose its validity”

57. differentiate between negligence and omission

- An omission can indeed be performed intentionally or negligently; moreover a positive act can be

za
negligent, proving that negligence is not relevant only in regard to an omission.
ls s
o.
- An omission to take reasonable steps to prevent foreseeable harm (as part of the test for negligence)
ria al
should not be confused with an omission as species of conduct
.c
to ori
rtu ut
.g T

58. briefly discuss the general characteristics of the reasonable person dilgens paterfamilias as applied in
case law
w R

1. The reasonable person is not an exceptionally gifted, careful or developed person; neither is he
G

underdeveloped, nor someone who recklessly takes chances or who has no prudence

2. The reasonable person serves as the legal personification of those qualities which the community expects
from its members in their daily contact with one another
w

Weber v Santam Versekeringsmaatskappy Bpk “ we are not concerned with what the care of a legion of
reasonable person types would have been, such as a reasonable educated person, a reasonable illiterate
w

person, a reasonable skilled labourer, a reasonable unskilled labourer, a reasonable adult or a


reasonable child. There is only one abstract, objective criterion, and that is the Courts judgement of what
is reasonable, because the court places itself in the position of the diligens paterfamilias”

- The reasonable person has a certain minimum knowledge and mental capacity which enables him to
appreciate the dangerous potential of certain actions

- Everyone is required to conform to the objective standard of the reasonable person

- There is no authority for the view that the physical characteristics of the wrongdoer play a part in the
reasonable person test

A person suffering from a physical disability may thus still be negligent where he engages in an activity which a
reasonable person in his position would not have regarded as safe

Page | 18
Page 19 of 50
19

59. discuss in detail the reasonable-person test as applied to children with reference to case law and apply to
practical examples

- Whether the fact that the wrongdoer is a child should play a role in the application of the reasonable
person test…

- Before 1965 there was a tendency in our courts to take into account the youthfulness of a wrongdoer in
determining negligence

- In 1965 the Appellate division gave judgement on this matter in Jones v Santam Bpk it adopted a new approach
to the determination of negligence in regard to children. The court implied that the criterion for the
determination of negligence is always objective in the sense that all situations the test of reasonable
person is applied.

- In determining whether the child acted with culpa, it must first be determined:

1. whether the child concerned met the standard of care required of the reasonable person,
2. it must be asked whether the child, if the care shown by him did not meet the requirements of the first test,
was culpa capax (accountable for his actions)

za
- To determine whether the child acted negligently it must be asked whether the conduct of the child
measures up to the standard of care of a reasonable person. The fact that the conduct in question is that of a
ls s
o.
child is irrelevant at this stage of the enquiry
ria al
-
.c
It must be ascertained whether the negligent conduct may be imputed to the wrongdoer that is whether
to ori

he or she is juridically responsible for his acts. Here all the subjective qualities of the child itself are taken into
account to determine his accountability
rtu ut

- The new approach has received a critical reception. 2aspects are noteworthy :
.g T

⇒ It would seem that the earlier test of the reasonable child is more acceptable than that of a reasonable
person because a child, even though he may be fully accountable cannot realistically be measured up
w R

against an adult standard.


G

⇒ It is clear that the court in the Jones case placed the cart before the horse by first enquiring into fault and then
into accountability

- In the Weber v Santam Versekeringsmaatskappy Bpk case the court held that the Jones case did not
materially depart from common law and confirmed the approach in this case “ when the child’s conduct is judged
w

according to the criterion of an adult, the enquiry must be whether he was mature enough to comply with
the criterion in respect of the specific situation
w

60. Discuss in detail the negligence test as applied to experts with reference to case law and apply it to
practical sets of facts

- Whether the fact that the wrongdoer possesses proficiency or expertise in regard to the allegedly
negligent conduct, affects the application of the reasonable person test.

- In the case of an expertise the test for negligence in regard to the exercise of the expert activity is the
test of the so-called reasonable expert: the reasonable expert is identical to the reasonable person in all
respects, except that a reasonable measure of the relevant expertise is added

- Imperitia culpae adnumeratur: ignorance or lack of skill is deemed to be negligence. This is misleading
because our law does not accept that mere ignorance constitutes negligence. The principle embodied in this
maxim applies where a person undertakes an activity for which expert knowledge is required while such person
knows or should reasonable know that he lacks the requisite expert knowledge and should therefore not
undertake the activity in question

Page | 19
Page 20 of 50
20
- Where it is reasonable for a lay person to undertake an activity for which an expert skill is required, it
is expected from such lay person that he acts as a reasonable lay person in the particular circumstances
and as long as he exercises the care of an ordinary lay person in the particular situation he is not
negligent.

61. name the 2 legs on which the test for negligence stands

the reasonable foreseeability and reasonable preventability of damage

62. describe the nature and applicability of the abstract and concrete approaches to foreseeability

- Foreseeability: two diverging views exist as to the nature of the foreseeability test.

⇒ The abstract (absolute) approach: the question whether someone acted negligently must be answered by
determining whether harm to others was in general reasonably foreseeable; whether his conduct in general
created an unreasonable risk of harm to others. On this approach it is clearly not a requirement for
negligence that the extent of the damage or a particular consequence that actually occurred should
have been reasonably foreseeable; it suffices if damage in general was reasonably foreseeable.

⇒ The question whether the defendant is liable for a specific consequence is answered with reference to legal

za
causation rather than by enquiring whether the defendant was negligent with regard to that specific
consequence. This view is not generally accepted by our courts
ls s
o.
ria al
⇒ The concrete (or relative) approach: based on the premise that a person’s conduct may only be
.c
to ori

described as negligent in regard to a specific consequence or consequences; it is therefore a


prerequisite for negligence that the occurrence of a particular consequence must be reasonably
foreseeable
rtu ut
.g T

⇒ The concrete approach is preferred for the following reasons: the question whether the reasonable
person in the position of the wrongdoer would have acted differently in order to prevent damage, may
w R

only be answered by in a meaningful way by reference to the consequences or consequences that


were indeed reasonably foreseeable (and not merely by reference to damage in general as required in
G

the abstract approach).

63. name the 4 considerations that play a role in the preventability aspect of the test for negligence and apply
them to factual complexes
w

- Preventability: whether the reasonable person would have taken precautionary steps to prevent the damage
w

from occurring

- Whether, in an instance of reasonable foreseeable damage, the defendant took adequate reasonable
steps to prevent the materialisation of the damage

- Van der walt and Midgely: identified 4 factors particularly relevant to preventability leg of the test for negligence:

⇒ The nature and extent of the risk


⇒ The seriousness of the damage: Lomagundi Sheetmetal and engineering v Basson the court held that
although the risk of the stover being ignited by the welding was not very great, the damage which was likely to
result from burning Stover would be fairly extensive. The reasonable person would thus have taken steps to
prevent the damage from occurring.
⇒ The relative importance and object of the wrongdoers conduct
⇒ The cost and difficulty of taking precautionary measures: Gordon v Da Mata and City of Salisabury v
King

Page | 20
Page 21 of 50
21

64. identify the general factors that are considered in determining whether negligence was present in a
particular case, and be able to apply this knowledge to practical sets of facts

- It must be self-evident that the negligence of conduct may only be evaluated in the light of all the relevant
circumstances of a particular case

- Cape Town Municipality v Butters: “ in considering the issues of negligence what is, or is not, reasonably
foreseeable and what steps, if any , ought to be taken by the reasonable person to avert such foreseeable harm
must always depend upon the particular circumstances of each case

- In each and every case one is still concerned with the question whether the wrongdoer’s behaviour was
in accordance with the standard of the reasonable person in his position

- Greater care is required when someone works with things which are inherently dangerous

- Greater care is expected when a person deals with individuals who suffer from some disability or
incapacity. Someone who knows or is reasonably expected to be aware of the special circumstances is required
to act with care

- Where a person has to take a decision in a situation of sudden emergency and there is insufficient
opportunity to consider all the consequences of his actions, this factor must be taken into account in

za
deciding whether he is negligent
ls s
o.
ria al
.c
65. explain and apply the principles relating to the so-called ‘doctrine of sudden emergency’
to ori

- the law cannot expect a person who has to act swiftly in a situation of imminent peril to show the same
rtu ut

judgement and skill as a person who is not acting in such urgent circumstances
.g T

- 3 requirements which must be satisfied in a case of sudden emergency for a wrongdoer’s conduct not to amount
w R

to negligence:
G

1. The wrongdoer must have faced a situation of imminent peril


2. the wrongdoer must not have caused the perilous situation by his own negligence or
imprudence
3. the wrongdoer must not have acted in a grossly unreasonable manner- must establish whether
the reasonable person in the same circumstances would have made the same error of
w

judgement as the wrongdoer


w

- generally speaking a person acts according to the standard of the reasonable person when he relies on
the fact that another person will act in a reasonable way (Butters v Cape Town Municipality )

- Ntsala v Mutual & Federal Insurance “a party to an action can only rely on the doctrine of sudden
emergency if and when the sudden emergency in which he finds himself is not of his own making. If his
actions or neglect are the reason or cause of the sudden emergency, he can for that reason also be
found to be negligent

- In general a person need not take steps to guard against the recklessness or gross negligence of others

- These principles also find application with regard to contributory negligence; they provide guidelines for
determining when a person is required to be on his guard against the possible negligence of others

- A person is required to act with extreme circumspection when there are clear indications that another is
not going
to obey traffic rules

- Where a person creates a situation which is not inherently dangerous, but which may become dangerous
when another person interferes, the former is obliged to take the precautionary steps which a reasonable
Page | 21
Page 22 of 50
22
person in his position would take

- The customs, usages and opinions of the community; generally a wrongdoer will be able to defend
himself against n allegation of negligence by proving that he acted in accordance with normal practices

- A person whose conduct is in fact negligent may not escape liability by relying on common practice-
whether the reasonable person would have acted in the same way as the defendant

- It is not clear in our law whether conduct contrary to a statutory provision is per se negligent or whether
the provision merely affords proof of negligence. It should probably be accepted that in such a situation
it is incorrect to speak of statutory negligence and the statutory provisions at best only provides
evidentiary material

66. explain the English law ‘duty of care’, the criticism thereof, and the difference between ‘duty of care’ and
‘legal duty’

- Negligence is generally determined using the reasonable person test; our courts have on occasion used
the so-called duty of care doctrine.

- The so-called duty of care doctrine: according to this approach one must first establish whether the defendant
owed the plaintiff a duty of care (the duty-issue), and thereafter whether there was a breach of this duty (the
negligence- issue). If both questions are answered in the affirmative, negligence is said to be present

za
- Whether a duty of care was owed, the criterion was traditionally whether the reasonable person in the
ls s
o.
defendant’s position would have foreseen that his conduct might cause damage to the plaintiff. This issue
ria al
is a policy-based judgement, in which foreseeability plays no role, as to whether interests should be protected
.c
against negligent conduct.
to ori

- Whether there was a breach of the duty or care, the court considers whether the wrongdoer exercised the
rtu ut

standard of care that the reasonable person would have exercise in order to prevent damage
.g T

- The duty of care is not a general duty; it is a duty towards certain people or classes of people and not
towards every person
w RG

- From a historical point of view the application of these principles must be rejected. The reason to reject the
application of the duty of care in our law is that in its traditional form it is unnecessary and roundabout
way of establishing what may be established directly by means of the reasonable person test for
negligence. The use of the duty of care doctrine may confuse the test for wrongfulness (breach of legal duty)
with the test for negligence
w

- It is submitted that there is no reason why the duty of care approach should be used in the determination
w

of negligence and at present it would appear that in most cases our courts simply use the test of the
reasonable person

67. write brief notes on the application of the onus of proof in the case of negligence, and in particular the res
ipsa loquitur maxim

- The onus is on the plaintiff to prove on a preponderance of probabilities that the defendant was
negligent. Where there is a statutory presumption of negligence, the onus rests on the defendant to rebut the
presumption of negligence in order to escape liability.

- Res ipsa loquitur (the facts speak for themselves) usually refers to a situation where the facts of the matter
are such as to give rise to an inference of negligent conduct and finds application “where the only known
facts, relating to negligence consist of the occurrence itself”

- It has been contended that res ipsa loquitur creates a presumption of negligence, but the current legal
position is that there is no shift in the onus of proof in such cases and that there is not even a prima facie case in

Page | 22
Page 23 of 50
23
favour of the plaintiff; the phrase is merely an argument on the probabilities that the plaintiff, who may
have little evidence at his disposal, may use in order to convince the court that the defendant acted
negligently

68. explain the difference between wrongfulness and negligence

- Wrongfulness is determined by means of the criterion of objective reasonableness, while the test for
negligence is that of the (objective) reasonable person- an objective criterion of reasonableness is used in
the determination of both wrongfulness and negligence

- In the case of wrongfulness the defendant’s conduct is determined by means of a weighing up of


conflicting interests in the light of the legal convictions of the community (boni mores); with negligence
the reasonable person’s conduct is determined with reference to the reasonable foreseeability and preventability
of damage

- Wrongfulness is concerned with the determination of the legal reprehensibility of the conduct (wrongfulness
thus qualifies conduct); negligence is usually seen as the determination of the legal blameworthiness of the
defendant for his wrongful conduct (negligence thus qualifies the defendant or wrongdoer)

- Wrongfulness concerns the legal reprehensibility of a person’s conduct determined in light of all the

za
relevant facts and circumstances that are actually present and all the consequences which actually
ensued;
ls s
o.
ria al
- negligence concerns the legal blameworthiness of the wrongdoer, it is determined with reference to the
.c
position in which the defendant actually found himself, by placing the reasonable person in the position
to ori

of the wrongdoer at the time of the commission of the act and by taking cognisance only of those facts
and circumstances which the wrongdoer knew of, amplified by facts about which the reasonable person
rtu ut

is his position would have known


.g T

- Wrongfulness is determined on the basis of actual facts or realities and negligence on the basis of
probabilities
w RG

- Wrongfulness is determined before negligence

- The test for wrongfulness is narrower that the test for negligence

- S v Goliath A threatened by B assisted in killing C, the court held that compulsion may be a defence to the
w

killing of a human being, but was not prepared to express an opinion on the question whether compulsion is a
ground of justification or ground excluding fault
w

- the conclusion is that because of the differences between the test for wrongfulness and the test for
negligence, a defendant may be said to have acted unreasonably for the purposes of wrongfulness but
reasonably (like the reasonable person) for the purposes of negligence

- an omission is unreasonable and thus wrongful where, according to the boni mores test, a legal duty rested
on the defendant to act positively in order to prevent harm and he neglected to comply with such a duty and
his attempt coincided with what the reasonable person would have done, his (unreasonable) wrongful act is
not accompanied by (unreasonable) negligent conduct and he will escape liability Minister of forestry v
Quathlamba

69. write brief notes on the meaning and relevance of the term “contributory fault”

⇒ Contributory fault refers to the conduct of the plaintiff. Primarily relevant in limiting the extent of the defendant’s
liability and is thus of considerable importance in legal practice. regulated by the apportionment of damages act
34 of 1956

Page | 23
Page 24 of 50
24

70. explain the common-law position regarding contributory fault and be able to apply this knowledge to
factual examples

common law position


⇒ The general rule in Roman Dutch law was that fault on the part of the plaintiff precluded him from
claiming damages from the defendant who was also to blame for causing the damage

⇒ The doctrine of contributory negligence as applied initially in our courts was taken over from English law.
Developed from the judgement of Davies v Mann- the plaintiff had negligently left his haltered donkey in the road.
The defendant, driving his wagon, collided with the donkey. There was negligence on both sides. The court
adopted a new approach. Since the defendant had the last opportunity (last opportunity rule) to avoid the collision,
the plaintiff’s negligence was ignored and the defendant incurred full liability for the damage.

⇒ In 1945 English legislature replaced this rule with the principle of proportional division of damages in accordance
with each parties degree of fault

⇒ Our courts initially accepted that if the negligence of two persons contributed to the causing of a
particular result, and one or both of them suffered damage as a consequence thereof, neither party could
institute action unless the negligence of one of them was the decisive cause of the accident. In the event
that the negligence of the other party was completely ignored and he could succeed in full with his claim

za
⇒ The so-called last opportunity rule did not work in practise and in time resulted in an untenable situation,
ls s
o.
so legislature intervene
ria al
.c
to ori

71. explain the terms, meaning and effect of the Apportionment of Damages act 34 of 1956 and be able to
rtu ut

apply this knowledge to factual situations


.g T

The Apportionment of Damages Act 34 of 1956


w R

⇒ Has made considerable changes I the common law position concerning contributory fault (in the form of
contributory negligence) on the part of the plaintiff
G

Provisions and meaning of S1 (1) (a) and (b)

S 1(1) (a)
“where any person suffers damage which is caused partly by his own fault and partly by the fault of the other
w

person, a claim in respect of that damage shall to be defeated by reason of the fault of the claimant but the
damages recoverable in respect thereof shall be reduced by the court to such extent as the court may deem
w

just and equitable having regard to the degree in which the claimant was at fault in relation to the damage”

S 1(1) (b)
“Damage shall not for the purpose of (a) be regarded as having been caused by a persons fault
notwithstanding the fact that another person had an opportunity of avoiding the consequences thereof and
negligently failed to do so”

⇒ The effect of these two subsections is to abolish the all or nothing principle

⇒ Meaning of “Fault”: in general the term fault encompasses both intent and negligence.

⇒ Whether a defendant who has intentionally caused damage to the plaintiff may raise a defence of
contributory negligence on the part of the plaintiff- A defendant who has intentionally caused harm to the
plaintiff will not be able to ask for a reduction in damages because of contributory negligence

⇒ Whether the section under discussion applies in the case if the so-called defence of contributory intent-
A distinction must be made between 2 situations:
1. where a plaintiff intentionally contributed towards his own loss while the defendant was merely negligent-
the plaintiff forfeits his claim
Page | 24
Page 25 of 50
25
2. where the defendant caused the loss intentionally and the plaintiff’s unreasonable conduct causing loss
was also intentional
⇒ it would appear that legislature intended to make provision for contributory negligence and not the
defence of contributory intent

⇒ in Greater Johannesburg Transitional Metropolitan Council v ABSA Bank ltd the court held that S 1(1) (a)
applies when the form of fault on the part of both the plaintiff and the defendant is intent

⇒ Meaning of “apportionment of damages”: the process concerns a reduction of damages received by the plaintiff
because of his own fault (negligence) in regard to the damage he sustained

⇒ Criteria for the “apportionment of damages”; the reasonable person test for negligence. The implication is
that S 1(1) (a) applies only to damage caused partly by the fault of the plaintiff and partly by that of the defendant,
the act cannot apply where liability does not depend on the defendant’s fault. Thus the act does not apply in the
case of strict liability

⇒ Insofar as the objective reasonable person test applies one is dealing with the deviation from the standard of
care which applies to all persons in the community

⇒ In the case of the plaintiff as well as the defendant one is concerned with a negligent act or omission that
is casually linked to the damage. This casual nexus is determined according to the usual test, and not, as was
previously the case, in terms of the so-called last opportunity rile

⇒ The court does not take into account degrees of causation

za
ls s
o.
ria al
⇒ The method of determining who should bear what portion of the damages, involves a comparison of the
.c
respective degrees of negligence of the parties involved
to ori

⇒ Prior to Jones v Santam ( in the South British co v Smit) the AD accepted that once the plaintiff’s degree of
rtu ut

negligence had been established, it was unnecessary to inquire into the extent to which the defendant’s conduct
had deviated from the standard of the reasonable person
.g T

⇒ In the Jones case a completely new approach to determining the degree of fault shown by the plaintiff and
w R

defendant was followed. The fact that the plaintiff was 30 % negligent does not automatically imply that the
G

defendant was 70% negligent. In order to establish respective degrees of negligence the carefulness of the
conduct of each party must be measured separately against the standard of the reasonable person

⇒ AA Mutual Insurance association ltd v Nomeka the AD confirmed the approach followed by Smit, that the
degree of the plaintiff’s fault automatically determines the degree of fault by the defendant
w

⇒ It is submitted that the Jones case should be followed


w

⇒ The courts determine the degree of negligence of each part on the evidence and then go through a mental
process, ascertain each one’s deviation from the standard of the reasonable person, the result of which is then
without further ado given as a result

⇒ General Accident Versekeringmatskappy SA Bpk v Uijs the extent of the plaintiffs fault is merely one of a
number of factors which the court may take into account in order to reduce the plaintiff’s damages in a just and
equitable manner. The approach by Van Heerden JA may be justified in the light of criteria such as fairness and
equity

⇒ The extent to which negligence on the part of a third party may be taken into account in a delictual action. In
rd
general it may not be raised as a defence to a delictual claim unless the 3 party is the servant of the plaintiff and
acted within the scope of employment

Page | 25
Page 26 of 50
26
⇒ Onus of proof; the defendant raises he defence of contributory negligence on the part of the plaintiff; he
has to prove such a defence on a balance of probabilities. The AD has held that contributory negligence may
be taken into account even where the defendant has not expressly pleaded such a defence

⇒ The concept of contributory “negligence”:

strictly speaking an act can only be negligent where it is also wrongful and it is clear that a person cannot act
wrongfully in regard to himself. “contributory negligence” is thus the result of an approach used to
determine the extent of the defendant’s liability by means of a method which is analogous to that for
determining negligence; the method does not pertain to negligence stricto sensu because wrongfulness cannot
play a part

⇒ Fault in regard to “damage” or “damage-causing event” whether S1 (1) (a) is also applicable where the
plaintiff was negligent in respect of the damage-causing event itself but where his negligence increased the
damage
⇒ King v Pearl Insurance co the judge based his decision mainly on the argument that only negligence with regard
to the damage-causing event, as opposed to negligence with regard to the damage itself, is taken into
consideration for the purpose of the act
⇒ King was criticised in Bowkers Park Komga Cooperative v SAR “s 1(1) (a) leaves no doubt that contributory
negligence relates to fault with regard to damage and not fault with regard to the damage-causing event. This

za
approach was accepted in Union National South British Insurance v Vitoria
⇒ It must be emphasised that the contributory negligence of the plaintiff is only relevant in so far as that is
as led to an increase in the damage ls s
o.
ria al
⇒ Voster v AA Mutual Insurance Association- the plaintiff was not wearing a seatbelt when she was injured in an
.c
accident. Her contributory negligence was taken into account by the court in the apportionment of damages
to ori

⇒ The provisions and meaning of S(3):


“For the purpose of this section “fault” includes any act or omission which would, but for the provisions of
rtu ut

this section, have given rise to the defence of contributory negligence”


.g T

⇒ The words “but for this section” are meaningless


w R

⇒ The act erroneously construes fault as an act or omission. Fault is, generally the legal blameworthiness of a
G

person for his wrongful conduct

⇒ An examination of conduct alone is an insufficient basis on which to determine the fault of the actor. In addition to
an act or omission, other relevant factors must also be taken into account in determining fault
w

⇒ The dependant’s action: the common law position remained unchanged until 1971 and the last opportunity rule
was still applied in these cases. This position was changed by an amendment in 1971 of the apportionment of
w

damages act

⇒ Breach of contract: whether the apportionment of damages at applies to damages for loss suffered as a result of
breach of contract. The essence of the court’s decision was that the act was historically not intended to apply to
claims for breach of contract, but only to amend the law of delict.

⇒ However, a strong case can be argued in favour of intervention by the legislature to provide for cases where the
defendant’s breach of contract is described in terms of his negligence while the plaintiff by his own negligence
contributed to the damage.

Legal causation: not every negligent act of a plaintiff related to his damage is relevant for the purpose of S1
(1) (a) of the act. A clear distinction must be made between a plaintiff’s negligent conduct before the damage-causing
event and such conduct after the event. Only the former conduct is relevant for the purposes of apportionment of
damages, while the latter must be taken into account when determining legal causation

Page | 26
Page 27 of 50
27

72. distinguish between the following concepts: volenti non fit iniuria, consent to injury, consent to the risk of
injury, voluntary assumption of risk, contributory intent and contributory negligence

⇒ Consent to injury and consent to the risk of injury as a ground of justification. Such consent is sometimes
referred to as voluntary assumption of risk.

⇒ In regard to contributory fault, voluntary assumption of risk is a ground that cancels fault and is not a
ground of justification

⇒ Assumption of risk in this sense implies that the requirements for a ground of justification are absent

⇒ Where a plaintiff does not act with contributory intent, the fault of the defendant (in the form of
negligence) is eliminated by the contributory intent of the plaintiff.

⇒ The contributory intent (at least dolus eventualis) or assumption of risk by the plaintiff therefore cancels
the defendants fault.

⇒ There is little authority for the so-called defence of contributory intent in our law where the defendant
acted negligently, and it would appear that our courts are not prepared to recognise it in terms of the
Apportionment of damages act. The principle that the conscious taking of an unreasonable risk by the
plaintiff cancels fault on the part of the defendant, is a principle of common law and functions
independently of the act.

za
ls s
o.
ria al
73. distinguish between voluntary assumption of risk by the plaintiff as ground of justification excluding
.c
to ori

wrongfulness, and voluntary assumption of risk on the part of the plaintiff as a type of ground excluding
fault which excludes the defendants negligence
rtu ut

⇒ Lampert v Hefer whether the injured part should have realised that Hefer was unable to control the motor cycle
.g T

properly; if this was the case; she was guilty of contributory negligence. This was not the courts finding. Fagan JA
“she must have or should have appreciated that risk”. Either she did appreciate is and then there is assumption of
w R

risk or she ought to have done so, and then there is contributory negligence.
G

⇒ Schreiner JA distinguished clearly between voluntary assumption risk and contributory negligence.
Although he inclined in that direction, he did not regard assumption of risk as a separate defence over and above
consent (ground of justification) theoretically it is more correct o draw the distinction

⇒ It is important only to determine whether there was contributory intent, this must be answered in the
w

affirmative, because the plaintiff, well aware of the danger and possible injury, nevertheless decided to expose
herself to the risk. The remedy should therefore not be available to her.
w

74. discuss the case law as an illustration of contributory fault

⇒ Fagan JA: discussed voluntary assumption of risk (contributory intent) and contributory negligence, and
stated that these two defences may overlap. However, where there is an actual assumption of risk, the injured
party chooses freely, with full knowledge of the danger, to run the risk- which is not negligence but an
intentional exposure to risk. In this sense, there is thus no overlap with contributory negligence. Where,
however, the injured party should have been aware of the danger, but was not, there is clearly o
assumption of risk, but only contributory negligence.

⇒ Netherlands Insurance co of SA v Van Der Vyver: AD directed its attention to 2 form of volenti non fit iniuria ,
namely, the consent to the risk of injury (a ground of justification) and contributory intent or voluntary assumption
of the risk (which cancels fault).

⇒ No authority from our case law was cited for the statement that contributory intent is an independent defence, nor
was reference made to any of the authoritative sources of our law recognising it. The AD was therefore not

Page | 27
Page 28 of 50
28
prepared in principle to acknowledge such a defence

⇒ While contributory intent is not found as a defence in our authorities, the concept has been developed in
law in order to explain a form of the well-known defence of volenti non fit iniuria

⇒ Contributory intent is present in a case where the defendant escapes liability because the plaintiff has exposed
himself voluntarily and “intentionally” to the risk of injury. In the Lampert case the plaintiff failed precisely because
of the defence of volenti non fit iniuria. On close analysis her claim failed because of her contributory intent

75. discuss the connection between the doctrine of voluntary assumption of risk and the so-called rescue
cases

⇒ The connection between the doctrine of voluntary assumption of risk and so-called “rescue cases”

⇒ Y entered the house to rescue a baby and was injured by the flames. Here X would have also been liable because
he should have foreseen that there could be a baby in the house and that someone would enter the house to
rescue the baby.

Could it be argued that there was contributory fault on Y’s part that would exclude X’s liability? In terms of such an
argument, Y’s contributory fault could take the form either of contributory intent in that he knowingly exposed himself
to the risk of injury or of contributory negligence in that the reasonable man would not have acted likewise. However,

za
there is no question of contributory fault here; as far as contributory negligence is concerned, Y acted like a
reasonable man; as far as contributory intention is concerned, his will was directed towards a lawful goal and he did
ls s
not therefore act consciously unreasonably (that is “consciousness of wrongfulness” is absent)

o.
ria al
.c
to ori

76. explain the importance of the decision in Greater Johannesburg Transitional Metropolitan Council v ABSA
in respect of the defence of contributory intent
rtu ut

⇒ in Greater Johannesburg Transitional Metropolitan Council v ABSA Bank ltd the court held that S 1(1) (a)
.g T

applies when the form of fault on the part of both the plaintiff and the defendant is intent
w RG

Causation

77. distinguish between factual and legal causation

o A causal nexus between conduct and damage is required for a delict. A person can thus not be liable if he
w

has not caused any damage


o Various so-called theories of causation have been developed and the most important are the conditio sine
w

qua non theory, the adequacy theory, the direct consequence theory, the foreseeability theory and the “flexible
approach”

o the question of legal causation arises when determining which harmful consequence actually caused by
the wrongdoer’s wrongful, culpable act he should be held liable for i.e. which consequences should be
imputed to him
o factual causation is present in a given case if it has been proved on a preponderance of probabilities that
the act concerned has caused the relevant consequence; legal causation concerns the question whether the
actor should be held liable for the damage he has caused in a wrongful and culpable manner

Factual Causation
o There can be no question of delictual liability if it s not proved that the conduct of the wrongdoer or
defendant caused the damage of the person suffering the harm
o The courts usually succeed admirably to determine, on the basis of the evidence and the probabilities of
the given case, whether a casual link exists between the wrongdoers conduct and the damage
o Conditio sin qua non to inquire whether one fact follows from another. This is indeed the natural way to
determine a casual link

Page | 28
Page 29 of 50
29
o Most writers and courts are in favour of the conditio sin qua non theory in determining or describing
factual causation. However, our courts have also accepter that the conditio sin qua non approach is not the only
way to determine factual causation
o It is not always clear precisely what interpretation the courts give to the conditio sin qua non method they
employ, which variant of the method they employ, or precisely how the test works

78. explain the operation of the conditio sine qua non doctrine, and be able to apply it to factual situations

o Also known as the ‘but for’ test

o Van der Merwe: An act is the cause of a result if the act cannot be thought away without the result disappearing
simultaneously. The act must in other words be conditio sin qua non of the result
o
International Shipping co v Bentley: the court formulated the conditio sin qua non approach as follows:
st
o the 1 enquiry is a factual one and relates to the question whether the defendant’s wrongful act was a
cause of the plaintiff’s loss. In order to apply this test one must make a hypothetical enquiry as to what probably
would have happened but for the wrongful conduct of the defendant. This enquiry may involve the mental
elimination of the wrongful conduct and the substitution of a hypothetical course of lawful conduct and
the posing of the question as to whether upon such a hypothesis plaintiff’s loss would have ensued or
not

o In the case of “positive” conduct or a comissio on the part of the defendant, the conduct must be
“removed” in the mind to determine whether the relevant consequence would still have resulted

za
o ls s
If hypothetical positive conduct of the defendant could have prevented the damage, it can be said that the

o.
ria al
defendant’s omission was the cause of the damage. This inquiry requires a retrospective analysis of what
would probably have happened, based upon the evidence and what could have been expected in the ordinary
.c
to ori

course of human endeavour

Whether the hypothetical positive conduct must be determined objectively or subjectively, that is,
rtu ut

o
according to what a reasonable person would have done or what the relevant person (wrongdoer or defendant)
.g T

would have done. Constitutional court preferred an objective test


w R

o However, “inserting” reasonable conduct of the wrongdoer into the set of facts appears to have the
potential to cause a confusion of factual causation and negligence. First it must be determined whether the
G

wrongdoer could have done anything to prevent the relevant consequence (causation), and only then whether the
reasonable person in the position of the wrongdoer would have prevented the consequence (negligence)
w

79. write brief notes on the criticism of the conditio sine qua non doctrine
w

♣ The conditio sine qua non theory is based on a clumsy, indirect process of thought that results in
circular logic

♣ The condition sine qua non test fails completely in cases of so-called cumulative causation-
cumulative causation occurs where more than one act actually causes a particular consequence,
for example where X and Y simultaneously, but independently of each other, fire a fatal shot at Z

♣ The conditio sine qua non test is in fact not a test of causation because it is merely an ex post
facto way of expressing a predetermined casual nexus

One can only employ conditio sine qua non after one has in some other way determined the cause
of a particular consequence. Without prior knowledge of what actually caused a result, conditio
sine qua non offers no solution;

Conditio sine qua non is merely a convenient and known way of expressing an already determined
casual link. The conditio sine qua non is not a practical test of causation, but rather, an expression
of a conclusion already made on other grounds

Page | 29
Page 30 of 50
30
European jurists reject conditio sine qua non as a test- according to them it is a best a method
of controlling the correctness of one’s conclusion about a factual casual link on the evidence

za
ls s
o.
ria al
.c
to ori
rtu ut
.g T
w RG
w
w

Page | 30
Page 31 of 50
31

80. explain the apparent application of conditio sine qua non in the case of an omission, and be able to apply
this knowledge to factual situations

o S v Van As- the court attempted to test the casual connection between the omission and the death by asking
whether a reasonable search would have prevented the children’s death, the court inserted positive conduct in the
place of the omission. This approach is viewed as an application of condition sine qua non by our courts

o Every person knows that he can prevent certain consequences by interrupting a casual chain of events

o The ‘inserting’ method used by our courts to determine causation in instances of omission is nevertheless a
realistic approach to causation and is logically more well-founded that the “removing” of something that is
obviously the cause of a consequence in order to demonstrate the existence of factual causation

81. explain the correct method of determining a factual causal relationship and be apply to apply it to factual
examples

Factual causation concerns a particular kind of link or connection between at least two facts or sets of facts, namely
the link existing when, stated succinctly, one fact arises out of another.

za
o The courts usually determine (or test) on the basis of the evidence and probabilities whether a factual
casual link between the act and the harmful consequence exists without really employing the method of
ls s
o.
the conditio sine qua non
ria al
o
.c
It stands to reason that knowledge and experience, as well as reliable evidence, are required to determine
to ori

a casual link. This knowledge may be of a simple nature or it may be of an expert nature
rtu ut

The existence of such a nexus will be dependant on the facts of a particular case and a characteristic of a casual
nexus is that one fact arises out of another, established according to human experience in general and that of the
.g T

judge of the facts in particular


w R

o in determining factual causation in a case of omission, the court must in general determine what the alleged
G

wrongdoer could in the circumstances have done something (in the form of positive conduct) to change the
factual course of events to a meaningful extent, do the questions concerning legal duty and reasonable
conduct (wrongfulness and negligence) arise

o determining factual causation in the case of an omission can in general be formulated as follows: it entails a
retrospective analysis of what would probably have happened if the alleged wrongdoer had acted
w

positively in light of the available evidence and the probabilities originating from human behaviour and
w

related circumstances

o it is usually sufficient for the purposes of factual causation if a defendant’s conduct has in any way contributed to
the damage sustained by the plaintiff; for causation it is unnecessary that his conduct should be the only cause, or
the main cause, or a direct cause

82. discuss the meaning, operation and function of legal causation

o the question of legal causation arises when determining which harmful consequence actually caused by the
wrongdoer’s wrongful, culpable act he should be held liable for i.e. which consequences should be imputed to
him

o factual causation is present in a given case if it has been proved on a preponderance of probabilities that
the act concerned has caused the relevant consequence; legal causation concerns the question whether the
actor should be held liable for the damage he has caused in a wrongful and culpable manner

Page | 31
Page 32 of 50
32

o Truck v Commissioner for Inland Revenue it is generally recognised that causation in the law of delict gives
rise to 2 distinct enquiries. the first one often termed ‘causation in fact’ or ‘factual causation’, is whether there is a
factual link of cause and effect between the act or omission of the party concerned and the harm for which he is
sought to be held liable;

and in this sphere the generally recognised test is that of the conditio sine qua non or the but for test.

This is essentially a factual enquiry. Generally speaking no act or omission can be regarded as a cause in fact
unless it passes this test.

o The second enquiry postulates that the act or omission is a conditio sine qua non and raises the question as to
whether the link between the act or omission and the harm is sufficiently close or direct for legal liability to ensue;
or whether the harm is as it is said, ‘to remote’.

o This enquiry is concerned basically with a juridical problem in which considerations of legal policy ay play a part
(see International shipping co v Bentley and S v Mokgethi)

o In a sense the question of legal causation is tacitly dealt with within the framework of the investigation into the
other elements of a delict- especially wrongfulness and fault. Normally legal causation is only problematic where a
chain of consecutive or remote consequences results from the wrongdoer’s conduct, and where it is alleged that
he should be held liable for all the consequences

za
o ls s
The concepts of ‘ legal causation’, ‘limitation of liability’ and ‘ imputability of harm’ are used synonymously to

o.
ria al
indicate the process whereby the court determines which of the heads of damage caused by an actor he should
be held liable .c
to ori

o Minister of Safety and Security v Carmichele legal causation deals with the question whether “the wrongful act
rtu ut

is linked sufficiently closely or directly to the loss for legal liability to ensue or whether, as it is said, the loss is to
remote”
.g T

o the limitless liability which could have been brought about y factual causation in itself, is “limited” by those
w R

elements of a delict which establish liability


G

83. name the different tests for legal causation

o The best known theories for determining legal causation are :


w
w

o the flexible approach, based on policy considerations, reasonableness, fairness and justice,
o the theory of adequate causation, the “direct consequences” criterion;
o the theory of fault and the reasonable foreseeability criterion.

o Until recently it has generally been accepted that the reasonable foreseeability criterion is preferred by the courts,
however, the AD has now expressed itself in favour of a flexible approach, in terms of which there is no single
criterion which can be applied to all situations

Page | 32
Page 33 of 50
33

84. explain the flexible approach to legal causation as applied by the courts, and be able to apply it

o S v Mokgethi: Van Heerden- there is no single and general criterion for the legal causation which is
applicable in all instances. A flexible approach is accordingly suggested.

o Basic question: whether the is a close enough relationship between the wrongdoer’s conduct and its
consequences for such consequences to be imputed to the wrongdoer in view of policy considerations
based on reasonableness, fairness and justice

o The flexile criteria according to which, the court determines, whether a sufficiently close link exists
between an act and a consequence with reference to policy considerations

o Whether one regards reasonable foreseeability (or any other test for legal causation) as a subsidiary test, or
simply as a factor in determining legal causation, the AD formulation an application of the flexible approach makes
it clear that these tests or factors merely function as aids in answering the basic question of imputability of harm

o This means that the theories should be regarded as pointers or criteria reflecting legal policy and legal convictions
as to When damage should be imputed to a person; damage is imputable when, depending on the
circumstances, it is a direct consequence of the conduct, or reasonably foreseeable, or if it is an adequate
relationship to the conduct, or for a combination of such reasons, or simply for reasons of legal policy

o The flexible approach can accommodate the divergent needs for legal causation to different legal fields as

za
the law of delict, criminal law and insurance law
ls s
o.
ria al
.c
to ori

85. explain adequate causation as a specific test for legal causation, and be able to apply it
rtu ut

o A consequence which has in fact been caused by the wrongdoer, is imputed to him if the consequence is
.g T

“adequately” connected to the conduct


w R

o “adequate” if according to human experience, in the normal course of events the act has the tendency of bringing
G

about that type of consequence. In order to determine whether the act had such a tendency, the following
question, for example, asked: was the damage the reasonably-to-be-expected consequence of the act; did the
damage fall within the expected field of protection envisioned b the legal norm that was infringed; were
the consequences “juridically relevant” with reference to the cause?

o The adequate consequence theory entails an objective prognostic test: “ one looks forward as from the
w

moment of the act and ask whether that type of result was to be expected
w

o No substantial difference between the theory of adequate causation and the test of reasonable
foreseeability

o Generally speaking, one would be able to say that a result normally to be expected is also a reasonably
foreseeable result, and visa versa

o Criterion for legal causation can be more easily distinguished from negligence (where a reasonable foreseeability
criterion is also applied) with the theory of adequate causation that the criterion of reasonable foreseeability

Page | 33
Page 34 of 50
34

86. explain the direct consequences theory as a test for legal causation, and be able to apply it

o Stems from English law, an actor is liable for all the “direct consequences” of his negligent conduct.

o Liability is not necessarily limited to the foreseeable consequences of his conduct.

o A consequence need not follow the cause immediately in time and space to be a “direct consequence” thereof

o The possibly wide effect of the direct consequences test has also been limited b the “foreseeable plaintiff”
doctrine- an actor does to act negligently towards a plaintiff unless it is reasonably foreseeable that the particular
plaintiff will be injured

o In 1961 the direct consequence theory was rejected by the privy council in Overseas Tankship v Motor Dock and
Engineering Co in favour of the principle that only foreseeable damage was recoverable

o In South African law of delict, the direct consequences theory was unequivocally accepted in (probably)
only one reported case Frenkel & co v Cadle

o The direct consequences theory does not serve as a general test for the imputability of harm, but like the
other tests, fulfils a subsidiary role in establishing legal causation in terms of the prevailing flexible
approach

za
ls s
o.
ria al
87. write brief notes of the content of the so-called fault in relation to the loss approach to legal causation
.c
to ori

⇒ The wrongdoer is liable only for those consequences in respect of which he had fault; in other words, those
consequences covered by his fault are imputed to him,
rtu ut

⇒ Van der Merwe and Olivier: liability must therefore be limited to the consequences willed by a person
.g T

whilst aware of their wrongfulness, and the wrongful consequences that he reasonably should have
foreseen and prevented
w RG

⇒ It is a fact that in most cases of delictual liability legal causation is not expressly raised because it is
evident that the consequences caused wrongfully and culpably must be imputed to the actor

⇒ Intent and negligence cannot serve as satisfactory criteria for legal causation
w
w

88. explain why intent cannot serve as criterion for legal causation

⇒ Intent cannot serve as a criterion for legal causation

⇒ The court determined the limits of the defendants liability with reference to the reasonable foreseeability
test (which is subordinate to the flexible criterion whereby legal causation is presently established)

⇒ Van der Merwe on the basis of an argument that the intentional defendant acted negligently with reference to
those circumstances which he did not intend: “a person can have intent in respect of some of the wrongful
consequences of his act, while being negligent or even innocent in respect of others. From the fact that intent
and negligence cannot overlap conceptually, it must be concluded that an alleged wrongdoer who caused
consequences A and B intentionally, cannot be negligent in respect of consequences C and D, which he had not
willed”

⇒ The question is no longer whether the wrongdoer has to be blamed for causing the consequences
concerned (as a result of his intent this is an established fact), but rather whether the consequences of
his intentional (blameworthy) conduct can ne imputed to him
Page | 34
Page 35 of 50
35

89. explain why negligence cannot serve as criterion for legal causation

⇒ The criterion for negligence cannot, as in the case of intent, serve as a test for determining the
imputability of damage. The test for negligence is whether the reasonable person ,in the same position as
the wrongdoer, would have foreseen and prevented either injury to another in general (abstract approach)
or the consequence concerned (concrete approach)

⇒ It is sufficient according to the abstract approach, if damage in general is reasonably foreseeable, the question
whether a wrongdoer is liable for a specific consequence has to be determined separately by applying one or
other of the different criteria for legal causation and not with reference to the question whether the wrongdoer had
been negligent with reference to that specific consequence

⇒ The concrete (“relative”) approach renders an investigation into legal causation unnecessary because
wrongfulness and negligence are determined with reference to a specific consequence. According to this
approach, it is unnecessary to undertake an independent investigation into imputability of damage because of the
concrete test for wrongfulness and negligence supposedly contain all the elements necessary to keep liability
within acceptable limits

⇒ Even those who prefer the abstract approach to negligence seldom need to apply legal causation expressly,
because in most cases it is apparent that the damage that has been caused by the conduct of the actor, who
acted wrongfully and negligently, must be imputed to him, with the result that it is unnecessary to enter into the

za
matter separately. This applies especially where the concrete approach to negligence is followed
ls s
o.
⇒ The question whether a wrongdoer should be held liable for a “remote consequence”, is completely different from
ria al
the question: .c
to ori

∗ whether the wrongdoer’s conduct was unreasonable according to the legal convictions of the
community (the question of wrongfulness), from the question
∗ whether the wrongdoer should be legally blamed because he foresaw and reconciled himself with the
rtu ut

consequence and the possible wrongfulness thereof (the question of intent),


∗ and from the question whether injury was foreseeable with such a degree of probability that the
.g T

reasonable man would have taken steps to avoid injury (the question of negligence)
w R

The fact that legal causation entails considerations of a nature completely different from that of fault, is emphasised by
G

the necessity of its application in the case of strict liability

90. explain reasonable foreseeability as test for legal causation and be able to apply it
w

⇒ Has been used in a number of decisions as a criterion for legal causation, but in terms of the prevailing flexible
approach it plays a subsidiary role just like al the other traditional test for legal causation
w

⇒ It would be possible in a given matter, merely on the basis of legal policy, to impute liability in terms of
the flexible approach even where the damage was so exceptional that it could not be described as
reasonably foreseeable

⇒ The foreseeability test, the decision simply being that a specific result was foreseeable or not and that is
the end of the matter

Reasonable foreseeability may also serve as a (subsidiary) criterion for the imputability of harm in cases of intentional
wrongful conduct and liability without fault

Page | 35
Page 36 of 50
36

91. write brief notes on the relationship between reasonable foreseeability and the flexible approach to legal
causation

⇒ It would be possible in a given matter, merely on the basis of legal policy, to impute liability in terms of
the flexible approach even where the damage was so exceptional that it could not be described as
reasonably foreseeable

⇒ The foreseeability test, the decision simply being that a specific result was foreseeable or not and that is
the end of the matter

⇒ Van der Walt and Midgley: “it is not necessary that all the consequences of the defendant’s conduct
should have been foreseen; only the general nature or the kind of harm which actually occurred must
have been reasonably foreseeable. However, the risk of harm must have been a real risk, which a
reasonable person would not have brushed aside as being far fetched”

⇒ Van Rensburg general test: was the consequence, as well as the casual progression between the act and
the consequence, at the time of the act foreseeable with such a degree of probability that the
consequence can, in light of the circumstances, reasonably be imputed to the alleged wrongdoer. the
general rule should not apply that “an alleged wrongdoer is normally liable for all the consequences of his
culpable, wrongful act except for the consequences that were highly improbable

za
⇒ Reasonable foreseeability may also serve as a (subsidiary) criterion for the imputability of harm in cases of
ls s
o.
intentional wrongful conduct and liability without fault
ria al
.c
to ori

92. explain the meaning and role of an actus novus interveniens in the case of legal causation, and be able to
rtu ut

apply this knowledge to factual situations


.g T

⇒ Novus actus interveniens= new intervening cause- is an independent event which, after the wrongdoers act
w R

has been concluded, either caused or contributed to the consequence concerned.


G

⇒ Difficult to determine when a Novus actus interveniens influences the result to such an extent that the result
should no longer be imputed to the actor, although his conduct remains a factual cause of the result. The effect of
a Novus actus interveniens is obviously to limit the liability of the wrongdoer and, thus, it plays an
important role in legal causation
w

⇒ Whether the Novus actus has had the effect of severing the legal nexus with the result that the consequence
w

should not be imputed to the actor.

Ø When applying the flexible approach, the question is whether the novus actus between the defendants
conduct and the relevant consequence has been such that the consequence cannot be imputed
directly to the defendant on the basis of policy, reasonability, fairness and justice.
Ø In applying the direct consequences test, the question is whether the novus actus breaks the
“directness” of the consequence which is required for liability
Ø When applying foreseeability, the question is whether the novus actus influences the degree of
foreseeability to such an extent that it may be said that the consequence was not reasonably
foreseeable as a result of the novus actus.

⇒ In most cases such a novus actus is one of many circumstances which influence the question of
imputability of loss by means of the flexible approach. The primary question is whether the consequence
should reasonably be imputed to the defendant. The presence or not of a novus actus is therefore of secondary
importance and is merely one of a number of factors which may be considered in answering the primary question

⇒ An event will qualify as a novus actus interveniens only if the event was not reasonably foreseeable

Page | 36
Page 37 of 50
37

93. explain the meaning and role of the so-called egg-skull case of legal causation, and be able to apply this
knowledge to factual situations

⇒ Egg-skull cases arise where the plaintiff, because of one or other physical, psychological or financial
weakness, suffers more serious injury or loss as a result of the wrongdoer’s conduct than would have
been the case if the plaintiff did not suffer from such a weakness

⇒ The egg-skull rule had its origin in English decision Dulieu v White and Sons, expressed in the maxim
“the wrongdoer must take the victim as he finds him” (also identified as the talem qualem rule)

⇒ In such a case the wrongdoer should also be liable for the harm which maybe ascribed to the existence of
the weakness concerned

⇒ Van Rensburg: in these cases liability may still be explained with reference to the reasonable foreseeable
norm. as a result of the particular circumstances present in these cases, the precise manner in which the
consequences occur need not be foreseeable with the same degree of probability which applies in normal
cases

⇒ Van der Walt and Midgley link the egg-skull rule to the direct consequences theory, also “ take the victim
as you find him”

za
⇒ Van der Merwe and Olivier, who strictly adhere to fault as a criterion for the imputability of harm, contend that
“the reasonable man cannot be expected to foresee the unforeseeable” and declare that the notion that “
ls s
o.
you must take your victim as you find him” should be rejected insofar as the reasonable person would not
ria al
have foreseen the consequences concerned and that the injured party should bear the loss himself
.c
to ori

⇒ The most acceptable approach to the so-called egg-skull cases is made possible by the flexible criterion for legal
causation and illustrated by the judgement of Smit v Abrahams. The basic question is not whether the damage
rtu ut

was a direct consequence or reasonably foreseeable, but whether in light of all the circumstances of the
case , amongst others the egg-skill situation, the damage should reasonably be imputed to the defendant
.g T
w RG

Damages

94. write brief notes on the compensatory function of the law of delict
w

⇒ The general compensatory function of the law of delict implies that there must be some loss or damage
w

for which the law makes compensation available.

⇒ Compensation for damages: “damages” is a monetary equivalent of damage awarded to a person with the
object of elimination as fully as possible his past as well as future patrimonial and, where applicable, non-
patrimonial damage

⇒ Satisfaction: implies the reparation of damage in the form of injury to personality by inter alia effecting
retribution for the wrong suffered by the plaintiff and by satisfying the plaintiffs and/or community’s
sense of justice

95. define the concept of damage

Damage is the detrimental impact upon any patrimonial or personality interest deemed worthy of protection by law

Page | 37
Page 38 of 50
38

96. explain that damage is a wide concept including both patrimonial and non-patrimonial loss

⇒ The question arises whether “damage” indicates any for of harm whatsoever or whether some forms of harm
are excluded. The concept of damage does, included more than harm for which compensation is recoverable since
satisfaction may be awarded for some forms of damage

⇒ Damage is only that reduction of the utility of interests which has been brought about by n uncertain event. A reduction
in utility which is sure to take place because of, for example, wear and tear, illness due to natural causes, death and
consumption cannot be regarded as damage

⇒ Damage includes patrimonial (pecuniary) as well as non-patrimonial (non pecuniary) loss: damage is a broad
concept which consists of patrimonial as well as non-patrimonial loss (injury to personality). Thus damage is a
comprehensive concept with patrimonial and non-patrimonial loss as its two mutually exclusive components

97. define patrimonial loss

⇒ Patrimonial loss can be directly or naturally expresses in money, while non-patrimonial loss is at most only
indirectly measurable in this way

98. write brief notes on a persons patrimony

za
ls s
⇒ There is no generally accepted definition of a person’s patrimony. In terms of the juridical concept of patrimony it

o.
consists of all his patrimonial rights (subjective rights with a monetary value), his exceptions to acquire patrimonial
ria al
rights and all legally enforceable obligations (or exceptions) with monetary value
.c
to ori

⇒ Positive elements of someone’s patrimony: this refers to all a person’s patrimonial rights such as real rights,
immaterial property rights, and personal rights. Expectations of patrimonial benefits are also part of a person’s estate
rtu ut

and this is the legally accepted expectation to acquire patrimonial rights in future
.g T

⇒ Negative elements of someone’s patrimony: someone’s patrimony is burdened or reduced by the creation,
acceleration or increase of a monetary debt or liability. A debt constitutes damage even though the debtor has no assets
w R

to pay such debt. An expectation of debt is also part of a person’s patrimony


G

99. explain the methods by which patrimonial loss and the extent thereof are determined in a particular case, and be
able to apply these methods

⇒ The sum-formula approach: according to this approach damage consists in the negative difference between the
w

relevant person’s current patrimonial position (after the event complained of) and his hypothetical
patrimonial position that would have been his current position if the event had not taken place.
w

⇒ This test is recognised by the SCA- the reason why a hypothetical (potential) patrimonial position of the plaintiff
is used is to provide for prospective damage. Loss of profit and certain other forms of damage

⇒ A concrete concept of damage: the AD have used this method- the difference between the patrimonial position of
the prejudiced person before the wrongful act and thereafter... damage us the unfavourable difference caused by
the wrongful act

⇒ Van der Walt- he observes that the concept of damage logically relates to a comparative method- the real
question is what is compared and how the comparison takes place. What was must be compared with what is and
that the current position is not to be compared with the hypothetical position which would have existed had the
delict not been committed.

⇒ This is in accordance with actual legal practice. It is suggested that our law should adopt and follow the
concrete concept of damage except in instances of prospective loss, liability for misrepresentation and
loss of profit i.e. in all instances where the use of a hypothetical test is necessary

⇒ Time for the assessment of damage: according to current authority the date of commission of a delict is generally
the decisive moment for assessing damage (and this includes future loss). The date of commission of a delict

Page | 38
Page 39 of 50
39
is the earliest date on which all the elements of a delict are present. If all the other requirements of are
present, the date on which the first damage is manifested is used

100. explain the “once and for all” rule, and be able to apply it

⇒ “Once and for all” rule- a plaintiff, who claims damages on a specific cause of action, has only one chance
to claim damages for all damage already suffered as well as prospective loss.

⇒ Formulation and implications of the rule: the plaintiff must claim for all damages already sustained or
expected in the future insofar as it is based on a single cause of action

⇒ Prescription in regard to a claim for damages commences as soon as a cause of action accrues and the
debt in respect of the payment of damages is claimable. Prescription is concluded after 3 years

⇒ A plaintiff who has sued with or without success for a part of his damage, may not thereafter sue for
another part in both claims are based on a single cause of action

101. explain the collateral source rule in one sentence

⇒ A damage-causing event often not only causes loss but also result in the plaintiff receiving some benefit
⇒ There are practical guidelines on which benefits ay be taken into account in particular circumstances in reducing

za
the amount of damages to which the plaintiff is entitled or which benefits are to be ignored
ls s
o.
ria al
102. write brief notes on the plaintiff’s duty to mitigate
.c
to ori

o It is a principle of the law of delict that a plaintiff may not recover damages for a loss which is the factual
rtu ut

result of the defendant’s conduct but which could have been avoided if the plaintiff had taken reasonable
steps
.g T

o The plaintiff is obliged to take all reasonable steps to limit the damage caused by the defendant’s delict.
w R

This duty arises as soon as the plaintiff in fact suffers loss and knows or should reasonably know that he
has to mitigate his damage. A plaintiff who fails to mitigate his loss in this manner cannot recover damages in
G

respect of loss that he could reasonably have prevented

o A plaintiff who has taken steps to mitigate his loss may also recover damages for any loss caused by such
reasonable steps
w

o Where the plaintiff has reduced his damages by taking reasonable steps in mitigation, the defendant is only liable
to compensate him for the actual loss he sustained even if the plaintiff did more than the law required of him
w

o The onus of proving that the plaintiff did not properly fulfil his duty to mitigate rests on the defendant. If the
defendant has proved an unreasonable failure on the part of the plaintiff to mitigate his loss the plaintiff has to
prove what his loss would have bee had he taken reasonable steps
o

103. explain briefly what non-patrimonial loss (or injury to personality) is

Non-patrimonial (non-pecuniary) damage or injury to personality

o Non patrimonial damage is the detrimental impact (change in or factual disturbance of) personality interests
deemed worthy of protection by the law and which does not affect the patrimony

o Interest of personality: the different rights to personality provide an indication of the relevant personality interests
and thus also of non-patrimonial loss. There are rights of personality is regard to the following: physical-
mental integrity, liberty, reputation, dignity, privacy, identity and feelings

Page | 39
Page 40 of 50
40

Delictual remedies

104. name the different remedies that may be instituted on the basis of a delict (i.e. the so called 3 pillars on which
the law of delict rests, as well as the other delictual remedies)

The South African law of delict rests on the three pillars: the actio legis Aquiliae, the actio iniuriarium and the
action for pain and suffering

105. indicate whether the 3 main delictual actions are transmissible

o The question of their transmissibility (whether they are transferrable and cedable): the Aquilian action is actively
as well as passively heritable; similarly a claim under this action is freely cedable, litis contestatio (closing of
proceedings) has no effect in this regard. The actio iniuriarium and the action for pain and suffering are actively
as well as passively heritable only after litis contestatio, the claim therefore lapses if the plaintiff or the
defendant dies before litis contestatio. Claims under these actions are also not cedable, in any case not before
litis contestatio.

za
ls s
o.
ria al
106. briefly discuss the purpose, forms, function and requirements of an interdict
.c
to ori

o Is a legal remedy with which a person can avert and impending wrongful act or prevent the continuation
rtu ut

of a wrongful act that has already commenced


2 forms: namely prohibitory (prohibits the wrongdoer from committing a wrongful act at all or from
.g T

o
continuing with a wrongful act) and mandatory (requires a positive conduct on the part or the wrongdoer
w R

to terminate the continuing wrongfulness of an act that has already been committed
G

o Has a preventative function


o Fault is not a requirement in this regard in either our common law or case law
o 3 requirements for the granting of an interdict:
w

1. there must be an act by the respondent- either already commenced or threatening, may be a comissio or
w

an omissio
2. the act must be wrongful-there must be a threat to or an infringement of a so-called clear right of the
applicant
3. no other remedy must be available to the applicant- an interdict may either be final or temporary

107. write brief notes on concurrence of remedies

o One and the same act may in principle result in several- different or alternative- remedies. An act from
which various claims arise, each of which places a distinctive action at the plaintiff’s disposal gives rise to a
different remedy

o An act from which only one or more claims arise, but which offer a choice between different remedies,
results in alternative remedies

o Concurrence of the 3 most important delictual actions amongst themselves: the concurrence of the actio
iniuriarium and the contractual action. That of the action for pain and suffering and the contractual action,
Page | 40
Page 41 of 50
41
as well as the choice between the actio legis Aquiliae and the contractual action

108. write brief notes on a so-called exclusionary clause

o Exclusionary clauses: parties to a contract may restrict their liability- contractual as well as delictual-
through a so-called exclusionary clause. The precise restriction on the wrongdoer’s liability will depend on
the interpretation of the clause concerned

109. explain the principles concerning prescription of remedies, and apply them

o The Prescription act 68 of 1969, a delictual debt prescribes (and the delictual action is thus also
extinguished) 3 years after it originated
rd
o 3 party claims under the Road Accidents Fund act 56 of 1996 the period is 2 or 3 years.

o The period of prescription commences the moment all the elements of a delict are present had the
creditor has knowledge (or ought reasonably to know) of the identity of the wrongdoer and the facts of the

za
case
ls s
o.
ria al
Joint wrongdoers .c
to ori

110. explain what a “joint wrongdoer” is as defined in terms of the apportionment of damages act 34 of 1956
rtu ut

Where persons co-operate consciously to commit a delict, they were joint wrongdoer’s; where more than one
.g T

o
person by independent wrongful conduct contributed casually to the same harmful consequence, they were
w R

considered to be concurrent wrongdoer’s


G

o Presently the position is regulated by the Apportionment of Damages act 34 of 1956. The act abolishes the
common law distinction between joint wrongdoers and concurrent wrongdoers.

o Joint wrongdoers are now defined as person who are jointly liable in delict for the same damage
w
w

111. explain how joint wrongdoing is regulated in terns of the apportionment of Damages act and apply this
knowledge o factual situations

o Where more than one person therefore causes separate damaging consequences with regard to a
plaintiff, each is, according to the ordinary principles of delict, only liable for the specific damage he has
caused

o Joint wrongdoers are in solidum for the full damage. The plaintiff therefore has the right to sue whichever
joint wrongdoer he chooses for the full amount of damages. Joint wrongdoers may also be sued in the
same action

o If the court is satisfied that all the joint wrongdoers are before it, it may apportion the damages among
them on the basis of their relative degrees of fault, and may give judgement against every wrongdoer for
his part of the damages

If the plaintiff recovers only part of his damages from a wrongdoer, he may sue any other wrongdoer for the balance. If
a joint wrongdoer pays more than is justified by the degree of his fault, he may exercise his right of recourse against
any of the other joint wrongdoers

Page | 41
Page | 42
w
w G
w R
.g T
rtu ut
to ori
ria al
ls s
.c
o.
za
Page 42 of 50
42
Page 43 of 50
43

Psychological lesions

112. describe psychological lesions

⇒ Any recognisable harmful infringement of the brain and nervous system of a person. The existence of such a
lesion, should, as a rule, be proved by supporting psychiatric evidence

⇒ Emotional shock: a sudden painful emotion or fright resulting from the awareness or observation of an
overwhelming or disturbing event which causes unpleasant emotions such as fear, anxiety or grief.

⇒ Emotional shock may be caused by the prejudiced person’s fearing for his own safety, the safety of another
person, or even the safety of his property, by observing a gruesome accident, by learning of the death of a relative
or a loved one or by experiencing other disturbing events

113. name the locus classicus (trendsetting case) in the field of psychological lesions

The decision of the appeal court in Bester v Commercial Union Versekeringmatskappy van SA is the locus

za
classicus for the view point that impairment of personality and patrimonial loss resulting from psychiatric injury or
emotional shock caused wrongfully and negligently (or intentionally) founds the action for pain ad suffering and the
ls s
o.
actio legis Aquiliae in principle. Prior to Bester, the South African law of delict lacked clear principles in this field
ria al
.c
to ori
rtu ut
.g T

114. name the 2 artificial restrictions on the delictual principles which were initially applied by our courts in
w R

determining liability for psychological lesions


G

⇒ The courts consistently sought guidance from English law. This resulted n the imposition of 2 artificial restrictions
on liability for emotional shock (a) the shock (or psychological disturbance) must have originated from a physical
injury or resulted in harm to the physical constitution; and (b) the aggrieved party himself must have been in
personal danger of being physically injured
w
w

Page | 43
Page 44 of 50
44

115. describe the principles that were introduced by Bester v Commercial union in the place of the 2 old
restrictions

a) Wrongfulness: the requirement of physical harm indicates an infringement of the right to physical
integrity which is per se wrongful. This requirement was rejected in Bester- according to which the brain
and nervous system are as much a part of the physical body as an arm or a leg. As a result a physical
injury is not absolutely necessary to found liability

The effect of equating physical and psychological harm is that even a slightly emotional shock will in
principle also infringe the personality right to physical integrity and consequently be wrongful.
This is simply an application of the maxim de minimus non curat lex. To be actionable, the harm caused
by the shock must be reasonably serious

b) Negligence and legal causation: rejected in Bester ad replaced by the yardstick of reasonable
foreseeability of harm. Dealing with the question of either negligence or legal causation with regard to
emotional shock

The question of emotional shock arises where the shock or psychiatric injury is the only or at least (one of)
the first harmful consequence(s) of the wrongdoer’s conduct.

za
In order to establish negligence, the reasonable foreseeability and preventability of the psychological
lesions must be ascertained. ls s
o.
ria al
Where the emotional shock is further (subsequently or more remote) consequences of the wrongdoers
.c
to ori

already established negligent act, the question of legal causation is at hand, namely whether the
wrongdoer’s negligent act can be regarded as the legal cause of the psychological lesion
rtu ut

The view that the reasonable foreseeability of remote psychological lesions is concerned with legal causation is not
.g T

supported in Barnard- according to both the court a quo and the SCA one is still concerned with the question of
negligence.
w R

This view is subject to criticism, on the one hand because the question of the preventability of the psychological lesion
G

involved does not make sense in light of the wrongdoer’s already established negligence, and on the other, because
the determination of negligence with regard to the nervous shock through the application of the foreseeability test only,
incorrectly reduces negligence to reasonable foreseeability and consequently wrongly equates the two concepts. The
conclusion is that the test for negligence is not appropriate to ascertain liability for remote consequences
w
w

116. name the factors that may play a role in determining whether psychological lesions were reasonably
foreseeable

The following factors may play a role in this regard:

- the fact that the psychological lesion resulted from physical injury, was connected with such injury or sustained
together with it,
- the fact that the plaintiff was in personal danger of being physically injured, the fact that the plaintiff was informed of
the death or injury of a close friend or relative,
- the fact that the plaintiff personally witnessed the death or injury of someone with whom the plaintiff had a close
relationship

It must be noted that once the court has found that reasonably serious emotional shock was reasonably
foreseeable, the wrongdoer is then liable for any detrimental physical or mental consequences ensuing from
the emotional shock, regardless of whether such consequence was reasonably foreseeable as well

⇒ The so-called thin skull or talem qualem rule, namely that “the wrongdoer takes his victim as he finds him”
applies here. according to this rule the defendant cannot escape the liability by proving that the plaintiff
was particularly susceptible to the prejudicial consequences of the shock and the that the consequences
were therefore not reasonably foreseeable
Page | 44
Page 45 of 50
45

Injury or death of another; pure economic loss; negligent misrepresentation; interference with a contractual
relationship, unlawful competition, manufacturer’s liability

117. explain what is meant by pure economic loss

⇒ The Acquilian action is in principle available to claim damages for pure economic loss
⇒ On the one hand, pure economic loss may compromise patrimonial loss that does not result from damage to
property or impairment of personality. ( negligent misrep or unlawful competition)
⇒ On the other hand, pure economic loss may refer to financial loss that flows from damage to property or
impairment of personality, but which does not involve the plaintiff's property or person; or if it does, the defendant
did not cause such damage or injury

The wrongdoers conduct must comply with the general delictual requirements

118. name 5 other specific forms of damnum iniuria datum

za
5 specific forms of damnum iniuria datum
ls s
o.
1. Injury or death of another person
ria al
.c
to ori

2. Negligent misrepresentation

3. Interference with a contractual relationship


rtu ut

4. Unlawful competition
.g T

5. Manufacturer’s liability
w RG

The right to physical integrity; the right to a good name or fama; rights relating to dignitas
w
w

119. define defamation and give examples of this iniuria

o The intentional infringement of another person’s right to his good name

o Defamation is the wrongful, intentional publication of words or behaviour concerning another person
which has the effect on injuring his status, good name or reputation

120. name and discuss the elements of defamation

Elements
Publication
o Defamation will rise only if the defamatory statement or behaviour has been published or disclosed to a
rd
3 person

o This requirement is satisfied if the words or conduct are made known or disclosed to at least one person other that
the plaintiff himself

o This general principle is subject to important qualifications


Page | 45
Page 46 of 50
46
o The courts do not consider the disclosure of defamatory words or behaviour to an outsider who is
unaware of the defamatory character or meaning thereof in relation to the plaintiff as publication
rd
o The communication of defamatory words concerning a 3 party by one spouse to another does not constitute
publication according to the decision in Whittington v Bowles

o Once publication is established, the plaintiff must prove that the defendant was responsible for the
publication. As a general rule that publication is attributed to the defendant if he was aware or could
reasonably have expected that an outsider would take cognisance of the defamation. Whether the result
objected to was foreseen or was at least reasonable foreseeable.

o Not only the person from whom the defamatory mark originated, but also any person who repeats,
confirms or even draws attention to it, is in principle responsible for its publication

Defamatory effect: wrongfulness

o When determining wrongfulness, the question whether the good name of the person involved has in fact
(factually) been infringed is irrelevant.

o The only relevant question is whether, in the opinion of the reasonable man (person) with normal intelligence and
development the reputation of the person concerned has been injured (thus an objective approach) if so, the
words or behaviour are defamatory to, and in principle (prima facie) wrongful as against that person

o This test must be seen as a particular embodiment of the boni mores or reasonable criterion which is the

za
general yardstick for wrongfulness
ls s
o.
ria al
o The following principles have crystallised in practice with regard to the application of this test:
a) The reasonable person is the financial, normal, well-balanced and right-thinking person, who is
.c
to ori

neither hypercritical nor oversensitive, but someone with normal emotional reactions
b) The reasonable person is someone who subscribes to the norms and values of the constitution that must
inform all law. The constitutional principles must therefore be the basis upon which the values and views of
rtu ut

reasonable members of the community must be determined


.g T

c) The reasonable person is a member of society in general and not only of a certain group. The alleged
defamation must thus have the effect of harming the plaintiffs good name in the eyes of all reasonable
w R

persons in society
d) The reaction of the reasonable person is dependant upon the circumstance of the particular case. The alleged
G

defamation must therefore be interpreted in the context in which it is published


e) Verbal abuse is in most cases not defamatory because it normally does not have the effect of injuring
a person’s good name.
f) Words (or behaviour) are prima facie or according to their primary meaning defamatory or non defamatory.
Words may also have a secondary meaning which is an extraordinary meaning attached to the by a person
w

with knowledge of special circumstances. The plaintiff may show that words which are in their primary sense
non-defamatory have a secondary defamatory meaning (innuendo). Both the primary and secondary meaning
w

is ascertained objectively by means of the reasonable person test

If the words have ambiguous meaning, the meaning most favourable to the defendant must be followed.
The plaintiff who proves that words or behaviour are defamatory in the judgement of the reasonable person, does
not thereby prove that a wrongful act has been committed against him.

The plaintiff must therefore expressly aver and rove that the defamation pertains to his good name. the
test to ascertain this connection is again that of a reasonable person, namely, whether the defamatory publication
can be linked to the plaintiff according to the judgement of the reasonable person

Page | 46
Page 47 of 50
47

121. name, discuss and apply the traditional grounds of justification for defamation

o A presumption of wrongfulness then arises which places he onus of the defendant to rebut it. He may do this by
proving the existence of a ground of justification(privilege, truth and public interest and fair comment) for his
conduct

o Privilege or privileged occasion:

o Privilege exists where someone has a right, duty or interest to make a specific defamatory assertions and
the persons or people to whom the assertions are published have a corresponding right, duty or interest
to learn of such assertions

o A distinction must be made between absolute and relative privilege:


√ Absolute privilege means that the defendant is protected absolutely in the sense that liability for
defamation is completely excluded e.g. Members of parliament are given freedom of speech during
debates or other proceedings of parliament.

√ In the case of relative privilege the defendant enjoys only provisional or conditional protection, this
protection falls away as soon as the plaintiff proves that the defendant exceeded the bounds of the
privileged occasion. Catagories of this privilege:

∗ Discharge of a duty or furtherance of an interest: where a person has a legal, moral or social duty or a

za
legitimate interest in making defamatory assertions to another person who has a corresponding duty or
interest to learn of the assertions. The existence of a social or moral duty or interest must on the other
ls s
hand must be ascertained objectively by means of the reasonable person test

o.
ria al
.c
If it is proved that both parties had a corresponding duty or interest, then the defendant must further prove
to ori

that he acted within the scope or limits of the privilege- he must prove that the defamatory assertions were
relevant to, or reasonably connected with the discharge of the duty of the furtherance of the interest the
rtu ut

plaintiff may still show that the defendant in fact exceeded the limits of the privilege because he acted with
an improper motive
.g T

∗ Judicial or quasi-judicial proceedings: are defamatory statements made during the course of judicial or
w R

quasi-judicial proceeding and applies to all participants therein. The defendant need only prove that the
G

statements were relevant to the matter at issue. The plaintiff may the prove that, notwithstanding their
relevance, the statements were not supported by reasonable grounds. In the absence of relevance or
reasonable grounds the defendant exceeds the limits of this privilege and acts wrongfully. The plaintiff
may show that the defendant exceeded the limits because he acted with improper motive

∗ Privileged reports: defamation contained in the publication of the proceedings of the courts, parliament
w

and certain public bodies. The defendant must prove that the reporting was fair and substantially accurate
w

account of the proceedings. The provisional protection ill fall away if the plaintiff proves that the defendant
acted with an improper motive

Truth and public interest

⇒ The prima facie wrongfulness of the defendant’s conduct will be cancelled if he proves that the defamatory
remarks were true and in the public interest. The defendant need only prove that the remarks were substantially
not literally true. All depends on the convictions of the community (public policy) which depends on the time, the
manner and the occasion of the publication. Unlike in the case of privilege, the limits of this defence are not
exceeded if the defendant acted with malice

Media privilege

⇒ Concerns the reasonable publication of false or untrue defamatory statements by the media
⇒ This defence must be dealt with caution
⇒ When determining the reasonableness of the publications, the legal convictions of the community of our country
must be applied.
⇒ Several factors that the not meant to be decisive, can be considered in this regard: the public interest, the extent
and nature of the allegations, the nature of the information upon which the allegations were based, the nature and

Page | 47
Page 48 of 50
48
mass of the mediums used; the extent of distribution and the sector of the public at which the publication is
aimed; the reliability of the information, steps taken to verify the information, the extent to which other material
supports the allegations at the time of publication the opportunity given to the relevant person to react to the
allegations, the necessity or urgency to publish before the truth can be positively verified; the possibility that the
same objective could be reached in a less harmful manner, and the presence of a malicious motive

Political privilege

⇒ This defence is analogous to media privilege and entails the reasonable publication of (false or untrue)
defamatory allegations on the political terrain
⇒ The same factors as in media privilege are taken in account including that the publication must be made “with the
reasonable belief that the statements made are true

Fair comment

⇒ If the defendant proves that the defamation forms part of a fair comment on facts that are true and in the
public interest
⇒ 4 requirements:
1. The defamation must amount to comment and not to the assertion of an independent fact. The test is that of
the reasonable person
2. The comment must be fair- the comment must be relevant to the facts involved and convey the honest and
bona fide opinion of the defendant
3. The facts on which the comment is based must be true

za
4. These facts must be in the public interest

ls s
o.
ria al
122. discuss the grounds on which intent can be excluded in a case of defamation
.c
to ori

⇒ Grounds excluding intent:


rtu ut

v Mistake: if a person is unaware of the wrongfulness of his defamatory publication, because for whatever
reason, he bona fide thinks or believes that his conduct is lawful, consciousness of wrongfulness, an essential
.g T

element of intent, and therefore also intent, are absent as a result of this mistake
w R

v Mistake is determined subjectively


G

v In the case of an unreasonable mistake the defendant s held liable on the ground of his negligence

v Jest: if the defendant proves that he published the defamatory words in jest, I circumstances where his will
was not directed at the infringement of the prejudiced person’s right to good name, direction of will, as an
essential element of intent is absent and he should be able to rebut the presumption of aminus iniuriandi
w
w

v The courts incorrectly do not follow this approach. For a successful plea of jest, the courts require that the
(reasonable) bystander should also have regarded the words as a joke

Damage caused by animals

123. discuss the requirements for the actio de pauperie and apply them to a given factual situation

♣ The prejudiced person may claim damages fro the owner of a domestic animal which has caused damage.
♣ Fault on the part of the owner is not a requirement for liability

♣ Requirements to succeed with the Action de pauperie:

a) The defendant must be the owner of the animal when the damage is inflicted

b) The animal must be a domestic animal

c) The animal must act contra naturam sui generis when inflicting the damage

Page | 48
Page 49 of 50
49
♣ The animal involved must have acted objectively seen, contrary to what may be expected of a decent
and well-behaved animal of its kind
♣ The animal must have caused the damage spontaneously from “inward excitement or vice” or sponte
feriate commotai
♣ The animal does not act contra naturam if it reacting to external stimuli. The rule is, however, not
consistently applied by the courts
♣ Defences against the actio de pauperie relating to spontaneous conduct:

- culpable conduct on the part of the prejudiced person,


rd
- culpable conduct on the part of an outsider- where the animal is provoked the a 3 part the owner is not
rd
liable, the same applies where the damage may be attributed to the negligence of a 3 party provided
rd
that the 3 party was in charge or control of the animal and by his negligent conduct failed to prevent
the animal from injuring the victim ;and
- Provocation by another animal.

♣ These cases have the effect of excluding liability because the animal did not act from “inward
excitement or vice”. The defence of volenti non fit iniuria in the form of voluntary assumption of risk is
also available to the defendant

d) The prejudiced person or his property must be lawfully present at the location where the damage is
inflicted

♣ Some cases require a “lawful purpose” and others a “legal right” on the part of the prejudiced person in
order to establish a lawful presence at the location involved.

za
♣ The test for a “legal right” is narrower that “lawful purpose” since a person, who has a legitimate purpose,
ls s
o.
may not necessarily have the right to be at the place. The “legal right” approach is preferable because one
ria al
cannot always determine what the aim or purpose of property, being a lifeless object is
.c
to ori

♣ Both patrimonial damages and satisfaction may be claimed. The extent of the defendants liability should be
limited in accordance with the flexible criterion for legal causation
rtu ut
.g T

124. discuss the requirements for the actio de pastu and apply them to a given situation
w R

♣ Damages are claimed from the owner of an animal which caused loss by eating plants
G

♣ Still part of our law

♣ 3 requirements:
1. The defendant must be the owner of the animal when the damage is caused
w

2. the animal must cause damage by eating plants


3. the animal must act of its own volition when causing the damage
w

♣ Fault on the part of the prejudiced party constitutes complete defences against the actio de pastu. Culpable
conduct on the part of an outsider does not exclude the actio de pastu

Vicarious liability

125. define vicarious liability

⇒ The strict liability of one person for the delict of another

126. name 3 relationships where vicarious liability may apply

Applies where there is a particular relationship between two persons: employer-employee, principal-agent and motor
car owner- motor car driver

Page | 49
Page 50 of 50
50

127. name and discuss the requirements for an employers liability for a delict committed by an employee

⇒ Where an employee acting within the scope of his employment, commits a delict, his employer is fully liable for the
damage

⇒ The rationale for or basis of the employer’s liability is controversial. Best known one is: the employer liability is
founded on his own fault (culpa in eligendo)

⇒ Other theories/ rationale: the interest or profit theory according to which the employer must also bear the
burden of the employee’s services; the identification theory according t which the employee is only the
employer’s arm; and the solvency theory according to which an employer is liable because he is normally in a
better position financially than the employee

⇒ The convincing theory is the risk or danger theory which furnishes the true rationale for the employer liability- the
work entrusted to the employee creates certain risks of harm for which the employer should be held liable on the
rd
grounds of fairness and justice, as against injured 3 parties

⇒ 3 requirements for vicarious liability:


1. There must be an employer-employee relationship at the time when the delict is committed: a contract
of service must exist. A contract of mandate (involves an independent contractor) in terms of which one
person undertakes to render services to another for remuneration without being subject to the control of the

za
other, does not found vicarious liability
- The question of control, which does not mean factual control but the capacity or right of control
ls s
o.
- First the AD employed the dominant impression test to determine whether the dominant impression is that
ria al
or a contract of service or a contract of mandate. Later it was held that in determining the relationship
.c
to ori

between the parties is a multi-faceted test should be utilised, taking into account all relevant factors and
the circumstances of the specific case.
- The state is in the same position as other employers
rtu ut

2. the employee must commit a delict- due to the fact that the employee is also delictually liable, the employer
.g T

and employee are in principle regarded as joint wrongdoers as against the prejudiced party. However, a right
of recourse is only available to the employer
w RG

3. the employee must act within the scope of his employment when the delict is committed- if he acts in
the execution or fulfilment of his duties in terms of the employment contract

128. name the requirements for liability of the owner of a motor vehicle for a delict committed by the driver of
w

the motor vehicle


w

⇒ where a motor car owner allows someone else (who is not his employee) to drive his car and the driver negligently
causes an accident the owner is fully liable for the loss provided that

(a) the owner must request the driver to drive the vehicle or supervise his driving

(b) the vehicle must be driven in the interest of the owner

(c) the owner must retain a right (power) of control over the manner in which the vehicle is driven
vicarious liability may explained with reference to the risk theory

Page | 50
MAY/JUNE 2022

QUESTION 01

blamed for conduct which is careless; by giving insufficient attention to his actions and
Negligence is thetoblameworthy
failing to adhere the standardattitude
of care of someone
legally that of
required had acted
him. Thewrongfully
defendantand
is is
negligent if the reasonable person would have acted differently, if the unlawful causing
of damage was reasonably foreseeable and reasonably preventable.
The test of negligence was formulated in Kruger v Coetzee and is also known as
reasonable person test. The test follows three steps. A reasonable in the same position
as defendant would have:
(a) Foreseen the possibility of his conduct injuring another and causing him patrimonial
loss.
(b) Would take reasonable steps to prevent such occurrence
(c) And the defendant failed to take such steps.
John has foreseen the possibility that not taking his medication may cause him to have
black outs while on duty, but nevertheless he took a risk not to take his medication to
prevent such occurrence of having black outs at work and to prevent harm from
occurring, therefore John’s conduct was negligent.

QUESTION 02

An act can usually be wrongful if it has some consequence. The determination of


wrongfulness entails a dual investigation. Firstly it has to be ascertained whether the
perpetrator’s act was in fact the cause of a harmful result to another person and
secondly it must be ascertained whether the causing of harm took place in an
unreasonable or legally reprehensible way.
The question deals with wrongfulness of an omission. An omission is wrongful if the
defendant is under the legal duty to act positively to prevent harm suffered by the
plaintiff. The basic question to determine whether an omission is wrongful is whether
legal duty to act was present and was breached. This is determined with reference to
the legal convictions of the community (boni mores). Factors which may serve as
indications that a legal duty rested on the defendant include: prior conduct (omissio per
2|Page
commissio); control of a dangerous object; rules of law; a special relationship between
the parties; particular office; contractual undertaking for the safety of a third party; and
creating of an impression that the interests of a third person will be protected. If a
person’s conduct is contra boni mores that is to say if its contrary to the legal
convictions of the community, then that person would have acted in a wrongful manner.
The court follows an objective test based on reasonableness, based on legal
convictions of community and on all circumstances. The question asked is: Did the
defendant infringe interest of plaintiff reasonably or unreasonably? Court must weigh
conflicting interests in light of circumstances.

In the so-called municipality cases, prior conduct was considered to a prerequisite for
the wrongfulness of a commission. Prior conduct refers to the creation of a new source
of danger and failing to eliminate it.
In Halliwell v JHB municipality a wagon slipped on cobblestones and court held that a
legal duty that arises from prior conduct was needed.
In Minister van Polisie v Ewels the court held that the existence of a legal duty is
determined by boni mores and prior conduct is only an indication thereof. Not a
prerequisite.
In the Carmichele case the court held that the State was delictually liable for damages
arising out of the unlawful omissions of its servants. The Constitutional court made it
clear that boni mores must be informed by values underpinning the Bill of Rights.
A defendant is under a legal duty to act positively to prevent harm to the plaintiff if it is
reasonable to expect of the defendant to have taken positive measures to prevent the
harm. In the scenario in question the Shop-till-you-drop shopping mall the management
did not take all the necessary measures required to warn third parties of the puddle of
sanitizers. They should have put a sign indicating there’s puddle of water or mop the
sanitizers. Their prior conduct had imputed a legal duty on them to notify any third
parties of the harm an omission to do such constituted an act of wrongfulness. If the
principles of the Carmichele case are followed then the conduct of the Shop-Till-You-
Drop management was wrongful.

QUESTION 03

From the given facts, we can conclude that the defendant (Steady Wheels) has been
negligent, but the plaintiff (Mike) appears to have been negligent too. Thus we must
consider whether contributory negligence was present.

3|Page
Contributory negligence is negligence on the part of the plaintiff, and it is a defense that
the defendant can raise. The Apportionment of Damages Act 34 of 1956 is applicable.
This Act provides that a contributorily negligent plaintiff’s damages be apportioned. The
court will determine the degree of deviation from the reasonable person standard shown
by the conduct of both the defendant and the plaintiff, express the deviation as
percentages, and use these percentages as a basis for the apportionment.
According to the Smit and Nomeka cases, the percentages of negligence attributed to
the defendant and plaintiff respectively will always add up to a hundred per cent.
According to Jones NO v Santam Bpk 1965 case, both percentages must be assessed
independently, which could mean that, for example, a defendant may be 80% negligent
while the plaintiff is 30% negligent.
According to King v Pearl Insurance case a defense of contributory negligence could
not succeed where the plaintiff had omitted to wear a crash-helmet while driving a
scooter, but had not been negligent in respect of causing the accident. However, in
Bowkers Park Komga Cooperative Ltd v SAR and H case,the court held that
contributory negligence did not refer to negligence in respect of the damage-causing
event, such as a motorcar accident, but to negligence in respect of the damage itself,
and this was confirmed by the Appellate Division in Union National South British
Insurance Co Ltd v Vitoria 1982 and General Accident Versekeringsmaatskappy SA
Bpk v Uijs 1993 cases. Therefore, failure to wear a safety helmet would constitute
contributory negligence if it contributed to the plaintiff’s damage.
Applying these principles to the facts, we can conclude that Mike was contributorily
negligent. Steady wheels will not be liable for damages incurred. Mike’s claim will be
instituted against Steady wheels instead of David.

QUESTION 04

(a) Factual causation is the conditio sine qua non test, or ‘‘but for test’. Mentally
eliminating, or thinking away, the conduct. If the damage then also
disappears, a factual causal link is present between the conduct and the
damage. The condition sine qua non test is used to determine whether there
was factual causal link between two parties.
If the test is applied to the facts, we must conclude that if Tom had not did
not knocked the ladder that William was standing on, William would not have
fell and injured his leg and needed to go to the hospital and broke his leg after
being discharged from hospital, therefore a factual causal link is present
between Bargain buy’s conduct and Alex’s damage.

4|Page
(b) The test for legal causation is the so-called flexible approach, as formulated in
S v Mokgethi and International Shipping Co (Pty) Ltd v Bentley case. In
Mokgethi a bank robber shot a teller. The teller was rendered a paraplegic
and was discharged from hospital in a wheelchair. Subsequently, the
paraplegic man failed to shift his body position in the chair frequently and
developed pressure sores, eventually dying from complications. The question
that arose was whether the shot fired by the robber was the legal cause of the
teller’s death.

According to the court, the main question in respect of legal causation is


whether there is a close enough relationship between the wrongdoer’s
conduct and its consequence for such consequence to be imputed to the
wrongdoer in view of policy considerations based on reasonableness,
fairness and justice. Several other legal causation theories exist, such as
adequate causation, direct consequences, foreseeability and novus actus
interveniens. None of these criteria is suitable to be applied to all situations.
They may, however, be used as subsidiary aids when employing the flexible
approach. In the Mokgethi case, the court held that the shot was not a legal
cause of the death.
Applying the principles to the facts in the question, the conclusion is probably
that William broken leg was too remote and should not be imputed to the
wrongdoer. It could also be argued that a so-called novus actus interveniens,
that is, a new intervening act, was constituted by the fall on William’s conduct.

5|Page
QUESTION 05

Necesseity will be suitable ground of justification in the scenario


Ben will be able to raise necessity as a defence against Connie. Necessity must be
clearly distinguished from private defence. The distinction is that when acting in defence
the actor’s conduct is directed at an attack by the wrongdoer. When acting out of
necessity, his conduct violates the interests of an innocent party. The fact that Jenna
incited the dog means that the act was actually a human act by her using the dog as an
instrument, however, the dog is a legal object of Connie and therefore by killing the dog
Ben violated Connie’s (an innocent party) subjective right to his dog. Ben will still be
able to raise a defence of necessity.

QUESTION 06

Actio de pauperize action will be available to Jane. To be successful with


the actio de pauperie against the owner of a domestic animal that injured or
harmed a person, it has to be establish that the animal acted ferociously or
contrary to its nature and that the conduct of the animal caused the damages
suffered by the person.

To succeed in bringing the actio de pauperie, the following requirements must be


met:
(a) ) The defendant must be the owner of the animal when the damage is
inflicted.( Mere control over the animal is insufficient for the purpose of the
action)
(b) The animal must be domesticated animal.
(c) The animal must act contra naturam sui generis when inflicting the damage.
(d) ) The prejudiced person or his property must be lawfully present at the
location where the damage is inflicted

6|Page
QUESTION 07

(a) Provocation
(b) Unjustified Enrichment
(c) Private Defence

QUESTION 08

(a) Assault
(b) Invasion of Privacy

7|Page
Contents
PVL3702 – 201 – 2015...........................................................................................................................1
PVL3702 – 201 – 2018...........................................................................................................................5
PVL3702 – 201 – 2019.........................................................................................................................16
PVL3702 – Questions May 2013..........................................................................................................27
PVL3702_201_2009.............................................................................................................................28
PVL3702_201_2010.............................................................................................................................32
PVL3702_May 2013.............................................................................................................................35
PVL3702_Oct_2013.............................................................................................................................46
PVL3702_May 2014.............................................................................................................................54
PVL3702_Oct_2014.............................................................................................................................64
PVL3702_201_2016.............................................................................................................................74
PVL3702_201_2017.............................................................................................................................82
PVL3702_201_2018.............................................................................................................................91
PVL3702_201_2012.............................................................................................................................96
PVL3702_201_2013...........................................................................................................................100
PVL3702_201_2014...........................................................................................................................105
PVL3702_201_2011...........................................................................................................................109
PVL3702_ Various MCQ.....................................................................................................................113
ASSIGNMENT 1 – 2009......................................................................................................................117
ASSIGNMENT 2 – 2009......................................................................................................................121
ASSIGNMENT – 2010 – SEMESTER 1..................................................................................................128
ASSIGNMENT – 2010 – SEMESTER 2..................................................................................................133
ASSIGNMENT – 2008 – SEMESTER 1..................................................................................................136
PVL3702_May_2016..........................................................................................................................141
PVL3702_201_2013...........................................................................................................................147
PVL3702_201_2014...........................................................................................................................150
PVL3702_Revision Pack.....................................................................................................................153
PVL3702_May_2015..........................................................................................................................204
PVL3702_Oct_2015...........................................................................................................................237
PVL3702_Question from Study Unit..................................................................................................262
Study Unit 6 & 7_ OFFER AND ACCEPTANCE......................................................................................266
Case law.........................................................................................................................................266
1. CRAWLEY REX........................................................................................................................266
2. Carlill v. Carbolic Smoke Ball Co. [1893] Q.B. 256 (C.A.).........................................................266
3. Bloom v The American Swiss Watch Company......................................................................266
PROBLEM TYPES QUESTION: OFFER AND ACCEPTANCE.................................................................267
Question: ABSENCE OF CONSENSUS..............................................................................................269
Question: MISREPRESENTATION...................................................................................................271
Question: STUDY UNIT 13: DURESS...................................................................................................273
Question: Study Unit 14 UNDUE INFLUENCE.....................................................................................275
STUDY UNIT 15: COMMERCIAL BRIBERY............................................................................................276
Question: STUDY UNIT 17 and 18 : LEGALITY: ILLEGAL CONTRACTS THAT ARE VOID.........................276
STUDY UNIT 19: FORMALITIES...........................................................................................................277
Question STUDY UNIT 20: POSSIBILITY..............................................................................................278
STUDY UNIT 23: TERMS.....................................................................................................................279
3. Ticket cases....................................................................................................................................280
STUDY UNIT 24: INTERPRETATION.....................................................................................................281
STUDY UNIT 25-39: BREACH AND REMEDIES FOR BREACH................................................................281
SU 27: REPUDIATION.........................................................................................................................284
SU 28: PREVENTION OF PERFORMANCE............................................................................................284
SU 29: REMEDIES ON THE GROUND OF BREACH OF CONTRACT.......................................................285
SU 30: THE EXCEPTIO NON ADIMPLETI CONTRACTUS.......................................................................285
SU 31: RESCISSION (CANCEL).............................................................................................................285
SU 32: DAMAGES...............................................................................................................................285
SU 33: PENALTY CLAUSES...................................................................................................................304
SU 34: THE EXCEPTIO DOLI................................................................................................................305
SU 35: THE TRANSFER OF CLAIMS: CESSION......................................................................................305
SU 36, 37 & 38:..................................................................................................................................306
12 WAYS OF TERMINATING YOUR OBLIGATIONS:..........................................................................306
SU 36: THE TERMINATION OF OBLIGATIONS......................................................................................306
SU 37: THE TERMINATION OF OBLIGATIONS......................................................................................307
SU 38: THE TERMINATION OF OBLIGATIONS......................................................................................307
PVL3702_201_2019 – Second Semester............................................................................................311
ASSIGNMENT 02 – FIRST SEMESTER: UNIQUE NUMBER: 787088......................................................314
PVL3702_ Assignment 694865..........................................................................................................317
PVL3702_Oct 2016............................................................................................................................318
PVL3702_May 2017...........................................................................................................................330
PVL3702_Nov 2017............................................................................................................................340
PVL3702_May 2018...........................................................................................................................350
PROPERTY CASES:..............................................................................................................................359
REAL RIGHTS v CREDITORS RIGHTS................................................................................................359
OWNERSHIP AND LIMITATIONS ON OWNERSHIP...........................................................................359
Konstanz Property (Pty} Ltd:..........................................................................................................361
Info Plus:........................................................................................................................................361
Telkom SA Ltd v Xsinet (Pty) Ltd.....................................................................................................362
SERVITUDES: LIMITED REAL RIGHTS..............................................................................................362
Grant v Stonestreet....................................................................................................................362
Willoughby’s Consolidated........................................................................................................363
LIMITED REAL RIGHTS:...................................................................................................................363
REAL SECURITY...........................................................................................................................363
Mapenduka v Ashington............................................................................................................363
Qsry...........................................................................................................................................364
CONSTITUTIONAL LAW IN RESPECT OF PROPERTY........................................................................364
FNB v COMMISSIONER, SARS:....................................................................................................364

PVL3702 – 201 – 2015


Question

John, a racehorse owner, advertises for sale the horse Fire for R1.5 million. In the advertisement it is
stated that Fire is an offspring of the legendary July winner, Lightning. Peter is a horse breeder who
specifically wishes to introduce the bloodline of Lightning into his stud. He agrees orally with John to
buy Fire for R1.5 million. Later, in order to meet the requirements of the horse breeders’ association,
John has a written contract drawn up which Peter signs without reading. The contract makes no
mention of Fire's ancestry, but does contain a clause exempting John from liability for any
representations made during negotiations or in the contract. Peter's attention is not drawn to these
facts. A month later Peter finds out that Fire is in fact not an offspring of Lightning, although at the
time of the conclusion of the contract John genuinely and without any fault on his part believed that
to be the case. Advise Peter on whether the contract of sale is valid. Substantiate your advice and
refer to relevant case law. Apply the subjective approach of the courts in answering this question. Do
not apply the Consumer Protection Act to this question. (10)
Answer

Identifying the problem

The subjective approach of the courts involves the application of the will theory as qualified by
estoppel or quasi-mutual assent (Hutchison and Pretorius (eds) The law of Contract in South Africa
Oxford University Press Southern Africa 2012 90-97). A successful reliance on estoppel can only give
rise to a ‘fictional’ (thus not valid) contract (Hutchison and Pretorius Contract 94) and will thus not be
discussed.

The facts seemingly indicate that John and Peter have not reached consensus based on the will
theory. If that is the case, it is necessary to determine if Peter may be held bound to a contract with
John, based on the reliance theory.

Discussing the relevant law applicable to the problem AND applying the law to the facts of the
problem

The subjective approach implies that we first have to determine whether agreement between the
parties exists as required in terms of the will theory or whether a party acted under a material
mistake. Consensus has three elements (Hutchison and Pretorius Contract 14 85): the parties must
seriously intend to contract, be of one mind as to the material aspects of the proposed agreement
(the terms and the identity of the parties to it), and be conscious of the fact that their minds have
met.

In the present case the innocent misrepresentation regarding Fire’s lineage is irrelevant because it
caused a non-material mistake: a mistake regarding a characteristic (the lineage) of the thing sold,
Fire (an error in substantia). The parties wanted to buy and sell the same horse, Fire. See Eiselen GTS
et al Law of contract Only study guide for 3702 Unisa 2012 40: Hutchison and Pretorius Contract 88-
89.

The parties were, however, not in agreement as to the consequences they wished to create: Peter
did not know that there was a clause in the contract he signed exempting John from liability for any
representations made during negotiations or in the contract, but John knew that the sale included an
exemption clause. Peter made a mistake as to the obligations the parties wished to create which
excludes consensus between the parties (Hutchison and Pretorius Contract 86). No contract can thus
arise on the basis of the will theory.

The facts of our problem are very similar to that in Du Toit v Atkinson's Motors Bpk 1985 (2) SA 889
(A) where the appellant signed an agreement without reading it which contained a term excluding
the respondent’s liability for misrepresentation. The court held that the mistake regarding the
exemption clause was material.

This type of mistake also occurred in other cases. In Allen v Sixteen Stirling Investments (Pty) Ltd
1974 (4) SA 164 (D) the plaintiff believed that he was purchasing the erf shown to him by the seller's
agent, while the written contract that he signed indicated the correct erf which was a completely
different property. His mistake related to performance and was material. In Sonap Petroleum (SA)
(Pty) Ltd (formerly known as Sonarep (SA) (Pty) Ltd) v Pappadogianis 1992 (2) SA 234 (A) the
appellant erred with regard the period of the lease which was an aspect of the performance.

A valid contract could still arise in terms of the doctrine of quasi-mutual assent or direct reliance
theory (Hutchison and Pretorius Contract 95-97 103-105). The court stated the test in Sonap
Petroleum (SA) (Pty) Ltd (formerly known as Sonarep (SA) (Pty) Ltd) v Pappadogianis 1992 (2) SA 234
(A) 239-240 as follows:

In my view, therefore, the decisive question in a case like the present is this: did the party whose
actual intention did not conform to the common intention expressed, lead the other party, as a
reasonable man, to believe that his declared intention represented his actual intention? … To answer
this question, a three-fold enquiry is usually necessary, namely, firstly, was there a misrepresentation
as to one party’s intention; secondly, who made that representation; and thirdly, was the other party
misled thereby? … The last question postulates two possibilities: Was he actually misled and would a
reasonable man have been misled?

One of the parties to the contract, Peter, misrepresented his intention to be bound by the contract by
signing the contract. Although it could be argued that John was actually misled by this
misrepresentation of Peter, it is clear that a reasonable person in the position of John would not have
been misled thereby. John knew in fact that there was no exemption clause in the oral contract while
the written contract had such a clause. John should have realised that Peter could have thought that
the written contract was also without such a clause and he thus had a legal duty to point out to Peter
the presence of this clause in the written contract. There was either no actual or at least reasonable
reliance on the part of John.

The giving of appropriate advice

The written contract of sale is invalid because of the lack of actual and apparent consensus.

Total: [10]

Question 1

Which concept is NOT a value that informs the law of contract?

1 Freedom of contract.
2 Good faith.
3 Privity of contract.
4 The requirement that a contract must not be against public policy.
5 Pacta sunt servanda.

Question 2
Which statement is CORRECT regarding unlawful contracts?

1 All unlawful contracts are void.


2 Some contracts that have been criminalized by statute are not void.
3 Some unlawful contracts are unenforceable in terms of the in pari delicto potior condicio
possidentis rule.
4 All unlawful contracts are voidable.
5 All contracts that are prohibited by statute in order to protect the revenue of the state are void

Question 3
X makes an offer to Y to purchase Y’s car. X sends the offer by e-mail to Y. Y reads the offer on 13 May
and drafts a written acceptance on 14 May. Y posts his acceptance on 15 May to X. X receives the
acceptance on 17 May in his post-box and reads it on 18 May. When was the contract concluded?

1 13 May.
2 14 May.
3 15 May.
4 17 May.
5 18 May.

Question 4
In a contractual context, where the debtors are jointly liable only, and the co-creditors may only
claim performance jointly, this is a case of

1 proportionate liability.
2 simple joint liability.
3 in solidum liability.
4 joint and several liability.
5 collective joint liability.

Question 5
Which statement regarding the interpretation of contracts is INCORRECT?

1 The parol evidence rule has an integration and interpretation aspect.


2 The distinction between background and surrounding circumstances is imprecise.
3 The primary rule is to give effect to the intention of the party who drafted the contract.
4 Where a term is ambiguous it should be interpreted against the party who proposed it.
5 Where a term is ambiguous it should be given a meaning that makes it legally effective.

Question 6
X and Y agree that should X sell her leather couch, she (X) will offer to sell it to Y first, before making
an offer to sell the couch to any other person. X sells the couch to Z for R10 000 without first offering
it to Y for sale. Delivery of the couch has not yet taken place. Which statement is CORRECT?

1 X and Y concluded an option contract.


2 The contract between X and Z is voidable, because it breaches the contract between X and Y.
3 Y has a personal right against X, but Z has a real right against X.
4 Both Y and Z only have personal rights against X.
5 The personal right of Z enjoys preference above the personal right of Y, because it arises from a
contract of sale.

Question 7
Assume the same facts as in question (6). What remedy does Y have against X?

1 An interdict.
2 A claim for damages.
3 A claim for specific performance of the contract of sale after Y has stepped into Z’s shoes with a
unilateral declaration of intent made to X.
4 All the above remedies.
5 None of the above remedies

Question 8
Essentialia are:

1 All the terms of a contract apart from the naturalia.


2 Terms that identify a contract as belonging to a particular class of contracts.
3 Terms automatically imposed by law on the contracting parties unless contracting parties expressly
exclude them.
4 All the terms of a contract apart from the incidentalia.
5 Material terms and conditions of a contract.

Question 9
Y purchased from Z a specific painting for R150 000. At the time of contracting Y honestly believed it
to be an original Da Vinci painting, but Z did not know of Y’s belief. The painting was later found to be
a copy. Y argues that the contract is void whilst Z maintains that the contract is valid. Which answer
reflects the CORRECT legal position?

1 The contract is valid, because Y’s mistake regarding the painting is only an error in motive.
2 The contract is void, because Y and Z acted under a common error regarding the painting.
3 The contract is void, because of Y’s unilateral mistake regarding the painting.
4 The contract is void, because Y’s mistake regarding the painting is an error in corpore.
5 The contract is void, because of Y’s supposition regarding the painting.

Question 10
X, an organiser of art exhibitions, contracted with Y for an exhibition to be held on 24 to 27 July.
These dates were the only dates mentioned during the negotiations. After having been pressurized
by X, Y hurriedly signed the standard form contract without reading it. The contract contained a
clause permitting X to change the dates of the exhibition unilaterally. Thereafter X changed the dates.
X had no reason to believe that Y would have signed the contract if he had known of the term. Which
statement(s) is / are INCORRECT?

1 Y can cancel the contract because of her material mistake with regard to the presence of a clause in
the standard form contract allowing X to unilaterally change the dates of the exhibition.
2 Y can prove that a contract with X exists without the clause in the standard form contract allowing
X to unilaterally change the dates of the exhibition.
3 Y can use the iustus error approach to prove that a contract with X exists without the clause
allowing X to unilaterally change the dates of the exhibition.
4 Option 1 and 3.
5 Option 1, 2 and 3.

PVL3702 – 201 – 2018


Question
Read the judgment in Steyn v LSA Motors Ltd 1994 (1) SA 49 (A). Identify the judgment of the
Appellate Division case on which this judgement is based as well as the relevant section in the
prescribed textbook. Read both. You can find judgments in conventional law libraries, online at the
website of the Southern African Legal Information Institute (SAFLII) (www.saflii.org) or as an e-
resource on the Unisa Library site (choose Juta Law Online Publications and then South African Law
Reports). Then use those authorities to answer the following question:
John, a racehorse owner, advertises for sale the horse Fire for R1.5 million. In the advertisement it is
stated that Fire is an offspring of the legendary July winner, Lightning. Peter is a horse breeder who
specifically wishes to introduce the bloodline of Lightning into his stud. He agrees orally with John to
buy Fire for R1.5 million. Later, in order to meet the requirements of the horse breeders’ association,
John has a written contract drawn up which Peter signs in John’s presence without reading. The
contract makes no mention of Fire's ancestry, but does contain a clause exempting John from liability
for any representations made during negotiations or in the contract. Peter's attention is not drawn to
the provisions of this clause. A month later Peter finds out that Fire is in fact not an offspring of
Lightning, although at the time of the conclusion of the contract John genuinely and without any
fault on his part believed that to be the case. Advise Peter on whether the contract of sale is valid.
Substantiate your advice and refer to relevant case law. Do not apply the law with regard to
misrepresentation or the Consumer Protection Act 68 of 2008 to this question. (10)

Answer
In order to identify the law, which you should apply to the problem, you were asked to:
(1) read Steyn v LSA Motors Ltd 1994 (1) SA 49 (A) to determine the ratio decidendi of the case;
(2) identify the Appellate Division case on which the ratio decidendi of Steyn was based;
(3) identify the relevant section(s) in the textbook; and
(4) not to discuss misrepresentation.

(1) The ratio decidendi of the Steyn case


The court held that Steyn could not accept the offer, because the offer was not directed at him. A
reasonable person in Steyn’s position would have realised that the offer was open only to
professional golfers and, accordingly, that he had no contractual claim to the car. This answers the
third leg of the test in Sonap Petroleum (SA) (Pty) Ltd (formerly known as Sonarep (SA) (Pty) Ltd) v
Pappadogianis 1992 (2) SA 234 (A).

(2) Identify the AD case on which the ratio decidendi in Steyn was based
The decision in Steyn case, was thus based on Sonap Petroleum (SA) (Pty) Ltd (formerly known as
Sonarep (SA) (Pty) Ltd) v Pappadogianis 1992 (2) SA 234 (A).
The court stated the test as follows (239-240):
“In my view, therefore, the decisive question in a case like the present is this: did the party whose
actual intention did not conform to the common intention expressed, lead the other party, as a
reasonable man, to believe that his declared intention represented his actual intention? … To answer
this question, a three-fold enquiry is usually necessary, namely, firstly, was there a misrepresentation
as to one party’s intention; secondly, who made that representation; and thirdly, was the other party
misled thereby? …The last question postulates two possibilities: Was he actually misled and would a
reasonable man have been misled?”

(3) Identify the relevant section in the textbook


If the “Table of cases” is consulted at the end of Hutchison and Pretorius (eds) The law of Contract in
South Africa 3rd ed (2017) Oxford, Cape Town you will find the pages where Steyn and Sonap are
discussed in the textbook. One of the pages on which Steyn is discussed is page 109. In footnote 214
the Steyn case is identified as an example of an instance where there is no objective appearance of
agreement. Remember that, before the iustus error approach can apply, there must be a clear,
objective agreement between the parties, such as when the parties have signed a contractual
document. In Steyn, the court did not apply the iustus error approach, because no apparent or
ostensible contract existed between the parties. Steyn had one interpretation of the offer (that it was
open to all players) and the sponsor another (that it was only open to professional players).
Consequently, there was no clear, apparent contract and the iustus error approach could not be
applied.
In Hutchison and Pretorius Contract 106-108 a discussion of the Sonap case is to be found. In this
discussion the test quoted above in (2) is identified as the reliance theory.
It is thus clear from the above that the subjective approach of the courts (will theory as qualified by
the reliance theory) should be applied to the assignment problem.

(4) Do not discuss misrepresentation


This misrepresentation that you must not discuss cannot be the same as the misrepresentation in the
first leg of the three-fold test of Sonap (there must be a misrepresentation of the intention of one of
the parties), because we know we have to base our answer on Steyn and Sonap that both deal with
error.
So what does the instruction mean? In the given facts, Peter made an innocent misrepresentation
with regard to Fire’s lineage. This is thus the misrepresentation we need not discuss in so far it does
not cause a material mistake.

Identifying the problem


The subjective approach of the courts involves the application of the will theory as qualified by quasi-
mutual assent (Hutchison and Pretorius Contract 93-100).
The facts seemingly indicate that John and Peter have not reached consensus based on the will
theory. If that is the case, it is necessary to determine if Peter may be held bound to a contract with
John, based on the reliance theory.
Discussing the relevant law applicable to the problem AND applying the law to the facts of the
problem
The subjective approach implies that we first have to determine whether agreement between the
parties exists as required in terms of the will theory or whether a party acted under a material
mistake. Consensus has three elements (Hutchison and Pretorius Contract 87):
 The parties must seriously intend to contract,
 Be of one mind as to the material aspects of the proposed agreement (the terms and the
identity of the parties to it), and
 Be conscious of the fact that their minds have met.
In the present case the innocent misrepresentation regarding Fire’s lineage causes a mistake
regarding a characteristic (the lineage) of the thing sold, Fire (an error in substantia). There are two
views on whether such a mistake is material (Hutchison and Pretorius Contract 90-92). The one view
is that the mistake is not material (Trollip v Jordaan 1961 1 SA 238 (A); The other view is that such a
mistake is material (Spenmac (Pty) Ltd v Tatrim CC 2015 3 SA 46 (SCA). The latter view cannot be
correct as the parties wanted to buy and sell the same horse, Fire.
The parties were, however, not in agreement as to the consequences they wished to create: Peter
did not know that there was a clause in the contract he signed exempting John from liability for any
representations made during negotiations or in the contract, but John knew that the sale included an
exemption clause. Peter made a mistake as to the obligations the parties wished to create which
excludes consensus between the parties (Hutchison and Pretorius Contract 88). No contract can thus
arise on the basis of the will theory.
The facts of our problem are very similar to that in Du Toit v Atkinson's Motors Bpk 1985 2 SA 889 (A)
where the appellant signed an agreement without reading it which contained a term excluding the
respondent’s liability for misrepresentation. The court held that the mistake regarding the exemption
clause was material.
This type of mistake also occurred in other cases. In Allen v Sixteen Stirling Investments (Pty) Ltd
1974 (4) SA 164 (D) the plaintiff believed that he was purchasing the erf shown to him by the seller's
agent, while the written contract that he signed indicated the correct erf which was a completely
different property. His mistake related to performance and was material. In Sonap Petroleum (SA)
(Pty) Ltd (formerly known as Sonarep (SA) (Pty) Ltd) v Pappadogianis 1992 2 SA 234 (A) the appellant
erred with regard the period of the lease which was an aspect of the performance.
A valid contract could still arise in terms of the doctrine of quasi-mutual assent or direct reliance
theory (Hutchison and Pretorius Contract 98-100 and 106-108). The court stated the test in Sonap
Petroleum (SA) (Pty) Ltd (formerly known as Sonarep (SA) (Pty) Ltd) v Pappadogianis 1992 2 SA 234
(A) 239-240 as follows:
“In my view, therefore, the decisive question in a case like the present is this: did the party whose
actual intention did not conform to the common intention expressed, lead the other party, as a
reasonable man, to believe that his declared intention represented his actual intention? … To answer
this question, a three-fold enquiry is usually necessary, namely, firstly, was there a misrepresentation
as to one party’s intention; secondly, who made that representation; and thirdly, was the other party
misled thereby? … The last question postulates two possibilities: Was he actually misled and would a
reasonable man have been misled?”
One of the parties to the contract, Peter, misrepresented his intention to be bound by the contract by
signing the contract. Although it could be argued that John was actually misled by this
misrepresentation of Peter, it is clear that a reasonable person in the position of John would not have
been misled thereby. John knew in fact that there was no exemption clause in the oral contract while
the written contract had such a clause. John should have realised that Peter could have thought that
the written contract was also without such a clause and he thus had a legal duty to point out to Peter
the presence of this clause in the written contract. There was either no actual or at least reasonable
reliance on the part of John.
The giving of appropriate advice
The written contract of sale is invalid because of the lack of actual and apparent consensus.
Total: [10]

Question 1
Which statement is INCORRECT?

1 Obligationary agreements create one or more obligations.


2 Absolving agreements discharge or extinguish obligations.
3 Real agreements transfer rights.
4 Transfer agreements transfer rights.
5 All binding agreements are contracts.

Question 2
Which statement regarding the iustus error doctrine is CORRECT?

1 The iustus error doctrine qualifies the objective approach of our courts to error.
2 The iustus error doctrine is very similar to estoppel but does not require fault and prejudice as
estoppel does.
3 The iustus error doctrine can be used to prove the existence of a contract on the basis of quasi
mutual assent.
4 The iustus error doctrine amounts to a direct application of the reliance theory.
5 Option 1 and 4.

Question 3
Which cause of action is/are delictual?

1 Mistake.
2 Culpable misrepresentation.
3 Innocent misrepresentation.
4 Dictum et promissum.
5 All of the above.

Question 4
In which instance has consensus been obtained improperly?

1 A common error.
2 Rectification.
3 An error in substantia.
4 A threat that is not related to an imminent or inevitable evil.
5 An innocent misrepresentation.
Question 5
Which cause(s) of action may render the contract void?

1 A fraudulent and negligent misrepresentation.


2 Duress.
3 Undue influence.
4 A reasonable and material mistake.
5 Commercial bribery

Question 6
X is employed as a bookkeeper in Y's business. X steals money from the business's bank account over
a long period of time. Y gets forensic auditors in and they determine that X has stolen R50 000. Y
confronts X and threatens to lay a charge of theft against X at the police station unless X signs an
acknowledgement of debt for R50 000 and undertakes to pay back the money in monthly
instalments of R500 each. X signs because he is afraid to go to jail. Which statement regarding the
presence of the requirements for duress is INCORRECT?

1 X had a reasonable fear.


2 Ys threat weakened X’s power of resistance and rendered X's will compliant.
3 There was a threat of an imminent evil to X.
4 The threat against X was not contra bonos mores.
5 The pressure Y used against X caused X to suffer damage.

Question 7
X sent an offer by email to Z on 1 February. Z downloaded the email to her computer on 6 February,
but only read it on 7 February. Z sent an email to X on 8 February, in which she accepted the offer. Z’s
email reached X’s service provider on 9 February and could have been downloaded by X on that date.
X only downloaded Z’s email and read it on 11 February. When was the contract between X and Z
concluded?
1 6 February.
2 7 February.
3 8 February.
4 9 February.
5 11 February.

Question 8
X promises to give Y R10 000 if Y successfully passes her matric examination at the end of the year.
This is an obligation subject to a:

1 suspensive time clause.


2 resolutive time clause.
3 suspensive condition.
4 resolutive condition.
5 modus.
Question 9
Y sells his car to Z for R20 000 on 15 January. Y undertakes to deliver the car to Z on 17 January. The
undertaking to deliver the car on 17 January is a

1 suspensive time clause.


2 essentialium (singular for essentialia).
3 incidentalium (singular for incidentalia).
4 option 1 and 2.
5 option 1 and 3.

Question 10
The courts use the hypothetical bystander test when determining the possible existence of

1 essentialia.
2 naturalia.
3 terms implied by law.
4 tacit terms.
5 express terms.

Question 1
X, an organiser of art exhibitions, contracted with Y for an exhibition to be held on 24 to 27 July.
These dates were the only dates mentioned during the negotiations. After having been pressurized
by X, Y hurriedly signed the standard form contract without reading it. The contract contained a
clause permitting X to change the dates of the exhibition unilaterally. Thereafter X changed the dates.
X had no reason to believe that Y would have signed the contract if he had known of the term. Y
averred that the contract was void. Will Y succeed in his attempt to have the contract set aside?
Substantiate your answer and refer to relevant case law. Apply the direct reliance approach of the
courts in answering this question. Do not apply the Consumer Protection Act to this question. (10)

Answer
Identifying the problem
The facts seemingly indicate that X and Y have not reached consensus based on the will theory. If so,
it is necessary to determine if Y may be held bound to a contract with X, based on the reliance
theory, or whether Y will escape liability. Only the direct approach to the reliance theory will be
considered.

Discussing the relevant law applicable to the problem, referring to the relevant case law, AND
applying the law to the facts of the problem

The direct reliance approach can only be applied after it has been determined that Y acted under a
material mistake. It must thus be determined whether agreement (consensus ad idem) as a
contractual basis exists between the parties, as required in terms of the will theory.
The first step is to determine whether agreement (consensus ad idem) as a contractual basis exists
between the parties, as required in terms of the will theory. Consensus has three elements
(Hutchison and Pretorius (eds) The law of Contract in South Africa (Oxford University Press Southern
Africa 2012) 14 85): the parties must seriously intend to contract, be of one mind as to the material
aspects of the proposed agreement (the terms and the identity of the parties to it), and be conscious
of the fact that their minds have met.
In the present case the parties were not in agreement as to the consequences they wished to create:
Y thought that the dates for the art exhibition (X’s performance) was fixed, while X knew that the
contract allowed X to unilaterally change the dates. This is a mistake as to the obligations the parties
wished to create which excludes consensus between the parties (Hutchison and Pretorius Contract
86). No contract can arise on the basis of the will theory. This type of mistake can be illustrated with
a number of cases.
In Allen v Sixteen Stirling Investments (Pty) Ltd 1974 (4) SA 164 (D) the plaintiff believed that he was
purchasing the erf shown to him by the seller's agent, while the written contract that he signed
indicated a completely different property. His mistake related to performance and was material. See
also Du Toit v Atkinson's Motors Bpk 1985 (2) SA 889 (A). The appellant signed an agreement
containing a term excluding the respondent from liability for misrepresentation. Finally see Sonap
Petroleum (SA) (Pty) Ltd (formerly known as Sonarep (SA) (Pty) Ltd) v Pappadogianis 1992 (2) SA 234
(A) where the appellant erred with regard the period of the lease which was an aspect of the
performance.
The direct reliance approach can now be applied to the facts of the problem (Hutchison and
Pretorius Contract 18-19 95-97 103-105). The court stated the test in Sonap Petroleum (SA) (Pty) Ltd
(formerly known as Sonarep (SA) (Pty) Ltd) v Pappadogianis 1992 (2) SA 234 (A) 239-240 as follows:
In my view, therefore, the decisive question in a case like the present is this: did the party whose
actual intention did not conform to the common intention expressed, lead the other party, as a
reasonable man, to believe that his declared intention represented his actual intention? … To answer
this question, a three-fold enquiry is usually necessary, namely, firstly, was there a misrepresentation
as to one party’s intention; secondly, who made that representation; and thirdly, was the other party
misled thereby? … The last question postulates two possibilities: Was he actually misled and would a
reasonable man have been misled?
A discussion of Ridon v Van der Spuy and Partners (Wes-Kaap) Inc 2002 (2) SA 121 (K) and Steyn v LSA
Motors Ltd 1994 (1) SA 49 (A) will also be appropriate.
By signing the contract, Y, a party to the contract, misrepresented his intention to be bound by the
clause allowing X to unilaterally change the dates. X knew that the only dates mentioned during the
negotiations were 24 to 27 July, that Y hastily signed the contract and that the contract had a clause
allowing X to unilaterally change the dates. Although it could be argued that X was not actually
misled by Y’s misrepresentation, it is clear that a reasonable person would not have been misled in
any case. Indeed, X had no reason to believe that Y would have signed the contract had Y known of
the term allowing X to change the dates of the exhibition unilaterally. In fact X had a legal duty to
point out the presence of this clause in the agreement to Y. There was either no actual or at least
reasonable reliance on the part of X.

The giving of appropriate advice


Y is not bound by the agreement with X because of the lack of actual and apparent consensus.
Total: [10]

Question 2
X, an organiser of art exhibitions, contracted with Y for an exhibition to be held on 24 to 27 July.
These dates were the only dates mentioned during the negotiations. After having been pressurized
by X, Y hurriedly signed the standard form contract without reading it. The contract contained a
clause permitting X to change the dates of the exhibition unilaterally. Thereafter X changed the dates.
X had no reason to believe that Y would have signed the contract if he had known of the term. Y
averred that the contract was void. Will Y succeed in his attempt to have the contract set aside?
Substantiate your answer and refer to relevant case law. Apply the indirect reliance approach of the
courts in answering this question. Do not apply the Consumer Protection Act to this question. (10)

Answer
Identifying the problem
The facts seemingly indicate that X and Y have not reached consensus based on the will theory. If so,
it is necessary to determine if Y may be held bound to a contract with X, based on the reliance
theory, or whether Y will escape liability. Only the indirect approach to the reliance theory will be
considered.
Discussing the relevant law applicable to the problem, referring to the relevant case law, AND
applying the law to the facts of the problem
The indirect approach is the iustus error-approach. A party who acted under a mistake and wishes to
escape liability (Y in our case) must prove that his / her mistake is material and reasonable.
At the outset it must be determined whether agreement (consensus ad idem) as a contractual basis
exists between the parties, as required in terms of the will theory. Consensus has three elements
(Hutchison and Pretorius (eds) The law of Contract in South Africa Oxford University Press Southern
Africa 2012 14 85): the parties must seriously intend to contract, be of one mind as to the material
aspects of the proposed agreement (the terms and the identity of the parties to it), and be conscious
of the fact that their minds have met.
In the present case the parties were not in agreement as to the consequences they wished to create:
Y thought that the dates for the art exhibition (X’s performance) was fixed, while X knew that the
contract allowed X to unilaterally change the dates. This is a mistake as to the obligations the parties
wished to create which excludes consensus between the parties (Hutchison and Pretorius Contract
86). No contract can arise on the basis of the will theory. This type of mistake can be illustrated with
a number of cases.
In Allen v Sixteen Stirling Investments (Pty) Ltd 1974 (4) SA 164 (D) the mistake related to
performance and was thus material. The plaintiff believed that he was purchasing the erf shown to
him by the seller's agent, while the written contract that he signed indicated the correct erf which
was a completely different property. His mistake related to performance and was material. See also
in Du Toit v Atkinson's Motors Bpk 1985 (2) SA 889 (A), the appellant signed an agreement containing
a term excluding the respondent from liability for misrepresentation. Finally see Sonap Petroleum
(SA) (Pty) Ltd (formerly known as Sonarep (SA) (Pty) Ltd) v Pappadogianis 1992 (2) SA 234 (A) where
the appellant erred with regard the period of the lease which was an aspect of the performance.
However, the matter does not end here, because Y still has to prove that his mistake is reasonable. A
mistake will generally be reasonable (Hutchison and Pretorius Contract 100-103) in three instances of
which only one is relevant, to this question.
Where the mistake was induced or caused by the failure of the contract enforcer to remove an
incorrect impression (ommissio). Here it will only be wrongful if the contract enforcer breached a
legal duty to speak in the circumstances. Such a duty will usually exist (Hutchison and Pretorius
Contract 101-102) where the contract assertor knows or ought to know as a reasonable person that
the other party is mistaken, or where the contract assertor, before the conclusion of the
contract, created an impression which is in direct conflict with the agreement he or she seeks to
enforce. Under these circumstances, the contract assertor must draw the contract denier’s attention
to this discrepancy. (See the Du Toit case; Hutchison and Pretorius Contract 101-102).
In this question, the only dates mentioned during negotiations (for the exhibition) were 24-27 July.
Since X had no reason to believe that Y would have signed the contract had Y known of the term
allowing X to change the dates of the exhibition unilaterally, he (X) had a legal duty to point out this
clause to Y. X’s failure to do so, renders Y’s material mistake reasonable.

The giving of appropriate advice


Y is not bound by the agreement with X because of the lack of actual and apparent consensus.
Total: [10]

Question 3
X contracts with Y for the latter (Y) to build and fit a security gate for the entrance of her (X’s) home.
Y builds the gate and fits it with an electric motor which is activated with a remote control. X is
satisfied with the work and pays Y the contractual amount agreed upon. A week later the gate gets
stuck while it is half way open as a result of defective materials used to build the gate. When X
attempts to physically move the gate to close it fully, she suffers such severe damage to her left knee
that she has to have a knee operation. Her medical costs are R20 000. The costs of repairing the gate
amount to R15 000. X wants to claim both her medical costs, as well the cost of repairing the gate
from Y. Advise X if she will be successful with her claim. (20)

Answer
Identifying the problem
This question deals with one of the remedies for breach of contract by Y, as she used defective
materials to build the gate. As a result of this breach, X now seeks to claim damages. The type of
breach that has transpired is positive malperformance on the part of Y, and the damages claimable
by X will be determined on the basis of whether such damages constitute general or special
damages.
Discussing the relevant law applicable to the problem, referring to the relevant case law, AND
applying the law to the facts of the problem
Is X entitled to compensated for both her medical costs (R20 000), as well the cost of repairing the
gate (R15 000) from Y.
In order to succeed with a claim for damages as a result of a breach of contract the innocent party
must prove the following:
(1) Breach of contract has occurred.
(2) The innocent party) has suffered financial or patrimonial loss
(3) There is a factual causal link between the breach and the loss
(4) The loss is not too remote a consequence of the breach (legal causation)

(1) Breach of contract has occurred


Positive malperformance takes place where a contracting party does not comply with the terms of
the contract either by performing something in a manner which does not comply with the terms of
the contract, or by doing something which he undertook not to do.
In this question, it is either a tacit term or a term implied by law that the gate will not be built with
defective materials. Therefore Y’s breach constitutes positive malperformance.
(2) The innocent party must suffer patrimonial loss
Breach of contract per se does not give rise to a claim for damages, unless patrimonial loss has
actually been incurred. The plaintiff must prove actual pecuniary or patrimonial loss. Patrimonial loss
is a loss suffered by the estate of the person concerned.
To ascertain whether damage has been suffered as a result of breach of contract, one compares the
present value of the creditor's estate with the value it would have had, had the breach of contract
not occurred (the difference rule). In other words in the case of breach of contract one compares the
present value of the innocent party's estate with the value it would have had, had the contract been
carried out properly and on time. If the present value is less than it would have been, damage has
been suffered. The debtor must place the creditor in the same patrimonial position as he would have
been in had proper and timeous performance taken place. This is the measure or formula applied for
damages and is referred to as positive interest.
The innocent party has to receive his positive interest - in contrast with negative interest; that is the
compensation payable if the injured party would have to be placed in the position in which he would
have been, had the contract never been entered into.
In this problem it is clear that damages have been suffered by X, in the form of R20 000 (for the
medical costs) and R15 000 (for the cost of repairing the gate).
(3) Causation
Factual causation
There must be a causal connection between the breach of contract and damage. The damage must
be caused by the breach of contract. The question is whether the damage would have been incurred
if the guilty party had properly fulfilled her part of the contract. A certain result is caused by a certain
act if that result would not normally have ensued but for such act (conditio sine qua-non-test).
The innocent party needs to prove, on a balance of probabilities, that the loss would not have been
suffered but for the breach. If she fails to establish this causal link, that is the end of the enquiry and
the damages claim must fail.
Factual causality is present in this problem. If Y did not breach the contract, the gate would not have
needed repair. Furthermore, if Y did not breach the contract the gate would have not have got stuck,
X would not have tried to open the gate and X would not have been injured.
Legal causality
The question which arises after factual causation has been established is whether the innocent party
may hold the other party liable for all the consequences of the breach. In the interests of fairness to
the party that commits a breach of contract, a line must be drawn between damages caused by her
breach and for which he is to be held liable, and damages which, although caused by the breach, are
so remote from it that he should not be held liable for them. It is often very difficult to make this
distinction.
In regard to this issue, it is important to understand the distinction made by our courts between
general and special damages (Holmdene Brickworks (Pty) Ltd v Roberts Construction Co Ltd 1977 (3)
SA 670 (A)).
(a) General damages
General damages are those which flow naturally and generally from the specific kind of breach that
has been committed. They are the sort of damages that might be expected in the ordinary course of
things to result from the breach. As such, they would have been foreseeable to a reasonable person
entering into the contract as a probable consequence. The party that commits a breach of contract is
held liable, without further ado, for general damages.
An example of general damages is the cost of repairing or replacing defective goods. The repair costs
(R15 000) of the gate are thus general damages. X is likely to be successful in this claim.
It is not clear whether the medical costs are general damages or not. On the one hand it could be
argued that a reasonable person entering into the contract would foresee that the use of defective
materials could result in X sustaining injuries as a probable consequence, but the contrary could
more convincingly be argued. The question then arises whether the medical costs could be claimed
as special damages.
(b) Special damages
All damages that cannot be classified as general damages are special damages. Special damages are
those which do not flow naturally and generally from the specific kind of breach of contract. The
party that commits a breach of contract will be liable for special damages only in certain
circumstances. The innocent party must prove:
(i) the damages were actually foreseen or reasonable foreseeable at the time of entry into the
contract (the contemplation principle); and
(ii) the parties must have entered into the contract on the basis of their knowledge of the special
circumstances, and thus can be taken to have agreed, expressly or tacitly, that there would be
liability for damages arising from such special circumstances (the convention principle).
It is unlikely that X will succeed with her claim for special damages. X and Y did not actually foresee
that as a result of such a breach, X would physically injure herself (the contemplation principle). The
parties clearly did not agree explicitly or presumably that such damages would be paid (in line with
the convention principle). X’s claim for medical costs would thus be unsuccessful.

The giving of appropriate advice


X will succeed with her claim for the repair costs, but most probably not for her medical costs.
TOTAL (20)

PVL3702 – 201 – 2019


Question

Read the judgments in Bloom v American Swiss Watch Co 1915 AD 100; Laws v Rutherford 1924 AD
261 and The Fern Gold Mining Company v Tobias (1889-1890) 3 SAR TS 134. Identify and read the
relevant section(s) in the prescribed textbook. You can find judgments in conventional law libraries,
online at the website of the Southern African Legal Information Institute (SAFLII) (www.saflii.org) or
as an e-resource on the Unisa Library site (choose Juta Law Online Publications and then South
African Law Reports). Then use those cases and the textbook to answer the following question:

In a cash-in-transit heist, a gang of robbers attacks a van of XYZ Security in broad daylight on 10
January and robs R3.5 million in cash. The next day, XYZ Security, a well-known security company
offers a reward of R100 000 on the TV news to anyone who gives information to the police leading to
the arrest of the robbers. H watches the news and notices that a reward is being offered. On 14
January, XYZ Security withdraws the reward, which is reported on the TV news. H does not watch the
news and fails to hear about the revocation of the reward. On 15 January, H provides the police with
information, which indeed leads to the arrest of the robbers that very night. On 17 January, XYZ
Security hears that the information has been provided to the police. XYZ refuses to pay H the reward.
Advise H fully. Substantiate your advice and refer to relevant case law.

Answer

In order to identify the law, which you should apply to the problem, you were asked to:

(1) read Bloom v American Swiss Watch Co 1915 AD 100, Laws v Rutherford 1924 AD 261 and The
Fern Gold Mining Company v Tobias (1889-1890) 3 SAR TS 134; and

(2) identify the relevant section(s) in the prescribed textbook.

(1) Read three cases

In Bloom v American Swiss Watch Co 1915 AD 100 the American Swiss Watch Co offered a reward
after a robbery at their shop for the giving of information to the police which lead to the arrest of the
thieves and the recovery of the stolen goods. Bloom did so without knowing of the reward. He
claimed the reward, but his claim was refused. On appeal, the court held that the reward was an
offer open to the public. It could be accepted by providing information to the police. Bloom, however
had no intention of accepting the offer because he did not know of its existence when he gave the
information to the police.

In Laws v Rutherford 1924 AD 261 R gave L an option to accept an offer within 3 months by written
notice. L failed to notify R of his acceptance in the prescribed manner. The court found that no
contract arose.

In Fern Gold Mining Company v Tobias (1889-1890) 3 SAR TS 134 it was held that an offer can be
revoked at any time before acceptance and that acceptance must come to the notice of the offeree.

(2) Identify the relevant section in the textbook

If the “Table of Cases” is consulted at the end of Hutchison and Pretorius (eds) The law of Contract in
South Africa 3rd ed (Oxford Cape Town 2017) you will find the pages where Bloom, Laws and Tobias
are discussed in the textbook.

The first page on which Bloom appears in the text, is 54. The paragraph deals with rewards. It is
mentioned that an advertised reward is an offer to the public. The offer is accepted by performing
the required act and the reward must be certain. A discussion of the requirements of a valid offer
and acceptance seems appropriate. The second page on which it is discussed in the text is 58 which
deals with the requirement that the acceptance must be a conscious response to the offer. This is
again an indication that the requirements for a valid acceptance is relevant.

Laws appears in footnote 50 on page 58 as authority for the requirement that the acceptance must
be in the form prescribed by the offer.
Tobias appears in footnote 44 on page 56 as authority for the requirement that the revocation of an
offer must come to the notice of the offeror. Revocation of the offer is thus also relevant.

The relevant sections of the textbook are page 50– 54, 56 and 57-58 where the requirements of a
valid offer, the revocation of an offer and the requirements for a valid acceptance of an offer are
discussed.

Identifying the problem

First, we must determine whether the reward complies with the requirements for a valid offer. Then
if it does, we must determine whether the offer has validly been revoked. Only if it has not been
validly revoked, the question must be addressed whether the offer has been validly accepted.

Discussing the relevant law applicable to the problem AND applying the law to the facts of the
problem

The offer of reward of XYZ Security complies with the requirements for a valid offer:

1 The offer was firm. An offer must be made with the intention that its acceptance will result in a
binding contract (Hutchison and Pretorius (eds) The law of Contract in South Africa (Oxford Cape
Town 2017) 50). In Bloom v American Swiss Watch Co 1915 AD 100 the American Swiss Watch Co
offered a reward after a robbery at their shop for the giving of information to the police which would
lead to the arrest of the thieves and the recovery of the stolen goods. Bloom did so without knowing
of the reward. He claimed the reward, but his claim was refused. On appeal, the court held inter alia
that the reward was an offer open to the public. The offer of XYZ Security was on the news and was
not a tentative statement.

2 The offer was complete. An offer must include all material terms of the proposed agreement and
there cannot be additional matters that still have to be discussed before the agreement can take
effect (Hutchison and Pretorius Contract 50). The offer contained all the material terms: provide
information to the police leading to the arrest of the robbers in return for the reward of R100 000.
The offer will be accepted by providing the required information.

3 The offer was clear and certain. An offer is sufficiently clear and certain if the mere answer of ‘yes’
by the addressee brings a valid contract into existence (Hutchison and Pretorius Contract 50). If the
offer is unclear and cannot capture what the offeror has in mind, no acceptance of the offer can
create a binding contract. The offer by XYZ Security was so clear that H as a member of the public
could form a clear idea what XYZ Security had in mind: a reward of R100 000 was offered to any
member public for the giving of information to the police leading to arrest of the robbers. The giving
of the required information will also constitute acceptance of the offer. Furthermore, an offer such as
a promise of reward can be validly directed at undefined persons (the public).

The next question is whether XYZ Security validly revoked their offer (Hutchison and Pretorius
Contract 56). XYZ Security did not promise not to revoke their offer and they could revoke their offer
at any time. It is clear from Fern Gold Mining Company v Tobias (1889-1890) 3 SAR TS 134 that an
offer can be revoked at any time before acceptance, but acceptance must come to the notice of the
offeree. The revocation of the reward on 14 January did not come to H notice and therefore did not
take effect against H.
The question now arises whether H validly accepted the offer of reward. The requirements of a valid
acceptance have been met:

1 The acceptance must be unqualified. The acceptance is complete and unequivocal, where the
entire offer and nothing additional or less is accepted (Hutchison and Pretorius Contract 57). H
accepted the offer by providing the required information.

2 The acceptance must be by the person to whom the offer was made. Only the offeree can validly
accept the offer (Bird v Sumerville 1961 (3) SA 194 (A). The general rule is that an offer should be
addressed to a specific person, but an offer may also validly be directed to the public, such as an
offer of a reward (Bloom v American Swiss Watch Co 1915 AD 100). The offer was directed at H as a
member of the public and could be validly accepted by him.

3 The acceptance must be a conscious response to the offer. In Bloom v American Swiss Watch Co
1915 AD 100 the American Swiss Watch Co the court also held on appeal that that Bloom had no
intention of accepting the offer because he did not know of its existence when he gave the
information to the police. In our problem H, however, knew of the offer by XYZ Security and
consciously responded to the offer by providing the required information to the police.

4 The acceptance must be in the form prescribed by the offeror (if any). The offeror can prescribe any
method of acceptance he or she sees fit (Carlill v Carbolic Smoke Ball Co (1893) 1 QB 256; Bloom v
American Swiss Watch Co 1915 AD 100). In Laws v Rutherford 1924 AD 261 R gave L an option to
accept an offer within 3 months by written notice. L failed to notify R of his acceptance in the
prescribed manner. The court therefore found that no contract arose. By giving the police
information leading to the arrest of the robbers, H’s acceptance complied with the prescribed mode
of acceptance of XYZ Security. There was no need for the acceptance to come to the notice of XYZ
Security.

The giving of appropriate advice

A valid contract thus arose between H and XYZ Security on 15 January.

Total: [10]

Question 1
An absolving agreement is an agreement

1 only creating obligations.


2 whereby a right is transferred.
3 an example of which is cession.
4 extinguishing or discharging obligations.
5 that cannot entail the transfer of ownership of property.

Question 2
Which statement relating to the Bill of Rights in the Constitution of the Republic of South Africa,
1996, is CORRECT?
1 Vertical application relates to relationships between private persons, as in most contractual
situations.
2 Horizontal application relates to relationships between the state and the individual.
3 In Barkhuizen v Napier 2007 (5) SA 323 (CC), the majority in the Constitutional Court ruled that a
contractual term can be tested directly against a provision in the Bill of Rights.
4 In Barkhuizen v Napier 2007 (5) SA 323 (CC), the minority in the Constitutional Court preferred an
indirect application of the Constitution to the contractual dispute before them.
5 In Barkhuizen v Napier 2007 (5) SA 323 (CC), Ngcobo J reasoned that the proper approach to
constitutional challenges to contractual terms, is to determine whether the term challenged is
contrary to public policy; and what constitutes public policy must be discerned with reference to
the fundamental values embodied in the Constitution.

Question 3
The auction of a Porsche Carrera (a type of sports car) is advertised in the newspaper: The auction is
to take place on 1 May. On 1 May X, the auctioneer, announces before the start of the auction that
the auction will be subject to reserve and that the successful bidder must pay the price cash on
delivery. Y is the highest bidder. Which statement is INCORRECT?

1 X may validly call off the auction any time before the start of the auction on 1 May.
2 Y is not bound by the condition that the successful bidder must pay the price cash on delivery
because this condition has not been announced in the advertisement.
3 X may not change the conditions of the auction after the first bona fide bid has been made.
4 Y may retract his bid before the hammer falls, because Y makes an offer to buy the car by bidding.
5 X may refuse to accept Y’s offer if Y’s bid is lower than the reserve price.

Question 4
X makes an offer to sell her watch to Y for R2 500. X and Y agree on 15 May that X’s offer will be open
for acceptance until 31 May. On 20 May X informs Y in writing that she (X) intends to sell the watch
to Z on 30 May. On 21 May Y informs X that she (Y) accepts the offer, but X refuses to sell the watch
to her (Y). Which statement is CORRECT?

1 X and Y concluded a pre-emption contract on 15 May.


2 X and Y concluded a pre-emption contract and an option contract on 15 May.
3 X and Y concluded a pre-emption contract on 21 May.
4 X and Y concluded an option contract on 21 May.
5 X and Y concluded an option contract on 15 May.

Question 5
The notion that contracts are based on the concurring declaration of the parties, has given rise to
which theory?

1 The expedition theory.


2 The declaration theory.
3 The reliance theory.
4 The reception theory.
5 The will theory.

Question 6
In which instance has consensus been obtained improperly?

1 A common error.
2 Rectification.
3 An error in negotio.
4 A negligent misrepresentation.
5 A threat that is not related to an imminent or inevitable evil.

Question 7
S illegally sells uncut diamonds to P for R10 000. Both S and P know that the selling of uncut
diamonds is prohibited by statute. Although S delivers the diamonds to P, P fails to pay the purchase
price. Which statement is CORRECT?

1 S will be able to claim the purchase price from P.


2 The contract sale is void because of illegality.
3 The contract of sale is voidable because of illegality.
4 S shall be able to claim return of the diamonds from P with an enrichment action, because the
court as a rule will relax the par delictum rule.
5 S shall be able to claim return of the diamonds from P with an enrichment action because the par
delictum rule does not apply in the circumstances.

Question 8
X rents a house from Y for R10 000 per month until such time as his employer transfers him to Cape
Town. The lease is subject to a

1 suspensive time clause.


2 resolutive time clause.
3 suspensive condition.
4 resolutive condition.
5 modus.

Question 9
X donates R100 000 to the Salvation Army for the purpose of looking after homeless people. The
obligation to pay R100 000 is subject to a

1 modus.
2 supposition.
3 suspensive condition.
4 resolutive condition.
5 positive condition.

Question 10
A term implied ex lege into a contract

1 is imported into a contract by law and operates, unless the parties exclude it.
2 is imported into a contract by the parties and operates, unless the law operates to exclude it.
3 is also known as a naturalium (singular of naturalia) if it is implied by the common law.
4 is a tacit term.
5 option 1 and 3.

Question 1
X, an organiser of art exhibitions, contracted with Y for an exhibition to be held on 24 to 27 July.
These dates were the only dates mentioned during the negotiations. After having been pressurized
by X, Y hurriedly signed the standard form contract without reading it. The contract contained a
clause permitting X to change the dates of the exhibition unilaterally. Thereafter X changed the dates.
X had no reason to believe that Y would have signed the contract if he had known of the term. Y
averred that the contract was void. Will Y succeed in his attempt to have the contract set aside?
Substantiate your answer and refer to relevant case law. Apply the direct reliance approach of the
courts in answering this question. Do not apply the Consumer Protection Act to this question. (10)

Answer
Identifying the problem
The facts seemingly indicate that X and Y have not reached consensus based on the will theory. If so,
it is necessary to determine if Y may be held bound to a contract with X, based on the reliance
theory, or whether Y will escape liability. Only the direct approach to the reliance theory will be
considered.
Discussing the relevant law applicable to the problem, referring to the relevant case law, AND
applying the law to the facts of the problem
The direct reliance approach can only be applied after it has been determined that Y acted under a
material mistake. It must thus be determined whether agreement (consensus ad idem) as a
contractual basis exists between the parties, as required in terms of the will theory.
The first step is to determine whether agreement (consensus ad idem) as a contractual basis exists
between the parties, as required in terms of the will theory. Consensus has three elements
(Hutchison and Pretorius (eds) The law of Contract in South Africa (Oxford University Press Southern
Africa 2012) 14 85): the parties must seriously intend to contract, be of one mind as to the material
aspects of the proposed agreement (the terms and the identity of the parties to it), and be conscious
of the fact that their minds have met.
In the present case the parties were not in agreement as to the consequences they wished to create:
Y thought that the dates for the art exhibition (X’s performance) was fixed, while X knew that the
contract allowed X to unilaterally change the dates. This is a mistake as to the obligations the parties
wished to create which excludes consensus between the parties (Hutchison and Pretorius Contract
86). No contract can arise on the basis of the will theory. This type of mistake can be illustrated with
a number of cases.
In Allen v Sixteen Stirling Investments (Pty) Ltd 1974 (4) SA 164 (D) the plaintiff believed that he was
purchasing the erf shown to him by the seller's agent, while the written contract that he signed
indicated a completely different property. His mistake related to performance and was material. See
also Du Toit v Atkinson's Motors Bpk 1985 (2) SA 889 (A). The appellant signed an agreement
containing a term excluding the respondent from liability for misrepresentation. Finally see Sonap
Petroleum (SA) (Pty) Ltd (formerly known as Sonarep (SA) (Pty) Ltd) v Pappadogianis 1992 (2) SA 234
(A) where the appellant erred with regard the period of the lease which was an aspect of the
performance.
The direct reliance approach can now be applied to the facts of the problem (Hutchison and
Pretorius Contract 18-19 95-97 103-105). The court stated the test in Sonap Petroleum (SA) (Pty) Ltd
(formerly known as Sonarep (SA) (Pty) Ltd) v Pappadogianis 1992 (2) SA 234 (A) 239- 240 as follows:
In my view, therefore, the decisive question in a case like the present is this: did the party whose
actual intention did not conform to the common intention expressed, lead the other party, as a
reasonable man, to believe that his declared intention represented his actual intention? … To answer
this question, a three-fold enquiry is usually necessary, namely, firstly, was there a misrepresentation
as to one party’s intention; secondly, who made that representation; and thirdly, was the other party
misled thereby? … The last question postulates two possibilities: Was he actually misled and would a
reasonable man have been misled?
A discussion of Ridon v Van der Spuy and Partners (Wes-Kaap) Inc 2002 (2) SA 121 (K) and Steyn v LSA
Motors Ltd 1994 (1) SA 49 (A) will also be appropriate.
By signing the contract, Y, a party to the contract, misrepresented his intention to be bound by the
clause allowing X to unilaterally change the dates. X knew that the only dates mentioned during the
negotiations were 24 to 27 July, that Y hastily signed the contract and that the contract had a clause
allowing X to unilaterally change the dates. Although it could be argued that X was not actually
misled by Y’s misrepresentation, it is clear that a reasonable person would not have been misled in
any case. Indeed, X had no reason to believe that Y would have signed the contract had Y known of
the term allowing X to change the dates of the exhibition unilaterally. In fact X had a legal duty to
point out the presence of this clause in the agreement to Y. There was either no actual or at least
reasonable reliance on the part of X.
The giving of appropriate advice
Y is not bound by the agreement with X because of the lack of actual and apparent consensus.
Total: [10]

Question 2
X, an organiser of art exhibitions, contracted with Y for an exhibition to be held on 24 to 27 July.
These dates were the only dates mentioned during the negotiations. After having been pressurized
by X, Y hurriedly signed the standard form contract without reading it. The contract contained a
clause permitting X to change the dates of the exhibition unilaterally. Thereafter X changed the dates.
X had no reason to believe that Y would have signed the contract if he had known of the term. Y
averred that the contract was void. Will Y succeed in his attempt to have the contract set aside?
Substantiate your answer and refer to relevant case law. Apply the indirect reliance approach of the
courts in answering this question. Do not apply the Consumer Protection Act to this question. (10)

Answer
Identifying the problem
The facts seemingly indicate that X and Y have not reached consensus based on the will theory. If so,
it is necessary to determine if Y may be held bound to a contract with X, based on the reliance
theory, or whether Y will escape liability. Only the indirect approach to the reliance theory will be
considered.
Discussing the relevant law applicable to the problem, referring to the relevant case law, AND
applying the law to the facts of the problem
The indirect approach is the iustus error-approach. A party who acted under a mistake and wishes to
escape liability (Y in our case) must prove that his / her mistake is material and reasonable.
At the outset it must be determined whether agreement (consensus ad idem) as a contractual basis
exists between the parties, as required in terms of the will theory. Consensus has three elements
(Hutchison and Pretorius (eds) The law of Contract in South Africa Oxford University Press Southern
Africa 2012 14 85): the parties must seriously intend to contract, be of one mind as to the material
aspects of the proposed agreement (the terms and the identity of the parties to it), and be conscious
of the fact that their minds have met.
In the present case the parties were not in agreement as to the consequences they wished to create:
Y thought that the dates for the art exhibition (X’s performance) was fixed, while X knew that the
contract allowed X to unilaterally change the dates. This is a mistake as to the obligations the parties
wished to create which excludes consensus between the parties (Hutchison and Pretorius Contract
86). No contract can arise on the basis of the will theory. This type of mistake can be illustrated with
a number of cases.
In Allen v Sixteen Stirling Investments (Pty) Ltd 1974 (4) SA 164 (D) the mistake related to
performance and was thus material. The plaintiff believed that he was purchasing the erf shown to
him by the seller's agent, while the written contract that he signed indicated the correct erf which
was a completely different property. His mistake related to performance and was material. See also
in Du Toit v Atkinson's Motors Bpk 1985 (2) SA 889 (A), the appellant signed an agreement containing
a term excluding the respondent from liability for misrepresentation. Finally see Sonap Petroleum
(SA) (Pty) Ltd (formerly known as Sonarep (SA) (Pty) Ltd) v Pappadogianis 1992 (2) SA 234 (A) where
the appellant erred with regard the period of the lease which was an aspect of the performance.
However, the matter does not end here, because Y still has to prove that his mistake is reasonable. A
mistake will generally be reasonable (Hutchison and Pretorius Contract 100-103) in three instances of
which only one is relevant, to this question.
Where the mistake was induced or caused by the failure of the contract enforcer to remove an
incorrect impression (ommissio). Here it will only be wrongful if the contract enforcer breached a
legal duty to speak in the circumstances. Such a duty will usually exist (Hutchison and Pretorius
Contract 101-102) where the contract assertor knows or ought to know as a reasonable person that
the other party is mistaken, or where the contract assertor, before the conclusion of the
contract, created an impression which is in direct conflict with the agreement he or she seeks to
enforce. Under these circumstances, the contract assertor must draw the contract denier’s attention
to this discrepancy. (See the Du Toit case; Hutchison and Pretorius Contract 101-102).
In this question, the only dates mentioned during negotiations (for the exhibition) were 24-27 July.
Since X had no reason to believe that Y would have signed the contract had Y known of the term
allowing X to change the dates of the exhibition unilaterally, he (X) had a legal duty to point out this
clause to Y. X’s failure to do so, renders Y’s material mistake reasonable.
The giving of appropriate advice
Y is not bound by the agreement with X because of the lack of actual and apparent consensus.
Total: [10]

Question 3
X contracts with Y for the latter (Y) to build and fit a security gate for the entrance of her (X’s) home.
Y builds the gate and fits it with an electric motor which is activated with a remote control. X is
satisfied with the work and pays Y the contractual amount agreed upon. A week later the gate gets
stuck while it is half way open as a result of defective materials used to build the gate. When X
attempts to physically move the gate to close it fully, she suffers such severe damage to her left knee
that she has to have a knee operation. Her medical costs are R20 000. The costs of repairing the gate
amount to R15 000. X wants to claim both her medical costs, as well the cost of repairing the gate
from Y. Advise X if she will be successful with her claim. (20)

Answer
Identifying the problem
This question deals with one of the remedies for breach of contract by Y, as she used defective
materials to build the gate. As a result of this breach, X now seeks to claim damages. The type of
breach that has transpired is positive malperformance on the part of Y, and the damages claimable
by X will be determined on the basis of whether such damages constitute general or special
damages.
Discussing the relevant law applicable to the problem, referring to the relevant case law, AND
applying the law to the facts of the problem
Is X entitled to compensated for both her medical costs (R20 000), as well the cost of repairing the
gate (R15 000) from Y.
In order to succeed with a claim for damages as a result of a breach of contract the innocent party
must prove the following:
(1) Breach of contract has occurred.
(2) The innocent party) has suffered financial or patrimonial loss
(3) There is a factual causal link between the breach and the loss
(4) The loss is not too remote a consequence of the breach (legal causation)

(1) Breach of contract has occurred


Positive malperformance takes place where a contracting party does not comply with the terms of
the contract either by performing something in a manner which does not comply with the terms of
the contract, or by doing something which he undertook not to do.
In this question, it is either a tacit term or a term implied by law that the gate will not be built with
defective materials. Therefore Y’s breach constitutes positive malperformance.

(2) The innocent party must suffer patrimonial loss


Breach of contract per se does not give rise to a claim for damages, unless patrimonial loss has
actually been incurred. The plaintiff must prove actual pecuniary or patrimonial loss. Patrimonial loss
is a loss suffered by the estate of the person concerned.
To ascertain whether damage has been suffered as a result of breach of contract, one compares the
present value of the creditor's estate with the value it would have had, had the breach of contract
not occurred (the difference rule). In other words in the case of breach of contract one compares the
present value of the innocent party's estate with the value it would have had, had the contract been
carried out properly and on time. If the present value is less than it would have been, damage has
been suffered. The debtor must place the creditor in the same patrimonial position as he would have
been in had proper and timeous performance taken place. This is the measure or formula applied for
damages and is referred to as positive interest.
The innocent party has to receive his positive interest - in contrast with negative interest; that is the
compensation payable if the injured party would have to be placed in the position in which he would
have been, had the contract never been entered into.
In this problem it is clear that damages have been suffered by X, in the form of R20 000 (for the
medical costs) and R15 000 (for the cost of repairing the gate).

(3) Causation
Factual causation
There must be a causal connection between the breach of contract and damage. The damage must
be caused by the breach of contract. The question is whether the damage would have been incurred
if the guilty party had properly fulfilled her part of the contract. A certain result is caused by a certain
act if that result would not normally have ensued but for such act (conditio sine qua-non-test).
The innocent party needs to prove, on a balance of probabilities, that the loss would not have been
suffered but for the breach. If she fails to establish this causal link, that is the end of the enquiry and
the damages claim must fail.
Factual causality is present in this problem. If Y did not breach the contract, the gate would not have
needed repair. Furthermore, if Y did not breach the contract the gate would have not have got stuck,
X would not have tried to open the gate and X would not have been injured.

Legal causality
The question which arises after factual causation has been established is whether the innocent party
may hold the other party liable for all the consequences of the breach. In the interests of fairness to
the party that commits a breach of contract, a line must be drawn between damages caused by her
breach and for which he is to be held liable, and damages which, although caused by the breach, are
so remote from it that he should not be held liable for them. It is often very difficult to make this
distinction.
In regard to this issue, it is important to understand the distinction made by our courts between
general and special damages (Holmdene Brickworks (Pty) Ltd v Roberts Construction Co Ltd 1977 (3)
SA 670 (A)).
(a) General damages
General damages are those which flow naturally and generally from the specific kind of breach that
has been committed. They are the sort of damages that might be expected in the ordinary course of
things to result from the breach. As such, they would have been foreseeable to a reasonable person
entering into the contract as a probable consequence. The party that commits a breach of contract is
held liable, without further ado, for general damages.
An example of general damages is the cost of repairing or replacing defective goods. The repair costs
(R15 000) of the gate are thus general damages. X is likely to be successful in this claim.
It is not clear whether the medical costs are general damages or not. On the one hand it could be
argued that a reasonable person entering into the contract would foresee that the use of defective
materials could result in X sustaining injuries as a probable consequence, but the contrary could
more convincingly be argued. The question then arises whether the medical costs could be claimed
as special damages.
(b) Special damages
All damages that cannot be classified as general damages are special damages. Special damages are
those which do not flow naturally and generally from the specific kind of breach of contract. The
party that commits a breach of contract will be liable for special damages only in certain
circumstances. The innocent party must prove:
(i) the damages were actually foreseen or reasonable foreseeable at the time of entry into the
contract (the contemplation principle); and
(ii) the parties must have entered into the contract on the basis of their knowledge of the special
circumstances, and thus can be taken to have agreed, expressly or tacitly, that there would be
liability for damages arising from such special circumstances (the convention principle).
It is unlikely that X will succeed with her claim for special damages. X and Y did not actually foresee
that as a result of such a breach, X would physically injure herself (the contemplation principle). The
parties clearly did not agree explicitly or presumably that such damages would be paid (in line with
the convention principle). X’s claim for medical costs would thus be unsuccessful.
The giving of appropriate advice
X will succeed with her claim for the repair costs, but most probably not for her medical costs.
TOTAL (20)

PVL3702 – Questions May 2013

Ms REF Ugee is a citizen of a war-ravaged country in West Africa. She fled her country of birth,
moving gradually southwards and entered South Africa after a long journey. Ms REF Ugee applies for
asylum at the Department of Home Affairs in terms of section 21(1) of the Refugees Act 130 of 1998.
Pending the outcome her application for asylum, she was granted an asylum seeker permit in terms
of the Act, which allows her to stay temporarily in South Africa. Later, however, her application for
asylum is rejected by the authorized refugee status determination officer without any input by Ms
REF Ugee. No reasons were given, but it later transpired that the rejection was based on the informal
notes of the refugee reception officer working in the refugee reception officer responsible for issuing
her with the asylum seeker permit. From these notes the impression might be granted that the
decision had been influenced by certain irrelevant factors suggesting bias on the part of the
administrator. The Refugees Act makes provision for review by the Standing Committee and an
appeal to the Appeal Board of a decision by a refugee status determination officer.
You are a legal advisor working at a non-governmental organization (NGO) called Consortium for
Refugees and Migrants in South Africa (CRMSA). Ms Ugee approaches you for assistance on ways and
means on how the NGO might be able to come to her assistance.
Answer the following questions and substantiate your answers.

Question 1
1.1 Briefly explain what an administrative-law relationship is. Do you think Ms REF Ugee is a subject
of an administrative-law relationship? (6)

Answer
An administrative-law relationship is a relationship between two or more legal subjects in which one
is a person or body who is clothed with state authority and is able to exercise the authority over the
other. The exercise of power may affect the rights and/or interests of the person(s) in a subordinate
position. It is therefore an unequal relationship. In a general administrative-law relationship the legal
rules governing the relationship between the parties apply to all the subjects in a particular group. It
is created by, changed and terminated by legislation. In an individual administrative-law relationship
the rules apply personally and specifically between the parties. The relationship is created by
individual administrative decisions.
In the set of facts, the individual administrative-law relationship is applicable since an individual
relationship exists between Ms Ugee and the Department of Home Affairs.

1.2 Identify the administrative action in the in the set of facts. In your answer, you should give a full
definition of the concept “administrative action” with reference to the provisions of the Promotion of
Administrative Justice Act (PAJA) 3 of 2000. (13)

Answer
Section 1 of PAJA defines "administrative action” as any decision taken, or any failure to take a
decision, by –
(a) an organ of state, when-
(i) exercising a power in terms of the Constitution or a provincial constitution; or
(ii) exercising a public power or performing a public function in terms of any legislation; or
(b) a natural or juristic person, other than an organ of state, when exercising a public power or
performing a public function in terms of an empowering provision, which adversely affects the rights
of any person and which has a direct, external legal effect.
There are exceptions to the definition. These exceptions are, however, not applicable to the given set
of facts. The decision to reject Ms REF Ugee’s application for asylum amounts to administrative
action because it complies with the definition in that it involves a decision by an organ of state (the
Department of Home Affairs, authorized refugee status determination officer in the Department of
Home Affairs) exercising a public power or performing a public function in terms of any legislation
which has adversely affected the rights of a person (Ms REF Ugee’s) and which appears to have had a
direct external legal effect.

1.3 Identify the organs of state in the given set of facts. Explain your answers with reference to the
constitutional definition of organ of state. (6)

Answer
In terms of section 239 of the Constitution, the following are organs of state:
The Department of Home Affairs (any department of state or administration in the national,
provincial or local sphere of government)
The authorized refugee status determination official who made the decision, the refugee reception
officer, the Standing Committee and the Appeal Board in the Department of Home Affairs (any other
functionary or institution (ii) exercising a public power or performing a public function in terms of
any legislation)
PVL3702_201_2009
Question
X is on her way from work and sees a white bull terrier bitch hiding in a doorway. Being an animal
lover, she takes the dog home with her. The next day, she sees the following advertisement in the
newspaper: Lost in Johannesburg City Centre on 27 May. Pedigree white bull terrier bitch with black
patch over left eye. Answers to the name of Beauty. Reward of R1 000 for information leading to safe
return. Tel 011 555 5555.
She realises that the dog she found matches the description given. She calls the advertiser who
rushes over to be joyfully united with Beauty. In his joy, Beauty’s owner, Y, seems to forget about the
reward and X wishes to claim it from him. Will she be successful?
Substantiate your answer. (10)

Answer
X will only be successful in her claim if a valid contract arose between X and Y and this will be the
case if there was a valid acceptance of a valid offer. Y’s advertisement complies with the
requirements for a valid offer:
1 Y’s offer was definite and complete. The offer in this problem contained adequate information to
enable the addressee to form a clear idea of exactly what the offeror had in mind as it stated what
was required for the offer to be accepted (information leading to the safe return of Beauty) and what
amount the reward (R1 000) was.
2 Y’s offer contemplated acceptance and a resultant obligation. Although an advertisement is usually
only an invitation to do business (Crawley v Rex 1909 TS 1105), a promise of reward does constitute a
firm offer (Bloom v American Swiss Watch Co 1915 AD 100). Y’s offer was a firm offer and not a
tentative statement with a possible agreement in mind.
3 Y’s offer came to the attention of the addressee, X. The fact that an agreement is a conscious or
stated mutuality of consent (McKenzie v Farmers Cooperative Meat Industries Ltd 1922 AD 16) leads
to the requirement that the offeree must have knowledge of the offer to be able to react to it (Bloom
v The American Swiss Watch Co 1915 AD 100).
4 An offer such as a promise of reward can be validly directed at undefined persons.
The providing of information by X was a valid acceptance of Y’s offer:
1 X’s acceptance was unconditional and unequivocal.
2 X, as a member of the public, could accept the offer of reward because the offer was addressed to
the public in general.
3 X was aware of the offer of reward and her acceptance was thus a reaction to the offer (Bloom v
American Swiss Watch Co 1915 AD 100). X’s giving of information was a manifestation of her
intention to accept the offer.
4 X’s acceptance complies with the requirement set by Y that acceptance must take the form of the
provision of certain information (Carlill v Carbolic Smoke Ball Co (1893) 1 QB 256).
It can be concluded that a valid contract arose in this problem, because Y made a valid offer which
X validly accepted.

Question 1
X, who lives in Cape Town, writes a letter to Y, who lives in Johannesburg. In the letter, she offers to
sell her house to Y for R900 000. She posts the letter in Stellenbosch while visiting her daughter.
A week later Y receives the letter, reads it, and immediately phones X on her landline. Y requests X to
keep her offer open for a week and she undertakes to do. A day before the option expires, X phones
Y and revokes her offer to sell her house. Y immediately posts a letter of acceptance in which he
exercises the option. X only receives this letter a week later. Which statement is
CORRECT?

1 The substantive offer to sell the house does not contain all the essentialia of a contract of sale.
2 The option contract is valid.
3 The option contract was exercised too late as Y’s letter of acceptance only reached X after the
option lapsed.
4 According to the courts, X can validly revoke her offer to sell her house before Y exercises the
option.
5 The option contract is concluded in Cape Town.

Question 2
To the facts of which case can the iustus error approach not be applied?

1 Steyn v LSA Motors Ltd 1994 1 SA 49 (A).


2 Sonap Petroleum (SA) (Pty) Ltd v Pappadogianis 1992 3 SA 234 (A).
3 Du Toit v Atkinson’s Motors Bpk 1985 2 SA 893 (A).
4 George v Fairmead (Pty) Ltd 1958 2 SA 465 (A).
5 National and Overseas Distributors Corporation (Pty) Ltd v Potato Board 1958 2 SA 473 (A).

Question 3
Which case will you consult as authority on the exclusion of a party’s liability for misrepresentations
in a contractual clause?

1 Bayer South Africa (Pty) Ltd v Frost 1991 4 SA 449 (A).


2 Ranger v Wykerd 1977 2 SA 976 (A).
3 Trotman v Edwick 1951 1 SA 443 (A).
4 De Jager v Grunder 1964 1 SA 446 (A).
5 Wells v SA Alumenite Co 1927 AD 69.

Question 4
S illegally sells uncut diamonds to P for R10 000.00. Both S and P know that the selling of uncut
diamonds is prohibited by statute. Although S delivers the diamonds to P, P fails to pay the purchase
price. Which statement is CORRECT?

1 S will be able to claim the purchase price from P.


2 The contract sale is void on the basis of illegality.
3 The contract of sale is voidable on the basis of illegality.
4 S shall be able to claim return of the diamonds from P with an enrichment action, because the
court as a rule will relax the par delictum rule.
5 S shall be able to claim return of the diamonds from P with an enrichment action because par
delictum rule does not apply in the circumstances.
Question 5
X buys a stand from Y for R300 000. Both X and Y honestly believed that business rights existed on
the stand. They were also aware of each other’s belief. Later it emerges that no such rights exist.
Which statement is INCORRECT?

1 The contract is void if it does not comply with the Alienation of Land Act 68 of 1981.
2 Both X and Y act under a mistake regarding an attribute or characteristic of the subject matter of
the contract.
3 Dickensen Motors (Pty) Ltd v Oberholzer 1952 1 SA 443 (A) is a case which is applicable to this
problem.
4 Fourie v CDMO Homes (Pty) Ltd 1982 1 SA 21 (A) is a case which is applicable to this problem.
5 The contract is voidable because both X and Y has been unduly influenced by their belief in the
existence of business rights.

Question 6
X purchased Y’s shareholding in a particular company. The principle asset of the company was a
shopping centre which was rent producing. Y was aware of the fact that X was interested in the
shares because of the income the shopping centre generated. Y presented to X that the municipal
rates were much lower than what they actually were. X would not have bought the shares if he had
known the truth regarding the municipal rates. The contract of sale is voidable, because

(a) Y made an innocent misrepresentation.


(b) Y made a dictum et promissum.
(c) Y made a guilty misrepresentation.
(d) Y caused X to act under a mistake over a characteristic of the thing purchased.

1 All the possibilities.


2 (a), (b) and (c).
3 (b), (c) and (d).
4 (b) and (c).
5 (a) and (b).

Question 7
The facts of question 6 are based on the facts of:

1 Phame (Pty) Ltd v Paizes 1973 3 SA 397 (A).


2 Ranger v Wykerd 1977 2 SA 976 (A).
3 Trotman v Edwick 1951 1 SA 443 (A).
4 De Jager v Grunder 1964 1 SA 446 (A).
5 Bayer South Africa (Pty) Ltd v Frost 1991 4 SA 559 (A).

Question 8
P buys an erf from S for R300 000. During the negotiations S points a different erf out to P than the
one described in the written contract of sale. What type of mistake does P make?
1 Error in negotio.
2 Error in substantia.
3 Error in corpore.
4 Error in persona.
5 Error in motive

Question 9
Which statement is INCORRECT?

1 In Jurgens Eiendomsagente v Share 1990 4 SA 664 (A) the court held that the obligation to furnish
a guarantee of the payment of the purchase price was a suspensive time clause.
2 In Saambou-Nasionale Bouvereniging v Friedman 1979 3 SA 978 (A) the court had to decide
whether a cheque had a justa causa.
3 In Vasco Dry Cleaners v Twycross 1979 1 SA 603 (A) the court found that the contract of sale was
simulated.
4 In Broodryk v Smuts 1942 TPD 47 civil servants threatened the plaintiff with internment if he
refused to join the army.
5 The court held that the exceptio doli generalis was never part of the Roman-Dutch law in Bank of
Lisbon and South Africa Ltd v De Ornelas 1988 3 SA 580 (A).

Question 10
In which case was the hypothetical bystander test for tacit terms applied?

1 Fourie v CDMO Homes (Pty) Ltd 1982 (1) SA 21 (A).


2 Van den Berg v Tenner 1975 (2) SA 268 (A).
3 Jurgens Eiendomsagente v Share 1990 (4) SA 664 (A).
4 Minister van Landbou-Tegniese Dienste v Scholtz 1971 (3) SA 188 (A).
5 Magna Alloys and Research (SA) (Pty) Ltd v Ellis 1984 (4) SA 874 (A).

PVL3702_201_2010
Question
S, who lives in Upington, sends P, who lives in Grahamstown, a letter by post in which she offers to
sell him her (S’s) motorcycle, a collectors piece, for R100 000. She states in her letter that her offer
will expire on 1 February at 24h00. P phones S on 1 February at 19h00, but S and her husband is out
for the evening. P leaves a message on S’s voice mailbox accepting S’s offer. S only listens to P’s
message on 2 February. P tenders payment of R100 000 but S refuses to accept payment. Did a valid
contract of sale arise between S and P? Advise P. Substantiate your answer. (10)

Answer
The question is whether P has accepted S’s offer in time and thus whether S and P have reached
consensus. S’s offer lapses after the time that she has prescribed for acceptance (Van Aswegen et al
Law of Contract: only study guide for PVL301-W (1996) 23). The problem is that although P accepted
the offer in time, S was only informed of the acceptance after the time set for the lapse of the offer.
The general rule is that acceptance must be communicated to the offeror for consensus to arise (R v
Nel 1921 AD 339 344; Cape Explosives Works Ltd v South African Oil and Fat Industries Ltd;
Cape Explosives Works Ltd v Lever Brothers (South Africa) Ltd 1921 CPD 244 276; Smeiman v
Volkersz 1954 4 SA 170 (C) 179). This rule flows from the principle that actual and conscious
agreement between the parties form the primary basis for contractual liability (Van Aswegen et al
Law of Contract: only study guide for PVL301-W (1996) 15 and 29). The parties must be aware of
their unanimity (Van Aswegen et al Law of Contract: only study guide for PVL301-W (1996) 15).
This not a case of a contract concluded by telephone (S v Henckert 1981 (3) SA 445 (A)) as only the
acceptance by P was by telephone. The Electronic Communications and Transaction Act 25 of
2002, which applies the reception theory to contracts concluded with data messages, does not apply
furthermore as the definition of “data message” does not include purely voice messages such as
telephone conversations even where the message is stored as a voice message (Tutorial Letter
101/3/2010 32).
The offeror may, however, expressly or tacitly waive his right to notification of acceptance (R v Nel
1921 AD 339 344; Cape Explosives Works Ltd v South African Oil and Fat Industries Ltd; Cape
Explosives Works Ltd v Lever Brothers (South Africa) Ltd 1921 CPD 244 256 276; Smeiman v
Volkersz 1954 4 SA 170 (C) 176 179). S did not expressly do so as the offer contains no words to that
effect, but S tacitly did so by making her offer by post. She had tacitly indicated that her offer was
accepted as soon as P posted his letter of acceptance (Cape Explosives Works Ltd v South
African Oil and Fat Industries Ltd; Cape Explosives Works Ltd v Lever Brothers (South Africa) Ltd
1921 CPD 244 256 276; Smeiman v Volkersz 1954 4 SA 170 (C) 176 179; Van Aswegen et al Law
of Contract: only study guide for PVL301-W (1996) 30-31)). P is not obliged to accept by post and he
has not in fact done so (he phoned S and left a message on her voice mailbox). In such a case,
S’s tacit waiver does not apply but the general rule applies. We can conclude that S and P did not
reach agreement before S’s offer lapsed.

Question 1
In which case was it decided that advertisements are usually mere invitations to the public to do
business?

1 Crawley v Rex 1909 TS 1105.


2 Allen v Sixteen Stirling Investments (Pty) Ltd 1974 (4) SA 164 (D).
3 R v Nel 1921 AD 339.
4 Steyn v LSA Motors Ltd 1994 (1) SA 49 (A).
5 None of the above.

Question 2
In Bird v Sumerville 1961 (3) SA 194 (A) the court found that

1 a person cannot accept an offer of which he is not aware.


2 an offer must be accepted by the person to whom it was addressed.
3 an acceptance must be a reaction to the offer.
4 an acceptance must comply with any formalities set by law or by the offeror.
5 an acceptance must be unconditional and unequivocal.

Question 3
Y offers to sell his BMW motor car to Z on 1 July, for R50 000. One of the terms of Y’s offer is that the
offer lapses on 30 August. However, on 20 July Y notifies Z that the offer is cancelled. Z insists that
the offer is valid until 30 August and on 25 July Z notifies Y that he (Z) accepts the offer. Which
answer reflects the correct legal position?

1 A pre-emption contract was not concluded between Y and Z.


2 An option contract was not concluded between Y and Z.
3 An option contract was concluded between Y and Z.
4 1 and 2.
5 None of the above.

Question 4
John owns a business. During the period 1 February to 31 March he unknowingly operates his
business in contravention of a statute which requires him to have a trading license. This statute only
criminalises the operation of a business without a trade license. On 15 February John sells goods on
credit to Steve for R2000. John delivers the goods to Steve. When the price is due and payable on 20
February, Steve refuses to pay John. Steve argues that he is not obliged to pay John, as
John did not have a trading license on 15 February. Which answer is CORRECT?

1 John may always claim back the goods from Steve, or if it no longer exists, the value thereof, based
on an unjustified enrichment claim.
2 The contract is illegal and therefore unenforceable.
3 Under certain circumstances John may claim back the goods from Steve, or if it no longer exists,
the value thereof, based on an unjustified enrichment claim.
4 John may enforce the contract and claim R2000 from Steve.
5 2 and 3.

Question 5
Gary, a breeder of stud bulls, sells a bull to Piet for an agreed price. Gary knew that Piet required the
bull for breeding purposes, although this fact is not mentioned in the contract. Subsequently it turns
out that the bull is infertile and Piet wishes to cancel the contract. Which cause of action will
Piet be able to rely on?

1 Breach of a term implied by law that the stud bull is fertile.


2 Breach of a tacit term that the stud bull is fertile.
3 Breach of a tacit term that the stud bull will not have any latent defect.
4 Breach of an express term agreed upon by the conduct of the parties that the stud bull is fertile.
5 None of the above.
Question 6
Which case is applicable to the facts in question (5)?

1 Trotman v Edwick 1951 (1) SA 443 (A).


2 Van den Berg v Tenner 1975 (2) SA 268 (A).
3 Sweet v Ragerguhara NO 1978 (1) SA 131 (D).
4 Goldstein and Wolff v Maison Blanc (Pty) Ltd 1948 (4) SA 446 (C).
5 Minister van Landbou-Tegniese Dienste v Scholtz 1971 (3) SA 188 (A).

Question 7
The case in question (5) provides an excellent example of the distinction between

1 naturalia and tacit incidentalia of a contract.


2 accidentalia and incidentalia of a contract.
3 incidentalia and an essentialia of a contract.
4 essentialia and naturalia of a contract.
5 essentialia and accidentalia of a contract.

Question 8
Which case would you refer to as authority on the effect of a contractual clause excluding liability for
misrepresentation?

1 Goldblatt v Fremantle 1920 AD 123.


2 Neethling v Klopper 1967 (4) SA 459 (A).
3 Broodryk v Smuts 1942 TPD 47.
4 Wells v SA Alumenite Co 1927 AD 69.
5 Bank of Lisbon and South Africa Ltd v De Ornelas 1988 (3) SA 580 (A).

Question 9
While doing window-shopping, X and his fiancée, Y, see a ring displayed in a shop window. They
immediately enter the jeweller’s shop and offer to buy the ring at the displayed price. They find out
to their dismay that the “diamond” is a synthetic diamond afterwards. Which statement is
CORRECT:
1 The contract of sale is void.
2 The error is an error in substantia.
3 The error is an error in corpore.
4 The jeweller made a dictum et promissum that the ring was a diamond ring.
5 The parties to the contract did not want to buy and sell the same ring.

Question 10
“Our law allows a party to set up his own mistake in certain circumstances in order to escape liability
under a contract into which he has entered. But where the other party has not made any
misrepresentation and has not appreciated at the time of acceptance that his offer was being
accepted under a misapprehension, the scope for a defence of unilateral mistake is very narrow, if it
exists at all.” In which case does this statement appear?
1 Du Toit v Atkinson Motors Bpk 1985 2 SA 889 (A).
2 George v Fairmead (Pty) Ltd 1958 2 SA 465 (A).
3 Allen v Sixteen Stirling Investments (Pty) Ltd 1974 4 SA 164 (D).
4 National and Overseas Distributors Corporation (Pty) Ltd v Potato Board 1958 2 SA 473 (A).
5 Steyn LSA Motors Ltd 1994 1 SA 49 (A).

PVL3702_May 2013

(1) Which of the following IS NOT a requirement for a valid contract?

1 Consensus
2 Formalities
3 Possibility
4 Reciprocity
5 Certainty

(2) Which statement IS INCORRECT?

1 A contract is a unilateral or even bilateral Juristic act


2 A contract enta1ls prom1ses or undertak1ngs on one or both sides
3 An undertaking 1n a contract that a certain state of affairs exists, or has existed, IS known as a
warranty
4 The conclusion of a contract can be multilateral
5 Freedom of contract means that the parties can agree to anything that is possible and lawful

(3) Which statement is INCORRECT?

1 An obligation is a legal bond between a debtor and a creditor


2 With a contract of sale, the seller is the debtor in respect of the duty to deliver the thing sold, and
the seller is the creditor in respect to the obligation relating to the duty to pay the price
3 An obligation comprises a right and a corresponding duty the right of the cred1tor to demand a
performance by the debtor, and the duty of the debtor to make that performance
4 A natural obligation is unenforceable m a court of law
5 The right created by an obligation is a real right

(4) Which statement is INCORRECT?

1 A delict is wrongful and blameworthy conduct that causes harm to a person, and which obliges the
wrongdoer to compensate the injured party
2 Concurrent liability exists where the same conduct might constitute both a delict and a breach of
contract, as when a surgeon who has contracted to perform an operation negligently leaves a cotton
swab inside the patient’s body
3 The essential difference between contractual and delictual obligations is that the latter are, as a
general rule, voluntarily assumed by the parties themselves, whereas the former are imposed by
law, irrespective of the will of the parties
4 The courts have shown great reluctance to permit claims in delict for economic losses caused by a
breach of contract
5 Unjustified enrichment occurs when there is a shift of wealth from one person's estate to another's
without a good legal ground or cause for this shift

(5) Which statement relating to the Bill of Rights m the Constitution of the Republic of
South Africa, 1 996, 1s CORRECT?

1 Vertical application relates to relationships between private persons, as in most contractual


situations
2 Horizontal application relates to relationships between the state and the individual
3 In Barkhwzen v Nap1er 2007 (5) SA 323 (CC), the majority in the Constitutional Court ruled that a
contractual term can be tested directly against a provision in the Bill of Rights
4 In Barkhwzen v Nap1er 2007 (5) SA 323 (CC), the minority in the Constitutional Court preferred an
indirect application of the Constitution to the contractual dispute before them
5 In Barkhwzen v Nap1er 2007 (5) SA 323 (CC), Ngcobo J reasoned that the proper approach to
constitutional challenges to contractual terms, is to determine whether the term challenged is
contrary to public pol1cy, and what constitutes public policy must be discerned w1th reference to
the fundamental values embodied m the Constitution

(6) Cedric has joined a religious sect. Recently this sect took advantage of Cedric's good nature and
convinced him to donate his motor vehicle to the sect, which Cedric did Cedric approaches you for
legal advice, explaining that he wants his motor vehicle returned to him. The issue emanating from
these facts relates to

1 duress
2 undue Influence
3 commercial bribery
4 puffs
5 dicta et promissa

(7) Assume the same facts as in question (6) A court presiding over this case is likely to find that the
religious sect

1 threatened Cedric and more particularly his property


2 instilled reasonable fear in Cedric
3 coerced Cedric to donate his motor vehicle to the sect
4 obtained an influence over Cedric, and that this Influence weakened his powers of resistance and
rendered his will compliant
5 conducted itself in such a distasteful manner that it amounts to supervening impossibility of
performance
(8) Assume the same facts as in question (6) An order sought for the return of the motor vehicle by
Cedric amounts to an order for

1 restitution
2 specific performance
3 compensation for general damages
4 compensation for special damages
5 rectification

(9) Which of the following is NOT an element for commercial bribery?

1 A reward
2 The one party is the briber
3 The principal is able to exert Influence over a third party
4 The agent may be an agent in the true sense or merely a go-between or facilitator
5 A direct or indirect benefit IS sought for the briber

(10) If commercial bribery exists, the agreement between the briber and the agent is

1 voidable
2 Void
3 void and voidable
4 neither void nor voidable
5 for the benefit of a third party

(11) If commercial bribery exists, the agreement between the briber and the principal is

1 voidable
2 Void
3 vo1d and vo1dable
4 neither void nor voidable
5 for the benefit of a third party

(12) Kate makes an offer to sell her watch to Martha for R2 500. Kate and Martha agree on 15 May
that Kate's offer will be open for acceptance until 31 May. On 20 May Kate informs Martha in writing
that she (Kate) intends to sell the watch to Julia on 30 May. On 21 May Martha informs Kate that she
accepts the offer, but Kate refuses to sell the watch to her. Which statement is CORRECT?

1 Kate and Martha concluded a pre-emption contract on 15 May


2 Kate and Martha concluded a pre-emption contract and an option contract on 15 May
3 Kate and Martha concluded a pre-emption contract on 21 May
4 Kate and Martha concluded an option contract on 21 May
5 Kate and Martha concluded an option contract on 15 May
(13) Assume the same facts as in question (12) Martha wants to compel Kate to sell the watch to her.
How would you best advise Martha?

1 To cancel the contract and seek to interdict Kate from selling the watch to Julia
2 To enforce the contract and seek to interdict Kate from selling the watch to Julia
3 To cancel the contract and seek an order for specific performance against Kate
4 To enforce the contract and cancel the contract simultaneously through the use of an interdict
5 To cancel the pre-emption agreement between Kate and Julia

(14) Tony, a petrol attendant, sells dagga to Samuel for R1 000 Tony delivers the dagga to Samuel, but
Samuel refuses to pay Sect1on 5 of the Drugs and Drug Traff1ck1ng Act 140 of 1992 provides that no
person shall deal in (a) any dependence-producing substance or (b) any dangerous dependence-
producing substance or any undesirable dependence-producing substance, while section 4 prohibits
the possession of such substances Section 13 makes the contravention of both sections 4 and 5 a
crime Dagga is furthermore an undesirable dependence-producing substance Tony approaches a
court to assist him, either for the payment of R1 000 from Samuel or the return of the drugs. The
material focus of this dispute rests on which requirement relating to the validity of the agreement
between Tony and Samuel?

1 Consensus
2 Formalities
3 Possibility
4 Legality
5 Certainty

(15) Assume the same facts as in question (1 4). Most relevant to the dispute would be to consider
the application of

1 the ex turpt rule and the par delictum rule


2 statutory Illegality and restraint of trade agreements
3 public interest and rectification
4 the par delictum rule and capacity of the parties to contract
5 the ex turpt rule and stipulafio alten

(16) Assume the same facts as in question (14) Which of the following factors must be considered in
order to determine whether the legislator Impliedly Intended for the contract between Tony and
Samuel to be void?

(a) What is the object of the statute and what mischief (harm) is the statute directed against?
(b) Does the enactment 1mpose a cnm1nal sanct1on, or does the enactment merely serve to protect
the revenue of the State?
(c) Does the provision in the Act merely protect individuals or does it Involve a public interest that
requires protection by voiding the contract?
(d) What are the consequences of a particular interpretat1on of the contract?
1 Only (a)
2 Only (b)
3 (a), (b) and (c)
4 (a), (b) and (d)
5 (a), (b), (c) and (d)

(17) Assume the same facts as in question (14) The court hearing th1s dispute IS likely to find that

1 Tony can cla1m R1 000 contractual damages from Samuel


2 Tony will be granted an order for specific performance for the return of the dagga
3 Tony's request for the return of the dagga will fall because both parties were equally morally
guilty
4 Tony's claim for a R1 000 from Samuel will succeed, but his claim for the return of the dagga w1ll
fa1l because the contract between both part1es was only partially Illegal
5 1n order to achieve Justice between man and man, Samuel should pay Tony half of the claim,
which amounts to R500

(18) Consider the same set of facts as in question (14), the only difference be1ng that Tony was an
undercover policeman who sold the dagga to Samuel during a police entrapment operation. The
court hearing the matter is likely to rule that

1 the contractual claim for a R1 000 aga1nst Samuel is successful because it IS in the public Interest
and Tony was doing a good deed by trapping Samuel
2 the claim for the return of the dagga is unsuccessful in terms of an unjustified enrichment claim
3 the claim for a R 1 000 and the return of the dagga is successful
4 the claim for a R1 000 against Samuel IS unsuccessful because the contract is vo1d and
unenforceable
5 the claim for a R 1 000 and the claim for the return of the dagga are unsuccessful

(19) X has been leasing a commercial property from Z for the past three years. The lease will come to
an end on 31 May On 5 March, X phones Z and offers to renew the lease for a further three years,
which offer Z accepts. During this phone call, the material terms of the renewal agreement are
agreed upon and X and Z further agree that the said material terms must be reduced to writing and
signed by both parties Subsequently on 5 April, X IS shocked to rece1ve a letter from Z, advising X
that there Will be no renewal of the lease and that X should vacate the leased property on 31 May X
and Z never reduced the1r oral agreement to writing. Which is the most content1ous issue that
arises from th1s set of facts?

1 Whether a formality has been stipulated by X and Z


2 Whether a formality has been prescribed by law
3 Whether the Shifren principle applies to this dispute
4 Whether the contractual principles relating to restraint of trade applies to this dispute, based on
the Judgment in Goldblatt v Freemantle 1920 AD 123
5 Whether the principles relating to the exceptio doli generalis applies to this dispute
(20) Assume the same facts as in quest1on (19) Which statement(s) is/are CORRECT?

(a) Z bears the onus of proving that X and Z Intended that their oral agreement will only be binding if
it is reduced to writing and signed by them
(b) A court adjudicating upon this dispute is likely to find that X and Z intended that their oral
agreement will only be binding if it is reduced to writing and signed by them
(c) A court adjudicating upon this dispute is likely to find that the oral agreement is not enforceable
because it was not reduced to writing as s prescribed in section 12 of the Electronic Communications
and Transactions Act 25 of 2002
(d) In the given set of facts the law presumes that X and Z intended merely to facilitate proof of the
terms of the agreement since it is difficult to determine their intention

1 Only (a)
2 (a}, (b) and (c)
3 (a) and (b)
4 Only (d)
5 (a) and (d)

(21) Y and Z reach an agreement that should Y sell his leather couch, he (Y) Will offer to sell it to Z
first, before making an offer to sell the couch to any other person Which statement IS CORRECT?

1 Y and Z concluded an option contract


2 Y and Z concluded neither an option contract nor a pre-emption contract
3 Y and Z concluded a contract subject to a resolutive condition
4 Y and Z concluded an opt1on contract and a pre-emption contract
5 Y and Z concluded a pre-empt1on contract

(22) X contracts with Y for the latter (Y) to build and fit a security gate for the entrance of her (X's)
home Y builds the gate and fits it with an electric motor which IS activated with a remote control X is
satisfied with the work and pays Y the contractual amount agreed upon A week later the gate gets
stuck while it is half way open as a result of defective materials used to build the gate When X
attempts to physically move the gate to close it fully, she suffers such severe damage to her left knee
that she has to have a knee operation Her medical costs are R20 000 The costs of repa1nng the gate
amount to R15 000 X wants to cla1m both her medical costs, as well the cost of repairing the gate
from Y X IS likely pursue her claim for repairing the gate (R15 000), based on

1 the mitigation rule


2 special damages
3 general damages
4 the market value approach
5 the nominal damages rule

(23) Assume the same facts as in question (22) In order to claim her medical costs (R20 000), which
of the following requirements will be the most difficult for X to prove? You may assume that X has to
establish all of these requirements to be successful with her claim
1 That Y has committed a breach of contract
2 That X has suffered financial or patrimonial loss
3 That there is a factual causal link between the breach and the loss
4 That X's claim is not one for nominal damages
5 That the loss is not too remote as a consequence of the breach (that legal causation is present)

(24) Y lets premises to X The written lease contains a clause prohibiting X from subletting the
premises without the written consent of Y A further clause of the lease requires that any variation of
the terms of the lease (including this clause) has to be in writing and signed by both parties Later Y
and X agree orally that X can sub-let the premises After X has sub-let the premises to a third party, Y
changes his mind and informs X that both X and the sub-lessee (third party) should vacate the
premises because X has breached the contract by subletting the premises Which answer reflects the
correct legal position?

1 X does not breach the contract, because the oral agreement constitutes a binding contract, based
on the reliance theory
2 X breaches the contract, because the oral agreement is not legally binding
3 X does not breach the contract, because the oral agreement constitutes a binding tact contract
4 Y breaches the oral agreement and therefore X and the sub-lessee are entitled to remain on the
premises
5 X breaches the contract of lease and Y breaches the subsequent oral agreement. Therefore both
parties are in equal guilt, entitling X as the possessor to remain on the premises.

(25) X gives Y a cheque which is payable when X becomes 30 years old X IS 28 years old at present
The cheque is subject to a

1 suspensive time clause


2 resolutive time clause
3 modus (modal clause)
4 supposition
5 suspensive condition

QUESTION 1
Discuss the impact of the Consumer Protection Act 68 of 2008 upon the law of contract with
reference to its aims, objectives, scope, national regulatory institutions, and sanctions. [15]

Answer
The CPA is bound to have a huge impact on the conduct of businesses in South Africa, and the law of
contract.
The primary purpose of the Act is to protect consumers from exploitation in the marketplace, and to
promote their social and economic welfare. More specifically, it aims to:
• Establish a legal framework for the achievement and maintenance of a consumer market that is
fair, accessible, efficient, and responsible, for the benefit of consumers generally;
• Promote fair business practices;
• Protect consumers from unconscionable, unjust, or unreasonable business practices.
The scope of the Act is very wide. It applies to:
• Most transactions concluded in the ordinary course of business between suppliers and consumers
within South Africa, as well as;
• The promotion of goods and services that could lead to such transactions, and;
• The goods and services themselves once the transaction has been concluded.
A supplier is any person (including a juristic person, trust, and organ of State) who markets any goods
or services.
A consumer includes not only the end-consumer of goods and services but also:
• Franchisees
• Relatively small businesses in the supply chain (asset value or annual turnover below the threshold
determined by the Minister)
The Act does not apply to any transaction in terms of which goods and services are promoted or
supplied:
• To the State
• To a juristic person with an asset value or annual turnover above the threshold
• Employment contracts
• Credit agreements
• Transactions exempted by the Minister
These rights are protected and enforced not only through the courts, but the National Consumer
Commission and the National Consumer Tribunal. Failure to comply with provisions of the Act might
attract various sanctions, commencing with compliance notices and leading possibly to the
imposition of fines and criminal penalties. Contractual provisions in contravention of the Act may be
declared null and void to the extent of non-compliance.

QUESTION 2
State the elements of a fraudulent misrepresentation. [5]

Answer
1. A representation
2. which is, to the knowledge of the representer, false;
3. which the representer intended the representee to act upon;
4. which induced the representee to act; and
5. that the representee suffered damage as a result

QUESTION 3
State the different forms of breach of contract. [5]

Answer
1. Mora debitoris
2. Mora creditoris
3. Positive malperformance
4. Repudiation
5. Prevention of performance
QUESTION 4
X, an organiser of art exhibitions, contracted with Y for an exhibition to be held on 24 to 27 July.
These dates were the only dates mentioned during negotiations. After having been pressurized by X,
Y hurriedly signed a standard form contract without reading it. The contract contained a clause
permitting X to change the dates of the exhibition unilaterally. Thereafter, X changed the dates. X had
no reason to believe that Y would have signed the contract if he had known of the term. Y averred
that the contract was void. Will Y succeed in his attempt to have the contract set aside? Substantiate
your answer and refer to relevant case law. Do not apply the Consumer Protection Act to this
problem. [15]

Answer
The essence of this problem is whether X and Y have reached consensus. Y will not be contractually
bound if this requirement for a valid contract is absent.
At the outset, it must be considered whether agreement (consensus ad idem) as a contractual basis
exists between the parties, as required in terms of the will theory. Consensus has three elements:
1. The parties must seriously intend to contract
2. The parties must be of one mind as to the material aspects of the proposed agreement (the terms
and the identities of the parties to it)
3. The parties must be conscious of the fact that their minds have met.
In the present case, the parties were not in agreement as to the consequences they wished to
create: Y thought that the dates for the exhibition (X’s performance) was fixed, while X knew that the
contract allowed X to unilaterally change the dates. This is a mistake as to the obligations the parties
wished to create and is thus a material mistake, which excludes consensus between the parties. This
means that no contract could arise on the basis of the will theory.
This type of mistake can be illustrated with a number of cases:
In George v Fairmead, the appellant signed a hotel register without reading it.
The register contained a clause excluding the respondent from liability for certain acts. The appellant
was unaware of this term and his mistake related to a term that he believed would not be in the
contract and as such was material because it related to an aspect of performance.
In Allen v Sixteen Stirling Investments, the plaintiff believed he was purchasing the erf pointed out to
him by the seller’s agent, while the written contract that he signed indicated the correct erf, which
was a completely different property. His mistake related to performance and was material.
However, the matter does not end here. A party may be held contractually liable on the basis of a
supplementary ground for liability, namely the reliance theory.
In this regard, the direct or indirect approach to the reliance theory may be considered.
INDIRECT APPROACH (IUSTUS ERROR DOCTRINE):
In terms of this approach, a party may escape liability to be bound to a contract if it can be
established that the party labored under a mistake, which was both:
1. material and
2. reasonable.
It has already been shown that Y’s mistake is material in the discussion above. It still has to be
determined if Y’s mistake was reasonable.
The contract denier’s mistake will be reasonable in the following circumstances:
1. If caused by a misrepresentation on the part of the contract asserter (an unlawful
misrepresentation)
2. If the contract denier is not to blame for the mistake
3. If the contract denier did not cause the contract asserter to have a reasonable belief that the
contract denier assented to the contract.
If a legal duty to speak exists and a party has kept quiet when he ought to have spoken, that party
has made an unlawful negative misrepresentation. A legal duty to speak will usually exist where:
• The asserter knows or ought to know as a reasonable person that the other party is mistaken
• Where, prior to the conclusion of the agreement the asserter created an impression directly
conflicting with the provisions of the agreement, he must draw the contract denier’s attention to this
discrepancy (Du Toit v Atkinson’s Motors).
Since X had no reason to believe that Y would have signed the contract had Y known of the term
allowing X to change the dates of the exhibition unilaterally, X had a legal duty to point out this
clause to Y. X’s failure to do so renders Y’s material mistake reasonable.
DIRECT APPROACH:
With reference to the direct approach, contractual liability is based on the reasonable reliance that
consensus has been reached, which the one contractant (the contract denier) creates in the mind of
the other contractant (the contract asserter).
According to the Sonap case, the direct reliance approach involves a threefold enquiry:
1. Was there a misrepresentation regarding one party’s intention?
2. Who made this misrepresentation?
3. Was the other party actually misled by this misrepresentation, and if so, would a reasonable
person have been misled?
By signing the contract, Y, a party to the contract, misrepresented her intention to be bound by the
clause allowing X to unilaterally change the dates. X knew that the only dates mentioned during
negotiations were 24 to 27 July, that Y hastily signed the contract, and that the contract had a clause
allowing X to unilaterally change the dates. X was probably not actually misled by the
misrepresentation by Y, and nor would a reasonable person be misled in any event. There was
therefore no reasonable reliance on consensus on the part of Y.
Y is not bound by the agreement because of lack of apparent and actual consensus.

QUESTION 5
S, who lives in Upington, sends P, who lives in Grahamstown, a letter by private courier in which she
offers to sell him her (S’s) motorcycle, a collector’s piece, for R100 000. She states in her letter that
her offer will expire on 1 February. P accepts S’s offer by letter, which he posts on 31 January. S
receives the letter on 7 February and only reads it on the next day. P tenders payment of R100 000
but S refuses to accept payment. Did a valid contract arise between S and P? Substantiate your
answer. [15]

Answer
The question is whether P has accepted S’s offer in time and thus whether Sand P have reached
consensus.
Where the offeror has prescribed a time limit for acceptance, the offer lapses automatically if it is not
accepted within the prescribed period.
The general rule is that a contract comes into being only when the acceptance is communicated to
the mind of the offeror. The information theory, which is the general rule in our law, states that the
agreement is concluded when and where the offeror learns or is informed of the acceptance – in
other words, when the offeror reads the letter of acceptance.
On the other hand, the expedition theory applies to postal contracts. In terms of this theory,
introduced into our law in the Cape Explosive Works case, a contract comes into being when and
where the offeree posts the letter of acceptance. By making an offer through the post, the offeror is
deemed not only to have authorised acceptance by post, but also to have waived the requirement of
notification of acceptance.
The question that then arises is which theory applies. In our law, the general rule is that the
information theory applies, however the expedition theory will apply if the following four criteria are
met:
1. the offer is made by post or telegram
2. the postal services are operating normally
3. the offeror has not indicated a contrary intention, expressly or tacitly, and
4. the contract is a commercial one.
If any of these criteria are not met, the information theory applies. In this question, the offer was not
made by post, instead it was sent by private courier, and therefore the expedition theory does not
apply. It follows that the information theory must be applied. Because S only learnt of the
acceptance by P after expiry of the offer (when S read the letter on 8 February), the offer had already
lapsed and no valid contract arose between the parties.

PVL3702_Oct_2013
(1) The requirement that a contract must be legal for it to be valid means that,

1 the minds of the parties must meet (or at least appear to meet) on all material aspects of their
agreement
2 the parties must have the necessary capacity to contract
3 the agreement must not be prohibited by statute or common law
4 the obligations undertaken must be capable of performance when the agreement is entered into
5 the agreement must have a def1nrte or determinable content, so that the obligations can be
ascertained and enforced

(2) Which statement IS INCORRECT?

1 A contract can be a bilateral Juristic act


2 A contract entails promises or undertakings on one or both s1des
3 An undertaking m a contract that a certain state of affa1rs exists, or has existed, is known as a
warranty
4 The conclusion of a contract can never be multilateral
5 Freedom of contract means that the parties can agree to anything that is possible and lawful

(3) Which statement IS INCORRECT?


1 An obligation is a legal bond between a debtor and a creditor
2 With a contract of sale, the seller is the creditor in respect of the duty to deliver the thing sold,
and the seller is the debtor in respect to the duty to pay the price
3 An obligation comprises a right and a corresponding duty the right of the creditor to demand a
performance by the debtor, and the duty of the debtor to make that performance
4 A natural obligation 1s unenforceable m a court of law
5 The right created by an obligation us a personal right

(4) The declaration theory states that

1 the inner will or actual intention of the parties is relevant, but not what they say
2 the agreement comes into being when the letter of acceptance reaches the address of the offeror
3 the contract comes into being when and where the offerree posts his or her letter of acceptance
4 the focus is totally on a subjective approach
5 it IS Important what the parties say or do and not what they think

( 5) Which statement relating to the Constitution of the Republic of South Africa, 1996 is CORRECT?

1 In Everfresh Market Virginia (Pty) Ltd v Shoprite Checkers (Pty) Ltd 2012 (1) SA 256 (CC), the court
emphasised the central Importance of good faith in our contract law, and the desirability of
infusing the law of contract with constitutional values, including values of Ubuntu
2 Horizontal application relates to relationships between the state and the individual
3 In Barkhwzen v Nap1er 2007 (5) SA 323 (CC), the majority in the Constitutional Court ruled that a
contractual term can be tested directly against a provision in the Bill of Rights
4 In Barkhwzen v Nap1er 2007 (5) SA 323 (CC), the minority in the Constitutional Court preferred an
indirect application of the Constitution to the contractual dispute before them
5 Vertical application relates to relationships between private persons, as in most contractual
situations

(6) Which of the following does NOT lead to the termination of an offer?

1 Rectification of the offer


2 Rejection of the offer
3 Death of either the offeror or the offeree
4 Effluxion of the prescribed time , or of a reasonable time
5 Revocation of the offer

(7) Which of the following 1s NOT a requirement for the rescission (and restitution) of a contract
based on misrepresentation?

1 Misrepresentation by the other party


2 Inducement
3 intention to induce
4 The one party obtains an Influence over the other party
5 Materiality

(8) An option is

1 only an offer to sell something


2 only an agreement to keep an offer open for a certain period of time
3 both an offer to sell something, and an agreement to keep an offer open for a certain period of
time
4 an agreement to make an offer revocable for a certain period of time
5 none of the above

(9) Which of the following is NOT an element for commercial bribery?

1 A reward
2 The one party IS the briber
3 The agent is able to exert Influence over a third party
4 The agent may be an agent 1n the true sense or merely a go-between or fac1htator
5 The threat must be unlawful

(10) If commerc1al bribery exists, the agreement between the briber and the agent is

1 voidable
2 for the benefit of a third party
3 void and voidable
4 neither void nor voidable
5 unenforceable and has no legal effect

(11) If commercial bribery exists, the agreement between the briber and the principal is

1 voidable due to illegality


2 Voidable because consensus was improperly obtained
3 void due to Illegality
4 void because consensus was improperly obtained
5 for the benefit of a third party

(12) Kate and Martha agree orally that if Kate decides to sell her watch, she will offer to sell it to
Martha first This is an example of

1 an option contract
2 a pre-emption contract
3 an absolving agreement
4 a real agreement
5 a transfer agreement

(13) Assume the same facts as m question (12). Which statement IS INCORRECT?
1 This agreement gives Martha the right to be granted the first opportunity to buy the watch, if Kate
decides to sell it
2 An essential feature of this agreement IS that Kate is under an obligation to sell the watch within
a reasonable time from the date that she (Kate) concluded the agreement with Martha
3 This agreement which creates a right of first preference is a type of pactum de
contrahendo
4 This agreement has been concluded with an offer and acceptance
5 This agreement IS valid and legally binding even though it has not been concluded in writing

(14) Assume the same facts as in question (12) Assume further that Kate advises Martha that she
(Kate) is going to sell her watch to Julia, next week Martha wants to compel Kate to sell the watch to
her (Martha) How would you best advise Martha?

1 To cancel the contract and seek to interdict Kate from selling the watch to Julia
2 To enforce the contract and cancel the contract simultaneously through the use of an interdict
3 To cancel the contract and seek an order for specific performance against Kate
4 To enforce the contract and seek to interdict Kate from selling the watch to Julia
5 To cancel the pre-emption agreement between Kate and Julia

(15) Assume the same facts as in question (12) Assume further that Kate goes ahead and sells the
watch to Julia, without first offering it to Martha to purchase In terms of the agreement between
Kate and Martha, what type of breach has Kate committed?

1 Mora debitoris
2 Mora creditoris
3 Prevention of performance
4 Positive of performance
5 Repudiation

(16) Assume the same facts as in questions (12) and (15). Which statement is CORRECT?
1 Under no circumstances will a court place Martha into Julia's shoes through a unilateral declaration
of intent
2 In Associated SA Bakeries (Pty) Ltd v Oryx & Vereinigte Backereien (Pty) Ltd 1982
(3) SA 893 (A) the court had no doubt that the holder (Martha) could claim specific performance by
means of an order d1rect1ng the grantor (Kate) to make her an offer
3 If Kate has already delivered the watch to Julia, Martha would not be able to pursue the watch with
her personal right, unless Julia was aware of the agreement between Kate and Martha
4 If Kate has already delivered the watch to Julia, Martha can successfully claim the watch back from
Julia, as she (Martha) has a real right over the watch
5 If Kate has already delivered the watch to Julia, Martha can successfully claim the watch back from
Julia, as she (Martha) has both a real and personal right over the watch

(17) X is a keen golfer who has played at many golf tournaments over the years as an amateur. She is
very well Informed about the rules pertaining to her amateur status as a golfer and knows that
amateurs can only claim a maximum of R1 000 in prize money at golf tournaments X participated in a
recent golfing tournament wherein she achieved a hole-in-one at the ninth hole At this hole was an
advertising board which read "Hole-in-one prize sponsored by Speedy Motors to the value of
R90 000" The prize was parked next to this board in the form of a new car X claimed the prize from
Speedy Motors but they rejected her cla1m on the basis that the prize could only be claimed by
professional players and not amateur players. Regarding X's claim for delivery of the car, which is the
most contentious issue?

1 Whether the acceptance was in the form prescribed by the offeror


2 Whether the offer was firm
3 Whether the offer was complete
4 Whether the acceptance was unqualified
5 Who the offer was made to

(18) Assume the same facts in question (17) In order to enforce X's claim for the delivery of the car,
she IS likely to rely on the

1 application of the classical legal test that was formulated in Sonap Petroleum (SA) (Ply) Ltd v
Pappaddogianis 1992 (3) SA 234 (A)
2 1ndirect application of the reliance theory, if the will theory fails
3 the doctrine relating to reasonable and material mistake
4 direct application of the reception theory
5 application of the iustus error doctrine

(19) Assume the same facts in question (1 7) What is/are the most likely finding(s) that a court Will
reach?

(a) That objectively speaking, X's cla1m for delivery of the car is valid
(b) That X's claim for the car IS invalid
(c) That there was no real or actual consensus between X and Speedy Motors
(d) That a reasonable person in X's position would have realised that the offer was not open for
acceptance to amateur golfers
1 (a)
2 (c)
3 (a) and (d)
4 (c) and (d)
5 (b), (c) and (d)

(20) The Consumer Protection Act 68 of 2008 applies to

1 any transaction for goods supplied to the state


2 any transaction in terms of which services are promoted to a Juristic person with an asset value or
annual turnover above the prescribed threshold
3 employment contracts
4 credit agreements
5 relatively small businesses in the supply chain with an asset value or annual turnover below the
threshold determined by the Minister

(21) A contractual term in a contract which is regulated by the Consumer Protection Act 68 of 2008
("the Act"), that purports to limit or exclude the liability of a supplier (or those for whom he or she is
responsible) for harm caused by gross negligence is

1 valid and binding only if the supplier has made the customer aware that such a provision exists in
the contract, at the time of concluding the contract
2 void to the extent that the term is non-compliant with the Act
3 valid and binding, which is consistent with the current common law position
4 void and voidable at the Instance of the consumer
5 voidable to the extent that the term is non-compliant with the Act

(22) X contracts with Y for the latter (Y) to build and fit a security gate for the entrance of her (X's)
home Y builds the gate and fits it with an electric motor which is activated with a remote control X is
satisfied with the work and pays Y the contractual amount agreed upon A week later the gate gets
stuck while it is half way open as a result of defective materials used to build the gate When X
attempts to physically move the gate to close it fully, she suffers an Injury to her left knee that she
has to have a knee operation Her medical costs are R20 000 The costs of repairing the gate amount
to R15 000 X wants to claim both her medical costs, as well the cost of repairing the gate from Y X is
likely pursue her claim for repairing the gate (R15 000), based on

1 the mitigation rule


2 special damages
3 general damages
4 the market value approach
5 the nominal damages rule

(23) Assume the same facts as in question (22) In order to claim her medical costs (R20 000), which
of the following requirements will be the most difficult for X to prove? You may assume that X has to
establish all of these requirements to be successful with her claim

1 That Y has committed a breach of contract


2 That X has suffered financial or patrimonial loss
3 That there is a factual causal link between the breach and the loss
4 That X's claim is not one for nominal damages
5 That the loss is not too remote as a consequence of the breach (that legal causation is present)

(24) Assume the same facts as in question (22) In respect of X's claim for her medical costs (R20 000)
and in order to make it difficult for X to succeed with this claim, Y is likely to argue that this claim
relates to

1 the mitigation rule


2 special damages
3 general damages
4 the market value approach
5 the nominal damages rule

(25) Assume the same facts as in question (22) If X can prove that Y has committed a breach of their
agreement, what type of breach will X rely on?
1 mora debitoris
2 mora creditoris
3 prevention of performance
4 positive malperformance
5 repudiation

QUESTION 2
Briefly discuss tacit terms. [5]

Answer
A tacit term is one that the parties did not specifically agree upon, but which (without anything being
said) both or all of them expected to form part of their (oral or written) agreement. It is a wordless
understanding having the same legal effect as an express term.
In ascertaining whether a contract contains a tacit term, the courts often employ the officious
bystander test:
The court supposes that an impartial bystander had been present when the parties concluded their
agreement and had asked the parties what would happen in a situation they did not foresee and for
which their express agreement did not provide. If they were to agree that the answer to the
stranger’s question was self-evident, they are taken to have meant to incorporate the term into their
contract and to have tacitly agreed on it.

QUESTION 3
X has been leasing a commercial property from Z for the past three years. The leas will come to an
end on 31 May 2010. On 5 March 2010, X phones Z and offers to renew the lease for a further three
years, which offer Z accepts. During this phone call, the material terms of the renewal agreement are
agreed upon and X and Z further agree that the said material terms must be reduced to writing and
signed by both parties. Subsequently, on 5 April 2010, X is shocked to receive a letter from Z, advising
X that there will be no renewal of the lease and that X should vacate the leased property on 31 May
2010. X and Z never reduced their oral agreement to writing. Advise X if a binding agreement with Z
exists for the renewal of the lease for a further three years. Refer to Goldblatt v Fremantle. [15]

Answer
This question deals essentially with formalities stipulated by the parties for a valid creation of a
contract. The main question is whether a formality was stipulated in the oral agreement for the
renewal of lease between the parties, that for such agreement to be valid it should be reduced to
writing.
Parties to an oral agreement will often agree that their agreement should be reduced to writing, and
perhaps also signed. In doing so, they may have the following intentions:
1. To have a written record of their agreement to facilitate proof of its terms.
If so, the agreement is binding even if it is never reduced to writing.
2. Alternatively, they may intend that their oral agreement will not be binding upon them until it is
reduced to writing and signed by them. In Goldblatt v Fremantle, the Appellate Division held that no
contract existed because the parties intended their agreement to be concluded in writing, which also
involved signing by the parties.
In the absence of contrary evidence, the law presumes that the intention of the parties was merely
to facilitate proof of the terms of the agreement. The party who alleges otherwise bears the onus of
proof.
In our case no binding agreement exists because the parties agreed that the oral agreement must be
reduced to writing and signed, and this indicates their intention that the agreement will not be
binding if this formality is not complied with.

QUESTION 4
Andy and Craig conclude a contract wherein Andy agrees to paint Craig’s office block by 31 August,
and Craig agrees to pay Andy R10 000 upon completion of the work. When 80% of the work is
completed Andy suddenly falls ill and he is unable to complete the job by 31 August. Craig refuses to
pay Andy any money for his (Andy’s) services rendered, as Craig believes that Andy has breached the
contract by not completing the work. Craig hires another contractor at an amount of R3000 to
complete the job. Craig does not incur any other costs to complete the job, neither does his business
make any losses. Advise Andy as to what amount (if any) he may recover from Craig for the services
that he rendered, and on what basis. Discuss with reference to BK Tooling (Edms) Bpk v Scope
Precision Engineering (Edms) Bpk and other relevant case law. [15]

Answer
This contract is reciprocal in nature. Andy has rendered defective performance and the issue is
whether Craig has to compensate Andy for the work that has already been done.
This question deals with the exceptio non adimpleti contractus. The exceptio is a defence that can be
raised in the case of a reciprocal contract, where the performances due on either side are promised
in exchange for one another. It is a remedy that permits a party to withhold their performance and
ward off a claim for such performance until such time as the other party has either performed or
tendered performance of their obligations.
Where a party who has to perform first has only performed part of its obligations or has rendered
defective performance, which the party is in principle not entitled to claim counter-performance
until such time as he has performed in full. In practice, the innocent party often accepts part-
performance and starts using the performance. This sometimes leaves the breaching party in the
unfair position that it may be impractical or impossible to make full performance, but any claim for
counter-performance can be defended by the other party relying on the exceptio.
As a result, the courts have exercised discretion to relax the principle of reciprocity and order the
party making use of the defective or incomplete performance to pay a reduced amount to the party
in breach.
In BK Tooling, the Appellate Division confirmed this, and held that the courts have an equitable
discretion to award a reduced contract price, depending on the nature of the defect, and the cost of
repair, replacement, or substitute performance. The onus to prove the amount of reduction is on the
party in breach claiming the reduced price. The plaintiff must allege and prove:
• that the other party is using his performance
• the cost of remedying defects
• that it would be equitable to award some remuneration despite breach
• that the circumstances are such that the court should exercise its discretion
Based on the ruling in BK Tooling, Andy is entitled to be compensated by Craig because:
1. Craig is utilizing the defective performance
2. It would be equitable as Andy has completed most of the work
3. The counter-performance ought to be reduced by R3000 (the amount it cost to complete the job)
In the circumstances, Andy is entitled to receive R7000 from Craig, which represents the difference
between the contract price and the cost to complete the job.

PVL3702_May 2014

(1) A selects and pays for several items in a supermarket, and leaves. What type of agreement has
been concluded?

1 An agreement creating obligations


2 An agreement extinguishing a debt
3 A real agreement
4 All of the above

(2) Generally, an advertisement IS

1 an offer to the public


2 an invitation to do business
3 an offer directed at defined persons
4 an offer directed at undefined persons
5 1 and 4

(3) The notion that contracts are based on consensus, has given rise to which theory?

1 The declaration theory


2 The will theory
3 The reliance theory
4 The reception theory
5 The expedition theory

(4) X makes a written offer to Y to purchase Y's house X also states 1n the offer that Y has until 15
May to accept the offer X has the offer send by courier to Y. Y reads the offer on 13 May and drafts an
acceptance on 14 May Y posts his acceptance on 15 May to X. X receives the acceptance on 17 May
and reads it on 18 May. Which theory must be applied to determine whether a contract arose?
1 The declaration theory
2 The expedition theory
3 The reliance theory
4 The reception theory
5 The information theory

(5) X sells his shop (a business) in a shopping mall to Y. During the negotiations X Informs
Y of the overhead costs of running the shop but fails to mention that the rental for the shop space IS
soon to escalate dramatically In fact the rent will be so high that the shop will scarcely be able to turn
a profit What cause of action Will Y be able to rely on in the circumstances in order to claim
damages?

1 Dictum et promissum
2 Innocent misrepresentaton
3 Culpable misrepresentation
4 Material mistake
5 Non-material mistake

(6) Assume the same facts as in question (5). If Y would not have bought the shop had she (Y) known
of the rent escalation, according to what measure would Y's damages be calculated in the
Circumstances?

1 The actio quanti minoris


2 The actio redhibitoria
3 Positive interest
4 Dolus incidens
5 Dolus dans

(7) Assume the same facts as in question (5). If Y wanted to cancel the contract what would she
usually have to do?

1 Tender restitution
2 Invoke the exceptio non adimpleti contractus
3 Pay damages to X
4 Use the actio redhlbiloria
5 None of the above

{8) Which theory finds application under the Electronic Commumcations and Transactions
Act 2 5 of 2002?

1 The declaration theory


2 The exped1t1on theory
3 The 1nformat1on theory
4 The reception theory
5 The reliance theory
(9) Kate makes an offer to sell her watch to Martha for R2 500 Kate and Martha agree on 15 May that
Kate's offer will be open for acceptance until 31 May On 20 May Kate informs Martha in writing that
she (Kate) 1ntends to sell the watch to Julia on 30 May. On 21 May Martha informs Kate that she
accepts the offer, but Kate refuses to sell the watch to her Wh1ch statement 1s CORRECT?

1 Kate and Martha concluded a pre-empt1on contract on 15 May


2 Kate and Martha concluded a pre-empt1on contract and an opt1on contract on 15 May
3 Kate and Martha concluded a pre-empt1on contract on 21 May
4 Kate and Martha concluded an opt1on contract on 21 May
5 Kate and Martha concluded an opt1on contract on 15 May

(1 0) Assume the same facts as in question (9) Martha wants to compel Kate to sell the watch to her
How would you best advise Martha?

1 To cancel the contract and seek to interdict Kate from selling the watch to Julia
2 To enforce the contract and seek to interdict Kate from selling the watch to Julia
3 To cancel the contract and seek an order for specific performance against Kate
4 To enforce the contract and cancel the contract simultaneously through the use of an interdict
5 To cancel the pre-emption agreement between Kate and Julia

(11) Tony, a petrol attendant, sells dagga to Samuel for R1 000 Tony delivers the dagga to Samuel, but
Samuel refuses to pay Section 5 of the Drugs and Drug Trafficking Act 140 of 1992 provides that no
person shall deal in (a) any dependence-producing substance or (b) any dangerous dependence-
producing substance or any undesirable dependence-producing substance, while section 4 prohibits
the possession of such substances Section 13 makes the contravention of both sections 4 and 5 a
crime Dagga is furthermore an undesirable dependence-producing substance Tony approaches a
court to assist him, either for the payment of R1 000 from Samuel or the return of the drugs The
material focus of this dispute rests on which requirement relating to the validity of the agreement
between Tony and Samuel?

1 Consensus
2 Formalities
3 Possibility
4 Legality
5 Certainty

( 12) Assume the same facts as in question ( 11) Most relevant to the dispute would be to consider
the application of

1 the ex turpi rule and the par delictum rule


2 statutory illegality and restraint of trade agreements
3 public interest and rectification
4 the par delictum rule and capacity of the parties to contract
5 the ex turpi rule and stipulatio alten
(13) Assume the same facts as in question (11). Which of the following factors must be considered in
order to determine whether the legislator impliedly intended for the contract between Tony and
Samuel to be void?

(a) What IS the object of the statute and what mischief (harm) is the statute directed against?
(b) Does the enactment impose a criminal sanction, or does the enactment merely serve to protect
the revenue of the State?
(c) Does the provision in the Act merely protect individuals or does it involve a public interest that
requires protection by voiding the contract?
(d) What are the consequences of a particular interpretation of the contract?

1 Only (a)
2 Only (b)
3 (a), (b) and (c)
4 (a), (b) and (d)
5 (a), (b), (c) and (d)

( 14) Assume the same facts as in question ( 11) The court hearing this dispute is likely to find that

1 Tony can cla1m R1 000 contractual damages from Samuel


2 Tony will be granted an order for specific performance for the return of the dagga
3 Tony's request for the return of the dagga will fall because both parties were equally morally
guilty
4 Tony's claim for a R1 000 from Samuel will succeed, but his claim for the return of the dagga Will
fall because the contract between both parties was only partially Illegal
5 in order to achieve justice between man and man, Samuel should pay Tony half of the claim, which
amounts to R500

( 15) Consider the same set of facts as in question ( 11 ), the only difference being that Tony was an
undercover policeman who sold the dagga to Samuel during a police entrapment operation The
court hearing the matter is likely to rule that

1 the contractual claim for a R1 000 against Samuel is successful because it is in the public interest
and Tony was doing a good deed by trapping Samuel
2 the claim for the return of the dagga is unsuccessful in terms of an unjustified enrichment claim
3 the claim for a R1 000 and the return of the dagga is successful
4 the claim for a R 1 000 against Samuel is unsuccessful because the contract is void and
unenforceable
5 the claim for R 1 000 and the claim for the return of the dagga are unsuccessful

(16) X has been leasing a commercial property from Z for the past three years. The lease will come to
an end on 31 May On 5 March, X phones Z and offers to renew the lease for a further three years,
which offer Z accepts. During this phone call, the material terms of the renewal agreement are
agreed upon and X and Z further agree that the said material terms must be reduced to writing and
signed by both parties. Subsequently on 5 April, X is shocked to receive a letter from Z, advising X
that there will be no renewal of the lease and that X should vacate the leased property on 31 May X
and Z never reduced their oral agreement to writing. Which is the most contentious issue that arises
from this set of facts?

1 Whether a formality has been stipulated by X and Z


2 Whether a formality has been prescribed by law
3 Whether the Shifren principle applies to this dispute
4 Whether the contractual principles relating to restraint of trade applies to this dispute, based on
the Judgment in Goldblatt v Freemantle 1920 AD 123
5 Whether the principles relating to the exceptio doli generalis applies to this dispute

(17) Assume the same facts as in question (16). Which statement(s) is/are CORRECT?

(a) Z bears the onus of proving that X and Z intended that their oral agreement will only be binding if
it is reduced to writing and signed by them
(b) A court adjudicating upon this dispute IS likely to find that X and Z intended that their oral
agreement will only be binding if it is reduced to writing and signed by them
(c) A court adjudicating upon this dispute is likely to find that the oral agreement is not enforceable
because it was not reduced to writing as is prescribed in section 12 of the Electronic Communications
and Transactions Act 25 of 2002
(d) In the given set of facts the law presumes that X and Z Intended merely to facilitate proof of the
terms of the agreement since it is difficult to determine their intention

1 Only (a)
2 (a), (b) and (c)
3 (a) and (b)
4 Only (d)
5 (a) and (d)

(18) Y and Z reach an agreement that should Y sell his leather couch, he (Y) will offer to sell it to Z
first, before making an offer to sell the couch to any other person. Which statement IS CORRECT?

1 Y and Z concluded an option contract


2 Y and Z concluded neither an option contract nor a pre-emption contract
3 Y and Z concluded a contract subject to a resolutive condition
4 Y and Z concluded an option contract and a pre-emption contract
5 Y and Z concluded a pre-emption contract

(19) The majority in Barkhwzen v Nap1er 2007 (5) SA 323 (CC) held that

1 pacta sunt servanda is not a profoundly moral principle on which the coherence of any society
relies
2 the validity of a contractual term can be directly tested against a provision of the Bill of Rights in
the Constitution
3 unfair or unreasonable contractual provisions would always be enforced
4 self-autonomy or the ability to regulate one's own affairs, even to one's own detriment, is the very
essence of freedom and a vital part of dignity
5 public policy is not Informed by the concept of Ubuntu

(20) What is the general effect of supervening impossibility of performance?

1 Termination of the contractual obligation


2 The contract is rendered voidable at the instance of the creditor
3 The contract is rendered voidable at the instance of the debtor
4 The cred1tor IS guilty of contractual breach
5 The debtor IS guilty of contractual breach

(21) C, a builder, and 0 agree that C Will build a house on O's stand for R800 000, payable on
completion of the house The house must be completed by 30 April, but on that date the house IS
only 90% complete It would cost R 100 000 to complete the house O refuses to pay any amount to C
because C failed to complete the house timeously If C institutes legal proceedings against O for
payment, which defence IS O likely to raise?

1 The exceptio doli


2 The pactum de contrahendo
3 The stipulatio alten
4 The exceptio non adimpleti contractus
5 The pacta sunt servanda

(22) Assume the same facts in question (21.) The contract between the parties is a

1 contract that is both unilateral and bilateral


2 unilateral contract that is a reciprocal contract
3 bilateral contract that is not a reciprocal contract
4 unilateral contract that is not a reciprocal contract
5 bilateral contract that is a reciprocal contract

(23) Assume the same facts in question (21). Which answer reflects the CORRECT legal position?

1 As C did not complete the house timeously, the rule is absolute that C is not entitled to any
payment
2 C is entitled to a reduced counter performance, quantified as R720 000 (90% of R800 000)
provided inter alia that it is equitable
3 C is entitled to a reduced counter performance, quantified as R700 000, provided inter alia that O is
utilising the defective performance
4 Because C breached the contract, O can use her discretion to decide what amount to pay C
provided inter alia that it is a reasonable amount
5 C is entitled to a reduced counter performance, quantified as the difference between the market
value the house would have had in its completed state and the current market value of the house in
its uncompleted state

(24) X agrees with Y that Y will paint X's holiday home at the coast Unbeknown to either of them the
house had been destroyed in a storm the previous day. This is a case of

1 mutual mistake
2 impossibility of performance
3 prevention of performance
4 supervening impossibility of performance
5 an obligation subject to a resolutive condition

(25) X gives Y a cheque which is payable when X becomes 30 years old X is 28 years old at present
The cheque is subject to a

1 suspensive time clause


2 resolutive time clause
3 modus (modal clause)
4 supposition
5 suspensive condition

QUESTION 1
Discuss the impact of the Consumer Protection Act 68 of 2008 upon the law of contract with
reference to its aims, objectives, scope, national regulatory institutions, and sanctions. [10]

Answer
The CPA is bound to have a huge impact on the conduct of businesses in South Africa, and the law of
contract.
The primary purpose of the Act is to protect consumers from exploitation in the marketplace, and to
promote their social and economic welfare. More specifically, it aims to:
• Establish a legal framework for the achievement and maintenance of a consumer market that is
fair, accessible, efficient, and responsible, for the benefit of consumers generally;
• Promote fair business practices;
• Protect consumers from unconscionable, unjust, or unreasonable business practices.
The scope of the Act is very wide. It applies to:
• Most transactions concluded in the ordinary course of business between suppliers and consumers
within South Africa, as well as;
• The promotion of goods and services that could lead to such transactions, and;
• The goods and services themselves once the transaction has been concluded.
A supplier is any person (including a juristic person, trust, and organ of State) who markets any goods
or services.
A consumer includes not only the end-consumer of goods and services but also:
• Franchisees
• Relatively small businesses in the supply chain (asset value or annual turnover below the threshold
determined by the Minister)
The Act does not apply to any transaction in terms of which goods and services are promoted or
supplied:
• To the State
• To a juristic person with an asset value or annual turnover above the threshold
• Employment contracts
• Credit agreements
• Transactions exempted by the Minister
These rights are protected and enforced not only through the courts, but the National Consumer
Commission and the National Consumer Tribunal. Failure to comply with provisions of the Act might
attract various sanctions, commencing with compliance notices and leading possibly to the
imposition of fines and criminal penalties. Contractual provisions in contravention of the Act may be
declared null and void to the extent of non-compliance.

QUESTION 2
State the requirements for duress and undue influence. [10]

Answer
DURESS (improper pressure that amounts to intimidation):
1. Actual violence or reasonable fear
2. The fear must be caused by the threat of some considerable evil
3. It must be the threat of an imminent or inevitable evil
4. The threat or intimidation must be contra bonos mores
5. The moral pressure must have caused damage
UNDUE INFLUENCE (The party who seeks to set aside the contract must establish):
1. The other party obtained an influence over the party
2. This influence weakened his or her powers of resistance and rendered his will compliant
3. The other party used this influence in an unscrupulous manner to persuade him or her to agree to
a transaction that
a. was prejudicial to him or her
b. he or she would not have concluded with normal freedom of will

QUESTION 3
Albert takes his motor vehicle to Dodgy Motors for a service. On his arrival, he is asked to sign a “job
card” by the owner. Albert enquires why he is required to sign the “job card” and the owner explains
to him that by signing he is authorising them to conduct the service on his car, which will cost R1000.
He signs the “job card” without reading it. While servicing the car, the service manager finds faults
on the car (unrelated to the service) and he proceeds to do these additional repairs for a further
R2000. Albert refuses to pay for the additional repairs and argues that he did not authorise such
repairs. The owner of Dodgy Motors argues that Albert is obliged to pay for the work done as the
“job card” contains a contractual clause authorising Dodgy Motors to do any repairs on the motor
vehicle which they deem necessary, without asking the client’s authorisation, and requiring the client
to pay for such repairs. Advise Albert on whether he is liable on the contract to pay Dodgy Motors
R2000 for the additional repairs. Refer to George v Fairmead (Pty) Ltd, Sonap Petroleum (SA) Ltd (SA)
(Pty) Ltd v Pappadogianis, and other relevant case law in your answer. Do not apply the Consumer
Protection Act to this question. [15]

Answer
The essence of this problem is the question whether Albert and the owner of Dodgy Motors have
reached actual consensus or ostensible consensus. Albert will not be contractually bound to pay for
the additional repairs if this requirement for a valid contract is absent.
At the outset, it must be determined whether agreement (consensus ad idem) as a contractual basis
exists between the parties, as required in terms of the will theory. Consensus has three elements:
1. The parties must seriously intend to contract
2. The parties must be of one mind as to the material aspects of the proposed agreement (the terms
and the identity of the parties to it)
3. The parties must be conscious of the fact that their minds have met
In the present case, the parties were not in agreement as to the consequences they wished to
create; Albert thought that he was authorising Dodgy Motors to only service his car, while the owner
of Dodgy Motors knew that the contract also allowed Dodgy Motors to conduct repairs which they
deemed necessary and payable by Albert without any further authorisation from Albert. This is a
mistake as to the obligations the parties wished to create and is thus a material mistake, which
excludes consensus between the parties. This means that no contract could arise on the basis of the
will theory.
This type of mistake can be illustrated with a number of cases:
In George v Fairmead, the appellant signed a hotel register without reading it. The register contained
a term excluding the respondent from liability for certain acts. The appellant was unaware of this
term and his mistake related to a term that he believed would not be in the contract and as such was
material because it related to an aspect of performance.
In Allen v Sixteen Stirling Investments, the plaintiff believed he was purchasing the erf pointed out to
him by the seller’s agent, while the written contract that he signed indicated the correct erf, which
was a completely different property. His mistake related to performance and was material.
However, the matter does not end here. A party may be held contractually liable on the basis of a
supplementary ground for liability, namely the reliance theory.
In this regard, the direct or indirect approach to the reliance theory may be considered.
DIRECT APPROACH:
With reference to the direct approach, contractual liability is based on the reasonable reliance that
consensus has been reached, which the one contractant (the contract denier) creates in the mind of
the other contractant (the contract enforcer).
According to the Sonap case, the direct reliance approach entails a threefold enquiry:
1. Was there a misrepresentation regarding one party’s intention?
2. Who made the misrepresentation?
3. Was the other party misled by the misrepresentation, and if so, would a reasonable person have
been misled?
In our question, firstly, Albert made a misrepresentation by signing the contract, that his intention is
the same as that expressed in the contract.
Secondly, the owner of Dodgy Motors could actually have been misled by this misrepresentation, but
a reasonable man would have taken steps to point out to
Albert that the contract allows Dodgy Motors to unilaterally conduct repairs on the car, because
Albert enquired about the purpose of the “job card” and the owner of Dodgy Motors misled him to
believe that by signing the card he is merely authorising the service to be done. In Sonap the court
found that the contract enforcer knew that the contract denier was acting under a mistake and was
thus not misled.
In our case, Albert therefore did not create a reasonable reliance that he wished to be bound by the
contract he signed.
INDIRECT APPROACH (IUSTUS ERROR DOCTRINE):
In terms of this approach, a party may escape liability to be bound to a contract if it can be
established that the mistake is both:
1. Material, and
2. Reasonable
It has already been shown in the discussion above that Albert’s mistake is material. It still has to be
determined if his mistake was reasonable.
The contract denier’s mistake will be reasonable in the following instances:
1. If caused by a misrepresentation on the part of the contract enforcer (an unlawful
misrepresentation).
2. If the contract denier is not to blame for the mistake.
3. If the contract denier did not cause a reasonable belief in the contract asserter that the contract
denier assented to the agreement.
Fault is not a requirement for the misrepresentation by the contract enforcer, but unlawfulness is. If
the misrepresentation is a positive act it is unlawful in itself.
If a legal duty to speak exists and the party has kept quiet when he ought to have spoken, an
unlawful negative misrepresentation has occurred. A legal duty to speak exists in the following
instances:
• Where the contract asserter knows or ought to know as a reasonable person that the other party is
mistaken
• Where, prior to the conclusion of the agreement, the contract asserter created an impression
directly conflicting with the provisions of the agreements, he must draw the contract denier’s
attention to the discrepancy (Du Toit v Atkinson’s Motors).
In our problem, Albert enquired about the purpose of the “job card” and the owner of Dodgy Motors
misled him by answering that by signing he was merely authorising the service. The owner’s
misrepresentation was a positive act, and was therefore unlawful. Albert’s error was thus
reasonable.
Applying the indirect approach to the reliance theory we do not have a valid contract. Applying the
direct approach, we do not have a valid contract. Albert is not contractually liable to pay R2000 for
the repairs.
QUESTION 4
On 1 June M and Q conclude a contract whereby M undertakes to manufacture and install kitchen
cupboards in Q’s home for R50 000. The parties agree that the price will be paid as soon as the
kitchen cupboards are installed, but they do not determine a date for the completion of the work. M,
however, informs Q during the negotiations that she has some other work to complete and that she
will attend to the kitchen cupboards as soon as possible. Eight months has lapsed since the contract
was concluded and Q has not heard from M. Q runs out of patience and hires W to manufacture and
install the same kitchen cupboards for R60 000. After W has completed the job, M turns up to do the
work. Q claims R10 000 damages from M, but M institutes a counterclaim for R30 000 from Q for her
loss of profit. Who will succeed in this claim? Discuss. [15]

Answer
This question deals with damages for breach of contract. In order to determine who will succeed in
the claim for damages, we must ascertain which party committed the breach.
A plaintiff who wishes to claim damages for breach of contract must prove the following:
1. A breach of contract has been committed by the defendant
2. The plaintiff has suffered financial or patrimonial loss
3. There is a factual causal link between the breach and the loss
4. As a matter of legal causation, the loss is not too remote a consequence of the breach.
Did M or Q breach the contract?
M could possibly be in breach in the form of mora debitoris. Mora debitoris is the unjustifiable failure
of a debtor to make timeous performance of a positive obligation that is due and enforceable, and
still capable of performance in spite of such failure. Because performance has become impossible, it
is not capable of performance. Also, no date was stipulated for performance, nor did Q demand
performance, so M could neither be in mora ex re nor mora ex persona respectively.
M has not committed a breach of contract.
Q, by hiring W to manufacture the cupboards has committed two forms of breach: repudiation, and
prevention of performance. A party commits the breach of repudiation when, by words or conduct,
and without lawful excuse, he manifests an unequivocal intention no longer to be bound by the
contract or any obligation forming a part thereof.
Prevention of performance is a breach whereby, after conclusion of the contract, one of the parties,
owing to their fault, causes performance to become impossible.
M will therefore be able to claim damages from Q successfully, because he will be able to prove that
Q committed a breach of contract. The aim of damages is to place the innocent party in their
fulfillment position, that is, the position they would have been in had there been no breach. M’s
claim for loss of profit will probably be successful.

PVL3702_Oct_2014

(1) Which concept is NOT a value that informs the law of contract?

1 Freedom of contract
2 The requirement that a contract must not be against public policy
3 Privity of contract
4 Good faith
5 Pacta sunt servanda

(2) In case law the Constitution has already impacted on the law of contract in the following ways

1 Some High Courts have reversed the onus which rests on contract denier to prove that an
agreement in restraint of trade is against public policy to the position before Magna Alloys and
Research (SA) (Pty) Ltd v Eilts 1984 (4) SA 874 (A)
2 In ascertaining whether the exerc1se of contractual powers is against public policy, public policy
has been determined. With reference to the fundamental values embodied in the Constitution
3 In ascertaining whether a contractual term is contrary to public policy, public policy has been
determined with reference to a fundamental value embodied in the Constitution
4 2 and 3
5 1, 2 and 3

(3) Inequality of bargaining power between the parties is taken into account as such to determine if

1 a contract is illegal
2 undue influence has taken place
3 duress has taken place
4 1 and 2
5 1, 2 and 3

(4) X makes a written offer to Y to purchase Y's house X sends the offer by post to Y. Y reads the offer
on 13 May and drafts an acceptance on 14 May Y posts his acceptance on 15 May to X. X receives the
acceptance on 17 May and reads it on 18 May. When was the contract concluded?
1 13 May
2 14 May
3 15 May
4 17 May
5 18 May

(5) X finds himself stranded in the remote Richtersveld when his 4x4 vehicle breaks down. There is no
cell phone reception in the area. After five days have passed without seeing anyone, Y comes along
and offers to tow X to the nearest town for the exorbitant fee of R20 000 X initially turns the offer
down, but his fiancé, Z, makes such a scene that he relents and accepts Y's offer X refuses to pay Y
R20 000 after being towed to town A reasonable fee for tow1ng would have been R5 000, but Y
claims the full R20 000 from X The following defence ra1sed by X will probably be successful

1 The contract is void, because Z unduly influenced him to conclude the contract
2 The contract is void, because Y unduly influenced him to conclude the contract
3 The contract is voidable because of duress by Y
4 The contract is unenforceable because Y exploited the emergency situation X found himself in
5 None of the above

(6) Which statement is CORRECT regarding unlawful contracts?

1 All unlawful contracts are void


2 Some contracts that have been criminalized by statute are not void
3 Some unlawful contracts are unenforceable in terms of the in pan deltcto potior condicio
possidentis rule
4 All unlawful contracts are Voidable
5 All contracts that are prohibited by statute in order to protect the revenue of the state are void

(7) In a contractual context, where the debtors are jointly liable only, and the co-creditors may only
claim performance jointly, this is a case of
1 proportionate liability
2 simple Joint liability
3 in solidum liability
4 Joint and several liability
5 collective Joint liability

(8) Which statement regarding the interpretation of contracts is INCORRECT?

1 The parol evidence rule has an integration and interpretation aspect


2 The distinction between background and surrounding Circumstances is imprecise
3 The primary rule is to give effect to the intention of the party who drafted the contract
4 Where a term is ambiguous it should be interpreted against the party who proposed it
5 Where a term is ambiguous it should be given a meaning that makes it legally effective

(9) Mora ex re relates to

1 a time clause
2 mora debitoris
3 a stipulated fixed time for performance
4 mora creditoris and positive malperformance
5 1, 2 and 3

(10) X and Y agree that should X sell her leather couch, she (X) Will offer to sell it to Y first, before
making an offer to sell the couch to any other person X sells the couch to Z for R 10 000 without first
offering it to Y for sale. Delivery of the couch has not yet taken place Which statement IS CORRECT?

1 X and Y concluded an option contract


2 The contract between X and Z is voidable, because it breaches the contract between X and Y
3 Y has a personal right against X, but Z has a real right against X
4 Both Y and Z only have personal rights against X
5 The personal right of Z enjoys preference above the personal right of Y, because it arises from a
contract of sale

(11) Assume the same facts as in question (1 0). What remedy does Y have against X?

1 An interdict
2 A claim for damages
3 A claim for specific performance of the contract of sale after Y has stepped into Z's shoes with a
unilateral declaration of Intent made to X
4 All the above remed1es
5 None of the above remedies

(12) An option requires that it has to be exercised before a certain date. What is/are the legal
consequence(s) before it is exercised?

1 The substantive offer in the option cannot be validly revoked


2 If the grantee dies, the option can always be exercised
3 If the grantor dies, the option can always be exercised
4 The grantee can always cede his right to exercise the option to a third party
5 All of the above

(13) X sells his car to Y X fraudulently misrepresents the year model of his car to Y during the course
of their negotiations X's misrepresentation caused Y to agree to pay R220 000 for the car. Instead of
the R180 000 she would have offered to pay. Do not apply the Consumer Protection Act to this
question. X Will most probably be able to rescind the contract, because X made a(n)

(a) Innocent misrepresentation


(b) dictum et promissum
(c) fraudulent misrepresentation
1 (a)
2 (c)
3 (a} and (b)
4 (a) and (c)
5 None of the opt1ons

(14) X sells his car to Y X fraudulently misrepresents the year model of his car to Y during the course
of their negotiations X's misrepresentation caused Y to agree to pay R220 000 for the car, instead of
the R180 000 she would have offered to pay. The car is, in fact, worth only R150 000. The car would
have been worth R250 000 if the misrepresentation was true. What amount will Y be able to claim
from X based on fraudulent misrepresentation?

1 R0
2 R30 000
3 R40 000
4 R70 000
5 R100 000

(15) Assume the same facts as in question (14). What amount Will Y be able to claim from X based on
innocent misrepresentation (NOT a dictum et promissum)?

1 R0
2 R30 000
3 R40 000
4 R70 000
5 R100 000

(16) Consider the same set of facts as in question (14) What amount will Y be able to claim from X
based on X's dictum et promissum?

1 R0
2 R30 000
3 R40 000
4 R70 000
5 R100 000

(17) Assume the same facts as in question (14) X guarantees the year model of the car. What amount
will Y be able to claim from X based on X's breach of the guarantee?

1 R0
2 R30 000
3 R40 000
4 R70 000
5 R100 000

( 18) Essentialia are

1 All the terms of a contract apart from the naturalia


2 Terms that identify a contract as belonging to a particular class of contracts
3 Terms automatically imposed by law on the contracting parties unless contracting parties expressly
exclude them
4 All the terms of a contract apart from the incidentalia
5 Material terms and conditions of a contract

(19) Naturalia are

1 All the terms of a contract apart from the essentialia


2 Terms that identify a contract as belonging to a particular class of contracts
3 Terms automatically imposed by law on the contracting parties unless contracting parties
expressly exclude them
4 All the terms of a contract apart from the incidentalia
5 Material terms and conditions of a contract

(20) X and Y conclude a contract on 1 June 2014 m terms of wh1ch X undertakes to deliver 10 dozen
eggs per week to Y, a shop keeper, for a year On 1 August 2014 X Informs Y that she (X) is no longer
able to supply the eggs as she has sold all her chickens in order to pay for her Sister's funeral. This is
a case of

1 prevention of performance
2 supervening impossibility of performance
3 mora creditoris
4 mora debitoris
5 repudiation

(21) On 1 October, X and Y conclude a contract in terms of which X JS to paint the intenor of Y's flat
on 5 October in exchange for payment of R2 000 When X arrives to do the Job on 5 October, he finds
out that Y has left the flat locked up and that there is nobody to let him in This is a case of

1 repudiation
2 prevention of performance
3 mora creditoris
4 mora debutoris
5 none of the above options

(22) Assume the same facts as in question (21) Z, Y's neighbour, sees X's plight and offers X R400 to
paint one of Z's rooms X IS so angry that he declines Z's offer X claims R2 000 from Y in the small
claims court. Which statement is CORRECT?

1 X will not succeed in his claim, because X may not cancel the contract without taking any further
steps
2 X will not succeed in his claim, because X has not performed by painting Y's flat
3 X will succeed in his claim, because the damages X suffered flow naturally and generally from Y's
breach of contract
4 X will not succeed in his claim, because X's damages are special damages
5 X will not succeed in his claim, because X failed to mitigate his damages

(23) X works for Y as a domestic servant Y cedes her right to X's labour to her (Y's) mother, because
her mother has become very frail and needs someone help her clean her flat Y is easy-going, but Y' s
mother is a perfectionist and wants everything spotless DO NOT consider labour law when answering
this question X refuses to work for Y's mother, because

1 she (X) was not consulted before the cession


2 X's obligation to work is of such a personal nature that it cannot be ceded
3 there is a pactum de non cedendo in every employment contract
4 X will have to work much harder for Y's mother
5 X hates Y's mother
(24) X borrows R 10 000 from Y on 1 October 201 0 They agree that X Will pay back the money on 1
December 2010 X gets work overseas and leaves South Africa on 1 September 2010 without paying
back the money X returns to South Africa on 1 January 2014 Y institutes a claim against X for the
repayment of the R1 0 000 on 1 February 2014 Wh1ch statement IS CORRECT?

1 X can validly raise the defence of acquisitive prescription against Y


2 If X raises the defence of prescription it will succeed because the debt prescribes after three years
3 X need not plead prescnpt1on, because the court can raise it of its own accord
4 Prescription started to run on 1 October 2010
5 X's debt has not yet prescribed, because the running of prescription has been interrupted by X
going overseas

(25) X owes R700 to the municipality for electricity used in September it turns out that the
municipality owes X R400 as 1t over-charged him in respect of electricity used in August X therefore
only pays the municipality R300 This is a case of

1 Novation
2 Release
3 Merger
4 Set-off
5 Compromise

QUESTION 1
Discuss specific performance as a remedy for breach of contract. [10]

Answer
A claim for specific performance is the primary remedy for breach of contract. It may be a claim for
the payment of a sum of money, a claim for the performance of some positive act other the payment
of money, a claim for the performance of some positive act other than payment of money or a claim
to enforce a negative obligation.
- Specific performance will not be granted unless the innocent party is ready to perform and is
subjectively and objectively possible for the defendant.
- The courts have exercised an equitable discretion to refuse a claim for specific performance usually
on the grounds of impossibility, undue hardship or in claims for the enforcement of personal
services.
- Enforcement of an order for specific performance is in accordance with the ordinary rules of
procedure.
1) When specific performance has become impossible. Here the maxim ``the law does not force one
to do the impossible'' is applicable. A distinction should be drawn between impossibility of
performance which terminates the contract (Peters Flamman and Co v Kokstad Municipality 1919 AD
427; and the impossibility which prevents the issue of an order for specific performance. In the
Peters case, vis maior prevented the debtor from discharging the contract, and for that reason the
contract was terminated. Suppose, however, that A promises to deliver a cow to me and
subsequently slaughters the cow. He remains bound, but specific performance will not be ordered,
because it has become impossible (Farmer's Co-operative Society v Berry 1912 AD 343; Wireohms SA
(Pty) Ltd v Greenblatt 1959 (3) SA 909 (C)).
(2) Where it is impossible for the court to control specific performance. The court may send the
sheriff or the messenger of the court to fetch the horse from A and hand it over to me. Is the court
able to supervise A when he is ordered to rebuild my house, as he undertook to do? The court will
not order specific performance in a case where it is impossible to control the defendant (Barker v
Beckett and Co 1911 TPD 151 which was concerned with repairs to a house). In such a case the court
will order specific performance, but will add an alternative in case it is not executed, for example
repair the house or pay so much damages.
(3) Undue hardship. Specific performance will not be granted where it would operate harshly on the
defendant or where the agreement giving rise to the claim is unreasonable or where specific
performance would produce injustice or would be inequitable under all the circumstances (Haynes v
King Williams Town Municipality 1951 (2) SA 371 (A)).
(4) Inability to fulfill obligations. It is self-evident that specific performance in my favor will not be
ordered if I am not in a position to fulfill my own obligations.
(5) Where it concerns the freedom of the individual. All the textbooks state that promises to marry,
and master and servant contracts, etcetera are not specifically enforceable.

QUESTION 2
Discuss the requirements for a valid acceptance of an offer. [10]

Answer
1. The acceptance must be unconditional and unequivocal.
It stands to reason that consent is possible only where the whole offer and nothing more or less is
accepted. When the acceptance contains conditions or reservations, it is no acceptance but in fact a
counter-offer which the original offeror may accept or reject in turn (Boerne v Harris 1949 (1) SA 793
(A)).
Where the purported acceptance is equivocal (ie not positive and unambiguous) there is likewise no
valid acceptance (Van Jaarsveld v Ackerman 1975 (2) SA 753 (A)).
2. The offer must be accepted by the person to whom it was addressed
The offer cannot be accepted by anyone but the person to whom it was made (Bird v Summerville
1961 (3) SA 194 A)
However an offer addressed to the public in general; any member of the public may accept it.
3. The acceptance must be a reaction to the offer – a person cannot accept an offer which he is not
aware
This point is of special importance in reward cases as is aptly illustrated by the case of Bloom v
American Swiss Watch Co 1915 AD
4. The acceptance must comply with any formalities set by law or by the offeror
Where formalities are prescribed by law the acceptance must comply with such formalities for a valid
contract to arise (Brand v Spies 1960 (4) SA 14 (E)).
The acceptance may likewise require that acceptance must take a particular form, eg by requiring
that it must be communicated in a certain way.

QUESTION 3
John, a racehorse owner, advertises for sale the horse Fire for R1.5 million. In the advertisement it is
stated that Fire is an offspring of the legendary July winner, Lightning Peter is a horse breeder who
specifically wishes to introduce the bloodline of lightning into his stud. He agrees orally with John to
buy the Fire for R1.5 million. Latter in order to meet the requirements of the horse breeders’
association, John has written a contract drawn up which Peter signs without reading. The contract
makes no mention of Fire’s ancestry, but does contain a clause exempting John’s liability for any
representations made during negotiations or in the contract Peter’s attention is not drawn to these
facts. A month later Peter finds out that Fire is in fact not an offspring of Lightning, although at the
time of conclusion of the contract John genuinely and without any fault on his part believed that to
be the case. Advise Peter on whether the contract of sale is valid. Refer to Du Toit v Atkinsons Motors
Bkp 1985 (2) SA 893 (A), Sonap Petroleum (SA) Ltd (SA) (Pty) Ltd v Pappadogianis1992 (3) SA 234, and
any other relevant case law in your answer. Do not apply the Consumer Protection Act to this
question. [15]

Answer
The question is associated with whether Peter and John had reached actual consensus or ostensible
consensus. Peter will not be bound by the contract of sale if the valid requirements are absent.
The first step is to determine whether agreement (consensus ad idem) as a contractual basis exists
between the parties, as required in terms of the will theory. Consensus has three elements:
1. The parties must seriously intend to contract
2. The parties must be of one mind as to the material aspects of the proposed agreement (the terms
and the identity of the parties to it)
3. The parties must be conscious of the fact that their minds have met.
This type of mistake can be illustrated with a number of cases:
Du Toit v Atkinsons Motors Bkp
Facts of the Case
 A party (A) signed a contract without reading it since he believed that its terms coincided with the
content of the other party’s (B) advertisement.
 However - the contract contained a further material term of which A was unaware (Vis - a term
excluding the Respondent from liability for misrepresentation.
Finding of the court
 The court found [once again] that the mistake related to an aspect of performance and was thus
material.
 The court found (as the term was material) the contract to be void.
 The court found that B (in this case) had not been misled by A into believing that he (A) had agreed
to the term because B had not drawn A’s attention to that relevant material term.
 The court found (as the term was material) the contract to be void.
In George v Fairmead, the appellant signed a hotel register without reading it. The register contained
a term excluding the respondent from liability for certain acts. The appellant was unaware of this
term and his mistake related to a term that he believed would not be in the contract and as such was
material because it related to an aspect of performance.
In Allen v Sixteen Stirling Investments, the plaintiff believed he was purchasing the erf pointed out to
him by the seller’s agent, while the written contract that he signed indicated the correct erf, which
was a completely different property. His mistake related to performance and was material.
However, the matter does not end here. A party may be held contractually liable on the basis of a
supplementary ground for liability, namely the reliance theory.
In this regard, the direct or indirect approach to the reliance theory may be considered.
DIRECT APPROACH:
With reference to the direct approach, contractual liability is based on the reasonable reliance that
consensus has been reached, which the one contractant (the contract denier) creates in the mind of
the other contractant (the contract enforcer).
According to the Sonap case, the direct reliance approach entails a threefold enquiry:
1. Was there a misrepresentation regarding one party’s intention?
2. Who made the misrepresentation?
3. Was the other party misled by the misrepresentation, and if so, would a reasonable person have
been misled?
Peter made a misrepresentation by signing the contract, that his intention is the same as that
expressed in the contract.
John made a misrepresentation to Peter by placing an advert in which he alleged that Fire was an
offspring of the legendary winner Lightning of which it was otherwise. Apart from that John’s
misrepresentation also amounted to a fraudulent nature in which he presented facts which did not
exist at all. Peter there did not create a reasonable reliance that he wished to be bound by the
contract he signed. Since the mistake so as to the quality of the object of sale is material, the
contract of sale of the horse Fire by John to Peter is void. Peter may claim back his money and
damages from John.

QUESTION 4
John is the owner of a vacant stand and he concludes a contract with a builder, Peter, in terms of
which Peter must build a house on the stand according to a plan supplied by John. According to the
plan the house will be 200m2 in extent. In terms of the contract, Peter must complete the house by 1
July and John takes occupation of it, but it transpires that the house is 20m2 smaller than the
specifications required by the plan. It would cost R200 000 to have the house altered in order to
comply with the specifications. John refuses to pay Peter. Advise Peter fully. [15]

Answer
This question deals with a breach of contract through the form of positive malperformance by Peter.
Positive malperformance is when the debtor performs but in a defective or incomplete manner like
how Peter deviated from the plan and specifications that the house must be 200m2.
The remedies available to John are either rescission or fulfillment of the contract.
In rescission, the creditor will cancel the contract if there is a cancellation clause in the contract
providing for the creditor to cancel for the particular type of malperformance that has occurred,
even if the breach is not a material one. The creditor must give notice to the debtor informing him of
the intention to rescind if the breach cannot be rectified. If there is no cancellation clause, the
creditor may only cancel the contract the malperformance is of a serious nature that is failure to
perform a vital part of the obligations. The right to rescind is excluded where there has been
substantial performance of the contract. The test that can be applied by the court is objective one
that balances the competing interests of the parties in a manner that is fair and reasonable in the
circumstances of the case.
In the remedies aiming at the fulfillment, the creditor abides by the contract due to the facts that the
breach is not serious or simply because he prefers not to rescind. The creditor may accept the
defective performance as partial performance of the contractual obligation and claim as fulfillment
of the contract damages that are difference in value between proper performance and the
performance actually rendered. Apart from that the creditor may reject the defective performance
and demand either specific performance or damages in lieu of performance.
Damages that are awarded in lieu of complete performance are called surrogate damages to
distinguish them from the damages awarded in respect of other losses arising out of the breach
(consequential damages)
In the given case, there is no cancellation clause stated and the debtor, Peter had done substantial
performance of the contract which is also not a serious breach of the contract. This means that the
creditor, John may not rescind this contract under the given circumstances. John will however be left
with the remedies aimed at fulfillment of the contract in which he has two choices to choose from,
that is:
- accepting the defect performance together with a claim of damages
- Reject the defect performance and demand either specific performance or damages in lieu of
performance.

PVL3702_201_2016

Question
S, who lives in Upington, sends P, who lives in Grahamstown, a letter by post in which she offers to
sell him (P) her (S’s) motorcycle, a collectors piece, for R100 000. She states in her letter that her
offer will expire on 1 February at 24h00. P phones S on 1 February at 19h00, but S and her husband
are out for the evening. P leaves a message on S’s voice mailbox accepting S’s offer. S only listens to
P’s message on 2 February. P tenders payment of R100 000 but S refuses to accept payment. Did a
valid contract of sale arise between S and P? Advise P. Substantiate your answer. (10)

Answer
In the assignment you were inter alia referred to study unit 10 in answering the assignment. This was
a mistake and it should have been study unit 7. We did not notice this mistake until it was too late to
correct it. Some students failed to notice this mistake and focused on the subjective approach to
error of the courts, but others noticed this mistake and correctly discussed the time and place of
conclusion of the contract. Both approaches were marked as correct.
First possible answer
This is the answer we marked as correct if you failed to realise that we made a mistake.
Identifying the problem
The question is whether P accepted S's offer in time and S and P thus reached consensus. S did not
know that her offer was accepted by P on the date the offer expired (1 February) as she only became
aware of the acceptance on the next day. See Hutchison and Pretorius (eds) The law of Contract in
South Africa Oxford University Press Southern Africa 2012 54 56-60.
Discussing the relevant law applicable to the problem AND applying the law to the facts of the
problem
S may only refuse payment of the purchase price if a valid contract did not arise. In our problem S
made an offer to sell her motorcycle, but stated in her offer that her offer will expire on 1 February at
24h00. An offer which has a time limit for acceptance lapses automatically if it is not accepted within
the prescribed period (Hutchison and Pretorius Contract 54). The question is thus whether P's
acceptance of the offer which was recorded in the voicemail was a valid and timeous acceptance.
We will have to apply the subjective approach of the courts (the will theory) to the facts, because we
do not have an objective appearance of agreement (Hutchison and Pretorius Contract 106).
Consensus has three elements (Hutchison and Pretorius Contract 14 85): the parties must seriously
intend to contract, be of one mind as to the material aspects of the proposed agreement (the terms
and the identity of the parties to it), and be conscious of the fact that their minds have met. The
latter was absent on the date of expiry of the offer, because S only came to know of the acceptance
on the next day when she listened to her voicemail. The offer thus expired after effluxion of the
prescribed time (Hutchison and Pretorius Contract 54). The parties failed to reach subjective
consensus.
The next step is to apply estoppel and the doctrine of quasi mutual assent (Hutchison and Pretorius
Contract 92-97 103-104). Estoppel cannot be applied, because S misrepresented no facts from which
a reasonable inference of consensus could be made (Hutchison and Pretorius Contract 93-94). One
surely cannot draw such inference from the fact that S made an offer which would expire at a certain
specified time.
The doctrine of quasi mutual assent determines that contractual liability can arise if the one party
induces a reasonable belief on the part of the other party that the former party had agreed to the
contract in question (Hutchison and Pretorius Contract 95). In other words, did S create a reasonable
belief in P that she had agreed to the contract of sale? Again the answer is no, because she only
made an offer which would expire at a certain specified time. P could not reasonably deduce from
this that S agreed to the contract of sale even if the offer expired. The same result will be achieved if
the threefold test of Sonap Petroleum (SA) (Pty) Ltd v Pappadogianus 1992 (3) SA 234 (A) is applied
to our problem. The answer to the first test is negative as S did not misrepresent her intention to P
that she wishes to be bound to a contract even if the offer expires on the specified time.
The giving of appropriate advice
The written contract of sale is invalid, because actual and apparent consensus is lacking.
Total: [10]

Second possible answer


This is the answer we marked as correct if you realised that we made a mistake.
Identifying the problem
The question is whether P accepted S's offer in time and S and P thus reached consensus. To be more
specific, the question is relates to the legal rules pertaining to when and where the acceptance takes
effect (Hutchison and Pretorius (eds) The law of Contract in South Africa Oxford University Press
Southern Africa 2012 56-60).
Discussing the relevant law applicable to the problem AND applying the law to the facts of the
problem
In our problem S made an offer to sell her motorcycle by post, but stated in her offer that her offer
will expire on 1 February at 24h00. An offer which has a time limit for acceptance lapses
automatically if it is not accepted within the prescribed period (Hutchison and Pretorius Contract 54).
The question is thus whether P validly accepted the offer before 24h00 on 1 February. If P did so,
acceptance will take effect and a contract of sale will arise.
P's acceptance of the offer was a voice message left in the voice mailbox of S before 24h00 on 1
February. S only listened to the message on 2 February and thus did not know that her offer was
accepted at the time of termination of the offer.
The general rule is that a contract only comes into being when the offeror knows that his/her offer
has been accepted (Hutchison and Pretorius Contract 57). The theory which explains this rule is the
information theory. This general rule gives effect to one of the requirements of subjective consensus
(Hutchison and Pretorius Contract 13-14) which is the primary basis of contractual liability (Hutchison
and Pretorius Contract 19). If we apply the general rule, it is clear the offer expired before
acceptance.
But this is not the end of our enquiry because there are exceptions to the general rule (Hutchison
and Pretorius Contract 57-59). The offeror as dominus may dispense with the need of acceptance
being communicated to him /her or can indicate in the offer that the contract will come into being at
an earlier stage. This may be expressly indicated in the offer itself, but this is not the case in our
problem. It may also be implied from all the circumstances, the language of offer itself and the
nature of the contract. Where an offer is made through the post it is assumed (a legal fiction thus) if
certain the requirements are met that the offeror authorised acceptance by post as well as indicated
that the contract is concluded as soon as the acceptance is posted (Kergeulan Sealing and Whaling
Co v Commissioner of Inland Revenue 1939 AD 487). The expedition theory explains the postal rule.
All four the requirements for the application of the postal rule are present in our problem. The offer
firstly was made by post. Secondly, the fact that S's offer reached P indicates that the postal service
was operating normally (Bal v Van Staden 1902 TS 128). Thirdly, the contract that S envisaged in her
offer was a contract of sale which is a commercial contract (S v Henckert 1981 (3) SA 445 (A) 451).
Lastly, S did not expressly or tacitly indicate a contrary intention in her offer (A-Z Bazaars (Pty) v
Minister of Agriculture 1975 (3) SA 468 (A) 476).
P did not accept in the authorised manner by post. The leaving of a message in the voice mailbox of S
cannot be seen as an acceptance by post and the exception created by the offer by post thus cannot
apply.
As a consequence, the general rule must out of necessity apply: S had to hear that her offer was
accepted before the time of termination of her offer. S did not and her offer automatically lapsed and
there was no offer left to be accepted when she listened to her voicemail on 2 February and heard
that her offer was accepted.
The Electronic Communications and Transactions Act 25 of 2002 cannot be applicable to our
problem. Section 22(2) is only applicable to an agreement concluded between parties by means of
data messages. The contract of sale in our problem has not thus been concluded. The message in the
voicemail is a data message but the offer by post is certainly not.
The giving of appropriate advice
No contract of sale came into being because P failed to accept S's offer timeously. S may thus validly
refuse to accept P's performance in terms of an invalid contract.
General comments on the second possible answer
Some students experienced difficulty in solving this problem. We have to think logically and
systematically to solve this problem. We must also apply the following legal knowledge which we
should have acquired by now.
We should firstly understand that the law often regulates a situation in a certain way and then
creates exceptions to this the general rule. If one of the exceptions does not apply to a given set of
facts, the general rule applies.
We have to secondly understand that theories try to explain what the law does and that they are not
the law itself. In this problem we have different theories which explain when an acceptance has
effect and a contract comes into being. So for instance, an offer by post does not mean that the
expedition theory is authorised. No. The offeror is assumed to empower the addressee to accept the
offer by post and to provide that the contract comes into being as soon as the letter of acceptance is
posted. When we analyse this rule we can say that expedition theory applies. We must not confuse
the law and our explanation of the law. Some students incorrectly thought that the offer by post
authorised the expedition theory and then applied this theory to the acceptance left in the
voicemail. If the offer was accepted by letter but sent by courier to the offeror in our problem, we
cannot argue that the expedition theory was authorised and that the contract was concluded as soon
as the letter was given to the courier.
Thirdly, legislation often changes the common law, but only insofar as the Act is applicable. We must
thus determine whether an Act is applicable to a specific set of facts. If it is not, the common law
applies. So for instance section 22(2) of the Electronic Communications and Transactions Act 25 of
2002 is only applicable to agreements concluded between parties by means of data messages.
Total: [10]

Question 1
Z walks into a shop, and puts R10 on the counter and points to a packet of sweets. X (the owner of
the shop) takes the money and hands over the sweets to Z. Which statement is CORRECT?

1 There is an oral offer, and acceptance by conduct.


2 There is an express offer, and an oral acceptance.
3 There is both an offer, and acceptance by conduct.
4 There is an offer by conduct, and an express oral acceptance.
5 There is a firm oral offer, and an unqualified acceptance.
Discussion
Option 3 is correct. All the other options are incorrect because there is neither an oral offer, nor an
oral acceptance. No words were exchanged between the parties. The placing of the R10 on the
counter and pointing to the sweets constitutes an offer by conduct. Z taking the money amounts to
an acceptance by conduct. See Hutchison and Pretorius (eds) The law of Contract in South Africa
Oxford University Press Southern Africa 2012 46.

Question 2
Carol, an owner of an exclusive bicycle shop advertised a special limited edition bicycle for sale, and
invited the public to make offers for the bicycle. Jane and Portia were among many other people who
submitted written offers for the bicycle. Jane’s offer was for R150 000, and Portia’s offer was for R160
000. Although Carol intended to accept Portia’s offer, she erroneously accepted Jane’s offer. Carol’s
mistake is
(a) not material.
(b) an error in persona.
(c) an error in motive.
(d) material.

1 (a).
2 (b).
3 (c).
4 (b) and (d).
5 (a), (b) and (c).

Discussion
Options (b) and (d) are correct. Option (b) is correct because there is a mistake relating to the
identity of the parties with whom Carol intended to contract with (an error in persona) (Hutchison
and Pretorius Contract 88). Carol’s mistake was material because she inadvertently accepted the
offer of one party (Jane), whereas she intended to accept the offer of a completely different party
(Portia). The facts in this problem are similar to the facts in National and Overseas Distributors
Corporation (Pty) Ltd v Potato Board 1958 (2) SA 473 (A), where the Appellate Division found that the
mistake was material. Therefore option (b) is correct and option (a) is incorrect.
Option (c) is incorrect as the error did not relate to Carol's reason for concluding the contract
(Hutchison and Pretorius Contract 87).

Question 3
Assume the same facts as in question (2) and assume that Carol’s error was material. Which
statement(s) is/are CORRECT?
(a) Carol’s error was a iustus error.
(b) Carol’s error was not a iustus error.
(c) Carol misrepresented her intention to Jane by accepting Jane’s offer.
(d) Carol by accepting Jane’s offer, led Jane to reasonably believe that they have reached consensus.
1 (a).
2 (b).
3 (c) and (d).
4 (b) and (c).
5 (b), (c) and (d).

Discussion
The requirements of both iustus error and the doctrine of quasi mutual assent are relevant when
answering this question.
A mistake is a iustus error, if it is both material and reasonable (Hutchison and Pretorius Contract 99).
It has already been established from the previous answer that the error was material. The issue now
is whether the mistake was also reasonable. In National and Overseas Distributors Corporation (Pty)
Ltd v Potato Board 1958 (2) SA 473 (A), the court ruled under similar circumstances that the mistake
by the contract denier was not reasonable (Hutchison and Pretorius Contract 98-99). The mistake
furthermore does not fall into one of the recognised categories of reasonable mistakes (Hutchison
and Pretorius Contract 100-103). Therefore option (b) is correct.
Option (c) and (d) relate to requirements of the doctrine of quasi-mutual assent (Hutchison and
Pretorius Contract 95-97). This doctrine requires that the one party reasonably believes that the
other party had agreed to enter into the contract and that this belief must have been caused by the
latter party. Carol made a misrepresentation to Jane that she wanted to conclude a contract with
Jane by accepting Jane’s offer and this led Jane to reasonably believe that consensus had been
reached between the parties. Therefore both options (c) and (d) are correct. See the application of
the doctrine of quasi-mutual assent to the facts of National and Overseas Distributors Corporation
(Pty) Ltd v Potato Board 1958 (2) SA 473 (A) by Hutchison and Pretorius Contract 106.

Question 4
Assume the same facts as in question (2). Which case has similar facts?

1 National Overseas Distributors Corporation (Pty) Ltd v Potato Board 1958 (2) SA 473 (A).
2 Brink v Humphries & Jewell (Pty) Ltd 2005 (2) SA 419 (SCA).
3 George v Fairmead Hotel (Pty) Ltd 1958 (2) SA 465 (A).
4 Du Toit v Atkinson’s Motors Bpk 1985 (2) SA 893 (A).
5 Allen v Sixteen Stirling Investments (Pty) Ltd 1974 (4) SA 164 (D). (1)

Discussion
See Hutchison and Pretorius Contract 98.

Question 5
Gary points a loaded gun at Pete, and orders him to sign a written contract. Gary explains that the
document is for the sale of Pete’s car to him (Gary), at a price of R50 000 which is far below the
market value of the car. Pete, fearing for his life, signs the document. To establish a cause of action,
which of the following is NOT necessary for Pete to prove?

1 That Pete had a reasonable fear.


2 That the threat weakened Pete’s power of resistance and rendered his will compliant.
3 That there was a threat of an imminent evil.
4 That the threat was contra bonos mores.
5 That the pressure Gary used caused damage.

Discussion
The requirements for duress (Hutchison and Pretorius Contract 136-140) are relevant to this
question, as it appears that Pete did not freely conclude the contract, but through fear for his life.
Options 1, 3, 4 and 5 are all requirements that must be present to prove a cause of action based on
duress (Hutchison and Pretorius Contract 137). Option 2 is not a requirement to prove duress.
Instead it has a similar wording to a requirement, which is necessary to prove undue influence
(Hutchison and Pretorius Contract 141-142).

Question 6
X has a watch that Y likes. X offers to sell her watch to Y for R1 000, and X and Y agree that X’s offer
will be open for acceptance until 1 June. This is a case of

1 a pre-emption formed unilaterally.


2 an option formed bilaterally.
3 an option formed unilaterally.
4 neither an option, nor a pre-emption.
5 a pre-emption formed bilaterally. (1)

Discussion
From the facts of this problem it is clear that the parties have entered into an agreement to hold the
offer open. All contracts are bilateral juristic acts (Hutchison and Pretorius Contract 6).
An option is a contract to keep an offer open for acceptance for a certain period of time (Eiselen GTS
et al Law of contract. Only study guide for PVL3702 (University of South Africa 2012) 26). Clearly
there is an agreement between the parties to keep X’s offer open for acceptance until 1 June.
Therefore option 2 is correct.
In a pre-emption agreement the prospective seller undertakes to give a prospective buyer preference
in the event of deciding to sell the property (Eiselen et al Study Guide 31). Clearly X and Y did not
conclude such an agreement relating to the watch.

Question 7
X has a watch that Y likes. X offers to sell her watch to Y for R1 000, and Y accepts this offer. X and Y
agree that they will reduce their contract to writing and that they will both sign it (“the writing
clause”). The parties reduced their contract to writing, but failed to sign it. Which statement(s) is/are
CORRECT?
1 The law presumes that the X and Y intended the writing clause to facilitate proof of the terms of
their contract, because there is doubt about what the intended purpose of X and Y was with the
inclusion of the writing clause in their contract.
2 There is no binding contract because the formalities created by X and Y were not complied with.
3 The writing clause also constitutes a non-variation clause.
4 A binding contract was concluded even though X and Y only partially complied with the writing
clause.
5 Option 1 and 4.

Discussion
The question relates formalities stipulated by the parties (Hutchison and Pretorius Contract 164-165).
X and Y partially failed to comply with their formalities. The purpose of the parties with regard to
their formalities determines the consequences of their failure to comply. It is uncertain what the
parties intended in our problem and is such a case the law presumes that their intention was merely
to facilitate proof of the terms of their contract. This means that their oral contract was valid.
Therefore option 1 is correct and option 2 is incorrect. Option 4 is also correct, because it does not
matter whether the writing clause was not complied with partially, or in its entirety.
Option 3 is incorrect. The writing clause is not a non-variation clause, because it does not regulate
the variation of the contract between X and Y. For a discussion on non-variation clauses see
Hutchison and Pretorius Contract para 6.3.2.

Question 8
S negligently informs P that there are 1 000 fruit trees on his (S’s) farm, as a result of which P buys
the farm for R8 000 000. P pays the R8 000 000 and the farm is registered in her (P’s) name. It is later
found that there are only 800 fruit trees on the farm. P would not have bought the farm had she
known the truth about the number of fruit trees, but decides nevertheless to uphold the contract.
The value of the missing 200 fruit trees is R500 000. The market value of the farm is R7 800 000. The
farm would have been worth R8 400 000 if it had 1 000 fruit trees on it. What amount will P be able
to claim from S based on negligent misrepresentation? Do not apply the Consumer Protection Act to
this question.

1 R0.
2 R200 000.
3 R400 000.
4 R500 000.
5 R600 000.

Discussion
Negligent misrepresentation is a delict and damages may be claimed for negligent misrepresentation
in the same manner as damages for fraud (Hutchison and Pretorius Contract 131). The measure for
damages for negligent misrepresentation is the usual delictual measure: the victim is entitled to be
put in the financial position he/she would have been in had the delict (misrepresentation) not been
committed against him/her (Hutchison and Pretorius Contract 126). P’s damages will be calculated
based on dolus dans (Hutchison and Pretorius Contract 127), because she would not have contracted
at all had she known the truth about the number of trees. P should thus be placed in the financial
position that she would have been in had she not contracted at all. P would then have had the
purchase price of R8 000 000. P, however, upholds the contract, and thus has a farm which is only
worth R7 800 000. She thus suffers a loss of R200 000: R8 000 000 (the price) - R7 800 000 (the value
of the farm = R200 000. Therefore option 2 is correct.

Question 9
P’s claim against S in question (8) is based on

1 delict.
2 breach of contract.
3 unjustified enrichment.
4 monetary compensation.
5 all the above

Discussion
Negligent misrepresentation is a delict (Hutchison and Pretorius Contract 129-131).

Question 10
Incidentalia are:

1 All the terms of a contract apart from the naturalia and essentialia.
2 Terms that identify a contract as belonging to a particular class of contracts.
3 Terms automatically imposed by law on the contracting parties unless contracting parties expressly
exclude them.
4 All the terms of a contract apart from the naturalia.
5 All the material terms and conditions of a contract. (1)

Discussion
See the description of incidentialia in Hutchison and Pretorius Contract 238.

PVL3702_201_2017

Question
X is on her way from work and sees a white bull terrier bitch hiding in a doorway. Being an animal
lover, she takes the dog home with her. The next day, she sees the following advertisement in the
newspaper:
Lost in Johannesburg City Centre on 27 May. Pedigree white bull terrier bitch with black patch over
left eye. Answers to the name of Beauty. Reward of R1 000 for information leading to safe return. Tel
011 555 5555.
She realises that the dog she found matches the description given. She calls the advertiser who
rushes over to be joyfully united with Beauty. In his joy, Beauty’s owner, Y, seems to forget about the
reward and X wishes to claim it from him. Will she be successful? Substantiate your answer. (10)

Answer
Identifying the problem
X will only be able to claim the reward if a valid contract arose between X and Y. A valid contract will
arise if there was a valid acceptance of a valid offer.
Discussing the relevant law applicable to the problem AND applying the law to the facts of the
problem
Y’s advertisement complies with the requirements for a valid offer:
1 Y's offer was firm. The offer must be made with the intention that its acceptance will result in a
binding contract. Although an advertisement is usually only an invitation to do business (Crawley v
Rex 1909 TS 1105), a promise of reward does constitute a firm offer (Bloom v American Swiss Watch
Co 1915 AD 100). Y’s offer was thus a firm offer and not a tentative statement with a possible
agreement in mind.
2 Y's offer was complete. An offer must include all material terms of the proposed agreement and
there cannot be additional matters that still have to be discussed before the agreement can take
effect. Y’s offer contained all the material terms: providing information leading to the safe return of
Beauty in return for the reward of R1 000. The offer will be accepted by providing the required
information.
3 Y's offer was clear and certain. An offer is sufficiently clear and certain if the mere answer of ‘yes’
by the addressee brings a valid contract into existence. If the offer is unclear and cannot capture
what the offeror has in mind, no acceptance of the offer can create a binding contract. The offer by Y
was clear: a reward of R1000 was offered to the any member public for the giving of information
leading to the safe return of Beauty. The giving of the required information will also constitute
acceptance of the offer. Furthermore, it is clear that an offer such as a promise of reward can be
validly directed at undefined persons.
Providing the information by X was a valid acceptance of Y’s offer:
1 X's acceptance was unqualified, because X accepted the offer as it was by giving the required
information. The acceptance is complete and unequivocal, where the entire offer and nothing
additional or less is accepted.
2 X's acceptance was by the person to whom the offer was made because X was a member of the
public. Only the offeree can validly accept the offer (Bird v Sumerville 1961 (3) SA 194 (A). The
general rule is that an offer should be addressed to a specific person, but an offer may also validly be
directed to the public, such as an offer of a reward.
3 X's acceptance was a conscious response to the offer, because X was aware of the reward and
provided Y with the required information in response to it. In Bloom v American Swiss Watch Co
1915 AD 100, the plaintiff provided the information required in the offer of reward without knowing
of the reward. The court held that no contract came into being.
4 X accepted the offer in the form prescribed by the offeror by providing Y with the required
information which led to the dog's safe return to Y. The offeror is allowed to prescribe any method of
acceptance he or she sees fit (Carlill v Carbolic Smoke Ball Co (1893) 1 QB 256).
X and Y thus concluded a valid contract, because both the offer and acceptance was valid.
The giving of appropriate advice
X will be successful in claiming the reward from Y.
See Hutchison and Pretorius (eds) The law of Contract in South Africa 2nd ed (Oxford University Press
Southern Africa 2012) 48–9, 50-1, 52, 55-6.

Question 1
A real agreement is an agreement
1 only creating obligations.
2 whereby a right is transferred.
3 only extinguishing a debt.
4 an example of which is release or discharge.
5 that cannot entail the transfer of ownership of property

Question 2
Portia and Lucas are engaged and living together. They want to purchase Jerome’s car. Jerome offers
to sell his car to Lucas for R100 000. Lucas and Portia accept Jerome’s offer. Which statement is
CORRECT?

1 Only Lucas could accept the offer.


2 Either Lucas or Portia could accept the offer as they are engaged.
3 Since the offer was only made to Lucas, the acceptance from both Portia and Lucas will never
constitute a counter-offer.
4 A valid contract has been concluded, because Lucas is one of the parties that accepted the offer.
5 None of the above statements.
Question 3
X orally offers to sell her painting to Z for R20 000 on 1 April. In terms of the offer X stipulates that
her offer will lapse on 30 April. On 15 April X notifies Z orally that she (X) revokes her offer. On 25
April Z notifies X orally that he (Z) accepts the offer. Which statement reflects the CORRECT legal
position?

1 A right of pre-emption exists.


2 The revocation by X is valid, because an offer can be revoked where an option was given.
3 An option exists because X offered to keep her offer open until 30 April, which was accepted on 25
April.
4 The revocation by X is valid because no option contract was concluded.
5 The option is not valid because in Brandt v Spies 1960 (4) SA 14 (E) it was held that such an option
must be in writing.

Question 4
An innocent misrepresentation renders a contract

1 void.
2 void without a claim for damages.
3 voidable.
4 unenforceable.
5 neither void nor voidable, but it does give rise to a claim for damages.

Question 5
Which statement is/are CORRECT?

1 Commercial bribery amounts to an abuse of circumstances.


2 Commercial bribery amounts to misrepresentation.
3 Commercial bribery can lead to a claim for the setting aside of the contract and restitution.
4 Option 1 and 3.
5 Option 2 and 3.

Question 6
X, who lives in Cape Town, writes a letter to Y, who lives in Johannesburg. In the letter, she offers to
sell her house to Y for R900 000. She posts the letter in Stellenbosch while visiting her daughter. A
week later Y receives the letter, reads it, and immediately phones X on her landline. Y requests X to
keep her offer open for a week and she undertakes to do. A day before the option expires, X phones
Y and revokes her offer to sell her house. Y immediately posts a letter of acceptance in which he
exercises the option. X only receives this letter a week later. Which statement is CORRECT?

1 The substantive offer to sell the house does not contain all the essentialia of a contract of sale.
2 The option contract is valid.
3 The option contract was exercised too late as Y’s letter of acceptance only reached X after the
option lapsed.
4 According to the courts, X can validly revoke her offer to sell her house before Y exercises the
option.
5 The option contract is concluded in Cape Town.

Question 7
John agrees with Michael, that Michael will paint John's holiday home at the coast for R30 000.
Unbeknown to either of them the house had been destroyed in a storm the previous day. This is a
case of

1 mutual mistake.
2 an obligation subject to a suspensive time clause.
3 impossibility of performance.
4 an obligation subject to a resolutive condition.
5 an obligation subject to a suspensive condition

Question 8
C, a builder, and O agree that C will build a house on O’s stand for R800 000, payable on completion
of the house. Which statement is CORRECT?

1 The obligation to pay R800 000 is an alternative obligation.


2 The obligation to pay R800 000 is a facultative obligation.
3 The obligation to pay R800 000 is a generic obligation.
4 The contract is a reciprocal contract.
5 Option 3 and 4.

Question 9
X buys a stand from Y for R300 000 after being told by Y that business rights existed on the stand.
Later it emerges that no such rights exist. Y, however, believed in good faith that business rights did
exist on the stand. The contract contains a term which excludes Y’s liability for “any
misrepresentation”. X did not know of the existence of this clause because she did not read the
contract of sale before signing it in Y’s presence. The contract of sale complies with the formalities
prescribed for contracts of sale of land. What type(s) of mistake does X make?
1 Mistake regarding an attribute or characteristic of the subject matter of the contract if the
approach of Trollip v Jordaan 1961 (1) SA 238 (A) is followed.
2 Mistake regarding the legal consequences of the contract.
3 Mistake regarding the identity of the subject matter of the contract if the approach of the court in
Spenmac (Pty) Ltd v Tatrim CC 2015 (3) SA 46 (SCA) is followed.
4 Mistake regarding the nature of the contract.
5 Option 1, 2 and 3.

Question 10
Assume the same facts as in question 9. This question deals with X's mistake regarding the presence
of the exemption clause in the contract. Which statement(s) is/are CORRECT?

1 X made a misrepresentation regarding her intention to be bound to the contract of sale.


2 X's misrepresentation regarding her intention to be bound to the contract of sale probably did not
mislead Y and a reasonable person would also not have been misled thereby.
3 X's misrepresentation regarding her intention to be bound to the contract of sale misled Y and a
reasonable person would also have been misled thereby.
4 Option 1 and 2.
5 Option 1 and 3.

Question 1
X, an organiser of art exhibitions, contracted with Y for an exhibition to be held on 24 to 27 July.
These dates were the only dates mentioned during the negotiations. After having been pressurized
by X, Y hurriedly signed the standard form contract without reading it. The contract contained a
clause permitting X to change the dates of the exhibition unilaterally. Thereafter X changed the dates.
X had no reason to believe that Y would have signed the contract if he had known of the term. Y
averred that the contract was void. Will Y succeed in his attempt to have the contract set aside?
Substantiate your answer and refer to relevant case law. Apply the direct reliance approach of the
courts in answering this question. Do not apply the Consumer Protection Act to this question. (10)

Answer
Identifying the problem
The facts seemingly indicate that X and Y have not reached consensus based on the will theory. If so,
it is necessary to determine if Y may be held bound to a contract with X, based on the reliance
theory, or whether Y will escape liability. Only the direct approach to the reliance theory will be
considered.
Discussing the relevant law applicable to the problem, referring to the relevant case law, AND
applying the law to the facts of the problem
The direct reliance approach can only be applied after it has been determined that Y acted under a
material mistake. It must thus be determined whether agreement (consensus ad idem) as a
contractual basis exists between the parties, as required in terms of the will theory.
The first step is to determine whether agreement (consensus ad idem) as a contractual basis exists
between the parties, as required in terms of the will theory. Consensus has three elements
(Hutchison and Pretorius (eds) The law of Contract in South Africa (Oxford University Press Southern
Africa 2012) 14 85): the parties must seriously intend to contract, be of one mind as to the material
aspects of the proposed agreement (the terms and the identity of the parties to it), and be conscious
of the fact that their minds have met.
In the present case the parties were not in agreement as to the consequences they wished to create:
Y thought that the dates for the art exhibition (X’s performance) was fixed, while X knew that the
contract allowed X to unilaterally change the dates. This is a mistake as to the obligations the parties
wished to create which excludes consensus between the parties (Hutchison and Pretorius Contract
86). No contract can arise on the basis of the will theory. This type of mistake can be illustrated with
a number of cases.
In Allen v Sixteen Stirling Investments (Pty) Ltd 1974 (4) SA 164 (D) the plaintiff believed that he was
purchasing the erf shown to him by the seller's agent, while the written contract that he signed
indicated a completely different property. His mistake related to performance and was material. See
also Du Toit v Atkinson's Motors Bpk 1985 (2) SA 889 (A). The appellant signed an agreement
containing a term excluding the respondent from liability for misrepresentation. Finally see Sonap
Petroleum (SA) (Pty) Ltd (formerly known as Sonarep (SA) (Pty) Ltd) v Pappadogianis 1992 (2) SA 234
(A) where the appellant erred with regard the period of the lease which was an aspect of the
performance.
The direct reliance approach can now be applied to the facts of the problem (Hutchison and
Pretorius Contract 18-19 95-97 103-105). The court stated the test in Sonap Petroleum (SA) (Pty) Ltd
(formerly known as Sonarep (SA) (Pty) Ltd) v Pappadogianis 1992 (2) SA 234 (A) 239-240 as follows:
In my view, therefore, the decisive question in a case like the present is this: did the party whose
actual intention did not conform to the common intention expressed, lead the other party, as a
reasonable man, to believe that his declared intention represented his actual intention? … To answer
this question, a three-fold enquiry is usually necessary, namely, firstly, was there a misrepresentation
as to one party’s intention; secondly, who made that representation; and thirdly, was the other party
misled thereby? … The last question postulates two possibilities: Was he actually misled and would a
reasonable man have been misled?
A discussion of Ridon v Van der Spuy and Partners (Wes-Kaap) Inc 2002 (2) SA 121 (K) and Steyn v LSA
Motors Ltd 1994 (1) SA 49 (A) will also be appropriate.
By signing the contract, Y, a party to the contract, misrepresented his intention to be bound by the
clause allowing X to unilaterally change the dates. X knew that the only dates mentioned during the
negotiations were 24 to 27 July, that Y hastily signed the contract and that the contract had a clause
allowing X to unilaterally change the dates. Although it could be argued that X was not actually
misled by Y’s misrepresentation, it is clear that a reasonable person would not have been misled in
any case. Indeed, X had no reason to believe that Y would have signed the contract had Y known of
the term allowing X to change the dates of the exhibition unilaterally. In fact X had a legal duty to
point out the presence of this clause in the agreement to Y. There was either no actual or at least
reasonable reliance on the part of X.
The giving of appropriate advice
Y is not bound by the agreement with X because of the lack of actual and apparent consensus.
Total: [10]

Question 2
X, an organiser of art exhibitions, contracted with Y for an exhibition to be held on 24 to 27 July.
These dates were the only dates mentioned during the negotiations. After having been pressurized
by X, Y hurriedly signed the standard form contract without reading it. The contract contained a
clause permitting X to change the dates of the exhibition unilaterally. Thereafter X changed the dates.
X had no reason to believe that Y would have signed the contract if he had known of the term. Y
averred that the contract was void. Will Y succeed in his attempt to have the contract set aside?
Substantiate your answer and refer to relevant case law. Apply the indirect reliance approach of the
courts in answering this question. Do not apply the Consumer Protection Act to this question. (10)

Answer
Identifying the problem
The facts seemingly indicate that X and Y have not reached consensus based on the will theory. If so,
it is necessary to determine if Y may be held bound to a contract with X, based on the reliance
theory, or whether Y will escape liability. Only the indirect approach to the reliance theory will be
considered.
Discussing the relevant law applicable to the problem, referring to the relevant case law, AND
applying the law to the facts of the problem
The indirect approach is the iustus error-approach. A party who acted under a mistake and wishes to
escape liability (Y in our case) must prove that his / her mistake is material and reasonable.
At the outset it must be determined whether agreement (consensus ad idem) as a contractual basis
exists between the parties, as required in terms of the will theory. Consensus has three elements
(Hutchison and Pretorius (eds) The law of Contract in South Africa Oxford University Press Southern
Africa 2012 14 85): the parties must seriously intend to contract, be of one mind as to the material
aspects of the proposed agreement (the terms and the identity of the parties to it), and be conscious
of the fact that their minds have met.
In the present case the parties were not in agreement as to the consequences they wished to create:
Y thought that the dates for the art exhibition (X’s performance) was fixed, while X knew that the
contract allowed X to unilaterally change the dates. This is a mistake as to the obligations the parties
wished to create which excludes consensus between the parties (Hutchison and Pretorius Contract
86). No contract can arise on the basis of the will theory. This type of mistake can be illustrated with
a number of cases.
In Allen v Sixteen Stirling Investments (Pty) Ltd 1974 (4) SA 164 (D) the mistake related to
performance and was thus material. The plaintiff believed that he was purchasing the erf shown to
him by the seller's agent, while the written contract that he signed indicated the correct erf which
was a completely different property. His mistake related to performance and was material. See also
in Du Toit v Atkinson's Motors Bpk 1985 (2) SA 889 (A), the appellant signed an agreement containing
a term excluding the respondent from liability for misrepresentation. Finally see Sonap Petroleum
(SA) (Pty) Ltd (formerly known as Sonarep (SA) (Pty) Ltd) v Pappadogianis 1992 (2) SA 234 (A) where
the appellant erred with regard the period of the lease which was an aspect of the performance.
However, the matter does not end here, because Y still has to prove that his mistake is reasonable. A
mistake will generally be reasonable (Hutchison and Pretorius Contract 100-103) in three instances of
which only one is relevant, to this question.
Where the mistake was induced or caused by the failure of the contract enforcer to remove an
incorrect impression (ommissio). Here it will only be wrongful if the contract enforcer breached a
legal duty to speak in the circumstances. Such a duty will usually exist (Hutchison and Pretorius
Contract 101-102) where the contract assertor knows or ought to know as a reasonable person that
the other party is mistaken, or where the contract assertor, before the conclusion of the
contract, created an impression which is in direct conflict with the agreement he or she seeks to
enforce. Under these circumstances, the contract assertor must draw the contract denier’s attention
to this discrepancy. (See the Du Toit case; Hutchison and Pretorius Contract 101-102).
In this question, the only dates mentioned during negotiations (for the exhibition) were 24-27 July.
Since X had no reason to believe that Y would have signed the contract had Y known of the term
allowing X to change the dates of the exhibition unilaterally, he (X) had a legal duty to point out this
clause to Y. X’s failure to do so, renders Y’s material mistake reasonable.
The giving of appropriate advice
Y is not bound by the agreement with X because of the lack of actual and apparent consensus.
Total: [10]

Question 3
X contracts with Y for the latter (Y) to build and fit a security gate for the entrance of her (X’s) home.
Y builds the gate and fits it with an electric motor which is activated with a remote control. X is
satisfied with the work and pays Y the contractual amount agreed upon. A week later the gate gets
stuck while it is half way open as a result of defective materials used to build the gate. When X
attempts to physically move the gate to close it fully, she suffers such severe damage to her left knee
that she has to have a knee operation. Her medical costs are R20 000. The costs of repairing the gate
amount to R15 000. X wants to claim both her medical costs, as well the cost of repairing the gate
from Y. Advise X if she will be successful with her claim. (20)

Answer
Identifying the problem
This question deals with one of the remedies for breach of contract by Y, as she used defective
materials to build the gate. As a result of this breach, X now seeks to claim damages. The type of
breach that has transpired is positive malperformance on the part of Y, and the damages claimable
by X will be determined on the basis of whether such damages constitute general or special
damages.
Discussing the relevant law applicable to the problem, referring to the relevant case law, AND
applying the law to the facts of the problem
Is X entitled to compensated for both her medical costs (R20 000), as well the cost of repairing the
gate (R15 000) from Y.
In order to succeed with a claim for damages as a result of a breach of contract the innocent party
must prove the following:
(1) Breach of contract has occurred.
(2) The innocent party) has suffered financial or patrimonial loss
(3) There is a factual causal link between the breach and the loss
(4) The loss is not too remote a consequence of the breach (legal causation)
(1) Breach of contract has occurred
Positive malperformance takes place where a contracting party does not comply with the terms of
the contract either by performing something in a manner which does not comply with the terms of
the contract, or by doing something which he undertook not to do.
In this question, it is either a tacit term or a term implied by law that the gate will not be built with
defective materials. Therefore Y’s breach constitutes positive malperformance.
(2) The innocent party must suffer patrimonial loss
Breach of contract per se does not give rise to a claim for damages, unless patrimonial loss has
actually been incurred. The plaintiff must prove actual pecuniary or patrimonial loss. Patrimonial loss
is a loss suffered by the estate of the person concerned.
To ascertain whether damage has been suffered as a result of breach of contract, one compares the
present value of the creditor's estate with the value it would have had, had the breach of contract
not occurred (the difference rule). In other words in the case of breach of contract one compares the
present value of the innocent party's estate with the value it would have had, had the contract been
carried out properly and on time. If the present value is less than it would have been, damage has
been suffered. The debtor must place the creditor in the same patrimonial position as he would have
been in had proper and timeous performance taken place. This is the measure or formula applied for
damages and is referred to as positive interest.
The innocent party has to receive his positive interest - in contrast with negative interest; that is the
compensation payable if the injured party would have to be placed in the position in which he would
have been, had the contract never been entered into.
In this problem it is clear that damages have been suffered by X, in the form of R20 000 (for the
medical costs) and R15 000 (for the cost of repairing the gate).
(3) Causation
Factual causation
There must be a causal connection between the breach of contract and damage. The damage must
be caused by the breach of contract. The question is whether the damage would have been incurred
if the guilty party had properly fulfilled her part of the contract. A certain result is caused by a certain
act if that result would not normally have ensued but for such act (conditio sine qua-non-test).
The innocent party needs to prove, on a balance of probabilities, that the loss would not have been
suffered but for the breach. If she fails to establish this causal link, that is the end of the enquiry and
the damages claim must fail.
Factual causality is present in this problem. If Y did not breach the contract, the gate would not have
needed repair. Furthermore, if Y did not breach the contract the gate would have not have got stuck,
X would not have tried to open the gate and X would not have been injured.
Legal causality
The question which arises after factual causation has been established is whether the innocent party
may hold the other party liable for all the consequences of the breach. In the interests of fairness to
the party that commits a breach of contract, a line must be drawn between damages caused by her
breach and for which he is to be held liable, and damages which, although caused by the breach, are
so remote from it that he should not be held liable for them. It is often very difficult to make this
distinction.
In regard to this issue, it is important to understand the distinction made by our courts between
general and special damages (Holmdene Brickworks (Pty) Ltd v Roberts Construction Co Ltd 1977 (3)
SA 670 (A)).
(a) General damages
General damages are those which flow naturally and generally from the specific kind of breach that
has been committed. They are the sort of damages that might be expected in the ordinary course of
things to result from the breach. As such, they would have been foreseeable to a reasonable person
entering into the contract as a probable consequence. The party that commits a breach of contract is
held liable, without further ado, for general damages.
An example of general damages is the cost of repairing or replacing defective goods. The repair costs
(R15 000) of the gate are thus general damages. X is likely to be successful in this claim.
It is not clear whether the medical costs are general damages or not. On the one hand it could be
argued that a reasonable person entering into the contract would foresee that the use of defective
materials could result in X sustaining injuries as a probable consequence, but the contrary could
more convincingly be argued. The question then arises whether the medical costs could be claimed
as special damages.
(b) Special damages
All damages that cannot be classified as general damages are special damages. Special damages are
those which do not flow naturally and generally from the specific kind of breach of contract. The
party that commits a breach of contract will be liable for special damages only in certain
circumstances. The innocent party must prove:
(i) the damages were actually foreseen or reasonable foreseeable at the time of entry into the
contract (the contemplation principle); and
(ii) the parties must have entered into the contract on the basis of their knowledge of the special
circumstances, and thus can be taken to have agreed, expressly or tacitly, that there would be
liability for damages arising from such special circumstances (the convention principle).
It is unlikely that X will succeed with her claim for special damages. X and Y did not actually foresee
that as a result of such a breach, X would physically injure herself (the contemplation principle). The
parties clearly did not agree explicitly or presumably that such damages would be paid (in line with
the convention principle). X’s claim for medical costs would thus be unsuccessful.
The giving of appropriate advice
X will succeed with her claim for the repair costs, but most probably not for her medical costs.
TOTAL (20)

PVL3702_201_2018

Question
Read the judgment in Steyn v LSA Motors Ltd 1994 (1) SA 49 (A). Identify the judgment of the
Appellate Division case on which this judgement is based as well as the relevant section in the
prescribed textbook. Read both. You can find judgments in conventional law libraries, online at the
website of the Southern African Legal Information Institute (SAFLII) (www.saflii.org) or as an e-
resource on the Unisa Library site (choose Juta Law Online Publications and then South African Law
Reports). Then use those authorities to answer the following question:
John, a racehorse owner, advertises for sale the horse Fire for R1.5 million. In the advertisement it is
stated that Fire is an offspring of the legendary July winner, Lightning. Peter is a horse breeder who
specifically wishes to introduce the bloodline of Lightning into his stud. He agrees orally with John to
buy Fire for R1.5 million. Later, in order to meet the requirements of the horse breeders’ association,
John has a written contract drawn up which Peter signs in John’s presence without reading. The
contract makes no mention of Fire's ancestry, but does contain a clause exempting John from liability
for any representations made during negotiations or in the contract. Peter's attention is not drawn to
the provisions of this clause. A month later Peter finds out that Fire is in fact not an offspring of
Lightning, although at the time of the conclusion of the contract John genuinely and without any
fault on his part believed that to be the case. Advise Peter on whether the contract of sale is valid.
Substantiate your advice and refer to relevant case law. Do not apply the law with regard to
misrepresentation or the Consumer Protection Act 68 of 2008 to this question. (10)

Answer
In order to identify the law, which you should apply to the problem, you were asked to:
(1) read Steyn v LSA Motors Ltd 1994 (1) SA 49 (A) to determine the ratio decidendi of the case;
(2) identify the Appellate Division case on which the ratio decidendi of Steyn was based;
(3) identify the relevant section(s) in the textbook; and
(4) not to discuss misrepresentation.
(1) The ratio decidendi of the Steyn case
The court held that Steyn could not accept the offer, because the offer was not directed at him. A
reasonable person in Steyn’s position would have realised that the offer was open only to
professional golfers and, accordingly, that he had no contractual claim to the car. This answers the
third leg of the test in Sonap Petroleum (SA) (Pty) Ltd (formerly known as Sonarep (SA) (Pty) Ltd) v
Pappadogianis 1992 (2) SA 234 (A).
(2) Identify the AD case on which the ratio decidendi in Steyn was based
The decision in Steyn case, was thus based on Sonap Petroleum (SA) (Pty) Ltd (formerly known as
Sonarep (SA) (Pty) Ltd) v Pappadogianis 1992 (2) SA 234 (A).
The court stated the test as follows (239-240):
“In my view, therefore, the decisive question in a case like the present is this: did the party whose
actual intention did not conform to the common intention expressed, lead the other party, as a
reasonable man, to believe that his declared intention represented his actual intention? … To answer
this question, a three-fold enquiry is usually necessary, namely, firstly, was there a misrepresentation
as to one party’s intention; secondly, who made that representation; and thirdly, was the other party
misled thereby? …The last question postulates two possibilities: Was he actually misled and would a
reasonable man have been misled?”
(3) Identify the relevant section in the textbook
If the “Table of cases” is consulted at the end of Hutchison and Pretorius (eds) The law of Contract in
South Africa 3rd ed (2017) Oxford, Cape Town you will find the pages where Steyn and Sonap are
discussed in the textbook. One of the pages on which Steyn is discussed is page 109. In footnote 214
the Steyn case is identified as an example of an instance where there is no objective appearance of
agreement. Remember that, before the iustus error approach can apply, there must be a clear,
objective agreement between the parties, such as when the parties have signed a contractual
document. In Steyn, the court did not apply the iustus error approach, because no apparent or
ostensible contract existed between the parties. Steyn had one interpretation of the offer (that it was
open to all players) and the sponsor another (that it was only open to professional players).
Consequently, there was no clear, apparent contract and the iustus error approach could not be
applied.
In Hutchison and Pretorius Contract 106-108 a discussion of the Sonap case is to be found. In this
discussion the test quoted above in (2) is identified as the reliance theory.
It is thus clear from the above that the subjective approach of the courts (will theory as qualified by
the reliance theory) should be applied to the assignment problem.
(4) Do not discuss misrepresentation
This misrepresentation that you must not discuss cannot be the same as the misrepresentation in the
first leg of the three-fold test of Sonap (there must be a misrepresentation of the intention of one of
the parties), because we know we have to base our answer on Steyn and Sonap that both deal with
error.
So what does the instruction mean? In the given facts, Peter made an innocent misrepresentation
with regard to Fire’s lineage. This is thus the misrepresentation we need not discuss in so far it does
not cause a material mistake.
Identifying the problem
The subjective approach of the courts involves the application of the will theory as qualified by quasi-
mutual assent (Hutchison and Pretorius Contract 93-100).
The facts seemingly indicate that John and Peter have not reached consensus based on the will
theory. If that is the case, it is necessary to determine if Peter may be held bound to a contract with
John, based on the reliance theory.
Discussing the relevant law applicable to the problem AND applying the law to the facts of the
problem
The subjective approach implies that we first have to determine whether agreement between the
parties exists as required in terms of the will theory or whether a party acted under a material
mistake. Consensus has three elements (Hutchison and Pretorius Contract 87):
 The parties must seriously intend to contract,
 Be of one mind as to the material aspects of the proposed agreement (the terms and the identity
of the parties to it), and
 Be conscious of the fact that their minds have met.
In the present case the innocent misrepresentation regarding Fire’s lineage causes a mistake
regarding a characteristic (the lineage) of the thing sold, Fire (an error in substantia). There are two
views on whether such a mistake is material (Hutchison and Pretorius Contract 90-92). The one view
is that the mistake is not material (Trollip v Jordaan 1961 1 SA 238 (A); The other view is that such a
mistake is material (Spenmac (Pty) Ltd v Tatrim CC 2015 3 SA 46 (SCA). The latter view cannot be
correct as the parties wanted to buy and sell the same horse, Fire.
The parties were, however, not in agreement as to the consequences they wished to create: Peter
did not know that there was a clause in the contract he signed exempting John from liability for any
representations made during negotiations or in the contract, but John knew that the sale included an
exemption clause. Peter made a mistake as to the obligations the parties wished to create which
excludes consensus between the parties (Hutchison and Pretorius Contract 88). No contract can thus
arise on the basis of the will theory.
The facts of our problem are very similar to that in Du Toit v Atkinson's Motors Bpk 1985 2 SA 889 (A)
where the appellant signed an agreement without reading it which contained a term excluding the
respondent’s liability for misrepresentation. The court held that the mistake regarding the exemption
clause was material.
This type of mistake also occurred in other cases. In Allen v Sixteen Stirling Investments (Pty) Ltd
1974 (4) SA 164 (D) the plaintiff believed that he was purchasing the erf shown to him by the seller's
agent, while the written contract that he signed indicated the correct erf which was a completely
different property. His mistake related to performance and was material. In Sonap Petroleum (SA)
(Pty) Ltd (formerly known as Sonarep (SA) (Pty) Ltd) v Pappadogianis 1992 2 SA 234 (A) the appellant
erred with regard the period of the lease which was an aspect of the performance.
A valid contract could still arise in terms of the doctrine of quasi-mutual assent or direct reliance
theory (Hutchison and Pretorius Contract 98-100 and 106-108). The court stated the test in Sonap
Petroleum (SA) (Pty) Ltd (formerly known as Sonarep (SA) (Pty) Ltd) v Pappadogianis 1992 2 SA 234
(A) 239-240 as follows:
“In my view, therefore, the decisive question in a case like the present is this: did the party whose
actual intention did not conform to the common intention expressed, lead the other party, as a
reasonable man, to believe that his declared intention represented his actual intention? … To answer
this question, a three-fold enquiry is usually necessary, namely, firstly, was there a misrepresentation
as to one party’s intention; secondly, who made that representation; and thirdly, was the other party
misled thereby? … The last question postulates two possibilities: Was he actually misled and would a
reasonable man have been misled?”
One of the parties to the contract, Peter, misrepresented his intention to be bound by the contract by
signing the contract. Although it could be argued that John was actually misled by this
misrepresentation of Peter, it is clear that a reasonable person in the position of John would not have
been misled thereby. John knew in fact that there was no exemption clause in the oral contract while
the written contract had such a clause. John should have realised that Peter could have thought that
the written contract was also without such a clause and he thus had a legal duty to point out to Peter
the presence of this clause in the written contract. There was either no actual or at least reasonable
reliance on the part of John.
The giving of appropriate advice
The written contract of sale is invalid because of the lack of actual and apparent consensus.
Total: [10]

Question 1
Which statement is INCORRECT?

1 Obligationary agreements create one or more obligations.


2 Absolving agreements discharge or extinguish obligations.
3 Real agreements transfer rights.
4 Transfer agreements transfer rights.
5 All binding agreements are contracts.

Question 2
Which statement regarding the iustus error doctrine is CORRECT?

1 The iustus error doctrine qualifies the objective approach of our courts to error.
2 The iustus error doctrine is very similar to estoppel but does not require fault and prejudice as
estoppel does.
3 The iustus error doctrine can be used to prove the existence of a contract on the basis of quasi
mutual assent.
4 The iustus error doctrine amounts to a direct application of the reliance theory.
5 Option 1 and 4.

Question 3
Which cause of action is/are delictual?

1 Mistake.
2 Culpable misrepresentation.
3 Innocent misrepresentation.
4 Dictum et promissum.
5 All of the above.
Question 4
In which instance has consensus been obtained improperly?

1 A common error.
2 Rectification.
3 An error in substantia.
4 A threat that is not related to an imminent or inevitable evil.
5 An innocent misrepresentation.

Question 5
Which cause(s) of action may render the contract void?

1 A fraudulent and negligent misrepresentation.


2 Duress.
3 Undue influence.
4 A reasonable and material mistake.
5 Commercial bribery.

Question 6
X is employed as a bookkeeper in Y's business. X steals money from the business's bank account over
a long period of time. Y gets forensic auditors in and they determine that X has stolen R50 000. Y
confronts X and threatens to lay a charge of theft against X at the police station unless X signs an
acknowledgement of debt for R50 000 and undertakes to pay back the money in monthly
instalments of R500 each. X signs because he is afraid to go to jail. Which statement regarding the
presence of the requirements for duress is INCORRECT?

1 X had a reasonable fear.


2 Ys threat weakened X’s power of resistance and rendered X's will compliant.
3 There was a threat of an imminent evil to X.
4 The threat against X was not contra bonos mores.
5 The pressure Y used against X caused X to suffer damage

Question 7
X sent an offer by email to Z on 1 February. Z downloaded the email to her computer on 6 February,
but only read it on 7 February. Z sent an email to X on 8 February, in which she accepted the offer. Z’s
email reached X’s service provider on 9 February and could have been downloaded by X on that date.
X only downloaded Z’s email and read it on 11 February. When was the contract between X and Z
concluded?

1 6 February.
2 7 February.
3 8 February.
4 9 February.
5 11 February.
Question 8
X promises to give Y R10 000 if Y successfully passes her matric examination at the end of the year.
This is an obligation subject to a:

1 suspensive time clause.


2 resolutive time clause.
3 suspensive condition.
4 resolutive condition.
5 modus.

Question 9
Y sells his car to Z for R20 000 on 15 January. Y undertakes to deliver the car to Z on 17 January. The
undertaking to deliver the car on 17 January is a

1 suspensive time clause.


2 essentialium (singular for essentialia).
3 incidentalium (singular for incidentalia).
4 option 1 and 2.
5 option 1 and 3.

Question 10
The courts use the hypothetical bystander test when determining the possible existence of

1 essentialia.
2 naturalia.
3 terms implied by law.
4 tacit terms.
5 express terms.

PVL3702_201_2012

Question
X, an organiser of art exhibitions, contracted with Y for an exhibition to be held on 24 to 27 July.
These dates were the only dates mentioned during the negotiations. After having been pressurized
by X, Y hurriedly signed the standard form contract without reading it. The contract contained a
clause permitting X to change the dates of the exhibition unilaterally. Thereafter X changed the dates.
X had no reason to believe that Y would have signed the contract if he had known of the term. Y
averred that the contract was void. Will Y succeed in his attempt to have the contract set aside?
Substantiate your answer and refer to relevant case law. Apply the indirect reliance approach of the
courts in answering this question. (10)

Answer
Identifying the problem
The facts seemingly indicate that X and Y have not reached consensus based on the will theory. If so,
it is necessary to determine if Y may be held bound to a contract with X, based on the reliance
theory, or whether Y will escape liability. Only the indirect approach to the reliance theory will be
considered.
Discussing the relevant law applicable to the problem, referring to the relevant case law,
AND applying the law to the facts of the problem
The indirect approach is the iustus error-approach. A party who acted under a mistake and wishes to
escape liability (Y in our case) must prove that his / her mistake is material and reasonable.
At the outset it must be determined whether agreement (consensus ad idem) as a contractual basis
exists between the parties, as required in terms of the will theory. Consensus has three elements
(Hutchison and Pretorius (eds) The law of Contract in South Africa Oxford University
Press Southern Africa 2009 14 85): the parties must seriously intend to contract, be of one mind as to
the material aspects of the proposed agreement (the terms and the identity of the parties to it), and
be conscious of the fact that their minds have met.
In the present case the parties were not in agreement as to the consequences they wished to create:
Y thought that the dates for the art exhibition (X’s performance) was fixed, while X knew that the
contract allowed X to unilaterally change the dates. This is a mistake as to the obligations the parties
wished to create which excludes consensus between the parties. No contract can arise on the basis
of the will theory. This type of mistake can be illustrated with a number of cases.
In Allen v Sixteen Stirling Investments (Pty) Ltd 1974 (4) SA 164 (D) the mistake related to
performance and was thus material. The plaintiff believed that he was purchasing the erf shown to
him by the seller's agent, while the written contract that he signed indicated the correct erf which
was a completely different property. His mistake related to performance and was material.
See also in Du Toit v Atkinson's Motors Bpk 1985 (2) SA 889 (A), the appellant signed an agreement
containing a term excluding the respondent from liability for misrepresentation.
Finally see Sonap Petroleum (SA) (Pty) Ltd (formerly known as Sonarep (SA) (Pty) Ltd) v
Pappadogianis 1992 (2) SA 234 (A) where the appellant erred with regard the period of the lease
which was an aspect of the performance.
However, the matter does not end here, because Y still has to prove that his mistake is reasonable. A
mistake will generally be reasonable (Hutchison and Pretorius Contract 100-103) in three instances of
which only one is relevant, to this question.
Where the mistake was induced or caused by the failure of the contract enforcer to remove an
incorrect impression (ommissio). Here it will only be wrongful if the contract enforcer breached a
legal duty to speak in the circumstances. Such a duty will usually exist (Hutchison and Pretorius
Contract 101-102) where the contract assertor knows or ought to know as a reasonable person that
the other party is mistaken, or where the contract assertor, before the conclusion of the contract,
created an impression which is in direct conflict with the agreement he or she seeks to enforce.
Under these circumstances, the contract assertor must draw the contract denier’s attention to this
discrepancy. (See the Du Toit case; Hutchison and Pretorius Contract 101-102).
In this question, the only dates mentioned during negotiations (for the exhibition) were 24-27 July.
Since X had no reason to believe that Y would have signed the contract had Y known of the term
allowing X to change the dates of the exhibition unilaterally, he (X) had a legal duty to point out this
clause to Y. X’s failure to do so, renders Y’s material mistake reasonable.
The giving of appropriate advice
Y is not bound by the agreement with X because of the lack of actual and apparent consensus.
Total: [10]

Question 1
A real agreement is an agreement

1 only creating obligations.


2 whereby a right is transferred.
3 only extinguishing a debt.
4 an example of which is release or discharge.
5 that cannot entail the transfer of ownership of property.

Question 2
Which of the following is NOT a requirement for the creation of a valid contract?

1 There must be agreement or ostensible agreement between the parties.


2 The contracting parties must have capacity to act.
3 The performance must be possible after the contract is concluded.
4 The contract must be legal.
5 Constitutive formalities must be complied with.

Question 3
Portia and Lucas are married out of community of property. They want to purchase Jerome’s car.
Jerome offers to sell his car to Lucas for R100 000. Lucas and Portia accept Jerome’s offer.
Which statement is CORRECT?

1 Only Lucas could accept the offer.


2 Either Lucas or Portia could accept the offer as they are married.
3 Since the offer was only made to Lucas, the acceptance from both Portia and Lucas will never
constitute a counter-offer.
4 A valid contract has been concluded, because Lucas is one of the parties that accepted the offer.
5 None of the above statements

Question 4
X orally offers to sell her painting to Z for R20 000 on 1 April. In terms of the offer X stipulates
that her offer will lapse on 30 April. On 15 April X notifies Z orally that she (X) revokes her offer.
On 25 April Z notifies X orally that he (Z) accepts the offer. Which answer reflects the
CORRECT legal position?

1 X gave a right of pre-emption to Z.


2 The revocation by X is valid, even though an option was concluded.
3 An option exists because X offered to keep her offer open until 30 April, which was accepted on 25
April.
4 X did not give an option to Z.
5 An option exists because there is an agreement to keep the main offer open for a certain time.

Question 5
A right of pre-emption

1 is a right acquired by a contract.


2 is a right acquired by an offer.
3 is a right acquired in terms of an option.
4 is a preferential right entitling the grantee to compel the grantor to sell the thing in question at
anytime.
5 encapsulates all of the above statements.

Question 6
Y lets premises to X. The lease contains a clause prohibiting X from sub-letting the premises without
the written consent of Y. A further clause of the lease requires that any variation of the terms of the
lease (including this clause) has to be in writing and signed by both parties. Later Y and X agree orally
that X can sub-let the premises. After X has sub-let the premises to a third party, Y changes his mind
and informs X that both X and the sub-lessee (third party) should vacate the premises because X has
breached the contract by subletting the premises. Which answer reflects the correct legal position?

1 X does not breach the contract, because the oral agreement constitutes a binding contract, based
on the reliance theory.
2 X breaches the contract, because the oral agreement is not legally binding.
3 X does not breach the contract, because the oral agreement constitutes a binding tacit contract.
4 Y breaches the oral agreement and therefore X and the sub-lessee are entitled to remain on the
premises.
5 X breaches the contract of lease and Y breaches the subsequent oral agreement.
Therefore both parties are in equal guilt, entitling X as the possessor to remain on the premises.

Question 7
X and Y conclude a contract under circumstances wherein X threatened Y. This threat may be used to
prove an element of

1 duress.
2 undue influence.
3 consensus that has been obtained in an improper manner.
4 contractual illegality.
5 material mistake
Question 8
The claim in question (7) is probably

1 delictual.
2 contractual.
3 based on enrichment.
4 based on positive malperformance.
5 fraudulent.

Question 9
S, who lives in Upington, sends P, who lives in Grahamstown, a letter by private courier in which she
offers to sell him her (S’s) motorcycle, a collectors piece, for R100 000. She states in her letter that
her offer will expire on 1 February. P accepts S’s offer by letter which he posts on 31 January. S
receives the letter on 7 February and only reads it on the next day. P tenders payment of R100 000
but S refuses to accept payment. Which theory applies to determine if a valid contract was
concluded?

1 Expedition theory.
2 Reliance theory.
3 Information theory.
4 Reception theory.
5 Will theory.

Question 10
Assume the same facts in question (9). Which statement is CORRECT?

1 No valid contract was concluded.


2 A valid contract was concluded on 7 February in Upington.
3 A valid contract was concluded on the day after 7 February in Upington.
4 A valid contract was concluded on 31 January in Grahamstown.
5 An option contract was concluded in Grahamstown

PVL3702_201_2013
Question
Albert takes his motor vehicle to Dodgy Motors for a service. On his arrival, he is asked to sign a “job
card” by the owner. Albert enquires why he is required to sign the “job card” and the owner explains
to him that by signing he is authorising them to conduct the service on his car which will cost R1 000.
He signs the “job card” without reading it. The “job card” contains a contractual clause authorising
Dodgy Motors to do any repairs on the motor vehicle which they deem necessary without asking the
client’s authorisation and requiring the client to pay for such repairs. While servicing the car, the
service manager finds faults on the car (unrelated to the service) and he proceeds to do these
additional repairs for a further R2 000. Albert refuses to pay for the service and the additional repairs
and argues that he did not read the “job card” and did not know of the existence of all the terms of
the contract. Advise Dodgy Motors on whether
Albert is liable to pay Dodgy Motors R1 000 for the service of Albert’s car. Apply the direct reliance
approach of the courts in answering this question. Do not apply the Consumer
Protection Act to this question.(10)

Answer
Identifying the problem
The essence of this problem is the question whether Albert and the owner of Dodgy Motors have
reached actual consensus or ostensible consensus. Albert will not be contractually bound to pay for
the R1 000 service if this requirement for a valid contract is absent.
Discussing the relevant law applicable to the problem, referring to the relevant case law,
AND applying the law to the facts of the problem
At the outset it must be determined whether agreement (consensus ad idem) as a contractual basis
exists between the parties, as required in terms of the will theory. Consensus has three elements
(Hutchison and Pretorius (2ed) The law of Contract in South Africa Oxford University
Press Southern Africa 2012 13-14 85-87): the parties must seriously intend to contract, be of one
mind as to the material aspects of the proposed agreement (the terms and the identity of the parties
to it), and be conscious of the fact that their minds have met. There are two possible ways of
addressing this issue. Both motivations will be credited equally.
(a) In the present case the parties were in agreement as to the consequences they wished to create,
relating to the service for R1 000: Albert clearly authorised Dodgy Motors to service his motor
vehicle at a cost of R1 000, as it was expressly explained to Albert that by signing the “job card” he
was authorising Dodgy Motors to service his motor vehicle, at a cost of R1 000. It follows that when
Albert signed the job card, both parties clearly understood their obligations:
Dodgy motors would service the motor vehicle, and in return Albert would pay Dodgy Motors R1
000. This means that there was consensus between the parties, resulting in the existence of a valid
contract based on the will theory. Consensus existed between the parties as there was a clear offer
to service the motor vehicle for R1 000, which Albert accepted when he signed the “job card”.
Therefore Albert is liable to pay R1 000 for the service. Based on this conclusion it is not necessary to
consider the direct reliance approach, however, because the question requires same, it will be
considered below.
OR
(b) In the present case the parties were not in agreement as to the consequences they wished to
create: Albert thought that he was authorising Dodgy Motors to only service his car, while the owner
of Dodgy Motors knew that the contract also allowed Dodgy Motors to conduct repairs on the car for
which they deem necessary and payable by Albert without any further authorisation from Albert.
This is a mistake as to the obligations the parties wished to create and is a material mistake which
excludes consensus between the parties. This means that no contract could arise on the basis of the
will theory for both the service of the car and the additional repairs.
This type of mistake can be illustrated with a number of cases.
In George v Fairmead (Pty) Ltd 1958 (2) SA 465 (A) the appellant signed a hotel register without
reading it. The hotel register contained a term excluding the respondent from liability for certain
acts. The appellant was unaware of this term and his mistake related to a term which he believed
would not be in the contract and as such was material because it related to an aspect of
performance.
In Allen v Sixteen Stirling Investments (Pty) Ltd 1974 (4) SA 164 (D) the mistake related to
performance and was thus material. The plaintiff believed that he was purchasing the erf shown to
him by the seller's agent, while the written contract that he signed indicated the correct erf which
was a completely different property. His mistake related to performance and was material.
See also in Du Toit v Atkinson's Motors Bpk 1985 (2) SA 889 (A), the appellant signed an agreement
containing a term excluding the respondent from liability for misrepresentation.
Finally see Sonap Petroleum (SA) (Pty) Ltd (formerly known as Sonarep (SA) (Pty) Ltd) v
Pappadogianis 1992 (2) SA 234 (A) where the appellant erred with regard to the period of the lease
which was an aspect of the performance and thus material.
Whether the motivation under papragraph (a) or (b) above was used, it is still necessary to consider
if contractual liability exists based on the direct reliance approach (as per the instruction in the
question), relating to the service for R1 000.
Direct Approach
With reference to the direct approach, contractual liability is based on the reasonable reliance that
consensus has been reached which the one contractant (contract denier) creates in the mind of the
other contractant (contract enforcer).
According to the Sonap case the direct reliance approach entails a threefold enquiry:
∗ Was there a misrepresentation regarding one party’s intention? Because Albert is now disputing
liability to pay Dodgy Motors R1 000, it appears that by signing the contract Albert made a
misrepresentation that he would be bound to pay Dodgy Motors once the service to his car is
completed.
∗ Who made the misrepresentation? In the problem it was made by a party to the contract,
Albert.
∗ Was the other party actually misled by the misrepresentation and, if so, would a reasonable man
also have been misled? Dodgy Motors was misled and a reasonable man would also have been
misled to think that by signing the card, Albert was authorising the service to be done. This is the
case as Albert was told that by signing the “job card”, he is authorising
Dodgy Motors to conduct the service on his car, at a cost of R1 000.
In our problem Albert did create a reasonable reliance that he wished to be bound contractually for
the sum of R1 000. In any event, if Albert was not contractually bound, he may nevertheless be liable
to pay this amount to Dodgy Motors, based on an unjustified enrichment claim.
Additional marks are also awarded where it is motivated that that Albert is not contractually liable to
pay R2 000 for the additional repairs because he could not have expected the relevant term to be in
the contract (based on the correct application of the above three fold enquiry set out in the Sonap
case). However, this does not exclude the possibility that Albert may be liable for unjustifiable
enrichment for R2 000, for the additional repairs.
The giving of appropriate advice
Albert is bound by an agreement to pay Dodgy Motors R1 000 for the service conducted on his motor
vehicle. This is based on actual and/or apparent consensus.
Total: [10]

Question 1
A selects and pays for several items in a supermarket, and leaves. What type of agreement has been
concluded?
1 An agreement creating obligations.
2 An agreement extinguishing a debt.
3 A real agreement.
4 All of the above.
5 None of the above.

Question 2
The notion that contracts are based on consensus, has given rise to which theory?
1 The declaration theory.
2 The will theory.
3 The reliance theory.
4 The reception theory.
5 The expedition theory.

Question 3
X sells his shop (a business) in a shopping mall to Y. During the negotiations X informs Y of the
overhead costs of running the shop but fails to mention that the rental for the shop space is soon to
escalate dramatically. In fact the rent will be so high that the shop will scarcely be able to turn a
profit. What cause of action will Y be able to rely on in the circumstances?

1 Dictum et promissum.
2 Innocent misrepresentation.
3 Culpable misrepresentation.
4 Material mistake.
5 Non-material mistake

Question 4
Assume the same facts as in question (3). If Y would not have bought the shop had she (Y) known of
the rent escalation, according to what measure would Y’s damages be calculated in the
circumstances?

1 The actio quanti minoris.


2 The actio redhibitoria.
3 Positive interest.
4 Dolus incidens.
5 Dolus dans

Question 5
Assume the same facts as in question (4). If Y wanted to cancel the contract what would she usually
have to do?

1 Tender restitution.
2 Invoke the res vendita.
3 Pay damages to X.
4 Use the actio redhibitoria.
5 None of the above.

Question 6
Mark is engaged to Jane. Mark has a very strong personality and eventually persuades Jane to sell
and transfer her house that is worth R900 000 to him at a purchase price of a mere R20 000. After
registration of the property in Mark’s name he breaks off the engagement. Which of the following
requirement(s) is / are relevant for Jane to prove, in her pursuit to have the transfer of the house into
Mark’s name set aside?
(a) Mark exercised an influence over her.
(b) Mark exercised this influence over her, in an unscrupulous manner in order to induce her to
consent to a transaction which is to her detriment and which she, with normal free will, would not
have concluded.
(c) Mark gained this influence by standing in a position of trust in relation to her.
(d) This influence exercised by Mark over her, amounted to intimidation which was not imposed in
good faith.

1 (a) and (d).


2 (a) and (b).
3 (a), (b) and (c).
4 (a), (b) and (d).
5 (a), (b), (c) and (d).

Question 7
X promises to give Y R10 000 if Y successfully climbs Everest within two weeks. This is an obligation
subject to a:

1 suspensive time clause


2 resolutive time clause
3 suspensive condition
4 resolutive condition
5 modus

Question 8
Y offers to sell his BMW motor car to Z on 1 July, for R50 000. One of the terms of Y’s offer is that the
offer lapses on 30 August. However, on 20 July Y notifies Z that the offer is cancelled. Z insists that
the offer is valid until 30 August and on 25 July Z notifies Y that he (Z) accepts the offer. Which
statement reflects the CORRECT legal position?

1 Y and Z concluded an option contract.


2 Y and Z did not conclude an option contract.
3 Y and Z did not conclude a pre-emption contract.
4 Y and Z concluded a pre-emption contract.
5 2 and 3.

Question 9
A right of pre-emption

1 is a right acquired by a contract.


2 is a right acquired by an offer.
3 is a right acquired in terms of an option.
4 is a preferential right entitling the grantee to compel the grantor to sell the thing in question at
anytime.
5 encapsulates all of the above statements.

Question 10
Y lets premises to X. The lease contains a clause prohibiting X from sub-letting the premises without
the written consent of Y. A further clause of the lease requires that any variation of the terms of the
lease (including this clause) has to be in writing and signed by both parties. Later Y and X agree orally
that X can sub-let the premises. After X has sub-let the premises to a third party, Y changes his mind
and informs X that both X and the sub-lessee (third party) should vacate the premises because X has
breached the contract by subletting the premises. Which answer reflects the correct legal position?

1 X does not breach the contract, because the oral agreement constitutes a binding contract, based
on the reliance theory.
2 X breaches the contract, because the oral agreement is not legally binding.
3 X does not breach the contract, because the oral agreement constitutes a binding tacit contract.
4 Y breaches the oral agreement and therefore X and the sub-lessee are entitled to remain on the
premises.
5 X breaches the contract of lease and Y breaches the subsequent oral agreement. Therefore both
parties are in equal guilt, entitling X as the possessor to remain on the premises.

PVL3702_201_2014

Question
X, an organiser of art exhibitions, contracted with Y for an exhibition to be held on 24 to 27 July.
These dates were the only dates mentioned during the negotiations. After having been pressurized
by X, Y hurriedly signed the standard form contract without reading it. The contract contained a
clause permitting X to change the dates of the exhibition unilaterally. Thereafter X changed the dates.
X had no reason to believe that Y would have signed the contract if he had known of the term. Y
averred that the contract was void. Will Y succeed in his attempt to have the contract set aside?
Substantiate your answer and refer to relevant case law. Apply the indirect reliance approach of the
courts in answering this question. Do not apply the Consumer Protection Act to this question. (10)

Answer
Identifying the problem
The facts seemingly indicate that X and Y have not reached consensus based on the will theory. If so,
it is necessary to determine if Y may be held bound to a contract with X, based on the reliance
theory, or whether Y will escape liability. Only the indirect approach to the reliance theory will be
considered.
Discussing the relevant law applicable to the problem, referring to the relevant case law,
AND applying the law to the facts of the problem
The indirect approach is the iustus error-approach. A party who acted under a mistake and wishes to
escape liability (Y in our case) must prove that his / her mistake is material and reasonable.
At the outset it must be determined whether agreement (consensus ad idem) as a contractual basis
exists between the parties, as required in terms of the will theory. Consensus has three elements
(Hutchison and Pretorius (eds) The law of Contract in South Africa Oxford University
Press Southern Africa 2012 14 85): the parties must seriously intend to contract, be of one mind as to
the material aspects of the proposed agreement (the terms and the identity of the parties to it), and
be conscious of the fact that their minds have met.
In the present case the parties were not in agreement as to the consequences they wished to create:
Y thought that the dates for the art exhibition (X’s performance) was fixed, while X knew that the
contract allowed X to unilaterally change the dates. This is a mistake as to the obligations the parties
wished to create which excludes consensus between the parties
(Hutchison and Pretorius Contract 86). No contract can arise on the basis of the will theory. This type
of mistake can be illustrated with a number of cases.
In Allen v Sixteen Stirling Investments (Pty) Ltd 1974 (4) SA 164 (D) the mistake related to
performance and was thus material. The plaintiff believed that he was purchasing the erf shown to
him by the seller's agent, while the written contract that he signed indicated the correct erf which
was a completely different property. His mistake related to performance and was material.
See also in Du Toit v Atkinson's Motors Bpk 1985 (2) SA 889 (A), the appellant signed an agreement
containing a term excluding the respondent from liability for misrepresentation.
Finally see Sonap Petroleum (SA) (Pty) Ltd (formerly known as Sonarep (SA) (Pty) Ltd) v
Pappadogianis 1992 (2) SA 234 (A) where the appellant erred with regard the period of the lease
which was an aspect of the performance.
However, the matter does not end here, because Y still has to prove that his mistake is reasonable. A
mistake will generally be reasonable (Hutchison and Pretorius Contract 100-103) in three instances of
which only one is relevant, to this question.
Where the mistake was induced or caused by the failure of the contract enforcer to remove an
incorrect impression (ommissio). The mistake will only be wrongful if the contract enforcer breached
a legal duty to speak in the circumstances. Such a duty will usually exist (Hutchison and Pretorius
Contract 101-102) where the contract assertor knows or ought to know as a reasonable person that
the other party is mistaken, or where the contract assertor, before the conclusion of the contract,
created an impression which is in direct conflict with the agreement he or she seeks to enforce.
Under these circumstances, the contract assertor must draw the contract denier’s attention to this
discrepancy. (See the Du Toit case; Hutchison and Pretorius Contract 101-102).
In this question, the only dates mentioned during negotiations (for the exhibition) were 24-27 July.
Since X had no reason to believe that Y would have signed the contract had Y known of the term
allowing X to change the dates of the exhibition unilaterally, he (X) had a legal duty to point out this
clause to Y. X’s failure to do so, renders Y’s material mistake reasonable.
The giving of appropriate advice
Y is not bound by the agreement with X because of the lack of actual and apparent consensus.
Total: [10]
Question 1
Which of the following is NOT a requirement for a valid contract?

1 Consensus
2 Formalities
3 Possibility
4 Reciprocity
5 Certainty

Question 2
Which statement is INCORRECT?

1 A contract is a unilateral or even bilateral juristic act.


2 A contract entails promises or undertakings on one or both sides.
3 An undertaking in a contract that a certain state of affairs exists, or has existed, is known as a
warranty.
4 The conclusion of a contract can be multilateral.
5 Freedom of contract means that the parties can agree to anything that is possible and lawful.

Question 3
Which statement is INCORRECT?

1 An obligation is a legal bond between a debtor and a creditor.


2 With a contract of sale, the seller is the debtor in respect of the duty to deliver the thing sold, and
the seller is the creditor in respect to the obligation relating to the duty to pay the price.
3 An obligation comprises a right and a corresponding duty: the right of the creditor to demand a
performance by the debtor, and the duty of the debtor to make that performance.
4 A natural obligation is unenforceable in a court of law.
5 The right created by an obligation is a real right.

Question 4
Which statement is INCORRECT?

1 A delict is wrongful and blameworthy conduct that causes harm to a person, and which obliges the
wrongdoer to compensate the injured party.
2 Concurrent liability exists where the same conduct might constitute both a delict and a breach of
contract, as when a surgeon who has contracted to perform an operation negligently leaves a cotton
swab inside the patient’s body.
3 The essential difference between contractual and delictual obligations is that the latter are, as a
general rule, voluntarily assumed by the parties themselves, whereas the former are imposed by
law, irrespective of the will of the parties.
4 The courts have shown great reluctance to permit claims in delict for economic losses caused by a
breach of contract.
5 Unjustified enrichment occurs when there is a shift of wealth from one person’s estate to another’s
without a good legal ground or cause for this shift.
Question 5
Which statement relating to the Bill of Rights in the Constitution of the Republic of South Africa,
1996, is CORRECT?
1 Vertical application relates to relationships between private persons, as in most contractual
situations.
2 Horizontal application relates to relationships between the state and the individual.
3 In Barkhuizen v Napier 2007 (5) SA 323 (CC), the majority in the Constitutional Court ruled that a
contractual term can be tested directly against a provision in the Bill of Rights.
4 In Barkhuizen v Napier 2007 (5) SA 323 (CC), the minority in the Constitutional Court preferred an
indirect application of the Constitution to the contractual dispute before them.
5 In Barkhuizen v Napier 2007 (5) SA 323 (CC), Ngcobo J reasoned that the proper approach to
constitutional challenges to contractual terms, is to determine whether the term challenged is
contrary to public policy; and what constitutes public policy must be discerned with reference to
the fundamental values embodied in the Constitution.

Question 6
Cedric has joined a religious sect. Recently this sect took advantage of Cedric’s good nature and
convinced him to donate his motor vehicle to the sect, which Cedric did. Cedric approaches you for
legal advice, explaining that he wants his motor vehicle returned to him. The issue emanating from
these facts relates to

1 duress.
2 undue influence.
3 commercial bribery.
4 puffs.
5 dicta et promissa

Question 7
Assume the same facts as in question (6). A court presiding over this case is likely to find that the
religious sect

1 threatened Cedric and more particularly his property.


2 instilled reasonable fear in Cedric.
3 coerced Cedric to donate his motor vehicle to the sect.
4 obtained an influence over Cedric, and that this influence weakened his powers of resistance and
rendered his will compliant.
5 conducted itself in such a distasteful manner that it amounts to supervening impossibility of
performance.

Question 8
Which of the following is NOT an element for commercial bribery?

1 A reward.
2 The one party is the briber.
3 The principal is able to exert influence over a third party.
4 The agent may be an agent in the true sense or merely a go-between or facilitator.
5 A direct or indirect benefit is sought for the briber
Question 9
If commercial bribery exists, the agreement between the briber and the agent is

1 voidable.
2 void.
3 void and voidable.
4 neither void nor voidable.
5 for the benefit of a third party.

Question 10
If commercial bribery exists, the agreement between the briber and the principal is

1 voidable.
2 void.
3 void and voidable.
4 neither void nor voidable.
5 for the benefit of a third party.

PVL3702_201_2011

Question
Albert takes his motor vehicle to Dodgy Motors for a service. On his arrival, he is asked to sign a “job
card” by the owner. Albert enquires why he is required to sign the “job card” and the owner explains
to him that by signing he is authorising them to conduct the service on his car which will cost R1 000.
He signs the “job card” without reading it. While servicing the car, the service manager finds faults
on the car (unrelated to the service) and he proceeds to do these additional repairs for a further R2
000. Albert refuses to pay for the additional repairs and argues that he did not authorise such
repairs. The owner of Dodgy Motors argues that Albert is obliged to pay for the work done as the
“job card” contains a contractual clause authorising Dodgy Motors to do any repairs on the motor
vehicle which they deem necessary without asking the client’s authorisation and requiring the client
to pay for such repairs. Advise Albert on whether he is liable on the contract to pay Dodgy
Motors R2 000 for the additional repairs. Substantiate your answer and refer to relevant case law.
Apply the direct approach of the courts in answering this question. (10)

Answer
The essence of this problem is the question whether Albert and the owner of Dodgy Motors has
reached actual consensus or ostensible consensus. Albert will not be contractually bound to pay for
the additional repairs If this requirement for a valid contract is absent.
The first step is to determine whether agreement (consensus ad idem) exists between the parties, as
required in terms of the will theory. Agreement has three elements (Van Aswegen et al Law of
Contract: only study guide for PVL301-W (1996) 13):
∗ agreement between the parties as to the consequences they wish to create;
∗ agreement as to the intention of the parties to create legal consequences; and
∗ an awareness regarding their unanimity.
In the present case the parties were not in agreement as to the consequences they wished to create:
Albert thought that he was authorising Dodgy Motors to only service his car, while the owner of
Dodgy Motors knew that the contract also allowed Dodgy Motors to conduct repairs on the car
which they deem necessary and which should be paid by Albert without any further authorisation
from Albert. This was a mistake as to the obligations the parties wished to create and was a material
mistake which excludes consensus between the parties. This means that no contract could arise on
the basis of the will theory. This type of mistake can be illustrated with a number of cases.
In George v Fairmead (Pty) Ltd 1958 (2) SA 465 (A) the appellant signed a hotel register without
reading it. The hotel register contained a term excluding the respondent from liability for certain
acts. The appellant was unaware of this term and his mistake related to a term which he believed
would not be in the contract and as such was material because it related to an aspect of
performance.
In Allen v Sixteen Stirling Investments (Pty) Ltd 1974 (4) SA 164 (D) the plaintiff believed that he was
purchasing the erf shown to him by the seller's agent, while the written contract that he signed
indicated the another erf which was a completely different property. His mistake related to
performance and was material.
The appellant signed a contract without reading it in Du Toit v Atkinson's Motors Bpk 1985 (2) SA 889
(A). The contract contained a term excluding the respondent from liability for misrepresentation
Once again the mistake related to an aspect of performance.
In Sonap Petroleum (SA) (Pty) Ltd (formerly known as Sonarep (SA) (Pty) Ltd) v Pappadogianis
1992 (3) SA 234 (A) the parties concluded a 20-year notarial lease contract. A later addendum to the
contract drafted by the appellant's attorney incorrectly indicated that the period of the lease was
15 years. Again the appellant signed the addendum without reading it. The appellant erred with
regard the period of the lease which was an aspect of the performance.
However, the matter does not end here, because a party may be held contractually liable on the
basis of a supplementary ground for liability, namely the reliance theory. In this regard you were
asked to apply the direct reliance approach of the courts. Contractual liability is then based on the
reasonable reliance that consensus has been reached which the one contractant (contract denier)
creates in the mind of the other contractant (contract enforcer).
According to the Sonap case the direct reliance approach entails a threefold enquiry:
∗ Was there a misrepresentation regarding one party’s intention? In our problem Albert wanted his
car to be serviced only. Dodgy Motors wanted the power to also unilaterally conduct repairs to the
car, which it deemed necessary. By signing the contract Albert made a misrepresentation that his
intention is the same as that expressed in the contract. This happened in the Sonap case as well.
∗ Who made the misrepresentation? In the problem it was made by a party to the contract, Albert.
∗ Was the other party actually misled by the misrepresentation and, if so, would a reasonable man
also have been misled? Dodgy Motors in our problem could have been actually misled, but a
reasonable man would have taken steps to point out to Albert that the contract allows Dodgy
Motors to unilaterally conduct repairs on the car, because Albert enquired about the purpose of the
“job card” and the owner of Dodgy Motors misled him to believe that by signing the card he is
merely authorising the service to be done. In Sonap the court found that the contract enforcer knew
that the contract denier was acting under a mistake with regard to the reduction of the term of the
lease and consequently was not misled by the contents of the addendum. The addendum was thus
void.
In our problem Albert did not create a reasonable reliance that he wished to be bound to the
contract he signed. We can conclude that Albert is not contractually liable to pay R2 000 for the
repairs. [This does not exclude the possibility that Albert may be liable for unjustifiable enrichment]
Total: [10]

Question 1
Where a contract is concluded by email, which theory applies?

1 The declaration theory.


2 The expedition theory.
3 The reliance theory.
4 The reception theory.
5 The information theory.

Question 2
S sends a written offer by post to P, in which S offers to sell his radio to P for R3 000. In the written
offer, S stipulates that the acceptance must be communicated to him by 1 June. On 20 May, P posts a
written acceptance to S, to purchase the radio for R3 000. On 22 May, S phones P and cancels his
offer to sell his radio. P’s written acceptance reaches S on 25 May. Which answer reflects the
CORRECT legal position?

1 A contract is concluded between S and P, because the expedition theory applies.


2 A contract is not concluded between S and P, because the information theory applies.
3 A contract is concluded between S and P, because the information theory applies.
4 A contract is not concluded between S and P, because the expedition theory applies.
5 A contract is concluded between S and P, because the reception theory applies.

Question 3
A offers B her car for R20 000 cash. B answers: “I will buy your car for R19 000 cash.” By this answer,

1 B accepts A’s offer.


2 B accepts A’s offer unequivocally.
3 B accepts and rejects A’s offer.
4 B rejects A’s offer and makes her (A) a counter-offer.
5 B complies with the requirement that the acceptance must not be ambiguous.

Question 4
Where a culpable misrepresentation exists, the action which arises is based on

1 delictual liability.
2 unjustified enrichment.
3 estoppel.
4 statutory liability.
5 contractual liability.

Question 5
Sibongile is desperate to sell her house because of a termite infestation. Consequently she
knowingly conceals all signs of damage when a potential purchaser, Thandi, comes to inspect the
house and furthermore tells Thandi that there is nothing wrong with the house. Thandi purchases
the house, which she would never have done if she knew of the termite infestation. What cause of
action will Thandi be able to rely on in the circumstances?
(a) Dictum et promissum.
(b) Innocent misrepresentation.
(c) Culpable misrepresentation.
(d) Material mistake.

1 (a), (b) and (c).


2 (a) and (c).
3 (b) and (c).
4 Only (c).
5 Only (d).

Question 6
Assume the same facts as in question (5). What remedy or remedies are available to Thandi when
she discovers the termite infestation and damage to the house?

1 Thandi may only rescind the contract.


2 Thandi may only uphold the contract and claim damages.
3 Thandi may rescind the contract and claim damages.
4 Thandi may uphold the contract and claim damages.
5 Both 3 and 4.

Question 7
In which case did the seller not reveal to the purchaser that the swimming pool was not structurally
sound because it leaked?

1 Trotman and Another v Edwick 1951 (1) SA 443 (A).


2 De Jager v Grunder 1964 (1) SA 446 (A).
3 Ranger v Wykerd and Another 1977 (2) SA 976 (A).
4 Bayer South Africa (Pty) Ltd v Frost 1991 (4) SA 559 (A).
5 Phame (Pty) Ltd v Paizes 1973 (3) SA 397 (A).

Question 8
The aggrieved party may be successful in instituting

1 a delictual claim against the other contracting party, for the return of his performance in terms of
an illegal contract, if the in pari delicto rule is relaxed.
2 an unjustified enrichment claim against the other contracting party, for the return of his
performance in terms of an illegal contract, if the par delictum rule is relaxed.
3 a contractual claim against the other contracting party, for the return of his performance in terms
of an illegal contract, if the in pari delicto rule is relaxed.
4 a specific performance claim against the other contracting party, for the return of his performance
in terms of an illegal contract, if the in pari delicto rule is relaxed.
5 (3) and (4).

Question 9
The consequences of an illegally concluded contract were discussed in

1 Jajbhay v Cassim 1939 AD 537.


2 Brandt v Spies 1960 (4) SA 14 (E).
3 Neethling v Klopper 1967 (4) SA 459 (A).
4 Goldblatt v Fremantle 1920 AD 123.
5 Nel v Cloete 1972 (2) SA 150 (A).

Question 10
Where ambiguous words in a clause in a contract are interpreted in such a way that the least
possible burden is placed on the debtor, and the clause is interpreted against the party in whose
favour it was inserted, this is consistent with

1 the primary rules of interpretation.


2 the secondary rules of interpretation.
3 the tertiary rules of interpretation.
4 the parol evidence rule.
5 the pactum in favorem tertii.

PVL3702_ Various MCQ


VARIOUS MULTIPLE CHOICE QUESTIONS

Question 1:

Which statement best describes the basis on which unjustified enrichment law is based in SA law:

1. Unjustified enrichment provides an alternative claim to contractual and delictual claims in


cases where a party simultaneously has a contractual or delictual claim.
2. Unjustified enrichment provides a basis for a claim where the enrichment of the enriched
person has been obtained in an unlawful manner.
3. Unjustified enrichment provides a basis for a claim where there has been a transfer of
property or value from he impoverished party to the enriched party without a sufficient legal
ground.
4. Unjustified enrichment provides a basis for a claim where the enriched party obtained its
enrichment through the use of unfair contract terms.

The following facts are relevant for Questions 2 and 3:

A, an American tourist, has leased a vehicle from B. While travelling in the Northern Cape, the
vehicle breaks down. A contracts with C, a garage in Springbok, to repair the vehicle at a cost of
R12,000. After two days A leases another vehicle from X and completes his trip. A departs for
America. C wants to claim the R12,000 from B.

Question 2:

Which statement best explains whether C has a claim against B and the authority on which it is
based?
(1) In terms of the decision in Gouws v Jester Pool (Pty) Ltd 1968 it was held that C has no claim
against B because B has not been enriched at C’s expense.
(2) In terms of the decision in Gouws v Jester Pool (Pty) Ltd 1968 it was held that C has no
claim against B because B has not been enriched.
(3) The decision in the Gouws case was confirmed in Buzzard Electrical v 158 Jan Smuts avenue
Investments 1996.
(4) The decision in the Gouws case was rejected in Buzzard Electrical v 158 Jan Smuts avenue
Investments 1996.

Question 3:

Which statement best explains whether C has a retention right or whether he can exercise it?

(1) C can exercise a retention right over the vehicle against B until it has been paid in full
contract price.
(2) C can exercise a retention right over the vehicle against B until such time as it has been paid
for its necessary expenses.
(3) In terms of the decision in Buzzard Electrical v 158 Jan Smuts Avenue Investments 1996 party
C has no retention right because he has no enrichment action against B.
(4) C has no retention right under these circumstances.

Question 4:

In which one of the following circumstances can the condictio indebiti be used?

(1) Where an executor who is now functus officio has made a payment to heirs which were not
due because a creditor had lodged its claim too late.
(2) Where a person has made a payment in terms of a contract subject to a resolutive condition
and the contract has now been extinguished due to the condition being fulfilled.
(3) Where an undue payment has been made in circumstances where the mistake is not
excusable.
(4) Where a bank has made payment in terms of a cheque that has been stopped.

Question 5:

Which of the following is/are (a) prerequisite(s) for a claim in terms of the condictio indebiti?

(1) the enrichment was illegal


(2) the defendant was unlawfully enriched
(3) there was a causal link between the enrichment and impoverishment
(4) payment was made in terms of a valid contract

Question 6:

A has sold uncut diamonds to B for an amount of R100,000 in contravention of statutory law. B has
paid the amount but before the diamonds could be delivered, it was confiscated by the police during
a raid of A’s house. Which statement best explains the legal position of the possible claim B may have
against A?

(1) In circumstances like these a court may exercise an equitable judicial discretion to relax the
par delictum rule, depending on the relative turpitude of the parties’ conduct.
(2) B has a claim for damage against A due to a breach of contract.
(3) B has a claim against A in terms of the condictio sine cause specialis because there is no
other enrichment action at his disposal.
(4) B has a claim for damage against A based on delict.

Question 7:

In which one of the following circumstances can the condictio sine causa specialis be used?
(1) as a general enrichment action
(2) where a bank has made payment in terms of a cheque that had been stopped by the
drawer
(3) Where property has been transferred in terms of an illegal agreement.
(4) Where undue payment was made due to an excusable error.

The following facts are relevant for Question 8-10.

K is the owner of a farm adjacent to that of L. Unbeknown to K and L, K has been occupying part of
L’s land due to a fence that was mistakenly put up 10 years ago. K has effected the following
improvements on that part of the farm: (a) built a dam at a cost of R30, 000; (b) a luxury lapa on the
edge of the dam at a cost of R100,000; (c) a borehole at a cost of R20, 000; (d) planted fruit trees at a
cost of R15,000; (e) planted mealies which are almost ready to harvest at a cost of R60,000 (value
R120,000). During his tenure of the land he has harvested mealies worth R300,000 (production cost
R250,000) and fruit from the fruit trees sold at R55,000. L has now become aware of the true
situation and demands that K leaves the land.
Question 8:

Which statement best explains the nature of K’s possession or occupation of the land?

(1) K is a bona fide occupier of the land


(2) K is a mala fide occupier of the land
(3) K is a bona fide possessor of the land
(4) K is a lawful occupier of the land

Question 9:

Which statement best explains the nature and extent of K’s claim(s), if any?

(1) K has an enrichment action for all the expenses that he has incurred on the improvement of
L’s land.
(2) K has an enrichment action for the full amount of all the necessary and useful expenses he
has incurred.
(3) K has an enrichment action for the useful and necessary expenses he has incurred to the
extent that those expenses have increased the value of L’s land.
(4) K has no claim for the mealies which have not been harvested yet as they now belong to L.

Question 10:

Which statement best explains the amounts that may be brought into account against K’s claim, if
any?

(1) L is not entitled to subtract anything from K’s enrichment claim.


(2) L is entitled to reduce the enrichment claim against him by subtracting the value of K’s
occupation of the land.
(3) L is entitled to reduce the enrichment claim against him by subtracting the value of the
mealies and fruit harvested by K and the value of K’s occupation of the land.
(4) L is entitled to reduce the enrichment claim against him by subtracting the value of the
mealies harvested by K minus the production costs.

Question 11:

X and Y have concluded a contract in terms of which Y must perform certain building work for X
against payment of R200,000. The building work is not completed by Y and X has concluded a
contract with Z to complete the work at a cost of R30,000. X now refuses to pay Y because he has
failed to complete his contract. Indicate which one of the following statements best describes Y’s
possible claim:

(1) Y has no claim because he has committed a breach of contract.


(2) Y has a contractual claim against X for a reduced amount (R170,000) because X has
accepted the building work and has made use of it.
(3) Y has an enrichment claim for a reduced amount to be calculated on the basis of the amount
by which the property of X had in fact increased in value.
(4) Y is entitled to payment of the full contract price despite the fact that he has not completed
his contract.

ASSIGNMENT 1 – 2009

Choose the most correct option in every instance. If there is more than one correct option, choose
the appropriate combined option.
Question 1 to 3.

The following facts are relevant for questions 1 to 3.

A has demanded payment from B of an amount of R50,000 which he believes B is owing. B has
checked its records and has paid the amount in the bona fide belief that the amount is owing in
terms of their contract. Unbeknown to B, his bookkeeper, C had already paid the amount a week
earlier by way of an electronic funds transfer in to the account of A. At the time of the second
payment A's account was overdrawn in the amount of R30,000 and was therefore in credit of
R20,000 after the payment. A has taken R15,000 out of his account to pay his employees their
monthly wages. He has also paid R10,000 for a luxury weekend after realising that his account was in
credit.

Question 1:
Which statement best explains the nature of the claim against A?
1. B has a claim against A based on delict for a fraudulent misstatement.
2. B has contractual claim against A based on their contract.
3. B has an enrichment claim against A based on the condictio causa data causa non secuta.
4. B has an enrichment claim against A based on the condictio indebiti.
5. B has no claim against A because he paid the amount voluntarily.

Answer: This question deal with the condictio indebti and its requirements. The claim cannot be
delictual because A’s misrepresentation was innocently made. The claim can also not be based on
the contract because there had already been payment with which extinguished the duty to pay in
terms of the contract. Next evalute the answers against the requirements of the condictio indebti.
Here the one party made a bona fide payment that was not owning and under circumstances that
were excuasble, party because the mistake was induced by A’s misrepresentation.

Question 2:
Which statement regarding the requirements for an enrichment action is correct?

1. A has been enriched at the expense of B.


2. A has been enriched at the expense of C, who made the payment.
3. A's enrichment is not unjustified as there was a contract between A and B.
4. A's enrichment is unlawful because he made a demand for payment at a time that it was not
due.
5. B has been impoverished at the expense of the bank.

Answer: A's enrichment took place at the expense of B because B was the person who in law is
regarded as the one who made the payment, even if C physically made the payment. At the time of
the payment, the duty to pay had already been extinguished – the payment therefore cannot be in
terms of the agreement, even if B thought so. The enrichment is not unlawful because A's conduct
was not delictual in nature. The bank made payment in terms of its agreement with B and is
therefore entitled to debit B's account. Accordingly the bank was not impoverished. Consider
whether all of the other requirements for enrichment liability and the condictio indebiti have been
complied with.

Question 3:
Which statement best explains the calculation of the enrichment claim?

1. B can claim an amount of R50,000 from A with an enrichment action.


2. B can only claim R20,000 from A because his account was overdrawn and the bank received the
benefit of the other R30,000.
3. B can claim nothing as A has not been unjustifiably enriched at his expense.
4. B can claim only R25,000 because the rest of the enrichment amount has been spent on the wages
and A's holiday.
5. B can claim only R40,000 because the rest of the enrichment amount has been lost on the luxury
holiday.

Answer: A was initially enriched by an amount of R50,000 on receipt of the money. The fact that his
overdraft was extinguished does not diminish his enrichment as his debts have decreased by
R20,000. The payment of the wages also does not cause his enrichment to diminish as those are
expense he would have had in any event. The cost of the luxury holiday, however, does constitute an
extinction of his enrichment, as he would probably not have made these expenses if his account had
not been in credit. There is no indication on the facts provided that A should have realised that he
was enriched.

Question 4:
In order to be successful with a claim based on the condictio indebiti, the plaintiff must prove the
following fact(s) or requirement(s):

1. That the impoverished party made a payment that was not due.
2. That the enrichment was unlawful.
3. That the mistake of the impoverished party was excusable.
4. 1 and 3 are correct.
5. 1 and 2 and 3 are correct
Answer: Have another look at the requirements for the condictio indebiti. Unlawfulness is not a
requirement. For the condictio indebiti it is required that the impoverished party must have made a
payment that was not owing as a result of an excusable mistake.

Question 5:
In which one of the following circumstances can the condictio indebiti be used?

1. Where a bank has made payment in terms of countermanded cheque.


2. Where a party knowingly makes a payment that is not due, but under duress and protest.
3. Where a contract is rescinded due to a breach of contract.
4. Where a party has made an undue payment in terms of an illegal contract.
5. Where a party has made payment which is due but where the cause for the payment later falls
away.

Answer: In the case of stopped checks the appropriate action is the condictio sine causa specialis.
Where a contract is terminated due to breach, the action ground is contractual and not in
enrichment. Where one is dealing with illegal contracts, the appropriate action is the condictio ob
turpem vel iniustam causam. In instance number 5 the correct action is the condictio causa data
causa non secuta.

Question 6-7:

The following facts are relevant for Question 6 and 7.

X has concluded a contract with Y to build a tennis court at a cost of R40,000 on the property it is
renting from Z. It can be shown that the value of the property has increased by R20,000 due to the
improvement. X has disappeared before paying Y for the work done. Y now wants to lodge a claim
against Z, the owner of the property.

Answer: This case deals with indirect enrichment. Have another look at the decisions in Gouws v
Jester Pools and the Buzzard Electrical case. In the Gouws case it was decided that Y only had a
contractual claim against the lessee, X and no enrichment action against the owner, Z. In the Buzzard
Electrical case this issue was left undecided by the appellate division.

Question 6:
Which statement best explains the ground on which and amount that Y can claim?

1. Y has an enrichment claim against Z for an amount of R40,000.


2. Y has an enrichment claim against Z for an amount of R20,000.
3. Y has a contractual claim against X for R40,000.
4. X has an enrichment claim against Z for R40,000.
5. Y has an enrichment claim against X R 20,000.
Question 7:
Which statement best explains the authority on which you based your answer in question 6?

1. In terms of the decision in Gouws v Jester Pools (Pty) Ltd 1968 3 SA 63 (T) it was held that Y has
no claim against Z because Z had not been enriched at his expense.
2. In terms of the decision in Gouws v Jester Pools (Pty) Ltd 1968 3 SA 63 (T) it was held that Y has a
claim against Z because Z had been enriched at his expense.
3. The decision in the Gouws case was confirmed Buzzard Electrical v 158 Jan Smuts Avenue
Investments 1996 4 SA 19 (A)
4. The decision in the Gouws case was rejected in Buzzard Electrical v 158 Jan Smuts Avenue
Investments 1996 4 SA 19 (A)
5. The decision in the Gouws case was confirmed in Brooklyn House Furnishers Ltd v Knoetze & Sons
1970 3 SA 264 (A)

Question 8:
G has noticed that his neighbour's (H) stud bull is seriously ill. The neighbour is currently on a hiking
trip in Nepal and cannot be reached. G has called out a veterinary doctor to attend to the bull and
has paid all his bills as well as for the medication. The total cost was R12,000. Despite the treatment
the bull has died. Which statement best explains the basis of G's possible claim against H?

Answer: In this case G does not have to rely on an enrichment claim, because in that case he would
have no claim as the neighbour is not enriched any longer. G can rely on the real action for tending to
another's property, namely the actio negotiorum gestorum contraria. In terms of this action he can
reclaim all expenses reasonably made in the attempt to preserve his neighbour's property.

1. G has no claim against H because the bull has died and the expenses have been wasted.
2. G has an enrichment claim against H for his expenses as necessary expenses.
3. G has a claim against H in terms of the actio negotiorum gestorum contraria for R12,000.
4. G has a claim against H in terms of the actio negotiorum gestorum utilis for R12,000.
5. G's claim against H in terms of the actio negotiorum gestorum contraria will fail because the bull
died.

Question 9:
G has noticed that his neighbour's (H) stud bull is seriously ill. The neighbour is currently on a hiking
trip in Nepal and cannot be reached. G has called out a veterinary doctor to attend to the bull and
has paid all his bills as well as for the medication. The total cost was R12,000. Despite the treatment
the bull has died. G is a meddlesome neighbour and H has previously warned him not to do anything
on his farm under any circumstances, but rather to call K, if G should notice any problem. G did not
bother to call K. Which statement best explains the basis of G's possible claim against H?

Answer: In this case G cannot rely on the true actio negotiorum gestio because he has acted against
the express instructions of his neighbour. He can only rely on the actio negotiorum gestorum utilis,
which is a true enrichment action. Because the bull died, the neighbour is no longer enriched.

1. G has no claim against H because the bull has died and the expenses have been wasted.
2. G has an enrichment claim against H for his expenses as necessary expenses.
3. G has a claim against H in terms of the actio negotiorum gestorum contraria for R12,000.
4. G has a claim against H in terms of the actio negotiorum gestorum utilis for R12,000.
5. G's claim against H in terms of the actio negotiorum gestorum contraria will fail because the bull
died.

Question 10:
Which statement correctly explains the possession or occupation of another's property?

Answer: A bona fide occupier's possession is always unlawful. A possessor always occupies as if it is
the owner, not as a lessee.

1. A bona fide occupier is someone who lawfully occupies the immovable property of another
person.
2. A bona fide occupier is someone who unlawfully occupies the immovable property of another
person as if he is the owner thereof.
3. A bona fide possessor is someone who lawfully occupies the property of another person as if he is
the owner thereof.
4. A bona fide possessor is someone who unlawfully occupies the property of another person as if
he is the owner thereof.
5. A mala fide possessor is someone who unlawfully occupies the property of another person
temporarily as if he is entitled to occupy the property as a lessee.

ASSIGNMENT 2 – 2009

A, an American tourist, has leased a vehicle from B. While travelling in the Northern Cape, the
vehicle breaks down. A contracts with C, a garage in Springbok, to repair the vehicle at a cost of
R12,000. After two days A leases another vehicle from X and completes his trip. He departs for
America. C wants to claim the R12,000 from B.

Question 1:
Which statement best explains whether C has a claim against B and the authority on which it is
based?

1. In terms of the decision in Gouws v Jester Pools (Pty) Ltd 1968 3 SA 63 (T) it was held that C has no
claim against B because B had not been enriched.
2. In terms of the decision in Gouws v Jester Pools (Pty) Ltd 1968 3 SA 63 (T) it was held that C has
no claim against B because B has not been enriched at C's expense.
3. The decision in the Gouws case was confirmed in Buzzard Electrical v 158 Jan Smuts Avenue
Investments 1996 4 SA 19 (A)
4. The decision in the Gouws case was rejected in Buzzard Electrical v 158 Jan Smuts Avenue
Investments 1996 4 SA 19 (A)
5. The decision in the Gouws case was overruled in Brooklyn House Furnishers Ltd V Knoetze & Sons
1970 3 SA 264 (A)

Question 2:
Which statement best explains whether C has a retention right or whether he can exercise it?

1. C can exercise a retention right over the vehicle against B until such time as it has been paid for its
necessary expenses.
2. C can exercise a retention right over the vehicle against B until it has been paid the full contract
price.
3. In terms of the decision in Buzzard Electrical v 158 Jan Smuts Avenue Investments 1996 4 SA 19 (A)
C has no retention right because it has no enrichment claim against B.
4. An enrichment retention right is a personal right and can therefore be exercised only against the
creditor.
5. C has no retention right under these circumstances.

Question 3:
Which statement provides the most correct explanation of the current legal position?

1. Estoppel and waiver are one and the same


2. Estoppel cannot be used to maintain an impression that a right has been waived
3. Estoppel cannot apply to waiver because waiver relates to an existing right while estoppel is
merely a defence
4. A party may be estopped from denying waiver in certain circumstances
5. 2 and 3

Question 4:
In which one of the following circumstances can the condictio indebiti be used?

1. Where a person has made a payment in terms of a contract subject to a suspensive condition, and
the contract has now been extinguished due to the condition being fulfilled.
2. Where a person has made a payment in terms of a contract subject to a resolutive condition, and
the contract has now been extinguished due to the condition being fulfilled.
3. Where an undue payment has been made in circumstances where the mistake is not excusable.
4. Where an executor who is now functus officio has made a payment to heirs which were not due
because a creditor had lodged its claim too late.
5. Where a bank has made payment in terms of a forged cheque.

Question 5:
Which of the following is/are (a) prerequisite(s) for a claim in terms of the condictio indebiti?

1. The enrichment was unlawful.


2. The defendant was unjustifiably enriched.
3. There was a causal link between the enrichment and impoverishment
4. The mistake must have been excusable
5. 2 and 3 and 4 are correct

Question 6:
A has sold uncut diamonds to B for an amount of R100,000 in contravention of statutory law. B has
paid the amount but before the diamonds could be delivered, it was confiscated by the police during
a raid of A's house. Which statement best explains the nature of the claim against A?

1. In circumstances like these a court may exercise an equitable judicial discretion to relax the par
delictum rule, depending on the relative turpitude of the parties' conduct.
2. B has a claim for damages against A due to a breach of contract.
3. B has a claim against A in terms of the condictio ob turpem vel iniustam causam because it is
unfair that he should lose his money and get nothing.
4. B has a claim against A in terms of the condictio sine causa specialis because there is no other
enrichment action at his disposal.
5. B has a claim for damages against A based on delict.

Question 7:
In which one of the following circumstances can the condictio sine causa specialis be used?

1. As a general enrichment action.


2. Where property is transferred on the grounds of a valid cause which later falls away.
3. Where a contract is terminated due to a resolutive condition.
4. Where property has been transferred in terms of an illegal agreement.
5. Where undue payment was made due to an excusable error.

Question 8-10:
The following facts are relevant for questions 8-10.

K is the owner of a farm adjacent to that of L. Unbeknown to K and L, K has been occupying part of
L's land due to a fence that was mistakenly put up 10 years ago. K has affected the following
improvements on that part of the farm: (a) built a dam at a cost of R30,000; (b) A luxury little lapa on
the edge of the dam at a cost of R100,000; (c) a borehole at a cost of R20,000; (d) Planted fruit trees
at a cost of R15,000 (e) Planted mealies which are almost ready to harvest at a cost of R60,000
(value R120,000).

During his tenure of the land he has harvested mealies worth R300,000 (production cost R250,000)
and fruit from the fruit trees sold at R55,000. L has now become aware of the true situation and
demands that K leaves the land.

Question 8:
Which statement best explains the nature of K's possession or occupation of the land?

1. K is a bona fide occupier of the land.


2. K is a mala fide occupier of the land.
3. K is a bona fide possessor of the land.
4. K is a lawful occupier of the land.
5. K is a lawful possessor of the land.

Answer: K’s occupation or possession is unlawful because he has no right to occupy the land.
Because K believes that he owns the land, he is not merely an occupier, but a possessor, therefore a
bona fide possessor. If he knew that the land did not belong to him, he would have been a mala fide
possessor. Make sure that you know the difference between occupiers, holders and possessors.

Question 9:
Which statement best explains the nature and extent of K's claim(s), if any?

1. K has an enrichment action for all of the expenses that he has incurred on the improvement of L's
land.
2. K has an enrichment action for the full amount of all the necessary and useful expenses he has
incurred.
3. K has a choice to claim either the amount of his expenses incurred or the value by which L's land
has been increased, whichever is more.
4. K has an enrichment action for the useful and necessary expenses he has incurred to the extent
that hose expenses have increased the value of L's land.
5. K has no claim for the mealies which have not been harvested yet as they now belong to L.

Answer: Bona fide possessors do not have acclaim for all expenses, only for necessary and useful
expenses. In certain exceptional circumstances they may have an additional claim for luxurious
expenses. The amount of the claim however is limited to the amount of the expenses or the increase
in the value of the land, whichever is less and not for the full amount of expenses incurred. K will
have a claim for the value of crops still standing on the property as this is regarded as a useful
improvement.

Question 10:
Which statement best explains the amounts that may be brought into account against K's claim, if
any?

1. L is not entitled to subtract anything from K's enrichment claim.


2. L is entitled to reduce the enrichment claim against him by subtracting the value of K's occupation
of the land.
3. L is entitled to reduce the enrichment claim against him by subtracting the value of the mealies
and fruit harvested by K and the value of K's occupation of the land.
4. L is entitled to reduce the enrichment claim against him by subtracting the value of the mealies
harvested by K and the value of K's occupation of the land.
5. L is entitled to reduce the enrichment claim against him by subtracting the value of the mealies
harvested by K minus the production costs.
Answer: The owner is entitled to subtract the value of fruits drawn by the unlawful possessor minus
production costs from the unlawful possessor’s enrichment claim in terms of the common law. The
owner can only claim the value of the occupation of the land from the unlawful occupier.

Question 1:
Which statement is most correct

1. The protection of good faith is the basis of estoppel


2. The basis of estoppel is to be found in a delictual action for misrepresentation
3. The basis of estoppel is the exceptio doli
4. Estoppel is often seen as a doctrine of the law of evidence
5. The basis of estoppel is the maxim nemo contra suum factum venire debet

Question 2:
A local authority mistakenly and in conflict with its own policy granted a licence in respect of certain
areas to X when, thereafter, it attempted to cancel the licence, X raised estoppel to prevent it from
doing so. Which statement most correctly reflects the position in regard to X’s reliance on estoppel?

1. X will not be successful with its reliance on estoppel because estoppel is not allowed by law
in instances where a local authority must carry out a statutory duty
2. X wil not be successful with its reliance on estoppel because X did not act to its detriment
3. X will probably be successful with its reliance on estoppel
4. X will not be successful with its reliance on estoppel because the city council did not make a
misrepresentation
5. Estoppel will always suceed where a statutory body attempts to revoke its own decision

Question 3:
Which element of estoppel do the facts in question 2 specifically relate to?

1. Misrepresentation
2. Fault
3. Prejudice
4. Causation
5. Premissible in law

Question 4:
Chose the most correct statement

1. In Van Ryn Wine and Spirit Co v Chandos Bar 1928 TPD 417 estoppel succeeded
2. In Van Ryn Wine and Spirit Co v Chandos Bar 1928 TPD 417 estoppel did not succeed
3. In Van Ryn Wine and Spirit Co v Chandos Bar 1928 TPD 417 one of the parties was not
prevented from proving dissensus because his conduct had been reasonable and without fault
4. 1 and 3
5. 2 and 3

Question 5:
Estoppel is most similar to which theory?

1. The declaration theory


2. The will theory
3. The reliance theory
4. The reception theory
5. None of the above

Question 6:
A has sold his television set to B for R2,000. The contract stipulates that ownership will only pass to B
after the last instalment of R200 has been paid. A has given a letter to B stating the following
“Herewith I, A, confirm that I have sold Sony TV set No 123321 to B” After a period of six months and
payment of R1,200 B wants to sell the set to C and shows C the letter of A. C who is very cautious,
first phones A who again confirms the sale to B. C buys the set from B for R1,500. Thereafter B fails
to make any further payments to A. A now claims back his TV set from C with a rei vindicatio. Which
statements provide the most correct explanation of the current legal position?

1. A has committed a misrepresentation to C by giving the misleading letter to B while he


should have realised that B could abuse the letter according to the decision in Grosvenor Motors
(Potchefstroom) LTD v Douglas 1956 3 SA 420 (A)
2. A has committed a misrepresentation to C by giving the letter to B
3. A has committed a misrepresentation to C by giving the letter to B and failing to inform C at
the time when C phoned him, that the TC set had not yet been paid in full
4. A misrepresentation cannot be made in silenece
5. A misrepresentation cannot be made by conduct

Question 7:
Assume the facts in Question 6. Indicate which statement most correctly states the position in regard
to the fault requirement.

1. The person relying on estoppel must at least allege and provide negligence in cases where
the loss of ownership is involved
2. The person relying onestoppel must at least allege and prove intent in cases wher a loss of
ownership is involved
3. Fauls is never required for successful reliance on estoppel
4. Faulit is always required for successful reliance on estoppel
5. None of the above.

Question 8:
Assume the facts in Question 6. Indicate which statement most correctly states the position in regard
to the causality requirement.
1. The misrepresentation by the person denying estoppel must have been the only cause of the
detrimental conduct of the person relying on estoppel
2. In terms of the “proximate cause” as applied by the courts, the misrepresentation by the
person denying the estoppel must have been the only cause of the detrimental conduct of the
person relying on estoppel
3. In terms of the “proximate cause” as applied by the courts, it is sufficient that the
misrepresentation by the person denying estoppel made a material contribution to the detrimental
conduct of the person relying on estoppel
4. The “proximate cause” as applied by the courts, includes only factual causality and not legal
causality
5. The courts use the conditio sine qua non test to determine causality in general

Question 9:
Assume the facts in Question 6. Indicate which statement most correctly states the position in regard
to the detriment requirement.

1. It is sufficient to prove that the person relying on estoppel has changed his position to his
detriment even if he cannot prove concrete damage suffered
2. It is not sufficient to prove that the person relying on estoppel has changed his position to his
detriment even if he cannot prove concrete damage suffered
3. The person relying on estoppel must prove that he has already suffered damage as a result of
the misrepresentation in all instances
4. The person relying on estoppel must prove that he has already suffered either patrimonial
damage or personal damage
5. None of the above.

Question 10:
Assume the facts in Question 6. Indicate which statement most correctly indicated whether C’s
reliance on estoppel will be successful?

1. C has acted to his detriment because he has concluded the contract with B, made payment
and now possinly stands to lose the TV set
2. C has not acted to his detriment because he has not suffered any patrimonial damage
3. C has not acted to his detriment by concluding the contract with B, because he still has a
claim for breach of contract against B
4. C cannot rely on estoppel at all in cases where ownership is at stake
5. C cannot rely on estoppel when he has a contractual claim against a third party

Question 11:
Choose the correct statement

1. In Fawden v Lelyfeld 1937 TPD a plea of estoppel did not succeed


2. In Grosvenor Motors (Potchefstroom) Ltd v Douglas 1956 3 SA 420 (A) a plea of estoppel did
not succeed
3. In Johaadien v Stanley Porter (Paarl) (Pty) Ltd 1970 1 SA 394 (A) a plea of estoppel succeeded
4. In Morum Bros v Nepgen 1916 CPA a plea of estoppel succeeded
5. In Adams v Mocke 23 SC 722 a plea of estoppel succeeded

ASSIGNMENT – 2010 – SEMESTER 1

Choose the most correct option in every instance. If there is more than one correct option, choose
the appropriate combined option.

Question 1:
Indicate which one of the following statements most correctly describes the existence
of a general enrichment action in South African law:

1. In Nortjé v Pool 1966 (3) SA 96 (A) the Appellate Division recognised the existence of a general
enrichment action in South African law.
2. In Nortjé v Pool 1966 (3) SA 96 (A) the Appellate Division rejected the existence of a general
enrichment action in South African law.
3. In Willis Faber Enthoven (Pty) Ltd v Receiver of Revenue 1992 (4) SA 202 (A) the Appellate Division
recognised the existence of a general enrichment action in South African law.
4. In Willis Faber Enthoven (Pty) Ltd v Receiver of Revenue 1992 (4) SA 202 (A) the Appellate Division
rejected the existence of a general enrichment action in South African law.
5. Although the Appellate Division rejected the existence of a general enrichment action in South
African law in Nortjé v Pool 1966 (3) SA 96 (A), the existence of such an action has since been
recognised in the case law.

Answer: See Study guide 1, par 14.3.

Question 2:
S has concluded a contract with P for the sale of his horse, Big Boy, at a price of R 50,000. P
immediately paid the purchase price to S. Unknown to both parties at the time of the conclusion of
the contract, Big Boy had died the day before the conclusion of the contract when he was kicked by
another horse. S immediately used the purchase price to buy a new young foal for R15,000, to pay
his workers weekly wages of R 8,000, to pay his overdraft of R 10,000 and to pay for a luxury
weekend away of R 12,000. There remains R 5,000 of the money in his savings account with the
bank. This contract of sale is, however, void due to initial impossibility.

Which statement best explains the basis of P’s claim against S?

1. P has an enrichment claim against S for repayment of the purchase price based on the condictio
ob turpem vel iniustam causam.
2. P has an enrichment claim against S for repayment of the purchase price based on the condictio
causa data causa non secuta.
3. P has an enrichment claim against S for repayment of the purchase price based on the condictio
sine causa specialis.
4. P has an enrichment claim against S for repayment of the purchase price based on the condictio
indebiti.
5. P has an enrichment claim against S for repayment of the purchase price based on the actio
negiotiorum gestorum utilis.

Answer : The condictio ob turpem vel iniustam causam is used in cases of unlawful contracts (see
Study guide 1, par 5.4). The condictio causa data causa non secuta is used in cases where the
contract of sale was entered into subject to a condition (see Study guide 1, par 6.4). The condictio
sine causa specialis is used in instances where performance was due, but the causa for performance
fell away at a later stage (see Study guide 1, par 7.4). The condictio indebiti is used in instances
where performance is rendered in the mistaken belief that it was due or owing at the time it is done
(see Study guide 1, par 3.4). The actio negotiorum gestorum utilis is used in instances where a person
is managing someone else’s affairs (see Study guide 1, par 8.4). In this instance the contract of sale is
not unlawful, but void due to initial impossibility. It is thus not a case of performance that was due at
the time of performance and the causa fell away later, but rather a case of mistaken belief that
performance was owing at the time it was made. The correct option is therefore 4.

Question 3:
Assume the same facts as in Question 2. Indicate which statement best explains the quantum of P’s
enrichment claim:

1. P has an enrichment claim for the full R50,000 paid.


2. P has an enrichment claim only for the R5,000 left in the savings account, the R15,000 paid for the
foal and the weekly wages paid of R8,000.
3. P has an enrichment claim only for the R5,000 left in S’s savings account.
4. P has an enrichment claim only for the R5,000 left in the savings account and the R15,000 paid for
the foal.
5. P has an enrichment claim only for the R 5,000 left in the savings account, the R 15,000 paid for
the foal, the weekly wages paid of R 8,000 and the R 10,000 paid on the overdraft.

Answer : P will claim the full R50 000, but S could raise the defence that part of the enrichment has
fallen away. If S used the money to pay debts and wages, the defence will not succeed, because he
saved expenses he would have had in any event. If S used the money to buy something he will be
regarded as enriched to the extent of the value of the goods. If S spend the money on something
that has no market value anymore, and it is not something he would have done if he was aware of
the true facts, his defence will succeed. In this case S has been enriched by the R5 000 left in his
savings account, the R15 000 paid for the foal if we assume that the value of the foal is still R15 000,
the weekly wages of R8 000 and the R10 000 of the overdraft. The luxury weekend has no market
value afterwards and S will succeed with the defence that the enrichment has fallen away in this
respect, unless he booked and planned the holiday irrespective of whether he sold Big Boy or not.
The correct option is thus 5.
Question 4:
Indicate which one of the following is not a correct statement in respect of the condictio ob turpem
vel iniustam causam:

1. A party who acted with knowledge of the unlawfulness of the contract can never have an
enrichment claim against the other party.
2. Performance by the plaintiff must have taken place as a result of an unlawful agreement.
3. The plaintiff must offer to return any performance received when lodging this enrichment action.
4. The court has an equitable discretion to “do justice between man and man” when dealing with
claims based on this enrichment action.
5. 1 and 4 are both wrong.

Answer : Options 2, 3 and 4 are correct – see Study Guide 1, par 5.4 and Jajhbay v Cassim. Option 1 is
incorrect because of the word “never”. In instances where the par delictum rule is relaxed, a party
who acted with knowledge of the unlawfulness of the contract may still succeed if the court decides
that the turpitude on the part of the plaintiff was less than that on the part of the defendant.

Question 5:
L is renting a farm from O for an amount of R 10,000 per month. Without notifying O, L concludes a
contract with R to build a new storeroom at a cost of R 100,000 and to make repairs to the roof of
the house on the farm at a cost of R 15,000 because the roof is leaking and causing damage to the
interior of the house.

Indicate which statement best explains L’s presence on O’s land:

1. L is a lawful occupier of the farm.


2. L is a bona fide occupier of the farm.
3. L is a bona fide possessor of the farm.
4. L is a mala fide occupier of the farm because he did not have the permission of O to effect the
improvements and repairs.
5. 1 and 3 are both correct.

Answer : L is a lessee and thus a lawful occupier. A bona fide and mala fide occupier are both
unlawfully on the premises and that is not the case here. A possessor is someone that acts as if he is
the owner, which is also not the case here. The correct option is thus 1.

Question 6:
Assume the same facts as in Question 5. Indicate which statement best explains L’s possible claim:

1. L as lawful occupier has an enrichment action against O for the value of all of the improvements
affected to the farm.
2. L as bona fide occupier has an enrichment action against O to the extent that the improvements
increased the value of the farm.
3. L as lawful occupier has an enrichment action against O for only the value of the necessary
improvements effected to the farm, i.e. the repairs to the roof.
4. L as lessee of rural land has no claim against O in terms of the Roman-Dutch Placaaten that still
applies in South African law.
5. L as lessee of rural land has a claim against O in terms of the Roman-Dutch Placaaten that still
applies in South African law.

Answer : Because L is a lawful occupier, not a bona fide occupier, option 2 is incorrect immediately.
As a lessee of rural land the Placaaten will apply (see Study guide 1, par 10.2.1) and L will not have a
claim for his necessary and useful improvements because it was made without the permission of the
owner. The correct option is thus 4.

Question 7:
Assume the same facts as in Question 5. Assume further that L has absconded after the
improvements were effected and cannot be found as he has apparently emigrated.

Indicate which statement best explains the case law on whether R will have a claim against O under
these circumstances:

1. In Buzzard Electrical v 158 Jan Smuts Avenue Investments 1996 (4) SA 19 (A) it was held that under
these circumstances R has a claim against O for the value of the improvements made to the farm.
2. In Buzzard Electrical v 158 Jan Smuts Avenue Investments 1996 (4) SA 19 (A) the question on
whether R has a claim against O for the value of the improvements made to the farm under these
circumstances, was left undecided.
3. In Gouws v Jester Pools (Pty) Ltd 1968 (3) SA 563 (T) it was held that under these circumstances R
has a claim against O for the value of the improvements made to the farm.
4. In Gouws v Jester Pools (Pty) Ltd 1968 (3) SA 563 (T) it was held that under these circumstances R
has no claim against O for the value of the improvements made to the farm.
5. 2 and 4 are both correct.

Answer: See Study guide 1, par 2.2.3. Options 2 and 4 are both correct.

Question 8:
B has bought an operating business from S for R 1.5 million. After B had taken over the running of the
business, X, a major supplier to the business, refuses to supply B with any product until S has settled
a debt owed to X for goods delivered in an amount of R 50,000. B pays S’s debt with X because he
cannot operate the business without the product supplied by X. S refuses to repay the amount to B.

Indicate which statement best explains which enrichment action, if any, is available to B against X:

1. B has no claim against X in terms of any enrichment action.


2. B has a claim against X based on the condictio indebiti.
3. B has a claim against X based on the condictio sine causa specialis.
4. B has a claim against X based on the actio negiotiorum gestorum utilis (extended management of
affairs action).
5. B has a claim against X based on the actio negiotiorum gestorum contraria (true management of
affairs action).

Answer: B’s payment is not in terms of an unlawful, void or conditional contract. B also does not have
the intention to manage X’s affairs. B is paying a debt owed to X and therefore B has no enrichment
claim against X.

Question 9:
Assume the same facts as in question 8. Indicate which statement best explains which enrichment
action, if any, is available to B against S.

1. B has a claim against S based on the condictio indebiti.


2. B has a claim against S based on the condictio sine causa specialis.
3. B has a claim against S based on the actio negiotiorum gestorum utilis (extended management
of affairs action).
4. B has a claim against S based on the actio negiotiorum gestorum contraria (true management of
affairs action).
5. B has no claim against S because he is not entitled to meddle in the affairs of S without the
permission of S.

Answer: As already stated, B’s payment is not in terms of an unlawful, void or conditional contract. B
is paying a debt owed by S knowing that he (B) doesn’t owe the debt and therefore the condictio
indebiti will not be available. B’s payment is made to advance his own interests and therefore the
true management of affairs action will not be available to him. In Odendaal v Van Oudtshoorn 1968 3
SA 433 (T) the court held that the extended management of affairs action will be available in these
circumstances. See Study guide 1, par 8.4.2. The correct option is thus 3.

Question 10:
Assume the same facts as in Question 8. Further assume that the reason why S refused to pay X was
because the goods delivered were defective. S had a valid claim for the reduction of the purchase
price of those goods in an amount of R13,000.

Indicate which statement best explains the quantum of B’s claim, if any:
1. B has a claim against S for the full amount of R 50,000.
2. B has a claim against S for only R 37,000.
3. B has a claim against X for the full amount of R 50,000.
4. B has no claim against S because he is not entitled to meddle in the affairs of S without the
permission of S.
5. 3 and 4 are both correct.

Answer : B will claim the full R50 000. S will raise the defence that his enrichment was for a lesser
amount, because he had a valid claim for the reduction of the purchase price.
ASSIGNMENT – 2010 – SEMESTER 2

Choose the most correct option in every instance. If there is more than one correct option, choose
the appropriate combined option.

Question 1:
Which one of the following statements cannot be regarded as a general requirement for enrichment
liability?

1. The plaintiff must have been impoverished.


2. The enrichment must have taken place without a justifiable cause.
3. The enrichment must have taken place unlawfully.
4. The defendant must have been enriched.
5. The plaintiff is only entitled to the lesser of his impoverishment and the enrichment of the
defendant.

Question 2:
In order to be successful with a claim based on the condictio indebiti, the plaintiff must prove the
following fact(s) or requirement(s):

1. That the impoverished party made a payment that was not due.
2. That the enrichment was unlawful.
3. That the mistake of the impoverished party was excusable.
4. 1 and 3 are correct.
5. 1 and 2 and 3 are correct.

Question 3:
A has paid B an amount of R 40,000 by cheque. Before B could present the cheque to his bank, A
countermanded the cheque because B had delivered defective goods to him. X, a clerk at A’s bank
failed to notice the countermand notice and payment of the amount was made to B.

Indicate which statement best explains the nature of the possible claims by A or the bank:

1. B has been enriched at the expense of the bank, because the bank had no mandate to make a
payment from A’s account.
2. B has been enriched at the expense of A, from whose account the payment was made.
3. A has an enrichment claim against B for the full amount of R 40,000.
4. A has an enrichment claim against B for a reduced amount.
5. 2 and 4 are both correct.
Question 4:

E is an employee of M. E is paid a monthly salary of R 20,000. On 15 June 2009 M summarily


dismissed E because of theft of company assets. The dismissal was lawful in terms of the
employment contract and employment law.

Indicate which statement best explains the possible claim that E might have against his employer:

1. E has no claim for any part of his salary.


2. E has a contractual claim for the full amount of his salary for June 2009.
3. E has a pro rata claim for half of his salary of June 2009 based on the principle of unjustified
enrichment.
4. E has a contractual claim for a pro rata part of his salary for June 2009.
5. E has a claim for the full amount of his salary for June 2009 based on the principle of unjustified
enrichment.

A, an American tourist, has leased a vehicle from B. While travelling in the Northern Cape, the
vehicle breaks down. A contracts with C, a garage in Springbok, to repair the vehicle at a cost of
R12,000. After two days A leases another vehicle from X and completes his trip. He departs for
America. C wants to claim the R12,000 from B.

Question 5:
Which statement best explains whether C has a claim against B and the authority on which it is
based?

1. In terms of the decision in Gouws v Jester Pools (Pty) Ltd 1968 3 SA 63 (T) it was held that C has no
claim against B because B had not been enriched.
2. In terms of the decision in Gouws v Jester Pools (Pty) Ltd 1968 3 SA 63 (T) it was held that C has
no claim against B because B has not been enriched at C's expense.
3. The decision in the Gouws case was confirmed in Buzzard Electrical v 158 Jan Smuts Avenue
Investments 1996 4 SA 19 (A).
4. The decision in the Gouws case was rejected in Buzzard Electrical v 158 Jan Smuts Avenue
Investments 1996 4 SA 19 (A).
5. The decision in the Gouws case was overruled in Brooklyn House Furnishers Ltd v Knoetze & Sons
1970 3 SA 264 (A).

Question 6:
Which statement best explains whether C has a retention right or whether he can exercise it?

1. C can exercise a retention right over the vehicle against B until such time as it has been paid for its
necessary expenses.
2. C can exercise a retention right over the vehicle against B until it has been paid the full contract
price.
3. In terms of the decision in Buzzard Electrical v 158 Jan Smuts Avenue Investments 1996 4 SA 19 (A)
C has no retention right because it has no enrichment claim against B.
4. An enrichment retention right is a personal right and can therefore be exercised only against the
creditor.
5. C has no retention right under these circumstances.

Question 7:
A has sold uncut diamonds to B for an amount of R100,000 in contravention of statutory law. B has
paid the amount but before the diamonds could be delivered, it was confiscated by the police during
a raid of A's house. Which statement best explains the nature of the claim against A?

1. In circumstances like these a court may exercise an equitable judicial discretion to relax the par
delictum rule, depending on the relative turpitude of the parties' conduct.
2. B has a claim for damages against A due to a breach of contract.
3. B has a claim against A in terms of the condictio ob turpem vel iniustam causam because it is
unfair that he should lose his money and get nothing.
4. B has a claim against A in terms of the condictio sine causa specialis because there is no other
enrichment action at his disposal.
5. B has a claim for damages against A based on delict.

Question 8:
In which one of the following circumstances can the condictio sine causa specialis be used?

1. As a general enrichment action.


2. Where property is transferred on the grounds of a valid cause which later falls away.
3. Where a contract is terminated due to a resolutive condition.
4. Where property has been transferred in terms of an illegal agreement.
5. Where undue payment was made due to an excusable error.

Question 9:
Which statement correctly explains the possession or occupation of another's property?

1. A bona fide occupier is someone who lawfully occupies the immovable property of another
person.
2. A bona fide occupier is someone who unlawfully occupies the immovable property of another
person as if he is the owner thereof.
3. A bona fide possessor is someone who lawfully occupies the property of another person as if he is
the owner thereof.
4. A bona fide possessor is someone who unlawfully occupies the property of another person as if
he is the owner thereof.
5. A mala fide possessor is someone who unlawfully occupies the property of another person
temporarily as if he is entitled to occupy the property as a lessee.
Question 10:
Which statement best explains the legal position on the recognition of a general enrichment action in
South African law?

1. In Nortje v Pool 1966 3 SA 96 (A) the Appellate Division recognised the existence of a general
enrichment action in South Africa without any qualifications.
2. In Nortje v Pool 1966 3 SA 96 (A) the Appellate Division recognised the existence of a general
enrichment action in South Africa, but with certain qualifications.
3. In Kommissaris van Binnelandse Inkomste v Willers 1994 3 SA 283 (A) the Appellate Division
recognised the existence of a general enrichment action in South Africa.
4. In Kommissaris van Binnelandse Inkomste v Willers 1994 3 SA 283 (A) the Appellate Division
rejected the existence of a general enrichment action in South Africa but recognised that courts
can extend enrichment liability to circumstances where it is deemed necessary.
5. In Kommissaris van Binnelandse Inkomste v Willers 1994 3 SA 283 (A) the Appellate Division
rejected the existence of a general enrichment action in South Africa and also rejected the idea that
courts can extend enrichment liability to circumstances where it is deemed necessary.

ASSIGNMENT – 2008 – SEMESTER 1

Question 1 and 2
The following facts are relevant for Questions 1 and 2

A, an American tourist, has leased a vehicle from B. While travelling in the Northern Cape, the
vehicle breaks down. A contracts with C, a garage in Springbok, to repair the vehicle at a cost of
R12,000. After two days A leases another vehicle from X and completes his trip. He departs for
America. C wants to claim the R12,000 from B.

Answer:
This question deals with the problem of so-called indirect enrichment and causality or the
requirement that the enrichment must have been at the expense of the impoverished party. The
relevant case law here is Gouws v Jester Pools (Pty) Ltd 1968 3 SA 63 (T) and Buzzard Electrical v 158
Jan Smuts Avenue Investments 1996 4 SA 19 (A). See also New Club Garage
v Millborrow 1931 GWL 86.

Question 1:
Which statement best explains whether C has a claim against B and the authority on which it is
based?

1. In terms of the decision in Gouws v Jester Pools (Pty) Ltd 1968 3 SA 63 (T) it was held that C has no
claim against B because B had not been enriched.
2. In terms of the decision in Gouws v Jester Pools (Pty) Ltd 1968 3 SA 63 (T) it was held that C has
no claim against B because B has not been enriched at C's expense.
3. The decision in the Gouws case was confirmed in Buzzard Electrical v 158 Jan Smuts Avenue
Investments 1996 4 SA 19 (A)
4. The decision in the Gouws case was rejected in Buzzard Electrical v 158 Jan Smuts Avenue
Investments 1996 4 SA 19 (A)
5. The decision in the Gouws case was overruled in Brooklyn House Furnishers Ltd v Knoetze & Sons
1970 3 SA 264 (A)

Answer: In the Gouws case it was held that the owner of the property, B, was enriched, but not at
the expense of C, but rather at the expense of A because C had a contractual claim against A. The
fact that that claim existed in theory only, did not change the courts view on this aspect. In the
Buzzard case this issue was not dealt with. The court distinguished between the Gouws type of cases
and cases where one is dealing with main contractors and subcontractors as situation which should
be dealt with differently. It only made a decision on the sub-contractor type cases, and refrained
from rejecting or confirming the Gouws decision. In the Brooklyn House case, the appellate court
came to a different conclusion to the Gouws case, but did not overturn the Gouws case as the court
only dealt with the enrichment lien and not the existence of the underlying enrichment claim.

Question 2:
Which statement best explains whether C has a retention right or whether he can exercise it?

1. C can exercise a retention right over the vehicle against B until such time as it has been paid for
its necessary expenses.
2. C can exercise a retention right over the vehicle against B until it has been paid the full contract
price.
3. In terms of the decision in Buzzard Electrical v 158 Jan Smuts Avenue Investments 1996 4 SA 19 (A)
C has no retention right because it has no enrichment claim against B.
4. An enrichment retention right is a personal right and can therefore be exercised only against the
creditor.
5. C has no retention right under these circumstances.

Answer: In terms of the decision in the Brooklyn House case, C has an enrichment lien or retention
right over the property until such time as it has been paid for its impoverishment. C can therefore
not claim the full contract price from B. It only has a lien for the full contract price against A with
whom it had concluded the contract. The issue whether C has an enrichment lien against the owner
was left open in the Buzzard case. An enrichment retention right is a real right which can be
exercised against the world. A contractual lien is only a personal right which can be exercised against
the other contractual party only.

Question 3
In which one of the following circumstances can the condictio indebiti be used?

1. Where a person has made a payment in terms of a contract subject to a suspensive condition, and
the contract has now been extinguished due to the condition being fulfilled.
2. Where a person has made a payment in terms of a contract subject to a resolutive condition, and
the contract has now been extinguished due to the condition being fulfilled.
3. Where an undue payment has been made in circumstances where the mistake is not excusable.
4. Where an executor who is now functus officio has made a payment to heirs which were not due
because a creditor had lodged its claim too late.
5. Where a bank has made payment in terms of a forged cheque.

Answer: In the first two instances the correct enrichment action is the condictio causa data causa
non secuta. Where the mistake on which the impoverished party is relying, was not excusable under
the circumstances, the condictio indebiti is excluded. Where executors have made payments that are
not due and they have been discharged from their duties in the ordinary course, it is recognised that
the condictio indebiti is the appropriate enrichment action to use. Where payments have been made
in terms of a forged cheque the appropriate action according to case law is the condictio sine causa
specialis.

Question 4
Which of the following is/are (a) prerequisite(s) for a claim in terms of the condictio indebiti?
1. The enrichment was unlawful.
2. The defendant was unjustifiably enriched.
3. There was a causal link between the enrichment and impoverishment
4. The mistake must have been excusable
5. All of the above are correct
6. 2 and 3 and 4 are correct

Answer: Your mark sheet only allows 5 answers. This question has therefore been disregarded in the
calculation of your year mark. It is not a requirement for any of the enrichment actions that the
enrichment must have been unlawful, rather it must have been sine causa or without recognised
cause. All of the other requirements mentioned above are either general requirements which have
to be met first, or specific requirements of the condictio indebiti.

Question 5
A has sold uncut diamonds to B for an amount of R100,000 in contravention of statutory law. B has
paid the amount but before the diamonds could be delivered, it was confiscated by the police during
a raid of A's house. Which statement best explains the nature of the claim against A?

1. In circumstances like these a court may exercise an equitable judicial discretion to relax the par
delictum rule, depending on the relative turpitude of the parties' conduct.
2. B has a claim for damages against A due to a breach of contract.
3. B has a claim against A in terms of the condictio ob turpem vel iniustam causam because it is
unfair that he should lose his money and get nothing.
4. B has a claim against A in terms of the condictio sine causa specialis because there is no other
enrichment action at his disposal.
5. B has a claim for damages against A based on delict.
Answer: The contract between A and B is void due to illegality; there can therefore be no question of
a contractual claim. B does not have a claim in terms of the condictio ob turpem unless he is not a
turpis persona. However, courts do have an equitable discretion in terms of the common law as
developed by the case law to allow a claim under this condictio based on principles of fairness. A has
not committed any delict under these circumstances. Because these facts fall squarely within the
scope of application of the condictio ob turpem, the condictio sine causa specialis cannot be used.

Question 6
In which one of the following circumstances can the condictio sine causa specialis be used?

1. As a general enrichment action.


2. Where property is transferred on the grounds of a valid cause which later falls away.
3. Where a contract is terminated due to a resolutive condition.
4. Where property has been transferred in terms of an illegal agreement.
5. Where undue payment was made due to an excusable error.

Answer: Courts have repeatedly stated that this action is not a general enrichment action in disguise,
but has its own field of application. Where a contract is terminated due to a resolutive condition, the
appropriate action is the condictio causa data. Where property has been transferred due to an illegal
agreement, the appropriate action is the condictio ob turpem. In the last instance the appropriate
action is the condictio indebiti.

The following facts are relevant for questions 7-9.

K is the owner of a farm adjacent to that of L. Unbeknown to K and L, K has been occupying part of
L's land due to a fence that was mistakenly put up 10 years ago. K has effected the following
improvements on that part of the farm: (a) built a dam at a cost of R30,000; (b) a luxury little lapa on
the edge of the dam at a cost of R100,000; (c) a borehole at a cost of R20,000; (d) planted fruit trees
at a cost of R15,000 (e) planted mealies which are almost ready to harvest at a cost of R60,000 (value
R120,000). During his tenure of the land he has harvested mealies worth R300,000 (production cost
R250,000) and fruit from the fruit trees sold at R55,000. L has now become aware of the true
situation and demands that K leaves the land.

Answer: These facts deal with the enrichment action of a possessor or occupier of land and the
improvements effected to that property.

Question 7
Which statement best explains the nature of K's possession or occupation of the land?

1. K is a bona fide occupier of the land.


2. K is a mala fide occupier of the land.
3. K is a bona fide possessor of the land.
4. K is a lawful occupier of the land.
5. K is a lawful possessor of the land.
Answer: K’s occupation or possession is unlawful because he has no right to occupy the land.
Because K believes that he owns the land, he is not merely an occupier, but a possessor, therefore a
bona fide possessor. If he knew that the land did not belong to him, he would have been a mala fide
possessor. Make sure that you know the difference between occupiers, holders and possessors.

Question 8:
Which statement best explains the nature and extent of K's claim(s), if any?

1. K has an enrichment action for all of the expenses that he has incurred on the improvement of L's
land.
2. K has an enrichment action for the full amount of all the necessary and useful expenses he has
incurred.
3. K has a choice to claim either the amount of his expenses incurred or the value by which L's land
has been increased, whichever is more.
4. K has an enrichment action for the useful and necessary expenses he has incurred to the extent
that those expenses have increased the value of L's land.
5. K has no claim for the mealies which have not been harvested yet as they now belong to L.
6. K has no claim because he was on the land unlawfully.

Answer: Bona fide possessors do not have acclaim for all expenses, only for necessary and useful
expenses. In certain exceptional circumstances they may have an additional claim for luxurious
expenses. The amount of the claim however is limited to the amount of the expenses or the increase
in the value of the land, whichever is less and not for the full amount of expenses incurred. K will
have a claim for the value of crops still standing on the property as this is regarded as a useful
improvement.

Question 9:
What statement best explains the amounts that may be brought into account against K's claim, if
any?

1. L is not entitled to subtract anything from K's enrichment claim.


2. L is entitled to reduce the enrichment claim against him by subtracting the value of K's occupation
of the land.
3. L is entitled to reduce the enrichment claim against him by subtracting the value of the mealies
and fruit harvested by K and the value of K's occupation of the land.
4. L is entitled to reduce the enrichment claim against him by subtracting the value of the mealies
harvested by K and the value of K's occupation of the land.
5. L is entitled to reduce the enrichment claim against him by subtracting the value of the mealies
harvested by K minus the production costs.
6. L is entitled to reduce the enrichment claim against him by subtracting the full value of the mealies
harvested by K.
Answer: The owner is entitled to subtract the value of fruits drawn by the unlawful possessor minus
production costs from the unlawful possessor’s enrichment claim in terms of the common law. The
owner can only claim the value of the occupation of the land from the unlawful occupier.

Question 10
Which statement best explains the legal position on the recognition of a general enrichment action in
South African law?

1. In Nortje v Pool 1966 3 SA 96 (A) the Appellate Division recognised the existence of a general
enrichment action in South Africa without any qualifications.
2. In Nortje v Pool 1966 3 SA 96 (A) the Appellate Division recognised the existence of a general
enrichment action in South Africa, but with certain qualifications.
3. In Kommissaris van Binnelandse Inkomste v Willers 1994 3 SA 283 (A) the Appellate Division
recognised the existence of a general enrichment action in South Africa.
4. In Kommissaris van Binnelandse Inkomste v Willers 1994 3 SA 283 (A) the Appellate Division
rejected the existence of a general enrichment action in South Africa but recognised that courts
can extend enrichment liability to circumstances where it is deemed necessary.
5. In Kommissaris van Binnelandse Inkomste v Willers 1994 3 SA 283 (A) the Appellate Division
rejected the existence of a general enrichment action in South Africa and also rejected the idea that
courts can extend enrichment liability to circumstances where it is deemed necessary.

Answer: In the Nortje case the court rejected the existence of a general enrichment action, but
intimated that such an action may develop in future. In the Willers case the appellate division
confirmed its stance on the general enrichment action, but stated that the existing actions may be
extended in future to cover circumstances where an enrichment action had previously not been
available.

PVL3702_May_2016

QUESTION 1
A real agreement is an agreement

1 only creating obligation


2 whereby a right IS transferred
3 only extinguishing a debt
4 an example of which is release or discharge
5 that cannot entail the transfer of ownership of property

QUESTION 2
Which of the following is NOT a requirement for the creation of a valid contract?

1 There must be agreement or apparent agreement between the parties


2 The contracting parties must have capacity to act
3 The performance must be possible after the contract is concluded
4 The contract must be legal
5 The prescribed formalities must be complied with

QUESTION 3
Portia and Lucas are engaged and living together. They want to purchase Jerome's car Jerome offers
to sell his car to Lucas for R100 000. Lucas and Portia accept Jerome's offer. Which statement is
CORRECT?

1 Only Lucas could accept the offer


2 Either Lucas or Portia could accept the offer as they are engaged
3 Since the offer was only made to Lucas, the acceptance from both Portia and Lucas Will never
constitute a counter-offer
4 A valid contract has been concluded, because Lucas is one of the parties that accepted the offer
5 None of the above statements

QUESTION 4
X orally offers to sell her painting to Z for R20 000 on 1 April In terms of the offer X stipulates that her
offer will lapse on 30 April On 15 April X notifies Z orally that she (X) revokes her offer On 25 April Z
notifies X orally that he (Z) accepts the offer Which statement reflects the CORRECT legal position?

1 A right of pre-emption exists


2 The revocation by X IS valid, because an offer can be revoked where an option was given
3 An option exists because X offered to keep her offer open unt1l 30 April, wh1ch was accepted on
25 April
4 The revocat1on by X is valid because no option contract was concluded
5 The option is not valid because in Brandt v Sp1es 1960 (4) SA 14 (E) 1t was held that such an option
must be in writing

QUESTION 5
A right of pre-emption

1 is a right acquired by a contract


2 is a right acquired by an offer
3 is a right acquired in terms of an option
4 is a preferential right ent1thng the grantee to compel the grantor to sell the thing in question at
any time
5 encapsulates all of the above statements

QUESTION 6
An innocent misrepresentation renders a contract
1 Void
2 void without a claim for damages
3 voidable
4 unenforceable
5 neither void nor Voidable, but 1t does give rise to a claim for damages

QUESTION 7
X places an advertisement in the newspaper in which he offers to sell his car, a Porsche Carrera, for
R800 000 X states that the Porsche IS a 1995 model in the advertisement, while it is in fact a 1994
model which has been registered for the first time in 1995 X knows that that this statement in the
advertisement is untrue Y reads the advertisement, phones X and concludes a contract of sale with X.
The statement in the advertisement regard1ng the year model amounts to

1 fraud
2 mere puffing
3 a dictum et promissum
4 a guarantee in the contract
5 both fraud and a dictum et promissum

QUESTION 8
Which statement(s) is/are CORRECT?

1 Commercial bribery amounts to an abuse of Circumstances


2 Commercial bribery amounts to misrepresentation
3 Commercial bribery can lead to a cla1m for the setting aside of the contract and restitution
4 Option 1 and 3
5 Option 2 and 3

QUESTION 9
John agrees with Michael, that Michael Will paint John's holiday home at the coast for R30 000
Unbeknown to either of them the house had been destroyed in a storm the previous day. This is a
case of

1 mutual mistake
2 supervening impossibility of performance
3 impossibility of performance
4 an obligation subject to a resolutive condition
5 an obligation subject to a suspensive condition

QUESTION 10
Henry was arrested in a police trap for committing a crime when he sold drugs to a policeman for
R500 without authorisation. Both the drugs and the R500 were delivered and ownership of both
passed. The contract between Henry and the policeman IS
1 voidable because of misrepresentation on the part of Henry
2 void for vagueness
3 valid but unenforceable
4 voidable on the basis of illegality
5 void on the basis of illegality

QUESTION 11
The State may claim the R500 from Henry in question (10) with
1 a contractual action
2 an enrichment action
3 a delictual action
4 the rei vendicatio (real action of the owner)
5 a statutory action

QUESTION 12
X and Y conclude a contract. When will their contract Will be VOIDABLE?

1 X and Y have concluded a gambling contract which is prohibited by legislation


2 X's performance becomes absolutely impossible without his fault after the contract has been
concluded
3 The contract IS subject to a suspensive condition which has not yet been fulfilled
4 Y concluded the contract under undue Influence from X
5 The contract IS subject to a resolutive condition wi1ch has been fulfilled

QUESTION 13
X and Y conclude a contract, in terms of which X rents a house from Y for R2 000 a month until she
finds work in Cape Town The contract is subject to a

1 suspensive condition
2 resolutive condition
3 suspensive time clause
4 resolutive time clause
5 modus

QUESTION 14
X inherits a watch which Y is interested in buying Both X and Y believe that the watch to be a genuine
Rolex and know of each other's belief. When they conclude the contract, Y tells X that he is only
buying the watch because it is a Rolex However, it turns out to be only an imitation. This is most
probably a case of a
1 resolutive condition
2 error in motive
3 suspensive condition
4 common mistake
5 option 1 and 4

QUESTION 15
Koos and Ronald are owners of champion horses. They regularly buy horses from each other for
breeding purposes on one occasion Koos purchased a stallion from Ronald It turned out that the
stallion had a rare and deadly disease at the time of the purchase. Within two months of the
purchase, the stallion died from the illness Koos wishes to cancel the contract and claim back the
purchase price (R400 000) he paid, on the basis that Ronald breached a material term of the contract
that "the stallion will be free from any illness that is deadly" From the facts given, the material term
relied upon by Koos, could be
1 a tacit term or a naturalium (singular of naturalia}
2 an express term or an incidentalium (singular of incidentalia)
3 a term implied by law or an essentialium (singular of essentialia)
4 a naturalium or an express term
5 none of the above

QUESTION 16 / VRAAG 16
Assume the same facts in question (15) Assume further that the material term relied upon by Koos
does form part of the contract between Koos and Ronald as an express term The breach of this term
by Ronald is in the form of

1 positive mal-performance
2 negative mal-performance
3 reputation
4 prevention of performance
5 supervening impossibility of performance

QUESTION 17
Assume the same facts in questions (15) and (16) The market value of stallion at the time of its death
was R500 000 Consequently, Koos wants to claim R500 000 damages from Ronald Koos is likely to
succeed based on the application of the

1 rules regarding special damages


2 rules regarding general damages
3 rules regarding special and general damages
4 contemplation principle
5 convention principle

QUESTION 18 I VRAAG 18
X leases a video camera from Y, to be delivered on 23 March Y falls to deliver the camera on this day.
Which form of breach of contract has taken place?

1 Mora creditoris and mora ex re


2 Mora debitoris and mora ex persona
3 Mora creditoris and mora ex persona
4 Mora debitoris and mora ex re
5 Prevention of performance
QUESTION 19
Y lets premises to X for R10 000 per month The rental has to be paid in advance on the first day of
each month The written contract of lease conta1ns a clause requiring that any variation of any of the
terms of the lease (including this clause) has to be in writing and signed by both parties Later Y and X
agree orally that X may pay the rental on the third day of each month Y accepts payment for two
months on the third of the month Y then changes his mind and informs X that X should vacate the
premises because X has breached the contract by not paying the rental on the first of each month
Which answer reflects the CORRECT legal position?

1 X does not breach the contract, because the oral agreement constitutes a binding novation
2 X breaches the contract, because the oral agreement is not legally binding
3 X does not breach the contract, because the oral agreement constitutes a binding compromise
4 Y breaches the oral agreement and therefore X IS entitled to remain on the premises
5 X breaches the contract of lease and Y breaches the subsequent oral agreement. Therefore both
parties are in equal guilt, entitling X as the possessor to remain on the premises

QUESTION 20
Liability for which form(s) of misrepresentation may not be contractually excluded?

1 Innocent misrepresentation
2 Negligent misrepresentation
3 Grossly negligent misrepresentation
4 Fraudulent misrepresentation
5 Option 3 and 4

QUESTION 21
C, a builder, and O agree that C Will build a house on O's stand for R800 000, payable on completion
of the house The house must be completed by 30 April, but on that date the house is only 90%
complete. It would cost R1 00 000, for 0 to complete the house 0O refuses to pay any amount to C
because C failed to complete the house timeously If C institutes legal proceedings against 0 for
payment, which defence is O likely to raise?

1 The exceptio doll


2 The replicatio doll
3 The stipulatio alteri
4 The exceptio non adimpleti contractus
5 The pactum in favorem tertu

QUESTION 23
Assume the same facts in question (21) Which answer reflects the CORRECT legal position?

1 As C did not complete the house timeously, the rule IS absolute that C is not entitled to any
payment
2 C is entitled to a reduced counter performance, quantified as R720 000 {90% of R800 000) provided
inter alia that it is equitable
3 C IS entitled to a reduced counter performance, quantified as R700 000, provided inter alia that O
is utilising the defective performance
4 Because C breached the contract, O can use her discretion to decide what amount to pay C
provided inter alia that it is a reasonable amount
5 C is entitled to a reduced counter performance, quantified as the difference between the market
value the house would have had in its completed state and the current market value of the house in
its uncompleted state

QUESTION 24
X and Y conclude a contract under Circumstances where1n X threatened Y Th1s threat may be used
to prove an element of

1 duress
2 undue Influence
3 misrepresentation
4 contractual illegality
5 material mistake

QUESTION 25
X advertises her car for sale in the newspaper for the amount of R40 000 Y reads the advertisement,
phones X and accepts her advertisement to sell her car. Which statement most probably reflects the
CORRECT legal position?

1 X and Y concluded a contract because an offer and acceptance exsts


2 X made an offer to the public at large
3 Y IS the offeree
4 The advertisement IS an invitation to do business
5 Option 1 , 2 and 3

PVL3702_201_2013

Question 1
Generally, an advertisement is

1 an offer to the public


2 an invitation to do business
3 an offer directed at defined persons
4 an offer directed at undefined persons
5 1 and 4.

Question 2
X makes a written offer to Y to purchase Y’s house. X also states in the offer that Y has until 15 May to
accept the offer. X has the offer send by courier to Y. Y reads the offer on 13 May and drafts an
acceptance on 14 May. Y posts his acceptance on 15 May to X. X receives the acceptance on 17 May
and reads it on 18 May. Which theory must be applied to determine whether a contract arose?

1 The declaration theory.


2 The expedition theory.
3 The reliance theory.
4 The reception theory.
5 The information theory.

Question 3
Which theory finds application under the Electronic Communications and Transactions Act 25 of
2002?

1 The declaration theory.


2 The expedition theory.
3 The information theory.
4 The reception theory.
5 The reliance theory.

Question 4
Sibongile is desperate to sell her house because of a termite infestation. Consequently she knowingly
conceals all signs of damage when a potential purchaser, Thandi, comes to inspect the house and
furthermore tells Thandi that there is nothing wrong with the house. Thandi purchases the house,
which she would never have done if she knew of the termite infestation. What cause of action will
Thandi be able to rely on in the circumstances?
(a) Dictum et promissum.
(b) Innocent misrepresentation.
(c) Fraudulent misrepresentation.
(d) Material mistake.

1 (a), (b) and (c).


2 (a) and (c).
3 (b) and (c).
4 Only (c).
5 Only (d).

Question 5
Assume the same facts as in question (4). What remedy or remedies are available to Thandi when
she discovers the termite infestation and damage to the house?

1 Thandi may only rescind the contract.


2 Thandi may only uphold the contract and claim damages.
3 Thandi may rescind the contract and claim damages.
4 Thandi may uphold the contract and claim damages.
5 Both 3 and 4.
Question 6
Tony operates a general dealer business. From this business, Tony supplies Samuel with prohibited
drugs, which both knew was illegal. Subsequently when payment for the drugs is due, Samuel
refused to pay Tony. Any potential claim that Tony pursues for the return of the drugs, is based on

1 contractual principles.
2 delictual principles.
3 principles of unjustified enrichment.
4 principles of statutory illegality.
5 principles of freedom of trade.

Question 7
Assume the same facts in question (6). To reclaim the drugs, Tony will ask for a court order to

1 enforce the ex turpi rule.


2 relax the ex turpi rule.
3 enforce the par delictum rule.
4 partially relax both the ex turpi rule and the par delictum rule.
5 relax the par delictum rule.

Question 8
Assume the same facts in question (6). Which statement (s) is/are CORRECT?
(a) Tony cannot recover the money for the drugs from Samuel, because no action arises from an
illegal contract.
(b) It is unlikely that Tony will succeed in reclaiming the drugs, because both parties appear to be
equally guilty.
(c) In deciding whether to award Tony the return of the drugs, a court will consider public policy and
whether justice will be done between man and man.
(d) In deciding whether to order Samuel to pay Tony for the drugs supplied, a court will consider the
contractual principles relating to restraint of trade.

1 (a), (b), (c) and (d).


2 (a), (b) and (c).
3 (a) and (c).
4 Only (a).
5 Only (b).

Question 9
Y and Z reach an agreement that should Y sell his leather couch, he (Y) will offer to sell it to Z first,
before making an offer to sell the couch to any other person. Which statement is
CORRECT?

1 Y and Z concluded an option contract.


2 Y and Z concluded neither an option contract nor a pre-emption contract.
3 Y and Z concluded a contract subject to a resolutive condition.
4 Y and Z concluded an option contract and a pre-emption contract.
5 Y and Z concluded a pre-emption contract.

Question 10
Which of the following is NOT a requirement for the creation of a valid contract?
1 There must be agreement or ostensible agreement between the parties.
2 The contracting parties must have capacity to act.
3 The performance must be possible after the contract is concluded.
4 The contract must be legal.
5 Constitutive formalities must be complied with.

PVL3702_201_2014
Question 1
Which statement is INCORRECT?

1 Obligatory agreements create one or more obligations.


2 Absolving agreements discharge or extinguish obligations.
3 Real agreements transfer rights.
4 Transfer agreements transfer rights.
5 All binding agreements are contracts

Question 2
Which term / phrase is NOT linked to the characteristic features of a contract?

1 Juristic act.
2 Bilateral in nature.
3 Unilateral promise.
4 Multilateral in nature.
5 Promises or undertakings.

Question 3
What theory regards the inner wills of the parties as irrelevant on the basis that what the parties say
and do are important and not what the parties think?

1 The declaration theory.


2 The will theory.
3 The reliance theory.
4 The reception theory.
5 The expedition theory

Question 4
The majority in Barkhuizen v Napier 2007 (5) SA 323 (CC) held that

1 pacta sunt servanda is not a profoundly moral principle on which the coherence of any society
relies.
2 the validity of a contractual term can be directly tested against a provision of the Bill of Rights in
the Constitution.
3 unfair or unreasonable contractual provisions would always be enforced.
4 self-autonomy or the ability to regulate one’s own affairs, even to one’s own detriment, is the
very essence of freedom and a vital part of dignity.
5 public policy is not informed by the concept of ubuntu.

Question 5
S and V were staying in the same hotel in Cape Town, when V made a verbal offer to S. S returned to
Johannesburg where she resided to consider the offer. Three days before the final day for
acceptance, she tried to contact V telephonically to inform him of her acceptance, but he was
unavailable. She immediately wrote a letter of acceptance which she posted the same day, but it
reached V only after the deadline for acceptance had passed. Which theory is applicable to
determine if a valid contract was concluded?

1 The information theory.


2 The expedition theory.
3 The reception theory.
4 The reliance theory.
5 The declaration theory

Question 6
X sends an offer by email to Z on 1 February, and informs Z that the offer will lapse on
10 February. Z sends an email to X on 8 February, in which she accepts the offer. Z’s email reaches X’s
email address on 9 February. X only reads Z’s email on 11 February. Which is the
CORRECT statement?

1 Depending on the facts, either the expedition theory or the reception theory may be used to
determine if a valid contract was concluded.
2 A valid contract was concluded between X and Z, because the expedition theory applies.
3 A valid contract was concluded between X and Z, because the declaration theory applies.
4 A valid contract was concluded between X and Z, because the information theory applies.
5 A valid contract was concluded between X and Z, because a type of reception theory applies.

Question 7
A contractual term in a contract which is regulated by the Consumer Protection Act 68 of 2008 and
which is to the effect that the consumer acknowledges that no misrepresentation has been made to
him / her,

1 is effective only if the supplier has made the customer aware that such a provision exists in the
contract.
2 is effective only in the absence of fraud.
3 is prohibited.
4 could be effective or prohibited which outcome shall be decided by the discretion of the presiding
judicial officer.
5 is effective only if the customer acknowledged his / her awareness of such a term by signing or
initialling next to the term.

Question 8
Which statement about the Consumer Protection Act 68 of 2008 is INCORRECT?

1 The Act has a number of provisions that overlap considerably with the common law relating to
improperly obtained consensus.
2 The Act prohibits various forms of unconscionable conduct on the part of suppliers or their agents.
3 The Act prohibits non-disclosure of material facts to consumers.
4 It is not unconscionable for a supplier knowingly to take advantage of the fact that a consumer
was substantially unable to protect his or her own interests due to ignorance.
5 Suppliers suspected of engaging in prohibited conduct will be issued with compliance notices by a
regulatory body formed in terms of the Act.

Question 9
X is on her way from work and sees a white bull terrier bitch hiding in a doorway. Being an animal
lover, she takes the dog home with her. At home she notices a tag on the dog’s collar with the
owners contact details. She contacts the owner, Y, and Y and his dog are subsequently re-united. The
following day X sees the following advertisement in the newspaper: Lost in Johannesburg City Centre
on 27 May. Pedigree white bull terrier bitch with black patch over left eye. Answers to the name of
Beauty. Reward of R1 000 for information leading to safe return. Tel 011 555 5555.
She claims the R1 000 reward from Y who refuses to pay X any amount. Which statement is
INCORRECT?

1 The offer was clear and certain.


2 X has suffered financial or patrimonial loss.
3 An agreement was not concluded based on the reception theory.
4 The acceptance was a conscious response to the offer.
5 No agreement was concluded even though there was no revocation of the offer when the
acceptance was made.

Question 10
X has been leasing a commercial property from Z for the past three years. The lease will come to an
end on 31 May. On 5 March, X phones Z and offers to renew the lease for a further three years,
which offer Z accepts. During this phone call, the material terms of the renewal agreement are
agreed upon and X and Z further agree that the said material terms must be reduced to writing and
signed by both parties. Subsequently on 5 April, X is shocked to receive a letter from Z, advising X
that there will be no renewal of the lease and that X should vacate the leased property on 31 May. X
and Z never reduced their oral agreement to writing. Which is the most contentious issue that arises
from this set of facts?
1 Whether a formality has been stipulated by X and Z.
2 Whether a formality has been prescribed by law.
3 Whether the Shifren principle applies to this dispute.
4 Whether the contractual principles relating to restraint of trade applies to this dispute, based on
the judgment in Goldblatt v Freemantle 1920 AD 123.
5 Whether the principles relating to the exceptio doli generalis applies to this dispute.

PVL3702_Revision Pack

Question 1
Which of the following is NOT a requirement for a valid contract?
1 Consensus
2 Formalities
3 Possibility
4 Reciprocity
5 Certainty

Question 2
Which statement is INCORRECT?
1 A contract is a unilateral or even bilateral juristic act.
2 A contract entails promises or undertakings on one or both sides.
3 An undertaking in a contract that a certain state of affairs exists, or has existed, is known as a
warranty.
4 The conclusion of a contract can be multilateral.
5 Freedom of contract means that the parties can agree to anything that is possible and lawful.

Question 3
Which statement is INCORRECT?
1 An obligation is a legal bond between a debtor and a creditor.
2 With a contract of sale, the seller is the debtor in respect of the duty to deliver the thing sold, and
the seller is the creditor in respect to the obligation relating to the duty to pay the price.
3 An obligation comprises a right and a corresponding duty: the right of the creditor to demand a
performance by the debtor, and the duty of the debtor to make that performance.
4 A natural obligation is unenforceable in a court of law.
5 The right created by an obligation is a real right

Question 4
Which statement is INCORRECT?
1 A delict is wrongful and blameworthy conduct that causes harm to a person, and which obliges the
wrongdoer to compensate the injured party.
2 Concurrent liability exists where the same conduct might constitute both a delict and a breach of
contract, as when a surgeon who has contracted to perform an operation negligently leaves a cotton
swab inside the patient’s body.
3 The essential difference between contractual and delictual obligations is that the latter are, as a
general rule, voluntarily assumed by the parties themselves, whereas the former are imposed by
law, irrespective of the will of the parties.
4 The courts have shown great reluctance to permit claims in delict for economic losses caused by a
breach of contract.
5 Unjustified enrichment occurs when there is a shift of wealth from one person’s estate to another’s
without a good legal ground or cause for this shift.

Question 5
Which statement relating to the Bill of Rights in the Constitution of the Republic of South Africa,
1996, is CORRECT?
1 Vertical application relates to relationships between private persons, as in most contractual
situations.
2 Horizontal application relates to relationships between the state and the individual
3 In Barkhuizen v Napier 2007 (5) SA 323 (CC), the majority in the Constitutional Court ruled that a
contractual term can be tested directly against a provision in the Bill of Rights.
4 In Barkhuizen v Napier 2007 (5) SA 323 (CC), the minority in the Constitutional Court preferred an
indirect application of the Constitution to the contractual dispute before them.
5 In Barkhuizen v Napier 2007 (5) SA 323 (CC), Ngcobo J reasoned that the proper approach to
constitutional challenges to contractual terms, is to determine whether the term challenged is
contrary to public policy; and what constitutes public policy must be discerned with reference to
the fundamental values embodied in the Constitution

Question 6
Cedric has joined a religious sect. Recently this sect took advantage of Cedric’s good nature and
convinced him to donate his motor vehicle to the sect, which Cedric did. Cedric approaches you for
legal advice, explaining that he wants his motor vehicle returned to him. The issue emanating from
these facts relates to
1 duress.
2 undue influence.
3 commercial bribery.
4 puffs.
5 dicta et promissa.

Question 7
Assume the same facts as in question (6). A court presiding over this case is likely to find that the
religious sect
1 threatened Cedric and more particularly his property.
2 instilled reasonable fear in Cedric.
3 coerced Cedric to donate his motor vehicle to the sect.
4 obtained an influence over Cedric, and that this influence weakened his powers of resistance and
rendered his will compliant.
5 conducted itself in such a distasteful manner that it amounts to supervening impossibility of
performance
Question 8
Which of the following is NOT an element for commercial bribery?
1 A reward.
2 The one party is the briber.
3 The principal is able to exert influence over a third party.
4 The agent may be an agent in the true sense or merely a go-between or facilitator.
5 A direct or indirect benefit is sought for the briber

Question 9
If commercial bribery exists, the agreement between the briber and the agent is
1 voidable.
2 void.
3 void and voidable.
4 neither void nor voidable.
5 for the benefit of a third party

Question 10
If commercial bribery exists, the agreement between the briber and the principal is
1 voidable.
2 void.
3 void and voidable.
4 neither void nor voidable.
5 for the benefit of a third party

Question 11
Which concept is NOT a value that informs the law of contract?
1 Freedom of contract.
2 Good faith.
3 Privity of contract.
4 The requirement that a contract must not be against public policy.
5 Pacta sunt servanda.

Question 12
Which statement is CORRECT regarding unlawful contracts?
1 All unlawful contracts are void.
2 Some contracts that have been criminalized by statute are not void.
3 Some unlawful contracts are unenforceable in terms of the in pari delicto potior condicio
possidentis rule.
4 All unlawful contracts are voidable.
5 All contracts that are prohibited by statute in order to protect the revenue of the state are void

Question 13
X makes an offer to Y to purchase Y’s car. X sends the offer by e-mail to Y. Y reads the offer on 13 May
and drafts a written acceptance on 14 May. Y posts his acceptance on 15 May to X. X receives the
acceptance on 17 May in his post-box and reads it on 18 May. When was the contract concluded?
1 13 May.
2 14 May.
3 15 May.
4 17 May.
5 18 May

Question 14
In a contractual context, where the debtors are jointly liable only, and the co-creditors may only
claim performance jointly, this is a case of
1 proportionate liability.
2 simple joint liability.
3 in solidum liability.
4 joint and several liability.
5 collective joint liability

Question 15
Which statement regarding the interpretation of contracts is INCORRECT?
1 The parol evidence rule has an integration and interpretation aspect.
2 The distinction between background and surrounding circumstances is imprecise.
3 The primary rule is to give effect to the intention of the party who drafted the contract.
4 Where a term is ambiguous it should be interpreted against the party who proposed it.
5 Where a term is ambiguous it should be given a meaning that makes it legally effective.

Question 16
X and Y agree that should X sell her leather couch, she (X) will offer to sell it to Y first, before making
an offer to sell the couch to any other person. X sells the couch to Z for R10 000 without first offering
it to Y for sale. Delivery of the couch has not yet taken place. Which statement is CORRECT?
1 X and Y concluded an option contract.
2 The contract between X and Z is voidable, because it breaches the contract between X and Y.
3 Y has a personal right against X, but Z has a real right against X.
4 Both Y and Z only have personal rights against X.
5 The personal right of Z enjoys preference above the personal right of Y, because it arises from a
contract of sale.

Question 17
Assume the same facts as in question (6). What remedy does Y have against X?
1 An interdict.
2 A claim for damages.
3 A claim for specific performance of the contract of sale after Y has stepped into Z’s shoes with a
unilateral declaration of intent made to X.
4 All the above remedies.
5 None of the above remedies
Question 18
Essentialia are:
1 All the terms of a contract apart from the naturalia.
2 Terms that identify a contract as belonging to a particular class of contracts.
3 Terms automatically imposed by law on the contracting parties unless contracting parties expressly
exclude them.
4 All the terms of a contract apart from the incidentalia.
5 Material terms and conditions of a contract

Question 19
Y purchased from Z a specific painting for R150 000. At the time of contracting Y honestly believed it
to be an original Da Vinci painting, but Z did not know of Y’s belief. The painting was later found to be
a copy. Y argues that the contract is void whilst Z maintains that the contract is valid. Which answer
reflects the CORRECT legal position?
1 The contract is valid, because Y’s mistake regarding the painting is only an error in motive.
2 The contract is void, because Y and Z acted under a common error regarding the painting.
3 The contract is void, because of Y’s unilateral mistake regarding the painting.
4 The contract is void, because Y’s mistake regarding the painting is an error in corpore.
5 The contract is void, because of Y’s supposition regarding the painting

Question 20
X, an organiser of art exhibitions, contracted with Y for an exhibition to be held on 24 to 27 July.
These dates were the only dates mentioned during the negotiations. After having been pressurized
by X, Y hurriedly signed the standard form contract without reading it. The contract contained a
clause permitting X to change the dates of the exhibition unilaterally. Thereafter X changed the dates.
X had no reason to believe that Y would have signed the contract if he had known of the term. Which
statement(s) is / are INCORRECT?
1 Y can cancel the contract because of her material mistake with regard to the presence of a clause in
the standard form contract allowing X to unilaterally change the dates of the exhibition.
2 Y can prove that a contract with X exists without the clause in the standard form contract allowing
X to unilaterally change the dates of the exhibition.
3 Y can use the iustus error approach to prove that a contract with X exists without the clause
allowing X to unilaterally change the dates of the exhibition.
4 Option 1 and 3.
5 Option 1, 2 and 3

Question 21
In case law the Constitution has already impacted on the law of contract in the following ways:
1 Some High Courts have reversed the onus which rests on contract denier to prove that an
agreement in restraint of trade is against public policy to the position before Magna Alloys and
Research (SA) (Pty) Ltd v Ellis 1984 (4) SA 874 (A).
2 In ascertaining whether the exercise of contractual powers is against public policy, public policy has
been determined with reference to the fundamental values embodied in the Constitution.
3 In ascertaining whether a contractual term is contrary to public policy, public policy has been
determined with reference to a fundamental value embodied in the Constitution.
4 2 and 3.
5 1, 2 and 3.

Question 22
Inequality of bargaining power between the parties is taken into account as such to determine if
1 a contract is illegal.
2 undue influence has taken place.
3 duress has taken place.
4 1 and 2.
5 1, 2 and 3

Question 23
X makes a written offer to Y to purchase Y’s house. X sends the offer by post to Y. Y reads the offer on
13 May and drafts an acceptance on 14 May. Y posts his acceptance on 15 May to X. X receives the
acceptance on 17 May and reads it on 18 May. When was the contract concluded?
1 13 May.
2 14 May.
3 15 May.
4 17 May.
5 18 May

Question 24
An option requires that it has to be exercised before a certain date. What is / are the legal
consequence(s) before it is exercised?
1 The substantive offer in the option cannot be validly revoked.
2 If the grantee dies, the option can always be exercised.
3 If the grantor dies, the option can always be exercised.
4 The grantee can always cede his right to exercise the option to a third party.
5 All of the above

Question 25
X sells his car to Y. X fraudulently misrepresents the year model of his car to Y during the course of
their negotiations. X’s misrepresentation caused Y to agree to pay R220 000 for the car, instead of the
R180 000 she would have offered to pay. Y will most probably be able to rescind the contract,
because X made a(n)
(a) innocent misrepresentation.
(b) dictum et promissum.
(c) fraudulent misrepresentation.
1 (a).
2 (c).
3 (a) and (b).
4 (a) and (c).
5 None of the options.
Question 26
X sells his car to Y. X fraudulently misrepresents the year model of his car to Y during the course of
their negotiations. X’s misrepresentation caused Y to agree to pay R220 000 for the car, instead of the
R180 000 she would have offered to pay. The car is, in fact, worth only R150 000. The car would have
been worth R250 000 if the misrepresentation was true. What amount will Y be able to claim from X
based on fraudulent misrepresentation?
1 R0.
2 R30 000.
3 R40 000.
4 R70 000.
5 R100 000.

Question 27
Assume the same facts as in question (6). What amount will Y be able to claim from X based on
innocent misrepresentation (NOT a dictum et promissum)?
1 R0.
2 R30 000.
3 R40 000.
4 R70 000.
5 R100 000.

Question 28
Consider the same set of facts as in question (6). What amount will Y be able to claim from X based
on X’s dictum et promissum?
1 R0.
2 R30 000.
3 R40 000.
4 R70 000.
5 R100 000.

Question 29
Assume the same facts as in question (6). X guarantees the year model of the car. Y validly cancels
the contract. The price is returned to Y and the car to X. What amount will Y be able to claim from X
based on X’s breach of the guarantee?
1 R0.
2 R30 000.
3 R40 000.
4 R70 000.
5 R100 000.

Question 30
X, an organiser of art exhibitions, contracted with Y for an exhibition to be held on 24 to 27 July.
These dates were the only dates mentioned during the negotiations. After having been pressurized
by X, Y hurriedly signed the standard form contract without reading it. The contract contained a
clause permitting X to change the dates of the exhibition unilaterally. Thereafter X changed the dates.
X had no reason to believe that Y would have signed the contract if he had known of the term. Which
statement is CORRECT?
1 Y’s mistake with regard to the presence of a clause in the standard form contract allowing X to
unilaterally change the dates of the exhibition is a mistake in motive.
2 Y’s mistake with regard to with regard to the presence of a clause in the standard form contract
allowing X to unilaterally change the dates of the exhibition is an error in substantia.
3 Y can use the iustus error approach to prove that that a contract with X exists without the clause
allowing X to unilaterally change the dates of the exhibition.
4 Y’s mistake with regard to the presence of a clause in the standard form contract allowing X to
unilaterally change the dates of the exhibition is unreasonable.
5 X’s reliance that Y wanted to be bound by the standard form contract was unreasonable

Question 31
In which case was it decided that advertisements are usually mere invitations to the public to do
business?
1 Crawley v Rex 1909 TS 1105.
2 Allen v Sixteen Stirling Investments (Pty) Ltd 1974 (4) SA 164 (D).
3 R v Nel 1921 AD 339.
4 Steyn v LSA Motors Ltd 1994 (1) SA 49 (A).
5 None of the above.

Question 32
In Bird v Sumerville 1961 (3) SA 194 (A) the court found that
1 a person cannot accept an offer of which he is not aware.
2 an offer must be accepted by the person to whom it was addressed
3 an acceptance must be a reaction to the offer.
4 an acceptance must comply with any formalities set by law or by the offeror.
5 an acceptance must be unconditional and unequivocal.

Question 33
Y offers to sell his BMW motor car to Z on 1 July, for R50 000. One of the terms of Y’s offer is that the
offer lapses on 30 August. However, on 20 July Y notifies Z that the offer is cancelled. Z insists that
the offer is valid until 30 August and on 25 July Z notifies Y that he (Z) accepts the offer. Which
answer reflects the correct legal position?
1 A pre-emption contract was not concluded between Y and Z.
2 An option contract was not concluded between Y and Z.
3 An option contract was concluded between Y and Z.
4 1 and 2.
5 None of the above

Question 34
John owns a business. During the period 1 February to 31 March he unknowingly operates his
business in contravention of a statute which requires him to have a trading license. This statute only
criminalises the operation of a business without a trade license. On 15 February John sells goods on
credit to Steve for R2000. John delivers the goods to Steve. When the price is due and payable on 20
February, Steve refuses to pay John. Steve argues that he is not obliged to pay John, as
John did not have a trading license on 15 February.Which answer is CORRECT?
1 John may always claim back the goods from Steve, or if it no longer exists, the value thereof, based
on an unjustified enrichment claim.
2 The contract is illegal and therefore unenforceable.
3 Under certain circumstances John may claim back the goods from Steve, or if it no longer exists,
the value thereof, based on an unjustified enrichment claim.
4 John may enforce the contract and claim R2000 from Steve.
5 2 and 3

Question 35
Gary, a breeder of stud bulls, sells a bull to Piet for an agreed price. Gary knew that Piet required the
bull for breeding purposes, although this fact is not mentioned in the contract. Subsequently it turns
out that the bull is infertile and Piet wishes to cancel the contract. Which cause of action will Piet be
able to rely on?
1 Breach of a term implied by law that the stud bull is fertile.
2 Breach of a tacit term that the stud bull is fertile.
3 Breach of a tacit term that the stud bull will not have any latent defect.
4 Breach of an express term agreed upon by the conduct of the parties that the stud bull is fertile.
5 None of the above.

Question 36
Which case is applicable to the facts in question (5)?
1 Trotman v Edwick 1951 (1) SA 443 (A).
2 Van den Berg v Tenner 1975 (2) SA 268 (A).
3 Sweet v Ragerguhara NO 1978 (1) SA 131 (D).
4 Goldstein and Wolff v Maison Blanc (Pty) Ltd 1948 (4) SA 446 (C).
5 Minister van Landbou-Tegniese Dienste v Scholtz 1971 (3) SA 188 (A).

Question 37
The case in question (5) provides an excellent example of the distinction between
1 naturalia and tacit incidentalia of a contract.
2 accidentalia and incidentalia of a contract.
3 incidentalia and an essentialia of a contract.
4 essentialia and naturalia of a contract.
5 essentialia and accidentalia of a contract

Question 38
Which case would you refer to as authority on the effect of a contractual clause excluding liability for
misrepresentation?
1 Goldblatt v Fremantle 1920 AD 123.
2 Neethling v Klopper 1967 (4) SA 459 (A).
3 Broodryk v Smuts 1942 TPD 47.
4 Wells v SA Alumenite Co 1927 AD 69.
5 Bank of Lisbon and South Africa Ltd v De Ornelas 1988 (3) SA 580 (A).

Question 39
While doing window-shopping, X and his fiancée, Y, see a ring displayed in a shop window. They
immediately enter the jeweller’s shop and offer to buy the ring at the displayed price. They find out
to their dismay that the “diamond” is a synthetic diamond afterwards. Which statement is CORRECT:
1 The contract of sale is void.
2 The error is an error in substantia.
3 The error is an error in corpore.
4 The jeweller made a dictum et promissum that the ring was a diamond ring.
5 The parties to the contract did not want to buy and sell the same ring.
Question 40
“Our law allows a party to set up his own mistake in certain circumstances in order to escape liability
under a contract into which he has entered. But where the other party has not made any
misrepresentation and has not appreciated at the time of acceptance that his offer was being
accepted under a misapprehension, the scope for a defence of unilateral mistake is very narrow, if it
exists at all.” In which case does this statement appear?
1 Du Toit v Atkinson Motors Bpk 1985 2 SA 889 (A).
2 George v Fairmead (Pty) Ltd 1958 2 SA 465 (A).
3 Allen v Sixteen Stirling Investments (Pty) Ltd 1974 4 SA 164 (D).
4 National and Overseas Distributors Corporation (Pty) Ltd v Potato Board 1958 2 SA 473 (A).
5 Steyn LSA Motors Ltd 1994 1 SA 49 (A).

Question 41
Where a contract is concluded by email, which theory applies?
1 The declaration theory.
2 The expedition theory.
3 The reliance theory.
4 The reception theory.
5 The information theory

Question 42
S sends a written offer by post to P, in which S offers to sell his radio to P for R3 000. In the written
offer, S stipulates that the acceptance must be communicated to him by 1 June. On 20 May, P posts a
written acceptance to S, to purchase the radio for R3 000. On 22 May, S phones P and cancels his
offer to sell his radio. P’s written acceptance reaches S on 25 May. Which answer reflects the
CORRECT legal position?
1 A contract is concluded between S and P, because the expedition theory applies.
2 A contract is not concluded between S and P, because the information theory applies.
3 A contract is concluded between S and P, because the information theory applies.
4 A contract is not concluded between S and P, because the expedition theory applies.
5 A contract is concluded between S and P, because the reception theory applies.

Question 43
A offers B her car for R20 000 cash. B answers: “I will buy your car for R19 000 cash.” By this answer,
1 B accepts A’s offer.
2 B accepts A’s offer unequivocally.
3 B accepts and rejects A’s offer.
4 B rejects A’s offer and makes her (A) a counter-offer.
5 B complies with the requirement that the acceptance must not be ambiguous.

Question 44
Where a culpable misrepresentation exists, the action which arises is based on
1 delictual liability.
2 unjustified enrichment.
3 estoppel.
4 statutory liability.
5 contractual liability.

Question 45
Sibongile is desperate to sell her house because of a termite infestation. Consequently she knowingly
conceals all signs of damage when a potential purchaser, Thandi, comes to inspect the house and
furthermore tells Thandi that there is nothing wrong with the house. Thandi purchases the house,
which she would never have done if she knew of the termite infestation. What cause of
action will Thandi be able to rely on in the circumstances?
(a) Dictum et promissum.
(b) Innocent misrepresentation.
(c) Culpable misrepresentation.
(d) Material mistake.
1 (a), (b) and (c).
2 (a) and (c).
3 (b) and (c).
4 Only (c).
5 Only (d)

Question 46
Assume the same facts as in question (5). What remedy or remedies are available to Thandi when
she discovers the termite infestation and damage to the house?
1 Thandi may only rescind the contract.
2 Thandi may only uphold the contract and claim damages.
3 Thandi may rescind the contract and claim damages.
4 Thandi may uphold the contract and claim damages.
5 Both 3 and 4.

Question 47
In which case did the seller not reveal to the purchaser that the swimming pool was not structurally
sound because it leaked?
1 Trotman and Another v Edwick 1951 (1) SA 443 (A).
2 De Jager v Grunder 1964 (1) SA 446 (A).
3 Ranger v Wykerd and Another 1977 (2) SA 976 (A).
4 Bayer South Africa (Pty) Ltd v Frost 1991 (4) SA 559 (A).
5 Phame (Pty) Ltd v Paizes 1973 (3) SA 397 (A).

Question 48
The aggrieved party may be successful in instituting
1 a delictual claim against the other contracting party, for the return of his performance in terms of
an illegal contract, if the in pari delicto rule is relaxed.
2 an unjustified enrichment claim against the other contracting party, for the return of his
performance in terms of an illegal contract, if the par delictum rule is relaxed.
3 a contractual claim against the other contracting party, for the return of his performance in terms
of an illegal contract, if the in pari delicto rule is relaxed.
4 a specific performance claim against the other contracting party, for the return of his performance
in terms of an illegal contract, if the in pari delicto rule is relaxed.
5 (3) and (4).

Question 49
The consequences of an illegally concluded contract were discussed in
1 Jajbhay v Cassim 1939 AD 537.
2 Brandt v Spies 1960 (4) SA 14 (E).
3 Neethling v Klopper 1967 (4) SA 459 (A).
4 Goldblatt v Fremantle 1920 AD 123.
5 Nel v Cloete 1972 (2) SA 150 (A).

Question 50
Where ambiguous words in a clause in a contract are interpreted in such a way that the least
possible burden is placed on the debtor, and the clause is interpreted against the party in whose
favour it was inserted, this is consistent with
1 the primary rules of interpretation.
2 the secondary rules of interpretation.
3 the tertiary rules of interpretation.
4 the parol evidence rule.
5 the pactum in favorem tertii.

Question 51
An item displayed in a shop window, accompanied by presentation of payment for the item by a
customer, generally
1 results in a contract being concluded because an offer and acceptance exists.
2 infers that the customer is the offeror.
3 infers that the customer is the offeree.
4 infers that neither an offer nor an acceptance exists.
5 infers that the customer merely indicates an invitation to do business

Question 52
The courts use the hypothetical bystander test when determining the possible existence of
1 essentialia.
2 naturalia.
3 terms implied by law.
4 tacit terms.
5 express terms

Question 53
H lets a business premises to D for R5 000 per month. The written lease contains a clause prohibiting
D from sub-letting the premises without the written consent of H. A further clause requires, that for
any variation of the contract to be valid (including this clause), it has to be in writing and signed by
both parties. Subsequently H and D orally agreed that D can sub-let the premises, for which D will
pay an additional R2 000 per month. D then sublet the premises to Y. After H received a rental
income of R7 000 per month from D for a period of six months, he (H) cancels the lease agreement
with D because D breached the contract by sub-letting the premises. Which answer reflects the
correct legal position?
1 H validly cancelled the contract.
2 H’s attempted cancellation has no effect on the validity of the contract.
3 Because H orally agreed that D can sub-let the premises, it is in fact H that has breached the lease
agreement by cancelling the contract.
4 Because H collected an additional R2 000 per month for six months from D, the oral variation to the
contract is valid.
5 The contractual relationship between the parties has become void for vagueness.

Question 54
Assume the same facts in question (3). Which issue is NOT relevant to a question of this nature?
1 Whether the non-variation clause is itself entrenched against oral variations.
2 Whether a later agreement should take precedence over a prior agreement.
3 Policy grounds based on freedom of contract.
4 Whether a prior agreement should take precedence over a later agreement.
5 Whether H was enriched by receiving an additional R2 000 for six months, at the expense of D.

Question 55
Regarding restraint of trade agreements, what is the present legal position under South African law?
1 In line with English Law, freedom of trade takes precedence over the principle of sanctity of
contract.
2 If a restraint is unreasonable then it is automatically rendered to be contrary to public policy.
3 A restraint of trade agreement that is contrary to public policy is void and unenforceable.
4 The onus to prove that enforcing a restraint would be against public policy is on the party who
alleges that she is not bound by the restraint agreement.
5 A restraint of trade agreement cannot be partially enforceable.

Question 56
Mark is engaged to Jane. Mark has a very strong personality and eventually persuades Jane to sell
and transfer her house that is worth R900 000 to him at a purchase price of a mere R20 000. After
registration of the property in Mark’s name he breaks off the engagement. Which of the following
requirement(s) is / are relevant for Jane to prove, in her pursuit to have the transfer of the house into
Mark’s name set aside?
(a) Mark exercised an influence over her.
(b) Mark exercised this influence over her, in an unscrupulous manner in order to induce her to
consent to a transaction which is to her detriment and which she, with normal free will, would not
have concluded.
(c) Mark gained this influence by standing in a position of trust in relation to her.
(d) This influence exercised by Mark over her, amounted to intimidation which was not imposed in
good faith.
1 (a) and (d).
2 (a) and (b).
3 (a), (b) and (c).
4 (a), (b) and (d).
5 (a), (b), (c) and (d).

Question 57
Which cause of action is delictual?
1 Mistake.
2 Culpable misrepresentation.
3 Common error.
4 Dictum et promissum.
5 Impossibility of performance

Question 58
Which causes of action are likely to render the contract voidable?
1 Mistake and culpable misrepresentation.
2 The iustus error doctrine and duress.
3 The expedition theory and innocent misrepresentation.
4 Culpable misrepresentation and undue influence.
5 Undue influence and the reliance theory

Question 59
Which statement (s) is/are CORRECT regarding unlawful contracts?
(a) Some unlawful contracts are void.
(b) Some unlawful contracts are not void because the legislature intended that the conclusion of
such a contract should merely constitute a criminal offence that is punishable by way of a fine.
(c) All unlawful contracts are not enforceable.
(d) Some unlawful contracts are voidable.
1 (a) only.
2 (c) only.
3 (a) and (b).
4 (a), (b) and (d).
5 (a), (c) and (d).

Question 60
The aggrieved party may
1 be successful in instituting a delictual claim against the other contracting party, for the return of his
performance in terms of an illegal contract, if the in pari delicto rule is relaxed.
2 be successful in instituting an unjustified enrichment claim against the other contracting party,
for the return of his performance in terms of an illegal contract, if the in pari delicto rule is relaxed.
3 be successful in instituting a contractual claim against the other contracting party, for the return of
his performance in terms of an illegal contract, if the in pari delicto rule is relaxed.
4 be successful in instituting a specific performance claim against the other contracting party, for the
return of his performance in terms of an illegal contract, if the in pari delicto rule is relaxed.
5 not be successful in instituting any claim against the other contracting party, for the return of his
performance in terms of an illegal contract, if the in pari delicto rule is relaxed.
Question
Albert takes his motor vehicle to Dodgy Motors for a service. On his arrival, he is asked to sign a “job
card” by the owner. Albert enquires why he is required to sign the “job card” and the owner explains
to him that by signing he is authorising them to conduct the service on his car, which will cost R1000.
He signs the “job card” without reading it. While servicing the car, the service manager finds faults
on the car (unrelated to the service) and he proceeds to do these additional repairs for a further
R2000. Albert refuses to pay for the additional repairs and argues that he did not authorise such
repairs. The owner of Dodgy Motors argues that Albert is obliged to pay for the work done as the
“job card” contains a contractual clause authorising Dodgy Motors to do any repairs on the motor
vehicle which they deem necessary, without asking the client’s authorisation, and requiring the client
to pay for such repairs. Advise Albert on whether he is liable on the contract to pay Dodgy Motors
R2000 for the additional repairs. Refer to George v Fairmead (Pty) Ltd, Sonap Petroleum (SA) Ltd (SA)
(Pty) Ltd v Pappadogianis, and other relevant case law in your answer.
Do not apply the Consumer Protection Act to this question. [15]

Answer
The essence of this problem is the question whether Albert and the owner of Dodgy Motors have
reached actual consensus or ostensible consensus. Albert
will not be contractually bound to pay for the additional repairs if this requirement for a valid
contract is absent.
At the outset, it must be determined whether agreement (consensus ad idem) as a contractual basis
exists between the parties, as required in terms of the will theory. Consensus has three elements:
1. The parties must seriously intend to contract
2. The parties must be of one mind as to the material aspects of the proposed agreement (the terms
and the identity of the parties to it)
3. The parties must be conscious of the fact that their minds have met
In the present case, the parties were not in agreement as to the consequences they wished to
create; Albert thought that he was authorising Dodgy Motors to only service his car, while the owner
of Dodgy Motors knew that the contract also allowed Dodgy Motors to conduct repairs which they
deemed necessary and payable by Albert without any further authorisation from Albert. This is a
mistake as to the obligations the parties wished to create and is thus a material mistake, which
excludes consensus between the parties. This means that no contract could arise on the basis of the
will theory.
This type of mistake can be illustrated with a number of cases:
In George v Fairmead, the appellant signed a hotel register without reading it. The register contained
a term excluding the respondent from liability for certain acts. The appellant was unaware of this
term and his mistake related to a term that he believed would not be in the contract and as such was
material because it related to an aspect of performance.
In Allen v Sixteen Stirling Investments, the plaintiff believed he was purchasing the erf pointed out to
him by the seller’s agent, while the written contract that he signed indicated the correct erf, which
was a completely different property. His mistake related to performance and was material.
However, the matter does not end here. A party may be held contractually liable on the basis of a
supplementary ground for liability, namely the reliance theory. In this regard, the direct or indirect
approach to the reliance theory may be considered.
DIRECT APPROACH:
With reference to the direct approach, contractual liability is based on the reasonable reliance that
consensus has been reached, which the one contractant (the contract denier) creates in the mind of
the other contractant (the contract enforcer).
According to the Sonap case, the direct reliance approach entails a threefold enquiry:
1. Was there a misrepresentation regarding one party’s intention?
2. Who made the misrepresentation?
3. Was the other party misled by the misrepresentation, and if so, would a reasonable person have
been misled?
In our question, firstly, Albert made a misrepresentation by signing the contract, that his intention is
the same as that expressed in the contract.
Secondly, the owner of Dodgy Motors could actually have been misled by this misrepresentation, but
a reasonable man would have taken steps to point out to Albert that the contract allows Dodgy
Motors to unilaterally conduct repairs on the car, because Albert enquired about the purpose of the
“job card” and the owner of Dodgy Motors misled him to believe that by signing the card he is
merely authorising the service to be done. In Sonap the court found that the contract enforcer knew
that the contract denier was acting under a mistake and was thus not misled.
In our case, Albert therefore did not create a reasonable reliance that he wished to be bound by the
contract he signed.
INDIRECT APPROACH (IUSTUS ERROR DOCTRINE):
In terms of this approach, a party may escape liability to be bound to a contract if it can be
established that the mistake is both:
1. Material, and
2. Reasonable
It has already been shown in the discussion above that Albert’s mistake is material. It still has to be
determined if his mistake was reasonable.
The contract denier’s mistake will be reasonable in the following instances:
1. If caused by a misrepresentation on the part of the contract enforcer (an unlawful
misrepresentation).
2. If the contract denier is not to blame for the mistake.
3. If the contract denier did not cause a reasonable belief in the contract asserter that the contract
denier assented to the agreement.
Fault is not a requirement for the misrepresentation by the contract enforcer, but unlawfulness is. If
the misrepresentation is a positive act it is unlawful in itself. If a legal duty to speak exists and the
party has kept quiet when he ought to have spoken, an unlawful negative misrepresentation has
occurred. A legal duty to speak exists in the following instances:
• Where the contract asserter knows or ought to know as a reasonable person that the other party is
mistaken
• Where, prior to the conclusion of the agreement, the contract asserter created an impression
directly conflicting with the provisions of the agreements, he must draw the contract denier’s
attention to the discrepancy (Du Toit v Atkinson’s Motors).
In our problem, Albert enquired about the purpose of the “job card” and the owner of Dodgy Motors
misled him by answering that by signing he was merely authorising the service. The owner’s
misrepresentation was a positive act, and was therefore unlawful. Albert’s error was thus
reasonable.
Applying the indirect approach to the reliance theory we do not have a valid contract. Applying the
direct approach, we do not have a valid contract. Albert is not contractually liable to pay R2000 for
the repairs.

Question
X, an organiser of art exhibitions, contracted with Y for an exhibition to be held on 24 to 27 July.
These dates were the only dates mentioned during negotiations. After having been pressurised by X,
Y hurriedly signed a standard form contract without reading it. The contract contained a clause
permitting X to change the dates of the exhibition unilaterally. Thereafter, X changed the dates. X had
no reason to believe that Y would have signed the contract if he had known of the term. Y averred
that the contract was void. Will Y succeed in his attempt to have the contract set aside? Substantiate
your answer and refer to relevant case law. Do not apply the Consumer Protection Act to this
problem. [15]

Answer
The essence of this problem is whether X and Y have reached consensus. Y will not be contractually
bound if this requirement for a valid contract is absent.
At the outset, it must be considered whether agreement (consensus ad idem) as a contractual basis
exists between the parties, as required in terms of the will theory. Consensus has three elements:
1. The parties must seriously intend to contract
2. The parties must be of one mind as to the material aspects of the proposed agreement (the terms
and the identities of the parties to it)
3. The parties must be conscious of the fact that their minds have met.
In the present case, the parties were not in agreement as to the consequences they wished to
create: Y thought that the dates for the exhibition (X’s performance) was fixed, while X knew that the
contract allowed X to unilaterally change the dates. This is a mistake as to the obligations the parties
wished to create and is thus a material mistake, which excludes consensus between the parties. This
means that no contract could arise on the basis of the will theory.
This type of mistake can be illustrated with a number of cases:
In George v Fairmead, the appellant signed a hotel register without reading it. The register contained
a clause excluding the respondent from liability for certain acts. The appellant was unaware of this
term and his mistake related to a term that he believed would not be in the contract and as such was
material because it related to an aspect of performance.
In Allen v Sixteen Stirling Investments, the plaintiff believed he was purchasing the erf pointed out to
him by the seller’s agent, while the written contract that he signed indicated the correct erf, which
was a completely different property. His mistake related to performance and was material.
However, the matter does not end here. A party may be held contractually liable on the basis of a
supplementary ground for liability, namely the reliance theory. In this regard, the direct or indirect
approach to the reliance theory may be considered.
INDIRECT APPROACH (IUSTUS ERROR DOCTRINE):
In terms of this approach, a party may escape liability to be bound to a contract if it can be
established that the party laboured under a mistake, which was both: 1. material and
2. reasonable.
It has already been shown that Y’s mistake is material in the discussion above. It still has to be
determined if Y’s mistake was reasonable.
The contract denier’s mistake will be reasonable in the following circumstances:
1. If caused by a misrepresentation on the part of the contract asserter (an unlawful
misrepresentation)
2. If the contract denier is not to blame for the mistake
3. If the contract denier did not cause the contract asserter to have a reasonable belief that the
contract denier assented to the contract.
If a legal duty to speak exists and a party has kept quiet when he ought to have spoken, that party
has made an unlawful negative misrepresentation. A legal duty to speak will usually exist where:
• The asserter knows or ought to know as a reasonable person that the other party is mistaken
• Where, prior to the conclusion of the agreement the asserter created an impression directly
conflicting with the provisions of the agreement, he must draw the contract denier’s attention to this
discrepancy (Du Toit v Atkinson’s Motors).
Since X had no reason to believe that Y would have signed the contract had Y known of the term
allowing X to change the dates of the exhibition unilaterally, X had a legal duty to point out this
clause to Y. X’s failure to do so renders Y’s material mistake reasonable.
DIRECT APPROACH:
With reference to the direct approach, contractual liability is based on the reasonable reliance that
consensus has been reached, which the one contractant (the contract denier) creates in the mind of
the other contractant (the contract asserter).
According to the Sonap case, the direct reliance approach involves a threefold enquiry:
1. Was there a misrepresentation regarding one party’s intention?
2. Who made this misrepresentation?
3. Was the other party actually misled by this misrepresentation, and if so, would a reasonable
person have been misled?
By signing the contract, Y, a party to the contract, misrepresented her intention to be bound by the
clause allowing X to unilaterally change the dates. X knew that the only dates mentioned during
negotiations were 24 to 27 July, that Y hastily signed the contract, and that the contract had a clause
allowing X to unilaterally change the dates. X was probably not actually misled by the
misrepresentation by Y, and nor would a reasonable person be misled in any event. There was
therefore no reasonable reliance on consensus on the part of Y.
Y is not bound by the agreement because of lack of apparent and actual consensus.

Question
John, a racehorse owner, advertises for sale the horse Fire for R1.5 million. In the advertisement it is
stated that Fire is an offspring of the legendary July winner, Lightning. Peter is a horse breeder who
specifically wishes to introduce the bloodline of Lightning into his stud. He agrees orally with John to
buy Fire for R1.5 million. Later, in order to meet the requirements of the horse breeders’ association,
John has a written contract drawn up which Peter signs without reading. The contract makes no
mention of Fire's ancestry, but does contain a clause exempting John from liability for any
representations made during negotiations or in the contract. Peter's attention is not drawn to these
facts. A month later Peter finds out that Fire is in fact not an offspring of Lightning, although at the
time of the conclusion of the contract John genuinely and without any fault on his part believed that
to be the case. Advise Peter on whether the contract of sale is valid. Substantiate your advice and
refer to relevant case law. Apply the objective approach of the courts in answering this question. Do
not apply the Consumer Protection Act to this question.
Answer
Identifying the problem
The facts indicate that even though John and Peter have apparently reached consensus (there is a
written contract) they may not have reached consensus based on the will theory, because Peter
signed the contract without reading it. Peter also thought the horse was an offspring of Lightning.
The question thus deals with error.
A further clue is to be found in the direction to apply the objective approach. The objective approach
of the courts involves the application of declaration theory as qualified by the iustus error doctrine
(Hutchison and Pretorius (eds) The law of Contract in South Africa Oxford University Press Southern
Africa 2012 97-103). This question thus deals with error.
Discussing the relevant law applicable to the problem AND applying the law to the facts of the
problem
John and Peter have reached consensus according to the declaration theory. There is a signed
contract of sale which indicates that their declared intentions have concurred. The iustus error
doctrine now has to be applied because it qualifies the declaration theory. Peter erred with regard to
Fire's lineage and the presence of the exemption clause in the written contract of sale. Peter who
acted under a mistake and wishes to escape liability must prove that his mistake was material and
reasonable in terms of iustus error doctrine.
The error regarding Fire’s lineage is a non-material mistake is a mistake regarding a characteristic (the
lineage) of the thing sold, Fire (an error in substantia). The parties wanted to buy and sell the same
horse, Fire. See Eiselen GTS et al Law of contract Only study guide for 3702 Unisa 2012 40: Hutchison
and Pretorius Contract 88-89. The parties were, however, not in agreement as to the legal
consequences they wished to create: Peter did not know that there was an exemption clause in the
contract he signed, but John knew there was. Peter thus made a mistake as to the obligations the
parties wished to create which excludes consensus between the parties (Hutchison and Pretorius
Contract 86). There was thus no actual consensus.
The facts of our problem are very similar to that in Du Toit v Atkinson's Motors Bpk 1985 (2) SA 889
(A) where the appellant signed an agreement without reading it. The contract contained a term
excluding the respondent’s liability for misrepresentation. The court held that the mistake regarding
the exemption clause was material.
This type of mistake also occurred in other cases. In Allen v Sixteen Stirling Investments (Pty) Ltd
1974 (4) SA 164 (D) the plaintiff believed that he was purchasing the erf shown to him by the seller's
agent, while the written contract that he signed indicated the correct erf which was a completely
different property. His mistake related to performance and was material. In Sonap Petroleum (SA)
(Pty) Ltd (formerly known as Sonarep (SA) (Pty) Ltd) v Pappadogianis 1992 (2) SA 234 (A) the
appellant erred with regard the period of the lease which was an aspect of the performance.
However, the matter does not end here, because Y still has to prove that his mistake was reasonable.
This aspect of iustus error is an indirect application of the reliance theory. A mistake will inter alia be
reasonable (Hutchison and Pretorius Contract 100-103) where the mistake was caused by a
misrepresentation on the part of the contract assertor (John). Fault is not a requirement for this
misrepresentation, but wrongfulness is.
Here John failed to remove the incorrect impression that Peter had that the written contract did not
include an exemption clause. Such an ommissio will only be wrongful if John (the contract enforcer)
had a legal duty in the circumstances to speak to remove the incorrect impression. Such a duty will
exist where the contract assertor, before the conclusion of the contract, created an impression which
is in direct conflict with the agreement he or she seeks to enforce (Hutchison and Pretorius Contract
101-102). Under these circumstances, the contract assertor must draw the contract denier’s
attention to this discrepancy (Hutchison and Pretorius Contract 101-102).
Du Toit v Atkinson's Motors Bpk 1985 (2) SA 889 (A) contained a similar factual setting to this
assignment question. The Appellate Division reasoned (906) that by not saying anything about the
exemption clause, the contract assertor created the impression that the signed document did not
contradict the advertisement. The contract assertor misled the contract denier regarding the
contents of the contractual document by way of omission which rendered the contract deniers
mistake reasonable.
Peter’s mistake was thus reasonable as John failed to bring to his (Peter's) attention that the oral
contract arising from the advertisement, differed from the signed document which included an
exemption clause. There was thus no apparent consensus.
The giving of appropriate advice
The written contract of sale is invalid because Peter’s mistake was both material and reasonable and
both actual and apparent consensus lacked.

Question
X is a keen golfer who has played at many golf tournaments over the years as an amateur. She is very
well informed about the rules pertaining to her amateur status as a golfer, and knows that amateurs
can only claim a maximum of R1000 in prize money at golf tournaments. X participated in a recent
golfing tournament wherein she achieved a hole-in-one at the 9th hole. At this hole was an
advertising board, which read: “Hole-in-one prize sponsored by Speedy Motors to the value of R90
000”. The prize was parked next to this board in the form of a new car. X claimed the prize from
Speedy Motors but they rejected her claim on the basis that the prize could only be claimed by
professional players and not amateur players. Advise X. Refer to Steyn v LSA Motors and other
relevant case law. [15]

Answer
This problem deals with two questions: Was there a valid offer and acceptance? Was there
consensus between the parties?
Offer and acceptance:
The general rule in our law is that an advert constitutes merely an invitation to do business (Crawley
v Rex). However, following the reasoning in Carlill v Carbolic Smoke Ball Co, the court in Bloom v
American Swiss Watch Co held that the advertising of a reward might be construed as an offer to the
public.
An offer may only be accepted by a person or persons to whom it was directed (Bird v Summerville).
Although Speedy Motors intended the offer to be open only to professional players, the expressed
offer was apparently open to the public. Mistake is thus also relevant.
Mistake:
At the outset, it must be determined whether agreement (consensus ad idem) as a contractual basis
exists between the parties, as required in terms of the will theory. Consensus has three elements:
1. The parties must seriously intend to contract
2. The parties must be of one mind as to the material aspects of the proposed agreement (the terms
and the identities of the parties to it) 3. The parties must be conscious of the fact that their minds
have met
In our case, X and Speedy Motors were not in agreement as to the identity of the parties, and this is
a material mistake, which excludes consensus based on the will theory.
However, the matter does not end here. A party may be held contractually liable on the basis of a
supplementary ground for liability, namely the reliance theory. In this regard, the direct reliance
approach or the indirect reliance approach may be considered. Because the facts in this case are
similar to the case of Steyn v LSA Motors where it was held that the indirect approach couldn’t be
applied in instances where there is no objective appearance of agreement, only the direct approach
will be considered.
DIRECT APPROACH:
With reference to the direct approach, contractual liability is based on the reasonable reliance that
consensus has been reached, which the one contractant (the contract denier) creates in the mind of
the other contractant (the contract enforcer). According to the Sonap case, the direct reliance
approach entails a threefold enquiry:
1. Was there a misrepresentation regarding one party’s intention?
2. Who made this misrepresentation?
3. Was the other party actually misled by the misrepresentation, and if so, would a reasonable
person have been misled?
In our question, Speedy Motors made a misrepresentation regarding its intention that the offer is
made only to professional players, by advertising the reward to the public. Although it may be argued
that X was actually misled by the misrepresentation, it is certain that a reasonable person in X’s
position would not have been misled. X should know, as an experienced amateur golfer, that only
certain prizes are open to amateurs. There was therefore no reasonable reliance on consensus on
the part of X. X will not succeed in her claim for the prize.

Question
S, who lives in Upington, sends P, who lives in Grahamstown, a letter by private courier in which she
offers to sell him her (S’s) motorcycle, a collector’s piece, for R100 000. She states in her letter that
her offer will expire on 1 February. P accepts S’s offer by letter, which he posts on 31 January. S
receives the letter on 7 February and only reads it on the next day. P tenders payment of R100 000
but S refuses to accept payment. Did a valid contract arise between S and P? Substantiate your
answer. [15]
Answer
The question is whether P has accepted S’s offer in time and thus whether S and P have reached
consensus.
Where the offeror has prescribed a time limit for acceptance, the offer lapses automatically if it is not
accepted within the prescribed period.
The general rule is that a contract comes into being only when the acceptance is communicated to
the mind of the offeror. The information theory, which is the general rule in our law, states that the
agreement is concluded when and where the offeror learns or is informed of the acceptance – in
other words, when the offeror reads the letter of acceptance.
On the other hand, the expedition theory applies to postal contracts. In terms of this theory,
introduced into our law in the Cape Explosive Works case, a contract comes into being when and
where the offeree posts the letter of acceptance. By making an offer through the post, the offeror is
deemed not only to have authorised acceptance by post, but also to have waived the requirement of
notification of acceptance.
The question that then arises is which theory applies. In our law, the general rule is that the
information theory applies, however the expedition theory will apply if the following four criteria are
met:
1. the offer is made by post or telegram
2. the postal services are operating normally
3. the offeror has not indicated a contrary intention, expressly or tacitly, and
4. the contract is a commercial one.
If any of these criteria are not met, the information theory applies.
In this question, the offer was not made by post, instead it was sent by private courier, and therefore
the expedition theory does not apply. It follows that the information theory must be applied.
Because S only learnt of the acceptance by P after expiry of the offer (when S read the letter on 8
February), the offer had already lapsed and no valid contract arose between the parties.

Question
X is on her way from work and sees a white bull terrier bitch hiding in a doorway. Being an animal
lover, she takes the dog home with her. The next day, she sees the following advertisement in the
newspaper:
Lost in Johannesburg, on 27 May. Pedigree white bull terrier bitch with black patch over left eye.
Answers to the name of Beauty. Reward of R1000 for information leading to safe return. Tel 011 555
5555.
She realises that the dog she found matches the description given.
She calls the advertiser who rushes over to be joyfully united with Beauty. In his joy, Beauty’s owner,
Y, seems to forget the reward and X wishes to claim it from him. Will she be successful? Substantiate
your answer. Refer to Bloom v American Swiss Watch Co and other relevant case law in your answer.
[10]

Answer
X will only be successful in her claim if a valid contract arose between X and Y, and this will be the
case if there was a valid offer and acceptance.
The offer:
The offer was in the form of an advertisement. The general rule in our law is that an advertisement
constitutes an invitation to do business (Crawley v Rex). However, in Bloom v American Swiss it was
held that the advertising of a reward might be construed as an offer to the public. Offers to the
public at large can be made (Carlill v Carbolic Smoke Ball Co).
In our case, the offer was firm, complete, clear, and certain. The offer can therefore be said to have
been valid.
The providing of information by X was a valid acceptance of Y’s offer:
• X’s acceptance was unqualified
• X, as a member of the public to whom the offer was made, may accept
(offer may only be accepted by offeree – Bird v Summerville)
• X’s acceptance was a conscious response to the offer (he knew of the offer and could thus accept it
– unlike the situation of Bloom v American Swiss where the plaintiff returned the item but was
unaware there was a reward for doing so).
It can be concluded that a valid contract arose in this problem, because Y made a valid offer, which X
validly accepted.

Question
Y signs and delivers a written offer (including all the material terms) to Z on 1 July, for the purchase of
Z’s waterfront apartment. Y’s offer is for R800 000 and one of the terms of the offer states “This offer
lapses on 30 August”. However, whilst Z is still considering Y’s offer, Y delivers a letter to Z on 20 July,
advising Z that his (Y’s) offer is cancelled. Z insists that the offer is valid until 30 August, and on 25
July Z delivers a letter to Y, advising Y that he accepts Y’s offer. Has a valid contract of sale been
created between Y and Z? Discuss with reference to Brandt v Spies and other relevant case law. [10]

Answer
Contracting parties may enter into an agreement in terms of which the offeror undertakes not to
revoke his or her offer. In such cases, it is said that one party grants the other an option.
For this question, an option does not exist because there is no agreement in place that binds Y to
keep his offer open until 30 August. Y has unilaterally imposed this upon himself in the offer, but it
was certainly not an agreement by both parties to hold Y to keep his offer open until this date. This
means that no option contract was concluded.
Y validly revokes his offer to Z on 20 July and therefore there is no offer that Z can accept. The
requirements for a valid offer and acceptance for a contract have not been met, and no valid contract
has thus been created.

Question
X has been leasing a commercial property from Z for the past three years. The leas will come to an
end on 31 May 2010. On 5 March 2010, X phones Z and offers to renew the lease for a further three
years, which offer Z accepts. During this phone call, the material terms of the renewal agreement are
agreed upon and X and Z further agree that the said material terms must be reduced to writing and
signed by both parties. Subsequently, on 5 April 2010, X is shocked to receive a letter from Z, advising
X that there will be no renewal of the lease and that X should vacate the leased property on 31 May
2010. X and Z never reduced their oral agreement to writing. Advise X if a binding agreement with Z
exists for the renewal of the lease for a further three years. Refer to Goldblatt v Fremantle. [15]
Answer
This question deals essentially with formalities stipulated by the parties for a valid creation of a
contract. The main question is whether a formality was stipulated in the oral agreement for the
renewal of lease between the parties, that for such agreement to be valid it should be reduced to
writing.
Parties to an oral agreement will often agree that their agreement should be reduced to writing, and
perhaps also signed. In doing so, they may have the following intentions:
1. To have a written record of their agreement to facilitate proof of its terms. If so, the agreement is
binding even if it is never reduced to writing.
2. Alternatively, they may intend that their oral agreement will not be binding upon them until it is
reduced to writing and signed by them. In Goldblatt v Fremantle, the Appellate Division held that no
contract existed because the parties intended their agreement to be concluded in writing, which also
involved signing by the parties.
In the absence of contrary evidence, the law presumes that the intention of the parties was merely
to facilitate proof of the terms of the agreement. The party who alleges otherwise bears the onus of
proof.
In our case no binding agreement exists because the parties agreed that the oral agreement must be
reduced to writing and signed, and this indicates their intention that the agreement will not be
binding if this formality is not complied with.

Question
Y let premises to X. The lease contained a clause prohibiting X from sub-letting the premises without
the written consent of Y. A further clause of the lease required that any variation of the terms of the
lease had to be in writing and signed by both parties. Later Y told X that he (X) could sub-let a portion
of the premises. After X had sub-let a portion of the premises to a third party, Y changed his mind
and informed X that both X and the sub-lessee (third party) must vacate the premises because X had
breached the contract. Discuss X’s position with reference to Sa Sentrale Kooperatiewe
Graanmaatskappy Bpk v Shifren.

Answer
The facts correspond to a large extent with Shifren. The question is whether parties may orally
deviate from a written agreement that contains a clause that determines that the contract may only
be varied or terminated in a specific manner (non-variation clause). In such instances, the parties
have actually set formalities for the amendment or termination of their contract.
In the Shifren case, the court decided in favour of the lessor even though the lessor apparently gave
permission verbally for the amendment of a lease agreement, which contained such a provision. The
lessor was entitled to cancel the contract as a result of the lessee’s breach despite the oral variation.
The same results would apply to the present case.
In the Shifren case the court’s reasoning was as follows:
Where the parties insert a clause into their contract that provides that any amendment of the
contract, including the specific clause, must be in writing, they cannot later orally amend that clause
or any other provision. However, if the specific clause itself is not entrenched against oral variation,
the particular provision may be varied orally, with the result that thereafter the other provisions of
the contract may possibly also be varied orally.
Question
X, the owner of Tex-Mex Fried Chicken in Town A, sells her business as a running business to Y for
R100 000. The contract of sale provides that X may not conduct a similar business in Town A and
Town B for a period of two years. Six months later, X opens a similar business in Town B. X uses the
same recipe she used when preparing the chicken. Y seeks to enforce this clause in the contract with
an interdict. It appears that, at the time of conclusion of the sale, the Tex-Mex Fried Chicken drew its
customers only from Town A and that Tex-Mex chicken is not prepared according to a secret recipe.
Will Y succeed? Discuss.

Answer
Y will only succeed if the agreement in restraint of trade is reasonable, but the onus of proving that it
is unreasonable rests on X (the contract denier). In this regard, the Basson test should be applied to
the facts of this problem.
The first question is whether Y has a protectable interest. Goodwill definitely exists as part of the
running business. There is no right to a trade secret because although the recipe is useful and has
economic value, it is not secret (it is public knowledge).
The second question is whether the goodwill will be threatened by the conduct of Y. The opening of a
similar business in Town B directly infringes the restraint.
The third question involves a weighing up of the interests of X and Y. The business only drew its
customers from Town A. This shows that the restraint goes further than necessary to protect the
goodwill of the business.
The conclusion is thus that the restraint is not reasonable as between the parties. But the enquiry
does not end here. The fourth question that should be asked is whether there is any other relevant
aspect of public policy which indicates that the restraint should be enforced. In our problem, there is
none.
Y will not be successful in enforcing the restraint against X.

Question
Tony, a petrol attendant, sells dagga to Samuel for R1000. Tony delivers the dagga to Samuel but
Samuel refuses to pay. Section 5 of the Drugs and Drug Trafficking Act provides that no person shall
deal in dagga while section 4 prohibits possession of such substances. Section 13 makes the
contravention of both sections 4 and 5 a crime. Dagga is a substance as defined in section 5. Advise
Tony if he can sue Samuel for payment of R1000 or the return of the dagga. Would your advice be
different if Tony was an undercover policeman who sold dagga to Samuel during a police entrapment
operation? Discuss with reference to Jajbhay v Cassim and other relevant case law. [15]

Answer
This question involves an illegal contract of sale, which is void due to statutory illegality. The fact that
the legislator has enacted a criminal sanction for a contravention is a factor that would imply that the
legislator intended the contract to be void.
An illegal contract creates no obligations and it cannot be enforced. The ex turpi rule applies: from an
illegal cause no action arises. Neither party can institute an action on the contract or claim
performance from the other party. So for instance if a party has suffered damage as a result of such a
contract, he or she may not claim contractual damages from the other party. A court does not have
the discretion to relax this rule and there are no exceptions to it.
A party who has performed in terms of an illegal contract may however reclaim his performance, in
principle, with an enrichment action. However, such restitution will be prevented where the par
delictum rule applies. According to the par delictum rule: where two parties are equally morally
guilty, the one who is in possession is in the stronger position. If this is the case, restitution in terms
of an enrichment action is prevented.
In our case, Tony is precluded from instituting any contractual claim for R1000 from Samuel because
of the ex turpi rule, and also from an enrichment claim because of the par delictum rule.
The situation might differ if Tony was an undercover cop. In such a case, Tony would not be equally
morally guilty (Minister of Justice v Van Heerden) and so the par delictum rule would not apply.
In Jajbhay v Cassim, the Appellate Division held that the par delictum rule may be relaxed in
appropriate circumstances in order to justice “between man and man” if it would be in the interests
of public policy.

Question
X hands in her shocking pink suede jacket at the dry-cleaner. Y hands her a receipt. On the back of
the receipt is a clause excluding Y’s liability in the event of negligent damage to or theft of any goods
handed in for dry-cleaning. The same words appear on a big notice board in the shop, which is
clearly visible. When X fetches her jacket, she is dismayed to discover that the drycleaning process
has changed the jacket’s colour. Is she bound by the exemption clause? Discuss briefly. [5]

Answer
With so-called ticket contracts, one of the parties issues a ticket on which certain contractual terms
appear. The question is whether the other party may be held bound to such terms where that party
has not signed the ticket in question. Our courts use a three-legged test:
1. Did the person know there was writing on the ticket?
2. Did he know that the writing referred to terms of the contract?
If both answered in the affirmative, the terms form part of the contract. If either answered negative,
a further question follows:
3. Did the party who issued the ticket take reasonable steps to bring the reference to the terms to
the attention of the other party?
In the present case, X will probably be held bound because of the notice board that also refers to the
contractual terms.

Question
Andy and Craig conclude a contract wherein Andy agrees to paint
Craig’s office block by 31 August, and Craig agrees to pay Andy R10 000 upon completion of the
work. When 80% of the work is completed Andy suddenly falls ill and he is unable to complete the
job by 31 August. Craig refuses to pay Andy any money for his (Andy’s) services rendered, as Craig
believes that Andy has breached the contract by not completing the work. Craig hires another
contractor at an amount of R3000 to complete the job. Craig does not incur any other costs to
complete the job, neither does his business make any losses. Advise Andy as to what amount (if any)
he may recover from Craig for the services that he rendered, and on what basis. Discuss with
reference to BK Tooling (Edms) Bpk v Scope Precision Engineering (Edms) Bpk and other relevant case
law. [15]

Answer
This contract is reciprocal in nature. Andy has rendered defective performance and the issue is
whether Craig has to compensate Andy for the work that has already been done.
This question deals with the exceptio non adimpleti contractus. The exceptio is a defence that can be
raised in the case of a reciprocal contract, where the performances due on either side are promised
in exchange for one another. It is a remedy that permits a party to withhold their performance and
ward off a claim for such performance until such time as the other party has either performed or
tendered performance of their obligations.
Where a party who has to perform first has only performed part of its obligations or has rendered
defective performance, that party is in principle not entitled to claim counter-performance until such
time as he has performed in full. In practice, the innocent party often accepts part-performance and
starts using the performance. This sometimes leaves the breaching party in the unfair position that it
may be impractical or impossible to make full performance, but any claim for counter-performance
can be defended by the other party relying on the exceptio.
As a result, the courts have exercised a discretion to relax the principle of reciprocity and order the
party making use of the defective or incomplete performance to pay a reduced amount to the party
in breach.
In BK Tooling, the Appellate Division confirmed this, and held that the courts have an equitable
discretion to award a reduced contract price, depending on the nature of the defect, and the cost of
repair, replacement, or substitute performance. The onus to prove the amount of reduction is on the
party in breach claiming the reduced price. The plaintiff must allege and prove:
• that the other party is using his performance
• the cost of remedying defects
• that it would be equitable to award some remuneration despite breach
• that the circumstances are such that the court should exercise its discretion
Based on the ruling in BK Tooling, Andy is entitled to be compensated by Craig because:
1. Craig is utilising the defective performance
2. It would be equitable as Andy has completed most of the work
3. The counter-performance ought to be reduced by R3000 (the amount it cost to complete the job)
In the circumstances, Andy is entitled to receive R7000 from Craig, which represents the difference
between the contract price and the cost to complete the job.

Question
On 1 June M and Q conclude a contract whereby M undertakes to manufacture and install kitchen
cupboards in Q’s home for R50 000. The parties agree that the price will be paid as soon as the
kitchen cupboards are installed, but they do not determine a date for the completion of the work. M,
however, informs Q during the negotiations that she has some other work to complete and that she
will attend to the kitchen cupboards as soon as possible. Eight months has lapsed since the contract
was concluded and Q has not heard from M. Q runs out of patience and hires W to manufacture and
install the same kitchen cupboards for R60 000. After W has completed the job, M turns up to do the
work. Q claims R10 000 damages from M, but M institutes a counterclaim for R30 000 from Q for her
loss of profit. Who will succeed in this claim? Discuss.
Answer
This question deals with damages for breach of contract. In order to determine who will succeed in
the claim for damages, we must ascertain which party committed the breach.
A plaintiff who wishes to claim damages for breach of contract must prove the following:
1. A breach of contract has been committed by the defendant
2. The plaintiff has suffered financial or patrimonial loss
3. There is a factual causal link between the breach and the loss
4. As a matter of legal causation, the loss is not too remote a consequence of the breach.
Did M or Q breach the contract?
M could possibly be in breach in the form of mora debitoris. Mora debitoris is the unjustifiable failure
of a debtor to make timeous performance of a positive obligation that is due and enforceable, and
still capable of performance in spite of such failure.
Because performance has become impossible, it is not capable of performance. Also, no date was
stipulated for performance, nor did Q demand performance, so M could neither be in mora ex re nor
mora ex persona respectively. M has not committed a breach of contract.
Q, by hiring W to manufacture the cupboards has committed two forms of breach: repudiation, and
prevention of performance. A party commits the breach of repudiation when, by words or conduct,
and without lawful excuse, he manifests an unequivocal intention no longer to be bound by the
contract or any obligation forming a part thereof.
Prevention of performance is a breach whereby, after conclusion of the contract, one of the parties,
owing to their fault, causes performance to become impossible.
M will therefore be able to claim damages from Q successfully, because he will be able to prove that
Q committed a breach of contract. The aim of damages is to place the innocent party in their
fulfilment position, that is, the position they would have been in had there been no breach. M’s claim
for loss of profit will probably be successful.

Question
X contracts with Y for the latter (Y) to build and fit a security gate for the entrance of her (X’s) home.
Y builds the gate and fits it with an electric motor, which is activated with a remote control. X is
satisfied with the work and pays Y the contractual amount agreed upon. A week later, the gate gets
stuck while it is halfway open as a result of defective materials used to build the gate. When X
attempts to physically move the gate to close it fully, she suffers such severe damage to her left knee
that she has to have a knee operation. Her medical costs are R20 000. The costs of repairing the gate
amount to R15 000. X wants to claim both medical costs as well as the cost of repairing the gate from
Y. Advise X if she will be successful with her claim. Refer to Shatz Investments (Pty) Ltd v Kalovymas;
Holmdene Brickworks (Pty) Ltd v Roberts Construction Co, and other relevant case law in your
answer. [15]

Answer
This question deals with a claim for damages for breach of contract, and specifically, the element of
legal causation regarding special damages and general damages.
A plaintiff who wishes to claim damages for breach of contract must prove:
1. A breach of contract has been committed by the defendant
2. The plaintiff has suffered financial or patrimonial loss
3. There is a factual causal link between the breach and the loss
4. The loss is not too remote a consequence of the breach (legal causation).
Y has committed a breach of contract in the form of positive malperformance (the defective
materials used to build the gate).
In the law of contract, the approach to remoteness of consequences from breach (legal causation)
has been traditionally based on a distinction between general and special damages. The distinction
between general damages and special damages was stated in Holmdene Brickworks: general
damages are those damages that flow naturally and generally from the kind of breach in question
and which the law presumes the parties contemplated as a probable result of the breach; special
damages, on the other hand, are presumed to be too remote unless exceptional circumstances are
present.
X may claim the cost of repairing the gate as general damages.
If X wants to succeed in the claim for medical costs as special damages, X must prove that:
1. The damages were actually foreseen or reasonably foreseeable at the time of entry into the
contract (the contemplation principle); and
2. The parties can be taken to have agreed that there would be liability for damages arising from
special circumstances (the convention principle).

Question
Discuss the impact of the Consumer Protection Act 68 of 2008 upon the law of contract with
reference to its aims, objectives, scope, national regulatory institutions, and sanctions. [15]

Answer
The CPA is bound to have a huge impact on the conduct of businesses in South Africa, and the law of
contract.
The primary purpose of the Act is to protect consumers from exploitation in the marketplace, and to
promote their social and economic welfare. More specifically, it aims to:
• Establish a legal framework for the achievement and maintenance of a consumer market that is
fair, accessible, efficient, and responsible, for the benefit of consumers generally;
• Promote fair business practices;
• Protect consumers from unconscionable, unjust, or unreasonable business practices.
The scope of the Act is very wide. It applies to:
• Most transactions concluded in the ordinary course of business between suppliers and consumers
within South Africa, as well as;
• The promotion of goods and services that could lead to such transactions, and;
• The goods and services themselves once the transaction has been concluded.
A supplier is any person (including a juristic person, trust, and organ of State) who markets any goods
or services.
A consumer includes not only the end-consumer of goods and services but also:
• Franchisees
• Relatively small businesses in the supply chain (asset value or annual turnover below the threshold
determined by the Minister)
The Act does not apply to any transaction in terms of which goods and services are promoted or
supplied:
• To the State
• To a juristic person with an asset value or annual turnover above the threshold
• Employment contracts
• Credit agreements
• Transactions exempted by the Minister
These rights are protected and enforced not only through the courts, but the National Consumer
Commission and the National Consumer Tribunal. Failure to comply with provisions of the Act might
attract various sanctions, commencing with compliance notices and leading possibly to the
imposition of fines and criminal penalties. Contractual provisions in contravention of the Act may be
declared null and void to the extent of non-compliance.
Question
List and very briefly discuss the requirements for a valid offer and acceptance. [10]

Answer
OFFER:
• Must be firm.
(That is to say, with the intention that its acceptance will call into being a binding contract.)
• Must be complete.
(It must contain all the material terms of the proposed agreement.)
• Must be clear and certain.
(It should be enough for the addressee to answer merely “yes” for a contract to come into being.)
• Must meet the requirements of the Consumer Protection Act.
ACCEPTANCE:
• Must be unqualified.
(It must be a complete and unequivocal assent to every element of the offer.)
• Must be by the person to whom the offer was made – Bird v Summerville.
(E.g. the offer to sell farm A cannot be accepted by A and B jointly.)
• Must be a conscious response to the offer – Bloom v American Swiss Watch Co.
(A person cannot accept an offer if he was not aware of it.)
• Must be in the form prescribed by the offeror, if any.
State the ways an offer may be terminated.
1. Rejection of the offer
2. Acceptance of the offer
3. Effluxion of the prescribed time, or of a reasonable time
4. Death of either party
5. Revocation of the offer
6. Loss of legal capacity to act

Question
Discuss and distinguish between an option and a right of preemption. [10]

Answer
An option is a substantive offer, reinforced by an agreement in terms of which the offeror undertakes
to keep his offer open to the offeree for a specified period.
A right of pre-emption is a type of right of preference. It is given by a prospective seller to a
prospective purchaser, to give the purchaser preference if the prospective seller should decide to
sell.
There are significant differences between the two:
In the case of an option to buy, the grantor has already made a firm offer to the grantee, and the
power to conclude the sale lies exclusively in the hands of the grantee.
With a pre-emption agreement, however, there is as yet no firm offer “on the table” – merely an
undertaking to make an offer to the grantee if the trigger event occurs (usually, if the grantee decides
to sell the property). The grantor accordingly retains the power to decide whether or not to sell, and
cannot be compelled to do so unless or until the trigger event has occurred.

Remedies for breach:


Remedies for the breach of an option contract are governed by the general principles of the law of
contract. An attempted revocation of the substantive offer does not prevent the exercise of the
option, and the option holder may enforce the contract specifically by means of an interdict against
the grantor of the option. The option holder may also claim damages, if suffered, to place him or her
in the position that he or she would have been if the option had been exercised.
Remedies for the breach of a pre-emption contract by the grantor are a bit less certain. The
Appellate Division in Owsianick held that a right of pre-emption entails a restriction on alienation and
that the holder was entitled to an interdict to prevent the grantor from alienating the thing to a third
party. Furthermore, it held that the only other remedy available to the holder was a claim for
damages.
In Associated SA Bakeries the court also doubted whether the holder could claim specific
performance by means of an order directing the grantor to make him an offer, but granted a different
approach, which was set out as follows:
• If a seller concludes a contract of sale with a third party contrary to a preemptive right, the
purchaser can step into the shoes of the third party by a unilateral declaration of intent. A contract of
sale will then be deemed to have been concluded between the seller and the holder of the
preemptive right.
• Should delivery already have taken place, the holder of the right would not be able to pursue the
merx in the hands of the third party with his or her personal right, unless the third party was aware
of the existence of the pre-emptive right.
Formalities:
Option contract
In Brandt v Spies, the defendant orally granted an option to the plaintiff to purchase his farm.
Disregarding the option, he sold the farm to a third party. The plaintiff who exercised the option in
writing, then claimed damages for breach of contract, but an exception to his claim was upheld. The
court stated that an undertaking to keep open an offer that is invalid can itself confer no right upon
the grantee because there is nothing to keep open. It is submitted that this is correct in the case
where the offer and the offer to keep the first offer open are made simultaneously and orally.
However, where the offer to sell is made in writing on one occasion and is followed by an oral offer to
keep the offer to sell open at a later stage, it is submitted that the oral acceptance of the option
contract is valid.
Pre-emption contract
If the object of the envisaged sale is land, both the offer to buy (or sell) and the acceptance thereof
must be in writing. The contract from which the right of pre-emption arises also has to be in writing
(Hirschowitz v Moolman).

Question
State the requirements for duress and undue influence.

Answer
DURESS (improper pressure that amounts to intimidation):
1. Actual violence or reasonable fear
2. The fear must be caused by the threat of some considerable evil
3. It must be the threat of an imminent or inevitable evil
4. The threat or intimidation must be contra bonos mores
5. The moral pressure must have caused damage
UNDUE INFLUENCE (The party who seeks to set aside the contract must establish):
1. The other party obtained an influence over the party
2. This influence weakened his or her powers of resistance and rendered his will compliant
3. The other party used this influence in an unscrupulous manner to persuade him or her to agree to
a transaction that
a. was prejudicial to him or her
b. he or she would not have concluded with normal freedom of will

Question
State the elements for commercial bribery as held in Extel Industrial (Pty) Ltd v Crown Mills (Pty) Ltd.

Answer
1. A reward
2. paid or promised
3. by one party, the briber
4. to another, the agent (agent in true sense or merely a go-between)
5. who is able to exert influence over
6. a third party, the principal
7. without the principal’s knowledge, and
8. for the direct or indirect benefit of the briber
9. to enter into or maintain or alter a contractual relationship
10. with the briber, his principal, associate, or subordinate.

Question
State the requirements for restitutio in integrum.

Answer
1. Misrepresentation by the other party
2. Inducement
3. Intention to induce
4. Materiality

Question
State the elements of a fraudulent misrepresentation.

Answer
1. A representation
2. which is, to the knowledge of the representor, false;
3. which the representor intended the representee to act upon;
4. which induced the representee to act; and
5. that the representee suffered damage as a result

Question
Define misrepresentation.

Answer
A misrepresentation is generally a false statement of past or present fact (not law or opinion) made
by a contractual party to another prior to the conclusion of a contract and regarding some matter or
circumstance relating to the contract.

Question
Define dictum et promissum.

Answer
A material statement made by the seller to the buyer during negotiations, bearing on the quality of
the res vendita and going beyond mere praise and commendation.

Question
State the test to determine if a restraint of trade clause is enforceable (Basson test). [5]

Answer
1. Is there an interest of one party worthy of protection?
2. If so, is that interest threatened by the conduct of the other party?
3. If so, does such interest weigh up against the interest of the other party to be economically active
and productive?
4. Is there another aspect of public policy that requires that the restraint should be maintained or
rejected?

Question
Discuss the factors taken into consideration in determining whether the legislator impliedly intended
the contract to be void for statutory illegality.

Answer
1. What is the object of the statute and what mischief (harm) is the statute directed against? If the
validity of the contract brings about the harm the statute is directed against, it is an indication that
the legislator intended the contract to be void.
2. Does the enactment impose a criminal sanction? This is usually an indication that the legislator
intended the contract to be void. However, this is not the case where the sanction provides adequate
protection against the mischief that the statute is directed against.
3. Does the enactment merely serve to protect the revenue of the State? If the answer is in the
affirmative, it is an indication that the legislator intended the contract to be valid.
4. Does the provision merely protect individuals or does it involve a public interest that requires
protection by voiding the contract? If the provision is for the protection of the public, it would be an
indication that the legislator intended the contract to be void.
5. What are the consequences of a particular interpretation of the contract?
A balance-of-convenience test is employed that questions whether nullity of the contract would
cause greater inconvenience and justice than allowing the illegal conduct to stand

Question
Distinguish between initial impossibility of performance, supervening impossibility of performance,
and prevention of performance. [10]

Answer
If a performance is objectively impossible at the time of conclusion of a contract, no obligation
arises. To render performance impossible, it is not sufficient that a particular party cannot perform,
that is, subjective impossibility. The impossibility must be so serious that nobody can render the
performance – that is, it must be objectively impossible. An example of impossible performance is
where A agrees to sell his house to B, but unbeknown to them the house has already been destroyed
by a fire. Initial impossibility of performance prevents a contract from arising at all.
If, after the conclusion of the contract, performance becomes objectively impossible without the
fault of the debtor, as a result of an unavoidable and unforeseen event, this is known as supervening
impossibility of performance, and the obligation to perform is also, as a general rule, extinguished.
The requirements for supervening impossibility of performance are:
1. the performance must be objectively impossible; and
2. the impossibility must be unavoidable by a reasonable person.
If, after the conclusion of the contract, performance on either side becomes impossible owing to the
fault of either the debtor or the creditor, the contract is not terminated, but the party who rendered
the performance impossible is guilty of a breach of contract known as prevention of performance. It
is not necessary that the performance should be objectively impossible in order for the breach to
arise; subjective impossibility will suffice.

Question
Distinguish between suspensive conditions, resolutive conditions, suspensive time clauses, and
resolutive time clauses.

Answer
SUSPENSIVE CONDITION:
Performance of an obligation (which is an uncertain future event which may or may not occur) is
suspended, and enforceable only when that event has been fulfilled or has failed.
RESOLUTIVE CONDITION:
Performance of obligations should operate in full, but will come to an end if an uncertain future
event does or does not happen.
SUSPENSIVE TIME CLAUSE:
Performance of obligations postponed/suspended until an event or time that is certain to arrive in
the future.
RESOLUTIVE TIME CLAUSE:
Obligations terminate at a certain date or happening of a certain future event.

Question
Briefly discuss tacit terms. [5]

Answer
A tacit term is one that the parties did not specifically agree upon, but which
(without anything being said) both or all of them expected to form part of their (oral or written)
agreement. It is a wordless understanding having the same legal effect as an express term.
In ascertaining whether a contract contains a tacit term, the courts often employ the officious
bystander test:
The court supposes that an impartial bystander had been present when the parties concluded their
agreement and had asked the parties what would happen in a situation they did not foresee and for
which their express agreement did not provide. If they were to agree that the answer to the
stranger’s question was self-evident, they are taken to have meant to incorporate the term into their
contract and to have tacitly agreed on it.

Question
What is the parol evidence rule?

Answer
The parol evidence rule declares that where the parties intended their agreement to be fully and
finally embodied in writing, evidence to contradict, vary, add to, or subtract from the terms of the
writing is inadmissible.

Question
State the different forms of breach of contract.

Answer
1. Mora debitoris
2. Mora creditoris
3. Positive malperformance
4. Repudiation
5. Prevention of performance
Question
Discuss mora debitoris and mora creditoris and distinguish between them. [10]

Answer
MORA DEBITORIS:
Mora debitoris is the unjustifiable failure of a debtor to make timeous performance of a positive
obligation that is due and enforceable and still capable of performance in spite of such failure.
Requirements:
• The debt must be due and enforceable
• The time for performance must have been fixed, either in the contract or by a subsequent demand
for performance, and the debtor must have failed to perform timeously.
• Such failure to perform on time must be without legal justification.
Mora ex re occurs where the debtor fails to perform on or before the due date expressly or impliedly
stipulated by the parties in their contract.
Mora ex persona occurs where no time for performance has been stipulated, and the creditor
demands that the debtor perform on or before a definite date that is reasonable in the
circumstances (by means of a letter of demand, or oral demand).
MORA CREDITORIS:
Mora creditoris is a form of breach of contract by a creditor. It occurs in cases where a creditor is
obliged to lend his or her cooperation, and culpably fails to do so timeously.
Requirements:
• The debtor must be under an obligation to make the performance to the creditor (the performance
need not be enforceable or due, however).
• Cooperation of the creditor must be necessary for the performance by the debtor of his obligation.
• The debtor must tender performance to the creditor.
• The creditor must delay in accepting performance.
• The delay must be due to the fault of the creditor.

Question
Define repudiation.

Answer
Repudiation is the demonstration by a party, by words or conduct, and without lawful excuse, of an
unequivocal intention no longer to be bound by the contract or by any obligation forming part of the
contract.

Question
State the requirements than an innocent party must prove in order to succeed with a claim for
damages. [5]

Answer
1. A breach of contract has been committed by the defendant.
2. The plaintiff has suffered financial or patrimonial loss.
3. There is a factual link between the breach and the loss.
4. As a matter of legal causation, the loss is not too remote a consequence of the breach.

Question
Write notes on the difference between general damages and special damages. [5]

Answer
General damages are those which flow naturally and generally from the breach in question, and the
law presumes that the parties contemplated them as a possible result of the breach. The guilty party
is summarily held liable for general damages.
In contrast, special damages are those that do not flow naturally and generally from a specific form
of breach. The guilty party is only liable for special damages in certain circumstances. The courts use
two principles to determine the extent of liability in the case of special damages: the contemplation
principle, and the convention principle.
In terms of the contemplation principle, liability is restricted to damages that the parties actually or
reasonably must have contemplated as a probable consequence of the breach.
According to the convention principle, liability is limited to those damages that may be proved on the
basis of the contract. The innocent party has to prove either an express or implied provision
concerning the payment of damages.

Question
Discuss the exceptio non adimpleti contractus with regard to its definition, the principles of
reciprocity, how reciprocity is to be determined, as well as when the defence can be raised. Refer to
case law in your answer. [10]

Answer
The exceptio non adimpleti contractus is a defence that can be raised in the case of a reciprocal
contract. It is a remedy aimed at keeping the contract alive. It permits a party to withhold his or her
own performance, and to ward off a claim for such performance until such time as the other party
has either performed or tendered proper performance of his or her own obligations under the
contract.
The exceptio non adimpleti contractus is available when two requirements are met:
1. the two performances must be reciprocal to one another
2. the other party must be obliged to perform first, or at least simultaneously with the party raising
the exceptio. The exceptio may also be raised where a party has performed incompletely.
In BK Toolings (Edms) Bpk v Scope Precision Engineering (Edms) Bpk, the court stated that reciprocal
obligations are obligations that have been created in exchange for each other.
In order to determine whether an obligation is reciprocal or not, the (express or tacit) intention of
the parties must be determined by interpreting the agreement. The question to be asked is: did the
parties intend to create obligations in exchange for each other?

Question
State the requirements for a valid cession.

Answer
1. An entitlement by the cedent to dispose of the personal right
2. The capacity of the personal right to be ceded
3. A transfer agreement
4. Formalities
5. Legality
6. Absence of prejudice to the debtor

Question
State the ways in which obligations may be terminated.

Answer
1. By performance
2. By agreement
a. Release and waiver
b. Novation
c. Compromise
d. Effluxion of time
e. Notice
3. By law
a. Set-off
b. Merger
c. Supervening impossibility of performance
d. Prescription
e. Insolvency
f. Death

Question
Write brief notes on release and waiver.

Answer
A release is an express or tacit agreement that the debtor be freed from an obligation or obligations.
It therefore has the effect that the debtor need not perform. The debtor may be released in whole or
in part.
The term “waiver” is often used synonymously with the concept of a release agreement. However,
sometimes waiver is used to denote a unilateral act of abandoning a right or remedy that exists for
the sole benefit of the party abandoning the right or remedy.

Question
Write brief notes on novation.

Answer
A novation is an agreement to extinguish or replace one or more existing obligations with a new
obligation. Accessory obligations to the original debt, such as a pledge or suretyship, are
extinguished by an agreement to novate the debt.
The parties may agree to replace the debtor with a third party, provided of course that the third
party agrees to such novation. Replacement of a debtor by novation is called delegation.
If an original obligation is void, a novation of the obligation will also be void. But if the novation itself
is void, the original obligation will continue to exist.

Question
Write brief notes on compromise.

Answer
Compromise is an agreement in terms of which parties settle a dispute or some uncertainty between
themselves.
Compromise differs from true novation in that compromise does not require a valid old obligation to
have existed.
The purpose of a compromise is to secure a final settlement of a dispute or uncertainty, sometimes
as to whether there is a debt at all.

Question
Write brief notes on set-off, and the requirements therefor

Answer
Where two parties have claims against each other, and the requirements for setoff are met, the
debts can extinguish each other. If they are not for the same amount, the smaller debt is
extinguished and the larger debt is reduced by the amount of the smaller debt.
The following four requirements must be met for set-off to operate:
1. The debts must exist between the same two persons in the same capacities
2. The debts must be of the same kind or nature
3. Both debts must be due and enforceable
4. Both debts must be liquidated

Question
Albert takes his motor vehicle to Dodgy Motors for a service. On his arrival, he is asked to sign a “job
card” by the owner. Albert enquires why he is required to sign the “job card” and the owner explains
to him that by signing he is authorising them to conduct the service on his car, which will cost R1000.
He signs the “job card” without reading it. While servicing the car, the service manager finds faults
on the car (unrelated to the service) and he proceeds to do these additional repairs for a further
R2000. Albert refuses to pay for the additional repairs and argues that he did not authorise such
repairs. The owner of Dodgy Motors argues that Albert is obliged to pay for the work done as the
“job card” contains a contractual clause authorising Dodgy Motors to do any repairs on the motor
vehicle which they deem necessary, without asking the client’s authorisation, and requiring the client
to pay for such repairs. Advise Albert on whether he is liable on the contract to pay Dodgy Motors
R2000 for the additional repairs. Refer to George v Fairmead (Pty) Ltd, Sonap Petroleum (SA) Ltd (SA)
(Pty) Ltd v Pappadogianis, and other relevant case law in your answer.
Do not apply the Consumer Protection Act to this question. [15]

Answer
The essence of this problem is the question whether Albert and the owner of Dodgy Motors have
reached actual consensus or ostensible consensus. Albert will not be contractually bound to pay for
the additional repairs if this requirement for a valid contract is absent.
At the outset, it must be determined whether agreement (consensus ad idem) as a contractual basis
exists between the parties, as required in terms of the will theory. Consensus has three elements:
1. The parties must seriously intend to contract
2. The parties must be of one mind as to the material aspects of the proposed agreement (the terms
and the identity of the parties to it)
3. The parties must be conscious of the fact that their minds have met
In the present case, the parties were not in agreement as to the consequences they wished to
create; Albert thought that he was authorising Dodgy Motors to only service his car, while the owner
of Dodgy Motors knew that the contract also allowed Dodgy Motors to conduct repairs which they
deemed necessary and payable by Albert without any further authorisation from Albert. This is a
mistake as to the obligations the parties wished to create and is thus a material mistake, which
excludes consensus between the parties. This means that no contract could arise on the basis of the
will theory.
This type of mistake can be illustrated with a number of cases:
In George v Fairmead, the appellant signed a hotel register without reading it.
The register contained a term excluding the respondent from liability for certain acts. The appellant
was unaware of this term and his mistake related to a term that he believed would not be in the
contract and as such was material because it related to an aspect of performance.
In Allen v Sixteen Stirling Investments, the plaintiff believed he was purchasing the erf pointed out to
him by the seller’s agent, while the written contract that he signed indicated the correct erf, which
was a completely different property. His mistake related to performance and was material.
However, the matter does not end here. A party may be held contractually liable on the basis of a
supplementary ground for liability, namely the reliance theory.
In this regard, the direct or indirect approach to the reliance theory may be considered.
DIRECT APPROACH:
With reference to the direct approach, contractual liability is based on the reasonable reliance that
consensus has been reached, which the one contractant (the contract denier) creates in the mind of
the other contractant (the contract enforcer).
According to the Sonap case, the direct reliance approach entails a threefold enquiry:
1. Was there a misrepresentation regarding one party’s intention?
2. Who made the misrepresentation?
3. Was the other party misled by the misrepresentation, and if so, would a reasonable person have
been misled?
In our question, firstly, Albert made a misrepresentation by signing the contract, that his intention is
the same as that expressed in the contract.
Secondly, the owner of Dodgy Motors could actually have been misled by this misrepresentation, but
a reasonable man would have taken steps to point out to Albert that the contract allows Dodgy
Motors to unilaterally conduct repairs on the car, because Albert enquired about the purpose of the
“job card” and the owner of Dodgy Motors misled him to believe that by signing the card he is
merely authorising the service to be done. In Sonap the court found that the contract enforcer knew
that the contract denier was acting under a mistake and was thus not misled.
In our case, Albert therefore did not create a reasonable reliance that he wished to be bound by the
contract he signed.
INDIRECT APPROACH (IUSTUS ERROR DOCTRINE):
In terms of this approach, a party may escape liability to be bound to a contract if it can be
established that the mistake is both:
2. Reasonable
It has already been shown in the discussion above that Albert’s mistake is material. It still has to be
determined if his mistake was reasonable.
The contract denier’s mistake will be reasonable in the following instances:
1. If caused by a misrepresentation on the part of the contract enforcer (an unlawful
misrepresentation).
2. If the contract denier is not to blame for the mistake.
3. If the contract denier did not cause a reasonable belief in the contract asserter that the contract
denier assented to the agreement.
Fault is not a requirement for the misrepresentation by the contract enforcer, but unlawfulness is. If
the misrepresentation is a positive act it is unlawful in itself. If a legal duty to speak exists and the
party has kept quiet when he ought to have spoken, an unlawful negative misrepresentation has
occurred. A legal duty to speak exists in the following instances:
• Where the contract asserter knows or ought to know as a reasonable person that the other party is
mistaken
• Where, prior to the conclusion of the agreement, the contract asserter created an impression
directly conflicting with the provisions of the agreements, he must draw the contract denier’s
attention to the discrepancy (Du Toit v Atkinson’s Motors).
In our problem, Albert enquired about the purpose of the “job card” and the owner of Dodgy Motors
misled him by answering that by signing he was merely authorising the service. The owner’s
misrepresentation was a positive act, and was therefore unlawful. Albert’s error was thus
reasonable.
Applying the indirect approach to the reliance theory we do not have a valid contract. Applying the
direct approach, we do not have a valid contract. Albert is not contractually liable to pay R2000 for
the repairs.

Question
X, an organiser of art exhibitions, contracted with Y for an exhibition to be held on 24 to 27 July.
These dates were the only dates mentioned during negotiations. After having been pressurised by X,
Y hurriedly signed a standard form contract without reading it. The contract contained a clause
permitting X to change the dates of the exhibition unilaterally. Thereafter, X changed the dates. X had
no reason to believe that Y would have signed the contract if he had known of the term. Y averred
that the contract was void. Will Y succeed in his attempt to have the contract set aside? Substantiate
your answer and refer to relevant case law. Do not apply the Consumer Protection Act to this
problem. [15]

Answer
The essence of this problem is whether X and Y have reached consensus. Y will not be contractually
bound if this requirement for a valid contract is absent.
At the outset, it must be considered whether agreement (consensus ad idem) as a contractual basis
exists between the parties, as required in terms of the will theory. Consensus has three elements:
1. The parties must seriously intend to contract
2. The parties must be of one mind as to the material aspects of the proposed agreement (the terms
and the identities of the parties to it)
3. The parties must be conscious of the fact that their minds have met.
In the present case, the parties were not in agreement as to the consequences they wished to
create: Y thought that the dates for the exhibition (X’s performance) was fixed, while X knew that the
contract allowed X to unilaterally change the dates. This is a mistake as to the obligations the parties
wished to create and is thus a material mistake, which excludes consensus between the parties. This
means that no contract could arise on the basis of the will theory.
This type of mistake can be illustrated with a number of cases:
In George v Fairmead, the appellant signed a hotel register without reading it.
The register contained a clause excluding the respondent from liability for certain acts. The appellant
was unaware of this term and his mistake related to a term that he believed would not be in the
contract and as such was material because it related to an aspect of performance.
In Allen v Sixteen Stirling Investments, the plaintiff believed he was purchasing the erf pointed out to
him by the seller’s agent, while the written contract that he signed indicated the correct erf, which
was a completely different property. His mistake related to performance and was material.
However, the matter does not end here. A party may be held contractually liable on the basis of a
supplementary ground for liability, namely the reliance theory. In this regard, the direct or indirect
approach to the reliance theory may be considered.
INDIRECT APPROACH (IUSTUS ERROR DOCTRINE):
In terms of this approach, a party may escape liability to be bound to a contract if it can be
established that the party laboured under a mistake, which was both:
1. material and
2. reasonable.
It has already been shown that Y’s mistake is material in the discussion above. It still has to be
determined if Y’s mistake was reasonable.
The contract denier’s mistake will be reasonable in the following circumstances:
1. If caused by a misrepresentation on the part of the contract asserter (an unlawful
misrepresentation)
2. If the contract denier is not to blame for the mistake
3. If the contract denier did not cause the contract asserter to have a reasonable belief that the
contract denier assented to the contract.
If a legal duty to speak exists and a party has kept quiet when he ought to have spoken, that party
has made an unlawful negative misrepresentation. A legal duty to speak will usually exist where:
• The asserter knows or ought to know as a reasonable person that the other party is mistaken
• Where, prior to the conclusion of the agreement the asserter created an impression directly
conflicting with the provisions of the agreement, he must draw the contract denier’s attention to this
discrepancy (Du Toit v Atkinson’s Motors).
Since X had no reason to believe that Y would have signed the contract had Y known of the term
allowing X to change the dates of the exhibition unilaterally, X had a legal duty to point out this
clause to Y. X’s failure to do so renders Y’s material mistake reasonable.
DIRECT APPROACH:
With reference to the direct approach, contractual liability is based on the reasonable reliance that
consensus has been reached, which the one contractant (the contract denier) creates in the mind of
the other contractant (the contract asserter).
According to the Sonap case, the direct reliance approach involves a threefold enquiry:
1. Was there a misrepresentation regarding one party’s intention?
2. Who made this misrepresentation?
3. Was the other party actually misled by this misrepresentation, and if so, would a reasonable
person have been misled?
By signing the contract, Y, a party to the contract, misrepresented her intention to be bound by the
clause allowing X to unilaterally change the dates. X knew that the only dates mentioned during
negotiations were 24 to 27 July, that Y hastily signed the contract, and that the contract had a clause
allowing X to unilaterally change the dates. X was probably not actually misled by the
misrepresentation by Y, and nor would a reasonable person be misled in any event. There was
therefore no reasonable reliance on consensus on the part of Y.
Y is not bound by the agreement because of lack of apparent and actual consensus.

Question
X is a keen golfer who has played at many golf tournaments over the years as an amateur. She is very
well informed about the rules pertaining to her amateur status as a golfer, and knows that amateurs
can only claim a maximum of R1000 in prize money at golf tournaments. X participated in a recent
golfing tournament wherein she achieved a hole-in-one at the 9th hole. At this hole was an
advertising board, which read: “Hole-in-one prize sponsored by Speedy Motors to the value of R90
000”. The prize was parked next to this board in the form of a new car. X claimed the prize from
Speedy Motors but they rejected her claim on the basis that the prize could only be claimed by
professional players and not amateur players. Advise X. Refer to Steyn v LSA Motors and other
relevant case law. [15]

Answer
This problem deals with two questions: Was there a valid offer and acceptance?
Was there consensus between the parties?
Offer and acceptance:
The general rule in our law is that an advert constitutes merely an invitation to do business (Crawley
v Rex). However, following the reasoning in Carlill v Carbolic Smoke Ball Co, the court in Bloom v
American Swiss Watch Co held that the advertising of a reward might be construed as an offer to the
public. An offer may only be accepted by a person or persons to whom it was directed (Bird v
Summerville). Although Speedy Motors intended the offer to be open only to professional players,
the expressed offer was apparently open to the public.
Mistake is thus also relevant.
Mistake:
At the outset, it must be determined whether agreement (consensus ad idem) as a contractual basis
exists between the parties, as required in terms of the will theory. Consensus has three elements:
1. The parties must seriously intend to contract
2. The parties must be of one mind as to the material aspects of the proposed agreement (the terms
and the identities of the parties to it)
3. The parties must be conscious of the fact that their minds have met.
In our case, X and Speedy Motors were not in agreement as to the identity of the parties, and this is
a material mistake, which excludes consensus based on the will theory.
However, the matter does not end here. A party may be held contractually liable on the basis of a
supplementary ground for liability, namely the reliance theory.
In this regard, the direct reliance approach or the indirect reliance approach may be considered.
Because the facts in this case are similar to the case of Steyn v LSA Motors where it was held that the
indirect approach couldn’t be applied in instances where there is no objective appearance of
agreement, only the direct approach will be considered.
DIRECT APPROACH:
With reference to the direct approach, contractual liability is based on the reasonable reliance that
consensus has been reached, which the one contractant (the contract denier) creates in the mind of
the other contractant (the contract enforcer). According to the Sonap case, the direct reliance
approach entails a threefold enquiry:
1. Was there a misrepresentation regarding one party’s intention?
2. Who made this misrepresentation?
3. Was the other party actually misled by the misrepresentation, and if so, would a reasonable
person have been misled?
In our question, Speedy Motors made a misrepresentation regarding its intention that the offer is
made only to professional players, by advertising the reward to the public. Although it may be argued
that X was actually misled by the misrepresentation, it is certain that a reasonable person in X’s
position would not have been misled. X should know, as an experienced amateur golfer, that only
certain prizes are open to amateurs. There was therefore no reasonable reliance on consensus on
the part of X. X will not succeed in her claim for the prize.

Question
S, who lives in Upington, sends P, who lives in Grahamstown, a letter by private courier in which she
offers to sell him her (S’s) motorcycle, a collector’s piece, for R100 000. She states in her letter that
her offer will expire on 1 February. P accepts S’s offer by letter, which he posts on 31 January. S
receives the letter on 7 February and only reads it on the next day. P tenders payment of R100 000
but S refuses to accept payment. Did a valid contract arise between S and P? Substantiate your
answer. [15]

Answer
The question is whether P has accepted S’s offer in time and thus whether S and P have reached
consensus. Where the offeror has prescribed a time limit for acceptance, the offer lapses
automatically if it is not accepted within the prescribed period. The general rule is that a contract
comes into being only when the acceptance is communicated to the mind of the offeror. The
information theory, which is the general rule in our law, states that the agreement is concluded when
and where the offeror learns or is informed of the acceptance – in other words, when the offeror
reads the letter of acceptance.
On the other hand, the expedition theory applies to postal contracts. In terms of this theory,
introduced into our law in the Cape Explosive Works case, a contract comes into being when and
where the offeree posts the letter of acceptance. By making an offer through the post, the offeror is
deemed not only to have authorised acceptance by post, but also to have waived the requirement of
notification of acceptance.
The question that then arises is which theory applies. In our law, the general rule is that the
information theory applies, however the expedition theory will apply if the following four criteria are
met:
1. the offer is made by post or telegram
2. the postal services are operating normally
3. the offeror has not indicated a contrary intention, expressly or tacitly, and
4. the contract is a commercial one.
If any of these criteria are not met, the information theory applies.
In this question, the offer was not made by post, instead it was sent by private courier, and therefore
the expedition theory does not apply. It follows that the information theory must be applied.
Because S only learnt of the acceptance by P after expiry of the offer (when S read the letter on 8
February), the offer had already lapsed and no valid contract arose between the parties.

Question
X is on her way from work and sees a white bull terrier bitch hiding in a doorway. Being an animal
lover, she takes the dog home with her. The next day, she sees the following advertisement in the
newspaper:
Lost in Johannesburg, on 27 May. Pedigree white bull terrier bitch with black patch over left eye.
Answers to the name of Beauty. Reward of R1000 for information leading to safe return. Tel 011
555 5555. She realises that the dog she found matches the description given. She calls the advertiser
who rushes over to be joyfully united with Beauty. In his joy, Beauty’s owner, Y, seems to forget the
reward and X wishes to claim it from him. Will she be successful?
Substantiate your answer. Refer to Bloom v American Swiss Watch Co and other relevant case law in
your answer. [10]

Answer
X will only be successful in her claim if a valid contract arose between X and Y, and this will be the
case if there was a valid offer and acceptance.
The offer:
The offer was in the form of an advertisement. The general rule in our law is that an advertisement
constitutes an invitation to do business (Crawley v Rex).
However, in Bloom v American Swiss it was held that the advertising of a reward might be construed
as an offer to the public. Offers to the public at large can be made (Carlill v Carbolic Smoke Ball Co).
In our case, the offer was firm, complete, clear, and certain. The offer can therefore be said to have
been valid.
The providing of information by X was a valid acceptance of Y’s offer:
• X’s acceptance was unqualified
• X, as a member of the public to whom the offer was made, may accept (offer may only be accepted
by offeree – Bird v Summerville)
• X’s acceptance was a conscious response to the offer (he knew of the offer and could thus accept it
– unlike the situation of Bloom v American Swiss where the plaintiff returned the item but was
unaware there was a reward for doing so).
It can be concluded that a valid contract arose in this problem, because Y made a valid offer, which X
validly accepted.
Question
Y signs and delivers a written offer (including all the material terms) to Z on 1 July, for the purchase of
Z’s waterfront apartment. Y’s offer is for R800 000 and one of the terms of the offer states “This offer
lapses on 30 August”. However, whilst Z is still considering Y’s offer, Y delivers a letter to Z on 20 July,
advising Z that his (Y’s) offer is cancelled. Z insists that the offer is valid until 30 August, and on 25
July Z delivers a letter to Y, advising Y that he accepts Y’s offer. Has a valid contract of sale been
created between Y and Z? Discuss with reference to Brandt v Spies and other relevant case law. [10]

Answer
Contracting parties may enter into an agreement in terms of which the offeror undertakes not to
revoke his or her offer. In such cases, it is said that one party grants the other an option.
For this question, an option does not exist because there is no agreement in place that binds Y to
keep his offer open until 30 August. Y has unilaterally imposed this upon himself in the offer, but it
was certainly not an agreement by both parties to hold Y to keep his offer open until this date. This
means that no option contract was concluded.
Y validly revokes his offer to Z on 20 July and therefore there is no offer that Z can accept. The
requirements for a valid offer and acceptance for a contract have not been met, and no valid contract
has thus been created.

Question
X has been leasing a commercial property from Z for the past three years. The leas will come to an
end on 31 May 2010. On 5 March 2010, X phones Z and offers to renew the lease for a further three
years, which offer Z accepts. During this phone call, the material terms of the renewal agreement are
agreed upon and X and Z further agree that the said material terms must be reduced to writing and
signed by both parties. Subsequently, on 5 April 2010, X is shocked to receive a letter from Z, advising
X that there will be no renewal of the lease and that X should vacate the leased property on 31 May
2010. X and Z never reduced their oral agreement to writing. Advise X if a binding agreement with Z
exists for the renewal of the lease for a further three years. Refer to Goldblatt v Fremantle. [15]

Answer
This question deals essentially with formalities stipulated by the parties for a valid creation of a
contract. The main question is whether a formality was stipulated in the oral agreement for the
renewal of lease between the parties, that for such agreement to be valid it should be reduced to
writing.
Parties to an oral agreement will often agree that their agreement should be reduced to writing, and
perhaps also signed. In doing so, they may have the following intentions:
1. To have a written record of their agreement to facilitate proof of its terms.
If so, the agreement is binding even if it is never reduced to writing.
2. Alternatively, they may intend that their oral agreement will not be binding upon them until it is
reduced to writing and signed by them. In Goldblatt v
Fremantle, the Appellate Division held that no contract existed because the parties intended their
agreement to be concluded in writing, which also involved signing by the parties.
In the absence of contrary evidence, the law presumes that the intention of the parties was merely
to facilitate proof of the terms of the agreement. The party who alleges otherwise bears the onus of
proof.
In our case no binding agreement exists because the parties agreed that the oral agreement must be
reduced to writing and signed, and this indicates their intention that the agreement will not be
binding if this formality is not complied with.

Question
Y let premises to X. The lease contained a clause prohibiting X from sub-letting the premises without
the written consent of Y. A further clause of the lease required that any variation of the terms of the
lease had to be in writing and signed by both parties. Later Y told X that he (X) could sub-let a portion
of the premises. After X had sub-let a portion of the premises to a third party, Y changed his mind
and informed X that both X and the sub-lessee (third party) must vacate the premises because X had
breached the contract. Discuss X’s position with reference to Sa Sentrale Kooperatiewe
Graanmaatskappy Bpk v Shifren.

Answer
The facts correspond to a large extent with Shifren. The question is whether parties may orally
deviate from a written agreement that contains a clause that determines that the contract may only
be varied or terminated in a specific manner (non-variation clause). In such instances, the parties
have actually set formalities for the amendment or termination of their contract.
In the Shifren case, the court decided in favour of the lessor even though the lessor apparently gave
permission verbally for the amendment of a lease agreement, which contained such a provision. The
lessor was entitled to cancel the contract as a result of the lessee’s breach despite the oral variation.
The same results would apply to the present case.
In the Shifren case the court’s reasoning was as follows:
Where the parties insert a clause into their contract that provides that any amendment of the
contract, including the specific clause, must be in writing, they cannot later orally amend that clause
or any other provision. However, if the specific clause itself is not entrenched against oral variation,
the particular provision may be varied orally, with the result that thereafter the other provisions of
the contract may possibly also be varied orally.

Question
X, the owner of Tex-Mex Fried Chicken in Town A, sells her business as a running business to Y for
R100 000. The contract of sale provides that X may not conduct a similar business in Town A and
Town B for a period of two years. Six months later, X opens a similar business in Town B. X uses the
same recipe she used when preparing the chicken. Y seeks to enforce this clause in the contract with
an interdict. It appears that, at the time of conclusion of the sale, the Tex-Mex Fried Chicken drew its
customers only from Town A and that Tex-Mex chicken is not prepared according to a secret recipe.
Will Y succeed? Discuss.

Answer
Y will only succeed if the agreement in restraint of trade is reasonable, but the onus of proving that it
is unreasonable rests on X (the contract denier). In this regard, the Basson test should be applied to
the facts of this problem.
The first question is whether Y has a protectable interest. Goodwill definitely exists as part of the
running business. There is no right to a trade secret because although the recipe is useful and has
economic value, it is not secret (it is public knowledge).
The second question is whether the goodwill will be threatened by the conduct of Y. The opening of a
similar business in Town B directly infringes the restraint.
The third question involves a weighing up of the interests of X and Y. The business only drew its
customers from Town A. This shows that the restraint goes further than necessary to protect the
goodwill of the business.
The conclusion is thus that the restraint is not reasonable as between the parties. But the enquiry
does not end here. The fourth question that should be asked is whether there is any other relevant
aspect of public policy which indicates that the restraint should be enforced. In our problem, there is
none.
Y will not be successful in enforcing the restraint against X.

Question
Tony, a petrol attendant, sells dagga to Samuel for R1000. Tony delivers the dagga to Samuel but
Samuel refuses to pay. Section 5 of the Drugs and Drug Trafficking Act provides that no person shall
deal in dagga while section 4 prohibits possession of such substances. Section 13 makes the
contravention of both sections 4 and 5 a crime. Dagga is a substance as defined in section 5.
Advise Tony if he can sue Samuel for payment of R1000 or the return of the dagga. Would your
advice be different if Tony was an undercover policeman who sold dagga to Samuel during a police
entrapment operation? Discuss with reference to Jajbhay v Cassim and other relevant case law. [15]

Answer
This question involves an illegal contract of sale, which is void due to statutory illegality. The fact that
the legislator has enacted a criminal sanction for a contravention is a factor that would imply that the
legislator intended the contract to be void.
An illegal contract creates no obligations and it cannot be enforced. The ex turpi rule applies: from an
illegal cause no action arises. Neither party can institute an action on the contract or claim
performance from the other party. So for instance if a party has suffered damage as a result of such a
contract, he or she may not claim contractual damages from the other party. A court does not have
the discretion to relax this rule and there are no exceptions to it.
A party who has performed in terms of an illegal contract may however reclaim his performance, in
principle, with an enrichment action. However, such restitution will be prevented where the par
delictum rule applies. According to the par delictum rule: where two parties are equally morally
guilty, the one who is in possession is in the stronger position. If this is the case, restitution in terms
of an enrichment action is prevented.
In our case, Tony is precluded from instituting any contractual claim for R1000 from Samuel because
of the ex turpi rule, and also from an enrichment claim because of the par delictum rule.
The situation might differ if Tony was an undercover cop. In such a case, Tony would not be equally
morally guilty (Minister of Justice v Van Heerden) and so the par delictum rule would not apply.
In Jajbhay v Cassim, the Appellate Division held that the par delictum rule may be relaxed in
appropriate circumstances in order to justice “between man and man” if it would be in the interests
of public policy.
Question
X insures the contents of her house with Y Insurer. The insurance contract contains the following
clause:
If we reject liability for any claim made under this policy we will be released from liability unless
summons is served within 60 days of repudiation. X’s house is burgled and her TV and hi-fi are stolen.
X notifies Y
Insurer of the theft, but Y insurer rejects her claim. X only serves her summons 61 days after
rejection of her claim. X was involved during this period in a serious car accident and, as a
consequence, she was hospitalised for 30 days. Y Insurer raises the defence that
Y Insurer has been released from liability based on the relevant clause. Discuss whether the court
will uphold Y Insurer’s defence.

Answer
The facts of this problem are based on the Barkhuizen case. However, the time period for the
institution of the claim is far shorter, and the insured has a good reason for failing to be in time with
the institution of her claim.
The first question is whether the clause gives X a fair opportunity to seek legal redress.
The period is only two-thirds of the period in the Barkhuizen case (60 days versus 90 days), and it can
be validly argued that the short period in the clause amounts to a denial of the right to seek judicial
redress (a public interest informed by the constitutional right in section 34 of the Constitution).
It may furthermore be argued that the enforcement of the clause against X is unfair in the
circumstances, where X was in hospital for 30 days and only had 30 days to institute a claim against Y
Insurer, as this could amount to a denial of the right to seek judicial redress. X will have to have to
plead the illegality of the clause or the unfairness of enforcement. The onus will also rest on X to
prove illegality and unfairness. X will have to lead evidence on how long it usually takes to institute a
claim.
If we assume that the clause is illegal, a further question will arise: Can the clause be severed from
the rest of the insurance contract, as X would like to enforce the rest of the contract against Y insurer.
The guidelines that the courts use will have to be applied:
• Firstly, the clause is grammatically and notionally separate from the rest of the contract, as it forms
a separate clause.
• Secondly, if the clause is severed from the rest of the contract, the contract still stays an insurance
contract. The substantive character of the insurance contract remains intact.
• Thirdly, it could be argued that the insurer’s hypothetical intention would still be to enter into the
insurance contract, as no insurer can survive without business. Y Insurer would be faced with the
possibility that all its insurance contracts with such a clause would be invalid. X would, of course,
have entered into the insurance contract.
It can be concluded that the clause could be severed from the rest of the contract.

Question
X hands in her shocking pink suede jacket at the dry-cleaner. Y hands her a receipt. On the back of
the receipt is a clause excluding Y’s liability in the event of negligent damage to or theft of any goods
handed in for dry-cleaning. The same words appear on a big notice board in the shop, which is
clearly visible. When X fetches her jacket, she is dismayed to discover that the drycleaning process
has changed the jacket’s colour. Is she bound by the exemption clause? Discuss briefly. [5]
Answer
With so-called ticket contracts, one of the parties issues a ticket on which certain contractual terms
appear. The question is whether the other party may be held bound to such terms where that party
has not signed the ticket in question. Our courts use a three-legged test:
1. Did the person know there was writing on the ticket?
2. Did he know that the writing referred to terms of the contract?
If both answered in the affirmative, the terms form part of the contract. If either answered negative,
a further question follows:
3. Did the party who issued the ticket take reasonable steps to bring the reference to the terms to
the attention of the other party?
In the present case, X will probably be held bound because of the notice board that also refers to the
contractual terms.

Question
Andy and Craig conclude a contract wherein Andy agrees to paint Craig’s office block by 31 August,
and Craig agrees to pay Andy R10 000 upon completion of the work. When 80% of the work is
completed Andy suddenly falls ill and he is unable to complete the job by 31 August. Craig refuses to
pay Andy any money for his (Andy’s) services rendered, as Craig believes that Andy has breached the
contract by not completing the work. Craig hires another contractor at an amount of R3000 to
complete the job. Craig does not incur any other costs to complete the job, neither does his business
make any losses. Advise Andy as to what amount (if any) he may recover from Craig for the services
that he rendered, and on what basis. Discuss with reference to BK Tooling (Edms) Bpk v Scope
Precision Engineering (Edms) Bpk and other relevant case law. [15]

Answer
This contract is reciprocal in nature. Andy has rendered defective performance and the issue is
whether Craig has to compensate Andy for the work that has already been done.
This question deals with the exceptio non adimpleti contractus. The exceptio is a defence that can be
raised in the case of a reciprocal contract, where the performances due on either side are promised
in exchange for one another. It is a remedy that permits a party to withhold their performance and
ward off a claim for such performance until such time as the other party has either performed or
tendered performance of their obligations.
Where a party who has to perform first has only performed part of its obligations or has rendered
defective performance, that party is in principle not entitled to claim counter-performance until such
time as he has performed in full. In practice, the innocent party often accepts part-performance and
starts using the performance. This sometimes leaves the breaching party in the unfair position that it
may be impractical or impossible to make full performance, but any claim for counter-performance
can be defended by the other party relying on the exceptio.
As a result, the courts have exercised a discretion to relax the principle of reciprocity and order the
party making use of the defective or incomplete performance to pay a reduced amount to the party
in breach.
In BK Tooling, the Appellate Division confirmed this, and held that the courts have an equitable
discretion to award a reduced contract price, depending on the nature of the defect, and the cost of
repair, replacement, or substitute performance. The onus to prove the amount of reduction is on the
party in breach claiming the reduced price. The plaintiff must allege and prove:
• that the other party is using his performance
• the cost of remedying defects
• that it would be equitable to award some remuneration despite breach
• that the circumstances are such that the court should exercise its discretion
Based on the ruling in BK Tooling, Andy is entitled to be compensated by Craig because:
1. Craig is utilising the defective performance
2. It would be equitable as Andy has completed most of the work
3. The counter-performance ought to be reduced by R3000 (the amount it cost to complete the job)
In the circumstances, Andy is entitled to receive R7000 from Craig, which represents the difference
between the contract price and the cost to complete the job.

Question
On 1 June M and Q conclude a contract whereby M undertakes to manufacture and install kitchen
cupboards in Q’s home for R50 000. The parties agree that the price will be paid as soon as the
kitchen cupboards are installed, but they do not determine a date for the completion of the work. M,
however, informs Q during the negotiations that she has some other work to complete and that she
will attend to the kitchen cupboards as soon as possible. Eight months has lapsed since the contract
was concluded and Q has not heard from M. Q runs out of patience and hires W to manufacture and
install the same kitchen cupboards for R60 000.
After W has completed the job, M turns up to do the work. Q claims R10 000 damages from M, but
M institutes a counterclaim for R30 000 from Q for her loss of profit. Who will succeed in this claim?
Discuss.

Answer
This question deals with damages for breach of contract. In order to determine who will succeed in
the claim for damages, we must ascertain which party committed the breach.
A plaintiff who wishes to claim damages for breach of contract must prove the following:
1. A breach of contract has been committed by the defendant
2. The plaintiff has suffered financial or patrimonial loss
3. There is a factual causal link between the breach and the loss
4. As a matter of legal causation, the loss is not too remote a consequence of the breach.
Did M or Q breach the contract?
M could possibly be in breach in the form of mora debitoris. Mora debitoris is the unjustifiable failure
of a debtor to make timeous performance of a positive obligation that is due and enforceable, and
still capable of performance in spite of such failure.
Because performance has become impossible, it is not capable of performance.
Also, no date was stipulated for performance, nor did Q demand performance, so M could neither be
in mora ex re nor mora ex persona respectively.
M has not committed a breach of contract.
Q, by hiring W to manufacture the cupboards has committed two forms of breach: repudiation, and
prevention of performance. A party commits the breach of repudiation when, by words or conduct,
and without lawful excuse, he manifests an unequivocal intention no longer to be bound by the
contract or any obligation forming a part thereof.
Prevention of performance is a breach whereby, after conclusion of the contract, one of the parties,
owing to their fault, causes performance to become impossible.
M will therefore be able to claim damages from Q successfully, because he will be able to prove that
Q committed a breach of contract. The aim of damages is to place the innocent party in their
fulfilment position, that is, the position they would have been in had there been no breach. M’s claim
for loss of profit will probably be successful.

Question
X contracts with Y for the latter (Y) to build and fit a security gate for the entrance of her (X’s) home.
Y builds the gate and fits it with an electric motor, which is activated with a remote control. X is
satisfied with the work and pays Y the contractual amount agreed upon. A week later, the gate gets
stuck while it is halfway open as a result of defective materials used to build the gate. When X
attempts to physically move the gate to close it fully, she suffers such severe damage to her left knee
that she has to have a knee operation. Her medical costs are R20 000. The costs of repairing the gate
amount to R15 000. X wants to claim both medical costs as well as the cost of repairing the gate from
Y. Advise X if she will be successful with her claim. Refer to Shatz Investments (Pty) Ltd v Kalovymas;
Holmdene Brickworks (Pty) Ltd v Roberts Construction Co, and other relevant case law in your
answer. [15]

Answer
This question deals with a claim for damages for breach of contract, and specifically, the element of
legal causation regarding special damages and general damages.
A plaintiff who wishes to claim damages for breach of contract must prove:
1. A breach of contract has been committed by the defendant
2. The plaintiff has suffered financial or patrimonial loss
3. There is a factual causal link between the breach and the loss
4. The loss is not too remote a consequence of the breach (legal causation).
Y has committed a breach of contract in the form of positive malperformance (the defective
materials used to build the gate).
In the law of contract, the approach to remoteness of consequences from breach (legal causation)
has been traditionally based on a distinction between general and special damages. The distinction
between general damages and special damages was stated in Holmdene Brickworks: general
damages are those damages that flow naturally and generally from the kind of breach in question
and which the law presumes the parties contemplated as a probable result of the breach; special
damages, on the other hand, are presumed to be too remote unless exceptional circumstances are
present.
X may claim the cost of repairing the gate as general damages.
If X wants to succeed in the claim for medical costs as special damages, X must prove that:
1. The damages were actually foreseen or reasonably foreseeable at the time of entry into the
contract (the contemplation principle); and
2. The parties can be taken to have agreed that there would be liability for damages arising from
special circumstances (the convention principle).
PVL3702_May_2015

(1) Wh1ch of the following IS NOT a value which informs the law of contract?

1 Ubuntu
2 Fairness
3 Sanctity of contract
4 Legal and commercial certainty
5 The requirements that an offer must be firm

(2) The auction of a Porsche Carrera (a type of sports car) is advertised in the newspaper. The auction
is to take place on 1 May and will be subject to reserve On 1 May X, the
auctioneer, announces before the start of the auction that the auction will be subject to reserve and
that the successful bidder must pay the price cash on delivery Y is the highest bidder. Which
statement IS INCORRECT?

1 X may validly call off the auction any time before the start of the auction on 1 May
2 Y 1s not bound by the cond1t1on that the successful bidder must pay the price cash on delivery
because this condition has not been announced in the advertisement
3 X may not change the conditions of the auction after the first bona fide bid has been made
4 Y may retract his bid before the hammer falls, because Y makes an offer to buy the car by bidding
5 X may refuse to accept Y's offer 1f Y's bid is lower than the reserve price

(3) Which case deals with a notice promising a monetary reward in return for the furnishing of
certain reformation?

1 Carlil v Carbolic Smoke Ball Co [1893] 1 QB 256


2 Crawley v Rex 1909 TS 1105
3 Bloom v American Sw1ss Watch Co 1915 AD 1 00
4 Bird v Sumerville 1961 (3) SA 194 (A)
5 Smeiman v Volkersz 1954 (4) SA 170 (C)

(4) In B1rd v Sumervllle 1961 (3) SA 194 (A) the court found that

1 an offer must be accepted by the person to whom it was addressed


2 an acceptance must be a reaction to the offer
3 an acceptance must comply with any formalities set by law or by the offeror
4 an acceptance must be unconditional and unequivocal
5 a person cannot accept an offer of which he is not aware

(5) X makes a written offer to Y to purchase Y's house X hands Y the written offer on
12 May, but Y only reads 1t the next day (13 May). Y drafts an acceptance on 14 May and posts his
acceptance on 15 May to X. X receives the acceptance on 17 May and reads it on 18 May. When was
the contract concluded?
1 13 May
2 14 May
3 15 May
4 17 May
5 18 May

(6) S sold a second-hand tractor to P for R60 000. The maximum price for the tractor under the
relevant price control regulations was R40 000. The price control regulation prohibited the
conclusion of contracts of sale at a price in excess of the controlled price and provided that such
contracts were void P was aware of the price control regulation, but S was not S delivers the tractor,
but P refuses to pay the purchase price Wh1ch statement IS CORRECT?

1 P has to pays R40 000, because P would otherwise unjustifiably be enriched at the expense of S
2 S may cancel the contract and claim back the tractor from P
3 S may claim delictual damages from P, because P has refused pay the purchase price
4 S may claim back the tractor from P w1th an unjustified enrichment action
5 S may not claim the tractor back from P, because them pan delicto rule prohibits S from doing so

(7) S informs P in good faith that there are 1 000 fruit trees on his (S's) farm, as a result of which P
buys the farm from S for R8 000 000. It is later found that there are only 800 fruit trees on the farm.
This is a case of

1 an error in negotio
2 an Innocent misrepresentation
3 a dictum et promissum
4 option 2 and 3
5 opt1on 1, 2, and 3

(8) S negligently informs P that there are 1 000 fruit trees on his (S's) farm, as a result of which P buys
the farm for R8 000 000 P pays the R8 000 000 and the farm is registered m her (P's) name It is later
found that there are only 800 fruit trees on the farm P would not have bought the farm had she
known the truth about the number of fruit trees, but decides nevertheless to uphold the contract.
The value of the missing 200 fruit trees is R500 000. The market value of the farm is R7 800 000. The
farm would have been worth R8 400 000 if it had 1 000 fruit trees on it. What amount will P be able
to claim from S based on negligent misrepresentation? Do not apply the Consumer Protection Act to
this quest1on

1 R0
2 R200 000
3 R400 000
4 R500 000
5 R600 000

(9) P's claim against S in question (8) is based on


1 delict
2 breach of contract
3 unjustified enrichment
4 monetary compensation
5 all the above

(10) Assume the same facts as in question (8), but P would have paid only R7 900 000 for the farm
had she known the truth about the actual number of fruit trees on the farm P upholds the contract.
What amount will P be able to claim from S based on negligent misrepresentation? Do not apply the
Consumer Protection Act to this question.

1 R100 000
2 R200 000
3 R400 000
4 R500 000
5 R600 000

( 11) Assume the same facts as in question (8) S guarantees that there are 1 000 fruit trees on the
farm. Once fruit trees are full-grown they cannot successfully be transplanted. What amount will P
be able to claim from S based on S's breach of this guarantee? Do not apply the Consumer Protection
Act to this question.

1 R0
2 R200 000
3 R400 000
4 R500 000
5 R600 000

(12) Which theory must be applied to determine when and where a contract arose in the case of a
contract concluded by post?

1 The declaration theory


2 The rel1ance theory
3 The expedition theory
4 The reception theory
5 The information theory

(13) Which statement regarding cancellation for breach of contract is INCORRECT?

1 Where a party has a right to cancel a contract, he is never obliged to do so


2 A party may cancel a contract where the breach of contract on the part of the other party is
material
3 A party may cancel a contract where provision has been made in the contract for a right to resile in
the circumstances that exist
4 Cancellation can be claimed with the actio redhibitoria
5 Cancellation of a contract is available to a party only in exceptional circumstances

(14) Which one of the following contractual terms is certain?

1 X undertakes to pay a substantial amount of the purchase price of R600 000 for a house each
month until the price is pa1d off
2 Y will pay X a reasonable fee for the installation of Y's stove
3 X has an option to buy Y's farm for a price to be determined by agreement between X and Y X and Y
undertake to negotiate the price in good faith when X exercises the option
4 X, the seller of a car to Y, will determine the purchase price of his car
5 X has an option to buy Y's farm for a price to be determined by agreement between X and Y when X
exercises the option

(15) What IS the general effect of supervening impossiblity of performance?

1 The contract is rendered Voidable at the instance of the creditor


2 The contract is rendered voidable at the Instance of the debtor
3 The creditor is guilty of contractual breach
4 The debtor is guilty of contractual breach
5 Termination of the contractual obligation

(16) Which one of the following statements regarding tacit terms IS INCORRECT?

1 The courts often employ the officious bystander test in determining whether a contract contains a
tacit term
2 A tacit term Will only be read into a contract if it does not conflict any of the unambiguous express
terms of the contract
3 A tacit term will only be read into a contract if it is necessary in a business sense to give efficacy to
the contract
4 A tacit term must be capable of a reasonably clear and reasonably exact formulation
5 Trade usage is a fertile field for the importation of tacit terms

(17) Which one of the following statements regarding naturalia is INCORRECT?

1 The parties may generally exclude naturalia by express agreement


2 Naturalia are terms the law attaches to every contract of a particular class of contracts
3 Naturalia are terms used by the law in order to identify a contract as belonging to a particular
class of contracts
4 Naturalia often serve to protect one of the parties from a hazard typically associated with that type
of contract
5 Naturalia are ex lege terms

(18) lncidentalia are


1 All the terms of a contract apart from the naturalia and essentialia
2 Terms that identify a contract as belonging to a particular class of contracts
3 Terms automatically imposed by law on the contracting parties unless contracting parties expressly
exclude them
4 All the terms of a contract apart from the naturalia
5 Material terms and conditions of a contract

(19) S undertakes to transfer (del1ver) h1s car to P for R40 000 on 1 July. They agree that the car has
to be delivered top on 2 July, but they fail to agree on a date on which P has to pay S R40 000 S
guarantees that the car is a 2005 model. Which statement regarding this contract is INCORRECT?

1 The obligations to deliver the car and to pay R40 000 characterises this contract as a contract of
sale
2 The obligation to deliver the car and to pay R40 000 are the essentialia m this contract
3 Despite the parties' failure to agree on a date for the payment of R40 000, P must pay S when S
delivers the car on 2 July
4 The obligation to deliver the car on 2 July 1s one of the incidentalia in this contract
5 This contract has no naturalia

(20) X and Y conclude a contract on 1 June m terms of which X undertakes to deliver 10 dozen eggs
on every Monday to Y, a shopkeeper, for a year X falls to perform. This is a case of

1 repudiation
2 prevention of performance
3 mora creditoris
4 mora ex re
5 mora ex persona

(21) X and Y conclude a contract on 1 June m terms of which X undertakes to deliver 10 dozen eggs
on every Monday to Y, a shopkeeper, for a year X delivers the eggs as he has agreed to, but after a
month X Informs Y that he is not going to deliver the eggs anymore. This is a case of

1 repudiation
2 prevention of performance
3 mora creditoris
4 mora ex re
5 mora ex persona

(22) X undertakes to do the catering at Y's wedding on 5 June for R20 000 X falls to even pitch up at
the reception, because he takes his girlfriend on a picnic on 5 June. What type of breach of contract
does X commit?

1 Repudiation
2 Prevention of performance
3 Mora creditoris
4 Mora debitoris
5 None of the above options

(23) P buys S's painting for R50 000 on 1 June and pays S R50 000 immediately. The parties agree S
will deliver the painting to P at her (P's) house on 5 June S attempts to deliver the painting to P on 5
June, but P refuses to accept del1very as she is busy entertaining her guests. What type of breach of
contract does P commit?
1 Repudiation
2 Prevention of performance
3 Mora creditoris
4 Mora debitoris
5 None of the above options

(24) Cession is the transfer of

1 real rights
2 duties
3 both personal rights and duties
4 immaterial rights
5 personal rights that arise out of obligations

(25) X rents a house from Y for R8 000 per month X buys the house from Y and the property IS
transferred into X's name X stops paying Y the rent. This IS a case of

1 Merger
2 Release
3 Novation
4 Set-off
5 Compromise

QUESTION 1 (10)
Discuss the option contract

Answer
1.1 Description and requirements
Contracting parties may enter into an agreement in terms of which the offeror undertakes not to
revoke his offer. In such a case it is said that one party grants the other an option. In essence, in the
case of an option we are dealing with an offer which forms the subject matter of an agreement not
to revoke a substantive offer. The parties agree that the offeror will not revoke the offer, either
expressly or by implication, for example, by offering the same thing to a third party. It therefore
amounts to the person's granting the option undertaking to do nothing to prevent the coming into
existence, through acceptance of the offer by the grantee, of a contract capable of being performed
(substantive contract). The option contract must however comply with the requirements set for
contracts in general.
1.2 Definition
We could therefore define an option as an offer (substantive offer) reinforced by an agreement
(option contract) in terms of which the offeror (grantor) undertakes as against the offeree (grantee)
to keep open his offer (usually for a specific period) to the offeree, or, to put it differently, in terms of
which the offeree acquires the power to consider and accept the offer (usually within a specified
time period). Options may be granted in regard to a variety of contracts. It may be an option to buy,
to hire, to do work and so on.

Where A offers B an option, for instance to buy his farm, we are actually dealing with two offers
which can grow into two contracts on acceptance of the various offers, that is:

(1) an offer by A that B can purchase his farm for R300 000, subject to the terms expressed in
the offer

(2) an offer by A that the abovementioned offer will remain open to B for a certain period. A
may stipulate a remuneration for this right, for example R2 000.

If B now accepts the second offer, an option contract is created forthwith, on the one hand entitling
A to the R2 000 and on the other hand binding him to keep the offer to sell, open to B for the
stipulated period.

1.3 Consequences of an option


If B accepts the first offer (offer (1) above) by exercising the option, a contract of sale is created
immediately. Nothing more is required of the grantor for the formation of the contract of sale. Thus
it was decided in Scott v Artus 1964 (3) SA 384 (E) that a pactum de retrovendendo, that is an
agreement whereby the seller can repurchase the merx (thing sold) within a certain period, also
involves an option, since "... it embodies the two essential characteristics of what is commonly called
an option to purchase --- an offer to sell the property back to the plaintiff, coupled with a binding
agreement to keep that offer open for a stipulated period of time. A contract of resale comes into
being only if the plaintiff accepts the offer before it lapses."

De Wet and Van Wyk (33--34) state that an offer is made irrevocable by agreement (that is by the
conclusion of an option contract). This means that an attempted revocation of the substantive offer
would have no legal effect. Van der Merwe et al (3.3.2) state the following in this regard:
"An offer can be made irrevocable only by means of an option contract. [A] unilateral declaration by
an offeror that the offer is irrevocable is generally regarded as ineffective in our law."

Our courts also regard an offer supported by a pactum de contrahendo as irrevocable (see Thompson
v Van der Vyver 1954 (2) SA 192 (C). There is, however, the viewpoint that an option does not render
the substantive offer irrevocable but that revocation (or denial) of the substantive offer will
constitute breach of the option contract which should give rise to the normal remedies available in
the case of breach of contract. As it is possible, however, to commit breach of contract, the fact that
the offeror will be committing breach of contract should he unilaterally revoke his substantive offer,
should not mean that he cannot revoke it, and once he has revoked it there will no longer be an offer
for the offeree to accept (cf Lotz 1988 THRHR 237). In general, however, the opinion of writers and
the courts is that an option renders the substantive offer irrevocable.

Nevertheless, it should also be mentioned that in University of the North v Franks [2002] 8 BLLR 701
(LAC) at 720: “Where an offer is (either expressly or tacitly) stated to be irrevocable for a given period
and communicated to the offeree it becomes irrevocable upon receipt unless the offeree rejects the
irrevocability. To require a mental acceptance would be meaningless in practice as that cannot be
evidenced. Such requirement would merely pander to theory. To require notification of acceptance
of the irrevocability would set a standard which in normal business practice will not be followed and
will be regarded as rather foolish.”
Whether such an approach will become accepted law, is uncertain at this stage. In Oos-Vrystaat Kaap
Bedryf Bpk v Van Aswegen 2005 (4) SA 417 (O) the concept of an irrevocable offer was rejected, but
without reference to the Franks case.

1.4 Remedies for breach of an option


Breach of an option contract and the consequences thereof are governed by the general principles of
the law of contract. An attempted revocation of the sustantive offer does not prevent the exercise of
the option and the option holder may enforce the contract specifically by means of an interdict
against the grantor of the option. The option holder may also claim damages, if suffered, to place
him in the position that he would have been in if the option had been excercised (see *Study Unit
regarding damages for breach of contract and you may also consult Figure 2.3 in paragraph 2.5.1.8 of
the prescribed texbook).

1.5 Juristic nature of an option


In Hersch v Nel 1948 (3) SA 686 (A), Davis JA stated:
"An option has been analysed into an offer to sell together with an agreement to keep that offer
open for a certain time but perhaps a better way is to look at it simply as an agreement between the
giver and holder of the option by which the giver has bound himself to sell a certain thing to the
holder at a certain price if the holder shall require him to do so within the time fixed by the option;
by this agreement the giver grants and the holder acquires a right to buy."
To regard an option, however, as a "right to buy" with a corresponding "duty to sell" is to create the
impression that the person granting the option is contractually obliged to make the true offer to sell
only at the stage when the holder of the option wants to exercise the option, and that is not the
case. Moreover, the term "right to buy" is open to criticism from a legally scientific point of view, as is
the description "the grantee acquires the right to accept the offer to sell" (Brandt v Spies 1960 (4) SA
14 (E)). The grantee has only a power to accept the offer and not a subjective personal right to
acceptance.
The option contract does give the grantee a personal right, but it is not so much a "right to buy" as a
right to claim that the offer be kept intact as against him, the grantee, with a corresponding duty on
the grantor to keep the offer open to the grantee. The object of the right, the performance, is the
maintenance of the option as against the grantee for the duration of the option. The content of the
right is the power of the grantee to claim that the grantor shall do nothing to prevent the coming
into existence, through acceptance of the offer, of a contract capable of being performed. It is this
personal right which, as it were, ensconces or entrenches the offer. The offer is the basis of the right
and, as such, offer and claim share each other's fate as regards cession, termination and so on. The
option as such is simply an election; an election to accept or not to accept the entrenched offer.
The offer to sell and the offer not to revoke the offer to sell will normally occur simultaneously, but
there is no reason why this should necessarily be the case. It may happen, for example, that the offer
to sell is made first and that the option contract is concluded later. However, confusion often arises
where the transactions are combined, and where this happens, the word "option" serves only as a
name for this complex of juristic acts: an offer ensconced or backed by an agreement from which at
least one right arises, that is the right to maintenance of the offer (for a discussion of the nature of
an option see Venter v Birchholtz 1972 (1) 276 (A)).

1.6 Termination of options

1.6.1 Passage of time


Where the option contract fixes a time limit for the acceptance of the substantive offer, the option
lapses if it is not exercised within the prescribed period. What the position is where no such time
limit is fixed, has not yet been decided. It is submitted that it must then be accepted that the parties
intended the substantive offer to be kept open for a reasonable time.

1.6.2 Death of the grantor or grantee


In general, it is accepted that an option is actively and passively transmissible upon the death of the
offeror or offeree. This means that, in principle, the option does not automatically terminate upon
the death of either the grantor or the holder of the option. Unless the parties intended otherwise,
the obligation to keep the option open does not lapse with the death of the grantor (Beyers v Beyers
1933 CPD 31; Major's Estate v de Jager 1944 TPD 96). Whether the death of the grantee terminates
the option depends on whether the option is transferable. If it is not, it lapses; if it is, it devolves on
the grantee's heirs. On the question whether an option is transferable, see below regarding the
cession of an option.

1.6.3 Refusal
By his decision not to exercise the option, communicated to the grantor, the grantee waives his right.
The option therefore lapses on rejection.

1.6.4 Lapse of the right


The option will also lose its efficacy or terminate in any manner in which claims are normally
terminated.

1.7 Formalities concerning options


As we have already seen, two contracts come into play in the case of an option: the option contract
and the substantive contract created when the option is exercised. Both must comply with the usual
requirements for the formation of contracts in general. Where, in terms of a particular statute, the
substantive agreement must comply with certain formalities, the question arises whether the option
contract must likewise comply with the formalities in question.

In particular, the question arises whether an option to alienate land must be in writing and signed.
Section 2(1) of The Alienation of Land Act 68 of 1981 provides the following:
"No alienation of land after the commencement of this section shall, subject to the provisions of
section 28, be of any force or effect unless it is contained in a deed of alienation signed by the parties
thereto or by their agents acting on their written authority."

In Brandt v Spies (above) the defendant orally granted an option to the plaintiff to purchase his farm.
Disregarding the option, he sold the farm to a third party. Plaintiff, who had exercised the option in
writing, then claimed damages for breach of contract, but an exception to his claim was upheld. The
court stated as follows:

"If the offer is not in writing there is nothing which the offeree can accept so as to create a vinculum
juris between himself and the offeror. An undertaking to keep open an offer which is incapable of
forming the basis of a valid contract can itself confer no right upon the grantee --- for in law there is
nothing to keep open."
This judgment, it is submitted, is correct in the case where the offer to sell and the offer to keep the
first offer open are made simultaneously and orally. As regards the contract of sale, both offer and
acceptance must be in writing in accordance with the abovementioned legislation, and the offer has
been made orally in such a case. However, what of the case where the offer to sell is made in writing
on one occasion and is later followed by an oral offer to keep the offer to sell open? The Act provides
that no alienation of land will be valid unless it is reduced to writing. The option contract which
comes into question with the acceptance of the oral offer is not an alienation of land and
consequently one can conclude that it need not be in writing. What must be in writing is the offer to
sell.
In Hirschowitz v Moolman 1985 (3) SA 729 (A) the court, however, held that as a general rule pacta
de contrahendo must conform to any formalities prescribed for the substantive contract, including
option contracts relating to the purchase of land. This case dealt with a right of pre-emption and
therefore the statement of the court must be regarded as obiter as far as options are concerned. As
stated above, there seems to be no compelling reason why options relating to such contracts must
comply with section 2(1) of the Act in question.
Since the exercise of the option is nothing but an acceptance of the substantive offer to sell, it is
obvious that it must also be in writing before a contract of sale, in writing, is created. Subject to the
usual exceptions, this acceptance is binding only when it has been communicated to the offeror
(grantor of the option). The communication of acceptance need not, however, be in writing (Hersch v
Nel above).

1.8 Cession of options


As a rule claims may be freely ceded. The claim arising from the option contract in this case,
however, is so closely related to the substantive offer that the question whether it may be ceded
must be rendered dependent on the intention of the grantor: if it was his intention that the option
will be open to the grantee alone, then the option cannot be ceded; if it is immaterial to him who
exercises the option, the grantee or a third person, it may in fact be ceded. The intention must be
established in the usual manner (in this regard see Study Unit 3), and an important consideration will
naturally be whether the identity of the person who will be obliged to render the counter-
performance if the option is exercised is of any importance to the grantor. Thus the following was
alleged in Hersch v Nel (above):
"... an option to obtain a loan would obviously stand on an entirely different footing from an option
to buy for cash. And for the same reason an option to buy on credit also stands on a different footing
--- I come to the conclusion that where there is nothing in the option to show the intentions of the
party, an option to purchase for cash is ordinarily capable of being ceded."

(See further Dettman v Goldfain 1975 (3) SA 385 (A).)


Normally the cession of an option need not be in writing. Section 1 of Act 68 of 1957 provided,
however, that a cession in respect of land or any interest in land had to be in writing and signed. An
option in regard to land, being an offer to which a claim is attached, surely represents an interest in
land and it could therefore be argued that such option could only be ceded in writing. The words "or
cession" appearing in section 1 of Act 68 of 1957 have, however, been omitted from section 1(1) of
Act 71 of 1969 and from section 2(1) of Act 68 of 1981; consequently it is clear now that the cession
of an option in respect of land need not be in writing.
Thus far options granted by the prospective seller have been discussed. The prospective purchaser
acquires a personal right to have the offer kept open. The decision whether a contract of sale will be
created or not rests with the purchaser.
An option may, of course, also be granted by the prospective purchaser. This is the converse of the
above. Here it is the prospective purchaser who makes the offer to purchase and supports it with an
undertaking to keep it open to the prospective seller. The decision whether a contract of sale will be
created rests exclusively with the seller. He is entirely free to disregard the option and sell the thing
to a third party; that is the prospective seller acquires a right (that is that the person granting the
option may not revoke his offer, for example) but the prospective purchaser does not also acquire a
right as a matter of course. The prospective purchaser acquires a right only if he specifically
stipulates such a right, for example, a right of preference of some kind or other, as discussed below.

QUESTION 2
Discuss damages as a remedy for breach of contract (10)

Answer
Answer
Hutchison & Pretorius (eds) The Law of Contract in South Africa 2nd ed (2012) par 13.5:
13.5 Damages
Whether the innocent party elects to cancel or to claim specific performance of the contract, he or
she is in addition entitled to claim damages as compensation for any financial loss that might have
been suffered as a result of the breach, subject to various limitations that will be discussed below.
The claim for damages is probably the single most important remedy for breach of contract. The law
in regard to damages is complicated, however, and the innocent party will often experience difficulty
in quantifying the damage that he or she has suffered in consequence of the breach. Largely for this
reason, the parties to a contract will often include a penalty clause in their agreement, making
provision for the payment of a fixed sum of money in the event of breach. The penalty will then be
payable in lieu of the damages, and not in addition to them. If the penalty is disproportionate to the
prejudice actually suffered as a result of the breach, the court may reduce the amount of the penalty,
in terms of the governing legislation.
Figure 13.3 Overview of damages
13.5.1 The nature and purpose of contractual damages
The fundamental rule in regard to the award of damages for breach of contract, it has often been
held, is that the innocent party should be placed in the position he or she would have occupied had
the contract been properly performed, so far as this can be done by the payment of money and
without undue hardship to the defaulting party. The aim, in other words, is to place the innocent
party in his or her fulfilment position – that is, the position he or she would have occupied had there
been no breach.
The application of the fundamental rule to a given set of facts entails a comparison between two
financial positions of the plaintiff: the actual position in which the plaintiff now finds him or herself
subsequent to the breach, and the hypothetical position he or she would have occupied had there
been no breach. This method of quantifying the damage caused by a wrong is known as the
difference theory, and applies equally in the law of delict, where one asks: how does the plaintiff’s
present financial position compare with the hypothetical position he or she would have occupied had
the delict not been committed?
How then do contractual damages differ from delictual damages, if at all? In Trotman v Edwick, Van
den Heever JA stated:
A litigant who sues on contract sues to have his bargain or its equivalent in money or in money and
kind. The litigant who sues on delict sues to recover the loss which he has sustained because of
wrongful conduct of another, in other words that the amount by which his patrimony has been
diminished by such conduct should be restored to him.
In line with this dictum, it is often said that contractual damages are measured according to the
plaintiff’s positive or expectation interest, and include loss of profit, whereas delictual damages are
measured by the plaintiff’s negative or reliance interest, and are restricted to reliance or out-of-
pocket losses (wasted expenditure and the like). This statement requires some explanation as the
analysis is not quite so simple – as is explained below.
A person who has concluded a contract usually has an expectation that he or she will gain or profit
from the transaction. In order to realise that profit, however, he or she must perform his or her side
of the bargain, and in so performing or preparing to perform the party will often incur expenditure,
relying on the fact that he or she has a binding contract and that in due course this expenditure will
be recouped from the gross profits earned from the transaction. A building contractor, for example,
may hire staff and buy materials in order to fulfil his or her obligations under the building contract. If
the other party then unlawfully repudiates the contract, this expenditure will be wasted and the
contractor will suffer both a reliance loss (the wasted expenditure) and an expectation loss (the net
profit lost on the contract).
Positive interest damages are forward looking, in that they aim to place the plaintiff in the position
he or she would have occupied had the contract been properly and timeously fulfilled; negative
interest damages are backward looking in that they aim at placing the plaintiff in the position he or
she would have occupied had he or she not entered into the contract at all.
Applying the difference theory to a claim for damages for breach of contract will, as a simple matter
of causation, produce a forward-looking result: but for the breach, the plaintiff would have been in
the fulfilment position; and if he or she would have made a profit on the transaction, his or her
damages will include this expectation loss, in addition to any reliance losses he or she might have
suffered as a result of the breach.
Applying the difference theory to a claim for delictual damages on the grounds of a pre-contractual
wrong such as fraud that induced the contract (dolus dans) will, again as a simple matter of
causation, produce a backward-looking result: but for the fraud, there would have been no contract
at all; had the plaintiff not entered into the contract, he or she would not have incurred the
expenditure that is now wasted, but by the same token he or she could not have made any profit on
the transaction. The plaintiff’s damages are thus restricted to his or her reliance losses.

Position of the plaintiff had


contract not been concluded Position of the plaintiff had the
contract been properly performed
Pre-contractual wrong
(mostly out-of-pocket losses) Breach of contract
(includes compensation for loss of profit)
Positive and negative interest

This stark contrast in outcomes obtained by the application of the difference theory to a post-
contractual breach, on the one hand, and to a pre-contractual wrong like fraud, on the other, arises
not from some fundamental difference between contractual and delictual damages, but simply from
the fact that the causal effect of the two wrongs is so different in the particular context of a fraud or
other wrong that brings a contract into being on the one hand, and conduct that breaches
contractual obligations on the other. In other contexts, the commission of a delict may result in a
loss of profit, in which case that expectation loss will be recoverable in a delictual action for
damages.
Unfortunately, there has been a tendency at times to associate particular types of loss with particular
types of claim: loss of profit with a contractual claim and out-of-pocket losses with a delictual claim.
This has led to the misconception, not only that loss of profit is never recoverable in delict, but also
that a contractual claim for positive interest damages compensates only for loss of profit. This is
demonstrably false; properly applied positive interest should embrace reliance losses as well as
expectation losses. If the building contractor in the previous example can prove that his or her
reliance losses are say R30 000 and that, but for the breach, he or she would have made a net profit
of R10 000 on the contract, the amount of money that must be awarded to move from his or her
present actual position (-R30 000) to the hypothetical position had there been no breach (+R10 000)
is not R10 000 but rather R40 000.
Figure 13.5 Example: Contractual damages may include both expectation and reliance losses

Clearly, if the victim of a breach is to be properly compensated for the wrong committed against him
or her, he or she must be able to recover reliance losses. However, the misconception that positive
interest is concerned only with loss of profit has led some judges to hold that the victim of a breach
may elect to have damages measured according to his or her negative rather than his or her positive
interest, to enable the recovery of reliance losses. One case adds the proviso that the contract is
cancelled, another that these reliance damages can never exceed those recoverable under the
positive measure. The whole matter is controversial and awaits clarification by the Supreme Court of
Appeal.

Not all contracts are profitable.


In principle, the proposition that the plaintiff can demand to be placed financially in the position he
or she would have occupied had the contract never been concluded is open to the objection that it
would compensate the plaintiff for losses caused not by the breach, but also for losses that could
have ensued as a result of entering into the contract. In the case of a losing contract, where, but for
the breach, the plaintiff would in any event have made a net loss on the contract, an award of
damages that restores the plaintiff to his or her pre-contractual position would over-compensate him
or her and would permit him or her to escape the financial consequences of bad bargaining: hence
the qualification that the amount awarded under the negative measure can never exceed the
amount under the positive measure. However, this is a clear indication that the real aim is merely to
compensate the victim of a breach for reliance losses, not to place him or her in the position that
would have been occupied had the contract not been concluded.
13.5.2 Requirements for a damages claim
A plaintiff who wishes to claim damages for breach of contract must prove the following:
a breach of contract has been committed by the defendant;
the plaintiff has suffered financial or patrimonial loss;
there is a factual causal link between the breach and the loss; and
as a matter of legal causation, the loss is not too remote a consequence of the breach.
The latter three requirements are discussed in more detail below.
13.5.2.1 Financial loss
The nature of the loss on which a contractual claim for damages may be based is financial; not some
other measure of loss.
Actual loss; no nominal damages. The mere fact that the defendant has breached the contract does
not entitle the plaintiff to claim damages; the plaintiff must prove that the breach has caused him
actual loss or damage. South African law does not permit an award of nominal damages for breach
of contract.
In the nature of things, great difficulty will sometimes be experienced in quantifying the loss.
Provided it is clear that some loss has occurred, the fact that the extent of the loss cannot be
precisely calculated or even reliably estimated will not prevent the court from making an award. The
plaintiff need not prove his or her loss with mathematical precision, but must produce the best
evidence that is reasonably available so that the court is not faced with an exercise in pure
guesswork. Failure to do so when the evidence is available may result in an order of absolution from
the instance.
Financial or patrimonial loss. The loss must be financial or patrimonial loss. Refusing to follow the
lead taken by courts in other jurisdictions, the Supreme Court of Appeal has made it clear that in our
law, contractual damages may not be awarded for such things as physical inconvenience, discomfort,
mental stress, injured feelings or disappointment caused by a breach of contract. Such non-
patrimonial or sentimental losses are only recoverable, if at all, in a delictual claim for damages.
Future loss: the once-and-for-all-rule. In terms of the once-and-for-all-rule, the plaintiff must claim all
of his or her damages in one action. The plaintiff cannot claim damages in a piecemeal fashion, as
they occur. If not all of the loss has been suffered at the time that the action is lodged, the plaintiff
must include a claim for prospective losses in that action. If the plaintiff fails to do so, he or she will
not be allowed to claim such losses in future. This rule is aimed at preventing a multiplicity of
actions. It is accepted that prospective damages cannot be calculated with mathematical precision.
Nevertheless, in line with what was stated above, they must be proved with as much accuracy as is
possible in the circumstances.
Application of the difference rule: a more concrete approach. In terms of the difference rule, as we
have seen, the extent of the plaintiff’s patrimonial or financial loss is determined by comparing the
patrimonial position he or she presently occupies after the breach with the hypothetical patrimonial
position he or she would have occupied had the contract been properly performed.
A person’s patrimony or estate comprises all of his or her assets and liabilities. It includes
incorporeals such as personal rights and obligations as well as corporeal property, but excludes
personality rights. Thus, when a person enters into a contract, his or her personal right to a
performance under that contract forms part of his or her patrimony, as does his or her duty to make
counter-performance. The value of these rights and obligations must be taken into account in
determining the value of the estate.
Although the difference rule provides the fundamental and theoretical basis for the calculation of
damages, it is often difficult and cumbersome to apply in practice. Faced with the facts of a particular
case, the courts often use a more concrete approach in determining the quantum by focusing on the
actual element of the patrimony that is affected. According to this approach, damage is calculated
by comparing the value that the asset or obligation would have had if the contract had been properly
performed, with its actual value after the breach. Thus, the approach focuses on the specific asset or
right and not on the patrimony as a whole.
In practice, the quantification will depend on the nature of the loss that has occurred. For example,
the amount of damages may be determined with reference to:
the market value of the defective goods received as compared with the market value of the goods
without the defect;
the cost of repairing a defective performance or object to make it conform with the requirements of
the contract;
the amount of profit that a retailer could have made, but lost due to the breach;
the cost of completing an incomplete performance;
the additional price that must be paid for replacement goods, where defective goods are rejected or
where the contract is cancelled; and
interest paid on an overdraft by the innocent party, where late payment is made.
Whether it is more appropriate to use the difference method or the concrete method to calculate
damages will depend on the type of breach and the nature of the loss suffered. In the case of pure
economic loss, for instance, the application of the difference rule may be more appropriate. In Swart
v Van Der Vyver, the court stated that the calculation of damages is a practical matter and that even
though certain practical rules may have evolved in this regard, these rules are merely guidelines and
not a hard-and-fast formula.
Market value approach. In cases where the performance consists of marketable goods, the quantum
of damages is often calculated with reference to the market value of the goods. In terms of the
market value approach, the amount of damages is determined by the difference in the market value
of the goods as received and the market value they would have had if the goods had conformed with
the requirements of the contract.
The term ‘market’ is used loosely in this context to mean any place where a prospective party may go
to obtain the goods or services. The market value must be established with reference to similar
goods or services available at the time and place that the performance should have been rendered
by the party in breach. The market value approach can be used in regard to any type of performance
that has an objective market value and is not restricted to contracts of sale.
The market value approach is used as a point of departure by the courts in determining the amount
of damages, but the innocent party can persuade the court that a more appropriate alternative
method should be used under the specific circumstances. Where there is no appropriate market
value for the goods or services, alternative methods must be used.
Financial benefits flowing from the breach (lucra cum damno). It sometimes happens that a breach
of contract has beneficial financial consequences for the plaintiff, quite apart from any losses that it
might cause (for example, when the seller of a house fails to evict a tenant who subsequently pays
rent to the purchaser). In principle, when determining the overall financial position of the plaintiff
after the breach, account should be taken not only of the losses arising from the breach, but also of
these incidental gains, provided that they too are caused by the breach. In practice, the courts are
sometimes reluctant to do so, on grounds of public policy or fairness to the plaintiff.
13.5.2.2 Causation
The financial loss or damage suffered by the plaintiff must have been caused by the breach. As in
other fields of law, the question of causation can be a complicated one, since any event (including
the suffering of financial loss) will be preceded by a long chain of other events, all of which are
logically linked to the event, and which can to a greater or lesser extent be considered as causes of
that event.
The enquiry into causation in the law of contract is essentially the same as that in the law of delict.
The relevant principles were authoritatively set out by the Appellate Division in International
Shipping Co (Pty) Ltd v Bentley. The enquiry entails a two-stage process:
1. firstly, one asks whether as a matter of fact the breach causally contributed to the loss in
question; and
2. secondly, if the first question yields a positive answer, one asks whether, as a matter of law,
the causal connection between the breach and the loss is sufficiently close that the party in breach
should be held responsible for it.
The reason for the distinction between factual and legal causation is that a breach of contract, like a
delict, can give rise to a multitude of consequences, some of which might be quite unexpected. As a
matter of fairness, the defendant cannot be held liable for all these consequences, no matter how
remote they are from the breach. The requirement of legal causation serves the purpose of cutting
the causal chain at an appropriate point, and thereby limiting the liability of the defendant.
Factual causation. This is established by means of the ‘but-for’ test (or conditio sine qua non test).
The test enquires whether the loss in question would have been suffered had the breach of contract
not been committed. If, but for the breach, the loss would not have been suffered, the breach is a
factual cause of the loss; conversely, if the loss would in any event have been suffered, the breach is
not a factual cause of the loss. The innocent party needs to prove, on a balance of probabilities, that
the loss would not have been suffered, but for the breach. If it fails to establish this causal link, that is
the end of the enquiry and the damages claim must fail.
Legal causation. Once factual causation has been established, the focus of the inquiry shifts to legal
causation, where the aim is to protect the party in breach from liability that is too wide ranging and
unreasonable. The test for legal causation turns on the issue of remoteness – that is, whether the
causal connection that exists between the breach and the loss is sufficiently close to justify the
imposition of liability.
In the law of delict, as well as in a number of other fields of law, the approach of the courts to legal
causation has undergone considerable evolution in recent years. This has resulted in the
development of what has been termed a ‘flexible or supple’ test for determining the issue of
remoteness. This flexible test takes into account a variety of factors, including among others: the
foreseeability of the harm; whether the harm was a direct or indirect consequence of the wrong in
question; whether an intervening act (novus actus interveniens) broke the chain of causation; and
considerations of legal and public policy.
In the law of contract, on the other hand, the approach to remoteness has traditionally been based
on a distinction between general and special damages. In Thoroughbred Breeders’ Association v
Price Waterhouse, Nienaber JA expressed the view that the flexible test developed in other fields
might one day profitably be employed in the law of contract also. Until such time as the courts effect
such a change, however, remoteness in contract remains governed by the traditional ‘dichotomous
orthodox approach’.
In terms of the traditional approach, general damages flowing from the breach of contract are not
too remote and hence are recoverable as a matter of course; special damages, on the other hand,
are presumed to be too remote unless exceptional circumstances are present. The classic statement
of the law in this regard is that of Corbett CJ in Holmdene Brickworks (Pty) Ltd v Roberts Construction
Co Ltd:
To ensure that undue hardship is not imposed on the defaulting party … the defaulting party’s
liability is limited in terms of broad principles of causation and remoteness, to (a) those damages
that flow naturally and generally from the kind of breach in question and which the law presumes
the parties contemplated as a probable result of the breach, and (b) those damages that, although
caused by the breach of contract, are ordinarily regarded by the law as being too remote to be
recoverable unless, in the special circumstances attending the conclusion of the contract, the parties
actually or presumptively contemplated that they would probably result from the breach.
General damages, then, are those that flow naturally and generally from the kind of breach in
question. They are the sort of damages that might be expected in the ordinary course of things to
result from the breach. As such, they would have been foreseeable to a reasonable person entering
into the contract as a probable consequence (in the sense of ‘a realistic possibility’, something ‘not
unlikely to occur’) of the breach in question. The following examples serve to illustrate the
distinction between general and special damages:
The loss of the value of an architect's time and effort in preparing plans flows naturally from a failure
to transport those plans safely, but not the loss of the chance of winning a prize by entering the plans
in a competition.
The cost of demolishing and reconstructing a building flows naturally from a failure to deliver bricks
and cement of the required quality, but not the loss of profit resulting from a delay in completing the
building.
The cost of repairing or replacing defective goods flows naturally from a breach of a contract for the
delivery of new goods, but not a loss of production caused by the defect where it was not apparent
that the goods would be used for manufacturing purposes.
Loss of interest flows naturally from the failure to pay a money debt timeously.
Whether loss of profit constitutes general or special damages will depend on the particular
circumstances of each case. Where a wholesaler sells goods to a retailer, loss of profit from the
delivery of defective goods or non-delivery of the goods should be regarded as general damages.
However, where the lease of a residential property is breached by the lessor, loss of profit from a
business conducted from those premises will not be regarded as general damages.
All damages that cannot be classified as general damages are special damages – that is, damages
that would not normally be expected to flow from the type of breach in question, but which arise
due to the special circumstances of the case. The general rule is that the party in breach cannot be
held responsible for special damages. By way of exception, however, the innocent party will be able
to recover special damages, if he or she can prove that both of the following requirements were met:
1. the damages were actually foreseen or reasonably foreseeable at the time of entry into the
contract (the contemplation principle); and
2. the parties entered into the contract on the basis of their knowledge of the special
circumstances, and thus can be taken to have agreed, expressly or tacitly, that there would be
liability for damages arising from such special circumstances (the convention principle).
The convention principle was introduced into our law by the Appellate Division in the case of Lavery
& Co Ltd v Jungheinrich. Today, there is general agreement that the principle is unrealistically strict
in its insistence that an agreement must be shown before the breaching party can be held liable for
special damages. The courts themselves appear to share this view. However, until such time as the
Lavery decision is overruled, the convention principle must unfortunately be considered to be part of
our law.

The debate about legal causation rages on.


Christie convincingly argues that the convention principle operates too strictly and if applied literally
would lead to many claims for special damages being denied where it would generally be thought
fair to award them. The courts have somewhat softened the impact of this strict rule by sometimes
finding tacit agreements under circumstances where it was unlikely to have occurred.
There is strong support for an approach based simply on the contemplation principle. Christie states
that liability for special damages should be a policy decision based on the subject matter and terms
of the contract or the knowledge of both parties of the special circumstances involved – that is, on a
consideration of whether such damages were reasonably foreseeable at the time of the conclusion
of the contract for the parties involved.
Although the distinction between general damages and special damages is firmly entrenched in the
case law, there have been convincing arguments that the whole distinction (which has its origins with
the writer Pothier and subsequently in English law) is suspect and should be done a way with. De
Wet and Van Wyk argue that this distinction is artificial, theoretically tainted and not useful in
practice for the following reasons: (a) the distinction is based on a fiction, namely that there is a tacit
agreement that the guilty party will indemnify the innocent party for its damages; and (b) the
quantum of the damages ought to be determined at the time of the breach of the contract and not
at the time of the conclusion of the contract. In Thoroughbred Breeders’ Association v Price
Waterhouse, Nienaber JA indicated that it may be possible to do away with the distinction between
general and special damages and subsume the distinction within the test for remoteness. He states:
[51] That being so it is not strictly speaking necessary to revisit, in general, the dichotomous
orthodox approach of this Court to remoteness in contract. Nor is this the occasion, as it was not the
occasion in both Shatz Investments (supra at 554F-G) and Holmdene Brickworks (supra at 688A), to
review limb (b) in particular. (The ‘convention principle’ embraced by Wessels JA in Lavery & Co v
Jungheinrich (supra at 176) has long been discredited in England. Compare The Pegase (supra at 182-
3); Cartwright (op cit at 492).) But it may be worth noting that this Court’s approach to legal
causation within other disparate fields such as crime, delict, insurance and latterly, perhaps, estoppel
has undergone considerable evolution in recent years by the development of a new model for
causation sometimes termed the flexible or supple test. (Compare S v Mokgethi en Andere 1990 (1)
SA 32 (A) at 39I-41A; International Shipping Co (Pty) Ltd v Bentley 1990 (1) SA 680 (A) at 700H-701F;
Smit v Abrahams 1994 (4) SA 1 (A) at 15B-18H; Stellenbosch Farmers’ Winery Ltd v Vlachos t/a Liquor
Den case No 117/99, not yet reported.) * In Standard Chartered Bank of Canada v Nedperm Bank Ltd
1994 (4) SA 747 (A) at 765A-B the new test was described, again by Corbett CJ, as
‘. . . a flexible one in which factors such as reasonable foreseeability, directness, the absence or
presence of a novus actus interveniens, legal policy, reasonability, fairness and justice all play their
part’.
[52] When the matter, which was deferred for future consideration in Shatz’s case supra, does
eventually come before this Court as a pertinent issue, it may be appropriate to employ the learning
developed in those cases to good advantage. With breach of contract, as in delict and estoppel but
unlike insurance (which entails the interpretation of the terms of the policy – compare Napier v
Collett and Another 1995 (3) SA 140 (A)), the exercise would involve measuring the consequences of
wrongful conduct by a composite legal yardstick.
Apportionment. The innocent party needs to prove only that the breach of contract was a cause of
the loss, not that it was the dominant or preponderant cause of the loss.
In Thoroughbred Breeders’ Association v Price Waterhouse, the Supreme Court of Appeal
authoritatively decided that the Apportionment of Damages Act does not apply to a claim for
contractual damages. Thus, where the loss suffered by the plaintiff in a contractual claim results
partly from the breach of contract committed by the other party and partly from the plaintiff’s own
fault, there can be no apportionment or reduction of the damages. The party in breach is liable for
the full amount of the loss, even if the plaintiff’s fault was the dominant or pre-eminent cause of that
loss.
13.5.2.3 The mitigation rule
In Victoria Falls and Transvaal Power Co Ltd v Consolidated Langlaagte Mines Ltd, the court
authoritatively stated the mitigation rule: where a breach of contract has occurred, the innocent
party cannot merely sit back and allow his or her losses to accumulate; the party must take
reasonable positive steps to prevent the occurrence or accumulation of losses.
Should the innocent party fail to take such steps, the party in breach can raise the mitigation defence
against a claim for damages by the innocent party. If it is established that the innocent party failed to
mitigate losses, his or her claim for damages will be reduced or even totally wiped out, depending on
whether such putative action would have reduced or prevented such losses.
For instance, if a wholesaler fails to deliver certain products to a retailer, the retailer should take
reasonable steps to make a replacement purchase, even if it should be at a higher price. If it fails to
make a substitute purchase, it will probably not be successful in a claim for lost profits.
The rule does not require the innocent party to do anything more than a reasonable person would
do under the same circumstances. Reasonable expenses incurred in carrying out the mitigation steps
may be claimed as additional damages suffered. The onus of proving what steps could reasonably
have been taken, or that the expenses incurred were unreasonable, rests on the party in breach.

QUESTION 3
In his shop window M, a jeweller, displays two identical brooches, one made of diamonds, the other
of crystal. Walking into the shop N points out the crystal brooch to M and offers to buy it for R20 000
Nothing is said about the material from which the brooch is made N thinks he is buying a diamond
brooch, while M believes that N wants to buy the crystal brooch M accepts N's offer
3 1 did a VALID contract of sale arise? Substantiate your answer fully Do not discuss
misrepresentation. Do not apply the Consumer Protection Act (10)

Answer
Identifying the problem (maximum of 2 marks)

The question is whether the parties had reached consensus: Was N’s mistake regarding the material
the brooch was made of material?

Discussing the relevant law applicable to the problem, referring to the relevant case law, (maximum
of 5 marks) AND applying the law to the facts of the problem (maximum of 2 marks)

The question whether the contract is VALID indicates this question deals with one of the
requirements for a valid contract. The requirements for a valid contract are: consensus, capacity,
formalities, legality, possibility and certainty. The facts indicate that consensus could be absent: N
thinks he is buying a diamond brooch, while M believes that N wants to buy the crystal brooch.

Consensus has three elements: the parties must seriously intend to contract, be of one mind as to
the material aspects of the proposed agreement (the terms and the identity of the parties to it), and
be conscious of the fact that their minds have met.

A material mistake relates to or excludes an element of consensus, but a non-material mistake does
not exclude actual agreement between the parties, because it does not relate to an element of
consensus. A non-material mistake relates purely to the reason or motive of the mistaken party for
entering into an agreement and hence it is usually referred to as an error in motive.

In the problem it is clear that the parties had the serious intention to contract and that they were
conscious of each other’s agreement. The question is whether N thought that he was buying a
diamond brooch and M’s belief that N wanted to buy the crystal brooch excluded consensus
regarding the terms of the contract.

In short the question is whether we are dealing with an error in corpore or an error in substantia. An
error in corpore is a material mistake concerning the subject matter of the contract, or rather, the
object of the performance. The most prominent example is where property is purchased and the
parties have completely different properties in mind. For instance, in Maresky v Morkel, the
respondent was under the impression that he was purchasing property at site A, because of an
advertisement in a newspaper to that effect. The appellant’s agent failed to point out to him that the
property was in fact situated at site B. The court found that the respondent’s mistake was in corpore,
which vitiated his consent to the contract. An error in substantia It is a mistake regarding an attribute
or characteristic of the subject matter of the contract (object of performance) that is generally not
regarded as material. In Trollip v Jordaan, the appellant purchased a farm from the respondent in
terms of a deed of sale that correctly reflected the boundaries and extent of the farm. However, the
appellant was misled into believing that the farm included more afforested land than it did, because
the respondent’s agent had pointed out the boundaries of the farm incorrectly. The majority of the
court found that the appellant’s mistake was not in corpore. This decision implies that error in
substantia is not regarded as material in our law. Moreover, such a mistake usually will amount to an
error in motive, which is not material or operative. A useful question to ask is: Did both parties have
delivery of the same thing in mind? If so, there is no mistake which excludes consensus. If not, there
is a material mistake.

Did M and N want to buy and sell different brooches (N the diamond brooch and M the crystal
brooch)? If so, we have an error in corpore. Or did M and N want to buy the same brooch but
thought that this brooch had different characteristics (N that the material it was made of was a
diamond and M that the material it was made of was a diamond)? From the facts it is clear that
parties wanted to buy and sell the same brooch but they thought it was made from different
material. The mistake is non-material and thus consensus is present.

Discretionary Marks

The motivation in the memo which is correct states that there was no material mistake, therefore a
valid contract exists based on consensus. However, if a student states that there is a material mistake
resulting in no contract based on consensus (which is strictly wrong), but carries on to motivate that
a contract nevertheless exists in terms of the reliance theory, a maximum of 5 marks should be
awarded. Here the motivation basically is that despite the mistake, N led M reasonably to believe
that a binding contract did exist. N makes the misrepresentation that he is buying the crystal brooch;
M is misled by this misrepresentation; and a reasonable person would also have been misled to
believe that N was buying the crystal brooch. For this type of analysis, a student could get between 4
or 5 out of 10 marks, depending on how the law is discussed and presented.

Question
3 2 Would a VALID contract arise if both M and N believe that they are contracting for the diamond
brooch and both are aware of the other's belief? Substantiate your answer fully Do not apply the
Consumer Protection Act. ( 10)

Answer
Classification of mistake
Mistakes in contract can be classified in different ways. Our law has evolved to categorise mistakes
using a variety of distinctions and having a variety of consequences for contracting parties. It is
important to understand these distinctions and classifications in order to identify the legal
consequences correctly.
3.2.1 Unilateral, mutual and common mistake
Apparently under the influence of English law, the courts tend to categorise mistakes as being
unilateral, mutual or common. It seems that a unilateral mistake occurs where only one party is
mistaken, while the other party is aware of his or her mistake. Mutual mistake refers to the instance
where both parties are mistaken about each other’s intention and are at cross-purposes. In other
words, neither is aware of the other’s mistake.
The distinction between unilateral and mutual mistake is of little practical significance, since, in most
cases, a mutual mistake is present – as the following example illustrates. Suppose that A agrees to
purchase B’s property. A erroneously mistakes another property for B’s property, while in fact B’s
property is situated further down the road. If B is unaware of A’s misapprehension, B mistakenly
believes there is consensus and the case is therefore one of mutual mistake; but if B is aware of A’s
mistake, it is a case of unilateral mistake. Both scenarios, however, are cases of dissensus and the
distinction therefore has little importance. Unsurprisingly, the term unilateral mistake is sometimes
used to describe both instances.
A common mistake is one that is shared by the parties and differs fundamentally from unilateral or
mutual mistake, because it does not lead to dissensus. Nonetheless, it results in a contract being void
as it rests on a common underlying supposition that is later revealed to be incorrect. Since common
error is not a true case of mistake (or rather dissensus), it will be discussed separately.

Table 3.1 Common, mutual and unilateral mistake


A and B make the same mistake
There is consensus
The consensus is based on a common, false
supposition/assumption

A and B are at cross-purposes


Each is mistaken about the other’s intention
There is dissensus

A is mistaken about B’s intention


B knows of A’s mistake, but remains silent
There is dissensus

3.2.2 Irrelevant and relevant mistake


It is sometimes stated that a mistake does not negate consensus (and is therefore an irrelevant
mistake) if it did not affect the mistaken party’s decision to enter into a contract. If the mistaken
party would have entered into the contract, despite a mistake that causes dissensus, it seems that
the mistake may be regarded as irrelevant in so far as the question of agreement is concerned. In
Khan v Naidoo, the appellant signed an agreement as surety for the debt of her son while under the
impression (induced by the misrepresentation of her son) that she was consenting to something
completely different. Although the appellant’s mistake would normally indicate an absence of
consent on her part, the court held that she was bound to the suretyship because she would have
signed the document even if she had been aware of its true nature. From this, it seems that a
mistake must influence a party’s decision to conclude a contract in order to be relevant to the
question of possible dissensus.
3.2.3 Material and non-material mistake
If we accept that actual agreement is the primary basis of contractual liability in accordance with the
will theory, then the first question that arises when determining the existence of a valid contract is
whether the parties in fact reached consensus ad idem. On the basis of the will theory, if consensus
is reached, and provided the other requirements for a valid contract are met, a legally binding
contract arises. However, if the parties are not in agreement, then no contract will exist on the basis
of the will theory. In such circumstances, unless the contract can stand on the secondary basis of
reliance, it will be void ab initio.
The distinction between a material (operative or essential) mistake and a non-material mistake is
crucial to the question of consensus. A material mistake is an error that vitiates or negates actual
consensus between the parties. To this end, a material mistake must relate to or exclude an element
of consensus. Conversely, a non-material mistake does not exclude actual agreement between the
parties, because it does not relate to an element of consensus. Consequently, in the case of a non-
material mistake, a valid contract will still arise, although it may be voidable (rescindable) if
consensus has been obtained in an improper manner by way of misrepresentation, duress, undue
influence or commercial bribery.
The distinction between a material and non-material error is vital in determining the appropriate
resolution to a situation where a party was mistaken about something when entering into a contract.
On the one hand, if the mistake is material, the issue is one of dissensus or true ‘mistake’ and the
secondary principles of reliance have to be applied to determine whether a contract based on
reliance has arisen – or whether the mistaken party may be exonerated from contractual liability
because the potential contract lacks a legal basis. On the other hand, if the mistake is non-material, a
consensual contract exists and there is no need to apply the principles of reliance, because the
contract already has a legal basis (will theory). It is clear that materiality has a very specific
connotation in the context of mistake and it should not be confused with materiality as applied to
other areas of the law of contract.

Confusing aspects to the inducement requirement for a relevant mistake


The Khan v Naidoo approach referred to above requires a mistake (if it is to be relevant) to act as an
inducement to contract, in the sense that the mistake must influence a mistaken party’s decision to
enter into a contract. However, an inducement requirement within the context of material mistake
excluding agreement may lead to some confusion because inducement is also an accepted
prerequisite for determining the materiality of an actionable misrepresentation. Consequently, it is
suggested that inducement should be confined to non-material mistake, and that when determining
the materiality of a mistake, the appropriate route should be to focus merely on whether the mistake
was of a type that vitiates actual consent to the contract or not. Viewed from this perspective, the
mistake in Khan v Naidoo may well be regarded as having been material.
Figure 3.1 Mistake: material and non-material

3.2.3.1 Material mistake


Chapter 1 set out the elements of consensus, being that the parties must:
seriously intend to contract;
be of one mind as to the material aspects of the contract, namely, the terms of the proposed
agreement, and the identity of the parties to it; and
be conscious of the fact that their minds have met.
If for some reason or other, the parties are not in agreement about one (or more) of these elements,
there is a material mistake. It is worth taking a closer look at the elements of consensus and at some
examples of material mistake.
Serious intention to contract. If a contracting party lacks the intention to be legally bound by an
agreement (animus contrahendi), there can be no consensus. There are various reasons why a party
may lack the intention to be bound in law by an agreement. For instance, a party may make a
declaration to contract in jest (rixa) or merely intend a social agreement (gentlemen’s agreement) to
arise. In Mondorp Eiendomsagentskap (Edms) Bpk v Kemp en De Beer, the appellant’s
representative signed a document that the respondent alleged amounted to an undertaking to pay
certain moneys to the respondent. The appellant denied that the document contained such an
undertaking and contested its liability. The majority of the court found that no contract between the
parties had come into existence. However, Rumpff CJ clearly stated in his judgment that the
appellant’s representative did not intend to give an undertaking in terms of which the appellant
would become the debtor of the respondent. In other words, the representative did not sign the
document with the intention of incurring contractual liability for the appellant. Furthermore, it
should be mentioned that parties sometimes outwardly give the impression that they wish to create
certain legal consequences, while they actually intend different consequences or no juristic
consequences at all. Although not a case of mistake, it should be noted that the law judges such
simulated juristic acts in accordance with the actual intention of the parties and not the simulated
contract (plus valet quod agitur quam quod simulate concipitur).
Agreement as to the material aspects of the contract. The parties must be ad idem regarding the
consequences they intend to create; namely the persons between whom the obligations are to be
created and the content of the obligations – that is, the performance(s) to be rendered.
A mistake regarding the parties to a contract will usually be material. If a party inadvertently accepts
the offer of one party, while it actually intended to accept the offer of a completely different party,
the resultant error will be material. Similarly, if an offeror intends to make an offer to a specific
person only and the offer is purportedly accepted by the offeree and another party jointly, there
cannot be consensus.
The most common of operative mistakes are those that relate to the material terms of contracts,
and, more specifically, the aspect of contractual performance. For example, the parties may be of
different minds regarding the subject matter of the contract. In Allen v Sixteen Stirling Investments
(Pty) Ltd, the plaintiff believed he was purchasing the property pointed out to him by the
defendant’s agent. However, the deed of sale that he signed referred to another property, which the
plaintiff did not intend to buy. In other words, the parties were not in agreement as to what was
being bought and sold.
A further common type of material mistake deals with the situation where a contracting party does
not understand the legal consequences of a contractual provision, usually because the mistaken
party is unaware of the particular provision in a document that embodies the terms of the contract.
So, for instance, if a party is unaware of a contractual term that excludes remedies for
misrepresentation by the other party, the resultant mistake will be material. Similarly, where a
company director signs a credit application form on behalf of the company, but is unaware that the
application contains a suretyship clause binding him personally as surety for the debts of the
company, the director’s mistake will be material.
Generally speaking, any mistake regarding a contractual clause that permits a party unilaterally to
vary an aspect of performance, such as the date on which performance is to occur, or even entitling
a party to release itself from liability altogether in certain circumstances, will be material. In all these
examples, the parties are not ad idem as to a material aspect of the proposed contract between
them.
Consciousness of agreement. An integral element of consensus and the will theory is that the parties
must not just have coinciding declarations of intention, but must also be aware of each other’s
intention for a consensual contract to arise. So a mistake that precludes conscious agreement
between the parties will be material. The parties’ declarations of intention are usually expressed in
the form of an offer and an acceptance. However, the offeree must be aware of the offer before he
can accept it and, likewise, the offeror must be aware of the acceptance for a true meeting of the
minds (concursus animorum) to occur. A leading example of dissensus resulting from a lack of
conscious agreement is Bloom v American Swiss Watch Co. The respondent promised a reward to
any person who could provide information leading to the arrest of thieves who had stolen jewellery
from it. The appellant provided such information while he was unaware of the promise of reward.
Later, the appellant became aware of the offer and proceeded to claim the reward from the
respondent. The court found for the respondent; it held that the appellant could not accept an offer
until he knew of it; there was therefore no consensus and hence no contract.
3.2.3.2 Non-material mistake
Non-material mistakes, while usually influencing a party’s decision to enter into a contract, do not
affect an element of consensus. Notwithstanding a non-material mistake, if the parties are ad idem
as to the material aspects of the proposed contract, a consensual contract comes into being.
Typically, a mistake of this nature relates purely to the reason or motive of the mistaken party for
entering into an agreement and hence it is usually referred to as an error in motive.
In Diedericks v Minister of Lands, the defendant extended an offer to the plaintiff to purchase
property from the latter. However, the defendant could merely have invoked a clause in an existing
lease contract in terms of which the defendant would have repossessed the property at a much
cheaper rate. The plaintiff accepted the offer, but subsequently the defendant refused to honour its
undertaking, alleging that the offer was made as a result of a clerical error. The plaintiff asked the
court to declare the contract valid. The court found that the defendant’s mistake related to its motive
for making the offer, and did not exclude mutual assent (consensus) between the parties.
Consequently, a valid contract existed.
It should be stressed that just because the parties have reached consensus and a contract has arisen
does not necessarily mean that a mistaken party is without legal recourse. If the mistake in motive
was induced by the misrepresentation of the other party, the mistaken party may have a remedy,
provided the requirements for the particular form of misrepresentation have been complied with.
3.2.4 Traditional classification of material and non-material mistake
Historically, mistakes have been categorised according to type, and the materiality of a mistake has
been determined on the basis of the type of mistake in question. This does not mean that one is
dealing with a completely different set of mistakes to those previously discussed. It is merely a
different way of stating the same principles with reference to prominent instances. There are four
traditional classifications.
1. Error in corpore: This is a material mistake concerning the subject matter of the contract, or
rather, the object of the performance. The most prominent example is where property is purchased
and the parties have completely different properties in mind. For instance, in Maresky v Morkel, the
respondent was under the impression that he was purchasing property at site A, because of an
advertisement in a newspaper to that effect. The appellant’s agent failed to point out to him that the
property was in fact situated at site B. The court found that the respondent’s mistake was in corpore,
which vitiated his consent to the contract.
2. Error in negotio: This is a material mistake relating to the true nature of the contract (the
juristic act) concerned. An error in negotio is fairly rare, but Khan v Naidoo may be construed along
these lines. There, the appellant was illiterate and signed an agreement as surety for the debt of her
son, while under the impression that the document had to do with the transfer of property to her.
The appellant was clearly mistaken as to the real nature of the juristic act involved. Since such a
mistake will have a bearing on the performance(s) involved, it may be regarded as an example of
error in corpore.
3. Error in persona: This is a (usually material) mistake regarding the identity of the other party
to the contract. For example, in Kok v Osborne, the defendant believed that he was selling his
property to the plaintiff and another party jointly, whereas he was in fact only contracting with the
plaintiff as purchaser. The court found that the defendant was mistaken as to the identity of the
party or parties with whom he was contracting and that there was therefore dissensus. However, the
courts have complicated matters: at times, they have indicated that an error in persona is material
only if the identity of a party is of vital importance to the mistaken party. In so doing, they have
adopted the view of Pothier, which seems to have been in accord with the Roman-Dutch law. This
approach of the courts has also been linked to the notion that only mistakes that play a material role
in the decision of the mistaken party to enter into a contract can be regarded as legally relevant.

There may be further distinctions to be made regarding error in persona.


Joubert convincingly suggests that one should distinguish between an error in respect of the identity
of a contracting party, which is always material because the identity of a party is vital for consensus
to exist, and cases where there is merely a mistake regarding the name or, perhaps, some other
attribute of a party (in which case the mistake is not material).
4. Error in substantia (error in qualitate): This kind of mistake is not seen as material in our law.
It is a mistake regarding an attribute or characteristic of the subject matter of the contract (object of
performance) that is generally not regarded as material. In Trollip v Jordaan, the appellant
purchased a farm from the respondent in terms of a deed of sale that correctly reflected the
boundaries and extent of the farm. However, the appellant was misled into believing that the farm
included more afforested land than it did, because the respondent’s agent had pointed out the
boundaries of the farm incorrectly. The majority of the court found that the appellant’s mistake was
not in corpore. This decision implies that error in substantia is not regarded as material in our law.
Moreover, such a mistake usually will amount to an error in motive, which is not material or
operative.

The traditional classifications are not exhaustive.


The traditional list classifying mistake is certainly not exhaustive and, although useful in recognising
specific types of material and non-material mistake, the real focus should be on whether a particular
mistake relates to the content of an obligation (an operative mistake) or merely to the motive for
incurring such an obligation (a non-operative mistake). The question should always be whether there
was consensus, or conversely dissensus. In keeping with such an approach, it might also be prudent
to regard all instances of error in persona as material.

OR

The contract is valid.

3.2 Would a VALID contract arise if both M and N believe that they are contracting for the
diamond brooch and both are aware of the other’s belief? Substantiate your answer fully. Do not
apply the Consumer Protection Act. (10)

Answer
Did the parties make a common mistake?

Discussing the relevant law applicable to the problem, referring to the relevant case law, (maximum
of 5 marks) AND applying the law to the facts of the problem (maximum of 2 marks)
The parties, N and M, are now making the same mistake because they both believe the brooch is a
diamond brooch. This may be a common mistake because the mistake is shared by the parties and
does not relate to the intention of either party. Although a common mistake does not lead to
dissensus it nevertheless results in the contract being void as it rests on a common underlying
supposition that is later revealed to be incorrect. Common error is thus not a true case of mistake (or
rather dissensus).

In Dickinson Motors (Pty) Ltd v Oberholzer, the plaintiff’s son purchased two Plymouth motor cars on
hire purchase from two different parties – car A from the defendant and car B from a third party.
After exchanging car B for his father’s Hudson motor car, the son disappeared leaving unpaid
amounts on both vehicles. The defendant obtained judgment against the son in respect of car A and
then had the car in the plaintiff’s (the father’s) possession attached to satisfy the debt under the
mistaken impression that it was car A, whereas it was in fact car B. The plaintiff while under the same
mistaken belief that the car in his possession was car A, paid the outstanding debt to the defendant
to retain possession of the vehicle. Shortly thereafter, this same car was attached, but this time
legally at the instance of the third party who had sold the vehicle to the son. The plaintiff claimed
return of the amount which he had paid to the defendant. He succeeded and the Appellate Division
held that the contract between him and the defendant was void, because their contract was based
on a common mistake.

There are several views as to why common mistake renders the contract void. Probably the most
accepted explanation is the ‘implied term’ theory, which in general holds that the particular
circumstances may justify the inference that the parties implicitly (tacitly) agreed to make the
existence of their contract dependent on the truth of a material fact, or rather a supposition. A
supposition or assumption occurs where the parties render their contract dependent on the
presence (or absence) of some fact of the past or present. If the fact is present, there is a binding and
enforceable contract; if not, there is no binding contract and the parties may reclaim whatever they
have performed (restitution).

For a common error to have an effect on a contract, it must count as a term of the contract, either
expressly or by implication (tacitly). If a common error is not a term of a contract, it will usually
amount to little more than an error in motive, albeit a common error in motive. It will then have no
bearing on the validity of the contract. In Dickinson Motors (Pty) Ltd v Oberholzer, the court held
that the common mistake must have been ‘vital to the transaction’, in the sense that neither party
would have agreed to the contract if they had known the true situation. Presumably, if the common
mistake is fundamental in such a manner, the supposition would qualify as a tacit term.

In our problem, on the one hand it could be motivated that the common mistake was vital to the
transaction because neither party would have concluded the contract of sale had they known the
truth. This would mean that the contract is void. On the other hand, one could also possibly motivate
that the seller of the brooch would have nevertheless wanted to proceed with the sale, despite the
mistake. This would have meant that the transaction was not vital to the transaction, as one of the
parties would have contracted anyway. This means that the contract is valid. The facts are not
entirely clear on this issue, therefore both motivations should be allowed.
Discretionary Marks:

Where a student assumes that a purchase has been made for the diamond brooch (and not the
crystal brooch), and they motivate that a valid contract exists based on consensus and the will
theory, you can award a maximum of 5 marks, if the elements of consensus are set out, and the
application to the facts is done well.

Hutchison & Pretorius (eds) The Law of Contract in South Africa 2nd ed (2012) par 3.8 and 10.3.7.1:

3.8 Common mistake


A common mistake differs fundamentally from unilateral or mutual mistake, in that it does not lead
to dissensus; it nonetheless results in a contract being void. Therefore, common error is not a true
case of mistake and the principles discussed above do not apply. In such instances, both parties to a
contract make the same mistake, with the important proviso that the mistake does not relate to the
intention of either party. In fact the parties are in complete agreement (consensus ad idem) and each
knows the intention of the other and accepts it. However, both are mistaken about some underlying,
fundamental fact, in that they have a mistaken assumption about a present or past fact.
Common error results in the contract being void. For example, in Dickinson Motors (Pty) Ltd v
Oberholzer, the plaintiff’s son purchased two Plymouth motor cars on hire purchase from two
different parties – car A from the defendant and car B from a third party. After exchanging car B for
his father’s Hudson motor car, the son disappeared leaving unpaid amounts on both vehicles. The
defendant obtained judgment against the son in respect of car A and then had the car in the
plaintiff’s (the father’s) possession attached to satisfy the debt under the mistaken impression that it
was car A, whereas it was in fact car B. The plaintiff while under the same mistaken belief that the
car in his possession was car A, paid the outstanding debt to the defendant to retain possession of
the vehicle. Shortly thereafter, this same car was attached, but this time legally at the instance of the
third party who had sold the vehicle to the son. The plaintiff claimed return of the amount which he
had paid to the defendant. He succeeded and the Appellate Division held that the contract between
him and the defendant was void, because their contract was based on a common mistake.
There are several views as to why common mistake renders the contract void. Probably the most
accepted explanation is the ‘implied term’ theory, which in general holds that the particular
circumstances may justify the inference that the parties implicitly (tacitly) agreed to make the
existence of their contract dependent on the truth of a material fact, or rather a supposition. A
supposition or assumption occurs where the parties render their contract dependent on the
presence (or absence) of some fact of the past or present. If the fact is present, there is a binding and
enforceable contract; if not, there is no binding contract and the parties may reclaim whatever they
have performed (restitution).

A common error must amount to a term of the contract.


It is important to note that for a common error to have an effect on a contract, it must count as a
term of the contract, either expressly or by implication (tacitly). If a common error is not a term of a
contract, it will usually amount to little more than an error in motive, albeit a common error in
motive. It will then have no bearing on the validity of the contract. In Dickinson Motors (Pty) Ltd v
Oberholzer, the court held that the common mistake must have been ‘vital to the transaction’, in the
sense that neither party would have agreed to the contract if they had known the true situation.
Presumably, if the common mistake is fundamental in such a manner, the supposition would qualify
as a tacit term.

10.3.3.2 Tacit terms


A tacit term is one that the parties did not specifically agree upon, but which (without anything being
said) both or all of them expected to form part of their (oral or written) agreement. It is a wordless
understanding, an unarticulated term, having the same legal effect as an express term.
Tacit terms are established by considering the express terms and the circumstances that surrounded
the genesis of the contract. Some cases have suggested that the court may also have regard to the
subsequent conduct of the parties.
In ascertaining whether a contract contains a tacit term, the courts often employ the officious
bystander test. The court supposes that an impartial bystander had been present when the parties
concluded their agreement and had asked the parties what would happen in a situation which they
did not foresee and for which their express agreement consequently did not provide. If they were to
agree that the answer to the stranger’s question was self-evident, they are taken to have meant to
incorporate the term in their contract and to have tacitly agreed on it. This applies not only to terms
that the parties thought about, but found unnecessary to incorporate into the contract, but also to
terms that they would have incorporated if they had thought about them. A tacit term may therefore
express not only the parties’ actual intention but also what they may be presumed to have intended
– that is, an imputed intention.
In deciding whether a tacit term is to be imported into a contract, two things are necessary. Firstly, it
is obvious that the term sought to be imported must not conflict with an unambiguous express term
of the agreement: a tacit term after all only supplements the contract by providing a term that the
parties failed to agree upon expressly.
That a tacit term can only be imported into a contract if it is necessary in a business sense to give
efficacy to the contract was decided by Scrutton LJ in Reigate v Union Manufacturing (Ramsbottom).
This business efficacy test has been consistently applied by our courts. The implication must be
necessary, and not merely reasonable or convenient and much will depend on the express terms of
the agreement and the surrounding circumstances at the time it was entered into.

10.3.7.1 Suppositions
Parties may decide to conclude a contract on the supposition or assumption that a certain state of
affairs exists or existed. Thus, a second-hand copy of the third edition of a text book might be offered
for sale. A student, who needs a copy for one of his or her courses, is interested in purchasing the
book, but only wants to buy it if it is the latest edition. If neither the student nor the seller knows
whether it is the latest edition, they may contract on the basis that it is indeed the newest edition; if
it turns out that there is a later edition, the supposition is false and the contract will be void; if, on
the other hand, it is the latest edition, the contract will be valid.
A supposition or assumption must be shared by both parties in order to form the basis for the
contract. In other words, it must amount to a term of the contract. If only one of the parties has the
supposition in mind, it will be legally irrelevant and tantamount to an error in motive.
At first sight, a supposition seems similar to a condition. It is true that both may be either express or
tacit, and both make the effect of a contract depend on something that actually is uncertain. The
difference is that a condition relates to a future uncertain event, whereas a supposition relates to a
past or present state of affairs.

The contract is void, or the contract is void, depending on the motivation presented.

QUESTION 4
X, who is deeply troubled and in a highly emotional state, informs his minister (of religion) that he
has bought a number of shares at a fraction of their true value and that, although there was nothing
legally wrong with the transaction, he feels that he has acted in a way which was morally wrong. The
minister advises X to donate the shares to the church to rid him of the feelings of guilt, and X there
and then signs a contract in terms of which he donates all the shares to the church A month later X
seeks to have the contract SET ASIDE Will he succeed? Discuss fully. Do not apply the Consumer
Protection Act to this question. (10)

Answer

Identifying the problem (maximum of 2 marks)


Whether the minister has unduly influenced X in concluding this donation (if he has the contract is
voidable).
Discussing the relevant law applicable to the problem, referring to the relevant case law, (maximum
of 5 marks) AND applying the law to the facts of the problem (maximum of 2 marks)

X’s desire of having the contract set aside can relate to either having the contract declared void or
having the contract rescinded because consensus may have been obtained in an improper manner.

Undue influence

The facts of this problem indicate that X asked his minister for advice and the minister influenced him
to donate the shares to the church. This may be undue influence. South African courts have been
influenced by English law in recognising undue influence. Undue influence has been expressly
recognised as part of our law since a case in 1948 and this position has been confirmed by two later
Appellate Division cases.

In English law, the existence of a special relationship between the parties gives rise to a rebuttable
presumption of undue influence. It seems fairly clear that this is not the case in our law. The party
who alleges undue influence has the onus to prove that it occurred. The existence of a confidential
relationship between the parties is a relevant factor in discharging that onus.

There are two forms of such a close relationship: a fiduciary relationship (eg as between a doctor
and patient or attorney and client), or a relationship of respect and reverence (eg as in the case of
parent and child, guardian and ward or religious leader and disciple). In such relationships the one
party is entitled or obliged to advise and assist the other, and then misuses his or her superior
position to influence and undermine the will of the latter. In our problem a relationship of respect
existed between X and the minister.
Undue influence is a form of improper pressure brought to bear upon a person in order to induce
him or her to enter into a contract. According to AD cases a party who seeks to set aside a contract
on the ground of undue influence must establish:
1. that the other party obtained an influence over him or her;
2. that this influence weakened his or her powers of resistance and rendered his or her will
compliant; and
3. that the other party used this influence in an unscrupulous manner to persuade him or her
to agree to a transaction that was prejudicial to him or her, and which he or she would not have
concluded with normal freedom of will.

It is not clear why prejudice should be proved since the basis of the action is vitiated consent. Mere
inducement to contract should suffice.

The minister clearly had influence over X and this definitely weakened X’s power of resistance and
made his will compliant. The minister could have advised X to pay the seller of the shares the market
value of the shares but instead saw the opportunity of enriching the church. The minister thus
unscrupulously influenced X to donate the shares to the church. This donation was clearly to the
prejudice to of X: he stands to lose his shares. The fact that X wants to set the contract aside
indicates that X would not have donated the shares with his normal freedom of will.

Where the undue influence is brought to bear by an outsider, the party influenced may set aside the
contract only if the other contracting party was aware, at the time of the transaction, that the
influence had been exercised. The minister either acted as agent of the church or he was an
outsider.

On the one hand, it may seem that the minister was an outsider as there are no indications in the
facts of this problem that the minister acted on behalf of the church. There is also no indication that
the church knew of the undue influence.

On the other hand, the possibility exists that the minister acted as an authorised agent of the church.
Here the legal effect is that the actions of the minister are in fact the actions of the church. This
would mean that X was unduly influenced by the church.

The giving of appropriate advice (maximum of 1 mark)

If the minister is an outsider, X may not have the contract set aside. This is because there was either
no undue influence on the part of the church, nor did the church know of the minister’s undue
influence.

OR:

If the minister was acting as an agent of the church, then X may have the contract set aside. This is
because the contract would be voidable at his instance, due to the undue influence.
A maximum of 2 bonus marks should be awarded to students who have the presence of mind to
identify that there is a formality requirement to be complied with, for such a donation. But
remember that a maximum a student can get in total for this question is 10 marks.

Formalities
The requirements for a valid contract are: consensus, capacity, formalities, legality, possibility and
certainty. The facts of this problem indicate that the formalities may be implicated because X signs a
written contract of donation. This is an executory donation, because there is no indication in the
facts that the donation has been performed. In order to be valid such donation should be in writing
and signed by X. The donation complies with the required formalities and is thus valid.
6.2.1.3 Donation
Section 5 of the General Law Amendment Act provides that no executory contract of donation
entered into after 22 June 1956 will be valid unless its terms are embodied in a written document
signed by the donor or by a person acting on the donor’s written authority granted in the presence
of two witnesses. The deed of donation itself does not have to be witnessed. The donee has to
accept the offer, but need not sign the contract. An executory donation is one that has not yet been
carried out – that is, has not been completed by delivery of the promised benefit. The purpose of
this provision is apparently to make sure that the donor has a serious intention to conclude the
contract. However, an oral donation completed by delivery of the benefit is a binding contract.

Hutchison & Pretorius (eds) The Law of Contract in South Africa 2nd ed (2012) par 4.4:
4.4 Undue influence
Like duress, undue influence is a form of improper pressure brought to bear upon a person in order
to induce him or her to enter into a contract. However, in the case of undue influence, the pressure is
more subtle, involving an insidious erosion of the victim’s ability to exercise a free and independent
judgement in the matter, rather than threats or intimidation. In most instances of undue influence,
there is a close relationship between the parties – either a fiduciary relationship, as between a
doctor and patient or attorney and client, or one of respect and reverence, as in the case of parent
and child, guardian and ward or religious leader and disciple – which entitles or obliges the one party
to advise and assist the other, and the former then misuses his or her superior position to influence
and undermine the will of the latter.
4.4.1 Origins of the doctrine
The term ‘undue influence’, unknown to our old authorities, comes to us from English law where the
originally narrow scope of the common law concept of duress led the courts of equity to develop the
more elastic doctrine of undue influence. References to undue influence as a consensual defect
giving rise to restitutio in integrum gradually found their way into our law reports and textbooks, but
it was not until 1948 that a court firmly declared that a contract could be set aside on this ground.
Although controversial, this decision was confirmed by the Appellate Division in Preller v Jordaan,
where a majority of the court held that the grounds for restitutio in integrum in Roman-Dutch law
were wide enough to cover what is today referred to as undue influence. Whether or not this finding
is historically correct, the matter has been put beyond all dispute by the subsequent case of Patel v
Grobbelaar, which confirms that the doctrine of undue influence undoubtedly forms part of our
modern law. And indeed, the facts of Patel’s case indicate the need for such a doctrine and the
overall desirability of its recognition by our courts. The laissez-faire attitude displayed by Van den
Heever JA in Preller’s case is neither attractive nor appropriate in modern times, if it ever was.
4.4.2 Requirements
According to the Patel case, adopting the phraseology used in Preller v Jordaan, a party who seeks to
set aside a contract on the ground of undue influence must establish:
1. that the other party obtained an influence over him or her;
2. that this influence weakened his or her powers of resistance and rendered his or her will
compliant; and
3. that the other party used this influence in an unscrupulous manner to persuade him or her
to agree to a transaction that
was prejudicial to him or her, and
which he or she would not have concluded with normal freedom of will.
Since the basis of the action is vitiated consent, it is not clear why prejudice should be proved; once
again, it is submitted that mere inducement to contract should suffice.
In English law, the existence of a special relationship between the parties gives rise to a rebuttable
presumption of undue influence. In our law it seems fairly clear that no such principle applies, the
onus being on the party who alleges undue influence to prove that it occurred; but of course the
existence of a confidential relationship between the parties is a relevant factor in discharging that
onus. As this rule demonstrates, we have merely adopted the term ‘undue influence’ from English
law, and not the detailed rules in connection therewith.
Where the undue influence is brought to bear by an outsider, the party influenced may set aside the
contract only if the other contracting party was aware, at the time of the transaction, that the
influence had been exercised. Whether an action for damages will lie against the outsider in such
circumstances, or against the other party to the contract when it is he or she who has exercised the
influence, has not yet been decided; but it seems fairly clear that where the undue influence has
caused loss, the Aquilian action will lie since the conduct of the guilty party is both deliberate and
wrongful.
Being a somewhat elastic concept, undue influence may in some cases be almost indistinguishable
from fraud, while in others it may overlap with duress. As pointed out earlier, however, it is the
absence of intimidation that distinguishes it from duress. Thus, for example, pressure exerted by one
who is in a position of authority over another will usually constitute undue influence; but if the
pressure is accompanied by threats or intimidation, we have a case of duress: the so-called metus
reverentialis.

PVL3702_Oct_2015
QUESTION 1
Z walks into a shop, and puts R10 on the counter and points to a packet of sweets X (the owner of
the shop) takes the money and hands over the sweets to Z Wh1ch statement is CORRECT?

1 There is an oral offer, and acceptance by conduct


2 There is an express offer, and an oral acceptance
3 There is both an offer, and acceptance by conduct
4 There is an offer by conduct, and an express oral acceptance
5 There is a firm oral offer, and an unqualified acceptance
QUESTION 2
Assume the same facts as in question (1). At the stage when Z rece1ves the sweets from X, which
statement(s) is/are CORRECT?

1 An agreement creating obligations is concluded


2 There is an agreement extinguishing obligations
3 There is evidence of a real agreement, because ownership of the sweets has been transferred to z
4 Opt1on 1 and 3
5 Opt1on 1, 2 and 3

QUESTION 3
Which statement IS INCORRECT?

1 The obligations that arise from contract, delict, and unjustified enrichment, are always Imposed
by law, irrespective of the will of the parties
2 The law of contract, delict, and unjustified enrichment, all relate to the law of obligations
3 Breach of contract and a delict are very similar, in that they both constitute civil wrongs and, in
appropriate Circumstances, give rise to a duty to pay damages
4 Sometimes, concurrent contractual and delictual liability can arise, so that the plaintiff may sue on
either basis
5 In the absence of a valid contract, an unjustified enrichment claim may arise when there IS a
transfer of wealth from one person's estate to another person's estate, Without a good legal ground
or cause for this shift

QUESTION 4 I VRAAG 4
In Bloom v American Swiss Watch Co 1915 AD 100, the court decided that

1 an offer must be accepted by the person to whom it was addressed


2 an acceptance must comply with the method of acceptance prescribed by the offeror
3 an acceptance must comply with any formalities set by law or by the offeror
4 an acceptance must be unconditional and unequivocal
5 a person cannot accept an offer of which he/she is not aware

QUESTION 5
In an important case the parties were in each other's presence when a verbal offer was made. The
offer was posted before the deadline set for acceptance, but only reached the offeror after the
deadline. The court had to decide if a contract was concluded. This case IS

1 Carlil v Carbolic Smoke Ball Co [1893] 1 QB 256


2 Crawley v Rex 1909 TS 1105
3 Bloom v American Sw1ss watch Co 1915 AD 100
4 Bird v Sumerville 1961 (3) SA 194 (A)
5 Smeiman v Volkersz 1954 (4) SA 170 (C)
QUESTION 6 6
Which statement IS INCORRECT?

1 Freedom and sanctity of contract (pacta sunt servanda) are values that promote free choice in
entering into contractual relations, as well as certainty and the enforcement of contracts
2 In Brisley v Drotsky 2002 (4) SA 1 (SCA), the court held that concepts such as fairness, good faith
and reasonableness, were not only primary values but also Independent substantive rules in
contract law, which a Judge could apply to enforce contractual Justice between the parties
3 The values of fairness and good faith provide a counterbalance to freedom and sanctity of contract,
and could play a role in the development of a doctrine of unconscionability
4 In Bank of Lisbon and South Africa Ltd v De Ornelas 1988 (3) SA 580 (A), the court held that the
exceptio doli generalis had never been received into Roman-Dutch law and afforded no grounds for
the recognition of a substantive defence based on equity, m modern South African law
5 In Barkhuizen v Napier 2007 (5) SA 323 (CC), the majority held that, while public policy endorses
freedom and sanctity of contract (pacta sunt servanda), it would also preclude the enforcement of a
contractual term in circumstances where such enforcement would be unjust or unreasonable

QUESTION 7
Which statement relating to the Bill of Rights in the Constitution of the Republic of South Africa,
1996, is CORRECT?

1 Vertical application relates to relationships between private persons, as in most contractual


situations
2 Horizontal applications relates to relationships between the state and the individual
3 In Barkhwzen v Nap1er 2007 (5) SA 323 (CC), the majority in the Constitutional Court ruled that a
contractual term can be tested directly against a provision in the Bill of Rights
4 In Barkhwzen v Napier 2007 (5) SA 323 (CC), the minority in the Constitutional Court preferred an
indirect application of the Const1tut1on to the contractual dispute before them
5 In Barkhwzen v Napier 2007 (5) SA 323 (CC), Ngcobo J reasoned that the proper approach to
constitutional challenges to contractual terms, is to determine whether the term challenged is
contrary to public policy, and what constitutes public policy must be discerned with reference to
the fundamental values embodied in the Constitution.

QUESTION 8
Thandi writes a letter to John offering him R35 000 for h1s car One of Thandi's employees delivers
the letter by hand to John on Monday John posts a letter of acceptance on the same day to Thandi.
Thandi receives and reads John's letter of acceptance on Friday. However, on Saturday, Sam offers
John R40 000 for the same car John accepts Sam's offer for the purchase of his car, and John
immediately phones Thandi to inform her that he rejects her offer. Which theory applies to
determine if a valid contract has been concluded between Thandi and John?

1 The information theory


2 The expedition theory
3 The reception theory
4 The declaration theory
5 The reliance theory

QUESTION 9
Assume the same facts in question (8). Which statement IS CORRECT?

1 Two valid contracts were concluded and therefore only the later agreement between Sam and
John is enforceable
2 John rightly has the discretion to elect the higher offer of R40 000 from Sam, because there are two
contracts concluded
3 Neither of the two valid contracts is enforceable in order to avoid prejudicing all of the contracting
parties
4 An enforceable contract ex1sts between Thand1 and John, as there was a valid offer and a valid
acceptance of the offer
5 No valid contract between Sam and John arose, because a valid prior contract between Thandi and
John already existed

QUESTION 10
Assume the same facts as in question (8) Should Thandi and Sam try to enforce their respective
agreements with John, and claim delivery of John's car, the court order sought amounts to an order
for
1 restitution
2 specific performance
3 rectification
4 general damages
5 special damages

QUESTION 11
Thandi offers to buy John's car for R35 000 by email and John accepts the offer by email Thandi lives
in Cape Town and John in Johannesburg. Which theory is applicable to determine where the
contract was concluded?

1 The information theory


2 The expedition theory
3 The reception theory
4 The declaration theory
5 The reliance theory

QUESTION 12
Carol, an owner of an exclusive bicycle shop advertised a special limited edition bicycle for sale, and
Invited the public to make offers for the bicycle Jane and Portia were among many other people who
submitted written offers for the bicycle Jane's offer was for R150 000, and Portia's offer was for R160
000 Although Carol intended to accept Portia's offer, she erroneously accepted Jane's offer Carol's
mistake IS
(a) not material
(b) an error m persona
(c) an error in motive
(d) material

1 (a)
2 (b)
3 (c)
4 (b) and (d)
5 (a), (b) and (c)

QUESTION 13
Assume the same facts as m question (12) and assume that Carol's error was material. Which
statement(s) is/are CORRECT?
(a) Carol's error was a iustus error
(b) Carol's error was not a iustus error
(c) Carol misrepresented her intention to Jane by accepting Jane's offer
(d) Carol by accepting Jane's offer, led Jane to reasonably believe that they have reached consensus

1 (a)
2 (b)
3 (c) and (d)
4 (b) and (c)
5 (b), (c) and (d)

QUESTION 14
Assume the same facts as in question (12). Which case has similar facts?

1 National Overseas Distributors Corporat1on {Ply) Ltd v Potato Board 1958 (2) SA 473 (A)
2 Brink v Humphries & Jewell (Pty) Ltd 2005 (2) SA 419 (SCA)
3 George v Fairmead Hotel (Pty) Ltd 1958 (2) SA 465 (A)
4 Du Toit v Atkinson's Motors Bpk 1985 (2) SA 893 (A)
5 Allen v Sixteen Stirling Investments (Pty) Ltd 1974 (4) SA 164 (D)

QUESTION 15
Gary points a loaded gun at Pete, and orders him to sign a written contract Gary explains that the
document is for the sale of Pete's car to him (Gary), at a price of R50 000 which is far below the
market value of the car Pete, fearing for his life, signs the document To establish a cause of action,
which of the following IS NOT necessary for Pete to prove?

1 That Pete had a reasonable fear


2 That the threat weakened Pete's power of resistance and rendered his will compliant
3 That there was a threat of an imminent evil
4 That the threat was contra bonos mores
5 That the pressure Gary used caused damage
QUESTION 16

Assume the same facts as in question (15). If Pete proves the relevant cause of action , the contract
between Gary and Pete is

1 void at the Instance of Pete


2 void at the instance of both Gary and Pete
3 voidable at the instance of Pete
4 voidable at the instance of both Gary and Pete
5 neither void nor voidable, but unenforceable

QUESTION 17
A term implied ex lege

1 is imported into a contract by law and operates, unless the parties exclude it
2 is imported into a contract by the parties and operates, unless the law operates to exclude it
3 is derived from the common law or from statute
4 is a tacit term
5 option 1 and 3

QUESTION 18
Joanne and Zack conclude a contract, in which Joanne purchases Zack's car for R40 000 Prior to the
delivery of the car to Joanne, Zack receives a higher offer from Craig, which he (Zack) accepts Zack
delivers the car to Craig. Which form of breach of contract did Zack commit?

1 Mora debitoris
2 Prevention of performance
3 Mora creditoris
4 Positive malperformance
5 Mora ex re

QUESTION 19
Assume the same facts as in question (18). The market value of the car is R50 000 Joanne claims R 10
000 damages from Zack. This is a claim

1 for general damages


2 for special damages
3 for negative interest damages
4 relating to the application of the convention principle
5 option 2 and 4

QUESTION 20
Assume the same facts in question (18) and (19) Which of the following Will Joanne NOT have to
prove in pursuit of her claim for damages?
1 That Zach committed a breach of contract
2 That Joanne has suffered financial or patrimonial loss
3 That there was a factual causal link between the breach and the loss
4 That the breach of contract was material
5 That the loss was not too remote as a consequence of the breach (that legal causation was present)

QUESTION 21
X has a watch that Y likes X and Y agree that if X ever sells her watch, she will offer to sell it first to Y
for R5 000 This IS a case of

1 a pre-emption formed unilaterally


2 an option formed bilaterally
3 an option formed unilaterally
4 both an option and a pre-emption
5 a pre-emption formed bilaterally

QUESTION 22
X has a watch that Y likes X offers to sell her watch to Y for R1 000, and X and Y agree that X's offer
will be open for acceptance until 1 June. This IS a case of

1 a pre-emption formed unilaterally


2 an option formed bilaterally
3 an option formed unilaterally
4 neither an option, nor a pre-emption
5 a pre-emption formed bilaterally

QUESTION 23
X has a watch that Y likes X offers to sell her watch to Y for R1 000, and Y accepts this offer X and Y
agree that they will reduce their contract to writing and that they will both sign it ("the writing
clause")
The parties reduced their contract to writing, but failed to sign it. Which statement(s) is/are
CORRECT?

1 The law presumes that the X and Y intended the writing clause to facilitate proof of the terms of
their contract, because there is doubt about what the Intended purpose of X and Y was with the
inclusion of the writing clause in their contract
2 There is no binding contract because the formalities created by X and Y were not compiled with
3 The writing clause also constitutes a non-variation clause
4 A binding contract was concluded even though X and Y only partially complied with the writing
clause
5 Option 1 and 4

QUESTION 24
Mark is engaged to Jane Mark has a very strong personality and eventually persuades Jane to sell and
transfer her house that is worth R900 000 to him at a purchase price of a mere R20 000. After
registration of the property in Mark's name he breaks off the engagement. Which of the following
requirements does Jane have to prove in terms of the positive law, in her pursuit to have the transfer
of the house into Mark's name set aside?
(a) Mark exercised an Influence over her
(b) Mark exercised this Influence over her, in an unscrupulous manner m order to induce her to
consent to a transaction which is to her detriment and which she, with normal free will, would not
have concluded
(c) Mark gained this influence by standing in a position of trust in relation to her
(d) This Influence exercised by Mark over her, amounted to intimidation which was
(a) and (d)
2 (a) and (b)
3 (c) and (d)
4 (a), (b) and (c)
5 (a), (b), (c) and (d)

QUESTION 25
X and Y conclude a contract on 1 June in terms of which X undertakes to deliver 10 dozen eggs at
12h00 on 10 June to Y, at her house. When X arrives at Y's house on 10 June at 12h00 with the 10
dozen eggs there is no one at the house to receive the eggs This is a case of

1 repudiation
2 prevent1on of performance
3 mora creditoris
4 mora debitoris
5 positive malperformance

QUESTION 1
STATE the instances when a term is considered unfair, unjust or unreasonable, with reference to the
Consumer Protection Act 68 of 2008. (5)

Answer
Hutchison & Pretorius (eds) The Law of Contract in South Africa 2nd ed (2012) p34:

A term is unfair, unreasonable or unjust if: it is excessively one-sided in favour of the supplier; or it is
so adverse to the consumer as to be inequitable; or it was induced by a supplier’s false, misleading or
deceptive misrepresentation; or the existence, nature and effect of the term was not adequately
drawn to the attention of the consumer in a clear and conspicuous manner before the transaction
was entered into.
Marks should also be awarded to students who refer to unfair, unjust and unreasonable terms that
are prohibited outright (1 mark per correct instance). These include: terms aimed at defeating the
purposes and policy of the Act, or misleading the consumer, or subjecting the consumer to
fraudulent conduct; terms that purport to waive or deprive a consumer of rights under the Act, or to
avoid a supplier’s obligations under the Act; a term that purports to limit or exclude the liability of a
supplier (or those for whom he or she is responsible) for harm caused by gross negligence; and a
term that falsely expresses an acknowledgement by the consumer that no warranties or
misrepresentations were made in connection with the agreement.

QUESTION 2
Briefly discuss common mistake. (5)

Answer
Hutchison & Pretorius (eds) The Law of Contract in South Africa 2nd ed (2012) p108:
3.8 Common mistake
A common mistake differs fundamentally from unilateral or mutual mistake, in that it does not lead
to dissensus; it nonetheless results in a contract being void. Therefore, common error is not a true
case of mistake and the principles discussed above do not apply. In such instances, both parties to a
contract make the same mistake, with the important proviso that the mistake does not relate to the
intention of either party. In fact the parties are in complete agreement (consensus ad idem) and each
knows the intention of the other and accepts it. However, both are mistaken about some underlying,
fundamental fact, in that they have a mistaken assumption about a present or past fact.
Common error results in the contract being void. For example, in Dickinson Motors (Pty) Ltd v
Oberholzer, the plaintiff’s son purchased two Plymouth motor cars on hire purchase from two
different parties – car A from the defendant and car B from a third party. After exchanging car B for
his father’s Hudson motor car, the son disappeared leaving unpaid amounts on both vehicles. The
defendant obtained judgment against the son in respect of car A and then had the car in the
plaintiff’s (the father’s) possession attached to satisfy the debt under the mistaken impression that it
was car A, whereas it was in fact car B. The plaintiff while under the same mistaken belief that the
car in his possession was car A, paid the outstanding debt to the defendant to retain possession of
the vehicle. Shortly thereafter, this same car was attached, but this time legally at the instance of the
third party who had sold the vehicle to the son. The plaintiff claimed return of the amount which he
had paid to the defendant. He succeeded and the Appellate Division held that the contract between
him and the defendant was void, because their contract was based on a common mistake.
There are several views as to why common mistake renders the contract void. Probably the most
accepted explanation is the ‘implied term’ theory, which in general holds that the particular
circumstances may justify the inference that the parties implicitly (tacitly) agreed to make the
existence of their contract dependent on the truth of a material fact, or rather a supposition. A
supposition or assumption occurs where the parties render their contract dependent on the
presence (or absence) of some fact of the past or present. If the fact is present, there is a binding and
enforceable contract; if not, there is no binding contract and the parties may reclaim whatever they
have performed (restitution).

A common error must amount to a term of the contract.


It is important to note that for a common error to have an effect on a contract, it must count as a
term of the contract, either expressly or by implication (tacitly). If a common error is not a term of a
contract, it will usually amount to little more than an error in motive, albeit a common error in
motive. It will then have no bearing on the validity of the contract. In Dickinson Motors (Pty) Ltd v
Oberholzer, the court held that the common mistake must have been ‘vital to the transaction’, in the
sense that neither party would have agreed to the contract if they had known the true situation.
Presumably, if the common mistake is fundamental in such a manner, the supposition would qualify
as a tacit term.
10.3.3.2 Tacit terms
A tacit term is one that the parties did not specifically agree upon, but which (without anything being
said) both or all of them expected to form part of their (oral or written) agreement. It is a wordless
understanding, an unarticulated term, having the same legal effect as an express term.
Tacit terms are established by considering the express terms and the circumstances that surrounded
the genesis of the contract. Some cases have suggested that the court may also have regard to the
subsequent conduct of the parties.
In ascertaining whether a contract contains a tacit term, the courts often employ the officious
bystander test. The court supposes that an impartial bystander had been present when the parties
concluded their agreement and had asked the parties what would happen in a situation which they
did not foresee and for which their express agreement consequently did not provide. If they were to
agree that the answer to the stranger’s question was self-evident, they are taken to have meant to
incorporate the term in their contract and to have tacitly agreed on it. This applies not only to terms
that the parties thought about, but found unnecessary to incorporate into the contract, but also to
terms that they would have incorporated if they had thought about them. A tacit term may therefore
express not only the parties’ actual intention but also what they may be presumed to have intended
– that is, an imputed intention.
In deciding whether a tacit term is to be imported into a contract, two things are necessary. Firstly, it
is obvious that the term sought to be imported must not conflict with an unambiguous express term
of the agreement: a tacit term after all only supplements the contract by providing a term that the
parties failed to agree upon expressly.
That a tacit term can only be imported into a contract if it is necessary in a business sense to give
efficacy to the contract was decided by Scrutton LJ in Reigate v Union Manufacturing (Ramsbottom).
This business efficacy test has been consistently applied by our courts. The implication must be
necessary, and not merely reasonable or convenient and much will depend on the express terms of
the agreement and the surrounding circumstances at the time it was entered into.

10.3.7.1 Suppositions
Parties may decide to conclude a contract on the supposition or assumption that a certain state of
affairs exists or existed. Thus, a second-hand copy of the third edition of a text book might be offered
for sale. A student, who needs a copy for one of his or her courses, is interested in purchasing the
book, but only wants to buy it if it is the latest edition. If neither the student nor the seller knows
whether it is the latest edition, they may contract on the basis that it is indeed the newest edition; if
it turns out that there is a later edition, the supposition is false and the contract will be void; if, on
the other hand, it is the latest edition, the contract will be valid.
A supposition or assumption must be shared by both parties in order to form the basis for the
contract. In other words, it must amount to a term of the contract. If only one of the parties has the
supposition in mind, it will be legally irrelevant and tantamount to an error in motive.
At first sight, a supposition seems similar to a condition. It is true that both may be either express or
tacit, and both make the effect of a contract depend on something that actually is uncertain. The
difference is that a condition relates to a future uncertain event, whereas a supposition relates to a
past or present state of affairs.
QUESTION 3
Discuss damages as a remedy for misrepresentation Do not discuss the Consumer Protection Act
68 of 2008 (10)

Answer

Hutchison & Pretorius (eds) The Law of Contract in South Africa 2nd ed (2012) p125-134:
4.2.3.3 Damages
Where a misrepresentation exists, whether the representee chooses to cancel or to abide by the
contract, he or she may in addition be entitled to recover damages in respect of any patrimonial loss
caused by the misrepresentation, depending on the state of mind with which the representation was
made. In this regard, the distinction between fraudulent, negligent and innocent misrepresentation is
relevant.
1. Fraudulent misrepresentation. Deliberate deception that causes another financial harm is a
delict in our law, corresponding to the English tort of deceit, and is actionable under the actio legis
Aquiliae. Thus a representee who has been tricked into concluding a prejudicial contract is entitled
in principle to damages sounding in delict. The five essential elements of the cause of action are as
follows:
1. a representation;
2. which is, to the knowledge of the representor, false;
3. which the representor intended the representee to act upon;
4. which induced the representee to act; and
5. that the representee suffered damage as a result.
The motive of the representor is irrelevant; provided only that he or she made the assertion without
an honest belief in its truth and intended it to be acted upon, it matters not that he or she lacked an
intention to cause loss or damage to the representee; but such damage must have followed as a
result of the representee acting upon the misrepresentation.
Particularly in the case of an implied representation, if the representor was unaware of the import of
his or her representation, he or she cannot have intended it to be acted upon. However, such
intention need not always be dolus directus – that is, the representor need not have applied his or
her will to induce the representee to act upon the representation. It is sufficient if he or she
subjectively foresaw such a result and was reckless as to whether the result followed or not.
The right to claim damages for fraud is not dependent on the materiality of the representation; in
other words, it is no defence that the representee should not, as a reasonable person, have been
deceived so easily. Ex hypothesi, the representor intended to deceive the representee, and it does
not lie in his or her mouth to say that the latter ought not to have been so gullible.
An agreement that purports to exclude the right to claim redress for fraud is contrary to public policy
and hence unenforceable.
Since fraud is a delict, the measure of damages is the usual delictual rather than the contractual
measure. The fundamental difference between the two measures was explained by Van den Heever
JA in Trotman v Edwick:
A litigant who sues on contract sues to have his bargain or its equivalent in money or money and
kind. The litigant who sues on delict sues to recover the loss which he has sustained because of the
wrongful conduct of another, in other words that the amount by which his patrimony has been
diminished by such conduct should be restored to him.
The victim of fraudulent misrepresentation is thus entitled to be put in the financial position he or
she would have occupied had the representation not been made to him or her, but he or she cannot
have the representation ‘made good’ by being put in the position he or she would have occupied had
the representation been true, since that is the contractual measure, applicable to breach of
warranty.
This principle is easily stated, but its application in practice to the facts of a particular case of fraud
has often caused difficulty, especially where the contract is upheld. As a result, there is a strong
tendency to hold that the method of assessment to be employed depends on the particular facts of
the case in hand. Nevertheless, in spite of this rather pragmatic approach, a number of guiding
principles may be gleaned from the decided cases and the academic literature.
Whether the contract is rescinded or upheld, damages are recoverable in respect of the
consequential losses flowing from the fraud, provided that they are not too remote (for example,
the reduction in the value of the herd, where the representee has been deceived into buying a beast
with an infectious disease). Where the contract is rescinded and restitution ordered, the
representee’s loss on the transaction itself is generally wiped out by the process of restitution and his
or her damages are thus usually limited to wasted costs and other such consequential losses. Where
the contract is upheld, on the other hand, the representee may suffer a loss on the transaction itself
(for example, by paying more for a thing than it is worth), and it is the assessment of such intrinsic
loss that presents the most difficulty. In this regard, the distinction between dolus dans and dolus
incidens is once again of importance.
Since to be compensable at all, the loss in question must be causally connected with the fraud, care
must be taken to determine the precise effect of the fraud on the conclusion of the contract. In a
case of dolus dans, there would have been no contract at all but for the fraud. To remove the effect
of the delict, therefore, the representee should be placed in the financial position he or she would
have occupied had he or she not contracted. In other words, he or she should be given the financial
equivalent of rescission and restitution, as compensation for his or her net loss on the overall
transaction. This entails awarding the innocent party the value of his or her performance, less any
benefits which he or she has received from the other party under the contract. In a case of sale, for
example, his or her net loss is the price paid for the merx, less its actual or fair value at the time of
purchase. Both losses and benefits have to be taken into account, since both flow directly from the
misrepresentation; thus if, in spite of the misrepresentation, the representee has made an overall
profit on the transaction, he or she is not entitled to any damages – the so-called swings-and-
roundabouts principle. For example, if he or she has paid R800 000 for a house worth R900 000, and
he or she would not have bought at all but for a misrepresentation that the house had recently been
rewired, he or she cannot recover the R20 000 that it has cost to rewire the house, since what he or
she loses on the swings (R20 000) is more than compensated by what he or she gains on the
roundabouts (R100 000).
In a case of dolus incidens, on the other hand, where the effect of fraud was merely to influence the
terms of a contract which would in any event have been concluded, the representee’s loss cannot be
measured by comparing the values of the respective performances of the parties, since neither was
fully induced by the representation. Rather, one should measure the extent to which the
representation inflated the performance that the representee was prepared to make under the
contract. In other words, if the representee is a purchaser, his or her loss is the difference between
the price he or she actually paid and the price that he or she would have paid but for the
misrepresentation (the putative price).
Take the case of a bad bargainer. Suppose A offers to buy something worth R100 from B for a price of
R120; B makes a misrepresentation which, if true, would make the thing worth R150; A then buys it
for R180. The loss caused by the misrepresentation is not R80 (price paid less actual value) but R60
(price paid less price that would otherwise have been paid); the additional loss of R20 (putative price
less actual value) would in any event have been incurred due to A’s bad bargaining.
In the case of a good bargainer, on the other hand, the damages awarded may exceed the difference
between the price paid and the actual value of the merx, since they include an element of loss of
profit (lucrum cessans). In the example given above, if A’s initial offer was less than the actual value
of the thing, say R90, and his or her final, accepted offer after the misrepresentation was R135, then
the representee can recover not just R35 (price paid less actual value) but R45: the additional R10
represents the profit that he would have made but for the misrepresentation. This is not the same as
awarding him or her contractual damages, which would here be R50 (difference between the
represented and the actual value of the merx: R150 less R100). Indeed, even if the representee has
made an overall profit on the transaction, in spite of the misrepresentation, he or she may recover
damages to the extent that he or she has paid more for the thing than he or she would otherwise
have done, since he or she is financially worse off as a result of the fraud: but for the
misrepresentation his or her profit would have been even greater. One cannot set off against this
loss of profit the gain that the representee makes on the transaction, since that gain stems not from
the misrepresentation but from the conclusion of the contract (ex hypothesi the contract was not
induced by the misrepresentation). In a dolus incidens situation, therefore, the swings-and-
roundabouts principle is inapplicable.
Naturally, it will often be difficult to establish the terms on which the parties would have agreed, had
the misrepresentation not been made – particularly if there was no prior offer, as will usually be the
case. Strictly speaking, it is not sufficient simply to ask what the representee would have offered had
the misrepresentation not been made; one should also enquire whether the representor would have
accepted such lower offer, for if he or she would not have done so, no contract would have been
concluded: the dolus dans situation. However, this is taking logic a bit far, and the courts have
understandably baulked at saddling the innocent representee with the heavy onus of ‘proving ex
post facto, when the parties are at daggers drawn in litigation, the subjective state of mind of the
wrongdoer at the time of the contract’. It has accordingly been held that in the ordinary case, the
representee need merely establish the amount that he or she would have offered, but for the
misrepresentation; only if the representor then shows that such offer is lower than the market value
of the merx, to use the example of sale, will it be necessary for the representee to prove that his or
her lower offer would probably have been accepted. On the other hand, the courts will not simply
accept the ipse dixit of the representee as to what he or she would otherwise have offered. In the
absence of satisfactory evidence to the contrary, the courts will assume that the price actually paid
reflected the value of the thing in its represented condition, and therefore that, had the
misrepresentation not been made, the merx would have been sold at its true or market value. On
this assumption, that the parties bargained as reasonable persons, the measure of damages for dolus
incidens is the same as for dolus dans: price paid less actual value.

Do the courts merely pay lip service to these principles?


Although the foregoing principles are not hard-and-fast rules, they have attracted a wide measure of
judicial support. However, it has been said that, in attempting to apply them, the courts have often in
effect awarded the contractual measure of damages – a charge that cannot simply be dismissed with
the trite observation that, in a particular case, the delictual and contractual measures may yield the
same quantum of damages. Even when the courts adopt a different method of assessment, they
appear at times to fall into the same trap. In Ranger’s case, for example, it was held that where a
thing is misrepresented to be free from defects, the damages may be measured by the cost of
removing the defect. With respect, it is submitted that, despite the obvious practicality of such an
approach, it amounts to making the representation good; that is the contractual measure.
2. Negligent misrepresentation. Until the decision in Bayer South Africa (Pty) Ltd v Frost, there
remained some residual doubt as to whether delictual damages might be awarded in respect of a
negligent misrepresentation inducing a contract. The issue was but one aspect of a much wider
problem – namely, the actionability at all in delict of a negligent misstatement causing purely
economic or financial loss (as opposed to a negligent act causing injury to a person or property), but
with the added complication in this case, that the misstatement is made in a contractual context.
For many years, our courts were reluctant to grant delictual damages for a negligent misstatement
causing economic loss. To a large extent, this reluctance stemmed from a fear that the recognition of
such an action might result in ‘liability in an indeterminate amount for an indeterminate time to an
indeterminate class’. It was also strongly influenced by the decision of the House of Lords in Derry v
Peek, which was generally interpreted as ruling out damages in tort for a merely negligent, as
opposed to a fraudulent, statement. However, after English law dramatically changed course in
Hedley Byrne & Co Ltd v Heller and Partners Ltd, our law began to follow suit and in Administrateur,
Natal v Trust Bank van Afrika Bpk, the actionability in delict of a negligent misstatement causing
purely economic loss was at last recognised in unequivocal terms, subject to the usual elements of
Aquilian liability being satisfied.
With the broader question settled, the focus of attention shifted to the wrongfulness or otherwise of
a negligent misrepresentation inducing a contract. In Administrateur, Natal, that issue had been
deliberately left open, but the Appellate Division had earlier, in Hamman v Moolman, made its
attitude on the point quite plain:
The existing law grants what appears to be adequate protection in the field of contract to a party to
whom a misrepresentation is made. Thus a contracting party may safeguard himself against loss by
simply taking the elementary precaution of requiring the representor to guarantee the truth of his
representations. Adequate remedies are available where the misrepresentations are tainted with
dolus, and in appropriate circumstances an aggrieved party is granted relief in the case of an
innocent misrepresentation. … I am by no means satisfied that the practical necessity of a remedy of
the kind contended for has been demonstrated, nor that its recognition might not result in more ills
than the one it is intended to remedy, namely, the failure of the unwary representee to have proper
regard to his own interests in the field of contract.
These obiter dicta were severely criticised, and justifiably so. The existing remedies available to the
victim of a negligent misrepresentation could by no means be considered adequate: neither
rescission of the contract – a drastic step in itself – nor price reduction under the actio quanti
minoris, in the case of sale, are of much comfort to the representee who has suffered consequential
losses through reliance on the misrepresentation. Nor can one reasonably expect a contracting party
to insist that the other party guarantee every statement that he or she makes; if parties cannot rely
on statements made during the bargaining process, how can business be conducted in a spirit of
good faith? Of course, the law recognises that, in seeking the most advantageous deal possible,
contracting parties are prone to exaggeration and a certain looseness of language – hence the
immunity granted in respect of mere puffs. Some scepticism about pre-contractual statements is
thus called for; but it surely could not be the law that parties are free to pepper each other with
careless misstatements of fact.
For a while, the provincial courts reluctantly gave effect to the Hamman dicta. After the seminal
decision in Administrateur, Natal v Trust Bank van Afrika Bpk, however, the Cape Provincial Division
felt justified in departing from the Hamman line, holding that in certain circumstances a party
induced to contract by a negligent misrepresentation is entitled to delictual damages. The court was
fortified in its decision by the fact that in England the Court of Appeal had similarly extended the
Hedley Byrne doctrine into the pre-contractual sphere.
In 1992, this welcome development finally received the seal of approval from the Appellate Division
in the case of Bayer South Africa (Pty) Ltd v Frost. The court held that there was no good reason why
the general action for negligent misstatements recognised in the Trust Bank case should not be
applied in the pre-contractual sphere, to a negligent misrepresentation inducing a contract. This was
subject of course to all the requirements for Aquilian liability being satisfied, with particular
emphasis being placed on the elements of wrongfulness and causation affording the courts the
necessary means of controlling the scope of this new form of liability.
Since the new cause of action is delictual in character, the damages awarded for negligent
misrepresentation will be measured in the same manner as damages for fraud.
3. Innocent misrepresentation. A more difficult question is whether damages of some sort may
be recovered in respect of wholly innocent misrepresentations. That damages in the true sense of
the word cannot be recovered seems certain. Ex hypothesi, the misrepresentation is not a term of
the contract and is made without dolus or culpa, there is neither the breach of contract necessary for
contractual damages nor the culpable conduct required to sustain an Aquilian action for delictual
damages. Consequential losses suffered in reliance on an innocent misrepresentation are thus not
compensable, and must lie where they fall.
What remains to be settled, however, is whether a party induced to contract by an innocent
misrepresentation is entitled, instead of rescinding the contract, to abide by it and claim the financial
equivalent of rescission and restitution: ‘restitutional damages’, ‘partial restitution’, ‘reduction of
performance’ or ‘quanti minoris type damages’, as such relief has at various times been called.
In the special case of sale, as we shall see, such relief is now sometimes available under the actio
quanti minoris. However, outside the aedilitian context, the position is less clear. There are many
dicta denying damages for innocent misrepresentation. On the other hand, Maasdorp CJ took the
opposite view in Brink v Robinson:
Why should [the representee] not get damages, even if the representation was innocently made? He
has nothing to do with the representation. He asks for information, and he gets the
misrepresentation. It does not matter to him whether it was innocently made or not; and I fail to see
why he should be made to suffer damages for that.
The problem with almost all of these dicta is that they do not make it clear whether the damages to
which they refer are damages proper (compensatory damages) or merely restitutional damages. If
Maasdorp CJ had in mind an award of compensatory damages, his statement goes too far; but by
the same token most of the statements against allowing damages are open to the interpretation that
they refer only to damages proper, and not to restitutional damages.

Should restitutional damages be awarded for innocent misrepresentation?


In English law, the courts now have the power to award restitutional damages ‘in lieu of rescission’
for an innocent misrepresentation, under s 2(2) of the Misrepresentation Act of 1967, and calls for
similar relief have been made by several South African writers. Finding a doctrinal basis for such
relief presents a problem, however. Some writers have suggested basing it on enrichment, but that
cannot be done without undermining the requirement that the enrichment should be legally
unjustified or sine causa. Equally unconvincing is the argument that the making of a
misrepresentation constitutes a hazardous activity which so appreciably increases the risk of harm to
the representee that it justifies the imposition of strict delictual liability.
The best approach, it is submitted, is the non-dogmatic one favoured by many of the writers cited
above, namely that the right to restitutional damages (or ‘partial restitution’, or ‘reduction of
performance’) should be recognised as a logical and equitable extension of the existing right to
rescind a contract induced by an innocent misrepresentation. Having recognised the drastic right of
rescission on grounds of equity, the courts should on the same basis allow the representee to opt
instead for the less extreme course of affirming the contract and claiming the judicial equivalent of
restitutio in integrum. Otherwise, the representee may be driven to set aside an entire transaction in
order to obtain redress for a relatively minor, but none the less prejudicial misrepresentation, which
is surely not in the interests of either party.
The considerations of equity that have induced the courts to recognise a right of rescission apply
equally strongly to the remedy of restitutional damages. While it is of course true that the
representor has behaved innocently, so too has the representee; why should he have to suffer a loss
to the corresponding advantage of the representor? To order restitution would not punish the
representor or even make one of two innocent parties bear a loss, but would simply restore an
economic balance that has been disturbed. If it is morally wrong for a person to resist rescission of a
contract induced by his or her own innocent misrepresentation, it is surely equally so for him or her
to retain a benefit obtained in the same way.
The case for allowing restitutional damages is strengthened by the decision in Phame (Pty) Ltd v
Paizes. As we have seen, the Appellate Division decided in that case that a purchaser who has been
induced to contract by a seller’s unfounded dictum et promissum may invoke the aedilitian remedies:
if he or she would not otherwise have bought at all, he or she may cancel the contract with the actio
redhibitoria – a largely superfluous remedy, in view of the right to rescind for misrepresentation.
More significantly, the purchaser may instead choose to abide by the contract and claim a reduction
of the purchase price with the actio quanti minoris. Since a dictum et promissum (as defined in
Phame) is essentially a (positive) misrepresentation as to the quality of the merx and is actionable
even if made without fraud or negligence, the purchaser is in effect afforded a remedy in ‘damages’
for innocent misrepresentation. Indeed, it is when the dictum et promissum constitutes an innocent
misrepresentation that the purchaser is most likely to invoke the actio quanti minoris: if the
statement was warranted to be correct, he or she will surely sue in contract, and if it was made
fraudulently or even negligently, he or she is more likely to sue in delict, since he or she will then be
able to recover consequential losses as well.
The relief obtainable under the actio quanti minoris is limited to a reduction of the price; in the
absence of fraud or a warranty, the purchaser is not entitled to consequential damages, except
perhaps when the seller is a manufacturer or a merchant seller who publicly professes to have
attributes of skill and expert knowledge in relation to the kind of goods sold. The measure of
reduction is not the difference between the agreed price and the price that would have been paid
but for the misstatement, as in the cases of dolus incidens, but rather the difference between the
price paid and the actual value of the merx. Thus where the purchaser would in any event have paid
more for the merx than it was worth, it will suit him or her better to allege an innocent dictum et
promissum than a fraudulent misrepresentation – a result rightly considered ‘a little strange’.
The aedilitian actions apply only to the contract of sale (and perhaps exchange) and only to
misrepresentations that fall within the concept of a dictum et promissum. Thus the statement must
be made by a seller, and must concern the quality of the merx. It follows that the actio quanti
minoris can hardly serve as a general remedy for innocent misrepresentation. Nevertheless, the
recognition of a right to restitutional damages on the part of the purchaser raises the question why
other representees should not have a similar right. Apart from the historical one, what special factors
in the contract of sale justify this preferential treatment of the purchaser? It is noteworthy that the
court in Phame was clearly influenced by equitable considerations in effectively extending the scope
of aedilitian relief, though it based its decision on old authority, and expressly made no comment on
the alternative claim for the same sum as damages for innocent misrepresentation.

QUESTION 4 I VRAAG 4
Andy and Craig conclude a contract wherein Andy agrees to paint Craig's office block by 31 August,
and Craig agrees to pay Andy R10 000 upon completion of the work When 80% of the work IS
completed Andy suddenly decides to go on vacat1on for a week and he is consequently unable to
complete the Job by 31 August Craig validly cancels the contract because the work is not completed
on time and refuses to pay Andy any money for his (Andy's) services rendered Craig hires another
contractor at an amount of R3 000 to complete the job Craig does not incur any other costs to
complete the job, neither does his business make any losses. Advise Andy as to what amount (if any)
he may recover from Craig for the services that he rendered and on what basis. Discuss with
reference to BK Tooling (Edms) Bpk v Scope Precision Engineering (Edms) Bpk 1979 (1) SA 391 (A) and
other relevant case law Do not apply the Consumer Protection Act 68 of 2008 (15)

Answer
Identifying the problem (maximum of 3 marks)
This problem deals with the enrichment action of Andy

Defective performance and cancellation of the agreement. In the BK Tooling case, the Appellate
Division stated that one must carefully distinguish between circumstances where the contract is
cancelled due to a breach, and situations where the contract is kept alive and the exceptio non
adimpleti contractus is raised to fend off any claim for performance. In the case where the contract is
cancelled for material breach, the exceptio cannot be raised as a defence, as it is aimed at fulfilment
of the contract. Where the contract has been cancelled, a claim for performance will be resisted not
with the exceptio, but with the defence that, since the contract has been terminated, the
performance is no longer owed.
Where the contract is cancelled, the obligations to perform are terminated and both parties are
obliged to return any performance received. These obligations are contractual in nature and do not
arise from unjustified enrichment.
In certain circumstances, however, it may not be possible for a party to return the performance
received (for instance, in the case of services rendered or where property has become the property
of a third party through adhesion). Where the contract has been lawfully cancelled, the innocent
party who has received some value from the cancelled contract, then becomes liable to the party in
breach on the grounds of unjustified enrichment. It can be said that, after cancellation, the innocent
party has become enriched without a proper underlying cause and he or she therefore becomes
liable to the other party to the extent that he or she remains enriched at that time. It is said that the
enrichment is unjustified because the transfer of value is without legal cause – the original cause for
the transfer, the contract, has been cancelled and can no longer serve to justify the transfer of value.
This applies equally to the guilty party where the latter has received performance.
In terms of the enrichment claim, the impoverished party is entitled to claim the lesser of its own
expenditure – that is, the cost of performance to it – and the amount by which the enriched party
remains enriched at that time.

Application of the law to the facts (maximum of 3 marks)

Craig validly cancelled the contract, because of Andy's positive malperformance. The parties now
have to return the performance of whatever they have received from the other party. Craig cannot
return the paintwork done by Andy. Craig is thus enriched at Andy's expense. Andy will thus be able
to claim his expenses (the amount he has been impoverished) or the amount Craig has been
enriched, whatever is the least. The problem does not indicate what these two amounts are.

ALTERNATIVE ANSWER

Identifying the problem (maximum of 3 marks)

This question deals with the exceptio non adimpleti contractus.

This contract is reciprocal in nature. Andy’s has rendered defective performance and the issue is
whether Craig has to compensate Andy for the work that has already been done.

Discussion of law (maximum of 7 marks)

The exceptio non adimpleti contractus is a defence which may be raised in the case of reciprocal
contracts, that is contracts in which the parties create obligations in terms of which the parties must
either perform simultaneously or the one must perform before the other. A building contract is a
reciprocal contract and the building contractor must perform first.

Where a party is sued for performance he (the defendant) may withhold performance until the
claimant has tendered proper performance or has performed fully, provided the claimant has to
perform first or simultaneously, by raising the defence of the exceptio. The right to withhold
performance is referred to as the exceptio non adimpleti contractus.

To allow a defendant who has accepted and is using the plaintiff's performance (either because the
defect is not sufficiently serious to justify rejection of the performance and thus
rescission/cancellation, or because the defendant prefers to accept rather than reject the defective
performance) to raise the exceptio against the plaintiff's claim for counter performance might
operate extremely unfairly against the plaintiff. The defendant will have the (often considerable)
benefit of the plaintiff's performance while the plaintiff will receive nothing in return. On the other
hand, to order the defendant to render the full counter performance in return for the plaintiff's
defective performance will be unfair to the defendant.

This problem was determined as follows by the Appellate Division in BK Tooling v Scope Precision
Engineering 1979 (1) SA 391 (A):

Any contracting party has in principle a right to the specific performance undertaken by the other
party, that is he has the right, in principle, to enforce the contract strictly according to its terms. The
right of a party to a reciprocal contract to withhold his own performance until the other party
performs in full, is a powerful weapon to enforce full performance. In principle, therefore, a
defendant who has accepted the plaintiff's defective reciprocal performance is still entitled to raise
the exceptio non adimpleti contractus against the plaintiff's claim, even though the plaintiff's
performance is defective in minor aspects only. Where, however, fairness so requires, a court may, at
its discretion, refuse to allow a defendant to raise the exceptio and order him to render a reduced
counter performance.

The plaintiff in an action based on a reciprocal contract must, where he upholds the contract (ie
continues with the contract), have his pleadings in order and he must:

(1) allege in his particulars of claim that he has rendered full performance from his side or he must
tender full performance.

If he is unable to prove that he has indeed performed in full and wishes the court to exercise its
discretion in his favour by awarding him a reduced counter performance, he (the plaintiff) must
allege and prove:

(1) That the defendant is utilising the defective performance.

(2) That circumstances exist which render it fair (equitable) that the court should exercise its
discretion in his favour.

(3) By how much the counter-performance ought to be reduced. This would normally be by the
amount which it would cost the defendant to convert the plaintiff's defective or incomplete
performance into proper performance.

Marks should also be appropriately awarded to students who do not set out how the plaintiff’s case
should be pleaded, but nevertheless, discuss whether the defendant is utilising the defective
performance; the courts equitable discretion to consider awarding a reduced counter-performance
in the interests of fairness; and by how much the counter-performance ought to be reduced.

Application of the law to the facts (maximum of 3 marks)

The parties have concluded a building contract which is a reciprocal contract. Andy may therefore
not claim full performance from Craig as Craig may raise the exceptio.
Based on the ruling in the BK Tooling case, Andy is entitled to claim reduced performance from Craig
because:

(1) Craig is utilising the defective performance.


(2) It would be equitable as Andy has completed most of the work.
(3) The counter-performance ought to be reduced by R3000 (which is the amount it cost to
complete the job).

Giving appropriate advice (maximum of 2 marks)

Andy may claim a reduced performance of R7000 from Craig.

Extra marks:

Students should also be awarded up to a maximum of 2 marks where they state that Andy has
committed breach of contract in the form of positive malperformance and then explain positive
malperformance. Positive malperformance occurs when a party performs but his performance is
incomplete or defective. Andy has a single obligation, which he does not perform fully.

Below is the relevant pages (p316-p321) from the prescribed textbook (relevant for this question),
including a brief summary of BK Tooling (Edms) Bpk v Scope Precision Engineering (Edms) Bpk 1979
(1) SA 391 (A):

Exceptio non adimpleti contractus


The exceptio non adimpleti contractus is a defence that can be raised in the case of a reciprocal
contract, where the performances due on either side are promised in exchange for one another. It is
a remedy that permits a party to withhold his or her own performance, and to ward off a claim for
such performance, until such time as the other party has either performed or tendered proper
performance of his or her own obligations under the contract. In a sense, therefore, the exceptio is
aimed at securing specific performance from the other party.
Requirements for the exceptio non adimpleti contractus
The exceptio non adimpleti contractus is available when two requirements are met: the two
performances must be reciprocal to one another; and the other party must be obliged to perform
first, or at least simultaneously with the party raising the exceptio. The exceptio may also be used
where a party has performed incompletely.
1. Reciprocity of obligations. In the leading case of BK Tooling (Edms) Bpk v Scope Precision
Engineering (Edms) Bpk, the Appellate Division stated that reciprocal obligations are obligations that
have been created in exchange for each other. A contract that contains reciprocal obligations is also
called a reciprocal contract. The distinguishing factor of a reciprocal contract therefore, is that it is
essentially aimed at accomplishing an exchange of performances. The one performance must
therefore be given or promised in exchange for the other. Most contracts that create obligations for
both parties will be regarded as reciprocal. However, such contracts may have a number of different
obligations, not all of which are necessarily reciprocal. The difficulty then is to decide whether a
specific obligation within the contract can be regarded as reciprocal to a specific obligation of the
other party.
In order to determine whether an obligation is reciprocal or not, the (express or tacit)
intention of the parties must be determined by interpreting the agreement. Whether obligations are
reciprocal or not depends upon the intention of the parties, as reflected in the terms of the contract.
The question to be asked is the following: did the parties intend to create the obligations in exchange
for each other?
Reciprocal contracts are subject to the principle of reciprocity. In terms of this principle,
party A is not entitled to claim performance of a reciprocal obligation from party B where party A has
to perform his or her obligation first or simultaneously, unless A has already performed or is
tendering performance of his or her obligation. This also means that the party B is entitled to
withhold his or her performance until the counterparty (A) has performed in full or tenders full
performance. This brings us to the second issue to be determined – namely, what the sequence of
performance in terms of the contract is.
2. Sequence of performances. The general rule is that parties must perform pari passu
(simultaneously) unless (a) the parties have agreed otherwise, or (b) the naturalia of the contract in
question dictate otherwise. The following provide examples where the naturalia of certain types of
agreement may determine the sequence of performance:
Lease: The landlord must perform first by providing the use and enjoyment of the property to the
lessee. Payment of the rent is only owed at the end of the month or year.
Employment: The employee must perform first. The employer is only obliged to pay the salary or
wages at the end of the period.
Building contracts: The building contractor must perform first.
The parties are however free to change the sequence of performance as determined by the
naturalia in their agreement. The sequence of performance will therefore firstly be determined by
the intention of the parties and secondly by the naturalia of the contract if there is no specific
agreement. The intention of the parties is determined by the interpretation of their agreement.
The interpretation of the agreement is supported by two presumptions:
It is presumed that interdependent obligations are reciprocal unless there is evidence to the
contrary. Contracts such as sale, lease, mandate, insurance and a contract for work (locatio conductio
operis) provide examples of contracts containing reciprocal obligations.
There is a presumption that in any reciprocal contract the common intention is that neither party
shall be entitled to enforce performance unless that party has performed or is ready to perform –
that is, that performance should take place simultaneously. Of course, where there is a clear
indication that the one party has to perform first, the other party will be entitled to claim
performance before it has performed itself.
There is a clear exception to the second presumption in the case of contracts for work to be
performed (locatio conductio). The person who has to perform the work must perform first, unless
there is a clear indication in the contract to the contrary. Thus an agent, a builder or an employee
must perform first. Similarly, in the case of lease, the lessor must perform first.
If, at the time of the action, the plaintiff’s duty to perform is not yet due and enforceable,
the defendant cannot raise the exceptio non adimpleti contractus against the plaintiff. For example,
in the case of a cash sale, the buyer and seller must perform simultaneously and if the buyer should
institute action for delivery of the thing which he or she has bought without tendering payment, the
seller can raise the exceptio non adimpleti contractus – that is, he or she may resist the buyer’s claim
on the ground that the buyer has not made payment. In the case of a credit sale, however (that is,
where payment by the buyer has been deferred to some future date), the exceptio cannot be raised
against the buyer’s claim for delivery of the thing which he or she has bought, since payment is not
yet due.
Another exception to the general rule is the contract of lease, which is a reciprocal
agreement in terms of which one person, the lessor, undertakes to allow another, the lessee, the
enjoyment and use of a thing completely, or partially, temporarily, and for monetary compensation.
Where the contract of lease fails to stipulate a specific day for payment, the rent becomes due only
after expiry of the lease. The lessor must perform before the lessee is required to do so.
Where a party who must perform first or simultaneously claims specific performance from
the other party, it must allege in its pleadings that it has performed in full or that it is ready to do so.
If the plaintiff has not performed in full or is not in a position to do so, the defendant can raise the
defence of the exceptio non adimpleti contractus.
3. Incomplete performance. A defendant can rely on the exceptio non adimpleti contractus not
only where the other party has failed to perform at all (mora debitoris), but also in cases where the
other party has failed to perform properly, or in full (positive malperformance). The defective or
incomplete performance need not be so serious as to justify a cancellation of the contract; the
innocent party is permitted to withhold his or her own performance and to raise the exceptio even
where the defect or shortfall in the performance is relatively minor, provided only that it is not so
trivial as to fall within the principle de minimis lex non curat. This affords a powerful, extra-judicial
means of securing proper performance from the other party.
Factors affecting the application of the exceptio
Careful analysis is needed to determine whether the exceptio can be used in the particular
circumstances of any matter.
Acceptance of part-performance. The exceptio non adimpleti contractus is a remedy aimed at
ensuring full performance by the innocent party – that is, it is aimed at keeping the contract intact.
Where the innocent party has received part-performance or defective performance and has started
using the performance, even though it is aware of the defective performance, it is a strong indication
that the party has elected to keep the contract alive. Such a party who has elected to keep the
contract intact cannot thereafter cancel the contract, unless there is a further material breach of
contract. That party is thereafter restricted to relying on the remedies aimed at fulfilment and
damages. It is also entitled to raise the exceptio non adimpleti contractus as a defence.
Defective performance and cancellation of the agreement. In the BK Tooling case, the Appellate
Division stated that one must carefully distinguish between circumstances where the contract is
cancelled due to a breach, and situations where the contract is kept alive and the exceptio non
adimpleti contractus is raised to fend off any claim for performance. In the case where the contract is
cancelled for material breach, the exceptio cannot be raised as a defence, as it is aimed at fulfilment
of the contract. Where the contract has been cancelled, a claim for performance will be resisted not
with the exceptio, but with the defence that, since the contract has been terminated, the
performance is no longer owed.
Where the contract is cancelled, the obligations to perform are terminated and both parties are
obliged to return any performance received. These obligations are contractual in nature and do not
arise from unjustified enrichment.
In certain circumstances, however, it may not be possible for a party to return the performance
received (for instance, in the case of services rendered or where property has become the property
of a third party through adhesion). Where the contract has been lawfully cancelled, the innocent
party who has received some value from the cancelled contract, then becomes liable to the party in
breach on the grounds of unjustified enrichment. It can be said that, after cancellation, the innocent
party has become enriched without a proper underlying cause and he or she therefore becomes
liable to the other party to the extent that he or she remains enriched at that time. It is said that the
enrichment is unjustified because the transfer of value is without legal cause – the original cause for
the transfer, the contract, has been cancelled and can no longer serve to justify the transfer of value.
This applies equally to the guilty party where the latter has received performance.
In terms of the enrichment claim, the impoverished party is entitled to claim the lesser of its own
expenditure – that is, the cost of performance to it – and the amount by which the enriched party
remains enriched at that time.
Courts’ equitable discretion: reduced contract price
Where a party who has to perform first, has performed only part of its obligations or has rendered
defective performance, that party is in principle not entitled to claim counter-performance until such
time as he or she has performed in full. In practice, the innocent party often accepts part-
performance and starts utilising the performance. This sometimes leaves the party in breach in an
invidious position in that it may be impractical or impossible for the breaching party to make full
performance, but any claim for counter-performance can be defended by the other party relying on
the exceptio.
This is so patently unfair towards the party in breach that the courts, in a long line of cases, have
exercised a discretion to relax the principle of reciprocity and order the party making use of the
defective or incomplete performance to pay a reduced amount to the party in breach. The reduced
amount is usually determined by deducting from the contract price the cost of remedying the defect
or shortfall in the performance. In the BK Tooling case, the court reviewed the leading cases dealing
with this matter. The court stated firstly that a clear line should be drawn between, on the one hand,
cases where the contract had been cancelled and the subsequent claim was based on principles of
unjustified enrichment law, and, on the other hand, cases where the contract remained intact, but
had become difficult or impossible to fulfil.
In cases where the contract remains intact (for instance, where the innocent party has accepted and
started using defective performance), the Appellate Division held that courts have an equitable
discretion to award a reduced contract price, depending on the nature of the defect and the cost of
repair, replacement or substitute performance.
This remedy is contractual in nature and the price reduction is calculated on the basis of the contract
price. This must be clearly distinguished from the enrichment action discussed above, which is not
calculated on the basis of the contract price, but rather on the subjective enrichment and
impoverishment of the parties respectively. The onus to prove the amount of the reduction is on the
party in breach claiming the reduced contract price.
In the BK Tooling case, the one party had to manufacture certain moulds complying with very strict
specifications. The moulds were to be used in the manufacturing process of the other party. After
delivery of the moulds, it was discovered by this other party that the moulds did not comply with the
specifications. It then had the moulds modified by a third party. This, of course, made it impossible
for the original party to rectify the defect in performance. The original party’s subsequent claim for
payment was met by the exceptio non adimpleti contractus. The innocent party had clearly accepted
the defective performance and started using it, but without paying for the work done. The court held
that under these circumstances, the party in breach was entitled to a reduced contract price, the cost
of remedying the defects constituting an equitable reduction in the circumstances.
Scope of the exceptio
The right to withhold performance in terms of the principle of reciprocity is a temporary defence
aimed at obtaining full and proper performance before being obliged to render counter-
performance. The party relying on this right must therefore allow the party in breach an opportunity
to remedy the defect. The remedial steps may involve completing the performance (completing the
building operations, for instance), providing a substitute performance (replacing a defective product
with a new one, for instance) or repairing work (repairing the non-conforming goods). It is up to the
party in breach to decide how to effect proper and full performance in accordance with the
provisions of the agreement. Once performance is properly completed, the right to withhold
performance is extinguished.
The exceptio non adimpleti contractus is available in respect of all types of contract. The view that it
is available only in respect of contracts for work and services (locatio conductio operis or operarum)
was rejected in the case of Thompson v Scholtz. In that case, the exceptio was applied in respect of a
contract for the sale of a farm where the seller had failed to provide the buyer with occupation of the
farm house, but nevertheless claimed occupational rent.
The exceptio cannot be raised in cases where the failure of a party to perform properly or at all is
excused by law, or where the risk of such defective performance lies with the party who wishes to
raise the exceptio. For instance, in a contract of sale, the risk of damage to or destruction of the
goods passes to the buyer as soon as the sale is perfecta (unconditional). If, thereafter, the goods
should be destroyed by vis maior, the contract is not discharged and the buyer remains liable to pay
the contract price, even though the seller is excused from delivering the goods.
As noted earlier, the exceptio cannot be raised where a party has elected to cancel the agreement,
since it is aimed at fulfilment of the agreement.

QUESTION 5
S sold a second-hand tractor to P for R60 000. The maximum price for the tractor under the relevant
price control regulations was R40 000. The price control regulation prohibited the conclusion of
contracts of sale at a price in excess of the controlled price and provided that such contracts were
void Both S and P were aware of the provisions of the price control regulation S delivers the tractor,
but P refuses to pay the purchase price Advise S if he can recover the purchase price from P, or claim
the return of the tractor Discuss with reference to Jajbhay v Cassim 1939 AD 537 and other relevant
case law Do not apply the Consumer Protect1on Act 68 of 2008 (15)

Answer
Identifying the problem (maximum of 3 marks)

The question deals with whether or not the contract between S and P is illegal and void. Depending
on the outcome of this enquiry, it has to be considered if S can claim the R60 000 (performance)
from P, or if P can claim the tractor back (restitution).

Discussion of law (maximum of 7 marks) and application of the law to the facts (maximum of 3
marks)

(1) Illegality
The contract is illegal because the price control regulation prohibited the conclusion of contracts of
sale at a price in excess of the controlled price and provided that such contracts were void.
Application to facts of the problem: In this case the sale price (R60 000) is in excess of the maximum
controlled price (R40 000). Therefore the contract is void.

(2) The consequences of a contract that is void for illegality


(a) Contract cannot be enforced (ex turpi rule)

The law:
An illegal contract is void or invalid since one of the requirements for a valid contract is absent. An
illegal contract creates no obligations and, consequently, it cannot be enforced (Cape Dairy and
General Livestock Auctioneers v Sim 1924 AD 167). Neither party can institute action on the contract
or claim performance from the other party, because from an illegal cause, no action arises (ex turpi
causa non oritur actio – known as the ex turpi rule). So, for instance, if a party has suffered damage
as a result of such a contract, he or she may not claim contractual damages from the other party. A
court does not have the discretion to relax this rule and there are no exceptions to it. Performance by
one or even both the parties to the illegal contract does not make the contract legal.

Application to facts of the problem: S cannot claim payment of the price (R60 000) from S as this
amounts to the enforcement of a contract which is void.
(b) Reclaiming performance that has been made in terms of an illegal contract (the par
delictum rule)

Discussion of the law:


If a contract is void and there has been performance, restitution of what has been performed should,
in principle, be granted. Where ownership of the performance has not passed, the performance can
be reclaimed with the rei vindicatio (the action of the owner for the return of his or her property) –
for example, the return of leased property, where occupation has been given to the lessee. If
ownership did pass, the claim for the return of the performance will be based on unjustified
enrichment (for example, a claim for the repayment of the rental that has already been paid to the
lessor).

However, the par delictum rule, in pari delicto potior est conditio possidentis (where two parties are
equally morally guilty, the one who is in possession is in the stronger position), will prevent
restitution from taking place. This rule does not preclude the enforcement of an unlawful contract by
means of a claim for specific performance (that is the function of the ex turpi rule), but it does
prevent a party from reclaiming his or her performance in terms of an unlawful contract. The par
delictum rule is based on two considerations of public policy: a court will not assist those who
approach it with ‘unclean hands,’ and unlawful contracts should be discouraged.

Application to the facts of the problem:


S and P are equally morally guilty as they both were aware of the provisions of the price control
regulation. Therefore the par delictum rule applies and the possessor of the tractor is in the stronger
position. But the enquiry does not end here as the court may relax the par delictum rule.

Discussion of the law:


The par delictum rule may sometimes operate very harshly towards a party who claims the return of
his or her performance. Consequently, in Jajbhay v Cassim (1939 AD 537), the Appellate Division held
that this rule may be relaxed in appropriate circumstances in order to do justice between the parties.
The underlying principle of the par delictum rule is public policy, but public policy also demands that
‘justice shall be done between man and man’. The court considered the facts of the case. A sub-
lessor applied for the ejectment of a sub-lessee on the ground that the sub-lease was illegal. Both
parties were criminal offenders by being parties to the sub-lease. The sub-lessor did not terminate
the lease with a month’s notice as provided for in the illegal contract, while the sub-lessee complied
with his obligations under the lease. Neither party was enriched at the expense of the other party.
The court consequently refused to relax the rule.

This decision has been criticised because it is practically impossible to predict whether a court will be
prepared to relax the par delictum rule in a particular case. It will depend entirely on the facts.

The courts consider the following factors when deciding the question whether the par delictum rule
should be relaxed:
•whether the defendant will be enriched at the expense of the plaintiff if the rule is not relaxed;
•whether relaxing the rule would indirectly enforce the illegal contract; and
•any other consideration of public policy.
In Klokow v Sullivan, the contract of sale of a business was illegal for lack of compliance with
statutory requirements regarding the sale of a liquor-licensed business. The purchaser paid R250 000
of the purchase price and took possession of the business. The purchaser later returned the business
and claimed repayment of the R250 000. The Supreme Court of Appeal relaxed the par delictum rule
as the equities clearly favoured the purchaser (the seller was left with both the business and R250
000). The only factor that the court considered was that if the par delictum rule was not relaxed, the
defendant would have been enriched at the expense of the plaintiff.

Application to the facts of this problem:


P will clearly be enriched at the expense of S if the par delictum rule is not relaxed, as P has both the
tractor and the price (R250 000). Therefore, in line with the decision in Klokow v Sullivan, the par
delictum rule should be relaxed, and the tractor returned to S. Also, relaxing this rule would not
result in the enforcement of an illegal contract, but it would result in granting restitution to S in
terms of an enrichment claim, in order to do justice between man and man. There does not also
appear to be any other consideration of public policy which gravitates against the relaxation of the
par delictum rule.

Giving appropriate advice (Maximum of 2 marks):


S cannot claim the purchase price because the contract is void. However, S should successfully claim
the return of the tractor, based on an enrichment claim.
PVL3702_Question from Study Unit

Question
List the requirements for the formation of a valid contract (5)

Answer
1. There must be agreement (consensus) or ostensible agreement between the parties.
2. The parties must have capacity to act.
3. The performance must be possible at the time the contract was entered into.
4. The conclusion of the contract, the performance & the object of the contracting parties must be
lawful (contract must in other words be legal).
5. Constitutive formalities must be complied with.
6. The contents or consequences of a contract must be ascertained or be readily ascertainable, there
must in other words be, certainty as to the obligations to be created by the contract

1. Obligation:
An obligation is a juristic bond in terms of which the party or parties on the 1 side have a right to
performance & the party or parties on the other side have a duty to render that performance. In
other words it’s a legal relationship between 2 or more Legal Subjects.

2. Types of obligations
 Civil obligation is an enforceable agreement can be enforced in a court of law.
 Natural obligation does have some legal effect: it’s a legal relationship as opposed to a moral
relationship

3. Contract:
A contract is an agreement entered into with the intention of creating an obligation or obligations. A
contract is therefore a fact, an event.

4. Types of agreements
 Agreements creating obligations [ general agreements ] : agreements of this nature are nothing but
contracts.
 Agreements extinguishing a debt : terminates the obligation .e.g. release from a debt
 Real agreements: is an agreement whereby a right is transferred. E.g. when you buy a house
 Gentleman’s agreements’: are not enforceable because there is no animus contrhendi they are
binding in honour only.

Question
Discuss with the constitutional courts approach to the sanctity of a contract?

Answer
 Barkhuizen v Napier 2007 (5) SA 323 (CC)
This issue was dealt with in the Barkhuizen v Napier 2007 (5) SA 323 (CC)
Facts
Barkhuizen had insured his motor vehicle with Napier, a syndicate representing Lloyd's Underwriters
of London in South Africa. The vehicle was involved in an accident, and Barkhuizen timeously lodged
a claim with Napier. Napier repudiated the claim. Two years later, Barkhuizen instituted action
against Napier. Napier raised a special plea alleging that Napier had been released from liability
because Barkhuizen had failed to serve summons within 90 days of being notified of the repudiation
of his claim. The special plea was based on a clause in the contract
Barkhuizen argued that the time-limitation clause was unconstitutional and unenforceable because it
violated his right under section 34 of the Constitution of the Republic of South Africa Act, 1996 ('the
Constitution') to have the matter determined by a court.
The High Court upheld Barkhuizen's contention. On appeal to the Supreme Court of Appeal (SCA),
the SCA found that section 34 of the Constitution did not prevent time-limitation clauses or
provisions in contracts that were entered into freely and voluntarily, but that it could not be
determined on the evidence whether the clause under consideration had been entered into freely
and voluntarily. The SCA accordingly upheld the appeal (and the special plea), and Barkhuizen
approached the Constitutional Court (CC) for leave to appeal against the decision of the SCA.
Ngcobo J concluded that the 90-day time limitation was not manifestly unreasonable, nor was it
manifestly unfair. There was no evidence that the contract had not been freely concluded; that there
was unequal bargaining power between the parties or that the clause was not drawn to Barkhuizen's
attention.

Question
Discuss the impact of the of the Consumer Protection Act 68 of 2008 upon the law of contract with
reference to its aims, objectives, scope, national regulatory institutions and sanctions (10)

Answer
1. Discuss the Impact of the CPA
The primary purpose of the act is to protect consumers from exploitation in the marketplace, and to
promote their social and economic welfare.
It aims to: establish a legal framework for the achievement and maintenance of a consumer market
that is fair, accessible, efficient, and responsible, for the benefit of consumers generally;
• Promote fair business practices;
• Protect consumers from unconscionable, unjust, or unreasonable business practices.
 The scope applies to:
• Most transactions concluded in the ordinary course of business between suppliers and consumers
within south Africa, as well as;
A consumer includes not only the end-consumer of goods and services but also:
• To the state
• To a juristic person with an asset value or annual turnover above the threshold Franchisees
 Credit agreements
 Employment contracts
 Relatively small businesses in the supply chain (asset value or annual turnover below the threshold
determined by the minister) the act does not apply to any transaction in terms of which goods and
services are promoted or supplied:
Enforcement
1. Courts,
2. National consumer commission
3. National consumer tribunal.
Sanctions
Failure to comply with provisions of the act might attract various sanctions:
1. compliance notices
2. imposition of fines and
3. Criminal penalties.
4. Contractual provisions in contravention of the act may be declared null and void to the extent of
non-compliance.

Question
Explain the effect of the constitution of RSA on contract law

Answer
 PACTA SUNT SERVANDA
The Constitutional Court appears to prefer an indirect application of the Constitution between
private parties:
 an approach that tests the validity of a private contractual provision against the requirements of
public policy,
 but also recognises that public policy is now determined with reference to the fundamental values
embodied in the Constitution, and particularly in the Bill of Rights.
The courts have shown a willingness to intervene if a party exercises a contractual power in a
manner that fails to respect the constitutional rights of another party, and may even, in appropriate
circumstances, be willing to compel one party to contract with another on constitutional grounds.
Refere to the following sections
1, 2, and 8
Then state the provisions of section 39(2) when developing the common law of contract, courts are
required to do so in a manner that promotes the spirit, purport and objects of the Bill of Rights
.Common law principles of contract are informed by constitutional values. Thus it was recognised
that public policy, as informed by the Constitution, required that parties should comply with
contractual obligations that have been freely and voluntarily undertaken.
The doctrine of pacta sunt servanda (agreements must be honoured) also gives effect to the central
constitutional values of freedom and dignity
Parties to a contract are autonomous, and their self-autonomy, or the ability to determine the terms
of the contract to be entered into, was regarded by the Court as the very essence of freedom and a
vital part of dignity.
In this way, public policy endorses freedom of contract. Nevertheless, the doctrine of pacta sunt
servanda is not to be applied without exception.
FAIRNESS: BREDNKAMP CASE
Mr Bredenkamp held a number of accounts at the Standard Bank (both personal accounts as well as
accounts held in the name of various entities controlled by him). On 25 November 2008 the US
Department of Treasury listed Bredenkamp as a "specially designated national" ("SDN") because of
his association with President Mugabe, and because Bredenkamp was said to have provided financial
and logistical support to "Mugabe's regime". His facilities were withdrawn
Bredenkamp approached court for urgent relief, contending that the bank's decision to cancel his
facilities was unfair and therefore invalid, being "unconstitutional"
The SCA, in a well-reasoned judgment delivered by Harms DP, concluded that "fairness" is not a
freestanding requirement for the exercise by any party of a contractual right. There was no obligation
on the bank to afford Bredenkamp a "hearing" before electing to terminate its banking relationship
with the client.
The bank had a contract, which gave it the right to cancel. The bank had exercised its right to
termination in a bona fide manner. The termination, therefore, did not offend any identifiable
constitutional value and was not otherwise contrary to any other public policy consideration.

Study Unit 6 & 7_ OFFER AND ACCEPTANCE


Case law
1. CRAWLEY REX
Facts
Shopkeeper advertised sale of tobacco at reduced price. C bought half a KG and returned
immediately to buy more. Shopkeeper refused to sell to C again. C refused to leave the shop and was
arrested for remaining unlawfully on the premises
C argued that he had accepted the shopkeepers offer t sell tobacco and therefore a contract existed
between C and shopkeeper
Finding: Adverts are invitations to the public to do business and it is the client who makes an offer to
purchase, which the shopkeeper can then accept or reject

2. Carlill v. Carbolic Smoke Ball Co. [1893] Q.B. 256 (C.A.).


Carbolic Smoke Ball Co. (D) manufactured and sold The Carbolic Smoke Ball. The company placed ads
in various newspapers offering a reward of 100 pounds to any person who used the smoke ball three
times per day as directed and contracted influenza, colds, or any other disease. After seeing the ad
Carlill (P) purchased a ball and used it as directed. Carlill contracted influenza and made a claim for
the reward. Carbolic Smoke Ball refused to pay and Carlill sued for damages arising from breach of
contract. Judgment for 100 pounds was entered for Carlill and Carbolic Smoke Ball appealed.
Issue : Does one who makes a unilateral offer for the sale of goods by means of an advertisement
impliedly waive notification of acceptance, if his purpose is to sell as much product as possible?
Holding and Rule (Lindley) : Yes. One who makes a unilateral offer for the sale of goods by means of
an advertisement impliedly waives notification of acceptance if his purpose is to sell as much product
as possible. The court held that an advertisement is considered to be an offer when it specifies the
quantity of persons who are eligible to accept its terms. If such an advertisement requires
performance, the offeree is not required to give notice of his performance.
3. Bloom v The American Swiss Watch Company
Facts: Bloom claimed a reward in terms of a notice published by ASW promising a reward to any one
providing information which lead to the arrest of the thieves and recovery of jewelry stolen . Bloom
gave the information BEFORE he became aware of the notice
Finding and Rationale : He therefore, did not furnish the information in response to the notice and
could not therefore, be said to have accepted ASW’s offer to pay a reward Thus, no contract came
into being between Bloom and ASW and therefore, no basis on which he could claim the reward
Note : Parties who agree must be aware of their agreement
1. List the requirements of a valid offer
OFFER:
1. Must be firm. (That is to say, with the intention that its acceptance will call into being a binding
contract.)
2. Must be complete. (It must contain all the material terms of the proposed agreement.)
3. Must be clear and certain. (It should be enough for the addressee to answer merely “yes” for a
contract to come into being.)
4. Must meet the requirements of the Consumer Protection Act.

2. State the ways an offer may be terminated.


1. Rejection of the offer
2. Acceptance of the offer
3. Death of either party
4. Effluxion of the prescribed time, or of a reasonable time
5. Revocation of the offer
6. Loss of legal capacity to act

PROBLEM TYPES QUESTION: OFFER AND ACCEPTANCE


Question
X is on her way from work and sees a white bull terrier bitch hiding in a doorway. Being an animal
lover, she takes the dog home with her. The next day, she sees the following advertisement in the
newspaper: Lost in Johannesburg, on 27 May. Pedigree white bull terrier bitch with black patch over
left eye. Answers to the name of Beauty.
Reward of R1000 for information leading to safe return. Tel 011 555 5555. She realises that the dog
she found matches the description given. She calls the advertiser who rushes over to be joyfully
united with Beauty. In his joy, Beauty’s owner, Y, seems to forget the reward and X wishes to claim it
from him. Will she be successful?
Substantiate your answer. Refer to Bloom v American Swiss Watch Co and other relevant case law in
your answer. [10]

Answer
X will only be successful in her claim if a valid contract arose between X and Y, and this will be the
case if there was a valid offer and acceptance.
The offer: The offer was in the form of an advertisement. The general rule in our law is that an
advertisement constitutes an invitation to do business (Crawley v Rex).
However, in Bloom v American Swiss it was held that the advertising of a reward might be construed
as an offer to the public. Offers to the public at large can be made (Carlill v Carbolic Smoke Ball Co).
In our case, the offer was firm, complete, clear, and certain. The offer can therefore be said to have
been valid.
The providing of information by X was a valid acceptance of Y’s offer:
 X’s acceptance was unqualified
 X, as a member of the public to whom the offer was made, may accept (offer may only be accepted
by offeree – Bird v Summerville) reward for doing so).
 X’s acceptance was a conscious response to the offer (he knew of the offer and could thus accept it
– unlike the situation of Bloom v American Swiss where the plaintiff returned the item but was
unaware there was a rewardfor doing so). It can be concluded that a valid contract arose in this
problem, because Y made a valid offer, which X validly accepted.

Question
4. Problem type question: Expedition theory: Cape Explosive Works case
S, who lives in Upington, sends P, who lives in Grahamstown, a letter by private courier in which she
offers to sell him her (S’s) motorcycle, a collector’s piece, for R100 000. She states in her letter that
her offer will expire on 1 February. P accepts S’s offer by letter, which he posts on 31 January. S
receives the letter on 7
February and only reads it on the next day. P tenders payment of R100 000 but S refuses to accept
payment. Did a valid contract arise between S and P? Substantiate your answer. [15]

Answer
The question is whether P has accepted S’s offer in time and thus whether S and P have reached
consensus.
Where the offeror has prescribed a time limit for acceptance, the offer lapses automatically if it is not
accepted within the prescribed period.
The general rule is that a contract comes into being only when the acceptance is communicated to
the mind of the offeror. The information theory, which is the general rule in our law, states that the
agreement is concluded when and where the offeror learns or is informed of the acceptance – in
other words, when the offeror reads the letter of acceptance.
On the other hand, the expedition theory applies to postal contracts. In terms of this theory,
introduced into our law in the Cape Explosive Works case, a contract comes into being when and
where the offeree posts the letter of acceptance. By making an offer through the post, the offeror is
deemed not only to have authorised acceptance by post, but also to have waived the requirement of
notification of acceptance.
The question that then arises is which theory applies. In our law, the general rule is that the
information theory applies, however the expedition theory will apply if the following four criteria are
met:
1. the offer is made by post or telegram
2. the postal services are operating normally
3. the offeror has not indicated a contrary intention, expressly or tacitly, and
4. the contract is a commercial one.
If any of these criteria are not met, the information theory applies. In this question, the offer was not
made by post, instead it was sent by private courier, and therefore the expedition theory does not
apply. It follows that the information theory must be applied. Because S only learnt of the
acceptance by P after expiry of the offer (when S read the letter on 8 February), the offer had already
lapsed and no valid contract arose between the parties.

Question
Discuss and distinguish between an option and a right of pre-emption.

Answer
An option is a substantive offer, reinforced by an agreement in terms of which the offeror undertakes
to keep his offer open to the offeree for a specified period.
A right of pre-emption is a type of right of preference. It is given by a prospective seller to a
prospective purchaser, to give the purchaser preference if the prospective seller should decide to
sell.
Consequences
 The option contract obliges the grantor to maintain his substantive offer in accordance with the
terms of the option contract. A substantive contract arises if the grantee accepts the substantive
offer. The option holder determines whether a contract in accordance with the substantive offer
arises.
 A right of pre-emption does not place a duty on the grantor to sell the subject matter of the right;
the grantee merely acquires the preferential right to buy should the grantor decide to sell. The
obligation of the grantor is negative. The grantor may not alienate the thing to a third party except
under the conditions prescribed in the agreement creating the right. The grantor primarily
determines whether the parties will conclude a substantive agreement, usually after negotiating the
contractual terms.

Question
Must the option meet the requirements of the main contract?

Answer
Brand v Spies
S granted B an option to buy farm orally. S later repudiated the option. B sued S for damages on
ground of breach of contract
Court held:
 No contract existed because, contract of sale of land had to be in writing. For a contract to be in
writing, both offer and acceptance had to be in writing
 No written offer had existed which could be accepted by B to bring about a written contract

Question: ABSENCE OF CONSENSUS


Refer to George v Fairmead (Pty) Ltd, Sonap Petroleum (SA) Ltd (SA) (Pty) Ltd v Pappadogianis, and
other relevant case law in your answer
Albert takes his motor vehicle to Dodgy Motors for a service. On his arrival, he is asked to sign a “job
card” by the owner. Albert enquires why he is required to sign the “job card” and the owner explains
to him that by signing he is authorising them to conduct the service on his car, which will cost R1000.
He signs the “job card” without reading it. While servicing the car, the service manager finds faults
on the car (unrelated to the service) and he proceeds to do these additional repairs for a further
R2000. Albert refuses to pay for the additional repairs and argues that he did not authorise such
repairs. The owner of Dodgy Motors argues that Albert is obliged to pay for the work done as the
“job card” contains a contractual clause authorising Dodgy Motors to do any repairs on the motor
vehicle which they deem necessary, without asking the client’s authorisation, and requiring the client
to pay for such repairs. Advise Albert on whether he is liable on the contract to pay Dodgy Motors
R2000 for the additional repairs. Refer to George v Fairmead (Pty) Ltd, Sonap
Petroleum (SA) Ltd (SA) (Pty) Ltd v Pappadogianis, and other relevant case law in your answer. Do not
apply the Consumer Protection Act to this question. [15]

Answer
This deals with the question whether Albert and the owner of Dodgy Motors have reached actual
consensus or ostensible consensus.
At the outset, it must be determined whether agreement as a contractual basis exists between the
parties, as required in terms of the will theory. Consensus has three elements:
1. The parties must seriously intend to contract
2. The parties must be of one mind as to the material aspects of the proposed agreement (the terms
and the identity of the parties to it)
3. The parties must be conscious of the fact that their minds have met
In the present case, the parties were not in agreement as to the consequences they wished to
create; Albert thought that he was authorising Dodgy Motors to only service his car, while the owner
of Dodgy Motors knew that the contract also allowed Dodgy Motors to conduct repairs which they
deemed necessary and payable by Albert without any further authorisation from Albert. This is a
mistake as to the obligations the parties wished to create and is thus a material mistake, which
excludes consensus between the parties. This means that no contract could arise on the basis of the
will theory.
This type of mistake can be illustrated with a number of cases:
In George v Fairmead, the appellant signed a hotel register without reading it.
The register contained a term excluding the respondent from liability for certain acts. The appellant
was unaware of this term and his mistake related to a term that he believed would not be in the
contract and as such was material because it related to an aspect of performance.
In Allen v Sixteen Stirling Investments, the plaintiff believed he was purchasing the erf pointed out to
him by the seller’s agent, while the written contract that he signed indicated the correct erf, which
was a completely different property. His mistake related to performance and was material.
However, the matter does not end here. A party may be held contractually liable on the basis of a
supplementary ground for liability, namely the reliance theory.
In this regard, the direct or indirect approach to the reliance theory may be considered.
DIRECT APPROACH:
With reference to the direct approach, contractual liability is based on the reasonable reliance that
consensus has been reached, which the one contractant (the contract denier) creates in the mind of
the other contractant (the contract enforcer).
According to the Sonap case, the direct reliance approach entails a threefold enquiry:
1. Was there a misrepresentation regarding one party’s intention?
2. Who made the misrepresentation?
3. Was the other party misled by the misrepresentation, and if so, would a reasonable person have
been misled?
In our question, firstly, Albert made a misrepresentation by signing the contract, that his intention is
the same as that expressed in the contract.
Secondly, the owner of Dodgy Motors could actually have been misled by this misrepresentation, but
a reasonable man would have taken steps to point out to
Albert that the contract allows Dodgy Motors to unilaterally conduct repairs on the car, because
Albert enquired about the purpose of the “job card” and the owner of Dodgy Motors misled him to
believe that by signing the card he is merely authorising the service to be done.
In Sonap the court found that the contract enforcer knew that the contract denier was acting under a
mistake and was thus not misled.
In our case, Albert therefore did not create a reasonable reliance that he wished to be bound by the
contract he signed.
INDIRECT APPROACH (IUSTUS ERROR DOCTRINE):
In terms of this approach, a party may escape liability to be bound to a contract if it can be
established that the mistake is both:
1. Material, and
2. Reasonable
It has already been shown in the discussion above that Albert’s mistake is material. It still has to be
determined if his mistake was reasonable.
The contract denier’s mistake will be reasonable in the following instances:
1. If caused by a misrepresentation on the part of the contract enforcer (an unlawful
misrepresentation).
2. If the contract denier is not to blame for the mistake.
3. If the contract denier did not cause a reasonable belief in the contract asserter that the contract
denier assented to the agreement.
Fault is not a requirement for the misrepresentation by the contract enforcer, but unlawfulness is. If
the misrepresentation is a positive act it is unlawful in itself.
If a legal duty to speak exists and the party has kept quiet when he ought to have spoken, an
unlawful negative misrepresentation has occurred. A legal duty to speak exists in the following
instances:
 Where the contract asserter knows or ought to know as a reasonable person that the other party is
mistaken
 Where, prior to the conclusion of the agreement, the contract asserter created an impression
directly conflicting with the provisions of the agreements, he must draw the contract denier’s
attention to the discrepancy (Du Toit v Atkinson’s Motors).
In our problem, Albert enquired about the purpose of the “job card” and the owner of Dodgy Motors
misled him by answering that by signing he was merely authorising the service. The owner’s
misrepresentation was a positive act, and was therefore unlawful. Albert’s error was thus
reasonable.
Applying the indirect approach to the reliance theory we do not have a valid contract. Applying the
direct approach, we do not have a valid contract. Albert is not contractually liable to pay R2000 for
the repairs.
Question: MISREPRESENTATION
1. State the requirements for restitutio in integrum.

 Misrepresentation by the other party


 Inducement
 Intention to induce
 Materiality

Question
Refer to ‘Trotman v Edwick’ and ‘Phame v Paizes’. (10)
S sells his farm to P for R100000. There is a pine plantation on the farm and P purchases the farm for
the purpose of growing and selling pine wood. During negotiations S points out the boundaries of
the farm to P and includes a piece of land 300 square meters. Which in actual fact is part of an
adjoining farm, with the purpose of persuading P to conclude the contract. S knows that the piece of
land isn’t part of his farm. The market value of the farm is R900000. P would only have been
prepared to pay R800000 if she had known the true state of affairs. Discuss all the remedies which P
possibly may have. Refer to ‘Trotman v Edwick’ and ‘Phame v Paizes’. (10)

Answer
Misrepresentation: It’s an untrue statement of fact made by 1 party to the other during negotiations
which induces the other party to enter into a contract he would:
 Never have entered into had he known the truth or
 Would have entered into but on other terms.
Remedies available to the injured party are 2-fold:
1. he may rescind (cancel) the contract, &
2. Whether he rescinds or upholds the contract, he may claim damages to compensate him should
he have suffered actual loss.
1. Rescission:
Presence of culpable misrepresentation makes contract voidable. If aggrieved party rescinds contract
the obligations between the parties will be terminated with retrospective effect. Rescission leads to
restitution.
DOLUS DANS: Misrepesentee may elect to cancel where the facts show that if it had not been for
misrepresentation he would not have entered into the contract at all.
DOLUS INCIDENS: Isn’t so clear whether the misrepesentee has the right to elect to rescind where
the facts show that if there had been no misrepresentation he would still have entered into the
contract but on different terms.
Party has to cancel contract within a reasonable time after discovering the misrepresentation, if
delays might forfeit that right. In the case of rescission due to misrepresentation, as in the case of
cancellation on grounds of duress, undue influence, commercial bribery or breach of contract,
Reciprocal restitution takes place.
Restitution entails return of what has actually been received & also sum of money required to
restore the other contractant in full to his previous position.

2. Damages:
The case of Bayer SA v Frost allows damages for negligent misrepresentation. Victim of culpable is
entitled to damages whether he rescinds or upholds contract. Damages are calculated acc to the
plaintiff’s negative interest. Where breach of contract is concerned damages are calculated according
to the positive interest – aggrieved party must be placed in position he would have been in if
contract was carried out. A Rule Of Thumb id always to decide how much worse off the
misrepesentee is financially as a result of the misrepresentation.
Where contract is rescinded, restitution takes place & misrepesentee’s loss will take form of wasted
costs which he may have incurred with the conclusion & cancellation of the contract.
Where contract is upheld distinction between dolus dans & dolus incidence is NB when it comes to
assessing representee’s damages in the case where he decides to uphold the damages.
Test in case of Dolus Dans:
 If no misrepresentation – no contract.
 Loss determined by deducting value of performance made by misrepesentor from that made by
misrepesentee & adding to the difference any consequential loss (loss of profit) the misrepesentee
may have suffered.
P’s damages are: R1000000 – R900000 = R10000. (Trotman v Edwick).
Test in case of Dolus Incidens:
 If no misrepresentation – still enter contract but on different terms.
Loss determined by deducting value of performance that misrepesentee would have been prepared
to render had there been no misrepresentation, from performance that misrepesentee actually
rendered, & adding any consequential loss that the misrepesentee may have incurred.
Misrepesentee must proof that contractants would have agreed to the putative performance.
Misrepesentee is free to prove any putative price even 1 less than market value of the performance.
P’s damages are: R1000000 – R800000 = R200000. (De Jager v Grunder).
In the case of Innocent misrepresentation case would be:
Aedilitian actions are:
 Actio quanti minoris: misrepesentee could claim difference between price paid & actual value of
thing purchased.
 Actio redhibitoria: cancellation of contract & restitution was claimed.
After Phame v Paizes innocent misrepresentation situation is:
1. A buyer who has been misled by a representation amounting to a dictum et promissum (It’s a
material statement made by the seller to the buyer during the negotiations, bearing on the quality of
the res vendita (object) & going on beyond mere praise & commendation) made by the seller, may
claim rescission & restitution with the action redhibitoria or restitutional damages (abatement of the
purchase price) with the action quanti minoris. He will presumably be entitled to raise the exception
redhibitoria or the exception quanti minoris as a defence to an action by the seller.
2. If the seller’s representation doesn’t amount to a dictum et promissum or if the representation is
made to a party to any contract other than a contract of sale, the misrepesentee has a claim for
rescission & restitution based on innocent misrepresentation per se. He doesn’t, have an action for
restitutional damages.
Question: STUDY UNIT 13: DURESS
Thus John won’t be able to rely on this clause due to the intention factor.

Question
1. State the requirements for duress as set out in “Broodryk v Smuts”. (5)

Answer
In Broodryk v Smuts the court expressed the elements necessary to set aside or rescind a contract on
the ground of duress as follows:
1) There must be actual violence or reasonable fear.
2) The fear must be caused by the threat of some considerable evil to the contractant or his family.
3) It must be the threat of an imminent or inevitable evil.
4) The threat or intimidation must be contra boni mores.
5) The moral pressures used must have caused damages.

2. Problem type question


Peter wants a car just like his neighbour Sipho has. Sipho cheats on his income tax since Sipho often
boasts of this. Consequently Peter tells Sipho that if Sipho doesn’t sell his car to him for R20000 he
will report Sipho to the receiver of revenue. Sipho sells his car to Peter for the amount mentioned
although it’s worth R100000.

Question
On what basis will Sipho be able to attack the contract? Briefly discuss with reference to the court’s
requirements or elements of the cause of action in question. (5)

Answer
Sipho will be able to attack the contract on the basis of Duress. In Broodryk v Smuts the court
expressed the elements necessary to set aside or rescind a contract on the ground of duress as
follows:
1) There must be actual violence or reasonable fear.
2) The fear must be caused by the threat of some considerable evil to the contractant or his family.
3) It must be the threat of an imminent or inevitable evil.
4) The threat or intimidation must be contra boni mores.
5) The moral pressures used must have caused damages.

Question
Peter’s defence against Sipho’s action for rescission of the contract is that he, in fact, acted lawfully.
Will Sipho be successful with an action for rescission of the contract in the circumstances?
Substantiate. (4)

Answer
As in the case of culpable misrepresentation, a contractant who has agreed to a contract under some
threat or other may claim rescission of the contract & restitution of any performance made in terms
of the contract.
The case of the facts given complies with that of a vis compulsiva. It means that the aggrieved
contractant has indeed acted & has expressed his will even though his will has been obtained in an
improper manner. The expression of his will is valid. The contract is therefore not void but voidable.
Since the given set of facts comply with the requirements to set aside a contract on duress, Sipho will
be successful with an action for rescission of the contract, this means that the obligations of the
parties will be terminated with retrospective effect & the parties must restore to each other what
has already been performed in terms of the contract so that they are in the position they were
before the contract was concluded.

Question
Will Sipho successfully be able to claim damages from Peter in the circumstances? Discuss. (1)
Answer
If the aggrieved party has suffered a loss he should be able, as in the case with culpable
misrepresentation, to claim damages. So here Sipho has suffered a loss & will be able to claim
damages from Peter.

Question: Study Unit 14 UNDUE INFLUENCE


Question
1. State the requirements for undue influence. (3)

Answer
In Preller v Jordaan (reaffirmed in Patel v Grobbelaar) it was held that where a party to a contract
requests a court to set aside the contract on the ground of undue influence, an onus rests on that
party to prove:
1. That the other party exercised influence over him.
2. That this influence weakened his powers of resistance & made his will pliable.
3. That the other party exercised this influence in an unscrupulous manner in order to induce him to
consent to a transaction which firstly was to his detriment & secondly, which he with normal free will
would not have concluded.

2. PROBLEM TYPE QUESTION


Mark is engaged to Jane. Mark has a very strong personality and eventually persuades Jane to sell
and transfer her house worth R500 000 to him at a purchase price of a mere R20 000. After
registration of the property in Mark's name he breaks off the engagement. Does Jane have a remedy
available to her? Substantiate your answer with reference to case law.

Answer
Jane may have the contract cancelled or set aside on the basis of undue influence. In terms of Preller
v Jordaan, Jane must prove that:
(1) Mark exercised an influence over her.
(2) This influence weakened her powers of resistance and made her will pliable
(3) Mark exercised this influence in an unscrupulous manner in order to induce her to consent to a
transaction which is to her detriment and which she, with normal free will, would not have
concluded.
Although in English law there is a doctrine known as "undue influence", the Appellate Division in
Preller v Jordaan accepted that it is part of our law, as the sources of common law indicate, that the
concept of dolus is wide enough to cover instances which would be regarded as undue influence in
English law.
The courts have not expressly stated that undue influence constitutes a delict but the cases which
have come before our courts so far concerning this issue can be described in terms of delict

STUDY UNIT 15: COMMERCIAL BRIBERY


1. State the elements for commercial bribery as held in Extel Industrial (Pty) Ltd v Crown Mills (Pty)
Ltd.
1. A reward
2. paid or promised
3. by one party, the briber
4. to another, the agent (agent in true sense or merely a go-between)
5. who is able to exert influence over
6. a third party, the principal
7. without the principal’s knowledge, and
8. for the direct or indirect benefit of the briber
9. to enter into or maintain or alter a contractual relationship
10. With the briber, his principal, associate, or subordinate

Question: STUDY UNIT 17 and 18 : LEGALITY: ILLEGAL CONTRACTS


THAT ARE VOID.

Question
JAJBHAY V CASSIM AND OTHER RELEVANT CASE LAW. [15]

Tony, a petrol attendant, sells dagga to Samuel for R1000. Tony delivers the dagga to Samuel but
Samuel refuses to pay. Section 5 of the Drugs and Drug Trafficking Act provides that no person shall
deal in dagga while section 4 prohibits possession of such substances. Section 13 makes the
contravention of both sections 4 and 5 a crime. Dagga is a substance as defined in section 5.
Advise Tony if he can sue Samuel for payment of R1000 or the return of the dagga. Would your
advice be different if Tony was an undercover policeman who sold dagga to Samuel during a police
entrapment operation? Discuss with reference to Jajbhay v Cassim and other relevant case law. [15]

Answer
This question involves an illegal contract of sale, which is void due to statutory illegality. The fact that
the legislator has enacted a criminal sanction for a contravention is a factor that would imply that the
legislator intended the contract to be void.
An illegal contract creates no obligations and it cannot be enforced. The ex turpi rule applies: from an
illegal cause no action arises. Neither party can institute an action on the contract or claim
performance from the other party. So for instance if a party has suffered damage as a result of such a
contract, he or she may not claim contractual damages from the other party. A court does not have
the discretion to relax this rule and there are no exceptions to it.
A party who has performed in terms of an illegal contract may however reclaim his performance, in
principle, with an enrichment action. However, such restitution will be prevented where the par
delictum rule applies. According to the par delictum rule: where two parties are equally morally
guilty, the one who is in possession is in the stronger position. If this is the case, restitution in terms
of an enrichment action is prevented.
In our case, Tony is precluded from instituting any contractual claim for R1000 from Samuel because
of the ex turpi rule, and also from an enrichment claim because of the par delictum rule.

Question
State the test to determine if a restraint of trade clause is enforceable (Basson test).

1. Is there an interest of one party worthy of protection?


2. If so, is that interest threatened by the conduct of the other party?
3. If so, does such interest weigh up against the interest of the other party to be economically active
and productive?
4. Is there another aspect of public policy that requires that the restraint should be maintained or
rejected?

STUDY UNIT 19: FORMALITIES

1. Formalities
Question
X has been leasing a commercial property from Z for the past three years. The leas will come to an
end on 31 May 2010. On 5 March 2010, X phones Z and offers to renew the lease for a further three
years, which offer Z accepts. During this phone call, the material terms of the renewal agreement are
agreed upon and X and Z further agree that the said material terms must be reduced to writing and
signed by both parties. Subsequently, on 5 April 2010,
X is shocked to receive a letter from Z, advising X that there will be no renewal of the lease and that X
should vacate the leased property on 31 May 2010. X and Z never reduced their oral agreement to
writing. Advise X if a binding agreement with Z exists for the renewal of the lease for a further three
years. Refer to Goldblatt v Fremantle. [15]

Answer
This question deals essentially with formalities stipulated by the parties for a valid creation of a
contract. The main question is whether a formality was stipulated in the oral agreement for the
renewal of lease between the parties, that for such agreement to be valid it should be reduced to
writing.
Parties to an oral agreement will often agree that their agreement should be reduced to writing, and
perhaps also signed. In doing so, they may have the following intentions:
1. To have a written record of their agreement to facilitate proof of its terms. If so, the agreement is
binding even if it is never reduced to writing.
2. Alternatively, they may intend that their oral agreement will not be binding upon them until it is
reduced to writing and signed by them. In Goldblatt v Fremantle, the Appellate Division held that no
contract existed because the parties intended their agreement to be concluded in writing, which also
involved signing by the parties.
In the absence of contrary evidence, the law presumes that the intention of the parties was merely
to facilitate proof of the terms of the agreement. The party who alleges otherwise bears the onus of
proof.
In our case no binding agreement exists because the parties agreed that the oral agreement must be
reduced to writing and signed, and this indicates their intention that the agreement will not be
binding if this formality is not complied with.

Question
Discuss X’s position with reference to Sa Sentrale Kooperatiewe Graanmaatskappy Bpk v Shifren.

Y let premises to X. The lease contained a clause prohibiting X from sub-letting the premises without
the written consent of Y. A further clause of the lease required that any variation of the terms of the
lease had to be in writing and signed by both parties. Later Y told X that he (X) could sub-let a portion
of the premises. After X had sub-let a portion of the premises to a third party, Y changed his mind
and informed X that both X and the sub-lessee (third party) must vacate the premises because X had
breached the contract.

Answer
The facts correspond to a large extent with Shifren. The question is whether parties may orally
deviate from a written agreement that contains a clause that determines that the contract may only
be varied or terminated in a specific manner (non-variation clause). In such instances, the parties
have actually set formalities for the amendment or termination of their contract.
In the Shifren case, the court decided in favour of the lessor even though the lessor apparently gave
permission verbally for the amendment of a lease agreement, which contained such a provision. The
lessor was entitled to cancel the contract as a result of the lessee’s breach despite the oral variation.
The same results would apply to the present case.
In the Shifren case the court’s reasoning was as follows:
Where the parties insert a clause into their contract that provides that any amendment of the
contract, including the specific clause, must be in writing, they cannot later orally amend that clause
or any other provision. However, if the specific clause itself is not entrenched against oral variation,
the particular provision may be varied orally, with the result that thereafter the other provisions of
the contract may possibly also be varied orally.
Question STUDY UNIT 20: POSSIBILITY

Question
Distinguish between initial impossibility of performance, supervening impossibility of performance,
and prevention of performance.

Answer
If a performance is objectively impossible at the time of conclusion of a contract, no obligation
arises. To render performance impossible, it is not sufficient that a particular party cannot perform,
that is, subjective impossibility. The impossibility must be so serious that nobody can render the
performance – that is, it must be objectively impossible. An example of impossible performance is
where A agrees to sell his house to B, but unbeknown to them the house has already been destroyed
by a fire. Initial impossibility of performance prevents a contract from arising at all.
If, after the conclusion of the contract, performance becomes objectively impossible without the
fault of the debtor, as a result of an unavoidable and unforeseen event, this is known as supervening
impossibility of performance, and the obligation to perform is also, as a general rule, extinguished.
The requirements for supervening impossibility of performance are:
1. the performance must be objectively impossible; and
2. the impossibility must be unavoidable by a reasonable person. If, after the conclusion of the
contract, performance on either side becomes impossible owing to the fault of either the debtor or
the creditor, the contract is not terminated, but the party who rendered the performance impossible
is guilty of a breach of contract known as prevention of performance. It is not necessary that the
performance should be objectively impossible in order for the breach to arise; subjective
impossibility will suffice.

STUDY UNIT 23: TERMS


Question
Distinguish briefly between Essentialia, Naturalia and Incidentalia of a contract and give examples of
each.

Answer
Essentialia:
They are positive provisions of law. They are those terms which the law requires as essential to place
a contract in a certain category. E.g.: contract of sale – the purchase price and the thing.
Naturalia:
They are positive provisions of law. Terms which by law forms part of the contract but these
provisions may be changed by the parties, unless the law contains a provision to the contrary. But if
nothing is stipulated by the parties when concluding the agreement, the usual positive rules become
operative. E.g.: the seller’s duty to give undisturbed possession to the purchaser of the thing sold.
Incidentalia:
Here the law makes no positive provisions. These are the special agreements by the parties. The law
doesn’t lay down any rules in this connection but leaves it to the parties to make their own
arrangements. E.g.: it’s naturale of a sale that the potatoes must be edible (unless such warranty is
excluded by parties). Fact that potatoes are to be fetched by Johannesburg station on Friday,
however, is an incidentalia of the

Question
Discuss fully with reference to “Minister van Landbou v Scholtz”. Don’t discuss the Aedilitian
remedies for the latent defects.
S a breeder of stud bulls sold one to M. S knew that M intended to use the bull for breeding
purposes, subsequently, the bull proved to be infertile and M claimed cancellation of the sale and a
refund of the purchase price. On what basis will M be able to have the contract set aside? Discuss
fully with reference to “Minister van Landbou v Scholtz”. Don’t discuss the Aedilitian remedies for the
latent defects. (15)

Answer
To create a contract the parties must make their wills clearly known. But once it has been established
that a contract has come into being, a good deal of difficulty may arise in respect to the contents of
the contract. It isn’t necessary for the parties to make known all the contents of the contract. Our
courts hold the view that something which the parties regard as obvious may form part of the
contract, even if they have said nothing about it & even if they have made no other signs or gestures
relating to it. Our courts usually use the so-called HYPOTHETICAL BYSTANDER TEST to determine
whether a tacit term can be implied in a contract. This test was expressly applied in the Van den Berg
Case but not in the Shcoltz Case. Scholtz Case:
The same sets of facts apply to this case as was stated in the Question. AD Held: Seller had tacitly
guaranteed that the bull he sold was fertile.
1. It’s naturale of a contract of sale that seller is liable for latent defects in the thing sold without the
parties having agreed thereto.
2. Buyer would be entitled to cancel contract with action redhibitoria or get a reduction in Purchase
Price with the action quanti minoris.
3. The parties can also agree expressly or tacitly that seller will be liable for latent defects in the thing
sold – i.e.: the relevant naturale doesn’t prevent seller from expressly or tacitly guaranteeing that
thing sold is free of latent defects.
4. If it later becomes apparent that there are defects seller will be liable for breach of contract -
normal contractual action .
The tacit terms the court inferred from the facts & the facts the court regarded as relevant: Court
inferred a tacit guarantee that a bull was fertile from the following facts: Bull bought for breeding
purposes, parties assumed bull was fertile, but was never examined for fertility, seller expressed his
confidence that the purchaser would breed good calves from the bull, seller wouldn’t have sold bull
had he known it was fertile, & seller was a breeder of bulls which he sold for breeding purposes. This
case is an example of how our courts apply the test for tacit terms.M will be able to set aside the
contract due to the tacit terms of the contract.

3. Ticket cases
Question
A buys a ticket for a ride in a helicopter. On the back of the ticket there’s a clause excluding the pilot’s
liability in the event of an accident. Is A bound in terms of the ticket? Substantiate your answer. (5)

Answer
In certain contract s it’s customary that a ticket is issued in order to serve as evidence of the
existence of a contract (e.g.: contract s of carriage or deposits). Reference is usually made on such
tickets to certain terms (e.g.: railway regulations) to which the contract is said to be subject.
To determine this Question our courts, following the English decisions, have laid down a 3-fold test:
1. Did the person who received the ticket know there was writing or printing on the ticket?
2. Did he know the writing or printing referred to terms of the contract?
If both these Questions can be answered affirmatively the terms referred to form part of the
contract, but if either of them is answered in the negative, a 3rd Question is put.
3. Did the party issuing the ticket take the steps which were reasonably necessary to bring the
reference to the terms to the notice of the other party?
If this was done the terms form part of the contract, if not, the other party is not bound by them.
Whether the reasonably steps were taken for the purpose of Q3 is a Question of fact.

STUDY UNIT 24: INTERPRETATION


Explain briefly what you understand by the Parol Evidence Rule. (6)

Answer
Parole Evidence Rule:
Dispute about the contents of the agreement arises often. To settle such a dispute, the agreement
must be interpreted in accordance with the rules set out for the interpretation of contract s. But
where the contract is embodied in writing, whether writing is required by law or the parties
themselves have stipulated writing, the Question becomes even more difficult. In terms of the parole
evidence rule, the written document is the only admissible evidence about its contents that is the
terms of the written agreement. No extrinsic evidence (this is evidence going beyond the written
document evidencing the contract) of other agreements may be adduced to indicate that the
agreement was different or to explain precisely what the parties intended.

STUDY UNIT 25-39: BREACH AND REMEDIES FOR BREACH


List the five forms of breach of contract

Answer
1. Mora Debitoris
2. Mora Creditoris
3. Malperformance
4. Repudiation
5. Prevention of Performance

Question
Problem type question on Mora
On 1 June M and Q conclude a contract whereby M undertakes to manufacture and install kitchen
cupboards in Q’s home for R50 000. The parties agree that the price will be paid as soon as the
kitchen cupboards are installed, but they do not determine a date for the completion of the work. M,
however, informs Q during the negotiations that she has some other work to complete and that she
will attend to the kitchen cupboards as soon as possible. Eight months has lapsed since the contract
was concluded and Q has not heard from M. Q runs out of patience and hires W to manufacture and
install the same kitchen cupboards for R60 000.
After W has completed the job, M turns up to do the work. Q claims R10 000 damages from M, but
M institutes a counterclaim for R30 000 from Q for her loss of profit. Who will succeed in this claim?
Discuss.

Answer
This question deals with damages for breach of contract. In order to determine who will succeed in
the claim for damages, we must ascertain which party committed the breach.
A plaintiff who wishes to claim damages for breach of contract must prove the following:
1. A breach of contract has been committed by the defendant
2. The plaintiff has suffered financial or patrimonial loss
3. There is a factual causal link between the breach and the loss
4. As a matter of legal causation, the loss is not too remote a consequence of the breach.

Did M or Q breach the contract?


M could possibly be in breach in the form of mora debitoris. Mora debitoris is the unjustifiable failure
of a debtor to make timeous performance of a positive obligation that is due and enforceable, and
still capable of performance in spite of such failure.

Positive malperformance: BK tooling


Question
Andy and Craig conclude a contract wherein Andy agrees to paint Craig’s office block by 31 August,
and Craig agrees to pay Andy R10 000 upon completion of the work. When 80% of the work is
completed Andy suddenly falls ill and he is unable to complete the job by 31 August. Craig refuses to
pay Andy any money for his (Andy’s) services rendered, as Craig believes that Andy has breached the
contract by not completing the work. Craig hires another contractor at an amount of R3000 to
complete the job.
Craig does not incur any other costs to complete the job, neither does his business make any losses.
Advise Andy as to what amount (if any) he may recover from Craig for the services that he rendered,
and on what basis. Discuss with reference to BK Tooling (Edms) Bpk v Scope Precision Engineering
(Edms) Bpk and other relevant case law. [15]

Answer
This contract is reciprocal in nature. Andy has rendered defective performance and the issue is
whether Craig has to compensate Andy for the work that has already been done.
This question deals with the exceptio non adimpleti contractus. The exceptio is a defence that can be
raised in the case of a reciprocal contract, where the performances due on either side are promised
in exchange for one another. It is a remedy that permits a party to withhold their performance and
ward off a claim for such performance until such time as the other party has either performed or
tendered performance of their obligations.
Where a party who has to perform first has only performed part of its obligations or has rendered
defective performance that party is in principle not entitled to claim counter-performance until such
time as he has performed in full. In practice, the innocent party often accepts part-performance and
starts using the performance. This sometimes leaves the breaching party in the unfair position that it
may be impractical or impossible to make full performance, but any claim for counter-performance
can be defended by the other party relying on the exceptio.
As a result, the courts have exercised discretion to relax the principle of reciprocity and order the
party making use of the defective or incomplete performance to pay a reduced amount to the party
in breach. In BK Tooling, the Appellate Division confirmed this, and held that the courts have an
equitable discretion to award a reduced contract price, depending on the nature of the defect, and
the cost of repair, replacement, or substitute performance. The onus to prove the amount of
reduction is on the party in breach claiming the reduced price. The plaintiff must allege and prove:
• that the other party is using his performance
• the cost of remedying defects
• that it would be equitable to award some remuneration despite breach
• that the circumstances are such that the court should exercise its discretion
Based on the ruling in BK Tooling, Andy is entitled to be compensated by Craig because:
1. Craig is utilising the defective performance
2. It would be equitable as Andy has completed most of the work
3. The counter-performance ought to be reduced by R3000 (the amount it cost to complete the job)
In the circumstances, Andy is entitled to receive R7000 from Craig, which represents the difference
between the contract price and the cost to complete the job.

Question
Damages
X contracts with Y for the latter (Y) to build and fit a security gate for the entrance of her (X’s) home.
Y builds the gate and fits it with an electric motor, which is activated with a remote control. X is
satisfied with the work and pays Y the contractual amount agreed upon. A week later, the gate gets
stuck while it is halfway open as a result of defective materials used to build the gate. When X
attempts to physically move the gate to close it fully, she suffers such severe damage to her left knee
that she has to have a knee operation. Her medical costs are R20 000. The costs of repairing the gate
amount to R15 000. X wants to claim both medical costs as well as the cost of repairing the gate from
Y. Advise X if she will be successful with her claim. Refer to Shatz Investments (Pty) Ltd v Kalovymas;
Holmdene Brickworks (Pty) Ltd v Roberts Construction Co, and other relevant case law in your
answer. [15]

Answer
This question deals with a claim for damages for breach of contract, and specifically, the element of
legal causation regarding special damages and general damages.
A plaintiff who wishes to claim damages for breach of contract must prove:
1. A breach of contract has been committed by the defendant
2. The plaintiff has suffered financial or patrimonial loss
3. There is a factual causal link between the breach and the loss
4. The loss is not too remote a consequence of the breach (legal causation).
Y has committed a breach of contract in the form of positive malperformance (the defective
materials used to build the gate).
In the law of contract, the approach to remoteness of consequences from breach (legal causation)
has been traditionally based on a distinction between general and special damages. The distinction
between general damages and special damages was stated in Holmdene Brickworks: general
damages are those damages that flow naturally and generally from the kind of breach in question
and which the law presumes the parties contemplated as a probable result of the breach; special
damages, on the other hand, are presumed to be too remote unless exceptional circumstances are
present. X may claim the cost of repairing the gate as general damages.
If X wants to succeed in the claim for medical costs as special damages, X must prove that:
1. The damages were actually foreseen or reasonably foreseeable at the time of entry into the
contract (the contemplation principle); and
2. The parties can be taken to have agreed that there would be liability for damages arising from
special circumstances (the convention principle).

Positive Malperformance:
This is performance of something which doesn’t comply with the terms of the contract or it is the
doing of something which the contracting party undertook not to do.

SU 27: REPUDIATION
Repudiation:
Conduct of guilty party must indicate a refusal to perform.

SU 28: PREVENTION OF PERFORMANCE


Prevention of Performance:
This takes place where performance is made impossible by a contracting party after conclusion of the
contract. It’s a form of anticipatory breach of contract since it can take place either before, on or
after the date set for performance.
Debtor = commits this where the object which must be performed is destroyed as a result of his
fault.
Creditor = commits this where performance becomes impossible as a result of his delay.
Absolute / Objective prevention of performance:
Where performance is prevented permanently & as regards everyone.
Relative / Subjective prevention of performance:
Where it is only performance by the debtor which is rendered impossible.

SU 29: REMEDIES ON THE GROUND OF BREACH OF CONTRACT


Remedies:
1. Specific performance
2. Exceptio non adimpleti contractus
3. Rescission (cancel)
4. Damages
Specific Performance:
Performance of that which the parties agreed to in the contract.

SU 30: THE EXCEPTIO NON ADIMPLETI CONTRACTUS


Exceptio non adimpleti contractus:
This is a defence, which occurs in the case of reciprocal contracts, that is contracts in which the
parties create obligations in terms of which the parties must either perform simultaneously or the 1
must perform before the other.

SU 31: RESCISSION (CANCEL)


Rescission of a contract:
This is a juristic act as a result of which the consequences of a valid contract are terminated. Where
an obligation is not fulfilled the obligation is extinguished when the contract is cancelled.

SU 32: DAMAGES
Damnum emergens (patrimonial damage – entire interest):
That is the amount actually lost the amount by which the person’s assets have been diminished
(actual damage).
Lucrum cessans (patrimonial damage – entire interest):
Loss of profit or prospective damage.
Factual causality:
There must be a causal connection between the breach & damage.
Conditio sine qua non test (But For Test) Factual Causation:
A certain result is caused by a certain act if that result would not normally have ensued but for such
act.
Legal Causality:
May the innocent party hold the other party liable for all the consequences of breach – public policy
& Policy considerations.
General Damages:
Are those damages which flow naturally & generally from the specific kind of breach that has been
committed, & the law presumes that the parties contemplated them as a probable result of the
breach.
Special Damages:
Are those damages which do not flow naturally & generally from the specific kind of breach.
Contemplation Principle:
In terms of this principle, the liability of the party who commits a breach of contract is limited to
those damages which can fairly be said to have actually been contemplated by the parties, or my
reasonably be supposed to have been contemplated by them as a probable consequence of a breach
of contract.
Convention Principle:
This principle limits the liability of the party who commits a breach of contract to those damages
which the innocent party can prove as having been agreed on between them, explicitly or
presumably. The parties must have contracted from the premise that such damages would be paid.

QUESTIONS:
State and distinguish the 5 types of breach of contract that occur in our law. (5)

1. Mora Debitoris
2. Mora Creditoris
3. Malperformance
4. Repudiation
5. Prevention of Performance

Question
S sells his house to P and the parties conclude a contract of sale that complies with formal
requirements. Thereafter, S receives a higher offer for the house from Z, which he also accepts and
the parties concludes a contract of sale that also complies with the formal requirements. Z is
unaware of the contract between S and P and sells his own house and proceeds to contract a
removal company to assist him with the move at a price of R20000. Z doesn’t work for 3 days during
the move. He earns a R1000 on average per day as an insurance broker. When Z arrives at the house
of S to take occupation, he finds that it is already occupied by P, advise Z fully. (10)
This falls under the concept of damages. Thus Z can sue for damages.

Answer
Requirements for Damages:
1. Breach of contract has occurred.
2. That innocent party has suffered damages
3. That there is a causal link between the breach & the damage
4. That the damages are general damages.

1. Test for Patrimonial Loss:


In case of breach of contract 1 compares the present value of innocent party’s estate with the value
it would have had, had contract been carried out properly & on time. If present value is less than it
would have been, damage has been suffered. Debtor must place creditor in same position he would
have been if the contract was complied with. This formula for damages is referred to as Positive
Interest.
1.1. Types of Patrimonial Damages which may be incurred:
Patrimonial loss is not only the diminution of a person’s assets but also the amount by which it might
have been enlarged. Patrimonial damage is referred to as the person’s entire interest in the contract.

Entire interest in contract includes:


1. Damnum emergens: That is the amount actually lost the amount by which the person’s assets
have been diminished (actual damage).
2. Lucrum cessans: Loss of profit or prospective damage.
These 2 doesn’t have much effect in assessing damages. What is to be assessed it the extent of the
damage, namely whether the estate of the injured party is smaller now than it would have been if
breach hasn’t occurred.

2. Causality:
2.1. Factual causality:
There must be a causal connection between the breach & damage.
Conditio sine qua non test (But For Test) Factual Causation:
A certain result is caused by a certain act if that result would not normally have ensued but for such
act.
2.2. Legal Causality:
May the innocent party hold the other party liable for all the consequences of breach – public policy
& Policy considerations.
After causality has been established the Question arises whether the innocent party may hold the
other party liable for all the consequences of the breach. For fairness for the guilty party, a line must
be drawn between damages caused by his breach & for which he’s liable, and damages which,
although caused by breach, are so remote from it that he shouldn’t be held liable for it.
2.2.1. General Damages:
Are those damages which flow naturally & generally from the specific kind of breach that has been
committed, & the law presumes that the parties contemplated them as a probable result of the
breach.
Party that commits breach is held liable, without further ado, for general damages.
Difference between general & special damages was concluded in the Holmdene Brickworks Case.
2.2.2. Special Damages:
Are those damages which do not flow naturally & generally from the specific kind of breach.
Party that commits breach will be liable for special damages in certain circumstances. To ascertain
the circumstances, 2 principles can be applied, namely the contemplation principle & the convention
principle.
2.2.2.1. Contemplation Principle:
In terms of this principle, the liability of the party who commits a breach of contract is limited to
those damages which can fairly be said to have actually been contemplated by the parties, or my
reasonably be supposed to have been contemplated by them as a probable consequence of a breach
of contract.
To establish what parties actually contemplated or may be supposed to have contemplated regard
may be had to the subject matter & terms of the contract itself, or the special circumstances known
to both parties at the time they contracted.
2.2.2.2. Convention Principle:
This principle limits the liability of the party who commits a breach of contract to those damages
which the innocent party can prove as having been agreed on between them, explicitly or
presumably. The parties must have contracted from the premise that such damages would be paid.
Thus innocent party has to prove either an express or an implied provision concerning the damages.
In terms of case law it appears that liability for special damages is ltd by means of the convention
principle – Shatz Investment Case.
Authors are of the opinion that it should be limited by the contemplation principle.

Duty to limit, prevent or mitigate damage or loss:

Duty of injured party to prevent or mitigate the damages. But 1 should only do what is reasonable.
Guilty party not liable for damages which injured party could have limited or mitigated by exercising
reasonable care. Onus is on party in breach to prove that the injured party didn’t act reasonably.
Reasonable expenditure to mitigate damage or loss is recoverable.
Z did incur damage. He incurred patrimonial damage. His estate is now smaller than it would have
been if breach didn’t occur. Damnum emergens is the loss of his house which he lived in & the
amount which he paid to move his contents of his old house. Lucrum cessans is the loss of 3 days
income because he didn’t work because he was preparing to move.
S is the Factual Causation of the damage of Z.
S is the Legal Causation of the damages of Z. The damages of selling his house & paying for the
moving of the contents of his house are General damages. Thus S will be held liable for the general
damages. The amount Z lost because he didn’t work for 3 days constitutes Special Damages. Thus S
not working, falls under the Contemplation Principle.
Y let premises to X. The lease contained a clause prohibiting X from subletting the premises without
written consent of Y. A further clause of the lease required any variation of the terms of the lease,
had to be in writing and signed by both parties. Assume that X and Y did sign a variation of the lease
allowing X to sublet a portion of the premises and thereafter Y changed his mind and prohibited X
from subletting. Discuss X’s position with reference to “Tuckers Land Development Case”. (10)
Repudiation:
Conduct of guilty party must indicate a refusal to perform.
Repudiation is anticipatory in the sense that it foreshadows or pre-empts another form of breach
being committed.

Test to be applied:
An objective test is applied. The Question is:
Whether the party renouncing has acted in such a way as to lead a reasonable person to the
conclusion that he doesn’t intend to fulfill his part of the contract.
3 Remedies:
1. Upholding or cancellation of contract: If cancelling the contract it depends on whether the
defective performance which is anticipated would justify rescission.
2. Specific Performance: This must be ordered before the fixed time arrives so that the wrongdoer
can be forced to compel with the contract.
3. Damages: Amount of damages is calculated with reference to the date on which performance was
due & not with reference to the date of the repudiation.
Tucker’s Land Case:
This case deals with Breach: Repudiation, anticipatory breach.
Here H bought 2 stands in a proposed township from T (the developer) numbered 95 & 97. Contract
of sale was subject to a Suspensive condition that the township be proclaimed. H had to make
certain payments before fulfillment of condition. T had difficulty in getting proclamation & prepared
a new plan on which H’s 2 stands disappeared. H became aware of this & sued T on basis that T
repudiated the contract, & claimed repayment of amounts already paid. H succeeded in his claim.

Judge Said:
Doctrine of anticipatory breach is of an existing obligation. Our law accepts that anticipatory breach
of contract, that is repudiation, constitutes infringement of an existing obligation. H was entitled to
cancel the whole contract & reclaim what he had paid.
In this case Y’s actions indicated objectively that he intended repudiation. Thus X may use the
remedy of Specific Performance to compel Y to comply with the contract.

Question
Write notes on:
The exceptio non adempleti contractus. (10)

Answer
Exceptio non adimpleti contractus:
This is a defence which occurs in the case of reciprocal contracts, that is contract s in which the
parties create obligations in terms of which the parties must either perform simultaneously or the 1
must perform before the other.
Where a party is sued for performance he (Defendant) may withhold performance until the claimant
has tendered proper performance or has performed fully, provided that the claimant has to perform
1st or simultaneously by raising the defence of the exceptio. The right to withhold performance is
referred to as the exceptio non adimpleti contractus. This is a form of Specific Performance.
This is only available as a defence when a reciprocal contract requires both parties to perform
simultaneously or requires the Plaintiff to perform before the Defendant. If at the time of the action
the Plaintiff’s duty to perform is not yet enforceable the Defendant can’t raise the exceptio.
Eg’s:
1. Cash sale
2. Credit sale
3. contract of lease
4. contract of mandate
5. contract of service
When this defence can be raised:
1. Where a Plaintiff has not performed at all: Defendant can raise this defence against the Plaintiff
who has not performed at all is fair & reasonable.
2. Where the Plaintiff has performed defectively & the contract is upheld: To allow Defendant who
has accepted & is using the Plaintiff’s performance to raise the exceptio against the Plaintiff’s claim
for counter performance might operate extremely unfairly against the Plaintiff. The Defendant will
have the benefit of the Plaintiff’s performance while the Plaintiff will receive nothing in return.

(2) Above creates a Problem & the Solution to the problem is found in BK Tooling Case:
Any contracting party has in principle a right to the Specific Performance undertaken by the other
party in other words he has a right to enforce the contract strictly according to its terms. Right of
party to a reciprocal contract to withhold his own performance until other party performs in full is a
powerful weapon to enforce full performance. In principle, a Defendant who has accepted the
Plaintiff’s defective reciprocal performance is still entitled to raise the exceptio non adimpleti
contractus against Plaintiff’s claim. Where fairness so requires, a court may, at its discretion, refuse to
allow a Defendant to raise the exceptio & order him to render a reduced counter performance.
Plaintiff in action based on reciprocal contract must where he upholds the contract have his
pleadings in order & he must:
1. Allege in his particulars of claim that he has rendered full performance from his side or he must
tender full performance.
If he’s unable to prove that he has indeed performed in full & wishes the court to exercise its
discretion in his favour by awarding him a reduced counter performance, he (Plaintiff) must allege &
prove:
1. That the Defendant is utilising (using) the defective performance.
2. That the circumstances exist which render it fair that the court should exercise its discretion in his
favour.
3. By how much the counter performance must be reduced. (Market value of the thing).
Where contract is cancelled: If the breach is sufficiently serious the innocent party may cancel the
contract. Parties must restore what has been received & innocent party then has an action for
damages.
Where a contract which has created an obligation to do something e.g., building a house, is cancelled
by innocent party restoration of what has been received is impossible. Thus the innocent party
derived some benefit from the other’s incomplete or defective performance, but as contract has
been cancelled the guilty party is now unable to claim a reduced counter performance, cancelled
contract can’t be enforced. The Plaintiff has a remedy for unjustified enrichment. He may recover in
terms of his action the amount by which the Defendant was actually enriched at his expense or the
amount by which he himself has been impoverished whichever is the smaller.

Question
Discuss the principle of reciprocity. (5)

Answer
Principle of Reciprocity:
Contract’s can be divided into unilateral & bilateral contract s. A contract is unilateral (contract of
donation) when 1 party only assumes a duty to perform, whilst it is bilateral (contract of sale) when
both parties assume such a duty.
Bilateral contract s is divided into reciprocal contract s (contract of sale & contract of lease) &
contract s that are not reciprocal (contract of loan).
Reciprocal contract’s is essentially aimed at accomplishing an exchange of performance - contract of
sale for e.g. is aimed at effecting the exchange of specific thing for specific sum of money - & the
implications of this characteristic of reciprocal contract’s is that there is no duty on 1 party to
perform unless the other party counter performs. This means that 1 party to this contract has a right
to either hold his performance until the other party performs unless of course he has agreed to
perform 1st.
The defence to which this right to withhold performance gives rise is known as the exceptio non
adimpleti contractus.

Question
Write notes on:
The court’s discretion to refuse an application for specific performance. (10)

Answer
Specific Performance:
Performance of that which the parties agreed to in the contract.
When Specific Performance will not be ordered:
Whenever Specific Performance is claimed by a party to the contract, court has a discretion to grant
or refuse the order. This discretion of the court isn’t confined to specific types of cases, nor is it
circumscribed by any rule. Each case must be judged in the light of its own circumstances.
Specific Performance will NOT be ordered in the following circumstances:
1. When Specific Performance has become impossible. (“Peters Case”).
2. Where it is impossible for the court to control Specific Performance.
3. Undue hardship
4. Inability to fulfill obligations.
5. Where it concerns the freedom of the individual.

Question
X and Y are involved in a protracted negotiation to conclude an intricate software contract; X spends
R8000 to do the required calculations. The parties are very close to agreement when Y suddenly
breaks off negotiations. Can X institute action against Y? Discuss. (4)

Answer
Yes X can institute action against Y for damages.
Requirements for Damages:
1. Breach of contract has occurred.
2. That innocent party has suffered damages
3. That there is a causal link between the breach & the damage
4. That the damages are general damages.
General Damages:
Are those damages which flow naturally & generally from the specific kind of breach that has been
committed, & the law presumes that the parties contemplated them as a probable result of the
breach.
Party that commits breach is held liable, without further ado, for general damages.
Discuss the concept of “Reasonable Time” with regards to mora debitoris. (10)
Mora debitoris:
This is when if the debtor neglects or fails to perform timeously in other words, when performance is
due, where performance remains possible in spite of such failure he is guilty of breach of contract.
Reasonable time falls under Mora Ex Persona which 1 of the 2 forms of Mora Debitoris.
Mora ex persona:
This occurs where no date for performance has been fixed.
Where no date for performance has been fixed the debtor must perform within a reasonable period
after the conclusion of the agreement. But if he fails to do so, he’s not automatically in mora. When
contract is silent about a time for performance, & debtor fails to perform within a reasonable time
the creditor must fix a time by making a demand (interpellatio) on the debtor to perform at a
specified time. When this time arrives & debtor fails to perform he falls in MORA EX PERSONA. Time
fixed in the demand must, however, leave debtor a reasonable period for performance, reasonable,
that is, taking into account the circumstances of which the parties were aware when the contract
was concluded or which they could reasonably have foreseen at that time. If time in the demand
isn’t reasonable, the demand is ineffective & a fresh demand has to be made by the creditor.
What will constitute a reasonable period?
Nel v Cloete:
In casu it was decided that in determining a reasonable period for performance, only circumstances
known to the parties at the time of contracting or reasonably foreseeable at the time can be taken
into account.
FACTS:
Contract concluded for purchase of house. Nel paid Cloete full Purchase Price. Cloete delayed
transfer because title deed was missing. He had to apply for a new 1. At time of contract neither of
the parties knew of the situation. Nel gave Cloete a 2 month period to sort everything out. Court
held that it was a reasonable period thus Nel could cancel contract.

Question
Write notes on the 2 principles, which can be applied to determine whether a party who commits
breach of contract will be liable to pay damages for special damages. (10)

Answer
Special Damages:
Are those damages which do not flow naturally & generally from the specific kind of breach.
Party that commits breach will be liable for special damages in certain circumstances. To ascertain
the circumstances, 2 principles can be applied, namely the contemplation principle & the convention
principle.
1. Contemplation Principle:
In terms of this principle, the liability of the party who commits a breach of contract is limited to
those damages which can fairly be said to have actually been contemplated by the parties, or my
reasonably be supposed to have been contemplated by them as a probable consequence of a breach
of contract.
To establish what parties actually contemplated or may be supposed to have contemplated regard
may be had to the subject matter & terms of the contract itself, or the special circumstances known
to both parties at the time they contracted.
2. Convention Principle:
This principle limits the liability of the party who commits a breach of contract to those damages
which the innocent party can prove as having been agreed on between them, explicitly or
presumably. The parties must have contracted from the premise that such damages would be paid.
Thus innocent party has to prove either an express or an implied provision concerning the damages.
In terms of case law it appears that liability for special damages is limited by means of the convention
principle – Shatz Investment Case.
Authors are of the opinion that it should be limited by the contemplation principle.

Question
Write notes on:
The difference between mora creditoris and prevention of performance. (5)

Answer
Mora Creditoris:
Creditor can also commit breach of contract by acting wrongfully. Where his co-operation is
necessary for the fulfillment of the obligation of the debtor, the creditor is guilty of breach if he fails
to co-operate timeously & performance remains possible.

Prevention of Performance:
This takes place where performance is made impossible by a contracting party after conclusion of the
contract. It’s a form of anticipatory breach of contract since it can take place either before, on or
after the date set for performance.
Debtor = commits this where the object which must be performed is destroyed as a result of his
fault.
Creditor = commits this where performance becomes impossible as a result of his delay.

Absolute / Objective prevention of performance:


Where performance is prevented permanently & as regards everyone.

Question
John is the owner of a vacant stand and he concludes a contract with a builder, Peter. In terms of
which Peter must build him a house on the stand according to a plan supplied by John. In terms of
the contract, Peter must complete the house by the 1st of July. Peter will be paid in full on the
completion of the building. Peter completes the building on the 1st of July but it transpires that the
house is 15 square meters smaller than the specifications required by the plan. Peter claims the full
contract price from John. Advise fully. (15)

Answer
This breach of contract is a form of Positive Malperformance.
Positive Malperformance:
This is performance of something which doesn’t comply with the terms of the contract or it is the
doing of something which the contracting party undertook not to do.
2 Forms of Positive Malperformance:
1. The debtor tenders faulty or defective performance.
2. Debtor does something he isn’t permitted to do in terms of the agreement.
Legal Remedies:
1. Creditor may retain the defective performance & sue for damages to compensate for the loss
caused by the defect.
2. He may reject the defective performance & claim proper performance.
3. He may reject the defective performance & claim damages from the other party as compensation.
4. He may resile from the contract if he has reserved to himself a right to resile OR if the breach of
contract is so serious that he can’t reasonably be expected to abide by the contract & be satisfied
with damages.
Here John has the remedy of the Exceptio Non Adimpleti contractus.
Exceptio non adimpleti contractus:
This is a defence which occurs in the case of reciprocal contracts, that is contracts in which the
parties create obligations in terms of which the parties must either perform simultaneously or the 1
must perform before the other.
Where a party is sued for performance he (Defendant) may withhold performance until the claimant
has tendered proper performance or has performed fully, provided that the claimant has to perform
1st or simultaneously by raising the defence of the exceptio. The right to withhold performance is
referred to as the exceptio non adimpleti contractus. This is a form of Specific Performance.
This is only available as a defence when a reciprocal contract requires both parties to perform
simultaneously or requires the Plaintiff to perform before the Defendant. If at the time of the action
the Plaintiff’s duty to perform is not yet enforceable the Defendant can’t raise the exceptio.

Question
What would the position be if John only discovered the defect in the house after he had paid Peter in
full. Advise Peter. (5)

Answer
John would be entitled to damages.
Requirements for damages:
1. Breach of contract has occurred.
2. That innocent party has suffered damages
3. That there is a causal link between the breach & the damage
4. That the damages are general damages.

General Damages:
Are those damages which flow naturally & generally from the specific kind of breach that has been
committed, & the law presumes that the parties contemplated them as a probable result of the
breach.
Party that commits breach is held liable, without further ado, for general damages.
Question
P buys a fridge from S for R500 on 18 October. P pays the purchase price immediately. They agree
furthermore that P will fetch the fridge on 20 October at S’s house. P fails to turn up at S’s house on
20 October. S is very glad because he has found another buyer who is prepared to pay R1000 and he
cancels the contract immediately. Was S entitled to cancel the contract and did S breach the contract
by doing so? Discuss. (10)

Answer
Since P failed to turn up, he was late & fault lies with him, which constitutes mora debitoris.
Mora Debitoris:
If the debtor fails to perform timeously in other words, when performance is due, where
performance remains possible in spite of such failure he is guilty of a breach of contract called Mora
Debitoris. He is then in mora. MD relates to the time of performance alone & not to the nature of
performance. For Mora Debitoris to arise the debt must be due & enforceable.
Mora Debitoris occurs:
When there is a delay in performance when the date of performance is fixed & that day has arrived.
2 Types:
1. Mora Ex Re: This occurs when a day for performance is fixed by the contract & the debtor fails to
perform on or before such day.
2. Mora Ex Persona: Occurs where no date for performance has been fixed.
S is now entitled to damages from P, because P was in Mora.
Requirements for damages:
1. Breach of contract has occurred.
2. That innocent party has suffered damages
3. That there is a causal link between the breach & the damage
4. That the damages are general damages.
General Damages:
Are those damages which flow naturally & generally from the specific kind of breach that has been
committed, & the law presumes that the parties contemplated them as a probable result of the
breach.
Party that commits breach is held liable, without further ado, for general damages. S could cancel.
Write notes on:

Question
The difference between general damages and special damages. (5)

General Damages:
Are those damages which flow naturally & generally from the specific kind of breach that has been
committed, & the law presumes that the parties contemplated them as a probable result of the
breach.
Party that commits breach is held liable, without further ado, for general damages. Difference
between general & special damages was concluded in the Holmdene Brickworks Case.
Special Damages:
Are those damages which do not flow naturally & generally from the specific kind of breach.
Party that commits breach will be liable for special damages in certain circumstances. To ascertain
the circumstances, 2 principles can be applied, namely the contemplation principle & the convention
principle.
Write notes on:

Question
Restitution by the party claiming rescission of a contract due to the other party’s breach of contract.
(5)

Answer
Rescission of a contract:
This is a juristic act as a result of which the consequences of a valid © are terminated. Where an
obligation is not fulfilled the obligation is extinguished when the contract is cancelled.
Impossibility of restitution:
If Party who can resile makes restitution of what he has received impossible he forfeited his right to
rescission. But he may resile if he is not to blame for inability to restore, provided that returns the
surrogate, if any, of what was to have been restored. Party who wishes to resile may do so in certain
circumstances where the return of the performance which he received has become impossible
through no fault on his part. This principal also applies if the innocent party resiles because of the
other party’s breach & can’t thereafter make restitution of what he has received under the contract.

Question
Special damages for breach of contract. (5)

Answer
Special Damages:
Are those damages which do not flow naturally & generally from the specific kind of breach.
Party that commits breach will be liable for special damages in certain circumstances. To ascertain
the circumstances, 2 principles can be applied, namely the contemplation principle & the convention
principle.
1. Contemplation Principle:
In terms of this principle, the liability of the party who commits a breach of contract is limited to
those damages which can fairly be said to have actually been contemplated by the parties, or my
reasonably be supposed to have been contemplated by them as a probable consequence of a breach
of contract.
To establish what parties actually contemplated or may be supposed to have contemplated regard
may be had to the subject matter & terms of the contract itself, or the special circumstances known
to both parties at the time they contracted.
2. Convention Principle:
This principle limits the liability of the party who commits a breach of contract to those damages
which the innocent party can prove as having been agreed on between them, explicitly or
presumably. The parties must have contracted from the premise that such damages would be paid.
Thus innocent party has to prove either an express or an implied provision concerning the damages.
In terms of case law it appears that liability for special damages is limited by means of the convention
principle – Shatz Investment Case. Authors are of the opinion that it should be limited by the
contemplation principle.
B builds a swimming pool for O. Parties agree that the pool must be completed on 27 July 2006.
Contract price is R20000 and O pays R5000 upon signing the agreement. On 27 July B and O
inspected the pool, but O isn’t satisfied. Paving around the pool is covered in cement splotches and
pool requires another layer of paint. O is of the opinion that B has committed breach of contract. He
refuses to pay B a cent further and forbids B to set foot on the premises again.

(A) Did B commit breach of contract? Substantiate answer. (4)

Yes, because…
Positive Malperformance:
This is performance of something which doesn’t comply with the terms of the contract or it is the
doing of something which the contracting party undertook not to do.
2 Forms of Positive Malperformance:
1. The debtor tenders faulty or defective performance. (This 1 applies here)
2. Debtor does something he isn’t permitted to do in terms of the agreement.

Can O cancel the agreement? Substantiate answer. (2)

Yes, because…
Legal Remedies:
1. Creditor may retain the defective performance & sue for damages to compensate for the loss
caused by the defect.
2. He may reject the defective performance & claim proper performance.
3. He may reject the defective performance & claim damages from the other party as compensation.
4. He may resile from the contract if he has reserved to himself a right to resile OR if the breach of
contract is so serious that he can’t reasonably be expected to abide by the contract & be satisfied
with damages.

Did O commit breach of contract if you assume that O may not validly cancel the agreement?
Substantiate your answer. (2)

What remedy/s does O have at his disposal Where B institutes action for the balance of the purchase
price and the agreement isn’t cancelled? Substantiate answer. (6)

O has the remedy of damages to his disposal.


Requirements for damages:
1. Breach of contract has occurred.
2. That innocent party has suffered damages
3. That there is a causal link between the breach & the damage
4. That the damages are general damages.
General Damages:
Are those damages which flow naturally & generally from the specific kind of breach that has been
committed, & the law presumes that the parties contemplated them as a probable result of the
breach.
Party that commits breach is held liable, without further ado, for general damages.
Sibonki concludes an oral contract of employment with John in terms of which Sibonki employs John
as a waiter in her restaurant at a monthly salary of R2000. No other terms are specifically agreed.

(A) After working in the restaurant for 2 weeks John approaches Sibonki and orally demands R1000
from her as wages for 2 weeks. Sibonki refuses to pay and an irate John storms out of the restaurant
declaring that he will never again work for her.

1) Did John breach the © by demanding payment of R1000? If your answer is yes, what type of
breach of © did John commit? Discuss. (2)

Yes, because…
Positive Malperformance:
This is performance of something which doesn’t comply with the terms of the contract or it is the
doing of something which the contracting party undertook not to do.

2) Did Sibonki breach the contract by refusing to pay John the R1000? If your answer is yes, what
type of breach of contract did she commit? Discuss. (2)

Yes she did commit breach because she clearly indicated that she wasn’t going to perform.
Repudiation:
Conduct of guilty party must indicate a refusal to perform.
Repudiation is anticipatory in the sense that it foreshadows or pre-empts another form of breach
being committed.
Test to be applied:
An objective test is applied. The Question is:
Whether the party renouncing has acted in such a way as to lead a reasonable person to the
conclusion that he doesn’t intend to fulfill his part of the contract.

(B) As a result of John’s dramatic departure, Sibonki considers the contract cancelled and she hires
another waiter the very same day to take John’s place.

1) Assuming that Sibonki didn’t breach the contract by refusing to pay John, did she breach the
contract by cancelling it after John’s departure? If answer yes, what type of breach of contract did
she commit? (4)

Notice of Rescission:
Time can be made of the essence by the debtor’s being given a notice of rescission. Creditor must
take care that this notice is clear & unequivocal.
Sweet Case: Applicant sent a notice but since there was no mora it wasn’t possible to obtain a right
of rescission by sending a notice. The notice in this case was totally irrelevant & without effect.
Notice of rescission like a demand (interpellatio) must leave the debtor a reasonable time in which to
perform. Period that has already expired is taken into account in respect of the further reasonable
period which the creditor grants the debtor, in the notice of rescission.
Where a demand is 1st sent (to place debtor in mora) & is later followed by a notice of rescission (to
acquire a right to resile) debtor must twice be given a reasonable time in which to perform. Demand
& notice of rescission may be combined in 1 document, in which case 1 reasonable period serves for
both notices (Nel v Cloete).
Sibonki didn’t create a right to cancel the contract by means of a notice of rescission, thus she
committed breach.

2) Assuming that Sibonki validly cancelled the contract; may John still claim 2 weeks wages from her?
(1)

Yes, because of undue enrichment…


Where contract is cancelled: If the breach is sufficiently serious the innocent party may cancel the
contract.
Parties must restore what has been received & innocent party then has an action for damages.
Where a contract which has created an obligation to do something e.g., building a house, is cancelled
by innocent party restoration of what has been received is impossible. Thus the innocent party
derived some benefit from the other’s incomplete or defective performance, but as contract has
been cancelled the guilty party is now unable to claim a reduced counter performance, cancelled
contract can’t be enforced.
The Plaintiff has a remedy for unjustified enrichment. He may recover in terms of his action the
amount by which the Defendant was actually enriched at his expense or the amount by which he
himself has been impoverished whichever is the smaller.

3) Assuming that Sibonki validly cancelled the contract may she claim damages from John? (1)

Yes because her situation complies with the requirements for Damages.
Requirements for damages:
1. Breach of contract has occurred.
2. That innocent party has suffered damages
3. That there is a causal link between the breach & the damage
4. That the damages are general damages.

(c) John returns the next day to resume work but Sibonki has him removed from the premises.

1) Assuming that Sibonki couldn’t validly cancel the contract, did she breach the contract by
preventing John continuing work in the restaurant the next day? If answer yes, what type of breach
did she commit. (4)

Yes she committed breach in the form of prevention of performance.


Prevention of Performance:
This takes place where performance is made impossible by a contracting party after conclusion of the
contract. It’s a form of anticipatory breach of contract since it can take place either before, on or
after the date set for performance. Debtor = commits this where the object which must be
performed is destroyed as a result of his fault.
Creditor = commits this where performance becomes impossible as a result of his delay.
Thus she prevented John from performing his work.

2) Assuming that Sibonki breached the contract of service by having John removed; can John claim 2
weeks wages from her? Don’t discuss damages. (4)

Yes, because of undue enrichment…


Where contract is cancelled: If the breach is sufficiently serious the innocent party may cancel the
contract.
Parties must restore what has been received & innocent party then has an action for damages.
Where a contract which has created an obligation to do something e.g., building a house, is cancelled
by innocent party restoration of what has been received is impossible. Thus the innocent party
derived some benefit from the other’s incomplete or defective performance, but as contract has
been cancelled the guilty party is now unable to claim a reduced counter performance, cancelled
contract can’t be enforced. The Plaintiff has a remedy for unjustified enrichment. He may recover in
terms of this action the amount by which the Defendant was actually enriched at his expense or the
amount by which he himself has been impoverished whichever is the smaller.

3) Assuming that Sibonki breached the contract of service by having John removed; can John claim to
be reinstated in his work? Discuss. (4)

No because…this falls under Specific Performance.


Specific Performance: Performance of that which the parties agreed to in the contract.
When Specific Performance will not be ordered:
Specific Performance will NOT be ordered in the following circumstances:
1. When Specific Performance has become impossible. (“Peters Case”).
2. Where it is impossible for the court to control Specific Performance.
3. Undue hardship
4. Inability to fulfill obligations.
5. Where it concerns the freedom of the individual.

Here Sibonki’s freedom will be ltd if John is reinstated in his position at the restaurant.

Jude sells a new navigation system to Peter for R200000 and undertakes to install it on Peter’s
trawler. Jude warrants that the system complies with a number of specifications. Month after Jude
has installed system and Peter has paid Jude the R200000 the trawler is lost at sea during a storm. 3
years later, Peter finds out that the system didn’t comply with most of the specifications. Peter
cancels the contract in writing. Letter of rescission is delivered to Jude who never reads the letter
because the letter is lost amongst the correspondence on his untidy desk. Jude refuses to pay Peter
back the R200000.
(A)Jude alleges that the contract hasn’t been validly cancelled. Discuss the following reasons raised
by Jude:

1) Peter had no right to cancel the contract (don’t discuss latent defects or the remedies thereof). (3)

Mora Debitoris:
Notice of Rescission:
Time can be made of the essence by the debtor’s being given a notice of rescission. Creditor must
take care that this notice is clear & unequivocal.
Sweets Case: Applicant sent a notice but since there was no mora it wasn’t possible to obtain a right
of rescission by sending a notice. The notice in this case was totally irrelevant & without effect.
Notice of rescission like a demand (interpellatio) must leave the debtor a reasonable time in which to
perform. Period that has already expired is taken into account in respect of the further reasonable
period which the creditor grants the debtor, in the notice of rescission.
Where a demand is 1st sent (to place debtor in mora) & is later followed by a notice of rescission (to
acquire a right to resile) debtor must twice be given a reasonable time in which to perform. Demand
& notice of rescission may be combined in 1 document, in which case 1 reasonable period serves for
both notices (Nel v Cloete).
Thus Peter had a right to cancel the contract.

2) More than 3 years have lapsed since the installation of the navigation system before the contract
was cancelled. (2)

The lapse of a right of cancellation:


The right of rescission naturally lapses when the innocent party elects not to exercise it. The test is
whether a reasonable person would deduce that the right to rescission has been waived. Innocent
party doesn’t lose his right of cancellation through mere delay, even if a reasonable time has elapsed.

3) Peter can’t return the navigation system as the ship was lost at sea. (3)

Rescission of a contract:
This is a juristic act as a result of which the consequences of a valid contract are terminated. Where
an obligation is not fulfilled the obligation is extinguished when the contract is cancelled.
Impossibility of restitution:
If Party who can resile makes restitution of what he has received impossible he forfeited his right to
rescission. But he may resile if he is not to blame for inability to restore, provided that returns the
surrogate, if any, of what was to have been restored. Party who wishes to resile may do so in certain
circumstances where the return of the performance which he received has become impossible
through no fault on his part. This principal also applies if the innocent party resiles because of the
other party’s breach & can’t thereafter make restitution of what he has received under the contract.
Here the impossibility of the restitution of the navigation system wasn’t the fault of Peter thus he has
the right to cancel the contract.

4) Jude never read the letter cancelling the contract. (1)


How the innocent party exercises his right of cancellation:
He must notify the other party by means of a summons, orally or in writing. The rescission according
to the courts decision takes effect as soon as the guilty party obtains actual notice of the rescission
(Swart v Vosloo).
Thus Jude didn’t actually obtain the knowledge of the cancellation of the contract.

(B) Suppose trawler runs around because navigation system doesn’t comply with the specifications
and trawler damaged by the waves. Fortunately trawler can be salvaged at cost of R100000. Will cost
a further R50000 to repair damage to trawler. Peter wants to claim R150000 from Jude. Discuss
Jude’s defence that the damage is too remote from the breach of warranty. (3)

Jude’s defence is that of Special Damages.


Special Damages:
Are those damages which do not flow naturally & generally from the specific kind of breach.
Party that commits breach will be liable for special damages in certain circumstances. To ascertain
the circumstances, 2 principles can be applied, namely the contemplation principle & the convention
principle.
1. Contemplation Principle:
In terms of this principle, the liability of the party who commits a breach of contract is limited to
those damages which can fairly be said to have actually been contemplated by the parties, or my
reasonably be supposed to have been contemplated by them as a probable consequence of a breach
of contract.
To establish what parties actually contemplated or may be supposed to have contemplated regard
may be had to the subject matter & terms of the contract itself, or the special circumstances known
to both parties at the time they contracted.
Because Jude warranted that the navigation system complies with a number of specifications it could
in terms of the contemplation principle be said that damages would be payable in the case of a
default.

Question
K concludes a written contract with L in terms of which L will manufacture cricket hats to be sold
during the 2003 cricket world cup in South Africa. K has the exclusive right to sell such hats from the
cricket authorities at a cost of R100000. But K expects to make a profit of about R1000000 from the
sale of the hats. The contract between K and L doesn’t state the date on which L must deliver the
hats to K. but the hats clearly refer to the cricket world cup 2003 and the dates of the matches are
constantly announced in the media. At the time of the commencement of the world cup matches L
hasn’t delivered any hats to K. The cricket authorities immediately revoke K’s exclusive right to sell
cricket hats because he failed to comply with the terms of the agreement with the cricket
authorities. The contract between K and the cricket authorities also determines that upon
cancellation of the contract for any reason, K will forfeit the R100000 that he has paid for the right to
sell cricket hats. Thereafter L informs K that he is ready to deliver the hats to K. K refuses to take
delivery on the basis that L breached the contract. Discuss the legal position of both K and L with
reference to case law. (15)

Answer
Mora debitoris:
This is when if the debtor neglects or fails to perform timeously in other words, when performance is
due, where performance remains possible in spite of such failure he is guilty of breach of contract.
There are 2 forms of mora debitoris, namely Mora Ex Re & Mora Ex Persona. This situation complies
with mora ex re.
Mora ex re:
This occurs if a day for performance is fixed by the contract & the debtor fails to perform on or
before such day.
The date is not fixed per se, but the debtor knows exactly when the world cup is because it is
advertised every day on the TV, radio, etc.
Cancellation of a contract on the basis of mora debitoris is allowed when time is of the essence of
the contract, & time is of the essence in any of the following circumstances:
1. When the contract contains a cancellation clause
2. Where there is a tacit cancellation clause
3. Where the creditor has given the debtor a notice of rescission.
Here K gave L a tacit cancellation of the contract.
Tacit cancellation clause:
Thus, here, “time is of the essence” where all the circumstances indicate that the parties did intend
time to be of the essence regarding the contract & tacit cancellation clause. This is the case if the
nature of the agreement is such that a failure to perform timeously would justify the cancellation of
the contract. (Goldstein Case).
K is now entitled to damages.
Requirements for damages:
1. Breach of contract has occurred.
2. That innocent party has suffered damages
3. That there is a causal link between the breach & the damage
4. That the damages are general damages.

1. Test for Patrimonial Loss:


In case of breach of contract 1 compares the present value of innocent party’s estate with the value
it would have had, had contract been carried out properly & on time. If present value is less than it
would have been, damage has been suffered. Debtor must place creditor in same position he would
have been if the contract was complied with. This formula for damages is referred to as Positive
Interest.
1.1. Types of Patrimonial Damages which may be incurred:
Patrimonial loss is not only the diminution of a person’s assets but also the amount by which it might
have been enlarged. Patrimonial damage is referred to as the person’s entire interest in the contract.
Entire interest in contract includes:
1. Damnum emergens: That is the amount actually lost the amount by which the person’s assets
have been diminished (actual damage).
2. Lucrum cessans: Loss of profit or prospective damage.
These 2 doesn’t have much effect in assessing damages. What is to be assessed it the extent of the
damage, namely whether the estate of the injured party is smaller now than it would have been if
breach hasn’t occurred.
2. Causality:
2.1. Factual causality:
There must be a causal connection between the breach & damage.
Conditio sine qua non test (But For Test) Factual Causality:
A certain result is caused by a certain act if that result would not normally have ensued but for such
act.
2.2. Legal Causality:
May the innocent party hold the other party liable for all the consequences of breach – public policy
& Policy considerations.
After causality has been established the Question arises whether the innocent party may hold the
other party liable for all the consequences of the breach. For fairness for the guilty party, a line must
be drawn between damages caused by his breach & for which he’s liable, and damages which,
although caused by breach, are so remote from it that he shouldn’t be held liable for it.
2.2.1. General Damages:
Are those damages which flow naturally & generally from the specific kind of breach that has been
committed, & the law presumes that the parties contemplated them as a probable result of the
breach.
Party that commits breach is held liable, without further ado, for general damages. Difference
between general & special damages was concluded in the Holmdene Brickworks Case.
2.2.2. Special Damages:
Are those damages which do not flow naturally & generally from the specific kind of breach.
Party that commits breach will be liable for special damages in certain circumstances. To ascertain
the circumstances, 2 principles can be applied, namely the contemplation principle & the convention
principle.
3. Duty to limit, prevent or mitigate damage or loss:
Duty of injured party to prevent or mitigate the damages. But 1 should only do what is reasonable.
Guilty party not liable for damages which injured party could have limited or mitigated by exercising
reasonable care. Onus is on party in breach to prove that the injured party didn’t act reasonably.
Reasonable expenditure to mitigate damage or loss is recoverable.
Here K’s damages is that of general damages & thus he is entitled to damages from L

SU 33, 34 & 35

SU 33: PENALTY CLAUSES


QUESTIONS:
Write notes on:

Question
The reduction of a penalty amount in a contract (5)

Answer
The penalty is not necessarily enforceable to its full extent, since Sec 3 give the court the power to
reduce the penalty amount to what it deems “equitable”.
The person who bears the onus of proving the excessiveness or otherwise of the penalty stipulation:
The onus is on the debtor to prove that the penalty is out of proportion to the prejudice suffered by
the creditor & that it should consequently be reduced. Once the debtor has made out a prima facie
case that the penalty is excessive, there is an onus on the creditor to lead evidence in rebuttal of the
debtor’s prima facie case.
Reduction comes into Question only when it is apparent to the court that the penalty is out of
proportion to the prejudiced suffered by the innocent party.

SU 34: THE EXCEPTIO DOLI


QUESTIONS:
Write notes on:

Question
Application of the exceptio doli (5)

Answer
Definition:
The exceptio doli is a defence introduced in about 200 BC which could be raised by a Defendant if the
Plaintiff had acted contrary to the requirements of good faith at the moment the contract was
entered into, or at the moment of enforcing the action. This made the exceptio doli the means of
ensuring that an equitable decision was reached. The exceptio doli brought the requirements of
good faith into the law of contract.
Application:
In the Bank of Lisbon Case the Question arose whether the exceptio doli could be raised as a defence
in order to obtain an equitable decision. The Appellant Division came to the conclusion that the
exceptio doli didn’t form part of South African Law.

SU 35: THE TRANSFER OF CLAIMS: CESSION


DEFINITIONS:
Cession:
Every estate consists of rights & obligations, & to a large extent a person’s economic activities involve
the transfer of his rights to others. A claim (a personal right) is just as much an asset in one’s estate
as for e.g. a right of ownership to a car. For this reason there is a need for the passing of such assets
to other persons, for instance, on the holder’s death, insolvency, etc. In such cases the claim goes
over to the heir, trustee, etc by operation of law. But it may be of interest to the holder to transfer his
claim to somebody else by means of a juristic act. The juristic act can effect such a transfer of a
personal right (or claim) is known as a cession. Cession is the transfer of a personal right by means of
an agreement.

QUESTIONS:
Write notes on:
Security cession (4)
Cession is the transfer of a personal right by means of an agreement.
Cession may also be used as a form of security.
E.g.: A owes B a R100. B in his turn owes C R80. As security for the latter debt B cedes to C his claim
against A with this proviso that C should re-cede to him, B, the claim against A as soon as he pays
back the R80. Should B fail to settle his debt to C, C is entitled to enforce the ceded claim against A.
The transaction between B & C is known as an out-and-out security cession in securitatum debiti. A
security cession can also be in the form of a pledge. The dominium of the right is restrained by the
pledgor (cedent) while only the quasi-possession is transferred to the pledgee (cessionary). The
pledgee can realise (sell) the right to enforce it against the debtor when the pledgor fails to pay the
debt he owes the pledgee.

SU 36, 37 & 38:


12 WAYS OF TERMINATING YOUR OBLIGATIONS:
1. Discharge
2. Release
3. Novation
4. Delegation
5. Settlement
6. Set-Off
7. Merger
8. Prescription
9. Supervening Impossibility of Performance
10. Judgment
11. Insolvency & Subsequent Rehabilitation
12. Rescission, Termination & Cancellation Clause

SU 36: THE TERMINATION OF OBLIGATIONS


DEFINITIONS:
Discharge:
This is the normal manner in which obligations are terminated.

Release:
This is an agreement between the creditor & debtor according to which the creditor releases the
debtor from his obligations under their CONTRACT.

Novation (Novatio):
This is an agreement between a creditor & a debtor, to an existing obligation whereby the old debt
between them is extinguished & a new obligation is created in the place of the old one.
Delegation:
Here both rights & duties are transferred by agreement.

Settlement / Compromise (Transactio):


This is an agreement whereby the parties terminate a dispute between them. For a settlement to be
valid, it is not necessary that a valid old debt should have been in existence. Should a previous debt
exist, however, it is extinguished by the settlement.

SU 37: THE TERMINATION OF OBLIGATIONS


DEFINITIONS:
Set-Off (Compensatio):
This is the extinction of debts owed reciprocally by 2 parties.

SU 38: THE TERMINATION OF OBLIGATIONS


DEFINITIONS:
Merger (Confusio):
A confusio is “the occurance in the same person of the capacities of creditor & debtor in respect of
the same obligation”.

Prescription:
Contract terminates after 3 years has lapsed.

Supervening Impossibility of Performance:


If performance becomes impossible after conclusion of the contract, the obligation is terminated.
The object of the claim (personal right), that is the performance, has been extinguished & with that
the claim is terminated.

Vis maior:
This means “some force, power or agency which can’t be resisted or controlled by the ordinary
individual. This term is now used as including not only acts of nature, vis divina or acts of God, but
also the act of man.

Casus fortuitus:
This is “a species of vis maior & imports something exceptional, extraordinary or unforeseen, &
which human foresight can’t be expected to anticipate, or if it can be foreseen, it can’t be avoided by
the exercise of reasonable care or caution”. E.g.: Legislation.

Judgment:
This doesn’t substitute 1 obligation for another, but rather confirms or stabilises an existing
obligation.
Insolvency & Subsequent Rehabilitation:
This doesn’t always terminate the contract; it is up to the Trustee of the insolvent estate according to
the Insolvency Act.

Rescission, Termination & Cancellation Clause:


(Rescission is misrepresentation & Cancellation Clause is breach.)
The obligation is also terminated by cancellation by virtue of breach of contract or a cancellation
clause & by avoiding a voidable contract on some recognised ground.
Likewise, an obligation is terminated by a Resolutive time clause, on fulfillment of a Resolutive
condition or on the non-fulfillment of a suspensive condition.

QUESTIONS:
Write notes on: Novation (5)

Novation (Novatio):
This is an agreement between a creditor & a debtor, to an existing obligation whereby the old debt
between them is extinguished & a new obligation is created in the place of the old one.
E.g.: A has to deliver a horse to B. Parties reach a new agreement that A will deliver a cow instead of
a horse to B. Original agreement is extinguished by the new agreement.

Write notes on: Set-off (5)

Set-Off (Compensatio):
This is the extinction of debts owed reciprocally by 2 parties.
EG: A owes B R100 & B owes A R80. The R80 can be set of against the R100 which means A owes B
R20 after set-off.

There are 4 requirements for set-off:


1. The debt must be similar in nature: 2 monetary debts may set each other off, 2 debts for delivery
of the same kinds of goods may set each other off.
2. Liquidated debts: A liquidated debt is an amount which is fixed or can readily be fixed.
3. The debts must be due: A debt which is already due can’t be extinguished by one which is not yet
due.
4. Debts between the same persons & persons in the same capacity: Debts must be reciprocal, that
is, they must exist between the same parties in the same capacities. E.g.: set-off a debt owed to each
other in their personal capacities.

Question
Discuss “Peters, Flamman & Co v Kokstad Municipality” with regards to supervening impossibility of
performance (5)

Answer
Supervening Impossibility of Performance:
If performance becomes impossible after conclusion of the contract, the obligation is terminated.
The object of the claim (personal right), that is the performance, has been extinguished & with that
the claim is terminated.
The General Principle that a debtor is released & excused where performance becomes impossible
through no fault of his own is often stated differently, namely that the debtor is released where
performance is prevented by vis maior or casus fortuitus.

Vis maior:
This means “some force, power or agency which can’t be resisted or controlled by the ordinary
individual. This term is now used as including not only acts of nature, vis divina or acts of God, but
also the act of man.

Casus fortuitus:
This is “a species of vis maior & imports something exceptional, extraordinary or unforeseen, &
which human foresight can’t be expected to anticipate, or if it can be foreseen, it can’t be avoided by
the exercise of reasonable care or caution”. E.g.: Legislation.

Peters, Flamman & CO Case:


Facts:
Before World War 1 P & CO contracted with K municipalities to light their streets for a period of
20yrs. 1915 WW1 broke out & it became impossible for P & CO to perform their obligations. 1917
their partnership was wound up. K Municipalities sued for damages for breach of contract.
Held:
As soon as it has became impossible for P & CO to perform in terms of the ©, they were discharged
from liability.
The Judge said it became impossible for them to perform by virtue of an Act of State (casus fortuitus)
& thus their obligations under the contract were terminated.

Distinguish between delegation & cession (4)

Delegation:
Here both rights & duties are transferred by agreement.
(Now the parties change).
E.g.: A and B enter into a contract, A approaches B and asks whether C may take his place, if B agrees,
C takes over all A’s rights and obligations against B. Consent of all the parties is necessary. When C
takes A’s place, the obligations between A and B is terminated.

Cession:
Every estate consists of rights & obligations, & to a large extent a person’s economic activities involve
the transfer of his rights to others. A claim (a personal right) is just as much an asset in one’s estate
as for e.g. a right of ownership to a car. For this reason there is a need for the passing of such assets
to other persons, for instance, on the holder’s death, insolvency, etc. In such cases the claim goes
over to the heir, trustee, etc by operation of law. But it may be of interest to the holder to transfer his
claim to somebody else by means of a juristic act. The juristic act can effect such a transfer of a
personal right (or claim) is known as a cession. Cession is the transfer of a personal right by means of
an agreement.

Question
Indicate whether a valid & enforceable contract has been concluded in following instance:
John & Jack agree that Jack will man 1 of john’s hotdog stands at Ellis Park Rugby stadium during a
test match at a remuneration of R500. But structural defects are discovered in the stands at Ellis Park
& as a result the test match is cancelled. (3)

Answer
Yes a valid contract came into being, but due to a vis maior the obligations of both parties to the
agreement was terminated. No one can sue for breach of contract.
Supervening Impossibility of Performance:
If performance becomes impossible after conclusion of the contract, the obligation is terminated.
The object of the claim (personal right), that is the performance, has been extinguished & with that
the claim is terminated.

Vis maior:
This means “some force, power or agency which can’t be resisted or controlled by the ordinary
individual. This term is now used as including not only acts of nature, vis divina or acts of God, but
also the act of man.

Casus fortuitus:
This is “a species of vis maior & imports something exceptional, extraordinary or unforeseen, &
which human foresight can’t be expected to anticipate, or if it can be foreseen, it can’t be avoided by
the exercise of reasonable care or caution”. E.g.: Legislation.

Question
Distinguish between initial impossibility of performance, prevention of performance & supervening
impossibility of performance. (10)

Answer
1. Initial Impossibility of Performance:
This is Absolute or Objective impossibility.
Objective Impossibility:
Performance may be impossible for everyone, in which case we speak of absolute or objective
impossibility.
If performance is impossible at the time of contracting (initial), no obligations results with regards to
that performance. The contemplated obligation is in other words void. The obligation relating to the
counter-performance will also be void even if it is possible, which means that the contract is void.
2. Prevention of Performance:
This takes place where performance is made impossible by a contracting party after conclusion of the
contract. It’s a form of anticipatory breach of contract since it can take place either before, on or
after the date set for performance.
Debtor = commits this where the object which must be performed is destroyed as a result of his
fault.
Creditor = commits this where performance becomes impossible as a result of his delay.
3. Supervening Impossibility of Performance:
If performance becomes impossible after conclusion of the contract, the obligation is terminated.
The object of the claim (personal right), that is the performance, has been extinguished & with that
the claim is terminated.

Question
Write notes on: The distinction between in solatum datio, settlement & novation (6)

Answer
Settlement / Compromise (Transactio):
This is an agreement whereby the parties terminate a dispute between them. For a settlement to be
valid, it is not necessary that a valid old debt should have been in existence. Should a previous debt
exist, however, it is extinguished by the settlement.

Novation (Novatio):
This is an agreement between a creditor & a debtor, to an existing obligation whereby the old debt
between them is extinguished & a new obligation is created in the place of the old one.
For the difference between novation & in solatium datio (tendering in payment something other
than that which was due).

Question
E & O agree that E will work at 1 of O’s hotdog stalls at the FNB stadium during the variation games
of the soccer world cup at remuneration of R2000 per day. Since E who has been unemployed for 6
months is desperate for money, O guarantees that E will be given this job. But structural defects are
discovered in the stands at the FNB stadium & as a result all the games scheduled to be played there
are cancelled. Advise E. (6)

Answer
A valid contract came into being between O & E but due to a vis maior the obligations of both parties
to the agreement was terminated. No one can sue for breach of contract.
Supervening Impossibility of Performance:
If performance becomes impossible after conclusion of the contract, the obligation is terminated.
The object of the claim (personal right), that is the performance, has been extinguished & with that
the claim is terminated.
Vis maior:
This means “some force, power or agency which can’t be resisted or controlled by the ordinary
individual. This term is now used as including not only acts of nature, vis divina or acts of God, but
also the act of man.
Casus fortuitus:
This is “a species of vis maior & imports something exceptional, extraordinary or unforeseen, &
which human foresight can’t be expected to anticipate, or if it can be foreseen, it can’t be avoided by
the exercise of reasonable care or caution”. E.g.: Legislation.

PVL3702_201_2019 – Second Semester

Read the judgments in Bloom v American Swiss Watch Co 1915 AD 100; Laws v Rutherfurd 1924 AD
261 and The Fern Gold Mining Company v Tobias (1889-1890) 3 SAR TS 134. Identify and read the
relevant section(s) in the prescribed textbook. You can find judgments in conventional law libraries,
online at the website of the Southern African Legal Information Institute (SAFLII) (www.saflii.org) or
as an e-resource on the Unisa Library site (choose Juta Law Online Publications and then South
African Law Reports). Then use those cases and the textbook to answer the following question:
In a cash-in-transit heist, a gang of robbers attacks a van of XYZ Security in broad daylight on 10
January and robs R3.5 million in cash. The next day, XYZ Security, a well-known security company
offers a reward of R100 000 on the TV news to anyone who gives information to the police leading to
the arrest of the robbers. H watches the news and notices that a reward is being offered. On 14
January, XYZ Security withdraws the reward, which is reported on the TV news. H does not watch the
news and fails to hear about the revocation of the reward. On 15 January, H provides the police with
information, which indeed leads to the arrest of the robbers that very night. On 17 January, XYZ
Security hears that the information has been provided to the police. XYZ refuses to pay H the reward.
Advise H fully. Substantiate your advice and refer to relevant case law. (10)

Answer

In order to identify the law, which you should apply to the problem, you were asked to:
(1) read Bloom v American Swiss Watch Co 1915 AD 100, Laws v Rutherford 1924 AD 261 and The
Fern Gold Mining Company v Tobias (1889-1890) 3 SAR TS 134; and
(2) identify the relevant section(s) in the prescribed textbook.
(1) Read three cases
In Bloom v American Swiss Watch Co 1915 AD 100 the American Swiss Watch Co offered a reward
after a robbery at their shop for the giving of information to the police which lead to the arrest of the
thieves and the recovery of the stolen goods. Bloom did so without knowing of the reward. He
claimed the reward, but his claim was refused. On appeal, the court held that the reward was an
offer open to the public. It could be accepted by providing information to the police. Bloom, however
had no intention of accepting the offer because he did not know of its existence when he gave the
information to the police.
In Laws v Rutherford 1924 AD 261 R gave L an option to accept an offer within 3 months by written
notice. L failed to notify R of his acceptance in the prescribed manner. The court found that no
contract arose.
In Fern Gold Mining Company v Tobias (1889-1890) 3 SAR TS 134 it was held that an offer can be
revoked at any time before acceptance and that acceptance must come to the notice of the offeree.
(2) Identify the relevant section in the textbook
If the “Table of Cases” is consulted at the end of Hutchison and Pretorius (eds) The law of Contract in
South Africa 3rd ed (Oxford Cape Town 2017) you will find the pages where Bloom, Laws and Tobias
are discussed in the textbook.
The first page on which Bloom appears in the text, is 54. The paragraph deals with rewards. It is
mentioned that an advertised reward is an offer to the public. The offer is accepted by performing
the required act and the reward must be certain. A discussion of the requirements of a valid offer
and acceptance seems appropriate. The second page on which it is discussed in the text is 58 which
deals with the requirement that the acceptance must be a conscious response to the offer. This is
again an indication that the requirements for a valid acceptance is relevant.
Laws appears in footnote 50 on page 58 as authority for the requirement that the acceptance must
be in the form prescribed by the offer.
Tobias appears in footnote 44 on page 56 as authority for the requirement that the revocation of an
offer must come to the notice of the offeror. Revocation of the offer is thus also relevant.
The relevant sections of the textbook are page 50– 54, 56 and 57-58 where the requirements of a
valid offer, the revocation of an offer and the requirements for a valid acceptance of an offer are
discussed.
Identifying the problem
First, we must determine whether the reward complies with the requirements for a valid offer. Then
if it does, we must determine whether the offer has validly been revoked. Only if it has not been
validly revoked, the question must be addressed whether the offer has been validly accepted.
Discussing the relevant law applicable to the problem AND applying the law to the facts of the
problem
The offer of reward of XYZ Security complies with the requirements for a valid offer:
1 The offer was firm. An offer must be made with the intention that its acceptance will result in a
binding contract (Hutchison and Pretorius (eds) The law of Contract in South Africa (Oxford Cape
Town 2017) 50). In Bloom v American Swiss Watch Co 1915 AD 100 the American Swiss Watch Co
offered a reward after a robbery at their shop for the giving of information to the police which would
lead to the arrest of the thieves and the recovery of the stolen goods. Bloom did so without knowing
of the reward. He claimed the reward, but his claim was refused. On appeal, the court held inter alia
that the reward was an offer open to the public. The offer of XYZ Security was on the news and was
not a tentative statement.
2 The offer was complete. An offer must include all material terms of the proposed agreement and
there cannot be additional matters that still have to be discussed before the agreement can take
effect (Hutchison and Pretorius Contract 50). The offer contained all the material terms: provide
information to the police leading to the arrest of the robbers in return for the reward of R100 000.
The offer will be accepted by providing the required information.
3 The offer was clear and certain. An offer is sufficiently clear and certain if the mere answer of ‘yes’
by the addressee brings a valid contract into existence (Hutchison and Pretorius Contract 50). If the
offer is unclear and cannot capture what the offeror has in mind, no acceptance of the offer can
create a binding contract. The offer by XYZ Security was so clear that H as a member of the public
could form a clear idea what XYZ Security had in mind: a reward of R100 000 was offered to any
member public for the giving of information to the police leading to arrest of the robbers. The giving
of the required information will also constitute acceptance of the offer. Furthermore, an offer such as
a promise of reward can be validly directed at undefined persons (the public).
The next question is whether XYZ Security validly revoked their offer (Hutchison and Pretorius
Contract 56). XYZ Security did not promise not to revoke their offer and they could revoke their offer
at any time. It is clear from Fern Gold Mining Company v Tobias (1889-1890) 3 SAR TS 134 that an
offer can be revoked at any time before acceptance, but acceptance must come to the notice of the
offeree. The revocation of the reward on 14 January did not come to H notice and therefore did not
take effect against H.
The question now arises whether H validly accepted the offer of reward. The requirements of a valid
acceptance have been met:
1 The acceptance must be unqualified. The acceptance is complete and unequivocal, where the
entire offer and nothing additional or less is accepted (Hutchison and Pretorius Contract 57). H
accepted the offer by providing the required information.
2 The acceptance must be by the person to whom the offer was made. Only the offeree can validly
accept the offer (Bird v Sumerville 1961 (3) SA 194 (A). The general rule is that an offer should be
addressed to a specific person, but an offer may also validly be directed to the public, such as an
offer of a reward (Bloom v American Swiss Watch Co 1915 AD 100). The offer was directed at H as a
member of the public and could be validly accepted by him.
3 The acceptance must be a conscious response to the offer. In Bloom v American Swiss Watch Co
1915 AD 100 the American Swiss Watch Co the court also held on appeal that that Bloom had no
intention of accepting the offer because he did not know of its existence when he gave the
information to the police. In our problem H, however, knew of the offer by XYZ Security and
consciously responded to the offer by providing the required information to the police.
4 The acceptance must be in the form prescribed by the offeror (if any). The offeror can prescribe any
method of acceptance he or she sees fit (Carlill v Carbolic Smoke Ball Co (1893) 1 QB 256; Bloom v
American Swiss Watch Co 1915 AD 100). In Laws v Rutherford 1924 AD 261 R gave L an option to
accept an offer within 3 months by written notice. L failed to notify R of his acceptance in the
prescribed manner. The court therefore found that no contract arose. By giving the police
information leading to the arrest of the robbers, H’s acceptance complied with the prescribed mode
of acceptance of XYZ Security. There was no need for the acceptance to come to the notice of XYZ
Security.
The giving of appropriate advice
A valid contract thus arose between H and XYZ Security on 15 January.
Total: [10]

ASSIGNMENT 02 – FIRST SEMESTER: UNIQUE NUMBER: 787088

Question 1
Which statement is INCORRECT?

1 A contract can be a bilateral juristic act.


2 A contract entails promises or undertakings on one or both sides.
3 An undertaking in a contract that a certain state of affairs exists, or has existed, is known as a
warranty.
4 The conclusion of a contract can never be multilateral.
5 Freedom of contract means that the parties can agree to anything that is possible and lawful. (1)

Question 2
X buys Y’s car for R50 000. X pays the purchase price and Y delivers the car to X. Y has the intention to
transfer ownership and X the intention to receive ownership. What type of agreement(s) did X and Y
conclude when Y delivered the car to X?

1 A contract.
2 An agreement extinguishing a debt.
3 A real agreement.
4 Option 1, 2 and 3.
5 Option 2 and 3.

Question 3
X makes an offer to sell her watch to Y for R2 500. X and Y agree on 15 May that X’s offer will be open
for acceptance until 31 May. On 20 May X informs Y in writing that she (X) intends to sell the watch
to Z on 30 May. On 21 May Y informs X that she (Y) accepts the offer, but X refuses to sell the watch
to her (Y). Y wants to compel X to sell the watch to her. How would you best advise Y?
1 Cancel the contract and seek to interdict X from selling the watch to Z.
2 Enforce the contract and seek to interdict X from selling the watch to Z.
3 Cancel the contract and seek an order for specific performance against X.
4 Enforce the contract and cancel the contract simultaneously through the use of an interdict.
5 Cancel the pre-emption agreement between X and Z. (1)

Question 4
X and Y conclude a contract. When will their contract be VOID?

1 The conclusion of the contract is expressly prohibited by legislation as a crime.


2 X operates under an error in motive.
3 X threatens to kill Y if she does not conclude the contract.
4 X made an intentional misrepresentation to Y before the conclusion of the contract.
5 X made a dictum et promissum to Y before the conclusion of the contract. (1)

Question 5
Liability for which form of misrepresentation may NOT be contractually excluded?

1 An innocent misrepresentation.
2 A negligent misrepresentation.
3 A grossly negligent misrepresentation.
4 A fraudulent misrepresentation.
5 A dictum et promissum. (1)

Question 6
X and Y agree that should X sell her car, she (X) will offer to sell it to Y first, before making an offer to
sell the car to any other person. X sells the car to Z for R50 000 without first offering it to Y for sale.
Delivery of the car has not yet taken place. Which statement is INCORRECT?

1 X gave Y an option to purchase her (X’s) car.


2 X gave Y a right of pre-emption with regard to her (X’s) car.
3 Y can step into the shoes of Z with a unilateral declaration of intent.
4 Both Y and Z only have personal rights against X.
5 The personal right of Y enjoys preference above the personal right of Z.

Question 7
Which of the following must a party prove who seeks to set a contract aside on the ground of undue
influence?

1 The other party obtained influence over her.


2 The other party exercised this influence over her, in an unscrupulous manner in order to induce her
to consent to a transaction which is to her detriment and which she, with a normal free will, would
not have concluded.
3 The other party gained his influence over her by standing in a close relationship to her.
4 Options 1 and 2.
5 Options 1, 2 and 3.

Question 8
X sells his car to Y for R50 000 and undertakes to deliver the car to Y tomorrow morning. The
obligation to deliver the car tomorrow is subject to a

1 suspensive condition.
2 resolutive condition.
3 suspensive time clause.
4 resolutive time clause.
5 modus. (1)

Question 9
X sells his car to Y for R50 000 if it rains in the Karoo next Monday. This is a contract of sale subject to
a

1 suspensive condition.
2 resolutive condition.
3 suspensive time clause.
4 resolutive time clause.
5 modus

Question 10
Which one of the following statements regarding tacit terms is INCORRECT?
1 The courts often employ the officious bystander test in determining whether a contract contains a
tacit term.
2 Tacit terms will only be read into a contract if they do not conflict with any of the unambiguous
express terms of the contract.
3 Tacit terms will only be read into a contract if it is necessary in a business sense to give efficacy to
the contract.
4 Tacit term must be capable of a clear and exact formulation.
5 Tacit terms are the naturalia of the contract.

PVL3702_ Assignment 694865

Question 01
Read the judgment in South African Hang and Paragliding Association v Bewick 2015(3) SA 449 (SCA)
and write a discussion of 500-1000 words summarizing the facts of the case very briefly and then
summarizing what the court decided in respect of wrongfulness element only in that case. (10)
Answer
The facts of the case
In South African Hang and Paragliding Association v Bewick the respondent injured in a paragliding
accident while was transported as a passenger for reward. The paraglider experienced a so-called
wing collapse which affect it manoeurability and caused it to loose height. Driver swung the
paraglider back towards the hillside in an attempt to keep it loft. The respondent thought that she
could cushion the blow of the impending collision by putting her feet out obviously she did not
realize the speed at which they were already travelling when her feet hit the hillside she broke both
of her legs and also her spine. Her injuries left her paralyzed in a wheelchair. The respondent
instituted an action in the Western Cape Division of the High Court in which she claimed damages
from six defendants at hearing Gamble J ordered a separation of issues under uniform rule 33(4) of
the High Court Rules. At the end of the preliminary proceedings the Judge held in favour of the
respondent. The applicants thereupon escalated the matter to the Supreme Court of Appeal.

Wrongfulness
The Western Division of the High Court stated that the respondent’s case is therefore based on an
omission or failure to do something as opposed to positive culpable conduct. That brings about a
different approach to the delictual element of wrongfulness. As has by now become well established
negligent conduct manifesting itself in the form of positive act which causes physical injury raises a
presumption of wrongfulness. By contrast, in relation to liability for omission and pure economic loss
wrongfulness is not presumed and depends on existence of a legal duty.
The court started that to conclude that an accident of omission particularly in relation to public-law
duties was wrongful and impute delictual liability, was an exacting exercise that required a reflection
on a number important factors.

Some factors were:


1. Paragliding within South Africa fell under the direction and control of the two appellants.
2. Tanden paragliding for reward was illegal and the two appellants were aware that illegal activity
was going on.
3. The two appellants were under a legal duty to take reasonable steps to terminate and prevent this
illegal activity, but had negligently failed to do so.
4. Had the appellants done so the flight during which the respondent sustained her injuries would
have not occurred?

Evaluation
The decision of the SCA was differ in respect of wrongfulness and the appeal was upheld with costs,
including costs of the two counsels. The order of the court aquo was set aside and replaced by the
plaintiff’s claim against the fourth and fifth defendants was dismissed with costs including costs of
two counsel in respect of both defendants.

PVL3702_Oct 2016

QUESTION 1
A selects and pays for several items in a supermarket, and leaves. What type of agreement has been
concluded?
1 An agreement creating obligations
2 An agreement extinguishing a debt
3 A real agreement
4 All of the above
5 None of the above

QUESTION 2
Generally, an advertisement is
1 an offer to the public
2 an invitation to do business
3 an offer directed at defined persons
4 an offer directed at undefined persons
5 1 and 4

QUESTION 3
The notion that contracts are based on consensus, has given rise to which theory?
1 The declaration theory
2 The will theory
3 The reliance theory
4 The reception theory
5 The expedition theory

QUESTION 4
X makes a written offer to Y to purchase Y's house X also states in the offer that Y has until 15 May to
accept the offer X has the offer send by courier to Y. Y reads the offer on 13 May and drafts an
acceptance on 14 May Y posts its acceptance on 15 May to X. X receives the acceptance on 17 May
and reads it on 18 May Wh1ch theory explains the legal rules that determine whether a contract
arose here?
1 The declaration theory
2 The expedition theory
3 The reliance theory
4 The reception theory
5 The information theory

QUESTION 5
X sells has shop (a business) an a shopping mall to Y. During the negotiations X informs Y of the
overhead costs of running the shop but fails to mention that the rental for the shop space is soon to
escalate dramatically. In fact the rent wall be so high that the shop wall scarcely be able to turn a
profit. What cause of action wil Y be able to rely on in the circumstances?
1 Dictum et promissum
2 Innocent misrepresentation
3 Culpable misrepresentation
4 Material mistake
5 Non-material mistake

QUESTION 6
Assume the same facts as in question (5}. If Y would not have bought the shop had she (Y} known of
the rent escalation, according to what measure would Y's damages be calculated in the
circumstances?
1 The actio quanti minoris
2 The actio redhibitoria
3 Positive interest
4 Dolus incidens
5 Dolus dans

QUESTION 7
Assume the same facts as in question (5) If Y wanted to cancel the contract what would she usually
have to do?
1 Tender restitution
2 Invoke the exceptio non adimpleti contractus
3 Pay damages to X
4 Use the actio redhibitoria
5 None of the above

QUESTION 8
Which theory explains the provision in the Electronic Communications and Transactions Act 25 of
2002 that determines when a contract is concluded by means of electronic communication?
1 The declaration theory.
2 The expedition theory
3 The information theory
4 The reception theory
5 The reliance theory

QUESTION 9
Sibongile is desperate to sell her house because of a termite infestation Consequently she knowingly
conceals all signs of damage when a potential purchaser, Thandi, comes to inspect the house and
furthermore tells Thandi that there is nothing wrong with the house Thandi purchases the house,
which she would never have done if she knew of the termite infestation. What cause of action will
Thandi be able to rely on in the Circumstances?
1 Simplex commendatio
2 Innocent misrepresentation
3 Fraudulent misrepresentation
4 Duress
5 Puffery

QUESTION 10
Assume the same facts as in question (9) What remedy or remedies are available to Thandi when she
discovers the termite infestation and damage to the house?
1 Thandi may only rescind the contract
2 Thandi may only uphold the contract and claim damages
3 Thandi may rescind the contract and claim damages
4 Thandi may uphold the contract and claim damages
5 Both 3 and 4

QUESTION 11
Incidentalia are
1 All the terms of a contract apart from the naturalia and essentialia
2 Terms that identify a contract as belonging to a particular class of contracts
3 Terms automatically imposed by law on the contracting parties unless contracting parties expressly
exclude them
4 All the terms of a contract apart from the naturalia
5 Material terms and conditions of a contract

QUESTION 12
X contracts with Y for the latter (Y) to build and fit a security gate for the entrance of her (X's) home Y
builds the gate and fits it with an electric motor which is activated with a remote control X is satisfied
with the work and pays Y the contractual amount agreed upon A week later the gate gets stuck while
it is half way open as a result of defective materials used to build the gate When X attempts to
physically move the gate to close It fully, she suffers such severe damage to her left knee that she has
to have a knee operation Her medical costs are R20 000 The costs of repairing the gate amount to
R15 000 X wants to claim both her medical costs, as well the cost of repairing the gate from Y If Y has
breached the contract, what form of breach is it likely to be?
Prevention of performance
2 Mora debitoris
3 Breach of a suspensive condition
4 Positive malperformance
5 None of the above

QUESTION 13
Assume the same facts in question (12) X is likely pursue her claim for repairing the gate (R15 000),
based on
1 the mitigation rule
2 special damages
3 general damages
4 the market value approach
5 the nominal damages rule

QUESTION 14
Assume the same facts m question {12) In order to claim her medical costs (R20 000), which of the
following requirements Will be the most difficult for X to prove? You may assume that X has to
establish all of these requirements to be successful With her claim
1 That Y has committed a breach of contract
2 That X has suffered financial or patrimonial loss
3 That there is a factual causal link between the breach and the loss
4 That X's claim is not one for nominal damages
5 That the loss IS not too remote as a consequence of the breach (that legal causation is present}

QUESTION 15
Assume the same facts in question {12). Further assume that X cannot claim her medical costs
(R20 000) based on general damages. Which of the following must X prove in order to succeed with
her claim for medical costs?
1 That X and Y either expressly (explicitly) or presumably (tacitly) agreed that Y will be liable for
X's medical costs
2 That X's medical costs flow naturally and usually from the kind of breach that Y committed
3 That X's claim for the medical costs is legal
4 Option 1 and 3
5 None of the above options

QUESTION 16
O and C (an electrician), agrees that C Will replace the electrical wiring in O's house for R10 000 and
that the work Will commence on 1 June On 1 June, C arrives at O's house, but O refuses him entry
into the house and tells him that he must return in two months’ time to do the Job O's conduct
amounts to
1 mora debitoris
2 mora creditoris
3 prevention of performance
4 repudiation
5 positive malperformance
QUESTION 17
Assume the same facts as in question (16) C has the following remedy/remedies against O
1 An order for specific performance that allows C entry into O's house in order to do the electrical
wiring
2 A claim for damages for any loss suffered by C
3 A claim for the cancellation of the contract after a letter of demand and a notice of rescission have
been sent to O
4 Options 1 and 2
5 Options 1, 2 and 3

QUESTION 18
Tony operates a general dealer business. From this business, Tony supplies Samuel with prohibited
drugs, which both knew was Illegal. Subsequently when payment for the drugs is due, Samuel
refused to pay Tony. Any potential claim that Tony pursues for the return of the drugs, is based on

1 contractual principle
2 delictual principles
3 principles of unjustified enrichment
4 principles of statutory illegality
5 principles of freedom of trade

QUESTION 19
Assume the same facts in question (18) To reclaim the drugs, Tony Will ask for a court order to
1 enforce the ex turpi rule
2 relax the ex turpi rule
3 enforce the par delictum rule
4 partially relax both the ex turpi rule and the par delictum rule
5 relax the par delictum rule

QUESTION 20
Assume the same facts in question (18) Which statement(s) is/are INCORRECT?
1 Tony cannot recover the money for the drugs from Samuel, because no action arises from an illegal
contract
2 It is unlikely that Tony will succeed in reclaiming the drugs, because both parties appear to be
equally guilty
3 In deciding whether to award Tony the return of the drugs. A court will consider public policy and
whether Justice will be done between man and man
4 In deciding whether to order Samuel to pay Tony for the drugs supplied, a court will consider the
contractual principles relating to restraint of trade
5 In deciding whether to award Tony the return of the drugs, a court is likely to consider the
Judgment in Jajbhay v Cassim 1939 AD 537
QUESTION 21 I VRAAG 21
X promises to give Y R10 000 if Y successfully climbs Everest within two weeks. This is an obligation
subject to a suspensive time clause
2 resolutive time clause
3 suspensive condition
4 resolutive condition
5 modus

QUESTION 22
You buy a very famous painting from an art dealer for one million rand. You and she agree
furthermore that she will deliver the painting later to you at your house. You have agreed to no other
terms A week later you phone her and ask her when the painting will be delivered to you. She tells
you that she cannot do so anymore as she has sold and delivered the painting to X. Her conduct
amounts to
1 prevention of performance
2 positive malperformance
3 supervening impossibility of performance
4 the exercise of a lex commissoria
5 mora debitoris

QUESTION 23
Y offers to sell his BMW motor car to Z on 1 July, for R50 000 One of the terms of Y's offer IS that the
offer lapses on 30 August However, on 20 July Y notifies Z that the offer is cancelled Z insists that the
offer is valid until 30 August and on 25 July Z notifies Y that he (Z) accepts the offer Which statement
reflects the CORRECT legal position?
1 Y and Z concluded an option contract
2 Y and Z did not conclude an option contract
3 Y and Z did not conclude a pre-emption contract
4 Y and Z concluded a pre-emption contract
5 2 and 3

QUESTION 24
Y and Z reach an agreement that should Y sell his leather couch, he (Y) will offer to sell it to Z first,
before making an offer to sell the couch to any other person. Which statement is CORRECT?
Y and Z concluded an option contract
2 Y and Z concluded neither an option contract nor a pre-emption contract
3 Y and Z concluded a contract subject to a resolutive condition
4 Y and Z concluded an option contract and a pre-emption contract
5 Y and Z concluded a pre-emption contract

QUESTION 25
A right of pre-emption
1 is a right acquired by a contract
2 is a right acquired by an offer
3 is a right acquired in terms of an option
4 IS a preferential right entitling the grantee to compel the grantor to sell the thing in question at
anytime
5 encapsulates all of the above statements

QUESTION 1
STATE and very briefly discuss the requirements for a valid offer and acceptance {10)

Answer
An expression of intention will be regarded as sufficient for the purposes of a legally binding offer
only when it meets certain requirements.
2.2.2.1 The offer must be firm
The offer must be a firm one, made animo contrahendi – that is to say, with the intention that its
acceptance will call into being a binding contract. This requirement is not fulfilled if one of the
parties makes a tentative statement to the other with the intention of sounding the other out in
order to find out whether he or she would be prepared to enter into negotiations.3 Whether a
particular declaration amounts to a firm offer, or is merely a tentative indication of willingness to do
business, may not always be easy to determine. It is ultimately a question of fact to be decided in the
light of all the relevant circumstances.4
2.2.2.2 The offer must be complete
The offer must contain all the material terms of the proposed agreement – there cannot be further
matters that still have to be negotiated before the overall agreement can take effect.5 Often, when
large contracts are negotiated, various issues have to be settled before the deal can go ahead. In
such a case, it is said that ‘nothing is agreed until everything is agreed’. In other words, the fact that
the parties have reached agreement on issues A, B and C cannot give rise to binding obligations, if
issues D and E still have to be discussed, and the intention of the parties is that there will be no
binding contract until a comprehensive agreement is reached. However, if the intention of the parties
is that the preliminary agreement in respect of issues A, B and C should be binding on them,
irrespective of whether they ever reach consensus on outstanding issues D and E, then of course the
preliminary agreement will indeed constitute a binding contract. If agreement is subsequently
reached on issues D and E, the preliminary agreement will be incorporated into and superseded by
the more comprehensive agreement.6
2.2.2.3 The offer must be clear and certain
The offer must be sufficiently certain; it should be enough for the addressee merely to answer ‘yes’,
for a contract to come into being. If the offer is so vague that it fails to provide a reasonably clear
indication of what the offeror has in mind, no acceptance of the offer can create a binding obligation
because it will be impossible to determine the content of that obligation. For example, if Christine
says to Sepho: ‘I will buy your car if it suits me’, then Sepho cannot accept the offer, because it is too
vague.7 In this case, the agreement would be regarded as void for vagueness. Certainty may
accordingly be regarded as one of the requirements for a valid contract, and is dealt with as such in
Chapter 8.8 For present purposes, it suffices to note the following points.
Firstly, vagueness must not be confused with ambiguity; if a provision in a contract is capable of two
or more reasonable interpretations, the court will have regard to extrinsic evidence and the rules of
interpretation of contracts to determine the meaning of the provision. Secondly, the courts are
generally reluctant to strike down agreements that were intended to have legal effect; they recognise
that business people are not expert drafters, and are often content to conduct their affairs with only
roughly drawn-up and incomplete agreements in hand, relying on one another’s good faith and
commercial expediency to make such agreements work.9 In the case of Mouton v Hanekom,10 for
example, the Appellate Division held that an offer to perform when the offeror’s financial position
allows him or her to do so is not too vague.

Valid Acceptance
Definition: An acceptance is a clear and unambiguous declaration of intention by the offeree,
unequivocally assenting to all the terms of the proposal embodied in the offer. The offeree’s
intention to accept the offer may be expressly stated (for example, ‘I accept your offer’) or it may be
tacitly indicated (for example, where the offer is made in the presence of the offeree and he or she
nods his or her head). As noted earlier, silence cannot ordinarily be treated as acceptance. Thus the
offeror may not force a contract on the offeree by sending unsolicited goods through the post with a
stipulation that, unless the offeror hears to the contrary within a certain time, he or she will assume
that the offeree has agreed to buy the goods.
Requirements for a valid acceptance
Acceptance will give rise to the formation of a contract between the offeror and the offeree,
provided that certain requirements are fulfilled.
2.3.1.1 The acceptance must be unqualified
The acceptance must be a complete and unequivocal assent to every element of the offer; there can
only be a valid acceptance where the whole offer and nothing more or less is accepted. This is the so-
called mirror image rule. If the offeree’s acceptance is conditional or contains new terms or leaves
out original terms, then there is no clear acceptance and no consensus is reached. A qualified
acceptance constitutes a counter-offer, which the original offeror may accept or reject.46
For example: Dominic offers Didier his surf board for R5 000 cash. Didier answers, ‘I will buy
your surf board for R5 000, but I am short of cash and can only pay in six months’ time.’ By this
qualification, Didier rejects Dominic’s offer and makes him a counter-offer, which Dominic may
accept or turn down.
An ambiguous acceptance also does not qualify as a valid acceptance.47 An example of an
ambiguous acceptance is where Dominic offers Didier his surfboard for R5 000 and Didier replies,
‘I am very interested and will probably purchase your surfboard, but I must first see if I can afford it.’
2.3.1.2 The acceptance must be by the person to whom the offer was made
As noted earlier, where an offer is addressed to unspecified persons, such as the general public or a
class of the public, it may be accepted by any member of the public, or any member of that class.
Where it is addressed to a specific person or persons, it may be accepted only by that person or
group of persons; an offer to sell a farm to A cannot be accepted by A and B jointly, as was held in
Bird v Sumerville.48 In that case, the appellant, who wished to sell his property, was informed by an
estate agent that the first respondent was interested in buying. The appellant signed a written offer
to sell naming the first respondent as sole purchaser. However, the first and second respondents
both signed as buyers. At the time of making the offer, the appellant had been unaware of the
existence of the second respondent. The court found that although the appellant would not have
been prejudiced by both parties buying the property, the appellant was not bound to a contract of
sale to both the respondents because he never intended that his offer could be accepted by both of
them.
2.3.1.3 The acceptance must be a conscious response to the offer
As a matter of logic, a person cannot be said to accept an offer if he or she is unaware of it. This point
is especially relevant to offers of reward. In Bloom v American Swiss Watch Co,49 the respondent
company offered a reward to anyone who provided information leading to the arrest of thieves who
had stolen jewellery from the company. B furnished such information while ignorant of the offer of
reward. When he subsequently became aware of it, he claimed the reward from the company. The
court held that he could not recover the reward, because, ‘until the plaintiff knew of the offer he
could not accept it, and until he accepted it there could be no contract, for a contract requires that
there should be consensus of two minds, and if the one did not know what the other was proposing,
the two minds never came together.’
2.3.1.4 The acceptance must be in the form prescribed by the offeror (if any)
As dominus or initiator of the contracting process, the offeror is entitled to prescribe any method of
acceptance he or she sees fit; if the offeror does so, generally no other form of acceptance will
suffice. However, in Pillay v Shaik,51 the Supreme Court of Appeal held that the reliance theory can
be applied where acceptance does not take place in accordance with a prescribed mode of
acceptance, but where the offeree leads the offeror to a reasonable belief that a prescribed form of
acceptance has in fact been complied with. Note further that the offeror may authorise a particular
method of acceptance without thereby intending to prescribe it as the only acceptable method (for
example, ‘please let me know by return of post whether these terms are acceptable to you’).

QUESTION 2
DISCUSS AND DISTINGUISH between material and non-material mistakes (10}

A material mistake is an error that vitiates or negates actual consensus between the parties. To this
end, a material mistake must relate to or exclude an element of consensus. Conversely, a non-
material mistake does not exclude actual agreement between the parties because it does not relate
to an element of consensus. Consequently, in the case of a non-material mistake, a valid contract will
still arise, although it may be voidable (rescindable) if consensus has been obtained in an improper
manner by way of misrepresentation, duress, undue influence or commercial bribery.18
The distinction between a material and non-material error is vital in determining the appropriate
resolution to a situation where a party was mistaken about something when entering into a contract.
On the one hand, if the mistake is material, the issue is one of dissensus or true ‘mistake’ and the
secondary principles of reliance have to be applied to determine whether a contract based on
reliance has arisen – or whether the mistaken party may be exonerated from contractual liability
because the potential contract lacks a legal basis. On the other hand, if the mistake is non-material, a
consensual contract exists and there is no need to apply the principles of reliance because the
contract already has a legal basis (will theory).19 It is clear that materiality has a very specific
connotation in the context of mistake and it should not be confused with materiality as applied to
other areas of the law of contract.

Material mistake
The elements of consensus, being that the parties must:
• Seriously intend to contract;
• be of one mind as to the material aspects of the contract, namely, the terms of the proposed
agreement, and the identity of the parties to it; and
• be conscious of the fact that their minds have met.
If, for some reason or other, the parties are not in agreement about one (or more) of these elements,
there is a material mistake. It is worth taking a closer look at the elements of consensus and at some
examples of material mistake.

Non-material mistake
Non-material mistakes, while usually influencing a party’s decision to enter into a contract, do not
affect an element of consensus.44 Notwithstanding a non-material mistake, if the parties are ad
idem as to the material aspects of the proposed contract, a consensual contract comes into being.
Typically, a mistake of this nature relates purely to the reason or motive of the mistaken party for
entering into an agreement and hence it is usually referred to as an error in motive.45
In Diedericks v Minister of Lands,46 the defendant extended an offer to the plaintiff to purchase
property from the latter. However, the defendant could merely have invoked a clause in an existing
lease contract in terms of which the defendant would have repossessed the property at a much
cheaper rate. The plaintiff accepted the offer but subsequently the defendant refused to honour its
undertaking, alleging that the offer was made as a result of a clerical error. The plaintiff asked the
court to declare the contract valid. The court found that the defendant’s mistake related to its motive
for making the offer, and did not exclude mutual assent (consensus) between the parties.
Consequently, a valid contract existed.
It should be stressed that just because the parties have reached consensus and a contract has arisen
does not necessarily mean that a mistaken party is without legal recourse. If the mistake in motive
was induced by the misrepresentation of the other party, the mistaken party may have a remedy,
provided the requirements for the particular form of misrepresentation have been complied with.

QUESTION 3
Andy and Craig conclude a contract wherein Andy agrees to paint Craig’s office block by 31 August,
and Craig agrees to pay Andy R1 0 000 upon completion of the work When 40% of the work IS
completed. Andy refuses to continue painting unless Craig pays him R15 000 for the work Craig
refuses to pay Andy more for the work and Andy leaves in a rage On 1 September Andy insists on
payment for the work he has done, but Craig refuses to pay him anything Craig hires another
contractor at an amount of R6 000 to complete the Job Craig does not incur any other costs to
complete the job, neither does his business make any losses Advise ANDY as to what amount (1f any)
he may recover from Craig for the services that he rendered and on what basis Discuss with
reference to BK Tooling (Edms) Bpk v Scope Precision Engineering (Edms) Bpk 1979 (1) SA 391 (A) and
other relevant case law (15)

Answer
Exceptio Non Adimpleti Contractus: This is the first remedy aimed at keeping the contract intact and
forcing the other party to comply with its obligations. The concept of reciprocity is a requisite for this
remedy.
The exceptio non adimpleti contractus is available when two requirements are met: the two
performances must be reciprocal to each other; and the other party must be obliged to perform first,
or at least simultaneously with the party raising the exceptio. The exceptio may also be used where a
party has performed incompletely.
Context:
Termination or discharge of a contract.
The exception non adimpleti contractus.
Facts:
1. S Co and B Co entered into a contract in terms of which S Co agreed to manufacture certain steel
moulds for B Co in accordance with specifications
2. Although the moulds supplied didn’t accord specifications B Co made use of them.
Issue:
Should the strict principle of reciprocity be applied or should B Co only have to render reduced
performance?
Held:
1. The principle of reciprocity should be relaxed.
2. B Co therefore only had to render reduced counter performance, being the contract price less the
cost of rectifying the blocks.
Jansen JA:
It would be useful to consider a few aspects of the principle of reciprocity and its application through
the exceptio.
1. In contracts which create reciprocal obligations, it is a question of interpretation whether these
obligations are so closely linked that the principle of reciprocity applies.
2. The sequence of performance and counter performance also depends upon contractual
provisions.
3. The right of withholding the thing sold until the performance is made is a means of ensuring
counter performance and is analogous to pledge. But as long as performance remains possible and
there has been no cancellation, the other party may still perform.
4. If the right of withholding is regarded as being analogous to pledge, it would mean that a party’s
own performance could only be withheld until counter performance is made. A contractor must fully
perform before he is entitled to the contract price.
5. According to Voet, when the exceptio is raised against a plaintiff, the onus is on plaintiff to prove
that he has fully performed his part of the contract.
From earlier cases, there is agreement that a contractor could despite the principle of reciprocity, still
claim compensation for incomplete performance.
The whole basis of the relaxation of the principle of reciprocity and the possibility of instituting a
claim for reduced contract price rests on consideration of fairness.
Such considerations may well require that a contractor be compensated even though he has not fully
complied with his obligations, and knows that he has not fully complied.
The most satisfactory solution is that of Innes CJ, i.e. that the court, in the exercise of its discretion,
allows a relaxation of the strict principle of reciprocity where the other party has utilised the partial
performance.
If the contractor doesn’t succeed in convincing the court that he has fully complied with his part of
the contract and he wishes to claim a reduced contract price, he will have to prove:
1. That the incomplete performance is being utilised,
2. That in view of the circumstances it would be equitable for the court to exercise its discretion in
his favour,
3. What the reduced contract price should be.
Note:
The strict principle of reciprocity will therefore be relaxed when the above 3 elements are proved.

QUESTION 4
X, who is deeply troubled and m a highly emotional state, Informs his minister (of religion) that he
has bought a number of shares at a fraction of their true value and that, although there was nothing
legally wrong With the transaction, he feels that he has acted in a way which was morally wrong The
minister advises X to donate the shares to the church to rid him of the feelings of guilt, and X there
and then signs a contract in terms of which he donates all the shares to the church A month later X
seeks to have the contract SET ASIDE Will he succeed? Discuss fully Do not apply the Consumer
Protection Act of 2008 to this question. (15)

Answer
If the minister was acting as an agent of the church, then X may have the contract set aside. This is
because the contract would be voidable at his instance, due to the undue influence.

A maximum of 2 bonus marks should be awarded to students who have the presence of mind to
identify that there is a formality requirement to be complied with, for such a donation. But
remember that a maximum a student can get in total for this question is 10 marks.

Formalities
The requirements for a valid contract are: consensus, capacity, formalities, legality, possibility and
certainty. The facts of this problem indicate that the formalities may be implicated because X signs a
written contract of donation. This is an executory donation, because there is no indication in the
facts that the donation has been performed. In order to be valid such donation should be in writing
and signed by X. The donation complies with the required formalities and is thus valid.
6.2.1.3 Donation
Section 5 of the General Law Amendment Act provides that no executory contract of donation
entered into after 22 June 1956 will be valid unless its terms are embodied in a written document
signed by the donor or by a person acting on the donor’s written authority granted in the presence
of two witnesses. The deed of donation itself does not have to be witnessed. The donee has to
accept the offer, but need not sign the contract. An executory donation is one that has not yet been
carried out – that is, has not been completed by delivery of the promised benefit. The purpose of
this provision is apparently to make sure that the donor has a serious intention to conclude the
contract. However, an oral donation completed by delivery of the benefit is a binding contract.

Hutchison & Pretorius (eds) The Law of Contract in South Africa 2nd ed (2012) par 4.4:
4.4 Undue influence
Like duress, undue influence is a form of improper pressure brought to bear upon a person in order
to induce him or her to enter into a contract. However, in the case of undue influence, the pressure is
more subtle, involving an insidious erosion of the victim’s ability to exercise a free and independent
judgement in the matter, rather than threats or intimidation. In most instances of undue influence,
there is a close relationship between the parties – either a fiduciary relationship, as between a
doctor and patient or attorney and client, or one of respect and reverence, as in the case of parent
and child, guardian and ward or religious leader and disciple – which entitles or obliges the one party
to advise and assist the other, and the former then misuses his or her superior position to influence
and undermine the will of the latter.
4.4.1 Origins of the doctrine
The term ‘undue influence’, unknown to our old authorities, comes to us from English law where the
originally narrow scope of the common law concept of duress led the courts of equity to develop the
more elastic doctrine of undue influence. References to undue influence as a consensual defect
giving rise to restitutio in integrum gradually found their way into our law reports and textbooks, but
it was not until 1948 that a court firmly declared that a contract could be set aside on this ground.
Although controversial, this decision was confirmed by the Appellate Division in Preller v Jordaan,
where a majority of the court held that the grounds for restitutio in integrum in Roman-Dutch law
were wide enough to cover what is today referred to as undue influence. Whether or not this finding
is historically correct, the matter has been put beyond all dispute by the subsequent case of Patel v
Grobbelaar, which confirms that the doctrine of undue influence undoubtedly forms part of our
modern law. And indeed, the facts of Patel’s case indicate the need for such a doctrine and the
overall desirability of its recognition by our courts. The laissez-faire attitude displayed by Van den
Heever JA in Preller’s case is neither attractive nor appropriate in modern times, if it ever was.
4.4.2 Requirements
According to the Patel case, adopting the phraseology used in Preller v Jordaan, a party who seeks to
set aside a contract on the ground of undue influence must establish:
1. that the other party obtained an influence over him or her;
2. that this influence weakened his or her powers of resistance and rendered his or her will
compliant; and
3. that the other party used this influence in an unscrupulous manner to persuade him or her
to agree to a transaction that was prejudicial to him or her, and which he or she would not have
concluded with normal freedom of will.
Since the basis of the action is vitiated consent, it is not clear why prejudice should be proved; once
again, it is submitted that mere inducement to contract should suffice.
In English law, the existence of a special relationship between the parties gives rise to a rebuttable
presumption of undue influence. In our law it seems fairly clear that no such principle applies, the
onus being on the party who alleges undue influence to prove that it occurred; but of course the
existence of a confidential relationship between the parties is a relevant factor in discharging that
onus. As this rule demonstrates, we have merely adopted the term ‘undue influence’ from English
law, and not the detailed rules in connection therewith.
Where the undue influence is brought to bear by an outsider, the party influenced may set aside the
contract only if the other contracting party was aware, at the time of the transaction, that the
influence had been exercised. Whether an action for damages will lie against the outsider in such
circumstances, or against the other party to the contract when it is he or she who has exercised the
influence, has not yet been decided; but it seems fairly clear that where the undue influence has
caused loss, the Aquilian action will lie since the conduct of the guilty party is both deliberate and
wrongful.
Being a somewhat elastic concept, undue influence may in some cases be almost indistinguishable
from fraud, while in others it may overlap with duress. As pointed out earlier, however, it is the
absence of intimidation that distinguishes it from duress. Thus, for example, pressure exerted by one
who is in a position of authority over another will usually constitute undue influence; but if the
pressure is accompanied by threats or intimidation, we have a case of duress: the so-called metus
reverentialis.

PVL3702_May 2017

QUESTION 1
Which statement IS INCORRECT?
Obligationary agreements create one or more obligations
2 Absolving agreements discharge or extinguish obligations
3 Real agreements transfer rights
4 Transfer agreements transfer rights
5 All binding agreements are contracts

QUESTION 2
Which term/phrase IS NOT linked to the characteristic features of a contract?
1 Juristic act
2 Bilateral in nature
3 Unilateral promise
4 Multilateral in nature
5 Promises or undertakings

QUESTION 3
X attends the auction of a Porsche Carrera (a type of sports car). The auctioneer announces that
there is a reserve price set for the car and that the successful bidder must pay by way of bank
guaranteed cheque before the vehicle will be delivered X bids R50 000 for the car. Which
consequence does X's bid have?
1 It IS an offer to an undefined person
2 It causes a contract of sale to arise
3 It binds X to the auction conditions
4 It causes a contract of sale to arise subject to the auction conditions
5 It has no legal consequences

QUESTION 4
X sent an offer by email to Z on 1 February Z downloaded the email to her computer on 6 February,
but only read it on 7 February Z sent an ema1l to X on 8 February, in which she accepted the offer Z's
email reached X's service provider on 9 February and could have been downloaded by X on that date
X only downloaded Z:s email and read it on 11 February When was the contract between X and Z
concluded?

1 6 February
2 7 February
3 B February
4 9 February
5 11 February
QUESTION 5
What theory regards the inner wills of the parties as Irrelevant on the basis that what the parties say
and do are important and not what the parties think?
1 The declaration theory
2 The will theory
3 The reliance theory
4 The reception theory
5 The expedition theory
QUESTION 6
X is a keen golfer who has played at many golf tournaments over the years as an amateur. She is very
well informed about the rules pertaining to her amateur status as a golfer and knows that amateurs
can only claim a maximum of R1 000 in prize money at golf tournaments X participated in a recent
golfing tournament wherein she achieved a hole-in-one at the 9th hole. At this hole was an
advertising board which read "Hole-in-one prize sponsored by Speedy Motors to the value of R90
000" The prize was parked next to this board in the form of a new car X claimed the prize from
Speedy Motors but they rejected her claim on the basis that the prize could only be claimed by
professional players and not amateur players Regarding X's claim for delivery of the car. Which is the
most contentious issue?
1 Whether the acceptance was made by the person to whom the offer was made
2 Whether the offer was firm
3 Whether the offer was complete
4 Whether the acceptance was unqualified
5 Whether the acceptance was in the form prescribed by the offeror

QUESTION 7
Assume the same facts m question 6. Which statement is CORRECT?
1 Speedy Motors made a misrepresentation regarding their Intention to make an offer to
participating amateur golf players
2 Speedy Motors' misrepresentation regarding their intention to make an offer to participating
amateur golf players probably did not mislead X and a reasonable person would also not have been
misled thereby
3 Speedy Motors' misrepresentation regarding their Intention to make an offer to participating
amateur golf players probably mislead X and a reasonable person would also have been misled
thereby
4 Option 1 and 2
5 Option 1 and 3

QUESTION 8
Which statement regarding the iustus error doctrine is CORRECT?
1 The iustus error doctrine qualifies the objective approach of our courts to error
2 The iustus error doctrine is very similar to estoppel but does not require fault and prejudice as
estoppel does
3 The iustus error doctrine can be used to prove the existence of a contract on the basis of quasi
mutual assent
4 The iustus error doctrine amounts to a direct application of the reliance theory
5 Options 1 and 4

QUESTION 9
Which cause of action is delictual?
1 Mistake
2 Culpable misrepresentation
3 Repudiation
4 Dictum at promissum
5 Breach of contract

QUESTION 10
Which cause(s) of action may render the contract void?
1 A fraudulent and negligent misrepresentation
2 Duress
3 Undue Influence
4 A reasonable and material mistake
5 Commercial bribery

QUESTION 11
In which Instance has consensus been obtained improperly?
1 A common error
2 Rectification
3 An error in substantia
4 A threat that is not related to an imminent or inevitable evil
5 An innocent misrepresentation

QUESTION 12
X IS employed as a bookkeeper in Y's business X steals money from the business's bank account over
a long period of time Y gets forensic auditors in and they determine that X has stolen R50 000 Y
confronts X and threatens to lay a charge of theft against X at the police station unless X signs an
acknowledgement of debt for RSO 000 and undertakes to pay back the money in monthly
Instalments of R500 each X signs because he is afraid to go to Jail Which statement regarding the
presence of the requirements for duress is INCORRECT?
1 X had a reasonable fear
2 Ys threat weakened X's power of resistance and rendered X's w111 compliant
3 There was a threat of an 1mm1nent ev11 to X
4 The threat aga1nst X was contra bonos mores
5 The pressure Y used aga1nst X caused X to suffer damage

QUESTION 13
The courts use the hypothetical bystander test when determining the possible existence of
1 essentialia
2 naturalia
3 terms implied by law
4 tacit terms
5 express terms

QUESTION 14
Y sells his cat, Felix, to Z for R1 000 on 15 January Unbeknown to both parties, lightning killed the cat
on 14 January. This is a case of
1 mutual mistake
2 impossibility of performance
3 supervening impossibility of performance
4 prevention of performance
5 common mistake

QUESTION 15
X promises to give Y R1 0 000 If Y successfully passes her matric examination at the end of the year
This is an obligation subject to a
1 suspensive time clause
2 resolutive time clause
3 suspensive condition
4 resolutive condition
5 modus

QUESTION 16
Y sells his car to Z for R20 000 on 15 January Y undertakes to deliver the car to Z on 17 January The
undertaking to deliver the car on 17 January is a
1 suspensive time clause
2 essentialium (singular for essentialia)
3 incidentalium (s1ngular tor incidentalia)
4 option 1 and 2
5 option 1 and 3

QUESTION 17
Naturalia are
1 terms automatically Included in any contract belonging to one of the classes of specific contracts
traditionally recognised by our law
2 those distinctive terms used to Identify or classify a contract as one of the specific contracts
recognised by our common law
3 additional terms agreed upon by the parties that supplement the rights and duties incorporated by
law into the particular contract in question
4 terms specifically agreed upon by the parties and are either articulated in an oral contract or
written down
5 all terms that are not incidentalia

QUESTION 18
A contract for the benefit of a third party relates to which legal concept?
1 Set-off
2 Novatio
3 Delegation
4 Stipulatio alteri
5 Cession

QUESTION 19
Y sells his car to Z for R20 000 on 15 January Y undertakes to deliver the car to Z on 17 January, but
fails to do so. This failure amounts to
1 mora creditoris
2 subjective impossibility of performance
3 subjective supervening impossibility of performance
4 mora debitoris
5 prevention of performance

QUESTION 20
Y sells his car to Z for R20 000 on 15 January Y undertakes to deliver the car to Z on 17 January Y
destroys the car in a fit of anger on 16 January This act of Y amounts to
1 prevention of performance
2 impossibility of performance
3 supervening impossibility of performance
4 mora debitoris
5 positive malperformance

QUESTION 21 I VRAAG 21
Y sells her car to Z for R20 000 on 15 January Y undertakes to deliver the car to Z on 17 January On 16
January Y informs Z that she is not going to deliver the car at all, because she is no longer interested
in selling the car This act amounts to
1 mora debitoris
2 mora creditoris
3 prevention of performance
4 repudiation
5 none of the above options

QUESTION 22
Mpho and Craig conclude a contract wherein Mpho agrees to paint Craig's office block by 31 January,
and Craig agrees to pay Mpho R1 0 000 upon completion of the work When 40% of the work is
completed Mpho refuses to continue painting unless Craig pays him R15 000 for the work Craig
refuses to pay Mpho more for the work and Mpho leaves in a rage Craig never cancels the contract
with Mpho On 1 February Mpho insists on payment for the work he has done, but Craig refuses to
pay him anything Craig then hires another contractor at an amount of R7 000 to complete the job
Craig does not incur any other costs to complete the job, neither does his business make any losses
Craig never cancels the contract What amount can MPHO (If any) recover from Craig for the services
that he rendered?
1 R0
2 R3000
3 R4000
4 R10 000
5 R15 000

QUESTION 23
Assume the same facts in question 22 Mpho's claim is based on
1 delict
2 mora debitoris
3 mora creditoris
4 contract
5 unjustified enrichment

QUESTION 24
What is the general effect of supervening impossibility of performance?
1 Termination of the contractual obligation
2 The contract is rendered voidable at the instance of the creditor
3 The contract is rendered voidable at the Instance of the debtor
4 The creditor is guilty of contractual breach
5 The debtor IS guilty of contractual breach

QUESTION 25
Which one of the following IS NOT a requirement for a valid cession?
1 An entitlement by the cedent to dispose of the personal right
2 A transfer agreement
3 The capacity of the personal right to be ceded
4 Consent of the debtor not to be prejudiced
5 Legality

QUESTION 1
State the requirements for the formation of a valid contract (5)

Answer
The requirements for a binding contract are absolute, in the sense that they must all be complied
with for the creation of a valid contract.
In order to be recognised as a valid and binding contract, the agreement must satisfy the following
requirements, each of which will be considered at length in subsequent chapters:
• Consensus – the minds of the parties must meet (or at least appear to meet) on all material aspects
of their agreement;
• Capacity – the parties must have the necessary capacity to contract;
• Formalities – where the agreement is required, unusually, to be in a certain form (for example, in
writing and signed), these formalities must be observed;
• Legality – the agreement must be lawful – that is, not prohibited by statute or common law;
• Possibility – the obligations undertaken must be capable of performance when the agreement is
entered into; and
• Certainty – the agreement must have a definite or determinable content, so that the obligations
can be ascertained and enforced.

QUESTION 3
Y signs and delivers a written offer (including all the material terms) to Z on 1 July, for the purchase of
Z's apartment Y's offer is for R800 000 and one of the terms of the offer states "This offer lapses on
30 August" However, whilst Z is still considering Y's offer, Y delivers a letter to Z on 20 July, advising Z
that his (Y's) offer is cancelled Z insists that the offer is valid unt1l 30 August, and on 25 July Z delivers
a letter to Y, advising Y that he (Z) accepts Y's offer Has a valid contract of sate been created between
Y and Z? DISCUSS (5)

Answer
Contracting parties may enter into an agreement in terms of which the offeror undertakes not to
revoke his or her offer. In such cases, it is said that one party grants the other an option.
For this question, an option does not exist because there is no agreement in place that binds Y to
keep his offer open until 30 August. Y has unilaterally imposed this upon himself in the offer, but it
was certainly not an agreement by both parties to hold Y to keep his offer open until this date. This
means that no option contract was concluded.
Y validly revokes his offer to Z on 20 July and therefore there is no offer that Z can accept. The
requirements for a valid offer and acceptance for a contract have not been met, and no valid contract
has thus been created.

In Brandt v Spies (above), the defendant orally granted an option to the plaintiff to purchase his farm.
Disregarding the option, he sold the farm to a third party.
The plaintiff, who had exercised the option in writing, then claimed damages for breach of contract,
but an exception to his claim was upheld. The court stated as follows:
If the offer is not in writing there is nothing which the offeree can accept so as to create a vinculum
iuris between himself and the offeror. An undertaking to keep open an offer which is incapable of
forming the basis of a valid contract can itself confer no right upon the grantee for in law there is
nothing to keep open.
This judgment, it is submitted, is correct in the case where the offer to sell and the offer to keep the
first offer open are made simultaneously and orally. As regards the contract of sale, both offer and
acceptance must be in writing in accordance with the above-mentioned legislation, and the offer has
been made orally in such a case. However, what of the case where the offer to sell is made in writing
on one occasion and is later followed by an oral offer to keep the offer to sell open? The Act provides
that no alienation of land will be valid unless it is reduced to writing. The option contract which
comes into question with the acceptance of the oral offer is not an alienation of land and,
consequently, one can conclude that it need not be in writing. What must be in writing is the offer to
sell.

QUESTION 4
Tony, a petrol attendant, sells dagga to Samuel for R1 000 Tony delivers the dagga to Samuel, but
Samuel refuses to pay Section 5 of the Drugs and Drug Trafficking Act 140 of 1992 provides that no
person shall deal in (a) any dependence-producing substance or (b) any dangerous dependence
producing substance or any undesirable dependence-producing substance, while section 4 prohibits
the possession of such substances Section 13 makes the contravention of both sections 4 and 5 a
crime The Act furthermore lists dagga as an undesirable dependence-producing substance Tony
approaches a court to assist him, either for the payment of R1 000 from Samuel or the return of the
drugs by Samuel (15)

Answer
The ex turpi rule simply means that an illegal contract is void and that no obligations arise out of this
contract. There are NO exceptions to this rule. If one or both of the parties perform in terms of such
a void contract, performance will unjustifiably enrich the other party, as there is no obligation to
perform. This means that the party who has performed may claim back his or her performance with
an enrichment action. But, here, the par delictum rule comes into play and prohibits such a claim on
policy considerations in certain circumstances. However, fairness between the parties is also a policy
consideration and allows the courts to relax the par delictum rule in certain circumstances.

This question involves an illegal contract of sale, which is void due to statutory illegality. The fact that
the legislator has enacted a criminal sanction for a contravention is a factor that would imply that the
legislator intended the contract to be void.
An illegal contract creates no obligations and it cannot be enforced. The ex turpi rule applies: from an
illegal cause no action arises. Neither party can institute an action on the contract or claim
performance from the other party. So for instance if a party has suffered damage as a result of such a
contract, he or she may not claim contractual damages from the other party. A court does not have
the discretion to relax this rule and there are no exceptions to it.
A party who has performed in terms of an illegal contract may however reclaim his performance, in
principle, with an enrichment action. However, such restitution will be prevented where the par
delictum rule applies. According to the par delictum rule: where two parties are equally morally
guilty, the one who is in possession is in the stronger position. If this is the case, restitution in terms
of an enrichment action is prevented.
In our case, Tony is precluded from instituting any contractual claim for R1000 from Samuel because
of the ex turpi rule, and also from an enrichment claim because of the par delictum rule.
The situation might differ if Tony was an undercover cop. In such a case, Tony would not be equally
morally guilty (Minister of Justice v Van Heerden) and so the par delictum rule would not apply.
In Jajbhay v Cassim, the Appellate Division held that the par delictum rule may be relaxed in
appropriate circumstances in order to justice “between man and man” if it would be in the interests
of public policy.

The par delictum rule, in pari delicto potior est conditio possidentis (where two parties are equally
morally guilty, the one who is in possession is in the stronger position) will prevent restitution from
taking place. This rule does not preclude the enforcement of an unlawful contract by means of a
claim for specific performance (that is the function of the ex turpi rule), but it does prevent a party
from reclaiming his or her performance in terms of an unlawful contract.
The par delictum rule is based on two considerations of public policy: a court will not assist those
who approach it with ‘unclean hands’, and unlawful contracts should be discouraged.
The par delictum rule only applies where the parties are equally ‘guilty’ in concluding an illegal
agreement. Accordingly, the rule will not apply where the plaintiff is less guilty than the defendant
or is not morally guilty at all. A plaintiff’s guilt is diminished, for example, where he or she did not
know of the illegality of the contract or is reclaiming money paid to a blackmailer.

QUESTION 5
X, a manufacturer of navigation systems, sells a new navigation system to Y for R200 000 and
undertakes to install it on Y's fishing trawler X guarantees that the system complies with a number of
specifications. The fishing trawler runs aground because the navigation system does not comply with
the specifications and the trawler is damaged by the waves. Fortunately the trawler can be salvaged
at the cost of R 100 000. It will cost a further R50 000 to repair the damage to the trawler. To upgrade
the navigation system to comply With the specifications will cost R50 000 X and Y knew at the time of
conclusion of the contract that if the navigation system does not comply with the specifications it
would seriously malfunction, that a navigation system is essential for the safety of every fishing
trawler especially at night and during storms at sea, that fishing trawlers are used at night and are
sometimes caught in storms at sea Y wants to claim R200 000 damages from X. Will Y succeed?
Discuss Do not apply the Consumer Protection Act of 2008 to this question. ( 15)

Answer
This matter deals with the warranty/guarantee of navigation system complying with a number of
specifications.
Issue:
1. The navigation system does not comply with the specifications and the trawler is damaged
by the waves.
2. X and Y knew at the time of conclusion of the contract that if the navigation system does not
comply with the specifications it would seriously malfunction, that a navigation system is
essential for the safety of every fishing trawler especially at night and during storms at sea
Facts:
1. Y bought a navigation system from X
2. X guaranteed that the system complied with specifications
3. The system did not work as expected and created damages to the trawler
4. Y incurred financial loss in salvaging and repairing the damages as result of the defective
navigation system
5. Y wants to claim damages as a result of the damages caused by the navigation system which
did not comply with specifications

The conduct of the party who induced the contract by improper means will frequently constitute a
delict. Where this is the case, the innocent party may recover damages in respect of any financial
loss that he or she has suffered as a result of the delict, irrespective of whether he or she elects to
affirm or to rescind the contract. In spite of the contractual context, the damages are delictual and
therefore governed by the principles applicable to the actio legis Aquiliae.

Positive malperformance may take two forms. Where the debtor has a positive obligation, positive
malperformance occurs when the debtor duly performs but in an incomplete or defective manner.
The damages that are awarded in lieu of or to complete the performance are known as surrogate
damages. The view has been expressed that the creditor may reject the debtor’s performance only if
the defect (or shortfall) is sufficiently serious to merit rescission

In Minister Van Landbou-Tegniese Dienste V Scholz


The court held:
1) Buyer had not relied on the action redhibitoria (but on contract) and therefore his claim had not
prescribed.
2) Seller had tacitly guaranteed that the bull he sold was fertile.
3) Buyer therefore had brought the actio empti which had not prescribed.
1) It’s a naturale (ex lege term) of a contract of sale that seller is liable for latent defects in the thing
sold without the parties having agreed thereto.
2) Buyer would be entitled to cancel contract with action redhibitoria or get a reduction in purchase
price with actio quanti minoris.
3) The parties can also agree expressly or tacitly that seller will be liable for latent defects in the thing
sold – i.e.: the relevant naturale does not prevent seller from expressly or tacitly guaranteeing that
thing sold is free of latent defects.
4) If it later becomes apparent that there are defects, seller will be liable for breach of contract -
normal contractual action (in a sale = actio empti).

PVL3702_Nov 2017

QUESTION 1
Which obligations have NO legal consequences?
1 Civil obligations
2 Natural obligations
3 Facultative obligations
4 Moral obligations
5 Alternative obligations

QUESTION 2
Which of the following is NOT a requirement for a valid contract?
1 Consensus
2 Formalities
3 Reciprocity
4 Possibility
5 Certainty

QUESTION 3
X buys Y's car for R50 000 X pays the purchase price and Y delivers the car to X. Ownership of the car
has passed from Y to X, because
1 the contract of sale was valid
2 the parties concluded a valid real agreement by Y delivering the car to X and X receiving the car
3 the parties concluded a valid agreement extinguishing a debt by Y del1venng the car and X
receiving the car
4 the parties acted in good faith when the car was delivered
5 2 and 4

QUESTION 4
Generally, an advertisement is
1 an offer to the publ1c
2 an invitation to do business
3 an offer directed at specified persons
4 an offer directed at unspecified persons
5 1 and 4

QUESTION 5
Generally, an offer of reward is
1 an offer to the public
2 an invitation to do business
3 an offer directed at specified persons
4 an offer directed at unspecified persons
5 1 and 4

QUESTION 6
Generally, where is the place of formation of a contract?
1 Where the offeree decides to accept the offer
2 Where the last act necessary to constitute the agreement is formed
3 Where the offeree posts his acceptance
4 Where the offeree prepares to perform in terms of the contract
5 Where the offeror learns of the acceptance

QUESTION 7
Which theory explains the legal rules that determine when and where a contract is concluded by
telephone?
1 The declaration theory
2 The information theory
3 The reliance theory
4 The reception theory
5 The expedition theory

QUESTION 8
X wants to conclude a lucrative contract with company Y. To ensure that he is successful, X agrees
with the agent of Y, who is responsible for concluding contracts on behalf of Y, that for a fee of R50
000 the agent will ensure that the contract will be awarded to X. After the contract has been
concluded between X and Y, and both parties have performed, the financial director of Y discovers
how X secured the contract The issue emanating from these facts relates to
1 duress
2 undue Influence
3 commercial bribery
4 misrepresentation
5 mere commercial competition

QUESTION 9
Assume the same facts in question 8. Which statement(s) is/are CORRECT?
1 The contract between X and the agent is Illegal and void
2 The agent will be able claim payment of R50 000 from X
3 The contract between X and Y is voidable at the instance of Y
4 The contract between X and Y IS voidable at the Instance of both X and Y
5 Options 1 and 3

QUESTION 10
Which cause(s) of action may render the contract voidable?
1 A dictum et promissum
2 Illegality
3 Supervening impossibility of performance
4 Innocent misrepresentations
5 A reasonable and material mistake

QUESTION 11
Inequality of bargaining power between the parties is taken into account as such to determine if
1 a contract is against public policy
2 undue influence has taken place
3 duress has taken place
4 options 1 and 2
5 options 1, 2 and 3

QUESTION 12
What type of claim may the aggrieved party successfully institute against the other contracting party
for the return of his performance in terms of an Illegal contract, if the in pari delicto rule is relaxed?
1 A delictual claim
2 An unjustified enrichment claim
3 A contractual claim
4 A specific performance claim
5 None of the above

QUESTION 13
Cedric has joined a religious sect. Recently this sect took advantage of Cedric's good nature and
convinced him to donate his motor vehicle to the sect, which Cedric did Cedric approaches you for
legal advice, explaining that he wants his motor vehicle returned to him. The issue emanating from
these facts relates to
1 duress
2 undue Influence
3 commercial bribery
4 puffs
5 dicta et promissa

QUESTION 14
Assume the same facts as in question 13 An order sought for the return of the motor vehicle by
Cedric amounts to an order for
1 restitution
2 specific performance
3 compensation for general damages
4 compensation for special damages
5 rectification

Question 15
Which of the following must a party prove who seeks to set a contract aside on the ground of undue
influence?
1 The other party obtained influence over her
2 The other party exercised this Influence over her, in an unscrupulous manner in order to induce her
to consent to a transaction which is to her detriment and which she, with normal free will, would not
have concluded
3 The other party gained his influence over her by standing in a close relationship to her
4 Options I and 2
5 Options I, 2 and 3

QUESTION 16
What theory regards the inner wills of the parties as irrelevant on the basis that what the parties say
and do are important and not what the parties think?
1 The declaration theory
2 The will theory
3 The reliance theory
4 The reception theory
5 The expedition theory

QUESTION 17
An option is
1 only an offer to sell something
2 both an offer to sell something, and an agreement to keep this offer open
3 both an offer to sell something, and an agreement to keep this offer open for a certain period
4 an agreement to make an offer revocable for a certain period of time
5 none of the above

QUESTION 18
ABC Limited donates money to the XYZ Municipality. The grant is subject to the proviso that the
money be used to build a clinic for the local community. The donation is subject to
1 a resolutive condition
2 a suspensive condition
3 a modal clause (modus)
4 a counter performance
5 either 3 or 4

QUESTION 19
In ticket cases, to determine if the person receiving the ticket is bound by the terms, which of the
following is consistent with the three-fold test applied by the courts?
1 Did the person receiving the ticket know that there was writing on the ticket?
2 Did the person receiving the ticket sign and acknowledge receiving the ticket?
3 Did the person issuing the ticket take the steps that he believed were necessary to bring the terms
to the notice of the other party?
4 1, 2 and 3
5 1 and 3

QUESTION 20 I VRAAG 20
X undertakes to donate R100 000 to Y as soon as Y’s husband dies. This donation is subject to a
1 suspensive condition
2 resolutive condition
3 modal clause (modus)
4 suspensive time clause
5 resolutive time clause

QUESTION 21
X promises to give Y R10 000 if Y successfully climbs Everest. This is an obligation subject to a
1 suspensive time clause
2 resolutive time clause
3 suspensive condition
4 resolutive condition
5 modal clause (modus)

QUESTION 22
Y sells her car to Z for R20 000 on 15 January Y undertakes to deliver the car to Z on 17 January. On
16 January, Y informs Z that she IS not going to deliver the car at all, because she is no longer
interested in selling the car. This act amounts to
1 mora debitoris
2 mora creditoris
3 repudiation
4 prevention of performance
5 none of the above options

QUESTION 23
Y sells his car to Z for R20 000 on 15 January Y undertakes to deliver the car to Z on 17 January Y
destroys the car in a fit of anger on 16 January This act of Y amounts to
1 prevention of performance
2 impossibility of performance
3 supervening impossibility of performance
4 mora debitoris
5 positive malperformance

QUESTION 24
Ben and John conclude a contract In terms of the contract, the parties have agreed that John will
purchase Ben's Harley Davidson motorcycle for R200 000 However, after the contract is concluded
John is unable to raise the finance from his bank to purchase the motor cycle John wishes to get out
of the sale Do not apply the Consumer Protection Act of 2008 to this problem Which answer reflects
the CORRECT legal position?
1 No valid contract arose because the obligation to pay the price was practically or economically
impossible during conclusion of the contract
2 No valid contract arose because the obligation to pay the price was objectively impossible during
conclusion of the contract
3 The obligation to pay the price is extinguished by supervening impossibility of performance
because the performance has become objectively impossible
4 The obligation to pay the price is not extinguished by supervening impossibility of performance
because the performance has only become subjectively impossible
5 The obligation to pay the price is extinguished by supervening impossibility of performance
because of John's fault (his inability to raise finance)

QUESTION 25
X orders medication from a pharmacy but the pharmacy indicates that it only has the generic version
of the medicine that X requires In stock X indicates that he will accept the generic medicine in the
place of the prescribed medicine. This is an example of
1 novation
2 datio in solutum
3 compromise
4 release
5 none of the above

QUESTION 1
State the requirements for a valid offer and acceptance (5)

Answer
OFFER:
• Must be firm.
(That is to say, with the intention that its acceptance will call into being a binding contract.)
• Must be complete.
(It must contain all the material terms of the proposed agreement.)
• Must be clear and certain.
(It should be enough for the addressee to answer merely “yes” for a contract to come into being.)
• Must meet the requirements of the Consumer Protection Act.
ACCEPTANCE:
• Must be unqualified.
(It must be a complete and unequivocal assent to every element of the offer.)
• Must be by the person to whom the offer was made – Bird v Summerville.
(E.g. the offer to sell farm A cannot be accepted by A and B jointly.)
• Must be a conscious response to the offer – Bloom v American Swiss Watch Co.
(A person cannot accept an offer if he was not aware of it.)
• Must be in the form prescribed by the offeror, if any.
State the ways an offer may be terminated.
1. Rejection of the offer
2. Acceptance of the offer
3. Effluxion of the prescribed time, or of a reasonable time
4. Death of either party
5. Revocation of the offer
6. Loss of legal capacity to act

QUESTION 2
Discuss the objective approach of the courts to error as qualified by the iustus error doctrine (10)

Answer
The declaration theory or wholly objective approach to contractual liability grounds contractual
liability purely on concurring, objective declarations of will. The inner will or actual intention of a
party is irrelevant.
The case for this objective approach in South African law is essentially twofold. In the first instance,
there appears to be direct authority for it in the famous dictum of Wessels JA, in South
African Railways & Harbours v National Bank of South Africa Ltd:
The law does not concern itself with the working of the minds of parties to a contract, but with the
external manifestation of their minds. Even therefore if from a philosophical standpoint the minds of
the parties do not meet, yet, if by their acts their minds seem to have met, the law will, where fraud
is not alleged, look to their acts and assume that their minds did meet and that they contracted in
accordance with what the parties purport to accept as a record of their agreement. This is the only
practical way in which courts of law can determine the terms of a contract.

In terms of the iustus error doctrine, however, an objective contract may be rendered void for a
material and reasonable mistake.
A prominent example of this generally objective approach to contract is National and Overseas
Distributors Corporation (Pty) Ltd v Potato Board.146 The respondent invited tenders for the erection
of a steel shed. The appellant submitted its tender, which the respondent’s manager accepted on its
behalf by way of letter. The appellant set about making arrangements for the erection of the shed.
Thereafter, the appellant was informed by the respondent that the letter of acceptance had
inadvertently been addressed to the appellant as a result of an administrative error and that the
respondent had in fact accepted the tender of another firm. The appellant refused to accept that
such an error exonerated the respondent and claimed that a contract had arisen between the
parties, and that the respondent had breached it by denying its existence. In an action for damages
by the appellant, the respondent disputed inter alia that a contract had arisen and that the manager
had authority to enter into the contract on its behalf. The court a quo decided in favour of the
respondent. The Appellate Division upheld the appellant’s appeal and stated:
If the respondent had been a natural person who had accepted a tender according to its terms, there
is no doubt that a contract would have been made when the acceptance was communicated to the
tenderer, as by posting it. It would not be possible for such a natural person, if he repudiated, to
escape liability by proving that he had posted the wrong letter or the like. That follows from the
generally objective approach to the creation of contracts which our law follows. [own emphasis]

It seems that the iustus error approach to the problem of mistake in contract finds its origin in a text
dealing with restitutio in integrum on the grounds of reasonable error. Generally speaking, the iustus
error approach, like estoppel, is not in itself a theory of contractual liability.
However, it functions as a corrective measure in the case of dissensus and provides that a party will
not be held bound to an agreement if that party apparently (but mistakenly) gave his or her consent
and if his or her mistake is material and reasonable (iustus). Once the contract asserter has shown
that there is an ostensible agreement (consent from an objective viewpoint), or rather, that there is a
‘common intention expressed’,155 the contract denier bears the onus of proving that his or her
mistake is both material and reasonable in order to be absolved from liability in terms of the
apparent contract. If the contract denier succeeds, the contract is void ab initio, but if he or she fails,
the contract denier will be contractually bound on the terms originally proved by the contract
asserter.
However, the iustus error doctrine has never been comprehensively formulated and guidelines as to
what may be regarded as a reasonable mistake has developed piecemeal in case law. To begin with,
there are two dicta that are virtually synonymous with the iustus error approach. Firstly, there are
the words of Fagan CJ in George v Fairmead (Pty) Ltd:
When can an error be said to be justus for the purpose of entitling a man to repudiate his apparent
assent to a contractual term? As I read the decisions, our Courts, in applying the test, have taken into
account the fact that there is another party involved and have considered his position. They have, in
effect, said: Has the first party – the one who is trying to resile – been to blame in the sense that by
his conduct he has led the other party, as a reasonable man, to believe that he was binding himself?
… If his mistake is due to a misrepresentation, whether innocent or fraudulent, by the other party,
then, of course, it is the second party who is to blame, and the first party is not bound.
The second dictum is that of Schreiner JA in National and Overseas Distributors Corporation (Pty)
Ltd v Potato Board:
Our law allows a party to set up his own mistake in certain circumstances in order to escape liability
under a contract into which he has entered. But where the other party has not made any
misrepresentation and has not appreciated at the time of acceptance that his offer was being
accepted under a misapprehension, the scope for a defence of unilateral mistake is very narrow, if it
exists at all. At least the mistake (error) would have to be reasonable (justus) and it would have to be
pleaded.
Iustus error doctrine a doctrine in terms of which a material and reasonable mistake renders an
apparent contract void.

QUESTION 3
X is employed as a bookkeeper in Y's business X steals money from the business's bank account over
a long period of time. Y gets forensic auditors in and they determine that X has stolen R50 000. Y
confronts X and threatens to lay a charge of theft against X at the police station unless X signs an
acknowledgement of debt for R100 000 X signs because he is afraid to go to Jail. X refuses to pay the
R100 000. Can Y enforce the acknowledgement of debt against X? Discuss (10)

Answer
X committed a crime of theft, by stealing from Y. On the other hand Y did not institute a legal action
for the crime; instead his actions were to remedy the matter with a counter act of inflicting undue
influence/duress on X, by making him sign an acknowledgement of debt.
A person who enters a contract through fear inspired by a lawful threat has no ground for complaint
since the factor that renders a contract voidable for duress is not fear per se but the obtaining of the
victim’s consent by improper means.
A threat to enforce one’s rights by bringing a civil action is not unlawful – except perhaps in the most
unusual circumstances. Therefore, a settlement reached to stave off the threatened action is not
voidable for duress.195 A threat to institute a criminal prosecution unless the debt is satisfied is
another matter altogether. It is generally agreed that such a threat is unlawful if it is used to extort a
benefit to which the creditor is not entitled, or if the debtor is innocent of the alleged crime. But if
the debtor has in fact committed a crime and the creditor seeks to obtain no more than his or her
due (for example, where an employer threatens to prosecute his or her employee for theft unless the
latter repays, or agrees to repay, a definite sum of money that he or she has stolen from the
employer), there is a difference of opinion. In a number of Transvaal and Natal cases, the view has
been taken that in these circumstances, a threat of criminal prosecution is not unlawful. In the Cape,
on the other hand, some doubt has been expressed about the correctness of these cases, and this
doubt is shared by a number of academic writers.198 It is argued, persuasively, that the use of the
criminal process to enforce strictly private rights is contrary to the public interest and thus unlawful;
and that, quite apart from duress, a contract induced by a threat of a criminal prosecution will
normally be void for illegality, on the grounds that it amounts in effect to an agreement to stifle a
prosecution or to compound a crime.

QUESTION 4
X, a tiler, concludes a contract with Y, in terms of which X will , on 1 November, tile the kitchen of Y's
new house for R20 000. On 1 November, X arrives at Y's house with his two helpers and all his
materials and equipment, but nobody is at home. He tries to telephone Y, but she is unavailable. He
waits for an hour but nobody arrives to let him into the house. He is particularly annoyed because he
turned down another job for that day, and he has to pay his workers even though they had nothing
to do. Y apologises to X later and asks X to come do the thing the following week X approaches you
for advice X can prove that he would have made a profit R 10 000 on the contract with Y and a profit
of R 12 000 on the job he turned down. The cost of the material is R7 8OO. The cost of transporting
the materials and equipment to Y's house and back is R1 000. The pay of his two workers is R1 200
for the day. Do not apply the Consumer Protection Act of 2008 to this problem. Advise X with regard
to the following

4 1 Did Y commit a breach of contract, and if so, what type? Discuss (5)

Answer
Yes, Y committed a breach of contract (mora creditoris), she failed to avail herself so that X could
perform his obligations to the contract.
The positive cooperation of the creditor is required in order to enable the debtor to perform his or
her obligation. A culpable failure to do so timeously constitutes the form of breach known as mora
creditoris. The requirements for mora creditoris;
1. Obligation to make performance:
2. Cooperation: The cooperation of the creditor must be necessary for the proper performance
by the debtor of his or her obligation.
3. For mora creditoris to exist, the creditor must delay in accepting performance: this case does
not give information as to agreement of time, did Y agree that the specific time was that of
the contractual obligation to receive the tiler.
4. Fault: the delay must be due to the fault of the creditor – Y was unavailable to receive service
or let the tiler and his workers into the house.

4 2 Can X validly cancel the contract? Discuss (5)

Answer
The consequences of mora creditoris include;
1. Cancellation: The debtor may cancel the contract where it has subsequently been made of
the essence by means of a notice of rescission, with which the creditor has failed to comply.
2. Damages: X is entitled to damages for any loss that has been suffered as a result of the
mora. These will take the form of costs wasted in having to re-transport the goods from the
agreed place of delivery, or which were incurred in storing merchandise or feeding animals
that were to be delivered; but they may also include loss of profit on the overall transaction
where the contract is cancelled since the debtor is in principle entitled to recover his or her
full id quod interest.
3. The view taken in some cases, under the influence of the doctrine of fictional fulfilment of
conditions, that the debtor’s obligations should be regarded as fulfilled if the creditor refuses
to cooperate cannot be supported. Unless the contract is cancelled, the debtor’s duty to
perform remains, but his or her delay in performing obviously cannot constitute mora
debitoris, being caused by the fault of the creditor.

4 3 Can X claim damages from Y and, if so, what amount? Discuss cancelled the contract with Y
because of Y's breach of contract (15)

Answer
Where the innocent party chooses not to rescind the contract, his or her damages include also the
loss, if any, that he or she has incurred on the transaction in question. The aim, in other words, is to
place the innocent party in his or her fulfilment position – that is, the position he or she would have
occupied had there been no breach. The application of the fundamental rule to a given set of facts
entails a comparison between two financial positions of the plaintiff: the actual position in which the
plaintiff now finds him or herself subsequent to the breach, and the hypothetical position he or she
would have occupied had there been no breach.71 This method of quantifying the damage caused
by a wrong is known as the difference theory.
In Trotman v Edwick,73 Van den Heever JA stated:
A litigant who sues on contract sues to have his bargain or its equivalent in money or in money and
kind. The litigant who sues on delict sues to recover the loss which he has sustained because of
wrongful conduct of another, in other words that the amount by which his patrimony has been
diminished by such conduct should be restored to him.
In line with this dictum, it is often said that contractual damages are measured according to the
plaintiff’s positive or expectation interest,74 and include loss of profit.
A person who has concluded a contract usually has an expectation that he or she will gain or profit
from the transaction. In order to realise that profit, however, he or she must perform his or her side
of the bargain, and in so performing or preparing to perform the party will often incur expenditure,
relying on the fact that he or she has a binding contract and that in due course this expenditure will
be recouped from the gross profits earned from the transaction.
Contractual damages may include both expectation and reliance losses.
The requirements for a damages claim are:
• a breach of contract by defendant;
• financial or patrimonial loss by the plaintiff;
• a factual causal link between the breach and the loss; and
• legal causation (the loss must not be too remote a consequence of the breach).
In terms of the difference rule, a plaintiff’s financial loss is determined by comparing the patrimonial
position occupied after the breach with the hypothetical patrimonial position that would have been
occupied had the contract been properly performed.

Y may claim the amount for transportation and the workers for the day R100 + R 1200, in the event
that the contract is not cancelled.

PVL3702_May 2018
QUESTION 1
X buys Y's car for R50 000 X pays the purchase price and Y delivers the car to X Y has the intention to
transfer ownership and X the intention to receive ownership. What type of agreement(s) did X and Y
conclude when Y delivered the car to X?
1 A contract
2 An agreement extinguishing a debt
3 A real agreement
4 Option 2 and 3
5 Option 1 , 2 and 3

QUESTION 2
X and Y conclude a contract. When will their contract be VOID?
1 The conclusion of the contract IS expressly prohibited by legislation as a crime
2 X operates under an error in motive
3 X threatens to kill Y if she does not conclude the contract
4 X made an intentional misrepresentation to Y before the conclusion of the contract
5 X made a dictum et promissum to Y before the conclusion of the contract

QUESTION 3
X advertises his car in the newspaper as follows
"Hyunda1 110 Price R 100 000. 2015 model 20 000 km Mint condition Phone X at 073 57695 "
Y phones X, who is in Johannesburg and offers to buy the car X accepts Y's offer Y, who is in Cape
Town, hears X's acceptance of Y's offer. Which statement is CORRECT?
1 A mere acceptance of the advertisement by Y would create a valid contract of sale
2 The statements regarding the year model, mileage and condition of the car are mere puffing
3 The contract of sale is concluded in Cape Town
4 The contract of sale is concluded in Johannesburg
5 Option 1 and 3
QUESTION 4
Liability for which form of misrepresentation may NOT be contractually excluded?
1 An innocent misrepresentation
2 A negligent misrepresentation
3 A grossly negligent misrepresentation
4 A fraudulent misrepresentat1on
5 A dictum et promissum

QUESTION 5/ VRAAG 5
S Illegally sells uncut diamonds top for R10 000. Both Sand P know that the selling of uncut diamonds
is prohibited by statute. Although S delivers the diamonds to P, P falls to pay the purchase price.
Which statement is CORRECT?
1 S will be able to claim the purchase price from P.
2 The contract sale IS void because of Illegality
3 The contract of sale IS voidable because of illegality
4 S shall be able to claim return of the diamonds from P with an enrichment action, because the
court as a rule will relax the par delictum rule
5 S shall be able to claim return of the diamonds from P with an enrichment action because the
par delictum rule does not apply in the circumstances

QUESTION 6
Which of the following is NOT a requirement for the creation of a valid contract?
1 The prescribed formalities must be complied with
2 The performance must be possible after the contract is concluded
3 The contract must be certain
4 The contracting parties must have capacity to act
5 There must be agreement or apparent agreement between the parties

QUESTION 7
X and Y agree that should X sell her car, she (X) will offer to sell it to Y first, before making an offer to
sell the car to any other person X sells the car to Z for R50 000 without first offering it to Y for sale
Delivery of the car has not yet taken place. Which statement is INCORRECT?
1 X gave Y an option to purchase her (X's) car
2 X gave Y a right of pre-emption with regard to her (X's) car
3 X can step into the shoes of Z with a unilateral declaration of intent
4 Both Y and Z only have personal rights against X
5 The personal right of Y enjoys preference above the personal right of Z

QUESTION 8
Essentialia are
1 All the terms of a contract apart from the naturalia
2 Terms that identify a contract as belonging to a particular class of contracts
3 Terms automatically imposed by law on the contracting parties unless contracting parties expressly
exclude them
4 All the terms of a contract apart from the incidentalia
5 Material terms and conditions of a contract

QUESTION 9
X sells land to Y subject to the existence of pumping rights from a stream. Such a term is a
1 warranty
2 unilateral mistake
3 suspensive condition
4 suppositions
5 potestative suspensive condition

QUESTION 10
X finds himself stranded in the remote Richtersveld when his 4x4 vehicle breaks down. There is no
cell phone reception in the area. After five days have passed without seeing anyone, Y comes along
and offers to tow X to the nearest town for the exorbitant fee of R20 000, because Y knows that X is
running low on food and water X accepts Y's offer. Y's conduct will probably amount to
1 undue Influence
2 duress
3 abuse of circumstances
4 opt1on 1 and 3.
5 option 1, 2 and 3

QUESTION 11
In which Instance has consensus been obtained improperly?
1 A common error
2 Rectification
3 An error in negotio
4 A threat that is not related to an imminent or inevitable evil
5 A negligent misrepresentation
QUESTION 12
S informs P that there are 1 000 fruit trees on his (S's) farm, as a result of which P buys the farm from
S for R8 000 000. S knows that there are only 800 fruit trees on the farm. This is a case of
1 an error in negotio
2 a fraudulent misrepresentation
3 a negligent misrepresentation
4 option 1, 2 and 3
5 none of the above options

QUESTION 13
Assume the same facts as in question 12. The market value of the farm is R7 800 000 P claims
R200 000 from S P's claim against S is based on
1 delict
2 breach of contract
3 unjustified enrichment
4 monetary compensation
5 all the above

QUESTION 14
Y sells his cat, Felix, to Z for R1 000 on 15 January. Delivery is to take place on 17 January Lightning
kills the cat on 16 January This is a case of
1 mutual mistake
2 impossibility of performance
3 supervening impossibility of performance
4 prevention of performance
5 common mistake

QUESTION 15
X sells his car to Y for R50 000 and undertakes to deliver the car to Y tomorrow morning. The
obligation to deliver the car IS subject to a
1 suspensive time clause
2 resolutive time clause
3 suspensive condition
4 resolutive condition
5 modus

QUESTION 16
X sells his car to Y for R50 000 If it rains in the Karoo next Monday This is a contract of sale subject to
a
1 suspensive time clause
2 resolutive time clause
3 suspensive condition
4 resolutive condition
5 modus
QUESTION 17
Which one of the following statements regarding tacit terms is INCORRECT?
1 The courts often employ the officious bystander test in determining whether a contract contains a
tacit term
2 Tacit terms will only be read into a contract If they do not conflict with any of the unambiguous
express terms of the contract
3 Tacit terms will only be read into a contract if it IS necessary in a business sense to give efficacy to
the contract
4 Tacit term must be capable of a clear and exact formulation
5 Tacit terms are the naturafia of the contract

QUESTION 18
Which of the following must a party prove who seeks to set a contract aside on the ground of undue
influence?
1 The other party obtained influence over her
2 The other party exercised this influence over her, in an unscrupulous manner in order to induce her
to consent to a transaction which IS to her detriment and wh1ch she, with a normal free will, would
not have concluded
3 The other party gained his influence over her by standing in a close relationship to her
4 Options 1 and 2
5 Options 1, 2 and 3

QUESTION 19
Y sells his car to Z for R20 000 on 15 January Y undertakes to deliver the car to Z on 17 January, but Z
fails to accept delivery of the car. This failure of Z amounts to
1 mora creditoris
2 subjective impossibility of performance
3 subjective supervening impossibility of performance
4 mora debitoris
5 prevention of performance

QUESTION 20
B, a builder, and O agree that B will build a house on O's stand for R800 000 and that B will only be
paid after completion of the house, which must be no later than 1 August. On 1 August, the house is
only 20% complete O refuses to pay B any money, because B is in breach of contract O's refusal
amounts to
1 repudiation
2 positive malperformance
3 prevention of performance
4 mora debitoris
5 none of the above
QUESTION 21
Which statement is INCORRECT?
1 A contract can be a bilateral Juristic act
2 A contract entails prom1ses or undertakings on one or both sides
3 An undertaking in a contract that a certain state of affairs exists, or has existed, is known as a
warranty
4 The conclusion of a contract can never be multilateral
5 Freedom of contract means that the parties can agree to anything that is possible and lawful

QUESTION 22
Which of the following does NOT lead to the termination of an offer?
1 Extension of the time set for acceptance of the offer
2 Rejection of the offer
3 Death of either the offeror or the offeree
4 Effluxion of the prescribed time, or of a reasonable time
5 Revocation of the otter
QUESTION 23
Which statement regarding cancellation for breach of contract is INCORRECT?
1 Where a party has a right to cancel a contract, he/she has an election either to affirm or to cancel
the contract
2 A party may cancel a contract where the breach of contract on the part of the other party is
material
3 A party may cancel a contract where provision has been made in the contract for a right to resile in
the Circumstances that exist
4 Cancellation can be claimed with the actio quanti minoris
5 Cancellation of a contract is available to a party only in exceptional Circumstances

QUESTION 24
What IS the general effect of supervening impossibility of performance?
1 Termination of the contractual obligation
2 The contract IS rendered voidable at the Instance of the creditor
3 The contract is rendered voidable at the Instance of the debtor.
4 The creditor is guilty of contractual breach
5 The debtor IS guilty of contractual breach

QUESTION 25
X owes R700 to the municipality for electricity used in September It turns out that the municipality
owes X R400 as it over-charged him in respect of electricity used in August X therefore only pays the
municipality R300 This Is a case of
1 novation
2 release
3 merger
4 set-off
5 compromise
QUESTION 1
Y signs and delivers a written offer (including all the material terms) to Z on 1 July, for the purchase of
Z's apartment Y's offer IS for R800 000 and one of the terms of the offer states "This offer lapses on
30 August " Z accepts the offer by signing the contract Z posts the contract to Y on 28 August The
letter reaches Y on 1 September Y immediately opens the letter and not1ces that Z has signed the
contract Did Y and Z conclude a valid contract of sale? Discuss You may accept that the contract
complies with the required formalities (5)

Answer
Contracting parties may enter into an agreement in terms of which the offeror undertakes not to
revoke his or her offer. In such cases, it is said that one party grants the other an option.
For this question, an option does not exist because there is no agreement in place that binds Y to
keep his offer open until 30 August. Y has unilaterally imposed this upon himself in the offer, but it
was certainly not an agreement by both parties to hold Y to keep his offer open until this date. This
means that no option contract was concluded.
Y validly revokes his offer to Z on 20 July and therefore there is no offer that Z can accept. The
requirements for a valid offer and acceptance for a contract have not been met, and no valid contract
has thus been created.

In Brandt v Spies (above), the defendant orally granted an option to the plaintiff to purchase his farm.
Disregarding the option, he sold the farm to a third party.
The plaintiff, who had exercised the option in writing, then claimed damages for breach of contract,
but an exception to his claim was upheld. The court stated as follows:
If the offer is not in writing there is nothing which the offeree can accept so as to create a vinculum
iuris between himself and the offeror. An undertaking to keep open an offer which is incapable of
forming the basis of a valid contract can itself confer no right upon the grantee for in law there is
nothing to keep open.
This judgment, it is submitted, is correct in the case where the offer to sell and the offer to keep the
first offer open are made simultaneously and orally. As regards the contract of sale, both offer and
acceptance must be in writing in accordance with the above-mentioned legislation, and the offer has
been made orally in such a case. However, what of the case where the offer to sell is made in writing
on one occasion and is later followed by an oral offer to keep the offer to sell open? The Act provides
that no alienation of land will be valid unless it is reduced to writing. The option contract which
comes into question with the acceptance of the oral offer is not an alienation of land and,
consequently, one can conclude that it need not be in writing. What must be in writing is the offer to
sell.

QUESTION 2
X is employed as a bookkeeper in Y's business X steals money from the business's bank account over
a long period of tine Y gets forensic auditors in and they determine that X has stolen R50 000 Y
confronts X and threatens to lay a charge of theft against X at the police station unless X signs an
acknowledgement of debt for R50 000. X signs because he is afraid to go to Jail Did a valid contract
arise? Advise Y. Substantiate your answer (10).

Answer
X committed a crime of theft, by stealing from Y. On the other hand Y did not institute a legal action
for the crime; instead his actions were to remedy the matter with a counter act of inflicting undue
influence/duress on X, by making him sign an acknowledgement of debt.
A person who enters a contract through fear inspired by a lawful threat has no ground for complaint
since the factor that renders a contract voidable for duress is not fear per se but the obtaining of the
victim’s consent by improper means.
A threat to enforce one’s rights by bringing a civil action is not unlawful – except perhaps in the most
unusual circumstances. Therefore, a settlement reached to stave off the threatened action is not
voidable for duress.195 A threat to institute a criminal prosecution unless the debt is satisfied is
another matter altogether. It is generally agreed that such a threat is unlawful if it is used to extort a
benefit to which the creditor is not entitled, or if the debtor is innocent of the alleged crime. But if
the debtor has in fact committed a crime and the creditor seeks to obtain no more than his or her
due (for example, where an employer threatens to prosecute his or her employee for theft unless the
latter repays, or agrees to repay, a definite sum of money that he or she has stolen from the
employer), there is a difference of opinion. In a number of Transvaal and Natal cases, the view has
been taken that in these circumstances, a threat of criminal prosecution is not unlawful. In the Cape,
on the other hand, some doubt has been expressed about the correctness of these cases, and this
doubt is shared by a number of academic writers.198 It is argued, persuasively, that the use of the
criminal process to enforce strictly private rights is contrary to the public interest and thus unlawful;
and that, quite apart from duress, a contract induced by a threat of a criminal prosecution will
normally be void for illegality, on the grounds that it amounts in effect to an agreement to stifle a
prosecution or to compound a crime.

QUESTION 3
X IS a keen golfer who has played at many golf tournaments over the years as an amateur. She is very
well Informed about the rules pertaining to her amateur status as a golfer and knows that amateurs
can only claim a maximum of R1 000 in prize money at golf tournaments X participated in a recent
golfing tournament wherein she achieved a hole-an-one at the 9th hole. At this hole was an
advertising board, which read "Hole-In-one prize sponsored by Speedy Motors to the value of R90
000" The prize was parked next to this board in the form of a new car X claimed the prize from
Speedy Motors but they rejected her claim on the basis that the prize could only be claimed by
professional players and not amateur players. Did a valid contract arise? Advise X Substantiate your
answer (15)

Answer
This problem deals with two questions: Was there a valid offer and acceptance? Was there
consensus between the parties?
Offer and acceptance:
The general rule in our law is that an advert constitutes merely an invitation to do business (Crawley
v Rex). However, following the reasoning in Carlill v Carbolic Smoke Ball Co, the court in Bloom v
American Swiss Watch Co held that the advertising of a reward might be construed as an offer to the
public.
An offer may only be accepted by a person or persons to whom it was directed (Bird v Summerville).
Although Speedy Motors intended the offer to be open only to professional players, the expressed
offer was apparently open to the public. Mistake is thus also relevant.
Mistake:
At the outset, it must be determined whether agreement (consensus ad idem) as a contractual basis
exists between the parties, as required in terms of the will theory. Consensus has three elements:
1. The parties must seriously intend to contract
2. The parties must be of one mind as to the material aspects of the proposed agreement (the terms
and the identities of the parties to it) 3. The parties must be conscious of the fact that their minds
have met
In our case, X and Speedy Motors were not in agreement as to the identity of the parties, and this is
a material mistake, which excludes consensus based on the will theory.
However, the matter does not end here. A party may be held contractually liable on the basis of a
supplementary ground for liability, namely the reliance theory. In this regard, the direct reliance
approach or the indirect reliance approach may be considered. Because the facts in this case are
similar to the case of Steyn v LSA Motors where it was held that the indirect approach couldn’t be
applied in instances where there is no objective appearance of agreement, only the direct approach
will be considered.
DIRECT APPROACH:
With reference to the direct approach, contractual liability is based on the reasonable reliance that
consensus has been reached, which the one contractant (the contract denier) creates in the mind of
the other contractant (the contract enforcer). According to the Sonap case, the direct reliance
approach entails a threefold enquiry:
1. Was there a misrepresentation regarding one party’s intention?
2. Who made this misrepresentation?
3. Was the other party actually misled by the misrepresentation, and if so, would a reasonable
person have been misled?
In our question, Speedy Motors made a misrepresentation regarding its intention that the offer is
made only to professional players, by advertising the reward to the public. Although it may be argued
that X was actually misled by the misrepresentation, it is certain that a reasonable person in X’s
position would not have been misled. X should know, as an experienced amateur golfer, that only
certain prizes are open to amateurs. There was therefore no reasonable reliance on consensus on
the part of X. X will not succeed in her claim for the prize.

QUESTION 4
Mpho and Craig conclude a contract wherein Mpho agrees to paint Craig's office block by 31 January,
and Craig agrees to pay Mpho R 10 000 R5 000 at commencement of the contract and R5 000 upon
completion of the work. When 40% of the work is completed Mpho refuses to continue painting
unless Craig pays him R15 000 for the work Craig refuses to pay Mpho more for the work and Mpho
leaves in a rage, never to return. Did Mpho repudiate the contract (a form of breach the contract)?
Discuss (5)

Answer
Yes
A party to a contract commits the breach of repudiation when, by words or conduct, and without
lawful excuse, he or she manifests an unequivocal intention no longer to be bound by the contract or
by any obligation forming part of the contract. The intention to repudiate is judged objectively, the
test being whether the party accused of repudiation has acted in such a manner as to lead a
reasonable person to believe that he or she does not intend to fulfil, or completely fulfil, his or her
part of the contract. A refusal to perform ones end of the contract is a form of breach (repudiation).
According to the traditional approach, which still regards the act of repudiation as being analogous to
an offer to rescind, the breach is fully constituted or completed only when the repudiation is
accepted by the innocent party; and if rejected the repudiation is a nullity with no legal effect at all.

QUESTION 5
Assume the same facts as in question 4. Also, assume that Mpho has repudiated the contract. Can
Craig rescind the contract with Mpho? Discuss (5)

Answer
A repudiation of the entire contract will always entitle the innocent party to rescind. the agreement
would be terminated by mutual consent.

QUESTION 6
Assume the same facts as in question 4. Also, assume that Mpho has breached the contract and that
Craig has validly rescinded the contract. It costs Craig R7 000 to complete the paintwork. Can Craig
claim damages from Mpho and If so, what amount? Substantiate your answer {10).

PROPERTY CASES:
REAL RIGHTS v CREDITORS RIGHTS
Ex Parte Geldenhuis:
Facts: husband and wife left their land to their children in a will. The land would go to them as co-
owners when the eldest child reached majority. The land would then be divided into portions by
drawing lots. The child who got the land with the house on had to pay the others compensation.
2 conditions:
1. The time and manner of sub-division and usually co-owners are free to decide when to sub-divide-
if the obligation is on the land= subtraction from dominium and it can be registered.
Therefore subsequent owners are also bound.
2. The child with the house has to compensate the others- if the burden is on a specific person in his
personal capacity = creditors right and it can't be registered - subsequent owners won’t be bound.
The 1st obligation diminishes a co-owners rights of sub-division and was intended to do so =it
therefore affects all subsequent owners and is a burden on the land= REAL RIGHT AND MUST BE
REGISTERED
The 2nd obligation places a burden on one child only - it was intended to restore the balance- it's a
burden on a specific person in his personal capacity and doesn't subtract from ownership=
CREDITORS RIGHT AND THEREFORE CANT BE REGISTERED.
In this case the court decided to register them together for convenience.
In the Lorenz case: the court said that limited real rights could only be created when the result in the
subtraction amounted to a subtraction of the owner's physical use of the property.
In Pearly Beach case: court reverted back to the normal subtraction test as established in Geldenhuis.

OWNERSHIP AND LIMITATIONS ON OWNERSHIP


Pa alardo v Hau:
The resident of a township in Gauteng was ordered by the High court to allow his neighbour to insert
a drainage port in the wall, which was on the common boundary between them. He appealed the
courts decision.
The purpose of the drainage ports was to allow rainwater, which had gathered on the neighbour's
side to flow down the natural slope of the 2 properties onto the appellants land.
The High Court held: that the appellant as the owner of the lower lying property was obliged to
accept the water flowing from the higher lying neighbour based on the natural flow of the water.
On appeal the SCA: the applicant's obligation in this situation is limited to the NATURAL FLOW of
water between the 2 neighbouring properties.
Since there was no proof of a natural flow, the right to drain the water through the drainage ports
should NOT be allowed.
The appeal was successful
Malherbe v Ceres Municipality
Facts
The appellant, Malherbe, approached the court for an interdict ordering the respondent, Ceres
Municipality, to prevent acorns and leaves of oak trees growing next to the streets of Ceres from
falling onto his property. The appellant claimed that the oak trees constituted a nuisance on his
property in that the falling oak leaves blocked the gutters of his building, thereby causing rainwater
to damage the walls of the building.
Legal question
To determine whether falling leaves, acorns and protruding branches of trees growing next to streets
constitute a nuisance.
Ratio decidendi
General The law expects a degree of tolerance between neighbours in the exercise of their
entitlements of ownership.
Regarding leaves from trees next to the street
The planting of oak trees along the streets of towns and cities is considered to be compatible with
the natural and normal use of streets in the Western Province. Oak trees are benign, as well as being
ornamental and shade giving. If their leaves are blown onto neighbouring premises by the wind, then
the owners of those current owners of the farm cannot be held responsible for the damage caused
by the use of the property by a previous owner. The court further held that neighbour law is based
on the principle of reasonableness. If it was reasonably possible for the current owner to prevent the
damage from happening again in future, the failure to do so would amount to an unlawful act.
In the case of such an unlawful act the neighbour would be entitled to one or both of the following
remedies against the current owner:
(i) an interdict and/ or
(ii) a delictual claim for damages.
The court held that the current owner had acted reasonably and the application for an interdict did
not succeed.
Application of finding on relevant facts
X and Y cannot hold Q and R liable for damage caused by Z's use of the property. If it was reasonably
possible for Q and R to prevent the damage from happening again in future, the failure to do so
would amount to an unlawful act. This would then entitle X and Y to an interdict and/or a delictual
claim for damages against Q and R. In Regal v African Superslate (question based on the facts in this
judgment) the court held that the current owner, in our question Q and R, had acted reasonably.
Appellant objected to the valuation, arguing that the oil tanks weren't buildings and didn't constitute
immovable property.
Held: it was decided that the nature and physical features of the tanks in question, the method of
their construction and attachment and the difficulties involved in removing them = all indicate an
intention to attach them to the land permanently.
The court accepted that the intention is the intention of the annexor at the time of attachment.
On appeal the Supreme Court held that the tanks didn't constitute immovable property - there
would be no damage to the land to remove them.

Konstanz Property (Pty} Ltd:


Facts: the appellant through his farm manager concluded a contract with P (a close corporation) in
terms of which P was to install an irrigation system on the appellant's farm.
P bought the components of the system from the respondent, a wholesaler.
In terms of the purchase agreement, the respondent reserved ownership of each item pending
payment of the full purchase price, P installed the system on the farm and was paid by the appellant,
but failed to pay the respondent.
The respondent got default judgment against P, in terms of which the sheriff was ordered ion the
event of non-payment to attach the equipment.
The appellant applied for a declaratory order, to the effect that the appellant was the owner of the
items.
2 issues:
a) Whether the equipment had become part of the property through attachment
b) Whether the respondent was prevented on the basis of estoppel, from raising its reserved
ownership against an innocent purchaser of the property.
Held: the court a quo decided in favour of the respondent and ordered the appellant to allow the
respondent to remove the equipment.
The appellant appealed.
Remedies for the owner of a movable thing, who loses his ownership because of accession to the
landowner:
a) If its possible to detach the accessory thing, he can bring the rei vindicatio
b) He can claim an enrichment action for compensation for loss of his movable

Info Plus:
IP instituted an action against S for the delivery of a car, which they alleged, was their property. S
admitted possession of the car but denied IP's ownership.
IP bought the car from Wunder an instalment sale transaction in which W retained ownership until
the payment of the full purchase price. Before the payment of the last instalment IP delivered the car
to a dealer and instructed them to sell it.
M bought the car from the dealer on the strength of a fraudulent registration certificate, which
stated that the car had been paid for in full. M then sold and delivered the car to S.
When M was told that the car was allegedly stolen from IP, he paid the outstanding balance of the
original purchase price to W.
The question was if and when IP had become the owner of the car
The court held: IP couldn't prove ownership, as they had not been in possession of the car at the
time when the last instalment was paid restored. The courts therefore do not investigate the merits
of the rights of the parties. Relevant cases are Nino Bonino.

Question
What are the requirements that Z will have to prove to succeed with the spoliation remedy?

Answer
Z will have to prove that he had
1. (i) peaceful and undisturbed control of the billiard room and that
2. (ii) S disturbed his control in an unlawful manner
Question
(d) Will Z's aP.plication succeed? Refer to case law. (5)

Answer
Z will be successful because it is clear that S disturbed Z's peaceful and undisturbed control in an
unlawful manner. S cannot rely on the clause in the lease agreement entitling him to take control of
the premises. This will entitle to take the law into his own hands which is against public policy. The
clause would entitle S to be the judge in his own case since he can decide if there was a breach of
contract. Nino Bonino v De Lange.

Telkom SA Ltd v Xsinet (Pty) Ltd


X was an Internet provider and to do its business it required telecommunications from Telkom, X
claimed that they had paid all the charges to Telkom and claimed a contractual right to the use of the
systems and alleged that they were in peaceful and undisturbed control of the systems when they
were disconnected.
There was a dispute between the 2 regarding the payment of certain charges and Telkom advised X
that if the amount wasn't paid in terms of the contract they would disconnect their services.
The amount wasn't paid and X was disconnected, this caused a considerable loss for the company.
X alleged that the disconnection was unlawful deprivation and applied for a spoliation order.
The Court ordered that Telkom reconnect and restore the use and possession of the lines and was
liable to pay the costs of the application.
On appeal: the court said that X's use of the services amounted to a personal right in terms of the
contract and thus they were not allowed the protection of the mandement van spolie.
The appeal was allowed with costs
SERVITUDES: LIMITED REAL RIGHTS
Grant v Stonestreet
Facts: illustrate the application of the doctrine of knowledge.
A right of aqueduct was created by means of agreement in 1865 between the owners of farm A
(dominant tenement) and farm B, but it was never registered. The successor in title of the owner of
farm B had no knowledge of the right and it was never enforced against him – the next successor of
farm B did have knowledge of the written agreement. When the owner of farm A tried to enforce it
by means of a court order, he claimed the right was only a creditor's right and was only enforceable
between the contracting parties
The court decided:
a. The unregistered servitude agreement is enforceable against the contracting parties
b. Because the successor in title to the owner of farm B had no knowledge of the agreement, it's not
enforceable against him
c. The next successor of farm B did have knowledge of the agreement so its enforceable against him
because of the doctrine of knowledge- the agreement wasn't terminated because of the previous
owner of B's lack of knowledge of the agreement
d. The doctrine of knowledge will only be applicable if the contracting parties intended to bind their
successors in title.

Willoughby’s Consolidated
This case deals with acquiring a personal servitude- right to trade on a certain piece of land.
The defendants bought D stores with all its assets including the right to trade on the land belonging
to E.
Later the plaintiffs bought the assets of E, including the land.
The defendants claimed that they had the right t trade on the land, that the plaintiff knew of this
when they bought the land (DOCTRINE OF KONWDLEGE GRANT V STONESTREET ADD) and that the
plaintiff was bound to it just as E was.
The plaintiff applied for a declaratory order to state that the defendant didn't have servitude over its
property
A servitude (real right) can be created by an agreement, but such an agreement is only binding on
the contracting parties, in order for such.

LIMITED REAL RIGHTS:


REAL SECURITY
Ikea Trading Und Design v BOE Bank Ltd:
The court looked at Sl (1) of the Security BMO Movable Property Act.
The SCA held that notarial bond must describe the property in such a way that it can be recognized
from the bond itself.
The court found that Ikea didn't have a secured claim in an insolvent estate where the assets in
respect of which it claimed security wasn't identified in the bond in such a way that it could be
recognised by a 3rd party or creditor

Mapenduka v Ashington
Looking at a pactum commissorum for pledges: on non-payment of a loan secured by a pledge the
creditor could cancel the agreement and keep the property- in this case such an agreement is VOID

Qsry
S hands ostrich feathers to Z to sell on his behalf. Z is unable to sell all the feathers. Z advances a
large sum of money to Sand agrees that he willtry to sell the feathers, if necessary, at a public
auction. The feathers will serve asa pledge object for repayment of the money advanced to S. Z sells
the feathers at a public auction and buys them for a very low price. S argues that this sale is invalid
since it was executed in terms of an invalid summary execution clause.
Legal question
Is the summary execution clause valid?
Ratio decidendi
Sand Z agreed to a clause for summary execution. In terms of this clause the pledgee (Z) and the
pledgor (S) agreed that in the case of default on payment, the pledgee may sell the thing without an
execution order from the court. This type of clause is valid in principle. Should the pledgor (S) be
prejudiced by the sale, however, he can seek the protection of the court.
Application of decision to facts
In principle a summary execution clause is valid. S might, however, succeed if he can prove that he
has been prejudiced by the way Z carried out the sale of the feathers.

CONSTITUTIONAL LAW IN RESPECT OF PROPERTY


FNB v COMMISSIONER, SARS:
A deprivation can be described as a properly authorised and fairly imposed limitation on the use and
enjoyment or disposal of property for the sake of protecting and promoting public health and safety,
normally without compensation. Examples would be land-use planning, developmental and
environmental conservation measures
The following two requirements have to be met for a deprivation to be valid:
1. A deprivation must take place in terms of a law of general application. Deprivations of property
usually occur as a result of a statu tory provision.
2. A deprivation of property may not take place arbitrarily.

In First National Bank of SA Limited t a Wesbank v Commissioner for the South African Revenue
Services; First National Bank of SA Limited t/a Wesbank v Minster of Finance (2002) (7) BCLR 702 CC
the court concluded that a deprivation of property is "arbitrary" when the "law" that is referred to in
section 25 does not provide a sufficient reason for the deprivation or limitation or if it is procedurally
unfair.
FNB is a financial institution that sells and leases movables. They were the owner of 3 cars, which
were detained under S114 of the Customs and Excise Act. The commissioner and Minister of Finance
were the persons responsible for the administration. The commissioner intended to sell the cars to
recover unpaid duties and penalties by the importers.
The applicant contended that the detention and anticipated sale of the vehicles amounted to an
expropriation and was inconsistent with S25 and therefore invalid.
The court looked at the meaning of S25 broadly and after establishing the purpose of S25 established
that deprivation of the property had occurred =the dispossession of the owner of his, rights, use and
benefit to movable, corporeal property is an example of deprivation.
Expropriation occurs where the state forced the transfer of the property right to itself or a 3rd party.
The court held that deprivation of property is arbitrary if it doesn't provide sufficient reason for the
deprivation or the procedure is unfair.
The plaintiffs argued arbitrary deprivation and claimed just and equitable compensation or
constitutional damages,

Question
Sources of the law of things:
Answer
1. The Constitution of the Republic of South Africa, 1996 (1)
2. Statutory law (1)
3 . Case law (1)
4. Common law (Roman-Dutch law) (I)/ indigenous (customary) law (l)

Question
A real right is a lawful real (1) relationship between a legal subject and a thing (1), which confers
direct control ( 1) over the thing ( 1) on the legal subject, as well as the relationship between the
legal subject and all other legal subjects (1) who must respect this relationship.

Question
The function of the law of things can be summarised as follows:
1. It strives to harmonise or regulate various competing ownership rights, especially between
neighbouring owners
2 . It strives to harmonise or regulate an owner's rights in regard to his/her thing with the rights of
other limited real right holders to the same thing
3. It controls the acquisition, protection and extinction of things and real rights.
4 . Therefore a real right can be defined as a lawful real relationship between a legal subject and a
thing, which confers direct control over the thing on the legal subject, as well as the relationship
between the legal subject and all other legal subjects who must respect this relationship.

Question
A thing is a legal object characterised by its material (corporeal) nature. For a complete picture of a
thing in a legal sense, we define a thing as an independent part of the corporeal world, which is
external to humans and subject to human control, as well as useful and valuable to humans.

Question
Real and personal rights under the following headings:
1. Object
2. Absoluteness
3. Preference
4. Publicity
(i) Object

The object of a real right is a corporeal thing (1./2); whereas the object of a personal right is
performance. (1./2)
(ii) Absoluteness Real rights are absolute in principle: ( 1./2) the holder of the right can vindicate
his/her thing (subject to certain exceptions) from whomever is in control of the thing; while personal
rights are relative in principle: (1./2) the holder can enforce his/her right only against the person who
is obliged to perform in terms of an obligation (con tract or delict) .
(iii) Preference
In the case of insolvency, a real right enjoys preference over other rights . (1./2)
Moreover, the maxim, first in time is stronger in law (prior in tempore est potior in jure) is applied in
the case of two or more competing real rights . Apart from a few exceptions, this principle does not
apply to personal rights. (1./2)

(c) S
Limited real right: ( 1) The bank's limited real right of mortgage ( 1) limits S's ownership of Highlands.
During the currency of the mortgage (as long as S's principal debt to the bank has not been paid), S's
ownership is restricted in the sense that he is not entitled to sell or burden the farm without the
permission of the bank. (1) Furthermore, if S is unable to pay the loan (1) in terms of the mortgage,
the bank may approach the court to declare the farm executable.

QUESTION 2:
Briefly discuss the difference between ownership and limited real rights. (6)

Answer
The difference between ownership and limited real rights lies in the fact that ownership is a real right
over one's own thing, (1) whereas limited real rights are real rights to another person's thing. ( 1)
Furthermore ownership is the most comprehensive (1) real right a person can have to a thing,
whereas limited real rights are fundamentally limited (1) in scope. For example, in principle the
owner of a piece of land can use it as he/she wishes, (1) whereas the entitlements of a usufructuary
(limited real right holder in terms of a personal servitude) are clearly defined .
QUESTION 3:
How is the subtraction from the dominium (ownership) test formulated in Ex parte Geldenhuys (1926
OPD 155)? (7)

Answer
One has to look not so much to the right, (1) but to the correlative obligation (1). If that obligation is
a burden upon the land, (1) a subtraction from the dominium [ownership], (1) the corresponding
right is real and registrable; (1) if it is not such an obligation, but merely an obligation binding on
some person or other, ( 1) the corresponding right is a personal right, (1) or right in personam, and it
cannot as a rule be registered.

Ownership
Ownership is the most comprehensive (1) real right (1) a person can have with regard to a thing (1).
In principle, a person can act upon and with his thing as he/she pleases. (1) This apparent freedom is
restricted (1) however, by the law (1) and the rights of others. (1)

Co-ownership can be defined as the situation where two or more persons (1) own the same thing (1)
at the same time (1) in undivided shares. (1) Two forms of co-ownership can be distinguished ,
namely free co-ownership (1.12) and bound co-ownership. ( ½ )

Expropriation can be defined as an original method (1) of acquiring ownership in terms of which the
State (1) acquires ownership (1) of a movable (1.12) or immovable ( 1.12) thing without the consent
of the owner ( 1) against payment of compensation. ( 1)

Accession can be defined as an original method of acquiring ownership, which takes place when an
accessory thing becomes merged with a principal thing, with the result that the two things form one
entity. The accessory thing loses its
1. Minors ( 1)
2. Insane persons (1)
3. Married women with retention of the husband's marital power (although the marital power was
completely abolished in section 29 of the General Law Fourth Amendment Act 132 of 1993, this
should not be interpreted to the detriment of women against whom prescription was running during
the period when they were still subject to the marital power of their husbands) (1)
4. Persons absent from the country because of war, or those who are employed by the state (1)
5. Fideicommissaries - in the case where a fiduciarius has alienated fideicommissary goods without
the power to alienate, until such time as the fideicommissary goods are distributed. (1)

QUESTION 4:
Name the requirements for "attornment". (5)

Answer
A tripartite agreement (1)
Between the transferor, the transferee and the third party holder (1)
In terms of which the holder will continue to hold for the transferee (1)
And no longer for the transferor ( 1) -all three parties therefore consent to the transfer of ownership
(1).
The holder should exercise physical control (1) At the moment (1) of transfer from the transferor to
the transferee.

QUESTION 5:
Name the two categories of limitations on ownership and give one example of each (4)

Answer
These limitations on ownership can be categorised as follows:
Limitations imposed by law (1):
o Statutory limitations
o Limitations imposed in terms of neighbour law principles
Limitations imposed by the rights of other legal subjects (1)
o The limited real rights of third parties
o Limitations imposed by the personal rights (creditor's rights) of third parties

QUESTION 6:
A mining company erected an electrical substation and steel towers on its farm to convey electricity
to its mining operations. The installations and substation were erected in accordance with Eskom's
specifications. Eskom supplied power to the mining company. Subsequently the mining company
ceased operations and Eskom no longer supplied it with power.
In 1981 Rollomatic purchased certain steel towers, which had previously been part of the electrical
substation, from the mining company. In terms of the sales contract Rollomatic was responsible for
the removal of the steel towers and their concrete foundations and for restoring the land to its
original state.
Delivery of the steel towers to Rollomatic would take place after the latter had owner of the steel
towers and consequently it was not entitled to an order for delivery of the towers. (1)

QUESTION 7:
X and Y are seriously affected by baboons that destroy their maize plants. X installs an apparatus to
chase away the baboons on the boundary with his neighbour. The apparatus makes loud noises at
regular intervals during the day and the night. The neighbour writes to X and Y to complain about the
noise during the night, but X ignores the letter and refuses to speak to his neighbour on the
telephone. X and his neighbour are not on speaking terms because his neighbour seriously insulted
him a few years ago. The neighbour applies for an interdict ordering X andY to stop the noise. X andY
rely on their ownership of the farm. They argue that they are entitled to do as they please on their
property. Will this argument succeed? Substantiate your answer and refer to case law. (10)

Answer
Limitations on ownership
1. Statutory limits: Movables: Arms and Ammunition Act, Immovable's:
Physical Planning Act
2. Neighbour law
3. Limited real rights of others
4. Creditor's rights of others.

NEIGBOUR LAW NB :
Purpose: regulate and harmonize conflicting ownership rights. The court weighs up
the interests of the neighbors:
a) The owner must exercise his rights reasonably
b) The neighbor must tolerate this within reason

NUISANCE:
In the narrow sense: Gien v Gien
Nuisance occurs where neighbor's right to personality or his entitlement of use is infringed. E.g.
noise, smell, gas
The test is one of reasonableness.
The court looks at:
1. Nuisance must be repetitive
2. Location - residential/ industrial and the habits of the people living there
3. Were the acts of the respondent reasonable and fair (Gien)
4. Did the respondent act in bad faith (Gien)
5. Was the respondents use of the property normal (Gien)
6. The normal person must regard it as a nuisance - mustn't be over sensitive
7. Unreasonableness must be regarded as such in the eyes of the community
PV 3703 – LAW OF
DELICT
2022 – SEMESTER 2
FINAL PORTFOLIO
OCT/NOV 2022
DUE: 12 OCT 2022
QUESTION 1

Lesego works as a messenger and driver for the University of Summer. Her job is to
make deliveries on behalf of the University of Summer, mainly for the office of the
principal, Prof Bontle. While driving at 100km/h along a freeway to make a delivery at
the University of Winter, Lesego suffered a heart attack that caused her to lose
consciousness immediately. Her vehicle swerved out of control and crashed into a car
next to hers, moving in the same direction. Mpho, the driver of the other car, sustained
bodily injuries and was hospitalised for a period of six months. After his discharge from
hospital, Mpho instituted a delictual action against the University of Summer, the
employer of Lesego. Prof Bontle on behalf of the University of Summer raised the
defence that Mpho’s injuries were not caused by any conduct of its employee, Lesego,
because at the time of the collision, Lesego was unconscious. Discuss the merits of this
defence with specific reference to the theoretical background thereof.(15)

Conduct is defined as a voluntary human act or omission. “Voluntary” means that the
person must be able to control his muscular movements by means of his will. The act
of the wrongdoer must be voluntary to give rise to delictual liability. By raising the
defence of automatism, a defendant attempts to show that, according to the law, he
didn’t act. Defendant may argue that the conduct complained of doesn’t satisfy the
requirement of voluntariness. He relies on the defence of automatism - that he acted
mechanically.

Automatism doesn’t mean that there’s no voluntary act whatsoever by the defendant
which caused the damage, but only that the conduct in question wasn’t voluntary. Only
the voluntary act closest to the harmful consequence is of relevance, and it’s therefore
unnecessary to consider prior voluntary acts.

Defendant may argue that the conduct complained of doesn’t satisfy the requirement
of voluntariness. He relies on the defence of automatism - that he acted mechanically.
Conditions that may cause a person to act involuntarily as they render him incapable
of controlling his bodily movements: absolute compulsion), sleep, unconsciousness,
fainting fit, epileptic fit, serious intoxication, blackout, reflex movements, strong
emotional pressure, mental disease, hypnosis, and a heart attack.

According to Molefe v Mahaeng, the defendant does not bear the onus to prove that
he was in a state of so-called sane automatism.
The onus is on the plaintiff to prove that the defendant acted voluntarily.
.
In the Du Plessis case, he was charged with negligent driving as he had injured a
pedestrian. He experienced a blackout due to low-blood pressure. He was found not
guilty.

A person can’t rely on automatism if he/she negligently placed him in a mechanical


state ( If Lesego was on medication for her heart disease and did not take her
medictaion ). Thus, Lesego was probably negligent, or could even have had intention
in the form of dolus eventualis. Therefore, a reliance on automatism would fail in that
particular case.
In the given set of facts, we can conclude that Lesego did not act voluntarily when the
vehicle swerved out of control and crashed into the other car as she was unconscious .

QUESTION 2

Clearly differentiate between the wrongfulness criterion and the test for negligence.
First explain what the two respective criteria entail, and then discuss the differences
between them in detail.(15)

(a) The infringement of a subjective right • Has the subject-object relationship in fact
been disturbed? (factual infringement)
• Was the infringement in a legally reprehensible way? (norm-violation)

The general norm to determine whether a particular infringement is unlawful is the legal
convictions of the community or the boni mores. The boni mores test is an objective test
based on the criterion of reasonableness. The basic question is whether, according to
the legal convictions of the community and in the light of all the circumstances of the
case, the defendant infringed the interests of the plaintiff in a reasonable or
unreasonable manner.
The doctrines that are practical applications thereof are :

Fundamental premise of this doctrine is that the infringement of such a right is wrongful.
A dual investigation is necessary:

(a) The infringement of a subjective right

1. Has the subject-object relationship in fact been disturbed? (factual infringement)


2. Was the infringement in a legally reprehensible way? (norm-violation)

A subjective right is therefore infringed when the relationship between the holder of a
right and the object of the right has been infringed in a legally reprehensible manner.
(b) Wrongfulness as a breach of a legal duty

Sometimes the infringement of a subjective right is not appropriate to determine


wrongfulness, e.g. where no clearly defined right exists for instance in
misrepresentation cases, cases of omission or pure economic loss. In instances like
this, wrongfulness is determined by asking whether the defendant has a legal duty to
prevent loss. In general there is no duty to prevent loss. The question is whether
according to the boni mores or legal convictions of the community there was a legal
duty to act positively or to avoid pure economic loss.

The criterion for the test for negligence:

The reasonable person in the position of the defendant:

(a) would foresee the reasonable possibility of his conduct injuring another in his
person or property and causing him patrimonial loss; and
(b) would take reasonable steps to guard against such occurrence; and the defendant
failed to take such steps.

QUESTION 3

A bright 13-year-old boy, named Thabo, took an interest in the construction of


fireworks. He reached out to his father’s friend, who is a science Professor, to assist
him to build fireworks for Christmas. Thabo and the Professor built a firework rocket in
Thabo’s home backyard and launched it to test their creation. The rocket soared at
excessive speed to a height of 220 meters, erupted into flames and fell onto the
thatched roof of a lapa at a nearby tavern. The lapa burnt down. Can the standard
negligence test, as it is usually stated in South African case law, be applied to Thabo
and the Professor, or must the negligence test be adapted when applied to Thabo and
the Professor respectively? Discuss in detail, with reference to relevant case law.(25)

Prior to the case of Jones NO v Santam Bpk , the negligence of a child used to be
determined with reference to a reasonable child standard. In the Jones case, the
court held that the test for negligence remains objective, and the reasonable person
test must also be employed in the case of a child wrongdoer. The youthfulness of the
child wrongdoer is not specifically considered here. However, during the inquiry into
the accountability of the child, his or her youthfulness is taken into account.
The Jones case was criticised on 2 counts: firstly, many are of the opinion that a
reasonable adult standard for a child wrongdoer is unfair; secondly, the court put the
cart before the horse by testing for negligence first and, thereafter, for accountability.
In Roxa v Mtshayi, the court followed the correct order. In Weber v Santam, the
Jones case was confirmed in essence and the court said that if the principles were
applied with insight, the criticism would fall away. In Eskom Holdings Ltd v Hendricks
2005, the court reiterated that in each case it must be determined whether the child
has attained the emotional and intellectual maturity to appreciate the danger to be
avoided and to act accordingly. In respect of accountability, the Child Justice Act 75
of 2008 provides that a child of 9 or younger is irrebuttably presumed to be not
accountable, whereas a child over 9, but under 14, is rebuttably presumed to be not
accountable. Whether Thabo in our question would be held to have been negligent
would depend on all the circumstances of the case. If he was intelligent and mature
enough to be accountable, he was probably negligent, because, taken at face value,
his conduct deviated from that of the reasonable person in the circumstances.

Therefore in the given set of facts, the negligence test has to be adapted when
applied to Thabo and the Professor respectively.

QUESTION 4

4.1 Simultaneously, but independent of one another, Mxolisi and Vusi shoot at
Tau with heavy calibre pistols, both hitting Tau in the back of his head.

(a) Apply the theory of conditio sine qua non to determine whether both
Mxolisi and Vusi have caused Tau’s death. (4)

The conditio sine qua non is the test of factual causation. If we apply the test to the
facts, we must conclude that if Mxolisi and Vusu had not shot at Tau , he would not
have been hit on the back of his head and he should not have died. Therefore
a factual causal link is present between Mxolisisa md Vusi’s conduct and Tau’s
damage.
(b) Should the application of this test yield an unsatisfactory result, is
there a method by which it can be remedied? Discuss. (6)

Cumulative Causation:
In this case, the conditio sine qua non fails.
Mxolisi and Vusi acted both independently and without prior knowledge, shot Tau and
killed him. If we eliminate Mxolisi’s conduct, Tau would still be dead.
To answer this, medical evidence is needed to prove which bullet (or both) killed Tau.
If both bullets killed Tau simultaneously, Mxolisi and Vusi are both guilty of murder.

4.2 List five theories of legal causation. (5)

➢ Adequate causation,
➢ Direct consequences,
➢ foreseeability
➢ novus actus interveniens.
➢ Flexible approach

QUESTION 5

5 Concisely describe the ambit of the so-called “once and for all” rule and
briefly indicate what the practical implications of its application are?
(6)

The “once and for all” rule expresses that in all claims for compensation and satisfaction
arising out of a delict, the plaintiff must claim damages for all damage already sustained
and all future damages insofar as the claims are based on a single cause of action.

The rule has practical implications:


1. Prescription in regard to the claim for damages commences as soon as the cause of
action arises and the debt in respect of payment of damages is claimable = all the
elements of delict are present and the plaintiff is aware of or should reasonably be
aware of the identity of the debtor and the facts of the cause of action.
2. A plaintiff who has sued with or without success for part of his damage cannot
thereafter sue for another part, if both claims are based on a single cause of action.
5.1 Explain the concept of “mitigation of loss”. (2)

It is a principle of the law of delict that a plaintiff may not recover damages for a loss
which is the factual result of the defendant’s conduct but which could have been
prevented if the plaintiff had taken reasonable steps

QUESTION 6

Tom drove Connie’s sheep onto Jenni’s lucerne field, while mistakenly under the
impression that it was Connie’s lucerne field. The sheep consumed the lucerne. Briefly
discuss Jenni’s prospects of successfully instituting the actio de pastu against Connie.
(6

Damages are claimed from the owner of an animal, which caused loss, by eating
plants. Strict liability on the part of the owner.
Requirements:
1. The defendant must be the owner of the animal when the damage is caused
2. Animal must cause damage by eating plants
3. The animal must be of its own volition when causing the damage.

From the above set of facts, the 3 requirements has been met and therefore Jenni can
successfully institute the actio de pastu.

QUESTION 7

Define defamation and list the requirements that constitute defamation. (5)

Defamation is the intentional infringement of another person’s right to his good name.
Defamation is the wrongful, intentional publication or words or behavior concerning
another person, which has the effect of injuring his status, good name or reputation
Requirements
1. The defamation must amount to comment
2. The comment must be fair
3. The facts on which the comment is based must be true
4. These facts must be in public’s interest.
QUESTION 8

8.1 A character- and career-assassinating statement about Daphne was posted on


social media platforms, Twitter and Facebook, by Chuck. Daphne asked Chuck
to remove the post, but he refused to do so, even after Daphne’s attorneys had
sent him a letter instructing him to remove the post. According to Chuck, he
purely detailed the truth. Daphne maintains that the statement is defamatory and
makes her out to be a scammer, which she is not. It is also very harmful to her
family, especially her elderly mother who is a staunch Christian. What is
Daphne’s most immediate remedy? State the purpose of this remedy and list
the requirements that must be met for the court to grant this remedy.
(5)

The interdict
The interdict is directed at the prevention of a wrongful act (impending wrongful act or
to prevent the continuation of a wrongful act that has already commenced) and is
therefore the most suitable remedy. The interdict has a preventative function and as a
result, there is no need for the requirement of fault on the part of the wrongdoer to be
present. The requirements that need to be met for the granting of an interdict are: there
must be an act by the respondent, the act must be wrongful, and no other ordinary
remedy which would prevent the wrongful conduct must be available to the applicant.

8.2 Assume that Chuck persists in his refusal to remove the post. As a result of the
posts on Twitter and Facebook, Daphne’s reputation is damaged. The post
caused rumours to circulate and Daphne, being a well-known lawyer, lost many
of her clients and respect from her colleagues and community. In addition,
assume that Chuck’s post was unfounded. Which action(s) may Daphne
institute against Chuck?
(2)

In principle, Daphne must then institute two actions: the actio iniuriarum for satisfaction
(solatium) and the actio legis Aquiliae for patrimonial damages.

QUESTION 9

Write brief notes on the determination of the element of wrongfulness in respect of


the causing of emotional shock or psychological injury. (4

A psychological lesion is described as any recognizable harmful infringement of the


brain and nervous system of a person.
The existence of the lesion should be proved by supporting psychiatric evidence.
The injury can be sustained in various ways: nervous shock, fright or other mental
suffering.

Wrongfulness: the requirement of physical harm – infringement to the right to physical


integrity, which is wrongful – was rejected in Bester. According to Bester the brain and
nervous system are as much a part of the physical body as an arm and leg. As a result
a physical injury isn’t absolutely necessary to found liability.
PVL 3703 – LAW OF DELICT

MAY/JUNE 2022

EXAMINATION
QUESTION 1

John works in the control tower at the Take flight Airport. He suffers from a rare disease
that causes unexpected blackouts. However, he is on prescription medicine that
effectively eliminates the possibility of the blackouts. On one particular day, he does
not take his medicine. In the control tower, he has a blackout and is unable to ive the
necessary instructions to departing and arriving aircraft. A catastrophic airplane
accident takes place during his blackout because no instructions wereforthcoming. A
potential plaintiff approaches you for a legal opinion.

Write an opinion on the question whether John’s behaviour qualified as conduct for
the purpose of delictual liability. (10)
Conduct is defined as a voluntary human act or omission. “Voluntary” means that the
person must be able to control his muscular movements by means of his will. The act of
the wrongdoer must be voluntary to give rise to delictual liability. Conditions that may
cause a person to act involuntarily as they render him incapable of controlling his bodily
movements: absolute compulsion (vis absoluta), sleep, unconsciousness, fainting fit,
epileptic fit, serious intoxication, blackout, reflex movements, strong emotional pressure,
mental disease, hypnosis, and a heart attack.
However, John had been receiving medical treatment for a diagnosed illness but failed
to take his prescribed medication on that particular occasion. He therefore intentionally
placed himself in a mechanical state (actio libera in causa). Actio libera in causa:
Defence of automatism won’t succeed if defendant intentionally created the situation in
which he acts involuntarily in order to harm another. The defendant will be held liable for
his culpable conduct in creating the state of automatism which resulted in damage to the
plaintiff.
John was negligent regarding his automatic “conduct.” Where the reasonable man
would’ve foreseen the possibility of causing harm while in a state of automatism, e.g. in
Victor case, X was convicted of negligent driving despite causing the accident during an
epileptic fit, as he’d been suffering fits for 13 years and the reasonable man
would’ve foreseen the possibility of causing harm while in a state of automatism. In this
case, John knew he may have a blackout if his medication was taken.

Only the voluntary act closest to the harmful consequence is of relevance, and it’s
therefore unnecessary to consider prior voluntary acts.

John placed himself in a mechanical state by not taking his medication Thus, John was
probably negligent, or could even have had intention in the form of dolus eventualis.

QUESTION 2

At the Shop-Till-You-Drop Shopping Mall, hand sanitizer dispensers have been


installed at all the pedestrian entrances. Notices have been placed near the
dispensers, in which customers are urged to sanitize their hands at the dispensers
before entering the mall. At Entrance B, the dispenser has been malfunctioning for half
a week. It spills sanitising liquid onto the floor, in such large quantities that the liquid
does not evaporate immediately but forms a small puddle around the base of the
dispenser. Because the lighting at Entrance B is rather on the dim side, the puddle is
not readily visible unless one specifically looks out for it. The management of the mall
is aware of this situation but does not do anything about it. Ms C is an elderly lady
whose eyesight is not very good, and she is not particularly steady on her legs, but
she loves shopping. She arrives at entrance B, reads the notice, and proceeds to
sanitize her hands. This requires her to put her one foot on a pedal and push down on
it to activate the dispenser. As she attempts to do this, she slips in the puddle and falls.
She sustains serious injuries and is hospitalised. After a long and costly bout in
hospital, she is discharged. Ms C wishes to institute an action for damages against the
management of the Shop-Till-You-Drop Shopping Mall and approaches you for legal
advice.

Write an opinion, properly substantiated with reference to case law, only onthe
wrongfulness of the conduct of the management of the Shop-Till-You-Drop Shopping
Mall. (30)

This question deals with the wrongfulness of an omission. An omission is wrongful if


the defendant is under a legal duty to act positively to prevent the harm suffered by the
plaintiff. The basic question to determine whether an omission is wrongful is whether a
legal duty to act was present and was breached. As a general rule, a person does not
act wrongfully for the purposes of the law of delict if he omits to prevent harm to
another person. For liability to follow an act, prejudice must be caused in a wrongful
and unreasonable manner. Without wrongfulness the defendant cannot be held liable.
The legal convictions of the community (boni mores test) are used as the basic test for
wrongfulness. The general norm to see if an infringement of interests is unlawful is the
legal convictions of the community. Factors which may serve as indications that a legal
duty rested on the defendant include:
➢ prior conduct (omissio per commissionem)
➢ control of a dangerous object
➢ rules of law
➢ a special relationship between the parties
➢ particular office
➢ contractual undertaking for the safety of a third party
➢ and creating of an impression that the interests of a third person will be protected.
We will look at PRIOR CONDUCT more closely.
A person acts prima facie wrongfully when he creates a new source of danger and then
fails to eliminate that danger, with the result that harm is caused to another person. Prior
conduct is not a prerequisite for the existence of a legal duty, however at one stage this
was the only category where liability was imposed on a failure to act.
The Municipality cases are closely linked to this scenario.

In Halliwell v Johannesburg Municipality; the municipality laid cobblestones in


Johannesburg’s city centre but failed to maintain those stones. When Mr Halliwell
crossed the cobblestone path with his horse carriage on a Christmas morning, his
horse stumbled on the poorly maintained road and Mr Halliwell was hurt. The court
held that the municipality was liable for the failure to maintain the cobblestone road
which led to Mr Halliwell’s injuries. This was explicitly done on the basis of the prior-
conduct rule. This rule was also employed in the case of Silva’s Fishing Corporation v
Maweza, where the owner of a boat sent fishermen out to sea (the prior positive
conduct) but when the boat stopped working, the owner did nothing to save the
fishermen (the subsequent omission). The boat owner was held liable for his omission.
In Regal v African Superslate, the prominence of the prior-conduct rule as the only
ground on which liability for an omission could be established ended. In the judgment it
was noted that the prior-conduct rule was not the only basis on which an omission
would be actionable. For the first time, the Appellate Division noted that the prior-
conduct rule and the rule related to the control of dangerous property are both grounds
on which the wrongfulness of an omission can be established.
In Regal it was showed that the conduct element could be satisfied by simply showing
the factual existence of an act or an omission. The wrongfulness of the omission was
then established by means of the prior-conduct rule or the rule related to the control of
dangerous property. The principle from Regal was later also applied in the case of
Minister of Forestry v Quathlamba; where a fire started on one property, the owner of
that property failed to control or contain the fire, and the fire spread to a neighbouring
property. This re-inforced the Regal decision that prior conduct is not the only criterion
for establishing a legal duty.
An extension of the rules related to the wrongfulness of omissions featured in the case
of Minister van Polisie v Ewels. In that case police officers passively stood by and
watched as a fellow police officer brutally assaulted a civilian. The court held that those
two rules as shown in prior cases are just two of many factors to consider in determining
the wrongfulness of omissions. In Ewels, considerations that weighed in favour of a
finding of wrongfulness of the omission of the passive police officers included the legal
duty placed on the police to protect citizens from harm, the special relationship between
police officers and the public, and the fact that the passive police officers were in a
position to exercise authority or control over their assaultive colleague.
In Carmichele v Minister of Safety and Security (Centre for Applied Legal Studies
Intervening) 2001, the CC made it clear that the boni mores must now be informed by
the values underpinning the Bill of Rights in the Constitution.
APPLICATION TO THE FACTS
Looking at the above principles and the the given facts, we can conclude that the
omission of the Centre Management of the shopping mall was wrong. Management at
the Shopping Centre had a legal duty in terms of the legal convictions of the community
to prevent this accident from happening.

Question 3
Mike hires a mountain bike from the Steadywheels Mountain Bike Park and goes for
a ride on one of the mountain bikes trails in the Park. Even though Mike’s safety helmet
is in the boot of his car, he neglects to put on his helmet before he goes for the ride.
Along a steep part of the trail, the front wheel of the rental bike suddenly comes off.
Mike falls and sustains serious injuries. It transpires that David, an employee of
Steadywheels, had not fastened the front wheel of the bike properly when he had
serviced it. In addition, it transpires that Mike’s injuries would have been substantially
less serious if he had worn his helmet. Mike wants to institute a delictual claim against
Steadywheels.

Write a properly substantiated opinion on the feasibility of such a claim: (a) You may
accept that Steadywheels will argue that there was no fault on its part and will not
contest the other elements of delictual liability. (b) In addition, you should also consider
whether any defence is available to Steadywheels and what the effect of such a
defence, if available, would be. (c) Furthermore, you should give an opinion on the fact
that Mike’s claim will be instituted against Steadywheels, rather than David. (30)
From the given facts, we can conclude that the defendant has been negligent, but the
plaintiff appears to have been negligent too. Thus we must consider whether
contributory negligence was present.
Contributory negligence is negligence on the part of the plaintiff, and it is a defence that
the defendant can raise. The Apportionment of Damages Act 34 of 1956 is applicable.
This Act provides that a contributorily negligent plaintiff’s damages be apportioned. The
court will determine the degree of deviation from the reasonable person standard shown
by the conduct of both the defendant and the plaintiff, express the deviation as
percentages, and use these percentages as a basis for the apportionment.

According to the Smit 1962 and Nomeka 1976 cases, the percentages of negligence
attributed to the defendant and plaintiff respectively will always add up to 100%.
According to Jones NO v Santam Bpk 1965, both percentages must be assessed
independently, which could mean that, for example, a defendant may be 80% negligent
while the plaintiff is 30% negligent. According to Neethling and Potgieter, the approach
in Jones is to be preferred, but the 2 approaches can be reconciled. According to King v
Pearl Insurance Co Ltd 1970, a defence of contributory negligence could not succeed
where the plaintiff had omitted to wear a crash-helmet while driving a scooter but had
not been negligent in respect of causing the accident.

However, in Bowkers Park Komga Cooperative Ltd v SAR and H 1980, the court held
that contributory negligence didn’t refer to negligence in respect of the damage-causing
event, such as a motorcar accident, but to negligence in respect of the damage itself,
and this was confirmed by the AD in Union National South British Insurance Co Ltd v
Vitoria 1982 and General Accident Versekeringsmaatskappy SA Bpk v Uijs 1993.
Therefore, failure to wear a safety helmet would constitute contributory negligence if it
contributed to the plaintiff’s damage.
QUESTION 4

Tom inadvertently knocks over a ladder on which William is standing while painting a
wall. William falls and breaks a leg. In the hospital, the leg is set in plaster, William is
issued with crutches, and is discharged. At home, William slips with the crutches and
falls again, breaking an arm.

Discuss only the following: (a) whether there is a factual causal link between Tom’s
conduct and William’s broken arm; and (b) whether there is a legal causal link between
Tom’s conduct and William’s broken arm. (15)

Does a factual causal link exist between Tom’s conduct and Williams broken
leg?

The generally accepted test for factual causation is the conditio sine qua non test, or ‘‘but
for test’’. This entails mentally eliminating, or thinking away, the conduct. If the damage
then also disappears, a factual causal link is present between the conduct and the
damage. This test is subject to much criticism. Among others, it is said to be based on
circular logic and is, at best, a way to express the existence of a causal nexus that has
been determined in another way. Neethling and Potgieter argue that evidence and
human experience are sufficient to determine whether one fact flowed from another fact,
and that a so-called test of factual causation is superfluous. However, the courts
consistently state that the conditio sine qua non is the test of factual causation. If we
apply the test to the facts, we must conclude that if Tom had knocked over the ladder
that William was standing on, he would not have broken his leg, and therefore a factual
causal link is present between Tom’s conduct and William’s damage.

Does a legal causal link exist between Tom’s conduct and William’s broken leg?
The test for legal causation is the so-called flexible approach, as formulated in S v
Mokgethi 1990 and International Shipping Co (Pty) Ltd v Bentley 1990.
According to the court, the main question in respect of legal causation is whether there
is a close enough relationship between the wrongdoer’s conduct and its consequence
for such consequence to be imputed to the wrongdoer in view of policy considerations
based on reasonableness, fairness and justice. Several other legal causation theories
exist, such as adequate causation, direct consequences, foreseeability and novus actus
interveniens.
None of these criteria is suitable to be applied to all situations. They may, however, be
used as subsidiary aids when employing the flexible approach. In the Mokgethi case, the
court held that the shot was not a legal cause of the death. If these principles are applied
to the facts in the question, the conclusion is probably that William’s broken leg was too
remote and should not be imputed to the wrongdoer. It could also be argued that a so-
called novus actus interveniens, that is, a new intervening act, was constituted by
Williams second fall, and this strengthens the conclusion that there is no legal causal link
between Tom’s conduct and William’s broken leg

QUESTION 5

Jenna asks Connie to look after her dog while she is away on vacation. Connie takes
the dog for a walk in the park. She orders the dog, which is well-trained, to bite Ben,
against whom she bears a grudge. The dog immediately attacks Ben. Ben hits the dog
over the head with a baseball bat, killing the dog. On returning from her vacation,
Jenna institutes a claim for damages against Ben. Ben does some internet research,
but he is not sure whether to rely on private defence or necessity.

Which one of these two grounds of justification would you recommend to Ben, and
why? (5)

Ground of Justification - Necessity.

A state of necessity exists when defendant is placed in such a position by superior force
(vis maior) that he is able to protect his legally recognised interests (or those of
someone else) only by reasonably violating the interests of an innocent person. Thus,
Ben will be successful with necessity as a ground of justification as he had no other
option but ot protect himself from the attack

QUESTION 6
Jane’s dog bit Carol, inflicting serious injuries. Jane was not negligent in any way. Name
the action with which Carol can claim damages from Jane and list the requirements to
succeed with this action. (5)

Action - The actio de pauperie

The following requirements must be met:


1. The defendant must be the owner of the animal when the damage is inflicted.
2. The animal must be a domestic animal
3. The animal must act contrary to its own nature when inflicting damage – as a rule an
animal doesn’t act contrary to its own nature if it’s reacting to external stimuli –defences:
vis major, culpable conduct on the part of the prejudiced person and provocation.
4. The prejudiced person or his property must be lawfully present at the location when
the damage is inflicted.

QUESTION 7

Name 3 grounds of justification that are applicable in defamation law. (3)

• Privilege
• Truth and public interest
• Fair comment

QUESTION 8

Defamation is a specific form of iniuria. Name 2 other forms of iniuria. (2)

• Adultery
• Invasion of Privacy
May/June 2020

EXAM

PVL3703

Law of Delict

Questions 1 to 7 must be answered with reference to this set of facts:

The Cape Town Metro has neglected to keep a popular seaside promenade in a good
condition. Due to weathering, the surface of the promenade has become uneven. During
a pre-dawn run, Thabo trips and falls when running over crumbling paving on the
promenade. He breaks his right arm and his face hits a bench. He is treated in hospital
and his arm heals eventually, but his face remains permanently disfigured. He does not
belong to a medical aid scheme and is therefore personally liable for the hospital costs.
He is also unable to work for some time and suffers a temporary loss of income.

It is advisable to read all seven questions before answering any one of them.

1. If Thabo wishes to institute a delictual claim against the Metro, he may rely on the
following delictual remedy or remedies:
1. The actio legis Aquiliae and the actio iniuriarum.
2. The actio legis Aquiliae and the action for pain and suffering.
3. The actio legis Aquiliae only.
4. The actio de effusis vel deiectis only.
(2)

2. If Thabo institutes a delictual claim against the Metro, the question whether the
Metro’s conduct was wrongful will, according to the traditional approach to
wrongfulness, be answered with reference to the following:
1. Infringement of a subjective rialght.
2. Breach of a legal duty.
3. The reasonableness of holding the Metro liable.
4. The reasonable person test.
(2)
3. If Thabo institutes a delictual claim against the Metro, the question whether the
Metro’s conduct was wrongful will, according to the new approach to wrongfulness
that has often been followed since the judgment in Telematrix (Pty) Ltd t/a Matrix
Vehicle Tracking v Advertising Standards Authority SA 2006 1 SA 461(SA), be
answered with reference to the following:
1. Infringement of a subjective right.
2. Breach of a legal duty.
3. The reasonableness of holding the Metro liable.
4. The reasonable person test.
(2)

4. If Thabo institutes a delictual claim against the Metro, the question whether there was
fault on the Metro’s part, will be answered with reference to the following:
1. The ability to differentiate between right and wrong.
2. Breach of a legal duty.
3. Directing the will combined with consciousness of wrongfulness.
4. Foreseeability and preventability of harm.
(2)

5. Which one of the following has an important influence on the time at which Thabo
must claim for the different kinds of harm suffered by him? Select the most
appropriate one.
1. The “once and for all” rule.
2. The concrete approach to damage.
3. Mitigation of damage.
4. The sum-formula approach.
(2)

6. Assume that Thabo succeeds with a delictual claim against the Metro after he has
been discharged from hospital. The quantum of the claim for the hospital costs must
preferably be determined in accordance with:
1. The “once and for all” rule.
2. The concrete approach to damage.
3. Mitigation of damage.
4. The sum-formula approach.
(2)
7. If Thabo succeeds with a delictual claim against the Metro, after he has been
discharged from hospital but several months before he is able to work again, the
quantum of the claim for loss of income must preferably be determined in accordance
with:
1. The “once and for all” rule.
2. The concrete approach to damage.
3. Mitigation of damage.
4. The sum-formula approach.
(2)

Questions 8 to 15 must be answered with reference to this set of facts:

Charles works as a technician at Fani’s Funfair. One of his tasks is to inspect the
rollercoaster at the funfair for mechanical integrity and safety. However, as a result of
his lack of care and attention while conducting the inspection, Charles inadvertently
overlooks some signs that the rollercoaster is no longer in sound condition. Ria goes for
a ride on the rollercoaster. Neither on the ticket that Ria has purchased, nor on any
notice board on the premises is there any notice that Fani’s Funfair does not accept
liability for injury to its customers. The operator at the rollercoaster requests the persons
boarding the rollercoaster to fasten their safety harnesses securely. However, Ria
decides not to fasten her safety harness, because she wants to take good selfie pictures
during the ride. All the other persons on the rollercoaster fasten their safety harnesses.
During the ride, but before it can reach a high speed, the rollercoaster breaks down and
comes to an abrupt standstill. Ria’s head hits the front of the cart in which she is sitting.
Her nose is broken, and she sustains a concussion. All the other persons on the
rollercoaster are unharmed. She is hospitalised for treatment. After two days, Ria has
healed sufficiently to be discharged. However, Mandy, an inexperienced nurse, forgets
to close a window in Ria’s ward and Ria is exposed to an icy wind overnight. In the
morning, Ria is diagnosed with severe pneumonia and she must stay in hospital for
longer. Ria wishes to recover her damage by instituting a delictual claim.

It is advisable to read all 8 questions before answering any one of them.


8. The question whether Charles’ conduct was wrongful, will be answered with reference
to the following:
1. The boni mores.
2. The reasonable person test.
3. The conditio sine qua non test.
4. The doctrine of sudden emergency.
(2)

9. The question whether Charles’ conduct was negligent, will be answered by asking:
1. Whether Charles directed his will at the harm and was conscious of the
wrongfulness thereof.
2. Whether Charles would have foreseen and prevented the harm.
3. Whether the reasonable person in Charles’ position would have foreseen
and prevented the harm.
4. Whether the doctrine of sudden emergency is applicable.
(2)

10. Whether there was a factual causal link between Charles’ conduct and Ria’s harm,
will be determined with reference to:
1. The but for-test.
2. Adequate causation.
3. Direct consequences.
4. The flexible approach.
(2)

11. Choose the most correct and most complete alternative: There was a factual causal
link between Charles’ conduct and:
1. Ria’s broken nose and concussion.
2. Ria’s broken nose and concussion and the hospital costs incurred in respect of
treating the broken nose and concussion.
3. Ria’s broken nose and concussion and pneumonia and all the hospital costs.
4. Ria’s pneumonia and the hospital costs incurred in respect of treating the
pneumonia.
(2)
12. Whether there was a legal causal link between Charles’ conduct and Ria’s harm,
will be determined with reference to:
1. The but for-test.
2. Adequate causation.
3. Direct consequences.
4. The flexible approach.
(2)

13. Choose the most correct and most complete alternative: There was a legal causal
link between Charles’ conduct and:
1. Ria’s broken nose and concussion.
2. Ria’s broken nose, concussion and the hospital costs incurred in respect
of treating the broken nose and concussion.
3. Ria’s broken nose and concussion and pneumonia and all the hospital costs.
4. Ria’s pneumonia and the hospital costs incurred in respect of treating the
pneumonia.
(2)

14. Which one of the following defences may be available to the defendant(s)?
1. Pactum de non petendo.
2. Contributory negligence.
3. Doctrine of sudden emergency.
4. Official command.
(2)

15. What will the effect of a successful reliance of the available defence in question 14
be?
1. An apportionment of damages will take place.
2. Ria’s claim will fail completely.
3. Ria would have forfeited the right to institute a delictual claim.
4. The defendant(s) will be permitted to pay damages in instalments.
(2)
16. Thumi is an epilepsy sufferer. He neglects to take his prescription medicine and
decides to drive to the local mall to do some shopping. On his way, he suffers an
epileptic seizure. He loses control of his car and smashes into a corner shop, causing
considerable damage to the building and merchandise. The shop owner institutes a
delictual action against Thumi. Which one of the following statements is correct?

1. Thumi can escape liability by relying on automatism.


2. Thumi cannot rely on automatism because the defence actio libera in causa
applies.
3. Thumi cannot rely on automatism because he negligently placed himself in
a situation where his movements were mechanical.
4. Thumi can escape liability by proving that his bodily movements were not voluntary
at the time when the damage was caused.
(2)

17. Gezani incites his aggressive, pedigreed rottweiler dog to attack Derick. Derick
shoots and kills the dog. If Gezani institutes a delictual action against Derick for the
loss of his dog, Derick may rely on the following defence:

1. Necessity.
2. Private defence.
3. Provocation.
4. None of the above.
(2)

18. Linda asks Jane to look after Jock, her friendly and well-trained Staffordshire bull
terrier while Linda is away on vacation. The next day Jane takes Jock for a walk in a
park. Thomas approaches Jane, draws a knife and orders her to hand over her
handbag and phone. Jane orders Jock to attack Thomas. Jock responds
immediately, but Thomas kills Jock with his knife. Jane slips away, alerts a policeman
on patrol, and Thomas is apprehended. When Linda returns from vacation, she
institutes a delictual action against Thomas for the loss of her dog. Thomas may rely
on the following defence:

1. Necessity.
2. Private defence.
3. Provocation.
4. None of the above.
(2)
19. Tebogo asks Mary to look after Uli, her pedigreed and well-trained German
shepherd dog while Tebogo is away on vacation. The next day Mary takes Uli for a
walk in a park. Mary sees William, her ex-boyfriend, in the distance. Mary has a
grudge against William and orders Uli to attack him. William shoots and kills Uli.
Tebogo institutes a delictual action against William for the loss of her dog. William
may rely on the following defence:

1. Necessity.
2. Private defence.
3. Provocation.
4. None of the above.
(2)

20. Grace likes to shoot with a bow and arrow in her spare time. She asks Peter to
balance an apple on his head so that she can use it as a target. He agrees. The first
arrow shot by Grace misses the apple but penetrates Peter’s arm, necessitating
medical treatment. Peter institutes a delictual claim against Grace. Grace may rely
on the following defence:

1. Privilege.
2. Official capacity.
3. Consent.
4. None of the above.
(2)

21. X is suspected of infidelity by his wife. She hires a private detective, Y, to spy on X.
Y follows X in his car to a lonely spot in the veld. X has a woman with him in his car.
When Y approaches X’s car, X starts to drive off. Y vaults onto the bonnet in order
to obscure X’s view and so to make him stop. X, however, accelerates and begins
to swerve from side to side, clearly with the object of dislodging Y who is clinging on
for dear life. Six kilometres further, X succeeds in dislodging Y. Y suffers injuriesand
institutes a delictual claim against X.

According to Neethling and Potgieter, the theoretically correct position is:

1. X can successfully raise contributory negligence as a defence.


2. X can successfully raise contributory intention as a defence.
3. X can successfully raise a pactum de non petendo to ward off Y’s claim.
4. X can successfully raise tacit consent to the risk of injury as ground of justification.
(2)
22. Which of the following principles relates to the maxim that you must “take your victim
as you find him”?

1. The “sum formula” approach


2. The “mitigation” of loss principle.
3. The “talem qualem” rule.
4. The “once and for all” rule.
(2)

23. Luke and his brother Paul bear a grudge against Patrick and agree to teach him a
lesson he will not forget. While Patrick walks around in a shopping mall, Luke and
Paul damage Patrick’s motorbike by hitting it with hammers. If Patrick successfully
sues Luke and Paul in delict, they will incur:

1. Vicarious liability.
2. Joint and several liability.
3. Liability based on contributory intent.
4. Strict liability.
(2)

24. Rhulani’s goat eats and tramples Owen’s maize crops. Rhulani forgot to close the
gate between his and Owen’s land. Which remedy or remedies may be available to
Owen?

1. The actio de pauperie and actio legis Aquiliae.


2.The actio de pauperie only.
3. The actio de pastu only.
4. The actio de pastu and actio legis Aquiliae.
(2)

25. Which one of the following is incorrect?

1. An animal cannot act for purposes of the law of delict.


2. A public school can act for the purpose of the law of delict.
3. Behaviour must be willed to qualify as a voluntary act.
4. Conduct may consist of a commission or an omission.
(2)
26. While watching a soccer match, Tom, a staunch supporter of Orlando Pirates, tells
Ace, a staunch supporter of Chiefs, that he (Ace) is a wimp for supporting such a
useless team. Tom continues to insult Ace. After enduring Tom’s insults for about 30
minutes, Ace in a blind range, hits Tom in the face, breaking Tom’s nose. Tom
institutes a delictual action against Ace. Ace may rely on:
1. Private defence.
2. Necessity.
3. Provocation.
4. None of the above.
(2)

27. If a court must determine whether a brain surgeon was negligent while performing
brain surgery on a patient, the court will inquire whether he met the standard of care
of:
1. The reasonable person.
2. The reasonable doctor.
3. The reasonable brain surgeon.
4. The most well-known brain surgeon practicing in South Africa.
(2)

28. If a court must determine whether a brain surgeon was negligent while driving his
car, the court will inquire whether he met the standard of care of:
1. The reasonable person.
2. The reasonable doctor.
3. The reasonable brain surgeon.
4. The most well-known brain surgeon practicing in South Africa.
(2)
29. Matthew is a rep for a pharmaceutical company, and he uses his car on a daily basis
to earn an income. He takes his car to Mike’s Mechanics for a service. A fire breaks
out at the premises and destroys Matthew’s car. The fire resulted from negligent
conduct on Mike’s part. Matthew institutes a delictual claim against Mike’s
Mechanics. In addition to the value of his car, he also wishes to claim an amount
that he has paid to Richard’s Rentals for renting a car to enable him to continue
doing his work and thus earning an income while waiting for the outcome of the
litigation. In respect of this second amount:

1. Matthew will not succeed, because the rental of the car is regarded as res inter
alios acta.
2. Matthew will not succeed, because Mike’s Mechanics cannot be made a party to
the contractual relations between Matthew and Richard’s Rentals.
3. Matthew will succeed, because a legal causal link exists between the
conduct of Mike’s Mechanics and the amount that Matthew had to pay to
Richard’s Rentals.
4. Matthew will succeed, because of the mitigation of loss rule.
(2)

30. According to case law, if a motor vehicle is involved in an accident, the failure of a
passenger in that motor vehicle to fasten their seat belt:

1. Will always constitute contributory negligence for the purpose of an apportionment


of damages.
2. Can constitute contributory negligence for the purpose of an
apportionment of damages only if such failure contributed to the
passenger’s injuries.
3. Can constitute contributory negligence for the purpose of an apportionment of
damages only if such failure contributed to the accident.
4. Can never constitute contributory negligence for the purpose of an apportionment
of damages.
(2)

31. According to case law, the negligence of a child wrongdoer in delict cases is
determined:
1. According to a reasonable person standard, but the youthfulness of the
child is considered when determining accountability.
2. According to a reasonable child standard.
3. According to the standard of a reasonable child of the same age.
4. According to the boni mores standard.
(2)
32. Which statement is incorrect?
1. A legal causal link between an act and a consequence is determined by
mentally eliminating the act and inquiring whether the consequence will then
also disappear.
2. A legal causal link exists if there is a sufficiently close link between an act and a
consequence that the consequence may be imputed to the wrongdoer.
3. A legal causal link is determined with reference to policy considerations of
reasonableness, fairness and justice.
4. When determining legal causation, standards such as adequate causation and
direct consequences may be used as subsidiary tests.
(2)

33. According to the doctrine of sudden emergency:


1. When a court determines the negligence of the conduct of an alleged wrongdoer
who found himself in a situation of sudden emergency, it is permissible to deviate
slightly from the reasonable person standard.
2. The notional reasonable person can make an error of judgment in a
situation of sudden emergency.
3. The wrongfulness of the conduct of an alleged wrongdoer who had to act in a
situation of sudden emergency will be judged more leniently by a court of law.
4. The state president may provide for the adoption of regulations that absolve
residents from delictual liability for conduct that takes place in a situation of sudden
emergency.
(2)

34. According to Greater Johannesburg Transitional Metropolitan Council v ABSA Bank


Ltd t/a Volkskas Bank 1997 2 SA 691 (W), contributory intention:

1. Is not recognised as a defence in our law.


2. Can give rise to a 50% reduction of damages if the defendant acted negligently.
3. Can give rise to a 50% reduction of damages if the defendant acted intentionally.
4. Is a complete defence if the defendant acted intentionally.
(2)

35. “Contributory fault” refers to the fault of:

1. The defendant.
2. The plaintiff.
3. Both the defendant and the plaintiff.
4. None of the above.
(2)
36. Which is the odd one out?

1. Privilege.
2. Truth and public interest.
3. Fair comment.
4. Jest.
(2)

37. Since National Media Ltd v Bogoshi 1998 4 SA 1196 (SCA), liability of the media for
defamation:
1. Is strict liability.
2. Requires animus iniuriandi in the form of intention on the part of the media
defendant.
3. Requires at least fault in the form of negligence on the part of the media defendant.
4. Is based on a rebuttable presumption of intention on the part of the media
defendant.
(2)

38. Which of the following is not a requirement for the vicarious liability of an employer?
1. An employer-employee relationship.
2. The employee must have committed a delict.
3. The employee must have acted in the scope of his employment.
4. The employer must have been negligent in his supervision over the employee.
(2)

39. Which is the odd one out?

1. Liability for pure economic loss.


2. The actio de pauperie.
3. The actio de pastu.
4. Vicarious liability.
(2)

40. Which one of the following cases hampered the development of the actio legis
Aquiliae to its logical conclusion?

1. Union Government v Ocean Accident and Guarantee Corporation Ltd 1956


1 SA 577 (A).
2. Carmichele v Minister of Safety and Security (Centre for Applied Legal Studies
Intervening) 2001 4 SA 938 (CC).
3. Cape Town Municipality v Bakkerud 2000 3 SA 1049 (SCA).
4. Crown Chickens (Pty) Ltd t/a Rocklands Poultry v Rieck 2007 2 SA 118 (SCA).
(2)
41. In which one of the following cases did the Court state that it was uncertain whether
necessity excludes wrongfulness or negligence?

1. Union Government v Ocean Accident and Guarantee Corporation Ltd 1956 1 SA


577 (A).
2. Carmichele v Minister of Safety and Security (Centre for Applied Legal Studies
Intervening) 2001 4 SA 938 (CC).
3. Cape Town Municipality v Bakkerud 2000 3 SA 1049 (SCA).
4. Crown Chickens (Pty) Ltd t/a Rocklands Poultry v Rieck 2007 2 SA 118 (SCA).
(2)

42. Which one of the following cases dealt extensively with the importance of
Constitutional values for delictual liability?

1. Union Government v Ocean Accident and Guarantee Corporation Ltd 1956 1 SA


577 (A).
2. Carmichele v Minister of Safety and Security (Centre for Applied Legal
Studies Intervening) 2001 4 SA 938 (CC).
3. Cape Town Municipality v Bakkerud 2000 3 SA 1049 (SCA).
4. Crown Chickens (Pty) Ltd t/a Rocklands Poultry v Rieck 2007 2 SA 118 (SCA).
(2)

43. Which one of the following cases is most frequently cited for its clear formulation of
the test for negligence?

1. S v Goliath 1972 3 SA 1 (A).


2. Van Eeden v Minister of Safety and Security (Women’s Legal Centre Trust, as
amicus curiae) 2003 1 SA 389 (SCA).
3. First National Bank of South Africa Ltd v Duvenhage 2006 5 SA 319 (SCA).
4. Kruger v Coetzee 1966 2 SA 428 (A).
(2)

44. Which one of the following cases dealt explicitly with the sequence in which the
elements of delictual liability should be considered?

1. S v Goliath 1972 3 SA 1 (A).


2. Van Eeden v Minister of Safety and Security (Women’s Legal Centre Trust, as
amicus curiae) 2003 1 SA 389 (SCA).
3. First National Bank of South Africa Ltd v Duvenhage 2006 5 SA 319 (SCA).
4. Kruger v Coetzee 1966 2 SA 428 (A).
(2)
45. Which one of the following cases contains a clear exposition of the role of a legal
duty in delictual liability?

1. S v Goliath 1972 3 SA 1 (A).


2. Van Eeden v Minister of Safety and Security (Women’s Legal Centre Trust,
as amicus curiae) 2003 1 SA 389 (SCA).
3. First National Bank of South Africa Ltd v Duvenhage 2006 5 SA 319 (SCA).
4. Kruger v Coetzee 1966 2 SA 428 (A).
(2)

46. Which one of the following cases was a trend-setting judgment on necessity,
although aspects of the judgment may need to be revisited in the current
constitutional dispensation?

1. S v Goliath 1972 3 SA 1 (A).


2. Van Eeden v Minister of Safety and Security (Women’s Legal Centre Trust, as
amicus curiae) 2003 1 SA 389 (SCA).
3. First National Bank of South Africa Ltd v Duvenhage 2006 5 SA 319 (SCA).
4. Kruger v Coetzee 1966 2 SA 428 (A).
(2)

47. Which one of the following cases brought clarity in respect of the criterion to be used
when determining whether legal causation is present?

1. Jones NO v Santam Bpk 1965 2 SA 542 (A).


2. Roux v Hattingh 2012 6 SA 428 (SCA).
3. Lee v Minister of Correctional Services 2013 2 SA 144 (CC).
4. S v Mokgethi 1990 1 SA 32 (A).
(2)

48. Which one of the following cases is of great importance in respect of the negligence
of children?

1. Jones NO v Santam Bpk 1965 2 SA 542 (A).


2. Roux v Hattingh 2012 6 SA 428 (SCA).
3. Lee v Minister of Correctional Services 2013 2 SA 144 (CC).
4. S v Mokgethi 1990 1 SA 32 (A).
(2)
49. Which one of the following cases dealt with consent in the context of sports injuries?

1. Jones NO v Santam Bpk 1965 2 SA 542 (A).


2. Roux v Hattingh 2012 6 SA 428 (SCA).
3. Lee v Minister of Correctional Services 2013 2 SA 144 (CC).
4. S v Mokgethi 1990 1 SA 32 (A).
(2)

50. Which one of the following cases is of great importance in respect of the flexible
application of the conditio sine qua non test?

1. Jones NO v Santam Bpk 1965 2 SA 542 (A).


2. Roux v Hattingh 2012 6 SA 428 (SCA).
3. Lee v Minister of Correctional Services 2013 2 SA 144 (CC).
4. S v Mokgethi 1990 1 SA 32 (A).
(2)

[100]
Oct/Nov 2019
1 Which one of the following remedies is instituted for personality infringements?

1. The actio legis Aquiliae


2. The actio iniuriarum
3. The action for pain and suffering
4. The interdict

2 Which of the following statements is incorrect with regard to the defence of automatism?

1. A reflex movement may cause a person to behave involuntarily and he/she may rely on
automatism.
2. According to Van der Merwe and Olivier, automatism does not exclude conduct in delict.
3. A person may not rely on automatism if he/she intentionally created the situation in which
he/she behaves involuntarily.
4. The person relying on sane automatism in a delictual case, bears the onus of provingit.

3 Mpho plants a bomb in a busy restaurant. Before the bomb explodes and causes damage, it is
discovered and rendered harmless. Which of the following statements is correct with regard to
Mpho’s act?

1. Mpho’s act is delictually wrongful


2. Mpho’s act is not delictually wrongful
3. Even though Mpho’s act is delictually wrongful, no delict was committed
4. Mpho had the intention to cause harm, therefore he committed a delict

4 In which one of the following judgments did the court accept that wrongfulness consists in the
infringement of a right, breach of a legal duty, and the reasonableness of holding a defendant liable?

1. Minister van Polisie v Ewels 1975 3 SA 590 (A)


2. Telematrix (Pty) Ltd t/a Marix Vehicle Tracking v Advertising Standards Authority SA 2006 1 SA
461 (SCA)
3. Loureiro v Imvula Quality Protection (Pty) Ltd 2014 (3) SA 394 (CC)
4. Le Roux v Dey 2011 (3) SA 274 (CC)

5 In which of the following circumstances may X rely on “private defence”?

1. X, a prisoner whose prison term has expired, is not released. In order to escape prison, X
assaults the guard who refuses to release him.
2. The owner of a farm Z, while placing his hand on his rifle, tells X that he is trespassing on
his land and that if he does not leave within the next hour he will forcefully remove him. X
immediately reacts by striking Z with a knife.
3. X is attacked and bitten while playing with the neighbour’s dog. X reacts by striking the dog
with a stick, injuring it.
4. A policeman with a valid warrant of arrest for X, arrests X. X resists the arrest and strikes the
policeman, injuring him.
6 Which of the following statements is correct with regard to the requirements of consent?

1. There must be an agreement between the person consenting (the injured person) and the
actor
2. The person giving the consent must have full legal capacity to act
3. Mere knowledge of the risk of harm is sufficient for the requirement of “appreciation” of the
risk of harm
4. The person consenting must be able to express his will

7 Humbulani and Patricia are neighbours. Humbulani often has quarrels with Patricia and does not like
her. Humbulani builds a large shed for his chickens on his property in order to spoil Patricia’s beautiful
view. It appears that Humbulani did indeed need a shed, but that he could easily have built it
elsewhere on his property. Which of the following statements is correct with regard to Humbulani’s
act?

1. Humbulani’s improper motive renders his act wrongful


2. Humbulani furthered a reasonable interest of his own and thus did not actwrongfully
3. Humbulani exceeded his capacity as an owner and abused his right
4. Option 1 and 3 above

8 A terrorist places a time-bomb in a busy restaurant. The bomb explodes and injures Xholisi. In respect
of Xholisi’s injuries, the terrorist had:

1. dolus indirectus
2. dolus directus
3. dolus indeterminatus
4. dolus eventualis

9 In which of the following cases did the court decide that the test for negligence for a child is no longer
the reasonable child standard?

1. Bester v Commercial Union Versekeringsmaatskappy van SA Bpk 1973 (1) SA 769 (A)
2. Barnard v Santambank 1999 (1) SA 202 (SCA)
3. Jones NO v Santam Bpk 1965 (2) SA 542 (A)
4. Van Wyk v Lewis 1924 AD 438

10 Tshepo’s bull eats and tramples Grace’s maize crops. Tshepo forgot to close the gate between his and
Grace’s land. Which remedy or remedies may be available to Grace?

1. The actio de pauperie and actio legis Aquiliae


2. The actio de pauperie and interdict
3. The actio de pastu only
4. The actio de pastu and actio legis Aquiliae

11 Maureen inadvertently damages Lora’s drum. Which delictual action may be available to Lora?

1. The actio legis Aquiliae


2. The actio iniuriarum
3. The actio de pastu
4. None of the above
12 Musa negligently left his stationary donkey on the road. Shaun came along driving his new VW polo
and collided with the donkey. Musa now wishes to sue Shaun. Which defence may Shaun raise?

1. Private defence
2. Contributory intention
3. Contributory negligence
4. Consent

13 In Greater Johannesburg Transitional Metropolitan Council v ABSA Bank 1997 2 SA 691 (W), the court
held that:

1. a defence of contributory intention could succeed where both the plaintiff and the
defendant acted with intention
2. a person is guilty of negligence if his conduct falls short of that of the standard of the diligens
paterfamilias
3. for the purposes of the law of delict, intent and negligence may be present simultaneously
4. there is no single criterion for legal causation which is applicable in all instances

14 What is the criterion for determining factual causation?

1. Actio libera in causa.


2. Novus actus interveniens.
3. Conditio sine qua non.
4. The flexible approach.

15 In which one of the following cases did the court maintain that factual causation cannot always be
answered by strict adherence to logic but that common sense sometimes has to prevail?

1. International Shipping Co (Pty) Ltd v Bentley 1990 (1) SA 680 (A)


2. S v Van As 1967 4 SA 594 (A)
3. S v Mokgethi 1990 (1) SA 32 (A)
4. Lee v Minister of Correctional Services 2013 2 SA 144 (CC)

16 What does the talem qualem rule mean?

1. Damages have to be apportioned due to contributory fault


2. New intervening cause
3. You must take your victim as you find him
4. Reasonable foreseeability of harm

17 Prospective loss is best assessed in accordance with:

1. the sum-formula approach


2. the “once and for all” rule
3. the concrete approach to damage
4. res inter alios acta

18 Oratile and Mapule are farming on adjacent properties. Oratile bears a grudge against Mapule. He sets
up a device, which causes a loud bang every half an hour, on the common boundary between the 2
farms near Mapule’s homestead. As a result of this, Mapule and her family cannot sleep at night and
are irritated by day. Mapule asks Oratile to dismantle the device, but Oratile says that the device
serves the purpose of scaring baboons away from his orchards. Which delictual remedy may be
available to Mapule?

1. The actio de pauperie


2. The actio de feris
3. The actio de effuses vel deiectis
4. The interdict

19 Precious has a stall on the sidewalk where she sells homemade cakes. While Precious is tending to
customers, Donald and Jacob pass by and steal an entire tray full of cakes. While they run away,
Jacob’s hat falls off and Precious recognises him and reports the matter to the police. The police
manage to find Donald and Jacob, after they have consumed all the cakes. Precious wants to claim
damages from Jacob. Can she claim the whole amount of damages from Jacob?

1. Yes, Jacob is vicariously liable for the damage


2. Yes, joint wrongdoers are in solidum liable for the full damage
3. No, Jacob is known by Precious
4. No, Jacob and Donald are equally liable for the damage

20 Which one of the following is not a specific form of damnum iniuria datum?

1. Product liabilty
2. Pure economic loss
3. Unlawful competition
4. Defamation

Section B:

Question 1:

Lethabo suffers from epileptic fits. His doctor has prescribed him medication to control the onset of
the epileptic fits. One morning, while Lethabo is driving a forklift in the warehouse where he works,
he suffers an epileptic fit and the forklift veers into his co-worker Zanele, injuring her. It transpired
that Lethabo had not taken his medication prescribed by the doctor that morning. Zanele sustains
bodily injuries and is hospitalised. Zanele would like to institute a delictual action against Lethabo.
Discuss with reference to relevant authority whether Lethabo indeed acted for the purposes of the
law of delict. Restrict the scope of your answer to what is asked in the question, and note in
particular that the question does not deal with the element of wrongfulness. (10)

Conduct is defined as a voluntary human act or omission. “Voluntary” means that the person must be
able to control his muscular movements by means of his will. The act of the wrongdoer must be
voluntary to give rise to delictual liability. By raising the defence of automatism, a defendant attempts
to show that, according to the law, he didn’t act. Defendant may argue that the conduct complained of
doesn’t satisfy the requirement of voluntariness. He relies on the defence of automatism - that he
acted mechanically.

Conditions that may cause a person to act involuntarily as they render him incapable of controlling his
bodily movements: absolute compulsion (vis absoluta), sleep, unconsciousness, fainting fit, epileptic
fit, serious intoxication, blackout, reflex movements, strong emotional pressure, mental disease,
hypnosis, and a heart attack.
According to Molefe v Mahaeng, the defendant does not bear the onus to prove that he was in a state
of so-called sane automatism. The onus is on the plaintiff to prove that the defendant acted
voluntarily. In the Du Plessis case, X (72) was charged with negligent driving as he had injured a
pedestrian. He experienced a blackout due to low-blood pressure. He was found not guilty.

If we apply these principles to the given facts, Tumelo had been receiving medical treatment for a
diagnosed condition of insomnia, but failed to take his prescribed medication on that particular
occasion. A person can’t rely on automatism if he intentionally placed himself in a mechanical state
(actio libera in causa). Actio libera in causa: Defence of automatism won’t succeed if defendant
intentionally created the situation in which he acts involuntarily in order to harm another. The
defendant (Lethabo) will be held liable for his culpable conduct in creating the state of automatism
which resulted in damage to the plaintiff (Zanele).

Defendant may not successfully rely on the defence of automatism where he was negligent regarding
his automatic “conduct”. Where the reasonable man would’ve foreseen the possibility of causing harm
while in a state of automatism, eg. in Victor case, X was convicted of negligent driving despite causing
the accident during an epileptic fit, as he’d been suffering fits for 13 years and the reasonable man
would’ve foreseen the possibility of causing harm while in a state of automatism. In this case, X knew
he may suffer an epileptic fit and still drove a motor vehicle.

Automatism doesn’t mean that there’s no voluntary act whatsoever by the defendant which caused the
damage, but only that the conduct in question wasn’t voluntary. Only the voluntary act closest to the
harmful consequence is of relevance, and it’s therefore unnecessary to consider prior voluntary acts.

According to Van der Merwe and Olivier, automatism does not really exclude the element of conduct in
a delict, but rather wrongfulness or fault. This view may be illustrated with this eg: X buys a knife which
he keeps at his bedside when he retires at night. One night, X dreams that he’s being attacked. While
still half asleep, X grabs the knife and wounds Y (who sleeps on a bed next to him). According to Van
der Merwe and Olivier, the stabbing with the knife wasn’t a voluntary act, but that there are other
(prior) voluntary acts on X’s part which also caused Y’s injuries. Eg. the mere fact that X went to bed
with the knife next to him, is, according to the authors, a voluntary act which caused Y’s injuries and it
would thus be incorrect to conclude that X didn’t in any way act voluntarily with regard to such injuries.
This voluntary conduct by X wasn’t accompanied by fault and he won’t be held delictually liable.

However, a person can’t rely on automatism if he negligently placed himself in a mechanical state
(Lethabo didn’t take his medication). Thus, Lethabo was probably negligent, or could even have had
intention in the form of dolus eventualis. Therefore, a reliance on automatism would fail in this case.

Question 2:

Mina, while eight and half months pregnant, experienced severe abdominal and back pains. Her
husband immediately took her to Midlands Private Hospital. The medical staff attending to her
failed to perform a timely caesarean section, resulting in the baby sustaining brain damage. Answer
the following two questions with reference to these facts.

2.1 Was the conduct of the medical staff at Midlands Private Hospital wrongful? Discuss with reference
to relevant authority. Restrict the scope of your answer to the element of wrongfulness only. (10)

As a general rule, a person does not act wrongfully for the purposes of the law of delict if he omits to
prevent harm to another person. Liability only follows if the omission was in fact wrongful, and this will
be the case only if a legal duty rested on the defendant to act positively to prevent harm from
occurring and he failed to comply with that duty. The question of whether such a duty existed is
answered with reference to the criterion of the legal convictions of the community and legal policy.
This question deals with the wrongfulness of an omission. The basic question to determine whether an
omission is wrongful is whether a legal duty to act was present and was breached. This is determined
with reference to the legal convictions of the community, or the boni mores. Factors which may serve
as indications that a legal duty rested on the defendant include: prior conduct (omissio per
commissionem); control of a dangerous object; rules of law; a special relationship between the parties;
particular office; contractual undertaking for the safety of a third party; and creating of an impression
that the interests of a third person will be protected.

In the final analysis of whether a legal duty existed, we are dealing with the determination of the
reasonableness of the defendant’s failure to act in view of all the circumstances of the case. It is not
imperative that the omission in question falls into one of the crystallised categories. This well-known
example of the champion swimmer may serve as an example. A champion swimmer is walking
alongside a river and sees a small child drowning. He fails to rescue the child. As a result of his
omission, the child suffers serious brain damage and becomes quadriplegic. In determining whether a
duty rested on him to rescue the child, the swimmer’s conduct can’t be classified under any of the
stereotyped categories that indicate a legal duty. Consequently, recourse must be had to the general
test for wrongfulness. By means of the boni mores test, a balancing process must take place between
the interests of the swimmer (eg. inconvenience and damage to his clothes) and the interests of the
child (serious violation of his physical integrity) and the child’s parent/guardian (great financial cost
brought about by the child’s handicap). Public interest also plays a part here.

It must be decided whether the swimmer’s omission evokes not merely moral indignation but should
also be regarded as wrongful according to the legal convictions of the community and that he should
render compensation for the damage suffered. In this eg. it will be decided that a legal duty rested on
the swimmer to take steps to rescue the child. However, the scales may favour the defendant
(swimmer) if, eg. there were crocodiles in the vicinity and that he would place his own life in danger
were he to rescue the child; the law no longer requires that a person regard another’s life as more
important than his own. In principle, a defendant doesn’t act wrongfully when he fails to act positively
to prevent harm to another. If these principles are applied to the given facts, we can conclude that the
medical staff at Midlands Private Hospital acted as a reasonable medical staff would have done in the
particular circumstances and the conduct of the medical staff was probably not wrongful.

2.2 Did the medical staff at Midlands Private Hospital act negligently? Discuss with reference to relevant
authority. Restrict the scope of your answer to negligence only. (10)

Whether wrongdoer possesses proficiency or expertise in the allegedly negligent conduct, it affects
application of reasonable person test. The general test for negligence, ie. test of hypothetical
reasonable person in the position of wrongdoer, can’t be applied when considering the conduct of
defendant where such conduct calls for expertise. Thus, in the case of an expert, the test for
negligence in the exercise of expert activity (“professional negligence”) is the test of the reasonable
expert; eg. the reasonable medical doctor.

The reasonable expert is identical to the reasonable person in all respects, except that a reasonable
measure of the relevant expertise is added. The standard of expertise is described as “reasonable”
because regard is had, not to the highest degree of expertise in the relevant profession or occupation,
but to the general or average level of such expertise. In Van Wyk v Lewis, reference is made to “the
general level of skill and diligence possessed and exercised at the time by the members of the branch
of the profession to which the practitioner belongs”. In this case it was held that, as far as the medical
profession was concerned, the same expertise cannot be expected from a general practitioner as from
a specialist. In Durr v ABSA Bank Ltd, the SCA approved of the approach in Van Wyk v Lewis and
emphasised that it is for the court to decide what is reasonable under the circumstances. It will pay
much attention to the views of the profession but is not bound to adopt them.
The maxim imperitia culpae adnumeratur means that ignorance or lack of skill is deemed to be
negligence. However, this maxim is misleading because our law does not accept that mere ignorance
constitutes negligence. The principle embodied in this maxim applies where a person undertakes an
activity for which expert knowledge is required while he knows or should reasonably know that he
lacks the requisite expert knowledge and should therefore not undertake the activity in question. An
example of this is where X, who has no expertise in piloting an aircraft, flies an aircraft and causes an
accident. X’s blameworthiness in this example is not to be found in his incompetence in piloting an
aircraft, but in the fact that, while he knows or should reasonably know that he is incompetent, he
nevertheless attempts to perform the expert activity.

Question 3:

3.1 Tom and Hank play with their Frisbee in the front yard. During their game, the Frisbee gets stuck in a
tree. They try throwing a variety of objects at the Frisbee in an attempt to dislodge it. Finally, they
throw a baseball bat at it, which nudges the Frisbee out of the tree. The bat, however, damages the
window of Caroline’s house. Caroline institutes a delictual action against Tom and Hank for the
damage to the window. Tom and Hank explain that they were trying to get their Frisbee out of the
tree and raise necessity as a ground of justification. Indicate whether Tom and Hank will be successful
with necessity as a ground of justification. Discuss with reference to relevant authority. (5)

A state of necessity exists when defendant is placed in such a position by superior force (vis maior)
that he is able to protect his legally recognised interests (or those of someone else) only by reasonably
violating the interests of an innocent person. Thus, Tom and Hank will not be successful with necessity
as a ground of justification. If they foresaw the possibility of the bat breaking a window in Caroline’s
house, but decided that it will not happen and if the ball indeed breaks the window, Tom and Hank
had luxuria in respect of the damage.

3.2 Betty and Oscar go out to a fancy restaurant for dinner. Oscar finds the dessert not to his liking and
begins screaming at and insulting Betty as she chose the restaurant. Betty has had enough of his
behaviour and returns the insult. This angers Oscar and he institutes a delictual action against Betty
for insulting him in public. What defence may Betty rely on? Discuss with reference to relevant
authority. (5)

Provocation is present when a defendant is provoked or incited by words or actions to cause harm to
the plaintiff. Eg. X (Oscar) insults Y (Betty) and Y (Betty) returns the insult. Should X (Oscar) now
institute a claim against Y (Betty), Y (Betty) relies on provocation: she claims that X (Oscar) provoked
(enticed) her and that for this reason X’s (Oscar’s) claim should fail. Thus, Betty may rely on the
defence of provocation.

Question 4:

Kabelo is in a hurry to get out of the bus. He pushes Zama who is in front of him. Zama falls out of
the bus and twists her ankle badly. Zama is treated in a nearby hospital and is issued with a special
protective plastic boot to prevent further injury to her ankle while the healing is in progress. Zama
finds this boot very cumbersome and uncomfortable. The next day, Zama slips with the boot, falls
and breaks her left arm. Does a legal causal link exist between Kabelo’s conduct and Zama’s broken
arm? Discuss in detail with reference to case law. (10)

The test for legal causation is the so-called flexible approach, as formulated in S v Mokgethi 1990 and
International Shipping Co (Pty) Ltd v Bentley 1990. In Mokgethi, a bank robber shot a teller. The teller
was rendered a paraplegic and was discharged from hospital in a wheelchair. Subsequently, the
paraplegic man failed to shift his body position in the chair frequently and developed pressure sores,
eventually dying from complications. The question that arose was whether the shot fired by the
robber was the legal cause of the teller’s death. According to the court, the main question in respect
of legal causation is whether there is a close enough relationship between the wrongdoer’s conduct
and its consequence for such consequence to be imputed to the wrongdoer in view of policy
considerations based on reasonableness, fairness and justice. Several other legal causation theories
exist, such as adequate causation, direct consequences, foreseeability and novus actus interveniens.
None of these criteria is suitable to be applied to all situations. They may, however, be used as
subsidiary aids when employing the flexible approach. In the Mokgethi case, the court held that the
shot was not a legal cause of the death. If these principles are applied to the facts in the question, the
conclusion is probably that Zama’s broken arm was too remote and should not be imputed to the
wrongdoer. It could also be argued that a so-called novus actus interveniens (a new intervening act)
was constituted by Zama’s fall, and this strengthens the conclusion that there’s no legal causal link
between Kabelo’s conduct and Zama’s broken arm.

Question 5:

5.1 Name the requirements for the granting of an “interdict”. (3)

The courts state 3 requirements for the granting of an interdict:

• There must be an act (or threatening act) by the respondent.


• The act (or threatening act) must be wrongful.
• No other suitable remedy must be available to the applicant.

5.2 Name the requirements for the “actio de pauperie”. (3)

To succeed in bringing the actio de pauperie, these requirements must be met:

• Defendant must be the owner of the animal when the damage is inflicted.
• Animal must be a domestic animal.
• Animal must act contra naturam sui generis when inflicting the damage.
• Prejudiced person or his property must be lawfully present at location where the damage is
inflicted.

5.3 Name the requirements for “vicarious liability”. (3)

• There must be an employer-employee relationship at the time when the delict iscommitted.
• The employee must commit a delict.
• The employee must act within the scope of his employment when the delict is committed.

5.4 Explain the principle of “mitigation of loss” in one sentence. (1)

It is a principle of the law of delict that a plaintiff may not recover damages for a loss which is the
factual result of the defendant’s conduct but which could have been prevented if the plaintiff had
taken reasonable steps.
May/June 2019
1 Which of the following statements is correct about the South African law of delict:

1. A wrongdoer may be held liable only if his conduct satisfies the requirements of a specific
delict
2. The South African law of delict follows a casuistic approach
3. The law of delict belongs to the part of private law known as the “law ofobligations”
4. The law of delict consists of a set of separate delicts

2 Indicate the incorrect statement with regard to delictual liability:

1. The boni mores test is an objective test


2. Wrongfulness is always determined ex ante
3. A harmful consequence is itself insufficient to constitute wrongfulness
4. An act and its resulting consequence are always separated by time and space

3 In which one of the following cases did the court accept the doctrine of subjective rights?

1. Halliwell v Johannesburg Municipal Council 1912 AD 659


2. Minister van Polisie v Ewels 1975 3 SA 590 (A)
3. Telematrix (Pty) Ltd t/a Marix Vehicle Tracking v Advertising Standards Authority SA 2006 1 SA
461 (SCA)
4. Universiteit van Pretoria v Tommie Meyer Films (Edms) Bpk 1977 4 SA 376 (T)

4 In which of the following circumstances may X rely on “private defence”?

1. X is attacked and bitten while playing with the neighbour’s dog. X reacts by striking the dog
with a stick, injuring it.
2. A policeman with a valid warrant of arrest for X, arrests X. X resists the arrest and strikes the
policeman, injuring him.
3. X, a prisoner whose prison term has expired, is not released. In order to escape prison, X
assaults the guard who refuses to release him.
4. The owner of a farm Y, while placing his hand on his gun, tells X that he is trespassing on
his land and that if he does not leave within the next hour he will forcefully remove him.
X immediately reacts by striking Y with a cricket bat.

5 In which of the following circumstances may X not rely on the defence of “necessity”?

1. X is attacked and bitten while playing with the neighbour’s dog. X reacts by striking the dog
with a stick, injuring it.
2. X believes his daughter’s life is in danger as she accidently swallowed some liquid detergent.
X rushes to the hospital with his daughter and in his rush causes damage to Y’s car. It
transpired that X’s daughter’s life was not in danger as the liquid detergent was not harmful.
3. X damages his neighbour’s door while the neighbour is on holiday, in order to gain entry into
the house because there seems to be smoke coming from within.
4. Y, while holding a knife to X’s wife’s throat, tells X that if he does not kill Z, he will kill X’s wife.
X, out of fear for his wife’s life, follows Y’s instructions and kills Z.

6 Y verbally insults X who in retaliation slaps Y across the cheek. If Y institutes a delictual action for the
infringement of his personality rights, X may rely on:
1. Private defence
2. Necessity
3. Provocation
4. None of the above

7 Which of the following statements is correct with regard to the requirements of consent?

1. There must be an agreement between the person consenting (the injured person) and the
actor
2. The person giving the consent must have full legal capacity to act
3. Mere knowledge of the risk of harm is sufficient for the requirement of “appreciation” of the
risk of harm
4. The person consenting must be able to express his will

8 Luke places a time-bomb in a busy restaurant. The bomb explodes two hours later, causing injury to
the people in and in close proximity to the restaurant. Which one of the following forms of fault did X
have in respect of the injury to vistims of the explosion?

1. Dolus determinatus
2. Dolus eventualis
3. Dolus indeterminatus
4. Luxuria

9 In which one of the following cases did the defence of volenti non fit iniuria succeed?

1. Lampert v Hefer 1955 2 SA 507 (A)


2. Maartens v Pope 1992 4 SA 883 (N)
3. Netherlands Insurance Co of SA v Van der Vyver 1968 1 SA 412 (A)
4. Santam Insurance Co Ltd v Vorster 1973 4 SA 764 (A)

10 In which one of the following delictual remedies is there no need to prove harm, loss or damage?

1. Interdict
2. Actio de pastu
3. Actio de pauperie
4. Action for pain and suffering

11 If a person can differentiate between right and wrong, and act in accordance with this insight, he or
she:

1. is in a state of automatism
2. can rely on a ground of justification
3. is accountable
4. is negligent

12 Imraan, a financial adviser, negligently advises Trevor to make a bad investment. As a result of this,
Trevor suffers significant financial loss. Which one of the following delictual remedies may be available
if Trevor wishes to recover his financial loss from Imraan?
1. The actio legis Aquiliae
2. The actio iniuriarum
3. The action for pain and suffering
4. None of the above

13 The maxim, “you must take your victim as you find him”, is also known as the:

1. sum-formula approach
2. “talem qualem” rule
3. “once and for all” rule
4. Concrete approach to damage

14 Sylvester steals a laptop from an electronics store. To get to the laptop, he smashes a window made
from expensive glass. In respect of the damage to the window, Sylvester has:

1. Dolus indirectus
2. Dolus eventualis
3. Dolus indeterminatus
4. Luxuria

15 Defamation is an infringement of a person’s…?

1. feelings
2. good name
3. privacy
4. identity

16 Which of the following is not a ground of justification against defamation?

1. Privilege or privileged occasion


2. Truth and public interest
3. Media privilege
4. Provocation

17 If patrimonial loss is caused in a negligent manner, the most appropriate delictual remedy to consider
is:

1. The actio legis Aquiliae


2. The actio iniuriarum
3. The action for pain and suffering
4. None of the above

18 If bodily injury is caused in a negligent manner, the most appropriate delictual remedy to consider is:

1. The actio legis Aquiliae


2. The actio iniuriarum
3. The action for pain and suffering
4. None of the above
19 If the dignity of a person is infringed in a negligent manner, the most appropriate delictual remedy to
consider is:
1. The actio legis Aquiliae
2. The actio iniuriarum
3. The action for pain and suffering
4. None of the above

20 Tyron is training to be a tattoo artist. He asks Latitia whether he can practice his newly acquired skills
on her. Tyron explains to Latitia that he will apply a temporary tattoo on her back which can be easily
removed by washing it off with water. Latitia agrees to this. Tyron designs an eagle tattoo across
Latitia’s back. When Latitia discovers that the tattoo is permanent, she wants to institute a delictual
action against Tyron. Tyron may rely on the following ground of justification:

1. Necessity
2. Official capacity
3. Consent
4. None of the above

Section B:

Question 1:

1.1 Define a delict. (2)

A delict (wrongful conduct) is the act of a person which in a wrongful (legally reprehensible) and
culpable (legally blameworthy) way causes loss (damage) to another.

1.2 Damage can take one of two forms, name these two forms. (2)

Patrimonial loss (damnum iniuria datum) or Injury to personality (iniuria).

1.3 Name the requirements for “vicarious liability”. (3)

• There must be an employer-employee relationship at the time when the delict iscommitted.
• The employee must commit a delict.
• The employee must act within the scope of his employment when the delict is committed.

1.4 Name the requirements for the actio de pastu. (3)

• The defendant must be the owner of the animal when the damage is caused.
• The animal must cause damage by eating plants.
• The animal must act of its own volition when causing the damage.

1.5 Explain the difference between a delict and a contract. (2)

Breach of contract is only constituted by the non-fulfilment by a contractual party of a contractual


personal claim or an obligation to perform. A delict is constituted by the infringement of any legally
recognised interest of another party, excluding the non-fulfilment of a duty to perform by a
contractual party. The primary remedy for breach of contract is directed at the enforcement,
fulfilment, or execution of the contract (with a claim for damages playing a secondary part). Delictual
remedies are primarily directed at damages (or satisfaction), and not at fulfilment.
1.6 Explain the difference between a “constitutional wrong” and a “delict”. (3)

The requirements for a delict and a constitutional wrong differ. As a result, not every delict is
necessarily also a constitutional wrong, and vice versa. Besides, unlike a delictual remedy which is
aimed at compensation, a constitutional remedy (even in the form of damages) is directed at
affirming, enforcing, protecting and vindicating fundamental rights and at preventing or deterring
future violations of Chapter 2. A constitutional wrong and a delict (or their remedies) should therefore
not be treated alike and for conceptual clarity, the term constitutional “delict” or “tort” should rather
be avoided.

Question 2:

Tumelo suffers from insomnia and when he does sleep, often sleepwalks. His doctor has prescribed
him medication to control these conditions. Tumelo goes on a camping trip with his friends. On one
evening while sleepwalking, he thinks he is being attacked by a robber and stabs one of his friends,
Charles. In reality, there was no robber and Charles was trying to assist Tumelo to get back to his
bed. It transpired that Tumelo had not taken his medication prescribed by the doctor that morning.
Charles was seriously injured and hospitalised. Charles would like to institute a delictual action
against Tumelo. Discuss with reference to relevant authority whether Tumelo indeed acted for the
purposes of the law of delict. Restrict the scope of your answer to what is asked in the question, and
note in particular that the question does not deal with the element of wrongfulness. (10)

Conduct is defined as a voluntary human act or omission. “Voluntary” means that the person must be
able to control his muscular movements by means of his will. The act of the wrongdoer must be
voluntary to give rise to delictual liability. By raising the defence of automatism, a defendant attempts
to show that, according to the law, he didn’t act. Defendant may argue that the conduct complained of
doesn’t satisfy the requirement of voluntariness. He relies on the defence of automatism - that he
acted mechanically.

Conditions that may cause a person to act involuntarily as they render him incapable of controlling his
bodily movements: absolute compulsion (vis absoluta), sleep, unconsciousness, fainting fit, epileptic
fit, serious intoxication, blackout, reflex movements, strong emotional pressure, mental disease,
hypnosis, and a heart attack.

According to Molefe v Mahaeng, the defendant does not bear the onus to prove that he was in a state
of so-called sane automatism. The onus is on the plaintiff to prove that the defendant acted
voluntarily. In the Du Plessis case, X (72) was charged with negligent driving as he had injured a
pedestrian. He experienced a blackout due to low-blood pressure. He was found not guilty.
If we apply these principles to the given facts, Tumelo had been receiving medical treatment for a
diagnosed condition of insomnia, but failed to take his prescribed medication on that particular
occasion. A person can’t rely on automatism if he intentionally placed himself in a mechanical state
(actio libera in causa). Actio libera in causa: Defence of automatism won’t succeed if defendant
intentionally created the situation in which he acts involuntarily in order to harm another. The
defendant (Tumelo) will be held liable for his culpable conduct in creating the state of automatism
which resulted in damage to the plaintiff (Charles).

Defendant may not successfully rely on the defence of automatism where he was negligent regarding
his automatic “conduct”. Where the reasonable man would’ve foreseen the possibility of causing harm
while in a state of automatism, eg. in Victor case, X was convicted of negligent driving despite causing
the accident during an epileptic fit, as he’d been suffering fits for 13 years and the reasonable man
would’ve foreseen the possibility of causing harm while in a state of automatism. In this case, X knew
he may suffer an epileptic fit and still drove a motor vehicle.

Automatism doesn’t mean that there’s no voluntary act whatsoever by the defendant which caused
the damage, but only that the conduct in question wasn’t voluntary. Only the voluntary act closest to
the harmful consequence is of relevance, and it’s therefore unnecessary to consider prior voluntary
acts.

According to Van der Merwe and Olivier, automatism does not really exclude the element of conduct
in a delict, but rather wrongfulness or fault. This view may be illustrated with this eg: X buys a knife
which he keeps at his bedside when he retires at night. One night, X dreams that he’s being attacked.
While still half asleep, X grabs the knife and wounds Y (who sleeps on a bed next to him). According to
Van der Merwe and Olivier, the stabbing with the knife wasn’t a voluntary act, but that there are other
(prior) voluntary acts on X’s part which also caused Y’s injuries. Eg. the mere fact that X went to bed
with the knife next to him, is, according to the authors, a voluntary act which caused Y’s injuries and it
would thus be incorrect to conclude that X didn’t in any way act voluntarily with regard to such
injuries. This voluntary conduct by X wasn’t accompanied by fault and he won’t be held delictually
liable.

However, a person can’t rely on automatism if he negligently placed himself in a mechanical state
(Tumelo didn’t take his medication). Thus, Tumelo was probably negligent, or could even have had
intention in the form of dolus eventualis. Therefore, a reliance on automatism would fail in this case.

Question 3:

There is dry grass on Tebogo’s farm. Tebogo fails to cut the grass. A fire breaks out in the grass and
spreads to the neighbour’s land, Quinton’s farm, where it causes some damage. Can Tebogo be held
liable for the damage to Quinton’s farm? Discuss with reference to relevant authority. Restrict the
scope of your answer to the element of wrongfulness and liability for an omission. (10)

Liability for an omission follows only if the omission was in fact wrongful, and this will be the case only
if a legal duty rested on Tebogo to act positively (by cutting his grass) to prevent harm from occurring,
and Tebogo failed to comply with that duty. One of the factors that can indicate that a legal duty
existed to prevent prejudice in the case of an omission is control of a dangerous object. Firstly, there
must be actual control, and secondly, in light of such control there must be a legal duty on the
defendant to take steps to prevent damage resulting from his omission to exercise proper control.
Tebogo owns the potentially dangerous object of property with grass that could spread fire, so he has
control over it.
If person had knowledge or foresight that his omission might cause harm, is unreasonable and shows
wrongfulness of his conduct. Where a person was aware of a dangerous situation (eg. fire or
dangerous hole on his land), this may factor in determining whether he had to exercise control over
the danger and, then whether a legal duty rested on him to take steps to avert loss.

The law (either common law or statute) places an obligation upon a person to perform certain acts. Eg.
common law obliges owner of lower land to provide lateral support for neighbour’s land. If neighbour
suffers damage as a result of their failure to perform this duty, their conduct is prima facie wrongful.

A statutory provision can grant a delictual action, or it can justify a conclusion that a common law legal
duty exists. The question of whether it’s equitable and reasonable to award plaintiff a claim for
damages or not in view of the noncompliance with a legal provision must still be asked. The conduct
will be wrongful, not due to the non-compliance with the statutory legal duty, but because it’s
reasonable in the circumstances to compensate the plaintiff for the infringement of his right.
Reasonableness is determined with reference to the legal convictions of the community and legal
policy. Therefore, Tebogo can probably be held liable for Quinton’s damage because it will probably be
considered that proper exercise of control of this object would be necessary with reference to the
legal convictions of the community.

Question 4:

4.1 David, while fielding during the course of a social cricket match, was struck on the head by a cricket
ball. As a result of this, David sustained a serious head injury. His fellow player, Kagiso, struck the
cricket ball which caused the injury to David’s head. Both David and Kagiso were playing according the
rules and conventions of the sport. David wishes to claim delictual damages from Kagiso for the head
injury he sustained. Kagiso may raise a ground of justification against this claim. Discuss with
reference to authority which ground of justification Kagiso may rely on as well as the likelihood of the
court upholding the defence. Restrict the scope of your answer to the ground of justification. (7)

The principle that a defendant isn’t liable where the injured person has consented to injury or the risk
thereof, is embodied in the maxim volenti non fit iniuria (a willing person isn’t wronged; he who
consents can’t be injured). The maxim volenti non fit iniuria, in short volenti, is used to describe both
forms of consent. Volenti non fit iniuria may thus mean either consent to injury, or consent to the risk
of injury.

In the given facts, consent to the risk of injury applies. In the case of consent to the risk of injury, the
injured party consents to the risk of harm caused by the defendant’s conduct. Eg. a participant in sport
consents to the risks involved in such sport, like a cricket ball striking a cricket player in the head.
Should the risk contained in the sports injuries above ensue, the injured person won’t be able to hold
the defendant delictually liable, because he has consented to risk of such harm.

In the given facts, the defence of volenti non fit iniuria as a ground of justification will probably
succeed as Kagiso can state that David consented to the risk of injury and the injured person (David)
won’t be able to hold the defendant (Kagiso) delictually liable, because he (David) has consented to
risk of such harm.

4.2 Gerhard regularly invited Berg to his luxury farm. Wild ostriches, among other domesticated
animals, inhabit the farm. Gerhard warned Berg to be cautious around the ostriches and not to
startle them. Berg informs Gerhard that he knows how to deal with these ‘birds’ and always
interacted with one ostrich in particular during his visits. Berg always teased and tauntedthis
particular ostrich for self-amusement. During one of his visits, Berg picked up a stone and threw it in
the direction of the ostrich. The ostrich began to chase Berg. Berg then ran towards the house and
tripped and fell, injuring his achilles tendon. Berg was hospitalised and wishes to recover delictual
damages from Gerhard based upon Gerhard’s alleged negligent conduct of keeping wild animals in a
place that the public can have access to. Gerhard can raise a defence against this allegation. Which
defence may Gerhard rely upon and what will the effect of a successful reliance on the defence be?
Discuss in detail with reference to case law and legislation. (8)

With the actio de pauperie action, prejudiced person may claim damages from owner of a domestic
animal which has caused damage. Characteristic of actio is that fault on part of owner isn’t a
requirement for liability. This action entails strict liability.

To succeed in bringing the actio de pauperie, these requirements must be met:

• Defendant must be the owner of the animal when the damage is inflicted.
• Animal must be a domestic animal.
• Animal must act contra naturam sui generis when inflicting the damage.
• Prejudiced person or his property must be lawfully present at location where the damage is
inflicted.

The animal involved must have acted contrary to what may be expected of a decent and well-behaved
animal of its kind. A dog that bites, a horse that jumps and an ox that butts, in principle, act contra
naturam. However, this is qualified by the requirement that the animal must have caused the damage
spontaneously from “inward excitement or vice” or sponte feritate commota. As a rule, the animal
doesn’t act contra naturam if it’s reacting to external stimuli.

Defences against the actio de pauperie have developed from the requirement of spontaneous
conduct. They are culpable or provocative conduct on part of prejudiced person, culpable conduct on
part of an outsider and provocation by another animal. All these cases have the effect of excluding
liability, because the animal didn’t act from “inward excitement or vice” and thus didn’t act contra
naturam sui generis. Apart from these defences, the defence of volenti non fit iniuria in the form of
voluntary assumption of risk is also available to the defendant.

Therefore, culpable or provocative conduct on part of prejudiced person (Berg) is present as Berg
always teased and taunted the ostrich, and he picked up a stone and threw it in the direction of the
ostrich. Thus, this case has the effect of excluding liability because the animal (ostrich) didn’t act from
“inward excitement or vice” and didn’t act contra naturam sui generis. Gerhard can also rely on the
defence of volenti non fit iniuria in the form of voluntary assumption of risk.

Question 5:

In our law, the causing of damage through conduct is required for a delict. In other words, a causal
nexus between the conduct and the damage must be present. A person cannot be held liable if the
person did not cause any damage. Discuss the test used to determine factual causation and the test
used to determine legal causation. (10)

The generally accepted test for factual causation is the conditio sine qua non test, or ‘‘but for test’’.
This entails mentally eliminating, or thinking away, the conduct. If the damage then also disappears, a
factual causal link is present between the conduct and the damage. This test is subject to much
criticism. Among others, it is said to be based on circular logic and is, at best, a way to express the
existence of a causal nexus that has been determined in another way. Neethling and Potgieter argue
that evidence and human experience are sufficient to determine whether one fact flowed from
another fact, and that a so-called test of factual causation is superfluous. However, the courts
consistently state that the conditio sine qua non is the test of factual causation. For example, Max
bumps into Sam. Sam falls and breaks his leg. Thus, if Max hadn’t bumped into Sam, Sam would not
have fallen and broken his leg. Therefore a factual causal link is present between Max’s conduct and
Sam’s damage.

The test for legal causation is the flexible approach, as formulated in S v Mokgethi 1990 and
International Shipping Co (Pty) Ltd v Bentley 1990. In Mokgethi, a bank robber shot a teller. The teller
was rendered a paraplegic and was discharged from hospital in a wheelchair. Subsequently, the
paraplegic man failed to shift his body position in the chair frequently and developed pressure sores,
eventually dying from complications. The question arose whether the shot fired by the robber was the
legal cause of the teller’s death. According to the court, the main question in respect of legal causation
is whether there is a close enough relationship between the wrongdoer’s conduct and its consequence
for such consequence to be imputed to the wrongdoer based on reasonableness, fairness and justice.
Several other legal causation theories exist, such as adequate causation, direct consequences,
foreseeability and novus actus interveniens. None of these criteria is suitable to be applied to all
situations. They may, however, be used as subsidiary aids when employing the flexible approach. In
the Mokgethi case, the court held that the shot was not a legal cause of the death. For example, Max
bumps into Sam. Sam falls and breaks his leg. Sam is admitted to hospital. A nurse leaves the ward
windows open, and Sam contracts pneumonia. The conclusion is probably that Sam’s pneumonia was
too remote and should not be imputed to the wrongdoer (Max). It could also be argued that a novus
actus interveniens (a new intervening act) was constituted by a nurse leaving the ward windows open,
and this strengthens the conclusion that there is no legal causal link between Max’s conduct and Sam’s
pneumonia.

Oct/Nov 2018
1 Indicate the incorrect statement with regard to the law of delict and the constitution:

1. An infringement of a right may constitute a constitutional wrong and a delict


2. The requirements for a delict and a constitutional wrong differ materially
3. Direct application means that the state must generally respect fundamental rights and not
infringe them
4. Both a constitutional remedy and a delictual remedy are aimed primarily at compensation

2 Themba inadvertently damages Zanele’s bongo drum. Which delictual action may be available to
Zanele?

1. The actio iniuriarum


2. The actio legis Aquiliae
3. The action for pain and suffering
4. The actio de pastu

3 Which one of the following is incorrect?

1. An animal cannot act for purposes of the law of delict


2. A public school can act for the purpose of the law of delict
3. Behaviour must be willed to qualify as a voluntary act
4. Conduct may consist of a commission or an omission

4 Bongani’s cow eats and tramples Mandla’s maize crops. Bongani forgot to close the gate between his
and Mandla’s land. Which remedy or remedies may be available to Mandla?

1. The actio de pauperie and actio legis Aquiliae


2. The actio de pauperie and interdict
3. The actio de pastu only
4. The actio de pastu and actio legis Aquiliae

5 Which of the following is incorrect?

1. An epileptic seizure may cause a person to behave involuntarily and enables him to rely on
automatism.
2. A person may not rely on automatism if he intentionally created a situation in which he
behaves involuntarily.
3. A person relying on sane automatism as a defence in a delictual case has the onus to prove it.
4. According to Van der Merwe and Olivier, automatism does not exclude conduct in delict.
6 Mark employs electronic equipment to listen in on Johan’s telephone conversations. Which remedy or
remedies may be available for Johan?

1. The actio iniuriarum


2. The action for pain and suffering
3. Interdict
4. The action for pain and suffering and interdict

7 Which is the odd one out?

1. Necessity
2. Official capacity
3. Consent
4. Mistake

8 The following case is regarded as authority for the proposition that killing a person to protect property
may in certain circumstances be justified by private defence:

1. Ex parte Minister van Justisie: in re S v Van Wyk 1967 1 SA 488 (A)


2. S v Goliath 1972 3 SA 1 (A)
3. Molefe v Mahaeng 1999 1 SA 562 (SCA)
4. Carmichele v Minister of Safety and Security 2001 4 SA 938 (CC)

9 The following case is regarded as authority for the proposition that killing of an innocent person may
under certain circumstances be justified by necessity:

1. Ex parte Minister van Justisie: in re S v Van Wyk 1967 1 SA 488 (A)


2. S v Goliath 1972 3 SA 1 (A)
3. Molefe v Mahaeng 1999 1 SA 562 (SCA)
4. Carmichele v Minister of Safety and Security 2001 4 SA 938 (CC)

10 Which one of the following is incorrect?

1. In principle, the test for wrongfulness is objective in nature.


2. Subjective factors are usually irrelevant in the wrongfulness inquiry.
3. An improper motive may be relevant to the wrongfulness inquiry in delict cases.
4. Subjective knowledge that someone may be harmed is always irrelevant to the
wrongfulness inquiry.

11 Jaco insults Peter. Peter goes home feeling very depressed. An hour later, Peter phones Jaco and
retaliates by insulting Jaco. If Jaco institutes a delictual action for the infringement of his personality
rights, Jaco may rely on the following defence:

1. Private defence
2. Provocation
3. Necessity
4. None of the above
12 Melvin practices his golf swing in his back yard. He foresees the possibility of his ball breaking a
window in his neighbour’s house, but decides that it will not happen. If the ball indeed breaks the
window, Melvin had the following in respect of the damage:

1. Dolus directus
2. Dolus indirectus
3. Dolus eventualis
4. Luxuria

13 A terrorist places a time-bomb in a busy shopping mall. The bomb explodes and injures Kagiso. In
respect of Kagiso’s injuries, the terrorist had:

1. Dolus indeterminatus
2. Dolus directus
3. Gross negligence
4. Luxuria

14 Gawie wants to spy on Manie, his competitor in business. For this purpose, he installs a surveillance
device in the roof of the premises where Manie’s business is located. Gawie needs to break a hole into
the roof to accomplish this. He foresees the possibility that rainwater may now leak into the building
and may damage Manie’s merchandise, but reconciles himself with the possibility and proceeds
nevertheless. After three days, he has managed to learn all Manie’s trade secrets. In respect of the
damage to the roof, Gawie has:

1. Dolus directus
2. Dolus indirectus
3. Dolus eventualis
4. Dolus indeterminatus

15 Which one of the following is not a requirement for the interdict?

1. Conduct/threatening conduct
2. Fault/threatening fault
3. Wrongfulness/threatening wrongfulness
4. Unavailability of another suitable remedy

16 Which of the following principles relates to the rule that you must “take your victim as you find him”?

1. The “talem qualem” rule


2. The “once and for all” rule
3. The “sum-formula” approach
4. The “mitigation” principle

17 Zara’s two-year-old daughter falls down a flight of stairs. Zara believes that the child has sustained a
brain injury and rushes her to the hospital. On the way to the hospital, Zara drives over the
neighbour’s dog which subsequently dies. The neighbour institutes a delictual claim against Zara. It
later transpires that Zara’s daughter merely suffered from mild shock. Which one of the following
defences may Zara rely on?

1. Private defence
2. Provocation
3. Necessity
4. None of the above defences

18 Which of the following requirement/s must be present before provocation may be raised as a
defence?

1. The conduct of the defendant must be immediate


2. The defendant must be accountable
3. The defendant must not have contributed to the provocative conduct
4. 2 and 3 above

19 Isaac purchases a ticket to a boat ride. The ticket clearly states that the management of the
amusement park will not be held liable for any negligence on its part. As the ride comes to a stop, the
boat jerks due to a mechanical fault and Isaac bumps his head against the seat rendering him
unconscious. When Isaac awakens, he decides to institute an action for damages against the
management of the amusement park. In order to exclude or limit liability, the management may rely
on:

1. Consent to the risk of injury


2. Contributory negligence
3. Official capacity
4. None of the above

20 In which of the following cases did the court award a young boy compensation for emotional shock he
suffered as a result of witnessing his younger brother being injured in a motor vehicle accident?

1. Jones v Santam Bpk 1965 (2) SA 542 (A)


2. Bester v Commercial Union Versekeringsmaatskappy van SA Bpk 1973 (1) SA 769 (A)
3. Barnard v Santambank 1999 (1) SA 202 (SCA)
4. Weber v Sanatm Verskeringsmaatskappy Bpk 1983 (1) SA 381 (A)

Section B:

Question 1:

Mpho, a 16 year-old boy, suffered an epileptic fit while batting during a school cricket match. The
cricket ball had flung from his hand, into the air and struck a fellow player, Kwena on the head.
Kwena sustained a head injury and fell unconscious.

1.1 Did Mpho act for the purpose of the law of delict? Would it make a difference to your answer if
Mpho had already been under medical treatment for the epileptic fits he suffers, but failed to take
his prescribed medication when he should have done so earlier that morning? Discuss in detail with
reference to authority. Restrict the scope of your answer to what is asked in the question, and note
in particular that the question does not deal with the element of wrongfulness. (10)

Conduct is defined as a voluntary human act or omission. “Voluntary” means that the person must be
able to control his muscular movements by means of his will. The act of the wrongdoer must be
voluntary to give rise to delictual liability. By raising the defence of automatism, a defendant attempts
to show that, according to the law, he didn’t act. Defendant may argue that the conduct complained of
doesn’t satisfy the requirement of voluntariness. He relies on the defence of automatism - that he
acted mechanically. Conditions that may cause a person to act involuntarily as they render him
incapable of controlling his bodily movements include an epileptic fit. According to Molefe v Mahaeng,
the defendant does not bear the onus to prove that he was in a state of so-called sane automatism.
The onus is on the plaintiff to prove that the defendant acted voluntarily.

If we apply these principles to the given facts, we can conclude that Mpho did not act voluntarily when
the cricket ball struck Kwena on the head causing the head injury. However, the situation will indeed
change if Mpho had been receiving medical treatment for diagnosed epilepsy, but failed to take his
medication on that particular occasion. A person can’t rely on automatism if he intentionally placed
himself in a mechanical state (actio libera in causa). Actio libera in causa: Defence of automatism
won’t succeed if defendant intentionally created the situation in which he acts involuntarily in order to
harm another. The defendant (Mpho) will be held liable for his culpable conduct in creating the state
of automatism which resulted in damage to the plaintiff (Kwena).

Defendant may not successfully rely on the defence of automatism where he was negligent regarding
his automatic “conduct”. Where the reasonable man would’ve foreseen the possibility of causing harm
while in a state of automatism, eg. in Victor case, X was convicted of negligent driving despite causing
the accident during an epileptic fit, as he’d been suffering fits for 13 years and the reasonable man
would’ve foreseen the possibility of causing harm while in a state of automatism. In this case, X knew
he may suffer an epileptic fit and still drove a motor vehicle.

Automatism doesn’t mean that there’s no voluntary act whatsoever by the defendant which caused
the damage, but only that the conduct in question wasn’t voluntary. Only the voluntary act closest to
the harmful consequence is of relevance, and it’s therefore unnecessary to consider prior voluntary
acts.

A person can’t rely on automatism if he negligently placed him in a mechanical state (Mpho didn’t take
his medication). Thus, Mpho was probably negligent, or could even have had intention in the form of
dolus eventualis. Therefore, a reliance on automatism would fail in this case.

1.2 Kwena’s father instituted a delictual claim against Mpho as well as the school where the match took
place. The defence of volenti non fit iniuria as a ground of justification was raised. Assume that
Mpho did act for the purposes of the law of delict. Discuss the likelihood of this defence succeeding.
Restrict the scope of your answer to what is asked in the question, and note in particular that the
question does not deal with the element of fault. (10)

The principle that a defendant isn’t liable where the injured person has consented to injury or the risk
thereof, is embodied in the maxim volenti non fit iniuria (a willing person isn’t wronged; he who
consents can’t be injured). The maxim volenti non fit iniuria, in short volenti, is used to describe both
forms of consent. Volenti non fit iniuria may thus mean either consent to injury, or consent to the risk
of injury.

In the given facts, consent to the risk of injury applies. In the case of consent to the risk of injury, the
injured party consents to the risk of harm caused by the defendant’s conduct. Eg. a participant in sport
consents to the risks involved in such sport, like a cricket ball striking a cricket player in the head.
Should the risk contained in the sports injuries above ensue, the injured person won’t be able to hold
defendant delictually liable, because he has consented to risk of such harm.

In the given facts, the defence of volenti non fit iniuria as a ground of justification will probably
succeed as Mpho can state that Kwena consented to the risk of injury and the injured person (Kwena)
won’t be able to hold defendant (Mpho) delictually liable, because he (Kwena) has consented to risk of
such harm.
Question 2:

2.1 “Although the test for negligence is in principle uniform for all defendants, the test is adapted in the
case of children”. Discuss this statement with reference to relevant case law. (10)

The question whether wrongdoer is a child should play a role in application of reasonable person test.
This only arises in case of children 10 years and older, because law deems a child under 10 to be
culpae incapax.

Before 1965, courts took into account youthfulness of wrongdoer in determining his negligence; ie. to
test for negligence by means of conduct expected from a reasonable child of the wrongdoer’s age and
intellectual development.

In 1965, the AD passed judgment on this matter in Jones NO v Santam Bpk. The court adopted a new
approach to determination of negligence in respect of children. Court implied that the criterion for the
determination of negligence is always objective, in all situations the test of the reasonable person
(diligens paterfamilias) is applied. According to this approach, in determining whether a child acted
with culpa, it must first be ascertained whether child concerned met standard of care required of
reasonable person. Secondly, must be asked whether child, if care shown didn’t meet requirements of
first test, was culpae capax (accountable for his actions).

To determine whether a child acted negligently, question is whether conduct of child measures up to
the standard of care of the reasonable person. Once it’s established that conduct is negligent, it must
be ascertained whether negligent conduct may be imputed to the wrongdoer - whether he’s juridically
responsible for his acts. Here one’s dealing with accountability, and inquiry is whether child-
wrongdoer had the required intellect, maturity, experience, insight, etc to distinguish between right
and wrong and to act in accordance with such insight. All the subjective qualities of the child itself are
taken into account.

This (new) approach has had a critical reception. Firstly, it seems that the earlier test of reasonable
child is more acceptable than that of reasonable person, because a child, even though he may be fully
accountable, can’t be measured against an adult standard. Secondly, court in Jones placed the cart
before the horse by first inquiring into fault and then into accountability. Must first determine whether
a person is accountable before any question of fault. In Roxa v Mtshayi, the AD correctly accepted this
criticism by first testing for accountability and then for negligence.

In Weber v Santam Versekeringsmaatskappy Bpk, the court held that the Jones case didn’t materially
depart from common law and confirmed the approach in this case. Jansen JA substantiated his
judgment as follows:
When the child’s conduct is judged according to the criterion of an adult, the enquiry must be whether he was
mature enough to comply with that criterion in respect of the specific situation. If this approach is applied with
insight, many of the objections of our contemporary writers to the principles of the Jones case should fall away,
and it should also not be necessary to plead for the use of a reasonable child of relevant age, as in the Anglo-
American legal systems, instead of the criterion of the bonus paterfamilias to reach a fair result.

In Eskom Holdings Ltd v Hendricks, the SCA had to evaluate the alleged contributory negligence of a
child of 11 years of age. Court took cognisance of criticism of reasonable person test for children, but
referred to the approach in the above cases and reiterated that in “each case what had to be
determined was whether the child in question had developed the emotional and intellectual maturity
to appreciate the particular danger to be avoided and, if so, to act accordingly”.

2.2 Discuss the effect of a successful reliance on contributory fault in the following instances:
(a) the plaintiff has intent and the defendant is negligent (4)

The question is whether contributory fault applies in the case of defence of contributory intent. In the
situation where a plaintiff intentionally contributed towards his own loss while defendant was merely
negligent, the plaintiff forfeits his claim.

(b) both plaintiff and defendant are negligent (6)

The method of determining who should bear what portion of the damage involves a comparison of the
respective degrees of negligence of the parties involved. Each party’s degree of negligence is
determined by expressing its deviation from the standard of reasonable person as a %; the 2
percentages are then compared to allocate responsibility of the damage.

Prior to decision in Jones NO v Santam Bpk, the AD accepted that once plaintiff’s degree of negligence
had been established, it was unnecessary to inquire into extent to which defendant’s conduct had
deviated from standard of reasonable person. Eg. if court had established that plaintiff had been 40%
negligent (his conduct deviated 40% from standard of reasonable person), it was thought to follow
automatically that defendant was 60% negligent. However, in the Jones case a new approach to
determining degree of fault shown by plaintiff and defendant was followed. According to this decision,
the fact that plaintiff was, eg. 30% negligent, doesn’t automatically imply that defendant was 70%
negligent. To establish respective degrees of negligence, carefulness of conduct of each party must be
measured separately against standard of reasonable person. Eg. It’s possible that plaintiff’s conduct
deviated 70% while defendant’s conduct deviated 80%. In this case, the ratio between plaintiff’s and
defendant’s degree of fault is 70:80 (7:8 (15)). Plaintiff’s degree of fault is thus 7/15 × 100/1 = 46,7%,
and defendant’s 8/15 × 100/1 = 53,3% (or 100 – 46,7 = 53,3). Plaintiff thus receives compensation for
only 53,3% of damage he has suffered because he is 46,7% to blame for his loss.

If the defendant has been negligent but the plaintiff appears to have been negligent too, we must
consider whether contributory negligence was present. Contributory negligence is negligence on the
part of the plaintiff, and it’s a defence that the defendant can raise. The Apportionment of Damages
Act 34 of 1956 provides that a contributorily negligent plaintiff’s damages be apportioned. Court will
determine the degree of deviation from the reasonable person standard shown by the conduct of both
the defendant and the plaintiff, express the deviation as percentages, and use these percentages as a
basis for the apportionment. According to the Smit 1962 and Nomeka 1976 cases, the percentages of
negligence attributed to the defendant and plaintiff respectively will always add up to a hundred
percent.

According to Jones NO v Santam Bpk 1965, both percentages must be assessed independently, which
could mean that, for example, a defendant may be 80% negligent while the plaintiff is 30% negligent.
According to Neethling and Potgieter, the approach in Jones is to be preferred, but the 2 approaches
can be reconciled. According to King v Pearl Insurance Co Ltd 1970, a defence of contributory
negligence could not succeed where the plaintiff had omitted to wear a crash-helmet while driving a
scooter, but had not been negligent in respect of causing the accident.

However, in Bowkers Park Komga Cooperative Ltd v SAR and H 1980, the court held that contributory
negligence didn’t refer to negligence in respect of the damage-causing event, such as a motorcar
accident, but to negligence in respect of the damage itself, and this was confirmed by the AD in Union
National South British Insurance Co Ltd v Vitoria 1982 and General Accident Versekeringsmaatskappy
SA Bpk v Uijs 1993. Therefore, failure to wear a safety helmet would constitute contributory
negligence if it contributed to the plaintiff’s damage.
Question 3:

In his hurry to catch a train, Aaron bumps into Martha, a frail old lady. Martha falls and breaks a leg.
She is admitted to a hospital. The leg is set in plaster cast, she is given a set of crutches, and is
discharged from the hospital. A week later Martha slips with her crutches on a smooth floor, falls
again, and breaks her arm.

3.1 Does a factual causal link exist between Aaron’s conduct and Martha’s broken arm? Discuss. (5)

The generally accepted test for factual causation is the conditio sine qua non test, or ‘‘but for test’’.
This entails mentally eliminating, or thinking away, the conduct. If the damage then also disappears, a
factual causal link is present between the conduct and the damage. This test is subject to much
criticism. Among others, it is said to be based on circular logic and is, at best, a way to express the
existence of a causal nexus that has been determined in another way. Neethling and Potgieter argue
that evidence and human experience are sufficient to determine whether one fact flowed from
another fact, and that a so-called test of factual causation is superfluous. However, the courts
consistently state that the conditio sine qua non is the test of factual causation. If we apply the test to
the facts, we must conclude that if Aaron had not bumped Martha, she would not have broken her
arm, and therefore a factual causal link is present between Aaron’s conduct and Martha’s damage.

3.2 Does a legal causal link exist between Aaron’s conduct and Martha’s broken arm? Discuss in detail
with reference to case law. (10)

The test for legal causation is the so-called flexible approach, as formulated in S v Mokgethi 1990 and
International Shipping Co (Pty) Ltd v Bentley 1990. In Mokgethi, a bank robber shot a teller. The teller
was rendered a paraplegic and was discharged from hospital in a wheelchair. Subsequently, the
paraplegic man failed to shift his body position in the chair frequently and developed pressure sores,
eventually dying from complications. The question that arose was whether the shot fired by the
robber was the legal cause of the teller’s death. According to the court, the main question in respect of
legal causation is whether there is a close enough relationship between the wrongdoer’s conduct and
its consequence for such consequence to be imputed to the wrongdoer in view of policy
considerations based on reasonableness, fairness and justice. Several other legal causation theories
exist, such as adequate causation, direct consequences, foreseeability and novus actus interveniens.
None of these criteria is suitable to be applied to all situations. They may, however, be used as
subsidiary aids when employing the flexible approach. In the Mokgethi case, the court held that the
shot was not a legal cause of the death. If these principles are applied to the facts in the question, the
conclusion is probably that Martha’s broken arm was too remote and should not be imputed to the
wrongdoer. It could also be argued that a so-called novus actus interveniens, that is, a new intervening
act, was constituted by Martha’s second fall, and this strengthens the conclusion that there is no legal
causal link between Aaron’s conduct and Martha’s broken arm.

3.3 What is meant by the “sum-formula approach”? (2)

The sum-formula approach is a comparative method whereby patrimonial loss is established. The sum-
formula approach entails the comparison of an actual current patrimonial sum with a hypothetical
current patrimonial sum (the person’s current patrimonial position after the event, and his
hypothetical patrimonial position that would have been the current position if the event had not taken
place).

3.4 What is meant by the “once-and-for-all rule”? (2)

In terms of this rule, a plaintiff who claims damages on a specific cause of action has only 1 chance to
claim damages for all damage already suffered as well as all prospective loss.

3.5 What is vicarious liability? Explain briefly. (1)


Vicarious liability may be described as the strict liability of 1 person for the delict of another. The
former is thus indirectly or vicariously liable for the damage caused by the latter. This liability applies
where there is a particular relationship between 2 persons.

May/June 2018
1 Which one of the following is NOT described as one of the pillars of our law of delict?

1. The actio legis Aquiliae


2. The actio iniuriarum
3. The action for pain and suffering
4. The interdict

2 Janice threatens Queeneth with a gun and orders her to hand over her cellphone. Queeneth, in fear of
her life, hands over the cellphone. One week later, Queeneth spots Janice at a nearby shopping centre.
Queeneth rushes home and grabs a knife. She returns to the shopping centre and stabs Janice. Janice
wants to institute a delictual action against Queeneth. Queeneth may rely on the following ground of
justification:

1. Necessity
2. Provocation
3. Private defence
4. None of the above

3 Sibusiso is away on holiday. Frank, his neighbour, is keeping a watchful eye on his house for him. Frank
notices that the house is flooded with water and realises that a water pipe in the house has burst.
Frank breaks down the front door of Sibusiso’s house in order to turn off the water to prevent further
damage. If Sibusiso wants to institute a delictual action against Frank for breaking down his front door,
Frank may rely on the following ground of justification:

1. Statutory authority
2. Necessity
3. Provocation
4. Private defence

4 Which one of the following conditions may amount to automatism?

1. Provocation
2. Self-defence
3. Necessity
4. Hypnosis

5 The most suitable remedy for the prevention of harm is:

1. The actio legis Aquiliae


2. The actio de effuses vel deiectis
3. The interdict
4. Mitigation of loss
6 Choose the correct statement. In Carmichele v Minister of Safety and Security (Centre for Applied Legal
Studies Intervening) 2001 (4) SA 938 (CC), the Constitutional Court:

1. recognised a claim for Constitutional damages.


2. noted that the law of delict admirably reflected the spirit, purport and object of the Bill of
Rights.
3. declared that the courts of lower instance had erred by applying a pre-constitutional
concept of the boni mores.
4. recognised the concept of a Constitutional delict.

7 Absolute compulsion is one of the conditions that may cause a person’s behaviour to be involuntary
and hence not to qualify as conduct for the purpose of delictual liability. In which one of the following
instances is Y subject to absolute compulsion?

1. X points a gun at Y and orders Y to damage Z’s motor vehicle. In order to save his own life, Y
dents Z’s motor vehicle.
2. X pushes a baseball bat into Y’s hand and then, without Y being able to offer
resistance, X takes hold of Y’s hand and forces it to cause a dent on Z’s motor vehicle.
3. Y suffers an epileptic fit while driving his motor vehicle and in the process smashes into Z’s
motor vehicle.
4. Y is “blind drunk” and lies on the sidewalk, making involuntary movements with his arms and
legs. Z’s motor vehicle is parked next to Y and Y’s involuntary movements cause a dent in Z’s
motor vehicle.

8 Which of the following best describes what is meant by “psychological lesion”?

1. Pure economic loss


2. Negligent misrepresentation
3. Emotional shock
4. Infringement of the right to identity

9 In which one of the following situations can it be said that publication of defamatory words has taken
place?

1. Two German tourists visit South Africa. They start arguing in German in front of some South
Africans (who do not understand German) and the one tourist calls the other a liar and an
adulterer.
2. Mr Xolani tells his wife, Mrs Xolani, that Vanessa at his office is having an affair with her
secretary.
3. Mrs Brown tells her husband, Mr Brown, that Theresa, a work colleague of Mrs Brown, has
stolen money from the cash register on numerous occasions.
4. Bob meets Joseph at the local post office and tells Joseph that the manager of the post
office, Mrs Posh, is involved in an adulterous affair with Joseph’s neighbour Henry.

10 Dr Lucas, a well-known obstetrician, delivered Nicole’s daughter, Mandy. During delivery, the nerves in
Mandy’s right shoulder were injured resulting in the paralysis of her arm. All of this happened because
Dr Lucas failed to inform Nicole of the potential complications inherent in delivering a large baby.
Indicate the most correct statement: Negligence will be determined according to the standard of:

1. the reasonable doctor.


2. the reasonable person.
3. the reasonable man.
4. the reasonable child.

11 Carl, 17, left his Playstation in his locker at school. James, 15, wants to steal Carl’s Playstation. He
realises that he would have to damage Carl’s locker in order to get the Playstation. In respect of the
damage to the locker, James has:

1. Dolus directus
2. Dolus indirectus
3. Dolus eventualis
4. None of the above

12 Which one of the following qualifies as private defence?

1. John’s vicious dog charges to attack Zain. To protect himself, Zain picks up a baseball bat and
gives the dog a blow to the head.
2. Brian threatens Wilma with a knife and robs her of her cellphone. After 3 months, Wilma sees
Brian at a shopping centre and reacts by stabbing Brian with a high heeled shoe.
3. A child points a firearm at you and you grab his arm to prevent him from shooting you.
The child sustains injuries to his arm.
4. A police officer arrests Liam in the execution of a legitimate warrant of arrest. Liam resists the
arrest. He jabs the policeman, cracking one of the policeman’s ribs in an attempt to escape.

13 Gawie and Manie are farming on adjacent properties. Gawie bears a grudge against Manie. He sets up
a device, which causes a loud bang every half an hour, on the common boundary between the 2 farms
near Manie’s homestead. As a result of this, Manie and his family cannot sleep at night and are
irritated by day. Manie asks Gawie to dismantle the device, but Gawie says that the device serves the
purpose of scaring baboons away from his orchards. Which delictual remedy may be available to
Manie?

1. The actio de pauperie


2. The actio de feris
3. The actio de effuses vel deiectis
4. The interdict

14 Neil is chased by a fierce Jersey bull and jumps into Conrad’s scooter, which is parked next to the road,
in order to race away and save his own life. The scooter is damaged and Conrad institutes a delictual
claim against Neil. Neil may raise the following ground of justification:

1. Private defence
2. Necessity
3. Provocation
4. None of the above

15 Dick is so offended by William’s speech at a political rally, that he throws an apple at William. William
falls off the podium and sustains a broken arm as well as a few broken ribs. He is admitted to hospital.
While he is being treated there, a nurse leaves the windows of the ward open and William contracts
pneumonia. Which one of the following is incorrect?
1. Dick acted wrongfully
2. Dick had fault
3. There is a factual causal link between Dick’s conduct and William’s broken arm and ribs
4. There is no factual causal link between Dick’s conduct and William’s pneumonia

16 Greater Johannesburg Transitional Metropolitan Council v ABSA Bank Ltd 1997 (2) SA 591 (W) dealt
with:

1. Consent to injury
2. Consent to the risk of injury
3. Contributory intention
4. Contributory negligence

17 Which is the odd one out?

1. The adequate causation theory


2. The sum-formula approach
3. Direct consequences theory
4. Normative foreseeability

18 Joseph plays tennis in his back yard. He foresees the possibility of his ball breaking a window in his
neighbour’s house, but decides that it will not happen. If the ball indeed breaks the window, Joseph
had the following in respect of the damage:

1. dolus indeterminatus.
2. dolus eventualis.
3. dolus indirectus.
4. luxuria.

19 The principle dictating at what stage prospective loss must be claimed, is known as:

1. The sum-formula approach


2. The “once and for all” rule
3. Compensating advantages
4. The concrete approach to damage

20 John and Peter bear a grudge against Greg and agree to teach him a lesson he will not forget. While
Greg is in a shopping centre, John and Peter damage Greg’s motor car by hitting it with hammers. If
Greg successfully sues John and Peter in delict, they will incur:

1. vicarious liability.
2. joint and several liability.
3. liability based on contributory intent.
4. strict liability.
Section B:

Question 1:

The municipality of Cape Town is busy with excavations in the business centre. The municipal
workers do not erect any barrier or warning signs in the vicinity of the excavations. Mrs Baloyi, who
is near-sighted, falls into the excavations and sustains serious injuries. She wishes to institute a
delictual action against the municipality. Was the conduct of the municipality wrongful? Discuss in
detail. (15)

As a general rule, a person does not act wrongfully for the purposes of the law of delict if he omits to
prevent harm to another person. Liability only follows if the omission was in fact wrongful, and this will
be the case only if a legal duty rested on the defendant to act positively to prevent harm from
occurring and he failed to comply with that duty. The question of whether such a duty existed is
answered with reference to the criterion of the legal convictions of the community and legal policy.

This question deals with the wrongfulness of an omission. The basic question to determine whether an
omission is wrongful is whether a legal duty to act was present and was breached. This is determined
with reference to the legal convictions of the community, or the boni mores. Factors which may serve
as indications that a legal duty rested on the defendant include: prior conduct (omissio per
commissionem); control of a dangerous object; rules of law; a special relationship between the parties;
particular office; contractual undertaking for the safety of a third party; and creating of an impression
that the interests of a third person will be protected. In the so-called municipality cases, prior conduct
was considered to be a prerequisite for the wrongfulness of an omission. Prior conduct refers to
positive conduct that created a new source of danger, preceding subsequent omission to protect
others from being harmed by this new source of danger. The classic case in this respect is Halliwell v
Johannesburg Municipal Council 1912. The view that prior conduct was a prerequisite for wrongfulness
of an omission was eroded in Silva’s Fishing Corporation (Pty) Ltd v Maweza 1957; Regal v African
Superslate (Pty) Ltd 1963; and Minister of Forestry v Quathlamba 1973. In Minister van Polisie v Ewels
1975, the court finally held that the existence of a legal duty is determined by the boni mores, and
whereas the presence of prior conduct is a strong indication of the presence of wrongfulness, it is not
a prerequisite thereof. Subsequent judgments, such as Cape Town Municipality v Bakkerud 2000,
confirmed that the principles formulated in Ewels were applicable to municipality cases. An interplay
of different factors may also indicate the presence of a legal duty. In Carmichele v Minister of Safety
and Security (Centre for Applied Legal Studies Intervening) 2001, the CC made it clear that the boni
mores must now be informed by the values underpinning the Bill of Rights in the Constitution. If these
principles are applied to the given facts, we can probably conclude that the omission of the
municipality was indeed wrongful.
Question 2:

2.1 David works at the control tower of the OR Tambo Airport. His job is to regulate the movements of
the aeroplanes landing and taking off. Due to a sudden sharp drop in David’s blood pressure, he
loses consciousness for five minutes. During this period of unconsciousness, two aeroplanes collide
because David did not give the pilots the correct instructions. Did David act for the purpose of the
law of delict?

Would it make a difference to your answer if David has already been under medical treatment for a
diagnosed condition of low blood pressure, but failed to take his prescribed blood pressure
medication when he should have done so earlier that morning? Discuss in detail with reference to
authority. Restrict the scope of your answer to what is asked in the question, and note in particular
that the question does not deal with the element of wrongfulness. (10)
Conduct is defined as a voluntary human act or omission. “Voluntary” means that the person must be
able to control his muscular movements by means of his will. The act of the wrongdoer must be
voluntary to give rise to delictual liability. By raising the defence of automatism, a defendant attempts
to show that, according to the law, he didn’t act. Defendant may argue that the conduct complained of
doesn’t satisfy the requirement of voluntariness. He relies on the defence of automatism - that he
acted mechanically. Conditions that may cause a person to act involuntarily as they render him
incapable of controlling his bodily movements: absolute compulsion (vis absoluta), sleep,
unconsciousness, fainting fit, epileptic fit, serious intoxication, blackout, reflex movements, strong
emotional pressure, mental disease, hypnosis, and a heart attack.

According to Molefe v Mahaeng, the defendant does not bear the onus to prove that he was in a state
of so-called sane automatism. The onus is on the plaintiff to prove that the defendant acted
voluntarily. In the Du Plessis case, X (72) was charged with negligent driving as he had injured a
pedestrian. He experienced a blackout due to low-blood pressure. He was found not guilty.

If we apply these principles to the given facts, we can conclude that David did not act voluntarily when
the two aeroplanes collide because David did not give the pilots the correct instructions as he was
unconscious. However, the situation will indeed change if David had been receiving medical treatment
for a diagnosed condition of low blood pressure, but failed to take his prescribed blood pressure
medication on that particular occasion. A person can’t rely on automatism if he intentionally placed
himself in a mechanical state (actio libera in causa). Actio libera in causa: Defence of automatism
won’t succeed if defendant intentionally created the situation in which he acts involuntarily in order to
harm another. The defendant (David) will be held liable for his culpable conduct in creating the state
of automatism which resulted in damage to the plaintiff. Defendant may not successfully rely on the
defence of automatism where he was negligent regarding his automatic “conduct”. Where the
reasonable man would’ve foreseen the possibility of causing harm while in a state of automatism, eg.
in Victor case, X was convicted of negligent driving despite causing the accident during an epileptic fit,
as he’d been suffering fits for 13 years and the reasonable man would’ve foreseen the possibility of
causing harm while in a state of automatism. In this case, X knew he may suffer an epileptic fit and still
drove a motor vehicle.

Automatism doesn’t mean that there’s no voluntary act whatsoever by the defendant which caused
the damage, but only that the conduct in question wasn’t voluntary. Only the voluntary act closest to
the harmful consequence is of relevance, and it’s therefore unnecessary to consider prior voluntary
acts.

A person can’t rely on automatism if he negligently placed him in a mechanical state (David didn’t take
his medication). Thus, David was probably negligent, or could even have had intention in the form of
dolus eventualis. Therefore, a reliance on automatism would fail in this case.

2.2 Joseph is an intelligent twelve year-old boy. His dream is to be a world-class cricket player one day.
While practicing in his parents’ back yard, he hits a ball with all his might. The ball shatters a big
window in the neighbour’s house. Discuss in detail whether Joseph was negligent. (15)

Before Jones NO v Santam Bpk 1965, the negligence of a child used to be determined with reference to
a reasonable child standard. In the Jones case, the court held that the test for negligence remains
objective, and the reasonable person test (the diligens paterfamilias test) must also be employed in
the case of a child wrongdoer. The youthfulness of the child wrongdoer is not specifically considered
here. However, during the inquiry into the accountability of the child, his or her youthfulness is taken
into account. The Jones case was criticised on 2 counts: firstly, many are of the opinion that a
reasonable adult standard for a child wrongdoer is unfair; secondly, the court put the cart before the
horse by testing for negligence first and, thereafter, for accountability. In Roxa v Mtshayi 1975, the
court followed the correct order. In Weber v Santam Versekeringsmaatskappy Bpk 1983, the Jones
case was confirmed in essence and the court said that if the principles were applied with insight, the
criticism would fall away. In Eskom Holdings Ltd v Hendricks 2005, the court reiterated that in each
case it must be determined whether the child has attained the emotional and intellectual maturity to
appreciate the danger to be avoided and to act accordingly. In respect of accountability, the Child
Justice Act 75 of 2008 provides that a child of 9 or younger is irrebuttably presumed to be culpae
incapax/not accountable, whereas a child over 9, but under 14, is rebuttably presumed to be culpae
incapax/not accountable. Whether Joseph in our question would be held to have been negligent
would depend on all the circumstances of the case. If he was intelligent and mature enough to be
accountable/culpae capax, he was probably negligent, because, taken at face value, his conduct
deviated from that of the reasonable person in the circumstances.

Question 3:

Celia is so offended by Nadia’s speech at a political rally, that she throws her shoe at Nadia. Nadia
falls off the podium and sustains a broken arm as well as a few ribs. She is admitted to hospital.
While she is being treated there, a nurse leaves the windows of the ward open and Nadia contracts
pneumonia.

3.1 Does a factual causal link exist between Celia’s conduct and Nadia’s pneumonia? Discuss. (5)

The generally accepted test for factual causation is the conditio sine qua non test, or ‘‘but for test’’.
This entails mentally eliminating, or thinking away, the conduct. If the damage then also disappears, a
factual causal link is present between the conduct and the damage. This test is subject to much
criticism. Among others, it is said to be based on circular logic and is, at best, a way to express the
existence of a causal nexus that has been determined in another way. Neethling and Potgieter argue
that evidence and human experience are sufficient to determine whether one fact flowed from
another fact, and that a so-called test of factual causation is superfluous. However, the courts
consistently state that the conditio sine qua non is the test of factual causation. If we apply the test to
the facts, we must conclude that if Celia had not throne her shoe at Nadia, she would not have broken
her arm and a few ribs, and therefore a factual causal link is present between Celia’s conduct and
Nadia’s damage.

3.2 Does a legal causal link exist between Celia’s conduct and Nadia’s pneumonia? Discuss in detail with
reference to case law. (15)

The test for legal causation is the so-called flexible approach, as formulated in S v Mokgethi 1990 and
International Shipping Co (Pty) Ltd v Bentley 1990. In Mokgethi, a bank robber shot a teller. The teller
was rendered a paraplegic and was discharged from hospital in a wheelchair. Subsequently, the
paraplegic man failed to shift his body position in the chair frequently and developed pressure sores,
eventually dying from complications. The question that arose was whether the shot fired by the
robber was the legal cause of the teller’s death. According to the court, the main question in respect of
legal causation is whether there is a close enough relationship between the wrongdoer’s conduct and
its consequence for such consequence to be imputed to the wrongdoer in view of policy
considerations based on reasonableness, fairness and justice. Several other legal causation theories
exist, such as adequate causation, direct consequences, foreseeability and novus actus interveniens.
None of these criteria is suitable to be applied to all situations. They may, however, be used as
subsidiary aids when employing the flexible approach. In the Mokgethi case, the court held that the
shot was not a legal cause of the death. If these principles are applied to the facts in the question, the
conclusion is probably that Nadia’s pneumonia was too remote and should not be imputed to the
wrongdoer (Celia). It could also be argued that a so-called novus actus interveniens (a new intervening
act) was constituted by a nurse leaving the windows of the ward open, and this strengthens the
conclusion that there is no legal causal link between Celia’s conduct and Nadia’s pneumonia.
Oct/Nov 2017:
1 Absolute compulsion is one of the conditions that may cause a person’s behaviour to be involuntary
and hence not to qualify as conduct for the purpose of delictual liability. In which one of the following
instances is Y subject to absolute compulsion?

1. X points a gun at Y and orders Y to damage Z’s motor vehicle. In order to save his own life, Y
dents Z’s motor vehicle.
2. X pushes a baseball bat into Y’s hand and then, without Y being able to offer
resistance, X takes hold of Y’s hand and forces it to cause a dent on Z’s motor vehicle.
3. Y suffers an epileptic fit while driving his motor vehicle and in the process smashes into Z’s
motor vehicle.
4. Y is “blind drunk” and lies on the sidewalk, making involuntary movements with his arms and
legs. Z’s motor vehicle is parked next to Y and Y’s involuntary movements cause a dent in Z’s
motor vehicle.

2 Arnold threatens Betty with a gun and orders Betty to hand over her handbag and cellphone. Betty, in
fear of her life, hands over the handbag and cellphone. One week later, Betty spots Arnold at a nearby
shopping centre. Betty rushes home and grabs a knife. She returns to the shopping centre and stabs
Arnold. Arnold wants to institute a delictual action against Betty. Betty may rely on the following
ground of justification:

1. Necessity
2. Provocation
3. Private defence
4. None of the above

3 Sibusiso is away on holiday. Mapule, his neighbour, is keeping a watchful eye on his house for him.
Mapule notices that the house is flooded with water and realises that a water pipe in the house has
burst. Mapule breaks down the front door of Sibusiso’s house in order to turn off the water to prevent
further damage. If Sibusiso wants to institute a delictual action against Mapule for breaking down his
front door, Mapule may rely on the following ground of justification:

1. Statutory authority
2. Necessity
3. Provocation
4. Private defence

4 Doctor Suliman is an obstetrician and Nadia is a pregnant patient of hers. Nadia is rushed to hospital
late at night as she experiences severe abdominal pains. Doctor Suliman realises that Nadia is in labour
and performs an emergency caesarean section. Nadia lays a complaint against Doctor Suliman with the
Health Professions Council. The council finds that the doctor acted as a reasonable obstetrician would
have done in the particular circumstances. Doctor Suliman wishes to sue Nadia for defamation. Nadia
may rely on the following ground of justification:

1. Relative privilege
2. Absolute privilege
3. Fair comment
4. None of the above
5 The most suitable remedy for the prevention of harm is:

1. The actio legis Aquiliae


2. The actio de effuses vel deiectis
3. The interdict
4. Mitigation of loss

6 Which statement is incorrect?

1. Only an act or omission that has been willed, can give rise to delictual liability
2. An act or an omission that is irrational or inexplicable may give rise to delictual liability
3. The behaviour of an animal can never qualify as conduct for the purpose of the law of delict
4. A company can act for the purpose of the law of delict

7 Dick is so offended by William’s speech at a political rally, that he throws an apple at William. William
falls off the podium and sustains a broken arm as well as a few broken ribs. He is admitted to hospital.
While he is being treated there, a nurse leaves the windows of the ward open and William contracts
pneumonia. Which one of the following is incorrect?

1. Dick acted wrongfully


2. Dick had fault
3. There is a factual causal link between Dick’s conduct and William’s broken arm and ribs
4. There is no factual causal link between Dick’s conduct and William’s pneumonia

8 Which one of the following statements is correct with regard to delictual liability?

1. The actio legis Aquiliae may be used as the primary remedy for delicts that cause injury to
personality
2. The actio iniuriarum may be used as the primary remedy for patrimonial loss
3. In South African law, delictual liability is governed by a generalising approach
4. In South African law, each delict has its own unique requirements.

9 In which one of the following situations can it be said that publication of defamatory words has taken
place?

1. Two German tourists visit South Africa. They start arguing in German in front of some South
Africans (who do not understand German) and the one tourist calls the other a liar and an
adulterer.
2. Mr Xolani tells his wife, Mrs Xolani, that Vanessa at his office is having an affair with her
secretary.
3. Mrs Brown tells her husband, Mr Brown, that Theresa, a work colleague of Mrs Brown, has
stolen money from the cash register on numerous occasions.
4. Bob meets Joseph at the local post office and tells Joseph that the manager of the post
office, Mrs Posh, is involved in an adulterous affair with Joseph’s neighbour Henry.

10 Nomsa’s two-year-old daughter falls down a flight of stairs. Nomsa believes that the child has
sustained a brain injury and rushes her to the hospital. On the way to the hospital, Nomsa drives over
the neighbour’s dog which subsequently dies. The neighbour institutes a delictual claim against
Nomsa. It later transpires that Nomsa’s daughter merely suffered from mild shock. Which one of the
following defences may Nomsa rely on?
1. Private defence
2. Provocation
3. Necessity
4. None of the above defences

11 Markus, a financial adviser, negligently advises Hans to make a bad investment. As a result of this,
Hans suffers a serious financial setback. However, there is no damage to Hans’s person or property.
Which one of the following delictual remedies may be available if Hans wishes to recover his financial
loss from Markus?

1. The actio legis Aquiliae


2. The actio iniuriarum
3. The action for pain and suffering
4. None of the above

12 Gawie and Manie are farming on adjacent properties. Gawie bears a grudge against Manie. He sets up
a device, which causes a loud bang every half an hour, on the common boundary between the 2 farms
near Manie’s homestead. As a result of this, Manie and his family cannot sleep at night and are
irritated by day. Manie asks Gawie to dismantle the device, but Gawie says that the device serves the
purpose of scaring baboons away from his orchards. Which delictual remedy may be available to
Manie?

1. The actio de pauperie


2. The actio de feris
3. The actio de effuses vel deiectis
4. The interdict

13 Which one of the following statements is correct?

1. Accountability is a prerequisite for fault


2. A person is accountable if he can differentiate between right and wrong and his behaviour is
susceptible to control of his will
3. Intoxication cannot influence accountability
4. A person is accountable if he directs his will at an unlawful result and is also conscious of
wrongfulness

14 Sean is employed by Raymond as a driver in Raymond’s courier business. Sean has the weekend off,
and decides to visit Mary on Saturday evening. On his way to Mary’s apartment, Sean is involved in an
accident due to his own negligence. Sean’s vehicle and the vehicle of Catherine, the other motorist
involved in the accident, are badly damaged. Catherine discovers that Sean’s financial position is not
good. Advise Catherine on the best course of action:

1. Institute an action against Raymond based on vicarious liability


2. Institute an action against Raymond and Sean as joint wrongdoers
3. Institute an action against Raymond based on culpa in eligendo
4. Institute an action against Sean

15 Greater Johannesburg Transitional Metropolitan Council v ABSA Bank Ltd 1997 (2) SA 591 (W) dealt
with:
1. Consent to injury
2. Consent to the risk of injury
3. Contributory intention
4. Contributory negligence

16 Jacob is training to be a tattoo artist. He asks Karen whether he can practice his newly acquired skills
on her. Karen enquires whether she will be able to remove the tattoo by washing, and Jacob replies in
the affirmative. Thereupon Karen is willing to comply, and Jacob makes a butterfly tattoo on Karen’s
arm. When Karen discovers that the tattoo is permanent, she wants to institute a delictual action
against Jacob. Jacob may rely on the following ground of justification:

1. Necessity
2. Official capacity
3. Consent
4. None of the above

17 Joseph plays tennis in his back yard. He foresees the possibility of his ball breaking a window in his
neighbour’s house, but decides that it will not happen. If the ball indeed breaks the window, Joseph
had the following in respect of the damage:

1. dolus indeterminatus.
2. dolus eventualis.
3. dolus indirectus.
4. luxuria.

18 Ben clambers over Neil’s fence with the aim of breaking into Neil’s house. Neil’s dog attacks Ben and
Ben sustains serious injuries. Ben wishes to recover his hospital expenses from Neil. Which remedy
may be available to Ben?

1. Actio de pauperie.
2. Actio de pastu.
3. Actio de feris.
4. None of the above

19 The principle dictating at what stage prospective loss must be claimed, is known as:

1. The sum-formula approach


2. The “once and for all” rule
3. Compensating advantages
4. The concrete approach to damage

20 John and Peter bear a grudge against Greg and agree to teach him a lesson he will not forget. While
Greg is in a shopping centre, John and Peter damage Greg’s motor car by hitting it with hammers. If
Greg successfully sues John and Peter in delict, they will incur:

1. vicarious liability.
2. joint and several liability.
3. liability based on contributory intent.
4. strict liability.
Section B:

Question 1:

The municipality of Tshwane is busy with excavations in the business centre. The municipal workers
do not erect any barrier or warning signs in the vicinity of the excavations. Mrs Ngonyama, who is
near-sighted, falls into the excavations and sustains serious injuries. She wishes to institute a
delictual action against the municipality. Was the conduct of the municipality wrongful? Discuss in
detail. (15)

As a general rule, a person does not act wrongfully for the purposes of the law of delict if he omits to
prevent harm to another person. Liability only follows if the omission was in fact wrongful, and this will
be the case only if a legal duty rested on the defendant to act positively to prevent harm from
occurring and he failed to comply with that duty. The question of whether such a duty existed is
answered with reference to the criterion of the legal convictions of the community and legal policy.

This question deals with the wrongfulness of an omission. The basic question to determine whether an
omission is wrongful is whether a legal duty to act was present and was breached. This is determined
with reference to the legal convictions of the community, or the boni mores. Factors which may serve
as indications that a legal duty rested on the defendant include: prior conduct (omissio per
commissionem); control of a dangerous object; rules of law; a special relationship between the parties;
particular office; contractual undertaking for the safety of a third party; and creating of an impression
that the interests of a third person will be protected. In the so-called municipality cases, prior conduct
was considered to be a prerequisite for the wrongfulness of an omission. Prior conduct refers to
positive conduct that created a new source of danger, preceding subsequent omission to protect
others from being harmed by this new source of danger. The classic case in this respect is Halliwell v
Johannesburg Municipal Council 1912. The view that prior conduct was a prerequisite for wrongfulness
of an omission was eroded in Silva’s Fishing Corporation (Pty) Ltd v Maweza 1957; Regal v African
Superslate (Pty) Ltd 1963; and Minister of Forestry v Quathlamba 1973. In Minister van Polisie v Ewels
1975, the court finally held that the existence of a legal duty is determined by the boni mores, and
whereas the presence of prior conduct is a strong indication of the presence of wrongfulness, it is not
a prerequisite thereof. Subsequent judgments, such as Cape Town Municipality v Bakkerud 2000,
confirmed that the principles formulated in Ewels were applicable to municipality cases. An interplay
of different factors may also indicate the presence of a legal duty. In Carmichele v Minister of Safety
and Security (Centre for Applied Legal Studies Intervening) 2001, the CC made it clear that the boni
mores must now be informed by the values underpinning the Bill of Rights in the Constitution. If these
principles are applied to the given facts, we can probably conclude that the omission of the
municipality was indeed wrongful.
Question 2:

2.1 Johnny is an intelligent thirteen year-old boy. His dream is to be a world-class soccer player one day.
While practicing in his parents’ back yard, he kicks a ball with all his might. The ball shatters a big
window in the neighbour’s house. Discuss in detail whether Johnny was negligent. (15)

Before Jones NO v Santam Bpk 1965, the negligence of a child used to be determined with reference to
a reasonable child standard. In the Jones case, the court held that the test for negligence remains
objective, and the reasonable person test (the diligens paterfamilias test) must also be employed in
the case of a child wrongdoer. The youthfulness of the child wrongdoer is not specifically considered
here. However, during the inquiry into the accountability of the child, his or her youthfulness is taken
into account. The Jones case was criticised on 2 counts: firstly, many are of the opinion that a
reasonable adult standard for a child wrongdoer is unfair; secondly, the court put the cart before the
horse by testing for negligence first and, thereafter, for accountability. In Roxa v Mtshayi 1975, the
court followed the correct order. In Weber v Santam Versekeringsmaatskappy Bpk 1983, the Jones
case was confirmed in essence and the court said that if the principles were applied with insight, the
criticism would fall away. In Eskom Holdings Ltd v Hendricks 2005, the court reiterated that in each
case it must be determined whether the child has attained the emotional and intellectual maturity to
appreciate the danger to be avoided and to act accordingly. In respect of accountability, the Child
Justice Act 75 of 2008 provides that a child of 9 or younger is irrebuttably presumed to be culpae
incapax/not accountable, whereas a child over 9, but under 14, is rebuttably presumed to be culpae
incapax/not accountable. Whether Johnny in our question would be held to have been negligent
would depend on all the circumstances of the case. If he was intelligent and mature enough to be
accountable/culpae capax, he was probably negligent, because, taken at face value, his conduct
deviated from that of the reasonable person in the circumstances.

2.2 The principal of the Bright Future High School has decided to have trees planted on a playground
where the energetic schoolboys play every day. To protect the young trees, circles of thin iron poles
are driven into the ground surrounding the plants. During the first break, Tim, one of the boys in
grade twelve, tries to sit on one of these poles. His friends warn him that the pole is too thin and has
a too sharp tip, but he nonetheless tries to sit on it. The pole penetrates his buttocks and causes a
serious injury. Tim’s parents institute a delictual action against the Bright Future High School for its
omission to guard against the poles injuring the schoolboys. If the management of the school
concedes that the omission was wrongful, what defence can the school nonetheless raise and what
will the effect of a successful reliance on that defence be? Discuss in detail. (15)

From the given facts, we can conclude that the defendant has been negligent, but the plaintiff appears
to have been negligent too. Thus we must consider whether contributory negligence was present.
Contributory negligence is negligence on the part of the plaintiff, and it is a defence that the defendant
can raise. The Apportionment of Damages Act 34 of 1956 is applicable. This Act provides that a
contributorily negligent plaintiff’s damages be apportioned. The court will determine the degree of
deviation from the reasonable person standard shown by the conduct of both the defendant and the
plaintiff, express the deviation as percentages, and use these percentages as a basis for the
apportionment. According to the Smit 1962 and Nomeka 1976 cases, the percentages of negligence
attributed to the defendant and plaintiff respectively will always add up to a 100%.

According to Jones NO v Santam Bpk 1965, both percentages must be assessed independently, which
could mean that, for example, a defendant may be 80% negligent while the plaintiff is 30% negligent.
According to Neethling and Potgieter, the approach in Jones is to be preferred, but the 2 approaches
can be reconciled. According to King v Pearl Insurance Co Ltd 1970, a defence of contributory
negligence could not succeed where the plaintiff had omitted to wear a crash-helmet while driving a
scooter, but had not been negligent in respect of causing the accident.

However, in Bowkers Park Komga Cooperative Ltd v SAR and H 1980, the court held that contributory
negligence didn’t refer to negligence in respect of the damage-causing event, such as a motorcar
accident, but to negligence in respect of the damage itself, and this was confirmed by the AD in Union
National South British Insurance Co Ltd v Vitoria 1982 and General Accident Versekeringsmaatskappy
SA Bpk v Uijs 1993. Therefore, failure to wear a safety helmet would constitute contributory
negligence if it contributed to the plaintiff’s damage.

Question 3:

Dick is so offended by William’s speech at a political rally, that he throws an apple at William.
William falls off the podium and sustains a broken arm as well as a few broken ribs. He is admitted
to hospital. While he is being treated there, a nurse leaves the windows of the ward open and
William contracts pneumonia. Was there a legal causal link between Dick’s conduct and William’s
pneumonia? Discuss in detail. (15)

The test for legal causation is the so-called flexible approach, as formulated in S v Mokgethi 1990 and
International Shipping Co (Pty) Ltd v Bentley 1990. In Mokgethi, a bank robber shot a teller. The teller
was rendered a paraplegic and was discharged from hospital in a wheelchair. Subsequently, the
paraplegic man failed to shift his body position in the chair frequently and developed pressure sores,
eventually dying from complications. The question that arose was whether the shot fired by the
robber was the legal cause of the teller’s death. According to the court, the main question in respect of
legal causation is whether there is a close enough relationship between the wrongdoer’s conduct and
its consequence for such consequence to be imputed to the wrongdoer in view of policy
considerations based on reasonableness, fairness and justice. Several other legal causation theories
exist, such as adequate causation, direct consequences, foreseeability and novus actus interveniens.
None of these criteria is suitable to be applied to all situations. They may, however, be used as
subsidiary aids when employing the flexible approach. In the Mokgethi case, the court held that the
shot was not a legal cause of the death. If these principles are applied to the facts in the question, the
conclusion is probably that William’s pneumonia was too remote and should not be imputed to the
wrongdoer. It could also be argued that a so-called novus actus interveniens (a new intervening act)
was constituted by a nurse leaving the windows of the ward open, and this strengthens the conclusion
that there is no legal causal link between Dick’s conduct and William’s pneumonia.

May/June 2017 – First paper


1 If patrimonial loss is caused in a negligent manner, the most appropriate delictual remedy to consider
is:

1. The actio legis Aquiliae


2. The actio iniuriarum
3. The action for pain and suffering
4. None of the above

2 If bodily injury is caused in a negligent manner, the most appropriate delictual remedy to consider is:

1. The actio legis Aquiliae


2. The actio iniuriarum
3. The action for pain and suffering
4. None of the above

3 The most suitable remedy for the prevention of harm is:

1. The actio legis Aquiliae


2. The actio de effuses vel deiectis
3. The interdict
4. Mitigation of loss

4 Kabelo’s prize bull breaks through a fence and ventures into Tino’s farm. The bull is aggressive and
charges at Tino’s employees. The employees clamber into a small tree to escape the wrath of the bull.
The bull begins to bash the trunk of the tree. As the tree appears to be about to topple, Tino shoots
and kills the bull to protect his employees. Kabelo institutes an action for damages against Tino. Tino
may raise the following ground of justification:

1. Necessity
2. Private defence
3. Provocation
4. Official capacity

5 Frans, a financial adviser, negligently advises Ngidi to make a bad investment. As a result of this, Ngidi
suffers a serious financial setback. However, there is no damage to Ngidi’s person or property. Which
one of the following delictual remedies may be available if Ngidi wishes to recover his financial loss
from Frans?

1. The actio legis Aquiliae


2. The actio iniuriarum
3. The action for pain and suffering
4. None of the above

6 Gregory and Mabula are farming on adjacent properties. Gregory bears a grudge against Mabula. He
sets up a device, which causes a loud bang every half an hour, on the common boundary between the
2 farms near Mabula’s homestead. As a result of this, Mabula and his family cannot sleep at night and
are irritated by day. Mabula asks Gregory to dismantle the device, but Gregory says that the device
serves the purpose of scaring baboons away from his orchards. Which delictual remedy may be
available to Mabula?

1. The actio de pauperie


2. The actio de feris
3. The actio de effuses vel deiectis
4. The interdict

7 Which one of the following statements is correct?

1. Accountability is a prerequisite for fault


2. A person is accountable if he can differentiate between right and wrong and his behaviour is
susceptible to control of his will
3. Intoxication cannot influence accountability
4. A person is accountable if he directs his will at an unlawful result and is also conscious of
wrongfulness
8 Max is training to be a tattoo artist. He asks Kanyiso whether he can practice his newly acquired skills
on her. Kanyiso enquires whether she will be able to remove the tattoo by washing, and Max replies in
the affirmative. Thereupon Kanyiso is willing to comply, and Max makes a butterfly tattoo on Kanyiso’s
arm. When Kanyiso discovers that the tattoo is permanent, she wants to institute a delictual action
against Max. Max may rely on the following ground of justification:

1. Necessity
2. Official capacity
3. Consent
4. None of the above
9 Betty plans to visit Kate, who is looking after Anne’s house. Kate mentions to Betty that there is a
vicious dog, named Roxy, on the premises, but promises to have Roxy locked up in the garage before
Betty arrives. Betty arrives at the appointed time, but on entering the premises, Roxy comes running
around the corner of the house and bites Betty’s leg. What action can Betty institute against Kate in
order to recover her resulting hospital costs?

1. Actio de pastu
2. Actio iniuriarum
3. Actio de pauperie
4. Actio legis Aquiliae

10 James, who is employed by Zando, delivers a parcel to Melissa’s house. Melissa’s dog, Snoopy, bites
James when he enters the property. What action may be available to James against Melissa?

1. Actio de pastu
2. Actio iniuriarum
3. Actio de pauperie
4. Actio legis Aquiliae

11 Precious has a stall on the sidewalk where she sells homemade cakes. While Precious is tending to
customers, Donald and Jacob pass by and steal an entire tray full of cakes. While they run away,
Jacob’s hat falls off and Precious recognises him and reports the matter to the police. The police
manage to find Jacob, but only after he and Donald have consumed all the cakes. Precious wants to
claim damages from Jacob. Can she claim the entire amount of damages from Jacob?

1. Yes, Jacob is vicariously liable for the damage


2. Yes, joint wrongdoers are in solidum liable for the full damage
3. No, she can only claim damage once the identity of both the perpetrators is known and then
she must claim from both of them simultaneously
4. No, Jacob and Donald are liable for the damage in equal proportions and hence she can claim
only half the amount of damages from Jacob

12 Zayn and Shiraz play in their backyard with a gas-activated pellet gun. They foresee that a pellet may
break a window in one of the neighbour’s houses or vehicles, but decide that it will not happen. If a
pellet indeed breaks the neighbour’s window, Zayn and Shiraz had the following in respect ofdamage:

1. Dolus indeterminatus
2. Dolus eventualis
3. Dolus indirectus
4. Luxuria

13 Ayanda insults Julia in front of her mother-in-law, Lerato. Lerato is very protective over her new
daughter-in-law and in retaliation to the insult, slaps Ayanda in the face. If Ayanda institutes a delictual
action against Lerato, Lerato may rely on the following defence:

1. Necessity
2. Private defence
3. Provocation
4. None of the above
14 In which one of the following cases did the court hold that an apportionment of damages could
succeed where both the plaintiff and the defendant acted with intention?

1. Greater Johannesburg Transitional Metropolitan Council v ABSA Bank 1997 2 SA 691 (W)
2. Wapnick v Durban City Garage 1984 2 SA 414 (D)
3. Lampert v Hefer 1955 2 SA 507 (A)
4. Netherlands Insurance Co of SA v Van der Vyver 1968 1 SA 412 (A)

15 X decides to test his home-made bomb in a deserted office building. A group of rebellious teenagers
enter the building and spray graffiti on the walls. The bomb is detonated while the teenagers are
inside the building and five of them are injured. The bomb also causes significant damage to the office
building. In respect of the injuries to the teenagers, the most likely form of fault that X can have is:

1. Dolus indirectus
2. Dolus directus
3. Dolus indeterminatus
4. Luxuria

16 Lucy is walking in a popular fashion store and she sees a designer handbag in a glass display unit. She
immediately decides to steal the handbag. In order to get to the handbag, she forces open the glass
display unit, damaging it significantly. In respect of the damage done to the glass display unit, Lucyhas:

1. Dolus indirectus
2. Dolus eventualis
3. Dolus indeterminatus
4. Luxuria

17 X wrongfully and negligently knocks over a ladder on which Y was standing. Y breaks an arm and a leg
and also sustains a back injury. Y will be unable to work until his arm and leg are healed. It is uncertain
how long the healing will take, and it is also unknown how much income he will lose. In addition, there
is a slight possibility that his back injury will prevent him from working for an even longer period of
time. Y wishes to claim delictual damages from X. which option is the most correct?

1. Y can only claim the hospital costs that have been incurred for the arm, leg and back injuries at
the stage that the claim is instituted
2. Y must claim for the hospital costs that have been incurred for the arm, leg and back injuries at
the stage that the claim is instituted, and can only claim for loss of income once he can work
again, when he will be able to quantify his loss of income
3. As soon as the first damage is evident, Y must claim all his damages, including all damage
that may materialise as a result of his fall from the ladder in the future
4. Y has no claim for damages

18 Indicate the correct statement with regard to the similarities and differences between delictual,
criminal and contractual liability:

1. A claim for damages is the primary remedy for delict


2. It is not possible for one and the same act to render the wrongdoer delictually as well as
contractually liable
3. One and the same act cannot found delictual as well as criminal liability
4. A claim for damages is the primary remedy in respect of a crime
19 Prospective loss is best assessed in accordance with:

1. the sum-formula approach


2. the “once and for all” rule
3. compensating advantages
4. mitigation

20 Ayesha tells Ketiwe that Rosemary, the wife of a famous singer, is involved in an extra-marital affair.
Ketiwe repeats the information to Xolani, a reporter. Xolani investigates and finds the information to
be true. He writes an article about this, and it is published in the local newspaper, The Know-It-All,
with knowledge of John, the editor. If Rosemary wishes to sue for defamation, which of the following
options is the most correct?

1. Xolani, The Know-It-All and John may incur liablity with the actio iniuriarum.
2. Ayesha, Xolani, The Know-It-All and John may incur liablity with the actio iniuriarum.
3. Ayesha, Ketiwe, Xolani, The Know-It-All and John may incur liablity with the actio iniuriarum.
4. No-one may incur liability with the actio iniuriarum.

Section B:

Question 1:

Jim works at the control tower of the Lanseria Airport. His job is to regulate the movements of the
aeroplanes landing and taking off. Due to a sudden sharp drop in Jim’s blood pressure, he loses
consciousness for five minutes. During this period of unconsciousness, two aeroplanes collide
because Jim did not give the pilots the correct instructions. Did David act for the purpose of the law
of delict? Would it make a difference to your answer if Jim has already been under medical
treatment for a diagnosed condition of low blood pressure, but failed to take his prescribed blood
pressure medication when he should have done so earlier that morning? Discuss in detail with
reference to authority. Restrict the scope of your answer to what is asked in the question, and note
in particular that the question does not deal with the element of wrongfulness. (10)

Conduct is defined as a voluntary human act or omission. “Voluntary” means that the person must be
able to control his muscular movements by means of his will. The act of the wrongdoer must be
voluntary to give rise to delictual liability. By raising the defence of automatism, a defendant attempts
to show that, according to the law, he didn’t act. Defendant may argue that the conduct complained of
doesn’t satisfy the requirement of voluntariness. He relies on the defence of automatism - that he
acted mechanically. Conditions that may cause a person to act involuntarily as they render him
incapable of controlling his bodily movements: absolute compulsion (vis absoluta), sleep,
unconsciousness, fainting fit, epileptic fit, serious intoxication, blackout, reflex movements, strong
emotional pressure, mental disease, hypnosis, and a heart attack.

According to Molefe v Mahaeng, the defendant does not bear the onus to prove that he was in a state
of so-called sane automatism. The onus is on the plaintiff to prove that the defendant acted
voluntarily. In the Du Plessis case, X (72) was charged with negligent driving as he had injured a
pedestrian. He experienced a blackout due to low-blood pressure. He was found not guilty.

If we apply these principles to the given facts, we can conclude that Jim did not act voluntarily when
the two aeroplanes collide because Jim did not give the pilots the correct instructions as he was
unconscious. However, the situation will indeed change if Jim had been receiving medical treatment
for a diagnosed condition of low blood pressure, but failed to take his prescribed blood pressure
medication on that particular occasion. A person can’t rely on automatism if he intentionally placed
himself in a mechanical state (actio libera in causa). Actio libera in causa: Defence of automatism
won’t succeed if defendant intentionally created the situation in which he acts involuntarily in order to
harm another. The defendant (Jim) will be held liable for his culpable conduct in creating the state of
automatism which resulted in damage to the plaintiff.

Defendant may not successfully rely on the defence of automatism where he was negligent regarding
his automatic “conduct”. Where the reasonable man would’ve foreseen the possibility of causing harm
while in a state of automatism, eg. in Victor case, X was convicted of negligent driving despite causing
the accident during an epileptic fit, as he’d been suffering fits for 13 years and the reasonable man
would’ve foreseen the possibility of causing harm while in a state of automatism. In this case, X knew
he may suffer an epileptic fit and still drove a motor vehicle.

Automatism doesn’t mean that there’s no voluntary act whatsoever by the defendant which caused
the damage, but only that the conduct in question wasn’t voluntary. Only the voluntary act closest to
the harmful consequence is of relevance, and it’s therefore unnecessary to consider prior voluntary
acts.

A person can’t rely on automatism if he negligently placed himself in a mechanical state (Jim didn’t
take his medication). Thus, Jim was probably negligent, or could even have had intention in the form
of dolus eventualis. Therefore, a reliance on automatism would fail in this case.

Question 2:

2.1 John works at the control tower of the Wonderboom Airport. His job is to regulate the movements
of the aeroplanes landing and taking off. Two aeroplanes collide because John failed to give the
pilots the correct instructions. Discuss only, but in detail and with reference to case law, whether
John’s conduct was delictually wrongful. (15)

As a general rule, a person does not act wrongfully for the purposes of the law of delict if he omits to
prevent harm to another person. Liability only follows if the omission was in fact wrongful, and this will
be the case only if a legal duty rested on the defendant to act positively to prevent harm from
occurring and he failed to comply with that duty. The question of whether such a duty existed is
answered with reference to the criterion of the legal convictions of the community and legal policy.

This question deals with the wrongfulness of an omission. The basic question to determine whether an
omission is wrongful is whether a legal duty to act was present and was breached. This is determined
with reference to the legal convictions of the community, or the boni mores. Factors which may serve
as indications that a legal duty rested on the defendant include: prior conduct (omissio per
commissionem); control of a dangerous object; rules of law; a special relationship between the parties;
particular office; contractual undertaking for the safety of a third party; and creating of an impression
that the interests of a third person will be protected. In the so-called municipality cases, prior conduct
was considered to be a prerequisite for the wrongfulness of an omission. Prior conduct refers to
positive conduct that created a new source of danger, preceding subsequent omission to protect
others from being harmed by this new source of danger. The classic case in this respect is Halliwell v
Johannesburg Municipal Council 1912. The view that prior conduct was a prerequisite for wrongfulness
of an omission was eroded in Silva’s Fishing

Corporation (Pty) Ltd v Maweza 1957; Regal v African Superslate (Pty) Ltd 1963; and Minister of
Forestry v Quathlamba 1973. In Minister van Polisie v Ewels 1975, the court finally held that the
existence of a legal duty is determined by the boni mores, and whereas the presence of prior conduct
is a strong indication of the presence of wrongfulness, it is not a prerequisite thereof. Subsequent
judgments, such as Cape Town Municipality v Bakkerud 2000, confirmed that the principles formulated
in Ewels were applicable to municipality cases. An interplay of different factors may also indicate the
presence of a legal duty. In Carmichele v Minister of Safety and Security (Centre for Applied Legal
Studies Intervening) 2001, the CC made it clear that the boni mores must now be informed by the
values underpinning the Bill of Rights in the Constitution. If these principles are applied to the given
facts, we can probably conclude that the omission of John was indeed wrongful.

2.2 Peter, a thirteen year-old boy, practices his golf swing in his parents’ garden in an upmarket suburb
in the north of Johannesburg. He hits a magnificent shot, but the trajectory of the ball ends in the
adjacent garden, where it kills the neighbour’s very rare and exceedingly expensive Pink Amazon
Parrot. Was Peter negligent? Discuss in detail with reference to case law. (15)

Before Jones NO v Santam Bpk 1965, the negligence of a child used to be determined with reference to
a reasonable child standard. In the Jones case, the court held that the test for negligence remains
objective, and the reasonable person test (the diligens paterfamilias test) must also be employed in
the case of a child wrongdoer. The youthfulness of the child wrongdoer is not specifically considered
here. However, during the inquiry into the accountability of the child, his or her youthfulness is taken
into account. The Jones case was criticised on 2 counts: firstly, many are of the opinion that a
reasonable adult standard for a child wrongdoer is unfair; secondly, the court put the cart before the
horse by testing for negligence first and, thereafter, for accountability. In Roxa v Mtshayi 1975, the
court followed the correct order. In Weber v Santam Versekeringsmaatskappy Bpk 1983, the Jones
case was confirmed in essence and the court said that if the principles were applied with insight, the
criticism would fall away. In Eskom Holdings Ltd v Hendricks 2005, the court reiterated that in each
case it must be determined whether the child has attained the emotional and intellectual maturity to
appreciate the danger to be avoided and to act accordingly. In respect of accountability, the Child
Justice Act 75 of 2008 provides that a child of 9 or younger is irrebuttably presumed to be culpae
incapax/not accountable, whereas a child over 9, but under 14, is rebuttably presumed to be culpae
incapax/not accountable. Whether Peter in our question would be held to have been negligent would
depend on all the circumstances of the case. If he was intelligent and mature enough to be
accountable/culpae capax, he was probably negligent, because, taken at face value, his conduct
deviated from that of the reasonable person in the circumstances.

Question 3:

In his haste to get to his seat on a soccer stadium, Max bumps into Sam. Sam falls and breaks a leg.
Sam is admitted to hospital. A nurse leaves the windows of the ward open, and Sam contracts
pneumonia.

3.1 Does a factual causal link exist between Max’s conduct and Sam’s pneumonia? Discuss. (5)

The generally accepted test for factual causation is the conditio sine qua non test, or ‘‘but for test’’.
This entails mentally eliminating, or thinking away, the conduct. If the damage then also disappears, a
factual causal link is present between the conduct and the damage. This test is subject to much
criticism. Among others, it is said to be based on circular logic and is, at best, a way to express the
existence of a causal nexus that has been determined in another way. Neethling and Potgieter argue
that evidence and human experience are sufficient to determine whether one fact flowed from
another fact, and that a so-called test of factual causation is superfluous. However, the courts
consistently state that the conditio sine qua non is the test of factual causation. If we apply the test to
the facts, we must conclude that if Max had not bumped Sam, he would not have broken a leg, and
therefore a factual causal link is present between Max’s conduct and Sam’s damage.
3.2 Does a legal causal link exist between Max’s conduct and Sam’s pneumonia? Discuss in detail with
reference to case law. (15)

The test for legal causation is the so-called flexible approach, as formulated in S v Mokgethi 1990 and
International Shipping Co (Pty) Ltd v Bentley 1990. In Mokgethi, a bank robber shot a teller. The teller
was rendered a paraplegic and was discharged from hospital in a wheelchair. Subsequently, the
paraplegic man failed to shift his body position in the chair frequently and developed pressure sores,
eventually dying from complications. The question that arose was whether the shot fired by the
robber was the legal cause of the teller’s death. According to the court, the main question in respect of
legal causation is whether there is a close enough relationship between the wrongdoer’s conduct and
its consequence for such consequence to be imputed to the wrongdoer in view of policy
considerations based on reasonableness, fairness and justice. Several other legal causation theories
exist, such as adequate causation, direct consequences, foreseeability and novus actus interveniens.
None of these criteria is suitable to be applied to all situations. They may, however, be used as
subsidiary aids when employing the flexible approach. In the Mokgethi case, the court held that the
shot was not a legal cause of the death. If these principles are applied to the facts in the question, the
conclusion is probably that Sam’s pneumonia was too remote and should not be imputed to the
wrongdoer (Max). It could also be argued that a so-called novus actus interveniens (a new intervening
act) was constituted by a nurse leaving the windows of the ward open, and this strengthens the
conclusion that there is no legal causal link between Max’s conduct and Sam’s pneumonia.

May/June 2017 – Second paper


1 If patrimonial loss is caused in a negligent manner, the most appropriate delictual remedy to consider
is:

1. The actio legis Aquiliae


2. The actio iniuriarum
3. The action for pain and suffering
4. None of the above

2 If bodily injury is caused in a negligent manner, the most appropriate delictual remedy to consider is:

1. The actio legis Aquiliae


2. The actio iniuriarum
3. The action for pain and suffering
4. None of the above

3 If the dignity of a person is infringed in a negligent manner, the most appropriate delictual remedy to
consider is:

1. The actio legis Aquiliae


2. The actio iniuriarum
3. The action for pain and suffering
4. None of the above

4 If a domestic animal has caused harm to a person and the owner of the animal was not negligent, the
most appropriate delictual remedy to consider is:
1. The actio legis Aquiliae
2. The actio de pauperie
3. The action de pastu
4. None of the above

5 The most suitable remedy for the prevention of harm is:

1. The actio legis Aquiliae


2. The actio de effuses vel deiectis
3. The interdict
4. Mitigation of loss

6 Which statement is incorrect?

1. Only an act or omission that has been willed, can give rise to delictual liability
2. An act or an omission that is irrational or inexplicable may give rise to delictual liability
3. The behaviour of an animal can never qualify as conduct for the purpose of the law ofdelict
4. A company can act for the purpose of the law of delict

7 Adam tells Bob that Charles, a well-known and married attorney, is having a secret affair with his
secretary. Bob repeats the information to Donald, a reporter. Donald writes an article about this, and
it is published in the local newspaper, The Event, with knowledge of Frank, the editor. Which of the
following options is the most correct?

1. Donald, The Event and Frank may incur liablity with the actio iniuriarum.
2. Adam, Donald, The Event and Frank may incur liablity with the actio iniuriarum.
3. Adam, Bob, Donald, The Event and Frank may incur liablity with the actio iniuriarum.
4. Bob, Donald, The Event and Frank may incur liability with the actio iniuriarum.

8 John’s prize bull breaks through a fence and ventures into David’s farm. The bull is aggressive and
charges at David’s employees. The employees clamber into a small tree to escape the wrath of the
bull. The bull begins to bash the trunk of the tree. As the tree appears to be about to topple, David
shoots and kills the bull to protect his employees. John institutes an action for damages against David.
David may raise the following ground of justification:

1. Necessity
2. Private defence
3. Provocation
4. Official capacity

9 Adrian insults Tumelo, who in retaliation slaps Adrian across the cheek. If Adrian institutes a delictual
action for the infringement of his personality rights, Tumelo may rely on the following ground of
justification:

1. Provocation.
2. Official capacity.
3. Private defence.
4. None of the above.
10 Jim insults Karen, Julie’s friend. Julie, who is angered by the insults, then insults Jim. If Jim institutes a
delictual action for the infringement of his personality rights, Julie may rely on the following ground of
justification:

1. Necessity.
2. Private defence.
3. Provocation.
4. None of the above.

11 Markus, a financial adviser, negligently advises Hans to make a bad investment. As a result of this,
Hans suffers a serious financial setback. However, there is no damage to Hans’s person or property.
Which one of the following delictual remedies may be available if Hans wishes to recover his financial
loss from Markus?

1. The actio legis Aquiliae


2. The actio iniuriarum
3. The action for pain and suffering
4. None of the above

12 Gawie and Manie are farming on adjacent properties. Gawie bears a grudge against Manie. He sets up
a device, which causes a loud bang every half an hour, on the common boundary between the 2 farms
near Manie’s homestead. As a result of this, Manie and his family cannot sleep at night and are
irritated by day. Manie asks Gawie to dismantle the device, but Gawie says that the device serves the
purpose of scaring baboons away from his orchards. Which delictual remedy may be available to
Manie?

1. The actio de pauperie


2. The actio de feris
3. The actio de effuses vel deiectis
4. The interdict

13 Which one of the following statements is correct?

1. Accountability is a prerequisite for fault


2. A person is accountable if he can differentiate between right and wrong and his behaviour is
susceptible to control of his will
3. Intoxication cannot influence accountability
4. A person is accountable if he directs his will at an unlawful result and is also conscious of
wrongfulness

14 Which is the odd one out?

1. The adequate causation theory


2. The sum-formula approach
3. Direct consequences theory
4. Normative foreseeability

15 Greater Johannesburg Transitional Metropolitan Council v ABSA Bank Ltd 1997 (2) SA 591 (W) dealt
with:
1. Consent to injury
2. Consent to the risk of injury
3. Contributory intention
4. Contributory negligence

16 Piet left his cellphone in his car. Gert wants to steal Piet’s cellphone. He realises that he would have to
damage Piet’s car in order to get the phone. In respect of the damage to the car, Gert has:

1. Dolus indeterminatus
2. Dolus eventualis
3. Dolus indirectus
4. Luxuria

17 Joseph plays tennis in his back yard. He foresees the possibility of his ball breaking a window in his
neighbour’s house, but decides that it will not happen. If the ball indeed breaks the window, Joseph
had the following in respect of the damage:

1. Dolus indeterminatus.
2. Dolus eventualis.
3. Dolus indirectus.
4. Luxuria.

18 Ben clambers over Neil’s fence with the aim of breaking into Neil’s house. Neil’s dog attacks Ben and
Ben sustains serious injuries. Ben wishes to recover his hospital expenses from Neil. Which remedy
may be available to Ben?

1. Actio de pauperie.
2. Actio de pastu.
3. Actio de feris.
4. None of the above

19 The principle dictating at what stage prospective loss must be claimed, is known as:

1. The sum-formula approach


2. The “once and for all” rule
3. Compensating advantages
4. The concrete approach to damage

20 John and Peter bear a grudge against Greg and agree to teach him a lesson he will not forget. While
Greg is in a shopping centre, John and Peter damage Greg’s motor car by hitting it with hammers. If
Greg successfully sues John and Peter in delict, they will incur:

1. vicarious liability.
2. joint and several liability.
3. liability based on contributory intent.
4. strict liability.
Section B:

Question 1:

The municipality of eThekwini is busy with excavations in the business centre. The municipal
workers do not erect any barrier or warning signs in the vicinity of the excavations. Mrs Ndlovu, who
is near-sighted, falls into the excavations and sustains serious injuries. She wishes to institute a
delictual action against the municipality.

1.1 Did the municipality act for the purposes of the law of delict? (5)

Conduct is defined as a voluntary human act or omission. “Voluntary” means that the person must be
able to control his muscular movements by means of his will. The act of the wrongdoer must be
voluntary to give rise to delictual liability. Any doubt about the applicability of the decision in the Ewels
case (that prior conduct is not indispensable for the existence of a legal duty) to the “municipality
cases”, was removed by the SCA in Cape Town Municipality v Bakkerud. The court held that the legal
convictions of the community could even in the absence of “prior conduct” (or a statutory duty) place
a legal duty on a municipality to, eg. repair roads or sidewalks or to warn against danger. Whether this
is the case, depends on the circumstances and must be determined ad hoc.

1.2 Was the conduct of the municipality wrongful? Discuss in detail. (15)

As a general rule, a person does not act wrongfully for the purposes of the law of delict if he omits to
prevent harm to another person. Liability only follows if the omission was in fact wrongful, and this will
be the case only if a legal duty rested on the defendant to act positively to prevent harm from
occurring and he failed to comply with that duty. The question of whether such a duty existed is
answered with reference to the criterion of the legal convictions of the community and legal policy.

This question deals with the wrongfulness of an omission. The basic question to determine whether an
omission is wrongful is whether a legal duty to act was present and was breached. This is determined
with reference to the legal convictions of the community, or the boni mores. Factors which may serve
as indications that a legal duty rested on the defendant include: prior conduct (omissio per
commissionem); control of a dangerous object; rules of law; a special relationship between the parties;
particular office; contractual undertaking for the safety of a third party; and creating of an impression
that the interests of a third person will be protected. In the so-called municipality cases, prior conduct
was considered to be a prerequisite for the wrongfulness of an omission. Prior conduct refers to
positive conduct that created a new source of danger, preceding subsequent omission to protect
others from being harmed by this new source of danger. The classic case in this respect is Halliwell v
Johannesburg Municipal Council 1912. The view that prior conduct was a prerequisite for wrongfulness
of an omission was eroded in Silva’s Fishing Corporation (Pty) Ltd v Maweza 1957; Regal v African
Superslate (Pty) Ltd 1963; and Minister of Forestry v Quathlamba 1973. In Minister van Polisie v Ewels
1975, the court finally held that the existence of a legal duty is determined by the boni mores, and
whereas the presence of prior conduct is a strong indication of the presence of wrongfulness, it is not
a prerequisite thereof. Subsequent judgments, such as Cape Town Municipality v Bakkerud 2000,
confirmed that the principles formulated in Ewels were applicable to municipality cases. An interplay
of different factors may also indicate the presence of a legal duty. In Carmichele v Minister of Safety
and Security (Centre for Applied Legal Studies Intervening) 2001, the CC made it clear that the boni
mores must now be informed by the values underpinning the Bill of Rights in the Constitution. If these
principles are applied to the given facts, we can probably conclude that the omission of the
municipality was indeed wrongful.
Question 2:

2.1 Johnny is an intelligent thirteen year-old boy. His dream is to be a great scientist one day. After
much research on the internet, Johnny builds a rocket in his parents’ back yard. One day he decides
to launch the rocket. The rocket ascends at great speed to a height of 200 metres. Then it bursts into
flames. It falls onto the thatched roof of the neighbour’s lapa. The lapa burns down. Discuss only,
but in detail, whether Johnny was negligent. (10)

Before Jones NO v Santam Bpk 1965, the negligence of a child used to be determined with reference to
a reasonable child standard. In the Jones case, the court held that the test for negligence remains
objective, and the reasonable person test (the diligens paterfamilias test) must also be employed in
the case of a child wrongdoer. The youthfulness of the child wrongdoer is not specifically considered
here. However, during the inquiry into the accountability of the child, his or her youthfulness is taken
into account. The Jones case was criticised on 2 counts: firstly, many are of the opinion that a
reasonable adult standard for a child wrongdoer is unfair; secondly, the court put the cart before the
horse by testing for negligence first and, thereafter, for accountability. In Roxa v Mtshayi 1975, the
court followed the correct order. In Weber v Santam Versekeringsmaatskappy Bpk 1983, the Jones
case was confirmed in essence and the court said that if the principles were applied with insight, the
criticism would fall away. In Eskom Holdings Ltd v Hendricks 2005, the court reiterated that in each
case it must be determined whether the child has attained the emotional and intellectual maturity to
appreciate the danger to be avoided and to act accordingly. In respect of accountability, the Child
Justice Act 75 of 2008 provides that a child of 9 or younger is irrebuttably presumed to be culpae
incapax/not accountable, whereas a child over 9, but under 14, is rebuttably presumed to be culpae
incapax/not accountable. Whether Johnny in our question would be held to have been negligent
would depend on all the circumstances of the case. If he was intelligent and mature enough to be
accountable/culpae capax, he was probably negligent, because, taken at face value, his conduct
deviated from that of the reasonable person in the circumstances.

2.2 Garth rents out mountain bikes. Phil hires one of the mountain bikes. On completing his ride, he is
unable to stop the bike, and he collides with a tree and sustains head injuries. It transpires that
Garth did not properly maintain the bikes, with the result that the brakes of the relevant bike were
malfunctioning. Phil, on the other hand, neglected to wear the safety helmet supplied by Garth. Phil
is hospitalised and his hospital costs amount to R10,000. His costs would have been R6,000 had he
worn the helmet. At the relevant time, Garth used no indemnity forms or notices excluding liability
in her business. Phil wishes to recover R10,000 in delictual damages from Garth. Will he be
successful? Discuss in detail with reference to case law and legislation. (15)

From the given facts, we can conclude that the defendant has been negligent, but the plaintiff appears
to have been negligent too. Thus we must consider whether contributory negligence was present.
Contributory negligence is negligence on the part of the plaintiff, and it is a defence that the defendant
can raise. The Apportionment of Damages Act 34 of 1956 is applicable. This Act provides that a
contributorily negligent plaintiff’s damages be apportioned. The court will determine the degree of
deviation from the reasonable person standard shown by the conduct of both the defendant and the
plaintiff, express the deviation as percentages, and use these percentages as a basis for the
apportionment. According to the Smit 1962 and Nomeka 1976 cases, the percentages of negligence
attributed to the defendant and plaintiff respectively will always add up to 100%.

According to Jones NO v Santam Bpk 1965, both percentages must be assessed independently, which
could mean that, for example, a defendant may be 80% negligent while the plaintiff is 30% negligent.
According to Neethling and Potgieter, the approach in Jones is to be preferred, but the 2 approaches
can be reconciled. According to King v Pearl Insurance Co Ltd 1970, a defence of contributory
negligence could not succeed where the plaintiff had omitted to wear a crash-helmet while driving a
scooter, but had not been negligent in respect of causing the accident.

However, in Bowkers Park Komga Cooperative Ltd v SAR and H 1980, the court held that contributory
negligence didn’t refer to negligence in respect of the damage-causing event, such as a motorcar
accident, but to negligence in respect of the damage itself, and this was confirmed by the AD in Union
National South British Insurance Co Ltd v Vitoria 1982 and General Accident Versekeringsmaatskappy
SA Bpk v Uijs 1993. Therefore, failure to wear a safety helmet would constitute contributory
negligence if it contributed to the plaintiff’s damage. Applying these principles to the facts, we can
conclude that Phil was contributorily negligent and that his damages will be apportioned. He will be
awarded R6 000 plus a portion of the R4 000 damage to which he contributed, taking into account his
and Garth’s respective degrees of negligence.

Question 3:

Dick is so offended by William’s speech at a political rally, that he throws a naartjie at William.
William falls off the podium and breaks an arm and a few ribs. He is admitted to hospital. While he
is being treated there, the nursing staff leaves the windows of the ward open and William contracts
pneumonia.

3.1 Was there a factual causal link between Dick’s conduct and William’s pneumonia? Discuss. (5)

The generally accepted test for factual causation is the conditio sine qua non test, or ‘‘but for test’’.
This entails mentally eliminating, or thinking away, the conduct. If the damage then also disappears, a
factual causal link is present between the conduct and the damage. This test is subject to much
criticism. Among others, it is said to be based on circular logic and is, at best, a way to express the
existence of a causal nexus that has been determined in another way. Neethling and Potgieter argue
that evidence and human experience are sufficient to determine whether one fact flowed from
another fact, and that a so-called test of factual causation is superfluous. However, the courts
consistently state that the conditio sine qua non is the test of factual causation. If we apply the test to
the facts, we must conclude that if Dick had not throne a naartjie at Willaim, he would not have
broken an arm and a few ribs, and therefore a factual causal link is present between Dick’s conduct
and William’s damage.

3.2 Was there a legal causal link between Dick’s conduct and William’s pneumonia? Discuss. (15)

The test for legal causation is the so-called flexible approach, as formulated in S v Mokgethi 1990 and
International Shipping Co (Pty) Ltd v Bentley 1990. In Mokgethi, a bank robber shot a teller. The teller
was rendered a paraplegic and was discharged from hospital in a wheelchair. Subsequently, the
paraplegic man failed to shift his body position in the chair frequently and developed pressure sores,
eventually dying from complications. The question that arose was whether the shot fired by the
robber was the legal cause of the teller’s death. According to the court, the main question in respect of
legal causation is whether there is a close enough relationship between the wrongdoer’s conduct and
its consequence for such consequence to be imputed to the wrongdoer in view of policy
considerations based on reasonableness, fairness and justice. Several other legal causation theories
exist, such as adequate causation, direct consequences, foreseeability and novus actus interveniens.
None of these criteria is suitable to be applied to all situations. They may, however, be used as
subsidiary aids when employing the flexible approach. In the Mokgethi case, the court held that the
shot was not a legal cause of the death. If these principles are applied to the facts in the question, the
conclusion is probably that William’s pneumonia was too remote and should not be imputed to the
wrongdoer (Dick). It could also be argued that a so-called novus actus interveniens (a new intervening
act) was constituted by the nursing staff leaving the windows of the ward open, and this strengthens
the conclusion that there is no legal causal link between Dick’s conduct and William’s pneumonia.

Oct/Nov 2016
1 Mary moves into a new apartment on the fourth floor. While cleaning her apartment, she throws a
non-functional computer out through the window. The computer causes damage to Brian’s vehicle.
Brian wishes to institute a delictual action. With which remedy is he most likely to succeed?

1. The actio legis Aquiliae.


2. The actio iniuriarum.
3. The action for pain and suffering.
4. The actio de effuses vel deiectis.

2 Which one of the following is not a requirement for the granting of an interdict?

1. There must be an act (or threatening act) by the respondent.


2. The act (or threatening act) must be wrongful.
3. The act (or threatening act) must be negligent.
4. No other suitable remedy must be available to the applicant.

3 If the dignity of a person is infringed in a negligent manner, the most appropriate delictual remedy to
consider is:

1. The actio legis Aquiliae


2. The actio iniuriarum
3. The action for pain and suffering
4. None of the above

4 Cecil the lion escapes from a nature reserve. The boundary fence of the reserve has been damaged a
month ago but the game rangers have not repaired it. Cecil wanders onto Tshepo’s farm and kills five
head of cattle. Tshepo would like to institute a delictual claim. With which remedy is he most likely to
succeed?

1. The actio legis Aquiliae


2. The actio de pauperie
3. The actio de pastu
4. The actio de feris

5 The most suitable remedy for the prevention of harm is:

1. The actio legis Aquiliae


2. The actio de effuses vel deiectis
3. The interdict
4. Mitigation of loss

6 Which statement is incorrect?

1. Only an act or omission that has been willed, can give rise to delictual liability
2. An act or an omission that is irrational or inexplicable may give rise to delictual liability
3. The behaviour of an animal can never qualify as conduct for the purpose of the law of delict
4. A company can act for the purpose of the law of delict

7 Adam tells Bob that Charles, a quiet and very reserved computer scientist, is having a secret affair with
an unmarried receptionist. Bob repeats the information to Donald, a reporter. Donald writes an article
about this, and it is published in the local newspaper, The Event, with knowledge of Frank, the editor.
Which of the following options is the most correct?

1. Donald, The Event and Frank may incur liability with the actio iniuriarum.
2. Adam, Donald, The Event and Frank may incur liability with the actio iniuriarum.
3. Adam, Bob, Donald, The Event and Frank may incur liability with the actio iniuriarum.
4. None of the above.

8 John’s prize bull breaks through a fence and ventures into David’s farm. The bull is aggressive and
charges at David’s employees. The employees clamber into a small tree to escape the wrath of the
bull. The bull begins to bash the trunk of the tree. As the tree appears to be about to topple, David
shoots and kills the bull to protect his employees. John institutes an action for damages against David.
David may raise the following ground of justification:

1. Necessity
2. Private defence
3. Provocation
4. Official capacity

9 Karen slaps Cayla in the face. Cayla immediately kicks Karen’s shin. If Karen institutes a delictual action
for the infringement of his personality rights, Cayla may rely on the following ground of justification:

1. Necessity
2. Private defence
3. Provocation.
4. None of the above.

10 Nick insults Sarah, Julie’s friend. Julie, who is angered by the insults, then insults Nick. If Nick institutes
a delictual action for the infringement of his personality rights, Julie may rely on the following ground
of justification:

1. Necessity.
2. Private defence.
3. Provocation.
4. None of the above.

11 Markus, a financial adviser, negligently advises Hans to make a bad investment. As a result of this,
Hans suffers a serious financial setback. However, there is no damage to Hans’s person or property.
Which one of the following delictual remedies may be available if Hans wishes to recover his financial
loss from Markus?

1. The actio legis Aquiliae


2. The actio iniuriarum
3. The actio ad exhibendum
4. None of the above
12 Sam and Rob live on adjacent agricultural smallholdings. Sam bears a grudge against Rob. He builds a
big cage near the boundary between the 2 properties, far from his house but close to Rob’s house. He
stocks the cage with parrots with piercing voices, and installs a bright light in the cage so that the
parrots will not sleep much at night. Because of the loud calls of the parrots, Rob and his family cannot
sleep at night and are irritated by day. Rob asks Sam to relocate the cage, but Sam replies that, as
owner of his own land, he is entitled to pursue his bird-breeding activities. Which delictual remedy
may be available to Rob?

1. The actio de pauperie


2. The actio de feris
3. The interdict
4. None of the above

13 Which one of the following statements is correct?

1. Accountability is a prerequisite for fault


2. A person is accountable if he can differentiate between right and wrong and his behaviour is
susceptible to control of his will
3. Intoxication cannot influence accountability
4. A person is accountable if he directs his will at an unlawful result and is also conscious of
wrongfulness

14 Which is the odd one out?

1. The adequate causation theory


2. The sum-formula approach
3. Direct consequences theory
4. Normative foreseeability

15 In Bester v Commercial Union Versekeringsmaatskappy van SA Bpk 1973 (1) SA 769 (A), the court held
that:

1. The elements of wrongfulness and negligence are of particular importance in determining


liability for negligent misrepresentation
2. An omission in the case of negligent misrepresentation can exist in the form of non-disclosure
of information
3. Impairment of personality and patrimonial loss resulting from emotional shock can found
the action for pain and suffering and the actio legis Aquiliae.
4. Intentional interference with contractual relations can give rise to delictual liability

16 Zaheer steals a motor car from a car dealership. To get the vehicle, he breaks open an expensive
security gate. In respect of the damage to the gate, Zaheer has:

1. Dolus indeterminatus
2. Dolus eventualis
3. Dolus indirectus
4. Luxuria
17 Joseph practices his golf swing in his back yard. He foresees the possibility of his ball breaking a
window in his neighbour’s house, but decides that it will not happen. If the ball indeed breaks the
window, Joseph had the following in respect of the damage:

1. Dolus indeterminatus
2. Dolus eventualis
3. Dolus indirectus
4. Luxuria

18 Which one of the following is not a requirement for the liability of a motor car owner for the delict of
the motor car driver?
1. The owner must request the driver to drive the vehicle or supervise the driving
2. The driver must be an employee of the motor car owner
3. The vehicle must be driven in the interest of the motor car owner
4. The owner must retain a right of control over the manner in which the vehicle is driven.

19 Phoebe surreptitiously enters Nellie’s apartment to steal Nellie’s jewellery. Nellie’s parrot attacks
Phoebe and disfigures her ear. Phoebe wishes to institute a delictual action against Nellie. Which
remedy may be available to Phoebe?

1. Actio de pauperie
2. Actio de pastu
3. Actio de feris
4. None of the above

20 In which one of the following cases did the court hold that an apportionment of damages could
succeed where both the plaintiff and the defendant acted with intention?

1. Greater Johannesburg Transitional Metropolitan Council v ABSA Bank 1997 2 SA 691 (W)
2. Wapnick v Durban City Garage 1984 2 SA 414 (D)
3. Lampert v Hefer 1955 2 SA 507 (A)
4. Netherlands Insurance Co of SA v Van der Vyver 1968 1 SA 412 (A)

21 What is the criterion for determining factual causation?

1. Actio libera in causa.


2. Novus actus interveniens.
3. Conditio sine qua non.
4. The flexible approach.

22 Jacob is training to be a tailor. He asks Karen whether he can practice his newly acquired skills to
convert her long skirt to a very short mini-skirt. Karen agrees, but after looking at herself in the mirror
wearing the short skirt, she regrets the change to her skirt and wants to claim damages from Jacob.
Jacob may rely on the following ground of justification:

1. Provocation
2. Official capacity
3. Consent
4. None of the above
23 The principle dictating at what stage prospective loss must be claimed, is known as:

1. The sum-formula approach


2. The “once and for all” rule
3. Compensating advantages
4. The concrete approach to damage

24 Prospective loss is best assessed in accordance with:

1. the sum-formula approach


2. the “once and for all” rule
3. compensating advantages
4. the concrete approach to damage

25 Mike, Mac and Mark steal and sell a truckload of Mel’s mealies. If Mel successfully sues Mike, Mac and
Mark in delict, they will incur:

1. vicarious liability.
2. joint and several liability.
3. liability based on contributory intent.
4. strict liability.

Section B:

Question 1:

Municipal workers of the city of Cape Town are busy repairing a coastal walkway that has been
damaged by a stormy sea during an exceptional high tide. During their lunch break, the workers
walk to a café to buy food. They leave a big cavity in the walkway, which they have been trying to fill
up, unattended and do not cordon it off or put up any signs warning pedestrians of this hazard. Jim,
a health-conscious engineer who works in a nearby office, goes for his customary lunch-hour jog
along the walkway. Jim is near-sighted, but because he perspires freely while he is exercising, he
leaves his spectacles in the office. While sprinting the final section of this jog, Jim sees the cavity in
the walkway too late, slips in a puddle of seawater and falls into the cavity. He breaks an arm,
fractures two ribs and sustains light concussion. Jim wishes to institute a delictual action against the
municipality. Was the conduct of the municipality wrongful? Restrict the scope of your answer to
what is asked in the question, but discuss in depth. (15)

As a general rule, a person does not act wrongfully for the purposes of the law of delict if he omits to
prevent harm to another person. Liability only follows if the omission was in fact wrongful, and this will
be the case only if a legal duty rested on the defendant to act positively to prevent harm from
occurring and he failed to comply with that duty. The question of whether such a duty existed is
answered with reference to the criterion of the legal convictions of the community and legal policy.

This question deals with the wrongfulness of an omission. The basic question to determine whether an
omission is wrongful is whether a legal duty to act was present and was breached. This is determined
with reference to the legal convictions of the community, or the boni mores. Factors which may serve
as indications that a legal duty rested on the defendant include: prior conduct (omissio per
commissionem); control of a dangerous object; rules of law; a special relationship between the parties;
particular office; contractual undertaking for the safety of a third party; and creating of an impression
that the interests of a third person will be protected. In the so-called municipality cases, prior conduct
was considered to be a prerequisite for the wrongfulness of an omission. Prior conduct refers to
positive conduct that created a new source of danger, preceding subsequent omission to protect
others from being harmed by this new source of danger. The classic case in this respect is Halliwell v
Johannesburg Municipal Council 1912. The view that prior conduct was a prerequisite for wrongfulness
of an omission was eroded in Silva’s Fishing

Corporation (Pty) Ltd v Maweza 1957; Regal v African Superslate (Pty) Ltd 1963; and Minister of
Forestry v Quathlamba 1973. In Minister van Polisie v Ewels 1975, the court finally held that the
existence of a legal duty is determined by the boni mores, and whereas the presence of prior conduct
is a strong indication of the presence of wrongfulness, it is not a prerequisite thereof. Subsequent
judgments, such as Cape Town Municipality v Bakkerud 2000, confirmed that the principles formulated
in Ewels were applicable to municipality cases. An interplay of different factors may also indicate the
presence of a legal duty. In Carmichele v Minister of Safety and Security (Centre for Applied Legal
Studies Intervening) 2001, the CC made it clear that the boni mores must now be informed by the
values underpinning the Bill of Rights in the Constitution. If these principles are applied to the given
facts, we can probably conclude that the omission of the municipality was indeed wrongful.

Question 2:

2.1 Tom is an energetic and intelligent thirteen year-old boy. He has built a sturdy soapbox-cart. He pulls
it to the top of a steep hill to test drive his new “vehicle”. He gives the cart a push, jumps in and
grabs hold of the steering wheel. The cart goes downhill at a terrific speed, but the steering
mechanism and the brakes do not operate very effectively. Tom crashes into the neighbour’s
expensive sports car that is, as usual, parked on the sidewalk. Tom causes a lot of damage. Discuss
only, but in detail, whether Tom was negligent. (10)

Before Jones NO v Santam Bpk 1965, the negligence of a child used to be determined with reference to
a reasonable child standard. In the Jones case, the court held that the test for negligence remains
objective, and the reasonable person test (the diligens paterfamilias test) must also be employed in
the case of a child wrongdoer. The youthfulness of the child wrongdoer is not specifically considered
here. However, during the inquiry into the accountability of the child, his or her youthfulness is taken
into account. The Jones case was criticised on 2 counts: firstly, many are of the opinion that a
reasonable adult standard for a child wrongdoer is unfair; secondly, the court put the cart before the
horse by testing for negligence first and, thereafter, for accountability. In Roxa v Mtshayi 1975, the
court followed the correct order. In Weber v Santam Versekeringsmaatskappy Bpk 1983, the Jones
case was confirmed in essence and the court said that if the principles were applied with insight, the
criticism would fall away. In Eskom Holdings Ltd v Hendricks 2005, the court reiterated that in each
case it must be determined whether the child has attained the emotional and intellectual maturity to
appreciate the danger to be avoided and to act accordingly. In respect of accountability, the Child
Justice Act 75 of 2008 provides that a child of 9 or younger is irrebuttably presumed to be culpae
incapax/not accountable, whereas a child over 9, but under 14, is rebuttably presumed to be culpae
incapax/not accountable. Whether Tom in our question would be held to have been negligent would
depend on all the circumstances of the case. If he was intelligent and mature enough to be
accountable/culpae capax, he was probably negligent, because, taken at face value, his conduct
deviated from that of the reasonable person in the circumstances.
2.2 The Mountain Air Hotel rents out riding horses to its guests. Ed, who used to be a keen rider in his
youth, hires a sprinted young stallion. The staff of Mountain Air neglect to inspect the quite old and
worn riding gear. However, they do warn Ed to wear a riding helmet for his own safety, but Ed
ignores this. During the ride, the saddle breaks. Ed tumbles headlong to the ground and sustains
head injuries. He is hospitalised and his hospital costs amount to R10,000. The doctor is of the
opinion that Ed’s costs would have been R7,000 had he worn the helmet. Ed wishes to recover
R10,000 in delictual damages from Mountain Air. You may assume that Ed did not give consent to
the risk of injury and that Mountain Air had no indemnity forms or notices excluding liability in use
at the time of the incident. What defence may be available to Mountain Air Hotel and what will the
effect of a successful reliance on the defence be? Discuss in detail with reference to case law and
legislation. (15)

From the given facts, we can conclude that the defendant has been negligent, but the plaintiff appears
to have been negligent too. Thus we must consider whether contributory negligence was present.
Contributory negligence is negligence on the part of the plaintiff, and it is a defence that the defendant
can raise. The Apportionment of Damages Act 34 of 1956 is applicable. This Act provides that a
contributorily negligent plaintiff’s damages be apportioned. The court will determine the degree of
deviation from the reasonable person standard shown by the conduct of both the defendant and the
plaintiff, express the deviation as percentages, and use these percentages as a basis for the
apportionment. According to the Smit 1962 and Nomeka 1976 cases, the percentages of negligence
attributed to the defendant and plaintiff respectively will always add up to 100%.

According to Jones NO v Santam Bpk 1965, both percentages must be assessed independently, which
could mean that, for example, a defendant may be 80% negligent while the plaintiff is 30% negligent.
According to Neethling and Potgieter, the approach in Jones is to be preferred, but the 2 approaches
can be reconciled. According to King v Pearl Insurance Co Ltd 1970, a defence of contributory
negligence could not succeed where the plaintiff had omitted to wear a crash-helmet while driving a
scooter, but had not been negligent in respect of causing the accident.

However, in Bowkers Park Komga Cooperative Ltd v SAR and H 1980, the court held that contributory
negligence didn’t refer to negligence in respect of the damage-causing event, such as a motorcar
accident, but to negligence in respect of the damage itself, and this was confirmed by the AD in Union
National South British Insurance Co Ltd v Vitoria 1982 and General Accident Versekeringsmaatskappy
SA Bpk v Uijs 1993. Therefore, failure to wear a safety helmet would constitute contributory
negligence if it contributed to the plaintiff’s damage. Applying these principles to the facts, we can
conclude that Ed was contributorily negligent and that his damages will be apportioned. He will be
awarded R7,000 plus a portion of the R3,000 damage to which he contributed, taking into account his
and Mountain Air’s respective degrees of negligence.

Question 3:

Sally and Sandy support rival soccer teams. While watching a game between their respective teams,
they get involved in a heated argument. Sally hits Sandy over the head with an umbrella. Sandy
sustains a bleeding head injury. Mary, an off-duty nurse, offers to take Sandy to the public toilet to
clean the wound and to cover it with a bandage. While Mary helps Sandy down the stadium, they
slip on some melted ice-cream and fall. Sandy breaks an arm. Discuss only whether there was a legal
causal link between Sally’s conduct and Sandy’s broken arm? (10)

The test for legal causation is the so-called flexible approach, as formulated in S v Mokgethi 1990 and
International Shipping Co (Pty) Ltd v Bentley 1990. In Mokgethi, a bank robber shot a teller. The teller
was rendered a paraplegic and was discharged from hospital in a wheelchair. Subsequently, the
paraplegic man failed to shift his body position in the chair frequently and developed pressure sores,
eventually dying from complications. The question that arose was whether the shot fired by the
robber was the legal cause of the teller’s death. According to the court, the main question in respect of
legal causation is whether there is a close enough relationship between the wrongdoer’s conduct and
its consequence for such consequence to be imputed to the wrongdoer in view of policy
considerations based on reasonableness, fairness and justice. Several other legal causation theories
exist, such as adequate causation, direct consequences, foreseeability and novus actus interveniens.
None of these criteria is suitable to be applied to all situations. They may, however, be used as
subsidiary aids when employing the flexible approach. In the Mokgethi case, the court held that the
shot was not a legal cause of the death. If these principles are applied to the facts in the question, the
conclusion is probably that Sandy’s broken arm was too remote and should not be imputed to the
wrongdoer. It could also be argued that a so-called novus actus interveniens (a new intervening act)
was constituted by Sandy’s fall, and this strengthens the conclusion that there’s no legal causal link
between Sally’s conduct and Sandy’s broken arm.

May/June 2016

1 The appropriate delictual remedy to claim compensation for the negligent infringement of bodily
integrity is:

1. The actio legis Aquiliae.


2. The actio iniuriarum.
3. The action for pain and suffering.
4. The interdict.

2 One night, while staying in a seaside hotel, Peter gets out of bed and walks in his sleep. He only wakes
up as he bumps into an expensive reading lamp and it falls crashing to the floor. The hotel
management wants to hold Peter delictually liable for the damage. What defence can Peter raise?

1. Consent
2. Automatism
3. Absence of accountability
4. No defence is available

3 Which one of the following acts amounts to an omission (omissio) on the part of the defendant?

1. X burns the overgrown grass on his property. It is a windy day and the fire spreads quickly and
uncontrollably and burns his neighbours’ crops
2. A good swimmer passes by a public swimming pool. He sees a child drowning but
simply ignores the incident as he does not want to be late for his date
3. Y is insulted by A and retaliates by slapping A across the face. A’s spectacles break and
splinters pierce his eyes
4. C suffers an epileptic fit while driving her vehicle. She causes an accident in which B’s vehicle is
badly damaged. C has not been diagnosed with epilepsy before this event.

4 Choose the correct statement. In Carmichele v Minister of Safety and Security (Centre for Applied Legal
Studies Intervening) 2001 (4) SA 938 (CC), the Constitutional Court:

1. recognised a claim for Constitutional damages.


2. noted that the law of delict admirably reflected the spirit, purport and object of the Bill of
Rights.
3. declared that the courts of lower instance had erred by applying a pre-constitutional
concept of the boni mores.
4. recognised the concept of a Constitutional delict.
5 Choose the correct statement:

1. The onus to prove sane automatism is on the defendant


2. Reliance on sane automatism will not succeed in instances of actio libera in causa.
3. Sane automatism is a ground of justification
4. Reliance on sane automatism will succeed if the defendant is negligent with regard to his or
her automatic body movements.

6 Negligence of a child wrongdoer is determined with reference to:

1. The reasonable person standard


2. The reasonable child standard
3. The reasonable expert standard
4. None of the above

7 Adam tells Bob that Charles, a well-known and married attorney, is having a secret affair with his
secretary. Bob repeats the information to Donald, a reporter. Donald writes an article about this, and
it is published in the local newspaper, The Event, with knowledge of Frank, the editor. Which of the
following options is the most correct?
1. Donald, The Event and Frank may incur liability with the actio iniuriarum.
2. Adam, Donald, The Event and Frank may incur liability with the actio iniuriarum.
3. Adam, Bob, Donald, The Event and Frank may incur liability with the actio iniuriarum.
4. Bob, Donald, The Event and Frank may incur liability with the actio iniuriarum.

8 Indicate which statement is incorrect:

1. Fault on the part of the aggressor is not a requirement for private defence
2. The attack need not be directed at the defendant for a reliance on private defence tosucceed
3. An incited animal may act for purposes of an attack giving rise to private defence
4. An imminently threatening attack may suffice for private defence

9 Dr Lucas, a well-known obstetrician, delivered Nicole’s daughter, Mandy. During delivery, the nerves
in Mandy’s right shoulder were injured resulting in the paralysis of her arm. All of this happened
because Dr Lucas failed to inform Nicole of the potential complications inherent in delivering a large
baby. Indicate the most correct statement: Negligence will be determined according to the standard
of:

1. the reasonable doctor test


2. the reasonable person test
3. the reasonable man test
4. the reasonable obstetrician test

10 Johnny, an intelligent 13 year-old boy, inadvertently breaks a priceless vase in Mr Khumalo’s house.
For the purpose of delictual liability, Johnny is:

1. Accountable
2. Culpae incapax.
3. Irrebuttably presumed to be culpae incapax.
4. Rebuttably presumed to be culpae incapax.
11 While repairing a stormwater drainage system, workers of the M municipality negligently sever an
electricity cable that supplies electricity to the F factory. The F factory suffers production losses and
loses several long-standing clients due to its inability to deliver products on time. However, the
severed cable is not the property of the F factory. In fact, there is no damage to any property of the
factory, and no employee of the factory is injured. Which one of the following delictual remedies may
be available if the F factory wishes to recover its loss from the M municipality?

1. The actio legis Aquiliae.


2. The actio iniuriarum.
3. The action for pain and suffering
4. None of the above

12 Gareth and Max are neighbours. They had a big quarrel years ago and have been enemies ever since.
Gareth plants a row of trees on the common boundary between the two properties, with the sole
purpose to make leaves fall constantly into Max’s swimming pool. What delictual remedy may be
available to Max?

1. The actio de pauperie


2. The actio de feris
3. The actio de effuses vel deiectis
4. The interdict
13 Carl, 17, left his Playstation in his locker at school. James, 15, wants to steal Carl’s Playstation. He
realises that he would have to damage Carl’s locker in order to get the Playstation. In respect of the
damage to the locker, James has:

1. Dolus directus
2. Dolus indirectus
3. Dolus eventualis
4. Luxuria

14 Which one of the following qualifies as private defence?

1. John’s vicious dog charges to attack Zain. To protect himself, Zain picks up a baseball bat and
gives the dog a blow to the head.
2. Brian threatens Wilma with a knife and robs her of her cellphone. After 3 months, Wilma sees
Brian at a shopping centre and reacts by stabbing Brian with a high heeled shoe.
3. A 15 year-old boy points a firearm at you and you grab his arm to prevent him from
shooting you. The boy sustains injuries to his arm.
4. A police officer arrests Liam in the execution of a legitimate warrant of arrest. Liam resists the
arrest. He jabs the policeman, cracking one of the policeman’s ribs in an attempt to escape.

15 Neil is chased by a mob of armed robbers. He jumps onto Conrad’s scooter, which is parked next to the
road, in order to race away. He escapes, but the scooter is damaged as a result of his wild driving.
Conrad institutes a delictual claim against Neil. Neil may raise the following ground of justification:

1. Private defence
2. Necessity
3. Provocation
4. Official capacity
16 In Greater Johannesburg Transitional Metropolitan Council v ABSA Bank 1997 2 SA 691 (W), the court
held that:

1. A defence of contributory intention could succeed where both the plaintiff and the
defendant acted with intention
2. A person is guilty of negligence if his conduct falls short of that of the standard of the diligens
paterfamilias.
3. For the purposes of the law of delict, intent and negligence may be present simultaneously
4. There is no single criterion for legal causation which is applicable in all instances

17 Which of the following concepts is best described as “patrimonial loss without injury to person or
property”?

1. Pure economic loss


2. Product liability
3. Emotional shock
4. None of the above

18 Sean is employed by Raymond as a driver in Raymond’s courier business. Sean has the weekend off,
and decides to visit Mary on Saturday evening. On his way to Mary’s apartment, Sean is involved in an
accident due to his own negligence. Sean’s vehicle and the vehicle of Catherine, the other motorist
involved in the accident, are badly damaged. Catherine discovers that Sean’s financial position is not
good. Advise Catherine on the best course of action:

1. Institute an action against Raymond based on vicarious liability


2. Institute an action against Raymond and Sean as joint wrongdoers
3. Institute an action against Raymond based on culpa in eligendo
4. Institute an action against Sean

19 Ben clambers over Neil’s fence with the aim of breaking into Neil’s house. Neil’s dog attacks Ben and
Ben sustains serious injuries. Ben wishes to recover his hospital expenses from Neil. Which remedy
may be available to Ben?

1. Actio de pauperie.
2. Actio de pastu.
3. Actio de feris.
4. None of the above

20 Phil’s bull tramples and eats Anne’s maize that she planted in her fields. Which remedy may be
available to Anne?

1. Actio de pauperie.
2. Actio de pastu.
3. Actio de effuses vel deiectis.
4. Actio de feris.

21 Which is the odd one out, with reference to the element of fault?

1. Actio iniuriarum.
2. Actio de pastu.
3. Actio de pauperie.
4. Vicarious liability.

22 Jacob is training to be a tattoo artist. He asks Karen whether he can practice his newly acquired skills
on her. Karen enquires whether she will be able to remove the tattoo by washing, and Jacob replies in
the affirmative. Thereupon Karen is willing to comply, and Jacob makes a butterfly tattoo on Karen’s
arm. When Karen discovers that the tattoo is permanent, she wants to institute a delictual action
against Jacob. Jacob may rely on the following ground of justification:

1. Necessity
2. Official capacity
3. Consent
4. None of the above

23 The principle dictating at what stage prospective loss must be claimed, is known as:

1. The sum-formula approach


2. The “once and for all” rule
3. Compensating advantages
4. The concrete approach to damage

24 Prospective loss is best assessed in accordance with:

1. the sum-formula approach


2. the “once and for all” rule
3. compensating advantages
4. the concrete approach to damage

25 Themba and Thabo make a fire and neglect to put it out. The fire spreads to Lindiwe’s flower-garden
and all the flowers die. Lindiwe would have sold the flowers to a cut-flower market and she suffers a
substantial loss of income. If Lindiwe successfully sues Themba and Thabo in delict, they willincur:

1. vicarious liability.
2. joint and several liability.
3. liability based on contributory intent.
4. strict liability.

Section B:

Questions 1 to 3 facts:

Alex enters the Bargain-Buy Store to buy some groceries. Bargain-Buy has a large section of fresh
fish on display. However, it is an exceptionally hot day and the ice that is used to keep the fish cold,
is melting faster than usual. A puddle of water has formed below the display table. The store
manager has put up a sign displaying the words “Watch out – water on floor”. In addition, the store
manager has instructed an employee of Bargain-Buy to dry the area with a mop at three-hourly
intervals. Alex reads the sign, and sees the water, but proceeds to walk to the fish display. He slips
and falls, breaking his left arm and fracturing two ribs. He is admitted to the CareMed hospital for
treatment. Diane, a nurse in the employ of CareMed, adjusts the airconditioning, and Alex contracts
pneumonia. Answer the following questions 1 to 3 below (with their subdivisions) with reference to
the set of facts.

Question 1:

Was the conduct of the Bargain-Buy Store wrongful? Discuss in detail and refer to applicable case
law. Do not discuss other delictual elements in your answer. (15)

As a general rule, a person does not act wrongfully for the purposes of the law of delict if he omits to
prevent harm to another person. Liability only follows if the omission was in fact wrongful, and this will
be the case only if a legal duty rested on the defendant to act positively to prevent harm from
occurring and he failed to comply with that duty. The question of whether such a duty existed is
answered with reference to the criterion of the legal convictions of the community and legal policy.

This question deals with the wrongfulness of an omission. The basic question to determine whether an
omission is wrongful is whether a legal duty to act was present and was breached. This is determined
with reference to the legal convictions of the community, or the boni mores. Factors which may serve
as indications that a legal duty rested on the defendant include: prior conduct (omissio per
commissionem); control of a dangerous object; rules of law; a special relationship between the parties;
particular office; contractual undertaking for the safety of a third party; and creating of an impression
that the interests of a third person will be protected. In the so-called municipality cases, prior conduct
was considered to be a prerequisite for the wrongfulness of an omission. Prior conduct refers to
positive conduct that created a new source of danger, preceding subsequent omission to protect
others from being harmed by this new source of danger. The classic case in this respect is Halliwell v
Johannesburg Municipal Council 1912. The view that prior conduct was a prerequisite for wrongfulness
of an omission was eroded in Silva’s Fishing Corporation (Pty) Ltd v Maweza 1957; Regal v African
Superslate (Pty) Ltd 1963; and Minister of Forestry v Quathlamba 1973. In Minister van Polisie v Ewels
1975, the court finally held that the existence of a legal duty is determined by the boni mores, and
whereas the presence of prior conduct is a strong indication of the presence of wrongfulness, it is not
a prerequisite thereof. Subsequent judgments, such as Cape Town Municipality v Bakkerud 2000,
confirmed that the principles formulated in Ewels were applicable to municipality cases. An interplay
of different factors may also indicate the presence of a legal duty. In Carmichele v Minister of Safety
and Security (Centre for Applied Legal Studies Intervening) 2001, the CC made it clear that the boni
mores must now be informed by the values underpinning the Bill of Rights in the Constitution. If these
principles are applied to the given facts, we can probably conclude that the omission of the Bargain-
Buy Store was indeed wrongful.

Question 2:

2.1 Assume that the conduct of Bargain-Buy was indeed wrongful. Was the conduct of Bargain-Buy
negligent? Discuss with reference to case law. (5)

Kruger v Coetzee, the test for negligence:

The reasonable person in the position of the defendant:

(a) would foresee the reasonable possibility of his conduct injuring another in his person or property
and causing him patrimonial loss; and

(b) would take reasonable steps to guard against such occurrence; and the defendant failed to take
such steps.

If these principles are applied to the given facts, we can probably conclude that the conduct of
Bargain-Buy was not negligent as the store manager foresaw the reasonable possibility of his conduct
injuring another and took reasonable steps to guard against such occurrence by putting up a sign
displaying the words “Watch out – water on floor”, and instructing an employee of Bargain-Buy to dry
the area with a mop at three-hourly intervals.

2.2 Assume that Bargain-But has acted wrongfully and negligently and that Bargain-Buy will not be able
to rely on the defence of consent to the risk of injury (or voluntary assumption of risk). Discuss
another defence that Bargain-Buy may raise. Refer to applicable legislation and case law. (15)

From the given facts, we can conclude that the defendant has been negligent, but the plaintiff appears
to have been negligent too. Thus we must consider whether contributory negligence was present.
Contributory negligence is negligence on the part of the plaintiff, and it is a defence that the defendant
can raise. The Apportionment of Damages Act 34 of 1956 is applicable. This Act provides that a
contributorily negligent plaintiff’s damages be apportioned. The court will determine the degree of
deviation from the reasonable person standard shown by the conduct of both the defendant and the
plaintiff, express the deviation as percentages, and use these percentages as a basis for the
apportionment. According to the Smit 1962 and Nomeka 1976 cases, the percentages of negligence
attributed to the defendant and plaintiff respectively will always add up to a 100%.

According to Jones NO v Santam Bpk 1965, both percentages must be assessed independently, which
could mean that, for example, a defendant may be 80% negligent while the plaintiff is 30% negligent.

According to Neethling and Potgieter, the approach in Jones is to be preferred, but the 2 approaches
can be reconciled. According to King v Pearl Insurance Co Ltd 1970, a defence of contributory
negligence could not succeed where the plaintiff had omitted to wear a crash-helmet while driving a
scooter, but had not been negligent in respect of causing the accident.

However, in Bowkers Park Komga Cooperative Ltd v SAR and H 1980, the court held that contributory
negligence didn’t refer to negligence in respect of the damage-causing event, such as a motorcar
accident, but to negligence in respect of the damage itself, and this was confirmed by the AD in Union
National South British Insurance Co Ltd v Vitoria 1982 and General Accident Versekeringsmaatskappy
SA Bpk v Uijs 1993. Therefore, Alex reads the sign, and sees the water, but proceeds to walk to the fish
display would constitute contributory negligence if it contributed to the plaintiff’s (Alex’s) damage.

Question 3:

3.1 Was there a factual causal link between Bargain-Buy’s conduct and the pneumonia that Alex has
contracted? Discuss. (5)

The generally accepted test for factual causation is the conditio sine qua non test, or ‘‘but for test’’.
This entails mentally eliminating, or thinking away, the conduct. If the damage then also disappears, a
factual causal link is present between the conduct and the damage. This test is subject to much
criticism. Among others, it is said to be based on circular logic and is, at best, a way to express the
existence of a causal nexus that has been determined in another way. Neethling and Potgieter argue
that evidence and human experience are sufficient to determine whether one fact flowed from
another fact, and that a so-called test of factual causation is superfluous. However, the courts
consistently state that the conditio sine qua non is the test of factual causation. If we apply the test to
the facts, we must conclude that if Bargain-Buy had mopped up the puddle of water, Alex would not
have broken his arm and fractured two ribs, and therefore a factual causal link is present between
Bargain-Buy’s conduct and Alex’s damage.
3.2 Was there a legal causal link between Bargain-Buy’s conduct and the pneumonia that Alex
has
contracted? Discuss with reference to case law. (10)

The test for legal causation is the so-called flexible approach, as formulated in S v
Mokgethi 1990 andInternational Shipping Co (Pty) Ltd v Bentley 1990. In Mokgethi, a bank
robber shot a teller. The tellerwas rendered a paraplegic and was discharged from
hospital in a wheelchair. Subsequently, the paraplegic man failed to shift his body position
in the chair frequently and developed pressure sores, eventually dying from
complications. The question that arose was whether the shot fired by the
robber was the legal cause of the teller’s death. According to the court, the main question
in respect oflegal causation is whether there is a close enough relationship between the
wrongdoer’s conduct and its consequence for such consequence to be imputed to the
wrongdoer in view of policy considerations based on reasonableness, fairness and justice.
Several other legal causation theories exist, such as adequate causation, direct
consequences, foreseeability and novus actus interveniens.
None of these criteria is suitable to be applied to all situations. They may, however, be
used as subsidiary aids when employing the flexible approach. In the Mokgethi case, the
court held that the shot was not a legal cause of the death. If these principles are applied
to the facts in the question, theconclusion is probably that Alex’s pneumonia was too
remote and should not be imputed to the wrongdoer (Bargain-Buy). It could also be
argued that a so-called novus actus interveniens (a new intervening act) was constituted
by a nurse adjusting the airconditioning, and this strengthens the conclusion that there is
no legal causal link between Bargain-Buy’s conduct and Alex’s pneumonia.
Law of Delict

PVL3703

2020 ASSIGNMENTS
1.1 ASSIGNMENT 01 (WRITTEN ASSIGNMENT)

COMMENTARY
ASSIGNMENT 01: PROBLEM TYPE QUESTION

Question
Read the judgment in Stedall and Another v Aspeling and Another 2018 (2) SA 75 (SCA) and
write a discussion of 500-1000 words. Your discussion should include:
• a discussion of what the court decided in respect of the wrongfulness element only in that case
[a maximum of 5 marks are awarded for the discussion of what the court decided in respect of
the wrongfulness element]; and
• a comparison of the court’s decision with that of Neethling and Potgieter in Neethling-
Potgieter-Visser Law of Delict on how wrongfulness should be established [a maximum of 5
marks are awarded for providing your own comparison].
You can find the judgment in conventional law libraries, or online by conducting a google search
or at the website of the Southern African Legal Information Institute (SAFLII) at www.saflii.org.

For an example of how a judgment should be summarised, refer to the discussion of the
judgment of the Constitutional Court in Carmichele v Minister of Safety and Security (Centre for
Applied Legal Studies Intervening) 2001 (4) SA 938 (CC) in Neethling, Potgieter and Visser
Neethling-Potgieter-Visser Law of Delict page 67 footnote 214.

Total for assignment 01: [10]

Answer
Facts of the case
Please note that no marks were awarded for providing the facts of the case. The facts are
provided hereunder merely as background information.
In Stedall and Another v Aspeling and Another 2018 (2) SA 75 (SCA), the respondents’ (the
Aspelings’) two and a half year-old daughter C, was involved in a swimming-pool accident on 27
July 2004. The accident occurred at the appellants’ (the Stedalls’) home in Constantia, Cape
Town. C’s mother had left her unattended for a short period of time. During that time, C had made
her way to the swimming pool and was later found floating, face down, in the swimming pool. C
did not drown but suffered severe, permanent brain damage. The Aspelings then sued the
Stedalls for their and C’s damages. The court a quo found the Stedalls twice as culpable as C’s
mother, in that they failed to secure the swimming pool’s gates. The court a quo also found C’s
mother negligent in failing to constantly watch over her. The Stedalls then appealed to the
Supreme Court of Appeal (hereinafter referred to as the “SCA”) disputing delictual liability on their
part (paras 1, 9, 11).

(a) Discussion of what the court decided in respect of the wrongfulness element
Please note that a maximum of 5 marks were awarded for the discussion on what the court
decided in respect of the wrongfulness element.
The SCA held that the court a quo overlooked the requirement that wrongfulness is an essential
and distinct element required for delictual liability (paras 11, 13) [1].
The court referred to Country Cloud Trading CC v MEC, Department of Infrastructure
Development, Gauteng (2015 (1) SA 1 (CC) paras 20-21) and Le Roux v Dey (Freedom of
Expression Institute and Restorative Justice Centre as Amici Curiae) (2011 (3) SA 274 (CC) para
122) where the Constitutional Court held that conduct will be considered wrongful if it would be
reasonable to impose liability on a defendant for the harm suffered [1] and that reasonableness
depends on considerations of public policy, legal policy and constitutional norms [1].
Wrongfulness and negligence should not be conflated (para 13) [1]. The court stated (at para 14)
that the SCA had previously pronounced that “foreseeability of harm”, an essential requirement
for negligence, should not be a requirement in determining wrongfulness (see Country Cloud
Trading CC v MEC, Department of Infrastructure Development 2014 (2) SA 214 (SCA) para 27,
and MTO Forestry (Pty) Ltd v Swart N.O. 2017 (5) SA 76 (SCA) para 18) [1].
A negligent omission, is not necessarily regarded as prima facie wrongful (para 15) [1]. A
negligent omission is regarded as wrongful only “if it occurs in circumstances that the law regards
as sufficient to give rise to a legal duty to avoid negligently causing harm” (see Minister of Safety
and Security v Van Duivenboden 2002 (6) SA 431 (SCA) para 12) [1]. With regard to an alleged
negligent omission, the respondents should not only have alleged that the negligent omissions
had been wrongful, but should also plead and prove the facts on which they rely on for holding
the omission wrongful (para 17-19) [1].
A “legal duty” in this context means that the omission must not be wrongful and involves public
and legal policy, consistent with constitutional norms (see Hawekwa Youth Camp v Byrne 2010
(6) SA 83 (SCA) para 22) [1]. A “legal duty” must not be confused with the English law concept of
“a duty of care” which encompasses both wrongfulness and negligence (para 16) [1].
The question was whether the failure to secure the swimming pool gates, was indeed wrongful in
the particular circumstances [1]. The pertinent circumstances were: a parent brought their child to
another’s private premises on a visit; aware that there was a swimming pool on the premises;
supervised the child; became momentarily distracted; and during that time the child wandered off,
fell into the swimming pool, and sustained brain damage [2]. The Court held that the following
factors must be considered: constitutional norms, in particular the best interests of the child [1];
whether the failure to ensure that the gate was secured evokes moral indignation [1]; whether the
legal convictions of the community demand that it be regarded as wrongful [1]; and whether it
would be overly burdensome to impose liability (para 22) [1].

The court referred to a number of foreign judgments, drawing attention to the fact that in those
cases where young children are in the care and under the supervision of their parents whilst
visiting the home of another, the duty to keep the child safe lies with the parents [1]. Public and
legal policy of these foreign countries reflect that it would be unreasonable to impose liability on
the homeowner and expect a homeowner to provide greater supervision than a parent (paras 29-
32) [1].
The court held that the Aspelings failed to establish the element of wrongfulness on the part of the
Stedalls and their claim had to fail on that basis alone. The appeal succeeded since the Stedalls’
conduct was not wrongful (para 33) [1].

(b) Neethling and Potgieter’s view on how wrongfulness should be established


Please note that a maximum of 5 marks were awarded for providing your own comparison.
Wrongfulness is a necessary, distinct requirement for delictual liability [1] which should be
determined ex post facto, that is, taking into account all the facts and circumstances present and
consequences that actually ensued (see Neethling, Potgieter and Visser Neethling-Potgieter-
Visser Law of Delict Chapter 3 para 1 ─ from here on, only the paragraph numbers will be
referred to from Chapter 3) [1].
In cases of liability for an omission, wrongfulness is determined by questioning whether according
to the boni mores (legal convictions of the community), the defendant had a legal duty to prevent
harm (see Van Eeden v Minister of Safety and Security (Women’s Legal Centre Trust, as amicus
curiae) 2003 (1) SA 389 (SCA) 395) [1]. There is no general duty to prevent harm by positive
conduct [1]. One must determine according to the circumstances of the case whether there was a
legal duty to act positively [1]. If it is found that the defendant had a legal duty and failed to
comply with that duty, without the applicability of a ground of justification, then that failure is
unreasonable, contra bonos mores and therefore wrongful [2] (see paras 5.1-5.2).
There are certain factors which have come to the fore over time and are indicative of a legal duty
to act positively. These are: prior positive conduct (para 5.2.1); control of a dangerous object
(para 5.2.2); knowledge and foresight of possible harm (para 5.2.3); rules of law (para 5.2.4); a
special relationship between the parties (para 5.2.5); assumption of a particular office (para
5.2.6); a contractual undertaking in respect of the safety of a third party (para 5.2.7); and the
creation of an impression that another will be protected (para 5.2.8) [maximum 3 marks for
mentioning the different factors]. There may be an interplay of the abovementioned various
factors which may indicate that the omission is wrongful (para 5.2.9) [1]. In the end we are
concerned with whether or not the defendant’s failure to act in the particular circumstances was
reasonable [1] and a balancing or weighing of interests takes place (para 5.2.10) [1].
The test for wrongfulness should not be reduced to a “legal duty not to act negligently” as this
creates the impression that the legal duty deals with the question of whether the defendant acted
negligently, thereby conflating wrongfulness and negligence [1]. This formulation of the test for
wrongfulness mirrors the English law “duty of care” concept which in effect combines the test for
wrongfulness and negligence and which was expressly rejected by the SCA (para 5.1) [1].
Most of what has been mentioned above by Neethling and Potgieter in determining wrongfulness
is essentially what the court did in fact reiterate in Stedall v Aspeling. For example, both the
judgment of Stedall v Aspeling and Neethling and Potgieter’s views are aligned in stating that
wrongfulness is a separate and distinct element; wrongfulness in cases of omission is determined
by establishing whether there was a breach of a legal duty; certain factors may play a role in
establishing wrongfulness; wrongfulness must be determined by considering all the
circumstances present; the boni mores, the Constitution, public and legal policy must be
considered; wrongfulness and negligence should not be conflated; and a breach of a legal duty is
not the same as a breach of the English duty of care [maximum 3 marks for expressly pointing
out the similarities]. Neethling and Potgieter however, do not agree with the courts recent
approach to determining wrongfulness, in that wrongfulness consists in the reasonableness of
holding a defendant liable. They are very critical of this approach and provide five reasons why it
should be rejected (see para 6) [1].
In addition, we may point out that Neethling and Potgieter accept in their textbook (para 5.2.9)
that foreseeability of prejudice can be a factor influencing wrongfulness, but in a more recent
contribution they agree with the SCA’s current view that foreseeability of harm should not play a
role in the wrongfulness inquiry (Neethling and Potgieter “Foreseeability: wrongfulness and
negligence of omissions in delict – the debate goes on” 2018 TRW 154-155).

[Maximum marks: 10]

1.2 ASSIGNMENT 02 (MULTIPLE CHOICE ASSIGNMENT)

COMMENTARY
ASSIGNMENT 02: MULTIPLE CHOICE QUESTIONS
UNIQUE ASSIGNMENT NUMBER: 528927

Question 1

1. Which one of the following conditions may amount to automatism?


1. Provocation.
2. Self-defence.
3. Black-out.
4. Necessity.
(1)

The correct alternative is [3]. It is accepted that a black-out may cause a person to act
involuntarily. Alternatives 1, 2 and 4 are generally considered as grounds of justification which
negate the element of wrongfulness. See Neethling, Potgieter and Visser Neethling-Potgieter-
Visser Law of Delict Chapter 2 para 3; Chapter 3 para 7; Study Guide, study units 3, 9-11.

Question 2

Choose the correct statement. In Carmichele v Minister of Safety and Security (Centre for
Applied Legal Studies Intervening) 2001 (4) SA 938 (CC) the Constitutional Court:
1. declared that the courts of lower instance had erred by applying a pre-constitutional concept
of the boni mores.
2. noted that the law of delict admirably reflected the spirit, purport and object of the Bill of
Rights.
3. recognised a claim for Constitutional damages.
4. recognised the concept of a Constitutional delict.

(1)
The correct alternative is [1]. See Neethling, Potgieter and Visser Neethling-Potgieter-Visser
Law of Delict Chapter 3 para 5.2.4 fn 214.

Question 3

Dr Berenice, a well-known obstetrician, delivered Bongi’s son, Kabelo. During delivery, the
nerves in Kabelo’s right shoulder were injured resulting in the paralysis of his arm. All of this
happened because Dr Berenice failed to inform Bongi of the potential complications inherent in
delivering a large baby. Indicate the most correct statement: Negligence will be determined
according to the standard of:
1. the reasonable doctor.
2. the reasonable person.
3. the reasonable man.
4. the reasonable child.
(1)

The correct alternative is [1]. In this scenario the wrongdoer is an expert, therefore the test is
adjusted and raised to that of the reasonable expert, that is, the reasonable doctor. See
Neethling, Potgieter and Visser Neethling-Potgieter-Visser Law of Delict Chapter 4 para 4.5.3;
Study Guide, study unit 16.

Question 4
Musa is away on holiday. Frank, his neighbour, is keeping a watchful eye on his house for him
and undertook to take care of Musa’s cat. Frank notices that the house is on fire and realises
that Musa’s cat is trapped in the burning house. Frank breaks down the front door of Musa’s
house in order to save the cat. If Musa wants to institute a delictual action against Frank for
breaking down his front door, Frank may rely on the following ground of justification:
1. statutory authority.
2. necessity.
3. provocation.
4. private defence.
(1)

The correct alternative is [2]. In this scenario, the defences of private defence, provocation
and statutory authority are not applicable. Necessity exists when the defendant is placed in
such a position that he or she is able to protect an interest (his or her own legally recognisable
interest or that of someone else) only by reasonably violating the interests of another person.
Frank acted out of necessity in protecting the life of the cat. All the requirements for necessity
are present. See Neethling, Potgieter and Visser Neethling-Potgieter-Visser Law of Delict
Chapter 3 para 7.3; Study Guide, study unit 10.
Question 5
Dick is so offended by William’s speech at a political rally, that he throws an apple at William.
William falls off the podium and sustains a broken arm as well as a few broken ribs. He is
admitted to hospital. While he is being treated there, a nurse leaves the windows of the ward
open and William contracts pneumonia. Which one of the following is incorrect?
1. Dick acted wrongfully.
2. Dick had fault.
3. There is a factual causal link between Dick’s conduct and William’s pneumonia.
4. There is a legal causal link between Dick’s conduct and William’s pneumonia.
(1)

The correct alternative is [4]. The emphasis here was on the incorrect statement. Dick did
act wrongfully when he threw the apple and he had fault (intention) when he directed the apple
at William. Dicks conduct has a factual causal link to William’s broken arm, ribs and pneumonia.
To determine factual causation, the conditio sine qua non theory (also known as the ‘but for’
test) is applied. With regard to William contracting pneumonia, Dick’s intentional wrongful
conduct was the factual cause but not the legal cause. The nurse’s conduct (leaving the window
open) may be considered a novus actus interveniens. Therefore, alternative 4 is the incorrect
statement and the correct alternative to this question. See Neethling, Potgieter and Visser
Neethling-Potgieter-Visser Law of Delict Chapter 5 para 2-3; Study Guide, study unit 21-22.

Question 6
The most suitable remedy for the prevention of harm is:
1. the actio legis Aquiliae.
2. the actio de effusis vel deiectis.
3. the interdict.
4. mitigation of loss.
(1)

The correct alternative is [3]. The interdict is directed at the prevention of a wrongful act
(impending wrongful act or to prevent the continuation of a wrongful act that has already
commenced) and is therefore the most suitable remedy. The interdict has a preventative
function and as a result, there is no need for the requirement of fault on the part of the
wrongdoer to be present. The requirements that need to be met for the granting of an interdict
are: there must be an act by the respondent, the act must be wrongful, and no other ordinary
remedy which would prevent the wrongful conduct must be available to the applicant. See
Neethling, Potgieter and Visser Neethling-Potgieter-Visser Law of Delict Chapter 7 para 2;
Study Guide, study unit 25.

Question 7

The principle dictating that all past and prospective loss must be claimed together stemming
from a single cause of action, is known as:
1. the sum-formula approach.
2. the ‘once and for all’ rule.
3. compensating advantages.
4. the concrete approach to damage.

(1)

The correct alternative is [2]. The “once and for all” rule expresses that in all claims for
compensation and satisfaction arising out of a delict, the plaintiff must claim damages for all
damage already sustained and all future damages insofar as the claims are based on a single
cause of action. See Neethling, Potgieter and Visser Neethling-Potgieter-Visser Law of Delict
Chapter 6 para 4.7; Study Guide, study unit 24.

Question 8
Which is the odd one out?
1. The adequate causation theory.
2. Normative foreseeability.
3. Direct consequences.
4. The sum-formula approach.
(1)

The correct alternative is [4]. Alternatives 1, 2 and 3 relate to establishing legal causation
while alternative 4 is an approach used in determining the element of harm/loss/damage. See in
general Study Guide, study units 22-24.
Question 9

Eddie and Robie bear a grudge against Greg and agree to teach him a lesson he will not forget.
While Greg is in a shopping centre, Eddie and Robie damage Greg’s motor car by hitting it with
hammers. If Greg successfully sues Eddie and Robie in delict, they will incur:
1. vicarious liability.
2. joint and several liability.
3. liability based on contributory intent.
4. strict liability.
(1)

The correct alternative is [2]. Both Eddie and Robie act together in causing damage to Greg’s
motor car. They are therefore deemed joint wrongdoers and are held jointly and severally liable
for the damage to Greg’s car. See Neethling, Potgieter and Visser Neethling-Potgieter-Visser
Law of Delict Chapter 8 para 1; Study Guide, study unit 26.

Question 10
In which one of the following situations can it be said that publication of defamatory words has
taken place?
1. Two German tourists visit South Africa. They start arguing in German in front of some South
Africans (who do not understand German) and the one tourist calls the other a liar and an
adulterer.
2. Mr Xolani tells his wife, Mrs Xolani, that Vanessa at his office is having an affair with her
secretary.
3. Bob meets Ricky at the local post office and tells Ricky that the manager of the post office,
Mrs Posh, is involved in an adulterous affair with Ricky’s neighbour Henry.
4. Mrs Brown tells her husband, Mr Brown, that Theresa, a work colleague of Mrs Brown, has
stolen money from the cash register on numerous occasions.
(1)

The correct alternative is [3]. A defamatory statement must be disclosed to a third person. In
alternatives 2 and 4, publication of defamatory words has not taken place because an exception
applies in respect of spouses. Publication of defamatory words has also not taken place in
alternative 1 as disclosure of defamatory words were heard by outsiders who are not aware of
the meaning of the defamatory words. See Neethling, Potgieter and Visser Neethling-Potgieter-
Visser Law of Delict Chapter 10 para 3.2.2.1; Study Guide, study unit 29.
TOTAL MARKS: [10]
2 RECENT DEVELOPMENTS

Accountability of a minor
The authors of the prescribed textbook, Neethling and Potgieter Neethling-Potgieter-Visser Law
of Delict in Chapter 4 para 2, state that “the Child Justice Act 75 of 2008 changed the common
law position with regard to the accountability of children”. According to the Child Justice Act, a
child who has not reached his/her ninth year cannot be held accountable, the child is culpae
incapax. A child from ten to fourteen years of age is presumed to lack accountability until the
contrary is proven. A child between fourteen to eighteen years of age is presumed to be
accountable (culpae capax), he/she is considered an adult in respect of accountability.
In terms of the common law, a child who has not reached his/her seventh year cannot be held
accountable. A child between seven and fourteen years of age is presumed to lack
accountability until the contrary is proven. A child between fourteen and eighteen years of age is
presumed to be accountable, he/she is considered an adult in respect of accountability.
Jansen and Neethling (Jansen R-M and Neethling J “Delictual capacity and contributing
negligence of minors” 2017 THRHR 474-482) submit that the Child Justice Act applies to the
accountability of children with regard to crimes. The common law position as stated above still
applies to delictual liability.
There is therefore uncertainty as to whether the Child Justice Act has indeed changed the
common law position. In light of this we advise that you to approach the prescribed textbook
with regard to the accountability of children with caution. In the exam, if you state that the
common law position is applicable or the Child Justice Act is applicable, you will not be
penalised, either will be correct.
1.1 ASSIGNMENT 01 (WRITTEN ASSIGNMENT)

ASSIGNMENT 01
Original due date: 21 August 2020

Unique number: 794118


(Compulsory) for Second Semester

Read the judgment in Life Healthcare Group (Pty) Ltd v Suliman 2019 (2) SA 185 (SCA) and
write a discussion of 500-1000 words. Your discussion should include:
• a discussion of what the court decided in respect of the wrongfulness element only in that case
[a maximum of 5 marks are awarded for the discussion of what the court decided in respect of
the wrongfulness element]; and
• your own comparison of the court’s decision with regards to the element of wrongfulness to the
discussion by Neethling and Potgieter in Neethling-Potgieter-Visser Law of Delict on how
wrongfulness should be established [a maximum of 5 marks are awarded for providing your own
comparison].
You can find the judgment in conventional law libraries, or online by conducting a google search
or at the website of the Southern African Legal Information Institute (SAFLII) at www.saflii.org.
For an example of how a judgment should be summarised, refer to the discussion of the
judgment of the Constitutional Court in Carmichele v Minister of Safety and Security (Centre for
Applied Legal Studies Intervening) 2001 (4) SA 938 (CC) in Neethling, Potgieter and Visser
Neethling-Potgieter-Visser Law of Delict page 67 footnote 214.
Total for assignment 01:[10]

ANSWER
Life Healthcare Group (Pty) Ltd v Suliman 2019 (2) SA 185 (SCA) was a medical negligence
case. The main issue for decision was whether the alleged negligent conduct of a doctor
caused the birth injuries of a child born with cerebral palsy. One bonus mark was awarded for
a satisfactory account of facts of the case.
What the court decided in respect of wrongfulness:
The court stated clearly that in our law a negligent omission is only unlawful if it occurs in
circumstances that the law regards as sufficient to give rise to a legal duty to avoid negligently
causing harm [1] citing Minister of Safety and Security v Van Duivenboden 2002 (6) SA 431
(SCA) para 25 and Oppelt v Head: Health, Department of Health, Provincial Administration:
Western Cape [2015] ZACC 33; 2015 (12) BCLR 1471 (CC). [1 mark for any one of these two
cases or both of them] Therefore the hospital bore the onus to prove that Dr Suliman owed a
legal duty [1] to the patient and that the duty was breached. [1] Dr Suliman contended that Mrs
S was not his patient because he was simply covering for another doctor and he therefore did
not assume normal responsibility for the patient. [1] The court remarked that the only logical
inference from Dr Suliman’s contention was that the patient was practically without a doctor or
specialist taking care of her, and the court was unable to agree with this contention. [1] The
court stated that the legal duty arose when Dr Suliman got involved in the treatment of the
patient [1] by agreeing with the request to cover for the other doctor, by positively responding to
a call from a sister that the patient had been admitted to hospital, by instructions to a nurse to
allow the labour to proceed and to sedate the patient, if necessary, and by prescribing
medication as mentioned earlier. This conduct of getting involved in the treatment of the patient
placed him in a position of being responsible for her and the baby. [1 for any reasonable
account of these points] The next question was whether the legal duty was breached, and the
court answered this question by inquiring whether Dr Suliman acted like a reasonable
obstetrician in the circumstances. [1] Dr Suliman first visited the patient more than 11 hours
after her admission. The explanation given by Dr Suliman was that he did not regard her as his
patient, but the court stated that if this was not gross negligence, then it was difficult to imagine
what would be. [1] [A maximum of 5 marks was awarded for the preceding section]
How the part of the judgment that dealt with wrongfulness compares to the views of the
authors of our textbook:
According to Neethling and Potgieter (p 33) wrongfulness lies in the infringement of a legally
protected interest in a legally reprehensible way. [1] The basic test for wrongfulness is the legal
convictions of the community or boni mores [1] and this test must reflect the values of the
Constitution and the Bill of Rights. [1 for mentioning either Constitution or Bill of Rights] In
instances of an omission, wrongfulness is determined with reference to breach of a legal duty
(Neethling and Potgieter 55, 58). [1] Insofar as the judgment dealt with an omission and
wrongfulness was determined with reference to a legal duty, the judgment is broadly in
agreement with the views of Neethling and Potgieter. [1] The court does not use the new
approach to wrongfulness, which is criticised by Neethling and Potgieter, and according to
which wrongfulness consists of the reasonableness, based on policy considerations, of holding
the plaintiff liable (Neethling and Potgieter 80). [1] The court does not specifically refer to the
boni mores or legal convictions of the community (Neethling and Potgieter 36). [1] The court
also does not refer to specific instances where liability for omission has been recognised by the
courts, which are dealt with in detail by Neethling and Potgieter 60-77. [1] However, several of
those specific instances could conceivably have been relied upon, when the court pointed out
that that the legal duty arose when Dr Suliman got involved in the treatment of the patient. [1]
They are: knowledge and foresight of possible harm (Neethling and Potgieter 65); a special
relationship between the parties (Neethling and Potgieter 69); contractual undertaking for the
safety of a third party (Neethling and Potgieter 71); creation of the impression that the interests
of a third party will be protected (Neethling and Potgieter 72); and interplay of factors (Neethling
and Potgieter 73). [maximum 2 marks for mentioning 2 of these and additional 1 mark for a
plausible substantiation why the particular instance is applicable to the facts of the case]
The court determined the breach of the legal duty by establishing that Dr Suliman had been
negligent. This makes it difficult to know where the wrongfulness inquiry ends and where the
fault (negligence, in this case) inquiry begins. This is similar to the duty of care approach of
English law where wrongfulness and negligence are not treated as distinct and independent
elements of liability. Neethling and Potgieter are usually critical of such an approach (e.g.
Neethling and Potgieter 57, see further 158). [A maximum of 2 was awarded for raising this
point, however this topic crosses over into negligence, which was not part of the
question and which most of students would not yet have studied when doing the
assignment. For this reason, full marks could have been attained without addressing this
point.]
1.2 ASSIGNMENT 02 (MULTIPLE CHOICE ASSIGNMENT)

ASSIGNMENT 02

1. Absolute compulsion is one of the conditions that may cause a person’s behaviour to be
involuntary and hence not to qualify as conduct for the purpose of delictual liability. In which one
of the following instances is Y subject to absolute compulsion?
1. X points a gun at Y and orders Y to damage Z’s motor vehicle. In order to save his own life, Y
dents Z’s motor vehicle.
2. Y suffers an epileptic fit while driving his motor vehicle and in the process smashes into Z’s
motor vehicle.
3. Y is ‘blind drunk’ and lies on the sidewalk, making involuntary movements with his arms and
legs. Z’s motor vehicle is parked next to Y and Y’s involuntary movements cause a dent in Z’s
motor vehicle.
4. X pushes a baseball bat into Y’s hand and then, without Y being able to offer resistance, X
takes hold of Y’s hand and forces it to cause a dent on Z’s motor vehicle.
Answer: 4; see Neethling and Potgieter p 27 (fn 21).

2. Which of the following best describes what is meant by “psychological lesion”?


1. Emotional shock.
2. Negligent misrepresentation.
3. Pure economic loss.
4. Infringement of the right to identity.
Answer: 1; see Neethling and Potgieter 300.

3. Which one of the following qualifies as private defence?


1. John’s vicious dog charges to attack Zain. To protect himself, Zain picks up a baseball bat
and gives the dog a blow to the head.
2. Brian threatens Wilma with a knife and robs her of her cell phone. After three months, Wilma
sees Brian at a shopping centre and reacts by stabbing Brian with a high heeled shoe.
3. A child points a firearm at you and you grab his arm to prevent him from shooting you. The
child sustains injuries to his arm.
4. A police officer arrests Liam in the execution of a legitimate warrant of arrest. Liam resists the
arrest. He jabs the policeman, cracking one of the policeman’s ribs, in an attempt to escape.
Answer: 3; see Neethling and Potgieter 88-97.

4. Neil is chased by a fierce Jersey bull and jumps onto Conrad’s scooter, which is parked next
to the road, in order to race away and save his own life. The scooter is damaged and Conrad
institutes a delictual claim against Neil. Neil may raise the following ground of justification:
1. private defence.
2. necessity.
3. provocation.
4. official capacity.
Answer: 2; see Neethling and Potgieter 97-102.

5. Carl, seventeen, left his PlayStation in his locker at school. James, fifteen, wants to steal
Carl’s PlayStation. He realises that he would have to damage Carl’s locker in order to get the
PlayStation. In respect of the damage to the locker, James has:
1. dolus directus.
2. dolus indirectus.
3. dolus eventualis.
4. none of the above.
Answer: 2; see Neethling and Potgieter 133.

6. Greater Johannesburg Transitional Metropolitan Council v ABSA Bank Ltd 1997 (2) SA 591
(W) dealt with:
1. contributory intention.
2. contributory negligence.
3. consent to injury.
4. consent to the risk of injury.
Answer: 1; see Neethling and Potgieter 169.
7. Joseph plays tennis in his back yard. He foresees the possibility of his ball breaking a window
in his neighbour’s house, but decides that it will not happen. If the ball indeed breaks the
window, Joseph had the following in respect of the damage:
1. dolus indeterminatus.
2. dolus eventualis.
3. dolus indirectus.
4. luxuria.
Answer: 4; see Neethling and Potgieter 134.

8. Gawie and Manie are farming on adjacent properties. Gawie bears a grudge against Manie.
He sets up a device, which causes a loud bang every half an hour, on the common boundary
between the two farms, near Manie’s homestead. As a result of this, Manie and his family
cannot sleep at night and are irritated by day. Manie asks Gawie to dismantle the device, but
Gawie says that the device serves the purpose of scaring baboons away from his orchards.
Which delictual remedy may be available to Manie?
1. The actio de pauperie.
2. The actio de feris.
3. The interdict.
4. The actio de effusis vel deiectis.
Answer: 3; see Neethling and Potgieter 269.

9. Jacob, an employee of Zanele, was on route to Rustenburg delivering bread when he


accidentally skipped a red robot and crashed into Dineo’s car. If Dineo successfully sues Zanele
(as the employer) for the damage to his car, Zanele will incur:
1. joint and several liability.
2. liability based on negligence.
3. vicarious liability.
4. risk liability.
Answer: 3; see Neethling and Potgieter 389-390.
10. Jonas threatens Lerato with a gun and orders her to hand over her cellphone. Lerato, in fear
of her life, hands over the cellphone. One week later, Lerato spots Jonas at a nearby shopping
centre. Lerato rushes home and grabs a knife. She returns to the shopping centre and stabs
Jonas. Jonas wants to institute a delictual action against Lerato. Lerato may rely on the
following ground of justification:
1. necessity.
2. provocation.
3. private defence.
4. none of the above.
Answer: 4; see Neethling and Potgieter 99; 106; 91.
PVL 3703

LAW OF DELICT

2021 ASSIGNMENTS
Question
Read the judgment in Bergrivier Municipality v Van Ryn Beck 2019 (4) SA 127 (SCA)
and write a discussion of 500-1000 words. You can find the judgment in conventional
law libraries, or online by conducting a google search or at the website of the
Southern African Legal Information Institute (SAFLII) at www.saflii.org.

In your discussion, you must:


(a) Discuss what the Supreme Court of Appeal decided in respect of the element of
wrongfulness only in Bergrivier Municipality v Van Ryn Beck 2019 (4) SA 127 (SCA).

In other words, we expect you to explain how the Supreme Court of Appeal came to
the conclusion that the Municipality’s conduct was not wrongful. Please note that no
marks will be awarded for providing the background facts of the case,
Bergrivier Municipality v Van Ryn Beck 2019 (4) SA 127 (SCA) unless they are
relevant to your discussion under point (a) above.

A maximum of 5 marks will be awarded for the discussion on what the court decided
in respect of the wrongfulness element. Therefore a maximum of 5 marks will be
awarded for part (a) of your discussion.

(b) Compare the court’s decision with that of Neethling and Potgieter (the prescribed
book, the Law of Delict) on how wrongfulness should be established. Here we
expect you to briefly lay out the manner in which Neethling and Potgieter
recommend that wrongfulness should be established, particularly in cases of
omissions. With regard to the comparison, you must point out any similarities
between the Supreme Court of Appeal’s principles in establishing wrongfulness and
Neethling and Potgieter’s principles in establishing wrongfulness. You must also
point out any marked differences if any.

A maximum of 5 marks will be awarded for providing your own comparison.

Total for assignment 01: [10]

Answer
Background facts
During April 2011, after heavy rainfall, Mr Van Ryn Beck's home was flooded. This
occurred as a result of the storm-water drainage system near his home being
overwhelmed with excessive rainfall (see paras 1, 6–7 and 15). Mr Van Ryn Beck's
property was flooded on two prior occasions, in December 2007 and in June 2009
(para 8). On both these occasions, the insurance company Mr Van Ryn Beck was
contracted with had covered the damages. After the damage to Mr Van Ryn Beck's
home on the second occasion in 2009, the insurance company was not willing to cover
any further damage to Mr Van Ryn Beck's home due to flooding (para 42). Mr Van Ryn
Beck alleged that the Bergrivier Municipality (the Municipality) had after these two
previous floods inspected his home and undertook to do what was necessary to avert
flooding in the future. This included, he alleged, “improving the storm-water drainage
system and ensuring regular maintenance of catch-pits and trenches” which was not
done (para 13).

Mr Van Ryn Beck sued the Municipality in the High Court alleging that the Municipality
had negligently and wrongfully failed to provide an effective water drainage system, to
maintain it and to effect flood-preventative measures (para 42). The High Court
dismissed the claim concluding that the evidence led did not prove negligence,
wrongfulness or causation (paras 3–4). Mr Van Ryn Beck then appealed to the full
bench which upheld the appeal (see para 5). The full bench found that there was
sufficient evidence “to have placed a legal duty on the respondent municipality to have
ensured that its drainage system was able to cater for flood waters that would have
emanated from the adjacent farmlands. It had not done so” (para 38). The Municipality
then appealed to the Supreme Court of Appeal (SCA). The question before the SCA
was whether the Municipality, should be held liable for the damages allegedly
sustained by Mr Van Ryn Beck as a result of the flooding of his home in 2011 (para
2).

Please note that no marks were awarded for stating the above background facts.

(a) Discussion of what the court decided in respect of wrongfulness

Please note that a maximum of 5 marks were awarded for the discussion on
what the court decided in respect of the element of wrongfulness.

The SCA stated that Mr Van Ryn Beck’s claim against the Municipality was based on
an omission and that liability for an omission will follow only if the omission was in fact
wrongful (para 43) [1 mark]. The omission is wrongful if in the circumstances “a legal
duty rested on a defendant to act positively to prevent harm from occurring and [the]
defendant failed to comply with that duty” (para 43) [2 marks]. The SCA stated that
even though conceptually, the inquiry into wrongfulness may occur before the enquiry
into negligence [1 mark], without negligence the question of wrongfulness does not
arise (para 44) [1 mark]. The conduct will not be wrongful if negligence is absent [1
mark]. In certain instances it may be appropriate to assume that there was a legal
duty and then consider negligence [1 mark] while in certain instances it may be
convenient to assume negligence when trying to establish wrongfulness (para 44) [1
mark].

The court held that even though counsel for both parties were of the view that the
issue in this case was with the element of wrongfulness, the evidence provided was
inadequate to establish not only wrongfulness [1 mark] but also fault or causation.
Therefore the Municipality could not be held delictually liabile (para 45) [1 mark].

Wrongfulness is “determined objectively, taking into account all the relevant facts and
circumstances and the consequences that ensued” (para 50) [1 mark]. In determining
whether an infringement of an interest is unlawful, the legal convictions of the
community are considered [1 mark]. The court referred to Lee v Minister of
Correctional Services 2013 (2) SA 144 (CC) and Minister van Polisie v Ewels 1975 (3)
SA 590 (A) with approval where those courts held that an omission is wrongful when
in the circumstances of the particular case, the legal convictions of the community
would demand that the omission be considered wrongful (para 50) [2 marks]. The
court held that in this case, the Municipality could not improve the water-drainage
system as requested by Mr Van Ryn Beck because the Municipality was restricted by
budgetary and sociological concerns (para 51) [1 mark]. There was a greater need to
use the limited funds that were allocated to the Municipality for the indigent, informal
settlement communities (para 51) [1 mark]. The legal convictions of the community
would not hold the Municipality liable under the circumstances as it was too onerous
a duty placed on it (para 51) [1 mark]. The court held that the full bench erred in
dismissing these concerns but at the same time this does not mean that municipalities
can ignore fulfilling their obligations by stating that they have budgetary constraints
(para 51) [1 mark].

The SCA found that the Municipality’s conduct was not wrongful and upheld the appeal
[1 mark].
[Maximum 5 marks]

(b) Neethling and Potgieter’s view on how wrongfulness should be established

Please note that a maximum of 5 marks were awarded for providing your own
comparison.

Wrongfulness is a necessary, distinct requirement for delictual liability [1 mark] which


should be determined ex post facto, that is, taking into account all the facts and
circumstances present and consequences that actually ensued (see Neethling and
Potgieter Neethling-Potgieter-Visser Law of Delict 7th edition and Neethling and
Potgieter Law of Delict 8th edition Chapter 3 para 1 ─ from here on, only the editions and
paragraph numbers will be referred to from Chapter 3) [1 mark].

In cases of liability for an omission, wrongfulness is determined by questioning whether


according to the boni mores (legal convictions of the community), the defendant had a
legal duty to prevent harm (see Van Eeden v Minister of Safety and Security (Women’s
Legal Centre Trust, as amicus curiae) 2003 (1) SA 389 (SCA) 395) [1 mark] and legal
policy [1 mark]. There is no general duty to prevent harm by positive conduct [1 mark].
One must determine according to the circumstances of the case whether there was a
legal duty to act positively [1 mark]. If it is found that the defendant had a legal duty and
failed to comply with that duty, without the applicability of a ground of justification, then
that failure is unreasonable, contra bonos mores and therefore wrongful [2 marks] (see
7th edition and 8th edition paras 5.1-5.2).

There are certain factors which have come to the fore over time and are indicative of a
legal duty to act positively. These are: prior positive conduct (7th edition and 8th edition
para 5.2.1); control of a dangerous object (7th edition and 8th edition para 5.2.2);
knowledge and foresight of possible harm (7th edition and 8th edition para 5.2.3); rules
of law (7th edition and 8th edition para 5.2.4); a special relationship between the parties
(7th edition and 8th edition para 5.2.5); assumption of a particular office (7th edition and
8th edition para 5.2.6); a contractual undertaking in respect of the safety of a third party
(7th edition and 8th edition para 5.2.7); and the creation of an impression that another will
be protected (7th edition and 8th edition para 5.2.8); danger of limitless liability (8th edition
para 5.2.9); vulnerability to risk of damage (8th edition para 5.2.10) [maximum 3 marks
for mentioning any of the different factors]. There may be an interplay of the
abovementioned various factors which may indicate that the omission is wrongful (7 th
edition para 5.2.9 and 8th edition para 5.2.11) [1 mark]. In the end, we are concerned
with whether or not the defendant’s failure to act in the particular circumstances was
reasonable [1 mark] and a balancing or weighing of interests takes place (7th edition
para 5.2.9 and 8th edition para 5.2.11) [1 mark].

The test for wrongfulness should not be reduced to a “legal duty not to act negligently”
as this creates the impression that the legal duty deals with the question of whether the
defendant acted negligently, thereby conflating wrongfulness and negligence [1 mark].
This formulation of the test for wrongfulness mirrors the English law “duty of care”
concept which in effect combines the test for wrongfulness and negligence and which
was expressly rejected by the SCA (7th edition and 8th edition para 5.1) [1 mark].

A number of factors mentioned above by Neethling and Potgieter in determining


wrongfulness is essentially what the court did in fact reiterate in Bergrivier Municipality
v Van Ryn Beck 2019 (4) SA 127 (SCA). For example, both the judgment of Bergrivier
Municipality v Van Ryn Beck and Neethling and Potgieter’s views are aligned in stating
wrongfulness in cases of omission is determined by establishing whether there was a
breach of a legal duty; wrongfulness is determined objectively; wrongfulness must be
determined by considering all the circumstances present; the legal convictions of the
community must be considered in determining wrongfulness [maximum 3 marks for
expressly pointing out the similarities]. Neethling and Potgieter however, would not
agree that without negligence the question of wrongfulness does not arise [1 mark] or
that conduct will not be wrongful if negligence is absent [1 mark] because they are of
the view that wrongfulness and negligence are separate and distinct elements that
should be established in their own right [1 mark]. Similarly, Neethling and Potgieter
would probably not agree that in certain instances it may be appropriate to assume that
there was a legal duty and then consider negligence [1 mark] or that in certain instances
it may be convenient to assume negligence when trying to establish wrongfulness [1
mark] because they are of the view that each element must be established in their own
right and the one element is not dependant on the other. Neethling and Potgieter do
however agree that at times, depending on the facts of the case, it may be more
convenient to establish the elements in a different order than the conventional order of
establishing the elements for delictual liability (see Neethling and Potgieter Neethling-
Potgieter-Visser Law of Delict 7th edition and Neethling and Potgieter Law of Delict 8th
edition Chapter 4 par 1) [1 mark].
[Maximum 5 marks]
PVL3703 - LAW OF
DELICT
2022
ASSIGNMENTS

Varsity Assist
Question

The municipality of eThekwini is busy with excavations in the business


centre. The Municipal workers do not erect any barrier or warning signs in
the vicinity of the excavation. Mrs Ndlovu, who is near-sighted falls into the
excavations and sustains serious injuries. She wishes to institute a delictual
action against the Municipality.

Write an opinion , properly substantiated with reference to case law, only on


wrongfulness of the conduct of the municipality of the eThekwini.

This question deals with the wrongfulness of an omission. An omission is wrongful


if the defendant is under a legal duty to act positively to prevent the harm suffered
by the plaintiff.1

The basic question to determine whether an omission is wrongful is whether a


legal duty to act was present and was breached. As a general rule, a person does
not act wrongfully for the purposes of the law of delict if he omits to prevent harm
to another person.2 For liability to follow an act, prejudice must be caused in a
wrongful and unreasonable manner. Without wrongfulness the defendant cannot
be held liable.

The legal convictions of the community (boni mores test) are used as the basic
test for wrongfulness. The general norm to see if an infringement of interests is
unlawful is the legal convictions of the community. Factors which may serve as
indications that a legal duty rested on the defendant include:

➢ prior conduct (omissio per commissionem)


➢ control of a dangerous object
➢ rules of law
➢ a special relationship between the parties
➢ particular office
➢ contractual undertaking for the safety of a third party

1 Neethling, Potgieter Law of Delict 8th ed Chapter 3 Pg 61.


2 Neethling, Potgieter Law of Delict 8th ed Chapter 3 Pg 64.

Varsity Assist
➢ and creating of an impression that the interests of a third person will be
protected.

We will look at PRIOR CONDUCT more closely.

A person acts prima facie wrongfully when he creates a new source of danger and
then fails to eliminate that danger, with the result that harm is caused to another
person. Prior conduct is not a prerequisite for the existence of a legal duty,
however at one stage this was the only category where liability was imposed on a
failure to act.

The Municipality cases are closely linked to this scenario.

In Halliwell v Johannesburg Municipality;3 the municipality laid cobblestones


inJohannesburg’s city centre but failed to maintain those stones. When Mr
Halliwell crossed the cobblestone path with his horse carriage on a Christmas
morning, his horse stumbled on the poorly maintained road and Mr Halliwell was
hurt. The court held that the municipality was liable for the failure to maintain the
cobblestone road which led to Mr Halliwell’s injuries. This was explicitly done on
the basis of the prior-conduct rule. This rule was also employed in the case of
Silva’s Fishing Corporation v Maweza,4 where the owner of a boat sent fishermen
out to sea (the prior positive conduct) but when the boat stopped working, the
owner did nothing to save the fishermen (the subsequent omission). The boat
owner was held liable for his omission.

In Regal v African Superslate,5 the prominence of the prior-conduct rule as the


only ground on which liability for an omission could be established ended. In the
judgment it was noted that the prior-conduct rule was not the only basis on which
an omission would be actionable. For the first time, the Appellate Division noted
that the prior-conduct rule and the rule related to the control of dangerous property
are both grounds on which the wrongfulness of an omission can be established.

3 Halliwell v Johannesburg Municipality 1912 AD 659.


4 Silva’s Fishing Corporation (Pty) Ltd v Maweza 1957 (2) SA 256 (A).
5 Regal v African Superslate (Pty) Ltd 1963 (1) SA 102 (A).
Varsity Assist
In Regal it was showed that the conduct element could be satisfied by simply
showing the factual existence of an act or an omission. The wrongfulness of the
omission was then established by means of the prior-conduct rule or the rule
related to the control of dangerous property. The principle from Regal was later
also applied in the case of Minister of Forestry v Quathlamba;6 where a fire started
on one property, the owner of that property failed to control or contain the fire, and
the fire spread to a neighbouring property. This re-inforced the Regal decision that
prior conduct is not the only criterion for establishing a legal duty.

An extension of the rules related to the wrongfulness of omissions featured in the


case of Minister van Polisie v Ewels.7 In that case police officers passively stood
by and watched as a fellow police officer brutally assaulted a civilian. The court
held that those two rules as shown in prior cases are just two of many factors to
consider in determining the wrongfulness of omissions. In Ewels, considerations
that weighed in favour of a finding of wrongfulness of the omission of the passive
police officers included the legal duty placed on the police to protect citizens from
harm, the special relationship between police officers and the public, and the fact
that the passive police officers were in a position to exercise authority or control
over their assaultive colleague.

In Carmichele v Minister of Safety and Security8 (Centre for Applied Legal Studies
Intervening) 2001, the CC made it clear that the boni mores must now be informed
by the values underpinning the Bill of Rights in the Constitution. 9

APPLICATION TO THE FACTS

Looking at the above principles and the the given facts, we can conclude that the
omission by the municipality was wrong. The municipality had a legal duty in
terms of the legal convictions of the community to prevent this accident from
happening and the municipality is indeed wrongful.

6 Minister of Forestry v Quathlamba 1973 (3) SA 69 (A).


7 Minister van Polisie v Ewels 1975 3 (SA) 590 (A).
8 Carmichele v Minister of Safety and Security 2001 (1) SA 489 (SCA).
9 Constitution of South Africa,1996.
Varsity Assist
BIBLIOGRAPHY

Books

Neethling, Potgieter, Law of Delict


Neethling J, Potgieter GM, Law of Delict (8th edn Lexis Nexis 2020)

Legislation

Constitution of the Republic of South Africa, 1996.

Case Law

1. Halliwell v Johannesburg Municipality 1912 AD 659.


2. Silva’s Fishing Corporation (Pty) Ltd v Maweza 1957 (2) SA 256 (A).
3. Regal v African Superslate (Pty) Ltd 1963 (1) SA 102 (A).
4. Minister of Forestry v Quathlamba 1973 (3) SA 69 (A).
5. Minister van Polisie v Ewels 3 (SA) 590 (A) 1975.
6. Carmichele v Minister of Safety and Security 2001 (1) SA 489 (SCA).

Varsity Assist
ACADEMIC DECLARATION OF HONESTY

Declaration:

1. I understand what academic dishonesty entails and am aware of UNISA’s policies


in this regard.

2. I declare that this assignment is my own, original work. Where I have used
someone else’s work, I have indicated this by using the prescribed style of
referencing. Every contribution to, and quotation in this assignment from the work
or works of other people has been referenced according to this style.

3. I have not allowed and will not allow anyone to copy my work with the intention
of passing it off as his or her own work.

4. I did not make use of another student’s work and submitted it as my own.

NAME:

STUDENT NUMBER:

Varsity Assist
MODULE CODE:

SIGNATURE:

Varsity Assist
PVL3703 - LAW OF DELICT
2022 – SEMESTER 1
ASSIGNMENT 2
WITH COMPLETE REFERENCES, FOOTNOTES
AND BIBLIOGRAPHY
Question
At the Shop-Till-You-Drop Shopping Mall, hand sanitizer dispensers have been installed at all
the pedestrian entrances…. Notices have been placed near the dispensers, in which
customers are urged to sanitize their hands at the dispensers before entering the Mall. At
Entrance B, the dispenser has been malfunctioning for half a week. It spills sanitizing liquid
onto the floor, in such large quantities that the liquid does not evaporate immediately but
forms a small puddle around the base of the dispenser. Because the lighting at Entrance B is
on the dim side, the puddle is not readily visible unless one specifically looks out for it.

The Management of the Mall is aware of the situation….but does not do anything about it. Ms
C is an elderly lady whose eyesight is not very good, and she is not particularly steady on
her legs, but she loves shopping. She arrives at Entrance B, reads the notice, and proceeds
to sanitize her hands. This requires her to put her one foot on a pedal and push down on it to
activate the dispenser. As she attempts to do this, she slips in the puddle and falls. She
sustains serious injuries and is hospitalised.

After a long and costly bout in hospital, she is discharged.. Ms C wishes to institute an action
for damages against the management of the Shop-Till-You -Drop Shopping Mall and
approaches you for legal advice.

Write an opinion, properly substantiated with reference to case law, only on the
WRONGFULNESS of the conduct of the management of The -Shop-Till-You-Drop Shopping
Mall.

This question deals with the wrongfulness of an omission. An omission is wrongful


if the defendant is under a legal duty to act positively to prevent the harm suffered
by the plaintiff.1 The basic question to determine whether an omission is wrongful
is whether a legal duty to act was present and was breached. As a general rule, a

1 Neethling, Potgieter Law of Delict 8th ed Chapter 3 Pg 61.

Varsity Assist
person does not act wrongfully for the purposes of the law of delict if he omits to
prevent harm to another person.2 For liability to follow an act, prejudice must be
caused in a wrongful and unreasonable manner. Without wrongfulness the
defendant cannot be held liable.

The legal convictions of the community (boni mores test) are used as the basic
test for wrongfulness. The general norm to see if an infringement of interests is
unlawful is the legal convictions of the community. Factors which may serve as
indications that a legal duty rested on the defendant include:

➢ prior conduct (omissio per commissionem)


➢ control of a dangerous object
➢ rules of law
➢ a special relationship between the parties
➢ particular office
➢ contractual undertaking for the safety of a third party
➢ and creating of an impression that the interests of a third person will be
protected.

We will look at PRIOR CONDUCT more closely.

A person acts prima facie wrongfully when he creates a new source of danger and
then fails to eliminate that danger, with the result that harm is caused to another
person. Prior conduct is not a prerequisite for the existence of a legal duty,
however at one stage this was the only category where liability was imposed on a
failure to act.

The Municipality cases are closely linked to this scenario.

In Halliwell v Johannesburg Municipality;3 the municipality laid cobblestones


inJohannesburg’s city centre but failed to maintain those stones. When Mr
Halliwell crossed the cobblestone path with his horse carriage on a Christmas
morning, his horse stumbled on the poorly maintained road and Mr Halliwell was
hurt. The court held that the municipality was liable for the failure to maintain the

2 Neethling, Potgieter Law of Delict 8th ed Chapter 3 Pg 64.


3 Halliwell v Johannesburg Municipality 1912 AD 659.

Varsity Assist
cobblestone road which led to Mr Halliwell’s injuries. This was explicitly done on
the basis of the prior-conduct rule. This rule was also employed in the case of
Silva’s Fishing Corporation v Maweza,4 where the owner of a boat sent fishermen
out to sea (the prior positive conduct) but when the boat stopped working, the
owner did nothing to save the fishermen (the subsequent omission). The boat
owner was held liable for his omission.

In Regal v African Superslate,5 the prominence of the prior-conduct rule as the


only ground on which liability for an omission could be established ended. In the
judgment it was noted that the prior-conduct rule was not the only basis on which
an omission would be actionable. For the first time, the Appellate Division noted
that the prior-conduct rule and the rule related to the control of dangerous property
are both grounds on which the wrongfulness of an omission can be established.

In Regal it was showed that the conduct element could be satisfied by simply
showing the factual existence of an act or an omission. The wrongfulness of the
omission was then established by means of the prior-conduct rule or the rule
related to the control of dangerous property. The principle from Regal was later
also applied in the case of Minister of Forestry v Quathlamba;6 where a fire started
on one property, the owner of that property failed to control or contain the fire, and
the fire spread to a neighbouring property. This re-inforced the Regal decision that
prior conduct is not the only criterion for establishing a legal duty.

An extension of the rules related to the wrongfulness of omissions featured in the


case of Minister van Polisie v Ewels.7 In that case police officers passively stood
by and watched as a fellow police officer brutally assaulted a civilian. The court
held that those two rules as shown in prior cases are just two of many factors to
consider in determining the wrongfulness of omissions. In Ewels, considerations
that weighed in favour of a finding of wrongfulness of the omission of the passive
police officers included the legal duty placed on the police to protect citizens from
harm, the special relationship between police officers and the public, and the fact

4 Silva’s Fishing Corporation (Pty) Ltd v Maweza 1957 (2) SA 256 (A).
5 Regal v African Superslate (Pty) Ltd 1963 (1) SA 102 (A).
6 Minister of Forestry v Quathlamba 1973 (3) SA 69 (A).
7 Minister van Polisie v Ewels 1975 3 (SA) 590 (A).
Varsity Assist
that the passive police officers were in a position to exercise authority or control
over their assaultive colleague.

In Carmichele v Minister of Safety and Security8 (Centre for Applied Legal Studies
Intervening) 2001, the CC made it clear that the boni mores must now be informed
by the values underpinning the Bill of Rights in the Constitution. 9

APPLICATION TO THE FACTS

Looking at the above principles and the the given facts, we can conclude that the
omission of the Centre Management of the shopping mall was wrong.
Management at the Shopping Centre had a legal duty in terms of the legal
convictions of the community to prevent this accident from happening.

8 Carmichele v Minister of Safety and Security 2001 (1) SA 489 (SCA).


9 Constitution of South Africa,1996.

Varsity Assist
BIBLIOGRAPHY

Books

Neethling, Potgieter, Law of Delict


Neethling J, Potgieter GM, Law of Delict (8th edn Lexis Nexis 2020)

Legislation

Constitution of the Republic of South Africa, 1996.

Case Law

1. Halliwell v Johannesburg Municipality 1912 AD 659.


2. Silva’s Fishing Corporation (Pty) Ltd v Maweza 1957 (2) SA 256 (A).
3. Regal v African Superslate (Pty) Ltd 1963 (1) SA 102 (A).
4. Minister of Forestry v Quathlamba 1973 (3) SA 69 (A).
5. Minister van Polisie v Ewels 3 (SA) 590 (A) 1975.
6. Carmichele v Minister of Safety and Security 2001 (1) SA 489 (SCA).

Varsity Assist
ACADEMIC DECLARATION OF HONESTY

Declaration:

1. I understand what academic dishonesty entails and am aware of UNISA’s policies


in this regard.

2. I declare that this assignment is my own, original work. Where I have used
someone else’s work, I have indicated this by using the prescribed style of
referencing. Every contribution to, and quotation in this assignment from the work
or works of other people has been referenced according to this style.

3. I have not allowed and will not allow anyone to copy my work with the intention
of passing it off as his or her own work.

4. I did not make use of another student’s work and submitted it as my own.

NAME:

STUDENT NUMBER:

MODULE CODE:

SIGNATURE:

Varsity Assist
PVL 3703

LAW OF DELICT

MCQ HEAVEN
Question 1
The delictual remedy used to claim damages for patrimonial loss caused wrongfully and
negligently is the:

1. actio legis Aquiliae


2. actio iniuriarum
3. action for pain and suffering
4. interdict
(1)

Patrimonial loss caused wrongfully and culpably is actionable with the Aquilian action.

The correct alternative is [1]. [see chap 1 par 4.2]

Question 2
The delictual remedy used to obtain a solatium for intentional infringement of personality
rights is the:

1. actio legis Aquiliae


2. actio iniuriarum
3. action for pain and suffering
4. interdict
(1
)

The delictual remedy used to claim a solatium for intentional infringement of personality rights
is the actio iniuriarum.

The correct answer is therefore alternative [2]. [see chap 1 par 4.3]

Question 3
The delictual remedy used to prevent wrongful causing of harm is the:

1. actio legis Aquiliae


2. actio iniuriarum
3. action for pain and suffering
4. interdict
(1
)

The delictual remedy used to prevent wrongful causing of harm is the interdict.
Therefore the correct alternative is [4]. [see study guide p 11 and chap 7 par 2]

Question 4
The delictual remedy used to claim compensation for negligent infringement of the
corpus is the:

1. actio legis Aquiliae


2. actio iniuriarum
3. action for pain and suffering
4. interdict
(1
)

The delictual remedy used to claim compensation for negligent infringement of the corpus is
the action for pain and suffering

The correct alternative is [3]. [see chap 1 par 4.4]

Question 5
A delictual remedy with which damages can be prevented without proving fault is the:

1. actio legis Aquiliae


2. actio iniuriarum
3. action for pain and suffering
4. interdict
(1
)

An interdict is directed at the prevention of a wrongful act, and not at the retribution for
wrongfulness already committed, fault is therefore not a requirement.

The correct alternative is [4]. [see chap 7 par 2]


Question 6
When the so-called open-ended delictual norms are given content in the light of the
basic values of chapter 2 of the Constitution, this process is known as:

1. direct application of the Bill of Rights


2. indirect application of the Bill of Rights
3. vertical application of the Bill of Rights
4. a constitutional delict
(1
)

The process of subjecting open-ended or flexible delictual principles to the Bill of Rights, and
giving such principles content in the light of basic values of the Bill of Rights, is known as
indirect application of the Bill of Rights.
The correct answer is therefore alternative [2]. [see chap 1 par 5(b)]

Question 7
The principle dictating at what stage prospective loss must be claimed is known as:

1. the sum-formula approach


2. the “once and for all” rule
3. compensating advantages
4. mitigation
(1
)

In a claim for compensation, the plaintiff must claim damages for all damage already
sustained as well as that expected in the future. This principle is known as the “once and for
all” rule, and it effectively dictates the stage at which prospective loss must be claimed.
The correct alternative is [2]. [see chap 6 par 4.7.1 and 4.6]

Question 8
Prospective loss is best assessed in accordance with:

1. the sum-formula approach


2. the “once and for all” rule
3. compensating advantages
4. mitigation
(1
)

The sum formula approach refers to a hypothetical (potential) patrimonial position of the
plaintiff in order to provide for the assessment of prospective damage.

The correct alternative is [1]. [see chap 6 par 4.5.1]

Question 9
Defamation is in the first place an infringement of a person’s:

1. bodily integrity
2. good name
3. privacy
4. feelings
(1
)

Defamation is the wrongful, intentional infringement of another person’s right to his/her


good name.

The correct alternative is [2]. [see chap 10 par


3.2.1] Question 10

If a person can differentiate between right and wrong and act in accordance with this
insight, he or she:
1. is in a state of automatism
2. can rely on a ground of justification
3. has intent
4. is accountable
(1
)

A person is accountable if he/she has the necessary mental ability to distinguish between right
and wrong and if he/she can also act in accordance with such appreciation.
The correct alternative is [4]. [see chap 4 par 2]
Question 1
Indicate the correct statement with regard to delictual remedies.

1. The actio iniuriarum is directed at ““satisfaction”” for the wrongful and intentional
injury to personality.
2. Intention is always a requirement for the actio legis Aquiliae.
3. South African law follows the casuistic approach with regard to delictual liability.
4. “ “Invasion of privacy” ” in practice is considered as a formdamnumof iniuria datum.
(1)

The correct alternative is [1].

Question 2
Which one of the following is not a requirement for an interdict?

1. There must be no other remedy available to the applicant.


2. There must be a “clear right’
3. “
There must be an infringement or a threat of an infringement of a clear right.

4. Intention or negligence must be present.


(1)

The correct answer is [4].

Question 3

Indicate the correct statement with regard to the concurrence of delictual, criminal and contractual
liability.

1. A claim for damages is the primary remedy for breach of contract.


2. One and the same act may render the wrongdoer delictually as well as contractually
liable.
3. One and the same act cannot establish delictual as well as criminal liability.
4. A claim for damages is the secondary remedy in respect of a crime.
(1)

The correct alternative is [2].

Question 4
Indicate the incorrect statement with regard to the law of delict and the Constitution.

1. An infringement of a right may constitute a constitutional wrong and a delict.


2. The requirements for a delict and a constitutional wrong differ materially.
3. An infringement of a right cannot constitute a constitutional wrong and a delict.
4. Indirect application means that the state must generally respect fundamental rights
and not infringe them.
(1)

Unfortunately a mistake slipped in here. [3] and [4] are both incorrect, and therefore we gave each student
who completed this assignment a mark for this question. We apologise for any inconvenience caused by this.

Question 5

Which one of the following is a requirement for a successful reliance on “ “


private
defence””?

1. There must be fault on the part of the aggressor.


2. The attack must be directed at the defender.
3. The attack must be wrongful.
(1)

The correct alternative is [3].

Question 6
Indicate the incorrect statement with regard to intent.

1. According to Neethling and Potgieter, consciousness of wrongfulness is a requirement for


intent.
2. Dolus eventualis exists where the wrongdoer directly wills one consequence of his
conduct but at the same time acknowledges that another consequence will unavoidably
occur.
3. Only an accountable person can act with intent.
4. Dolus indeterminatus exists where the wrongdoer’’s will is directed at the result
which he causes while he has no specific person or object in mind.
(1)

The correct answer is [2].

Question 7
Which of the following requirement/s must be present before provocation may be raised as
a defence?

1. The conduct of the defendant must be immediate.


2. The defendant must be accountable.
3. The defendant must not have contributed to the provocative conduct.
4. 2 and 3 above.
(1)

The correct alternative is [1].


Question 8
Which of the following acts is not an iniuria (infringement of a personality right)?

1. Defamation.
2. Negligent misrepresentation.
3. Invasion of privacy.
4. Adultery.
(1)

The correct alternative is [2].

Question 9
In which one of the following remedies is there no need to prove damage:

1. Interdict.
2. Actio de pastu.
3. Action for pain and suffering.
4. Actio de pauperie.
(1)

The correct alternative is [1].

Question 10
For which one of the following remedies is fault not a requirement?

1. Action for pain and suffering.


2. Actio de pauperie.
3. Actio legis Aquiliae.
4. Actio iniuriarum.
(1)

The correct alternative is [2].


TOTAL MARKS: [10]

13
Question 1
Andrew inadvertently damages Ben’s violin. Which delictual action may be available to Ben?
1. actio legis Aquiliae
2. actio iniuriarum
3. action for pain and suffering
4. actio de pastu
5. none of the above
(1)

The correct alternative is [1].

Question 2

James employs electronic equipment to listen in on Karl’s telephone conversations. Which


remedy or remedies may be available to Karl?
1. actio legis Aquiliae
2. actio iniuriarum
3. actio de pauperie
4. actio de effusis vel deiectis
5. none of the above
(1)

The correct answer is [2].

Question 3

Mark inadvertently pushes over a ladder on which Chris is standing. Chris breaks an arm.
Which remedy or remedies may be available to Chris?
1. action for pain and suffering
2. actio iniuriarum
3. action for pain and suffering and actio iniuriarum
4. action for pain and suffering and actio de effusis vel deiectis
5. none of the above
(1)
The correct alternative is [1].
Question 4
Jors allows Maans to fire some practice shots with his new pistol at an apple on Jors’s head.
The second shot makes a hole in Jors’s left ear. Jors institutes a delictual action against
Maans. Maans may rely on following defence:
1. private defence
2. execution of an official command
3. provocation
4. consent to the risk of injury
5. none of the above
(1)

The correct alternative is [5].

Question 5
The principle dictating at what stage prospective loss must be claimed, is known as:
1. the sum-formula approach
2. the “once and for all” rule
3. compensating advantages
4. the concrete approach to damage
5. res inter alios acta
(1)

The correct alternative is [2].

Question 6
According to the following case the onus is on the plaintiff to prove that the defendant acted
voluntarily, rather than on the defendant to prove the presence of automatism:
1. Ex parte Minister van Justisie: in re S v Van Wyk 1967 1 SA 488 (A)
2. Molefe v Mahaeng 1999 1 SA 562 (SCA)
3. National Media Ltd v Bogoshi 1988 4 SA 1196 (SCA)
4. Carmichele v Minister of Safety and Security 2001 4 SA 938 (CC)
5. S v Goliath 1972 3 SA 1 (A)
(1)

The correct answer is [2].


Question 7
The following case is regarded as authority for the proposition that killing a person to
protect property may be in certain circumstances be justified by private defence:

1. Ex parte Minister van Justisie: in re S v Van Wyk 1967 1 SA 488 (A)


2. Molefe v Mahaeng 1999 1 SA 562 (SCA)
3. National Media Ltd v Bogoshi 1988 4 SA 1196 (SCA)
4. Carmichele v Minister of Safety and Security 2001 4 SA 938 (CC)
5. S v Goliath 1972 3 SA 1 (A)
(1)

The correct alternative is [1].

Question 8
Alex incites Bert’s dog to bite Charles. Charles kills the dog with his golf club. Bert institutes a
delictual claim against Charles. What defence may be available to Charles?
1. private defence
2. necessity
3. provocation
4. volenti non fit iniuria

5. none of the above (1)

The correct alternative is [2].

Question 9
Alan causes harm to Bert in a wrongful and culpable manner. If Bert succeeds in holding
Charles, Alan’s employer, liable for the damage caused by Alan, Charles will incur:
1. joint and several liability
2. vicarious liability
3. liability based on contributory intent
4. liability based on contributory negligence
5. none of the above
(1)

The correct alternative is [2].


Question 10
When an infringement of a fundamental right constitutes a delict per se, this is known as:
1. direct application of the Bill of Rights
2. a constitutional delict
3. vertical application of the Bill of Rights
4. horizontal application of the Bill of Rights
5. indirect application of the Bill of Rights
(1)

The correct alternative is [2].


TOTAL MARKS: [10]
Question 1
Which one of the following statements is incorrect with regard to a delict?
1. Breach of contract is a species of the genus delict.
2. A “breach of a duty imposed by law” may constitute a delict.
3. One and the same act may result in a delict and a crime.
4. A delict and a constitutional wrong are two different concepts.
(1)
The correct alternative is [1].

Question 2

Nomsa’s two year old daughter falls down a flight of stairs. Nomsa believes that the
child has sustained a brain injury and rushes her to the hospital. On the way to the
hospital Nomsa drives over the neighbour’s dog which subsequently dies. The
neighbour institutes a delictual claim against Nomsa. It later transpires that Nomsa’s
daughter merely suffered from mild shock. Which one of the following defences may
Nomsa rely on?
1. Provocation.
2. Private defence.
3. Necessity.
4. None of the above defences.
(1)

The correct answer is [4].

Question 3

Themba punches David in the face in an attempt to stop David from hitting him with
a spade. David decides to institute a delictual action against Themba for the injury
sustained to his face. Which defence could Themba rely on?
1. Provocation.
2. Private defence.
3. Necessity.
4. None of the above defences.
(1)

The correct alternative is [2].


Question 4
Choose the correct alternative. Dolus eventualis is present when the wrongdoer…
1. desires a particular indirect result with regard to his conduct and continues with
his plan causing the indirect result.
2. does not desire a particular result but foresees the possibility of the result and
reconciles himself with that possibility, nevertheless performing the act.
3. directly intends the result of his conduct but simultaneously is aware that another
consequence will unfortunately occur.
4. Does not desire a particular result but foresees the possibility of the result,
reconciles himself with that possibility and later comes to the conclusion that the
result would not happen.
(1)

The correct alternative is [2].

Question 5
Which of the following statements is correct with regard to accountability?
1. A child under the age of seven is always considered to be culpae capax.
2. There is an irrebuttable presumption that a child between the age of seven and
under fourteen years lacks accountability.
3. A person cannot be at fault without being accountable.
4. A person who drinks and drives thereafter causing an accident can never be culpae
capax.
(1)

The correct alternative is [3].

Question 6
Which one of the following is not a requirement under the actio de pastu?
1. The defendant must be the owner of the animal at the time the damage occurs.
2. The animal must be a domestic animal.
3. The animal must cause damage by eating plants.
4. The animal must act of its own free will when causing the damage.
(1)

The correct answer is [2].

Question 7

Indicate the correct statement with regard to the concurrence of delictual, criminal and
contractual liability.
1. A claim for damages is the primary remedy for breach of contract.
2. One and the same act may render the wrongdoer delictually as well as contractually
liable.
3. One and the same act cannot found delictual as well as criminal liability.
4. A claim for damages is the secondary remedy in respect of a crime.
(1)

The correct alternative is [2].

Question 8
John’s donkey attacks some of the employees on the farm. Just as the donkey was about
to injure Charles (an employee), he shot and killed it with his rifle. John institutes an action
for damages against Charles. Charles may raise the following ground of justification:
1. Necessity.
2. Private defence.
3. Provocation.
4. None of the above.
(1)

The correct alternative is [1].

Question 9
Jacob insults Karen, Julie’s friend. Julie, who is angered by the insults, then insults Jacob.
If Jacob institutes a delictual action for the infringement of his personality rights, Julie may
rely on the following ground of justification:
1. Necessity.
2. Private defence.
3. Provocation.
4. None of the above.
(1)

The correct alternative is [3].

Question 10
Allan, an employee of Raymond, was en route to Rustenburg delivering bread when he
accidentally skipped a red robot and crashed into Mandla’s car. If Mandla successfully
sues Raymond (as the employer) for the damage to his car, Raymond will incur:
1. Vicarious liability.
2. Joint and several liability.
3. Liability based on negligence.
4. Risk liability.

The correct alternative is [1].


Question 1

Indicate the correct statement.

1. Irrational human behaviour cannot constitute conduct for the purposes of the law
of delict.
2. Human behaviour need not be willed to constitute conduct.
3. The behaviour of an animal can constitute conduct under certain circumstances.
4. The presence of a so-called actio libera in causa confirms the presence of
automatism.
(1)

The correct alternative is [2].

Question 2

Jacob is training to be a tattoo artist. He asks Karen whether he can practice his newly
acquired skills on her. Karen enquires whether she will be able to remove the tattoo by
washing, and Jacob replies in the affirmative. Thereupon Karen is willing to comply,
and Jacob makes a butterfly tattoo on Karen’s arm. When Karen discovers that the
tattoo is permanent, she wants to institute a delictual action against Jacob. Jacob may
rely on the following ground of justification:

1. Necessity.
2. Official authority.
3. Consent.
4. None of the above.
(1)

The correct answer is [4].


(1)

Question 4

Piet left his cell phone in his car. Gert wants to steal Piet’s cell phone. He realises that
he would have to damage Piet’s car in order to get the phone. In respect of the
damage to the car, Gert has:
1. Dolus indeterminatus.
2. Dolus eventualis.
3. Dolus indirectus.
4. Luxuria.
(1)

The correct alternative is [3].

Question 5
John’s prize bull breaks through a fence and ventures onto David’s farm. The bull is
aggressive and charges at David’s employees. The employees clamber into a small
tree to escape the wrath of the bull. The bull begins to bash the trunk of the tree. As
the tree appears to be about to topple, David shoots and kills the bull to save his
employees. John institutes an action for damages against David. David may raise the
following ground of justification:

1. Necessity.
2. Private defence.
3. Provocation.
4. Official capacity.
(1)

The correct alternative is [1].

Question 6
When so-called open-ended delictual norms are given content in light of the basic
values of chapter 2 of the Constitution, this is known as:
1. direct application of the Bill of Rights.
2. indirect application of the Bill of Rights.
3. vertical application of the Bill of Rights.
4. horizontal application of the Bill of Rights.
(1)
The correct answer is [2].

Question 7

Andrew inadvertently damages Ben’s violin. Which delictual action may be available to
Ben?
1. actio legis Aquiliae.
2. actio iniuriarum.
3. actio de pastu.
4. none of the above.
(1)

The correct alternative is [1].

Question 8

Mark inadvertently pushes over a ladder on which Chris is standing. Chris breaks an
arm. Which remedy or remedies may be available to Chris?
1. action for pain and suffering only.
2. actio iniuriarum only.
3. action for pain and suffering and actio iniuriarum.
4. none of the above.
(1)

The correct alternative is [1].

Question 9
According to the following case the onus is on the plaintiff to prove that the defendant
acted voluntarily, rather than on the defendant to prove the presence of automatism:
1. Ex parte Minister van Justisie: in re S v Van Wyk 1967 1 SA 488 (A).
2. Molefe v Mahaeng 1999 1 SA 562 (SCA).
3. National Media Ltd v Bogoshi 1988 4 SA 1196 (SCA).
4. S v Goliath 1972 3 SA 1 (A).
(1)

The correct alternative is [2].


Question 10
Prospective loss is best assessed in accordance with:
1. the sum-formula approach.
2. the “once and for all” rule.

3. the concrete approach to damage.


4. res inter alios acta.
(1)

The correct alternative is [1].


Question 1

John takes David’s big and rather aggressive dog for a walk. John incites the dog to
attack Garth. The dog charges at Garth, but Garth shoots the dog and kills it. If David
institutes a delictual action against Garth, on what ground of justification may Garth
rely?
1. Necessity.
2. Private defence.
3. Provocation.
4. None of the above.
(1)

The correct alternative is [1].

Question 2

Markus, a financial adviser, negligently advises Hans to make a bad investment. As a


result of this, Hans suffers a serious financial setback. However, there is no damage to
Hans’s person or property. Which one of the following delictual remedies may be
available if Hans wishes to recover his financial loss from Markus?
1. The actio legis Aquiliae.
2. The actio iniuriarum.
3. The action for pain and suffering.
4. None of the above.
(1)

The correct answer is [1].

Question 3

Shaun steals a big-screen television set from an electronics store. To get to the
television set, he smashes a window made from expensive security glass. In respect of
the damage to the window, Shaun has:
1. Dolus indirectus.
2. Dolus eventualis.
3. Dolus indeterminatus.
4. Luxuria.
(1)

The correct alternative is [1].


Question 4

Clive is employed by Raymond as a driver in Raymond’s courier business. Clive has


the weekend off, and decides to visit Mary on Saturday evening. On his way to Mary’s
apartment, Clive is involved in an accident due to his own negligence. Clive’s vehicle
and the vehicle of Catherine, the other motorist involved in the accident, are badly
damaged. Catherine discovers that Clive’s financial position is not good. Advise
Catherine on the best course of action:

1. Institute an action against Raymond based on vicarious liability.


2. Institute an action against Raymond and Clive as joint wrongdoers.
3. Institute an action against Raymond based on culpa in eligendo.
4. Institute an action against Clive.
(1)

The correct alternative is [4].

Question 5

Which is the odd one out?

1. The adequate causation theory.


2. The sum-formula approach.
3. Direct consequences.
4. Normative foreseeability.
(1)

The correct alternative is [2].


Question 6
Mike’s cow eats and tramples George’s crops. Mike forgot to close the gate between
his and George’s land. Which remedy or remedies may be available to George?
1. actio de pauperie and actio legis Aquiliae.
2. actio de pastu only.
3. actio de pauperie only.
4. actio de pastu and actio legis Aquiliae.
(1)

The correct answer is [4].

Question 7

Norman enters into a sexual relationship with Lex’s wife. Which remedy or remedies
may be available to Lex?
1. actio iniuriarum only.
2. action for pain and suffering only.
3. actio iniuriarum and action for pain and suffering.
4. none of the above.
(1)

The correct alternative is [3].

Question 8

Jors allows Maans to fire some practice shots with his new pistol at an apple on Jors’s
head. The second shot makes a hole in Jors’s left ear. Jors institutes a delictual action
against Maans. Maans may rely on following defence:
1. execution of an official command.

2. provocation.
3. consent to the risk of injury.
4. none of the above.
(1)

The correct alternative is [4].

Question 9

John and Peter bear a grudge against Greg and agree to teach him a lesson he will
not forget. While Greg is in a shopping centre, John and Peter damage Greg’s motor
car by hitting it with hammers. If Greg successfully sues John and Peter in delict, they
will incur:

1. vicarious liability.
2. joint and several liability.
3. liability based on contributory intent.
4. strict liability.
(1)

The correct alternative is [2].

Question 10

According to the following case liability of the media for defamation is based on negligence:
1. Kruger v Coetzee 1966 2 SA 428 (A).
2. Molefe v Mahaeng 1999 1 SA 562 (SCA).
3. National Media Ltd v Bogoshi 1988 4 SA 1196 (SCA).
4. S v Goliath 1972 3 SA 1 (A).
(1)

The correct alternative is [3].


TOTAL MARKS: [10]
Question 1
Indicate the correct statement with regard to the concurrence of delictual, criminal and
contractual liability.

1. A claim for damages is the primary remedy for breach of contract.

2. One and the same act may render the wrongdoer delictually as well as contractually
liable.
3. One and the same act cannot found delictual as well as criminal liability.

4. A claim for damages is the secondary remedy in respect of a crime.


(1)

The correct alternative is [2].

Question 2

Zahra’s dog bolts out of her yard and charges towards Michael. Just as the dog is about to
bite Michael, he shoots and kills the dog with his hand gun. Zahra institutes an action for
damages against Michael. Michael may rely on the following ground of justification:

1. Necessity.

2. Official capacity.

3. Private defence.

4. Provocation.
(1)

The correct answer is [1].

Question 3

Maseeha insults Taslima, who in retaliation slaps Maseeha across the cheek. Maseeha
institutes a delictual action for the infringement of her personality rights. Taslima may rely
on the following ground of justification:

1. Provocation.

2. Official capacity.

3. Private defence.

4. None of the above. (1)

The correct alternative is [4].


Question 4

Zayn left his laptop in his car. Neil wants to steal Zayn’s laptop. He realises that he would
have to damage Zayn’s car in order to get the laptop. In respect of the damage to the car,
Neil has:

1. Dolus indeterminatus.

2. Dolus eventualis.

3. Dolus indirectus.

4. Luxuria.
(1)

The correct alternative is [3].

Question 5

Andre is employed by Ricky as a driver in Ricky’s courier business. Andre has the weekend
off, and decides to visit Tebogo on Saturday evening. On his way to Tebogo’s apartment,
Andre is involved in an accident due to his own negligence. Andre’s vehicle and the vehicle
of Sylvia, the other motorist involved in the accident, are badly damaged. Sylvia discovers
that Andre’s financial position is not good. Advise Sylvia on the best course of action:

1. Institute an action against Ricky based on vicarious liability.

2. Institute an action against Ricky and Andre as joint wrongdoers.

3. Institute an action against Ricky based on culpa in eligendo.

4. Institute an action against Andre.


(1)

The correct alternative is [4].

Question 6
Choose the correct alternative Dolus eventualis is present when the wrongdoer:

1. desires a particular indirect result with regard to his conduct and continues with his plan
causing the indirect result.

2. does not desire a particular result but foresees the possibility of the result and reconciles
himself with that possibility nevertheless performing the act.

3. directly intends the result of his conduct but simultaneously is aware that another
consequence will unfortunately occur.

4. does not desire a particular result but foresees the possibility of the result, and
reconciles himself with that possibility and later comes to the conclusion that the result
would not happen.
(1)
The correct answer is [2].

Question 7

Justin places a home-made firework in a field where sheep are grazing. The firework goes
off damaging the crops and injuring the sheep. Which one of the following forms of fault did
Justin have in respect of the damage?

1. Dolus indeterminatus.

2. Dolus determinatus.

3. Luxuria.

4. Gross negligence
(1)

The correct alternative is [1].

Question 8

According to the following case the onus is on the plaintiff to prove that the defendant
acted voluntarily, rather than on the defendant to prove the presence of automatism:

1. Ex parte Minister van Justisie: in re S v Van Wyk 1967 1 SA 488 (A).

2. Molefe v Mahaeng 1999 1 SA 562 (SCA).

3. National Media Ltd v Bogoshi 1988 4 SA 1196 (SCA).

4. S v Goliath 1972 3 SA 1 (A).


(1)

The correct alternative is [2].

Question 9
Which one of the following statements is correct with regard to accountability?

1. A child under the age of nine is always considered to be culpae capax.

2. There is an irrebuttable presumption that a child between the age of nine and fourteen
years lacks accountability.

3. A person cannot be at fault without being accountable.

4. A person who drinks and drives thereafter causing an accident can never be culpae
capax.
(1)

The correct alternative is [3].

Question 10
Prospective loss is best assessed in accordance with:

1. the sum-formula approach.


2. the “once and for all” rule.

3. the concrete approach to damage.

4. res inter alios acta.

The correct alternative is [1].

Question 1
Jacob tells Cyril that Henry, the mayor of Pretoria, has embezzled money. Cyril repeats this
information to Zapiro, a cartoonist for a local newspaper. Zapiro investigates and finds the
information to be true. He draws a cartoon depicting this and publishes it in the newspaper,
The Daily Update, with the knowledge of the editor Piet. Which one of the following options is
the most correct?

1. Zapiro, The Daily Update and Piet will incur liability with the actio iniuriarum.

2. Jacob, Zapiro, The Daily Update and Piet will incur liability with the actio iniuriarum.

3. Jacob, Cyril, Zapiro, The Daily Update and Piet will incur liability with the actio
iniuriarum.

4. No one will incur liability with the actio iniuriarum.


(1)

The correct alternative is [4].

Question 2

Mary, a financial adviser, negligently advises Musti to make a bad investment. As a result of
this, Musti suffers a serious financial setback. However, there is no damage to Musti’s person or
property. Which one of the following delictual remedies may be available if Musti wishes to
recover his financial loss from Mary?

1. The actio legis Aquiliae.

2. The actio iniuriarum.

3. The action for pain and suffering.

4. None of the above.

(1)

The correct alternative is [1].


Question 3
Indicate the correct statement:

1. Irrational human behaviour cannot constitute conduct for the purposes of the law of
delict.

2. Human behaviour need not be willed to constitute conduct.

3. The behaviour of an animal can constitute conduct under certain circumstances.

4. The presence of a so-called actio libera in causa confirms the presence of automatism.

(1)

The correct alternative is [2].


Question 4
Harry’s prize bull breaks through a fence and ventures onto Mark’s farm. The bull is aggressive
and charges at Mark’s employees. The employees clamber into a small tree to escape the wrath
of the bull. The bull begins to bash the trunk of the tree. As the tree appears to be about to
topple, Mark shoots and kills the bull to save his employees. Harry institutes an action for
damages against Mark. Mark may raise the following ground of justification:

1. Necessity.

2. Private defence.

3. Provocation.

4. Official capacity
(1)

The correct alternative is [1].

Question 5
Which is the odd one out?

1. The adequate causation theory.

2. The sum-formula approach.

3. Direct consequences.

4. Normative foreseeability
(1)

The correct alternative is [2].


Question 6
Mike’s cow eats and tramples George’s crops. Mike forgot t o close the gate between his and
George’s land. Which remedy or remedies may be available to George?

1. actio de pauperie and actio legis Aquiliae.

2. actio de pastu only.

3. actio de pauperie only.

4. actio de pastu and actio legis Aquiliae.


(1)

The correct answer is [4].

Question 7

Gumba and Paul bear a grudge against Krishaan and agree to teach him a lesson he will not
forget. While Krishaan is shopping at Victoria’s shop, Gumba and Paul damage Krishaan’s
vehicle by hitting it with baseball bats. If Krishaan successfully sues Gumba and Paul in delict,
they will incur:

1. vicarious liability.

2. joint and several liability.

3. liability based on contributory intent.

4. strict liability.
(1)

The correct alternative is [2].

Question 8

According to the following case liability of the media for defamation is based on negligence:

1. Kruger v Coetzee 1966 2 SA 428 (A).

2. Molefe v Mahaeng 1999 1 SA 562 (SCA).

3. National Media Ltd v Bogoshi 1988 4 SA 1196 (SCA).

4. S v Goliath 1972 3 SA 1 (A).


(1)

The correct alternative is [3].


Question 1
When delictual norms, such as the boni mores test for wrongfulness, are given content
in the light of the basic values of Chapter 2 of the Constitution, this is known as:
1. direct application of the Bill of Rights
2. indirect application of the Bill of Rights
3. vertical application of the Bill of Rights
4. horizontal application of the Bill of Rights
(1)

The correct alternative is [2].

Question 2

Which statement is incorrect?

1. Only an act or omission that has been willed, can give rise to delictual liability.
2. An act or an omission that is irrational or inexplicable may give rise to delictual liability.
3. The behaviour of an animal can never qualify as conduct for the purpose of the law of
delict.
4. A company can act for the purpose of the law of delict.
(1)

The correct answer is [1].

Question 3

A places a baseball bat into B’s hand. Before B can offer resistance, A takes hold of B’s
hand that is holding the baseball bat and forces a hard blow to C’s head. In respect of the
injury to C, what defence can B raise?

1. Necessity.
2. Provocation.
3. Private defence.
4. Sane automatism.
(1)

The correct alternative is [4].

Question 4
In respect of which one of the following remedies is there an exception to the principle that
wrongfulness can only be ascertained after a harmful consequence has been caused?
1. The actio legis Aquiliae.
2. The actio de effusis vel deiectis.
3. The interdict.
4. Action for pain and suffering.
(1)

The correct alternative is [3].


Question 5
Which one of the following statements is correct?
1. Accountability is a prerequisite for fault.
2. A person is accountable if his behaviour is susceptible to control of his will.
3. Intoxication cannot influence accountability.
4. A person is accountable if he directs his will at an unlawful result.
(1)

The correct alternative is [1].

Question 6
S is employed by R as a driver in R’s business. S has the weekend off, and decides to visit M
on Saturday evening. On his way to M’s apartment, S is involved in an accident due to his own
negligence. S’s vehicle and the vehicle of C, the other motorist involved in the accident, are
badly damaged. C discovers that S’s financial position is not good. Advise C on the best course
of action:
1. Institute an action against R based on vicarious liability.
2. Institute an action against R and S as joint wrongdoers.
3. Institute an action against R based on culpa in eligendo.
4. Institute an action against S.
(1)

The correct answer is [4].

Question 7

P left his ipad in his car. G wants to steal P’s ipad. G realizes that he would have to break
through the window of P’s car to get to the ipad. In respect of the damage to the window of the
car, G has:

1. Dolus indeterminatus.
2. Dolus eventualis.
3. Dolus indirectus.
4. Luxuria.
(1)

The correct alternative is [3].

Question 8
X insults Y, who in retaliation slaps X across the cheek. If X institutes a delictual action for the
infringement of her personality rights, Y may rely on the following defence:
1. Necessity.
2. Private defence.
3. Provocation.
4. None of the above.
(1)

The correct alternative is [4].


Question 9
If a domestic animal has caused harm to a person and the owner of the animal was not
negligent, the most appropriate delictual remedy to consider is:
1. The actio legis Aquiliae.
2. The actio de pauperie.
3. The actio de pastu.
4. None of the above.
(1)

The correct alternative is [2].

Question 10
What is the criterion for determining factual causation?
1. Actio libera in causa.
2. Novus actus interveniens.
3. Conditio sine qua non.
4. The talem qualem rule.
(1)

The correct alternative is [3].


Question 1
Which one of the following statements is incorrect with regard to a delict?
1. Breach of contract is a species of the genus delict.
2. A “breach of a duty imposed by law” may constitute a delict.
3. One and the same act may result in a delict and a crime.
4. A delict and a constitutional wrong are two different concepts.
(1)

In a sense, a delict and breach of a contract are similar in that they constitute wrongful conduct
in private law. With a delict and breach of a contract, an act of a person in a wrongful and
culpable way causes harm to another. They are both species of the genus “wrongful conduct” in
private law. However, there are fundamental differences between a delict and breach of a
contract. Generally, a breach of contract occurs when one of the parties to the contract fails to
fulfil a contractual obligation. A delict occurs where there is an infringement of a party’s legally
recognised interest. Delictual remedies are primarily aimed at satisfaction or compensation,
while contractual remedies are primarily aimed at fulfilment of contractual obligations. Breach of
contract is dealt with under the law of contract with its own requirements and remedies that are
not applicable to a delict. See Neethling, Potgieter and Visser Neethling-Potgieter-Visser Law of
Delict Chapter 1 para 2; Study unit 2. Alternatives [2]-[4] are correct with regard to the law of
delict. In this question you were supposed to identify the incorrect statement, therefore
the correct alternative is [1].

Question 2

Fatima’s two year old daughter falls down a flight of stairs. Fatima believes that the child has
sustained a brain injury and rushes her to the hospital. On the way to the hospital Fatima drives
over the neighbour’s dog which subsequently dies. The neighbour institutes a delictual claim
against Fatima. It later transpires that Fatima’s daughter merely suffered from mild shock.
Which one of the following defences may Fatima rely on?

1. Provocation
2. Private defence
3. Necessity
4. None of the above defences.
Whether a state of necessity actually existed, must be determined objectively and not whether it
was subjectively present in Fatima’s mind. Therefore, Fatima’s conduct is wrongful as a state of
necessity from an objective viewpoint did not actually exist. Furthermore, the grounds of
justification “private defence” or “provocation” is not applicable in this instance. See Neethling,
Potgieter and Visser Neethling-Potgieter-Visser Law of Delict Chapter 3 para 7.3.2 and footnote
455; Study unit 10. The correct alternative is [4].

Question 3

Harry punches David in the face in an attempt to stop David from hitting him with a spade.
David decides to institute a delictual action against Harry for the injury sustained to his face.
Which defence could Harry rely on?
1. Provocation.
2. Private defence.
3. Necessity.
4. None of the above defences.
(1)

The requirements for the attack and the defensive conduct are present. Therefore Harry may
rely on “private defence” in order to avoid delictual liability. In this scenario, the defences of
“necessity” or “provocation” are not applicable. See Neethling, Potgieter and Visser Neethling-
Potgieter-Visser Law of Delict Chapter 3 para 7.2; Study unit 9. The correct alternative is [2].

Question 4
Choose the correct alternative. Dolus eventualis is present when the wrongdoer…

1. desires a particular indirect result with regard to his conduct and continues with his
plan causing the indirect result.
2. does not desire a particular result but foresees the possibility of the result and
reconciles himself with that possibility nevertheless performing the act.
3. directly intends the result of his conduct but simultaneously is aware that another
consequence will unfortunately occur.
4. does not desire a particular result but foresees the possibility of the result,
reconciles himself with that possibility and later comes to the conclusion that the
result would not happen.

(1)

Dolus eventualis is a form of intent where a particular consequence is not desired. The
wrongdoer foresees the possibility of the result and reconciles himself with that possibility
nevertheless performing the act. See Neethling, Potgieter and Visser Neethling-Potgieter-Visser
Law of Delict Chapter 4 para 3.1; Study unit 15. The correct alternative is [2].
Question 5

Which of the following statements is correct with regard to accountability?


1. A child under the age of nine is always considered to be culpae capax.
2. There is an irrebuttable presumption that a child between the age of nine and under
fourteen years lacks accountability.
3. A person cannot be at fault without being accountable.
4. A person who drinks and drives thereafter causing an accident can never be culpae
capax.
(1)

Accountability is a pre-requisite for fault. Therefore, if a person lacks accountability at the time
of the delict, then there is no fault on his part. Youth, intoxication, provocation, a mental illness
or mental impairment may depending on the circumstances lead to the lack of accountability.
See para 2 below (Recent developments) with regard to accountability as well Neethling,
Potgieter and Visser Neethling-Potgieter-Visser Law of Delict Chapter 4 para 2; Study unit 15.
In this question you were supposed to identify the correct statement, therefore the
correct alternative is [3].

Question 6
Which one of the following is not a requirement under the actio de pastu?
1. The defendant must be the owner of the animal at the time the damage occurs.
2. The animal must be a domestic animal.
3. The animal must cause damage by eating plants.
4. The animal must act of its own free will when causing the damage.
(1)

In terms of the requirements for the actio de pastu, the animal that causes damage need not be
a domestic animal. See Neethling, Potgieter and Visser Neethling-Potgieter-Visser Law of Delict
Chapter 11 para 2.1.1.2; Study unit 30. The correct alternative is [2].

Question 7

Indicate the correct statement with regard to the concurrence of delictual, criminal and
contractual liability.
1. A claim for damages is the primary remedy for breach of contract.
2. One and the same act may render the wrongdoer delictually as well as
contractually liable.
3. One and the same act cannot found delictual as well as criminal liability.
4. A claim for damages is the secondary remedy in respect of a crime.

(1)
One and the same act may render the wrongdoer, delictually, contractually or criminally liable. It
depends on whether the requirements are present for a delict, breach of a contract or particular
crime. In respect of a crime, the aim is to punish the offender for “his transgression against the
public interest”. Damages may be awarded when a crime has been committed but
compensation is not considered a secondary remedy in respect of a crime. See Neethling,
Potgieter and Visser Neethling-Potgieter-Visser Law of Delict Chapter 1 paras 2-3 as well as the
commentary to question 1 above. In this question, you were supposed to identify the
correct statement. The correct alternative is [2].

Question 8

Veronica’s donkey attacks some of the employees on the farm. Just as the donkey was
about to injure Charles (an employee), he shot and killed it with his rifle. Veronica institutes
an action for damages against Charles. Charles may raise the following ground of
justification:
1. Necessity.
2. Private defence.
3. Provocation.
4. None of the above.
(1)

In respect of the requirements for the attack with regard to “private defence”, the attack must
consist of a human act. An animal cannot act for the purposes of delict. Therefore, in this
scenario Charles cannot rely on “private defence” but can rely on “necessity”. “Provocation” as a
defence is not applicable in this scenario. See Neethling, Potgieter and Visser Neethling-
Potgieter-Visser Law of Delict Chapter 3 paras 7.2.2(a), 7.3.2; Study units 9 and 10. The
correct alternative is [1].

Question 9
Jacob insults Lefa, Julie’s friend. Julie, who is angered by the insults, then insults Jacob. If
Jacob institutes a delictual action for the infringement of his personality rights, Julie may
rely on the following ground of justification:

1. Necessity.
2. Private defence.
3. Provocation.
4. None of the above.
(1)

Julie may rely on the ground of justification, “provocation”. The requirements for provocation in
this instance are present. In particular, the interests violated in retortio are commensurate with
the initial violation. An insult in reaction to an insult. The insult in retortio is not out of proportion
in comparison to the provocative conduct (insult) on the part of Jacob. See Neethling, Potgieter
and Visser Neethling-Potgieter-Visser Law of Delict Chapter 3 paras 7.4.1, 7.4.3, footnote 521;
Study unit 11. The correct alternative is [3].
Question 10

Allan, an employee of Calvin, was en route to Rustenburg delivering bread when he


accidentally skipped a red robot and crashed into Mandla’s car. If Mandla successfully
sues Calvin (as the employer) for the damage to his car, Calvin will incur:

1. Vicarious liability.
2. Joint and several liability.
3. Liability based on negligence.
4. Risk liability.
(1)

The employer, Calvin is held vicariously liable for the damage caused by the employee, Allan.
There was an employer-employee relationship when the delict was committed, and the
employee, Allan was acting in the course and scope of employment. See Neethling, Potgieter
and Visser Neethling-Potgieter-Visser Law of Delict Chapter 11 para 2.1.7; Study unit 30. The
correct alternative is [1].

TOTAL MARKS: [10]


Question 1
1. Which one of the following conditions may amount to automatism?
1. Provocation.
2. Self-defence.
3. Necessity.
4. Hypnosis.
(1)

The correct alternative is [4]. Hypnosis is one of the recognised conditions that may cause a
person to act involuntarily. Where a person acts involuntarily, the defence of automatism may
apply. Automatism cancels the element of conduct. See Neethling, Potgieter and Visser
Neethling-Potgieter-Visser Law of Delict Chapter 2 para 3; Study Guide, study unit 3. The
other alternatives are defences which cancel the element of wrongfulness.

Question 2

Choose the correct statement. In Carmichele v Minister of Safety and Security (Centre for
Applied Legal Studies Intervening) 2001 (4) SA 938 (CC) the Constitutional Court:

1. recognised a claim for Constitutional damages.


2. noted that the law of delict admirably reflected the spirit, purport and object of the Bill of
Rights.
3. declared that the courts of lower instance had erred by applying a pre-constitutional concept
of the boni mores.
4. recognised the concept of a Constitutional delict.

(1)

The correct alternative is [3]. See Neethling, Potgieter and Visser Neethling-Potgieter-Visser
Law of Delict Chapter 3 para 5.2.4 fn 214.

Question 3

Janice threatens Queeneth with a gun and orders her to hand over her cellphone. Queeneth, in
fear of her life, hands over the cellphone. One week later, Queeneth spots Janice at a nearby
shopping centre. Queeneth rushes home and grabs a knife. She returns to the shopping centre
and stabs Janice. Janice wants to institute a delictual action against Queeneth. Queeneth may
rely on the following ground of justification:

1. necessity.
2. provocation.
3. private defence.
4. none of the above.
(1)
The correct alternative is [4]. Necessity and provocation are not applicable defences in this
scenario. With regard to private defence, it is not possible to act in defence once the attack has
ceased. See Neethling, Potgieter and Visser Neethling-Potgieter-Visser Law of Delict Chapter 3
para 7.2.2 (c); Study Guide, study unit 9.

Question 4

Sibusiso is away on holiday. Frank, his neighbour, is keeping a watchful eye on his house for
him. Frank notices that the house is flooded with water and realises that a water pipe in the
house has burst. Frank breaks down the front door of Sibusiso’s house in order to turn off the
water to prevent further damage. If Sibusiso wants to institute a delictual action against Frank
for breaking down his front door, Frank may rely on the following ground of justification:

1. statutory authority.
2. necessity.
3. provocation.
4. private defence.
(1)

The correct alternative is [2]. In this scenario, the defences of statutory authority, provocation
and private defence are not applicable. Frank acted out of necessity in protecting the interests
of his neighbour Sibusiso. All the requirements for necessity are present. See Neethling,
Potgieter and Visser Neethling-Potgieter-Visser Law of Delict Chapter 3 para 7.3.2; Study
Guide, study unit 10.

Question 5

Joseph plays tennis in his back yard. He foresees the possibility of his ball breaking a window in
his neighbour’s house, but decides that it will not happen. If the ball indeed breaks the window,
Joseph had the following in respect of the damage:

1. dolus indeterminatus.
2. dolus eventualis.
3. dolus indirectus.
4. luxuria.

(1)

The correct alternative is [4]. Joseph foresees the possibility of his ball breaking the window,
he is aware that his ball may break the window but comes to the conclusion that it will not
happen. Therefore, dolus eventualis will not be present but luxuria. Dolus indeterminatus and
dolus indirectus is not present. See Neethling, Potgieter and Visser Neethling-Potgieter-Visser
Law of Delict Chapter 4 para 3.1; Study Guide, study unit 15. All students obtained marks for
this question as a result of confusion over this same question last semester.
Question 6

Dr Lucas, a well-known obstetrician, delivered Nicole’s daughter, Mandy. During delivery,


the nerves in Mandy’s right shoulder were injured resulting in the paralysis of her arm. All
of this happened because Dr Lucas failed to inform Nicole of the potential complications
inherent in delivering a large baby. Indicate the most correct statement: Negligence will be
determined according to the standard of:
1. the reasonable doctor.
2. the reasonable person.
3. the reasonable man.
4. the reasonable child.

(1)

The correct alternative is [1]. In this scenario the wrongdoer is an expert, therefore the test is
adjusted and raised to that of the reasonable expert, that is, the reasonable doctor. See
Neethling, Potgieter and Visser Neethling-Potgieter-Visser Law of Delict Chapter 4 para 4.5.3;
Study Guide, study unit 16.

Question 7

Which of the following best describes what is meant by “psychological lesion”?


1. Pure economic loss.
2. Negligent misrepresentation.
3. Emotional shock.
4. Infringement of the right to identity.
(1)

The correct alternative is [3]. Psychological lesions/injuries may be sustained in a


number of ways, such as through shock, fright, or other mental suffering. In most cases
our courts have been faced with psychological lesions/injuries caused by emotional shock.
See Neethling, Potgieter and Visser Neethling-Potgieter-Visser Law of Delict Chapter 9
para 3; Study Guide, study unit 27.

Question 8
Which is the odd one out?
1. The adequate causation theory.
2. The sum-formula approach.
3. Direct consequences.
4. Normative foreseeability.
(1)

The correct alternative is [2]. Alternatives 1, 3 and 4 relate to establishing legal causation
while alternative 2 is an approach used in determining the element of harm/loss/damage. See in
general Study Guide, study units 22-24.
Question 9

In which one of the following situations can it be said that publication of defamatory words
has taken place?

1. Two German tourists visit South Africa. They start arguing in German in front of some
South Africans (who do not understand German) and the one tourist calls the other a liar
and an adulterer.
2. Mr Xolani tells his wife, Mrs Xolani, that Vanessa at his office is having an affair with her
secretary.
3. Mrs Brown tells her husband, Mr Brown, that Theresa, a work colleague of Mrs Brown,
has stolen money from the cash register on numerous occasions.
4. Bob meets Joseph at the local post office and tells Joseph that the manager of the
post office, Mrs Posh, is involved in an adulterous affair with Joseph’s neighbour Henry.
(1)

The correct alternative is [4]. A defamatory statement must be disclosed to a third person. In
alternatives 2 and 3, publication of defamatory words has not taken place because an exception
applies in respect of spouses. Publication of defamatory words has also not taken place in
alternative 1 as disclosure of defamatory words were heard by outsiders who are not aware of
the meaning of the defamatory words. See Neethling, Potgieter and Visser Neethling-Potgieter-
Visser Law of Delict Chapter 10 para 3.2.2.1; Study Guide, study unit 29.

Question 10

John and Peter bear a grudge against Greg and agree to teach him a lesson he will not
forget. While Greg is in a shopping centre, John and Peter damage Greg’s motor car by
hitting it with hammers. If Greg successfully sues John and Peter in delict, they will incur:
1. vicarious liability.
2. joint and several liability.
3. liability based on contributory intent.
4. strict liability.
(1)

The correct alternative is [2]. Both John and Peter act together in causing damage to Greg’s
motor car. They are therefore deemed joint wrongdoers and are held jointly and severally liable
for the damage to Greg’s car. See Neethling, Potgieter and Visser Neethling-Potgieter-Visser
Law of Delict Chapter 8 para 1; Study Guide, study unit 26.
Question 1
1. Which one of the following conditions may amount to automatism?
1. Provocation.
2. Self-defence.
3. Black-out.

4. Necessity.
(1)

The correct alternative is [3]. It is accepted that a black-out may cause a person to act
involuntarily. Alternatives 1, 2 and 4 are generally considered as grounds of justification which
negate the element of wrongfulness. See Neethling, Potgieter and Visser Neethling-Potgieter-
Visser Law of Delict Chapter 2 para 3; Chapter 3 para 7; Study Guide, study units 3, 9-11.

Question 2

Choose the correct statement. In Carmichele v Minister of Safety and Security (Centre for
Applied Legal Studies Intervening) 2001 (4) SA 938 (CC) the Constitutional Court:
1. declared that the courts of lower instance had erred by applying a pre-constitutional concept
of the boni mores.

2. noted that the law of delict admirably reflected the spirit, purport and object of the Bill of
Rights.
3. recognised a claim for Constitutional damages.
4. recognised the concept of a Constitutional delict.

(1)

The correct alternative is [1]. See Neethling, Potgieter and Visser Neethling-Potgieter-Visser
Law of Delict Chapter 3 para 5.2.4 fn 214.
Question 3

Dr Berenice, a well-known obstetrician, delivered Bongi’s son, Kabelo. During delivery, the
nerves in Kabelo’s right shoulder were injured resulting in the paralysis of his arm. All of this
happened because Dr Berenice failed to inform Bongi of the potential complications inherent in
delivering a large baby. Indicate the most correct statement: Negligence will be determined
according to the standard of:

1. the reasonable doctor.


2. the reasonable person.
3. the reasonable man.
4. the reasonable child.
(1)

The correct alternative is [1]. In this scenario the wrongdoer is an expert, therefore the test is
adjusted and raised to that of the reasonable expert, that is, the reasonable doctor. See
Neethling, Potgieter and Visser Neethling-Potgieter-Visser Law of Delict Chapter 4 para 4.5.3;
Study Guide, study unit 16.

Question 4

Musa is away on holiday. Frank, his neighbour, is keeping a watchful eye on his house for him
and undertook to take care of Musa’s cat. Frank notices that the house is on fire and realises
that Musa’s cat is trapped in the burning house. Frank breaks down the front door of Musa’s
house in order to save the cat. If Musa wants to institute a delictual action against Frank for
breaking down his front door, Frank may rely on the following ground of justification:
1. statutory authority.
2. necessity.
3. provocation.

4. private defence.
(1)

The correct alternative is [2]. In this scenario, the defences of private defence, provocation
and statutory authority are not applicable. Necessity exists when the defendant is placed in
such a position that he or she is able to protect an interest (his or her own legally recognisable
interest or that of someone else) only by reasonably violating the interests of another person.
Frank acted out of necessity in protecting the life of the cat. All the requirements for necessity
are present. See Neethling, Potgieter and Visser Neethling-Potgieter-Visser Law of Delict
Chapter 3 para 7.3; Study Guide, study unit 10.
Question 5

Dick is so offended by William’s speech at a political rally, that he throws an apple at William.
William falls off the podium and sustains a broken arm as well as a few broken ribs. He is
admitted to hospital. While he is being treated there, a nurse leaves the windows of the ward
open and William contracts pneumonia. Which one of the following is incorrect?

1. Dick acted wrongfully.


2. Dick had fault.
3. There is a factual causal link between Dick’s conduct and William’s pneumonia.
4. There is a legal causal link between Dick’s conduct and William’s pneumonia.
(1)

The correct alternative is [4]. The emphasis here was on the incorrect statement. Dick did
act wrongfully when he threw the apple and he had fault (intention) when he directed the apple
at William. Dicks conduct has a factual causal link to William’s broken arm, ribs and pneumonia.
To determine factual causation, the conditio sine qua non theory (also known as the ‘but for’
test) is applied. With regard to William contracting pneumonia, Dick’s intentional wrongful
conduct was the factual cause but not the legal cause. The nurse’s conduct (leaving the window
open) may be considered a novus actus interveniens. Therefore, alternative 4 is the incorrect
statement and the correct alternative to this question. See Neethling, Potgieter and Visser
Neethling-Potgieter-Visser Law of Delict Chapter 5 para 2-3; Study Guide, study unit 21-22.

Question 6

The most suitable remedy for the prevention of harm is:


1. the actio legis Aquiliae.
2. the actio de effusis vel deiectis.
3. the interdict.
4. mitigation of loss.
(1)

The correct alternative is [3]. The interdict is directed at the prevention of a wrongful act
(impending wrongful act or to prevent the continuation of a wrongful act that has already
commenced) and is therefore the most suitable remedy. The interdict has a preventative
function and as a result, there is no need for the requirement of fault on the part of the
wrongdoer to be present. The requirements that need to be met for the granting of an interdict
are: there must be an act by the respondent, the act must be wrongful, and no other ordinary
remedy which would prevent the wrongful conduct must be available to the applicant. See
Neethling, Potgieter and Visser Neethling-Potgieter-Visser Law of Delict Chapter 7 para 2;
Study Guide, study unit 25.
Question 7

The principle dictating that all past and prospective loss must be claimed together stemming
from a single cause of action, is known as:

1. the sum-formula approach.


2. the ‘once and for all’ rule.
3. compensating advantages.
4. the concrete approach to damage.

(1)

The correct alternative is [2]. The “once and for all” rule expresses that in all claims for
compensation and satisfaction arising out of a delict, the plaintiff must claim damages for all
damage already sustained and all future damages insofar as the claims are based on a single
cause of action. See Neethling, Potgieter and Visser Neethling-Potgieter-Visser Law of Delict
Chapter 6 para 4.7; Study Guide, study unit 24.

Question 8
Which is the odd one out?
1. The adequate causation theory.

2. Normative foreseeability.
3. Direct consequences.
4. The sum-formula approach.
(1)

The correct alternative is [4]. Alternatives 1, 2 and 3 relate to establishing legal causation
while alternative 4 is an approach used in determining the element of harm/loss/damage. See in
general Study Guide, study units 22-24.

Question 9

Eddie and Robie bear a grudge against Greg and agree to teach him a lesson he will not forget.
While Greg is in a shopping centre, Eddie and Robie damage Greg’s motor car by hitting it with
hammers. If Greg successfully sues Eddie and Robie in delict, they will incur:
1. vicarious liability.
2. joint and several liability.
3. liability based on contributory intent.
4. strict liability.
(1)
The correct alternative is [2]. Both Eddie and Robie act together in causing damage to Greg’s motor car.
They are therefore deemed joint wrongdoers and are held jointly and severally liable for the damage to
Greg’s car. See Neethling, Potgieter and Visser Neethling-Potgieter-Visser Law of Delict Chapter 8 para 1;
Study Guide, study unit 26.

Question 10
In which one of the following situations can it be said that publication of defamatory words has
taken place?

1. Two German tourists visit South Africa. They start arguing in German in front of some South
Africans (who do not understand German) and the one tourist calls the other a liar and an
adulterer.

2. Mr Xolani tells his wife, Mrs Xolani, that Vanessa at his office is having an affair with her
secretary.
3. Bob meets Ricky at the local post office and tells Ricky that the manager of the post office,
Mrs Posh, is involved in an adulterous affair with Ricky’s neighbour Henry.
4. Mrs Brown tells her husband, Mr Brown, that Theresa, a work colleague of Mrs Brown, has
stolen money from the cash register on numerous occasions.
(1)

The correct alternative is [3]. A defamatory statement must be disclosed to a third person. In alternatives
2 and 4, publication of defamatory words has not taken place because an exception applies in respect of
spouses. Publication of defamatory words has also not taken place in alternative 1 as disclosure of
defamatory words were heard by outsiders who are not aware of the meaning of the defamatory words. See
Neethling, Potgieter and Visser Neethling-Potgieter- Visser Law of Delict Chapter 10 para 3.2.2.1; Study
Guide, study unit 29.
Question 1
Which one of the following statements is incorrect with regard to a delict?
1. Breach of contract is a species of the genus delict.
2. A “breach of a duty imposed by law” may constitute a delict.
3. One and the same act may result in a delict and a crime.
4. A delict and a constitutional wrong are two different concepts.
Question 2
Fatima’s two year old daughter falls down a flight of stairs. Fatima believes that
the child has sustained a brain injury and rushes her to the hospital. On the way
to the hospital Fatima drives over the neighbour’s dog which subsequently dies.
The neighbour institutes a delictual claim against Fatima. It later transpires that
Fatima’s daughter merely suffered from mild shock. Which one of the following
defences may Fatima rely on?
1. Provocation
2. Private defence
3. Necessity
4. None of the above defences.
Question 3
Harry punches David in the face in an attempt to stop David from hitting him
with a spade. David decides to institute a delictual action against Harry for the
injury sustained to his face. Which defence could Harry rely on?
1. Provocation.
2. Private defence.
3. Necessity.
4. None of the above defences.
Question 4
Choose the correct alternative. Dolus eventualis is present when the
wrongdoer…
1. desires a particular indirect result with regard to his conduct and continues
with his plan causing the indirect result.
2. does not desire a particular result but foresees the possibility of the result
and reconciles himself with that possibility nevertheless performing the act.
3. directly intends the result of his conduct but simultaneously is aware that
another consequence will unfortunately occur.
4. does not desire a particular result but foresees the possibility of the result,
reconciles himself with that possibility and later comes to the conclusion that
the result would not happen.
Question 5
Which of the following statements is correct with regard to accountability?
1. A child under the age of nine is always considered to be culpae capax.
2. There is an irrebuttable presumption that a child between the age of nine and
under fourteen years lacks accountability.
3. A person cannot be at fault without being accountable.
4. A person who drinks and drives thereafter causing an accident can never be
culpae capax.
Question 6
Which one of the following is not a requirement under the actio de pastu?
1. The defendant must be the owner of the animal at the time the damage
occurs.
2. The animal must be a domestic animal.
3. The animal must cause damage by eating plants.
4. The animal must act of its own free will when causing the damage.
Question 7
Indicate the correct statement with regard to the concurrence of delictual,
criminal and contractual liability.
1. A claim for damages is the primary remedy for breach of contract.
2. One and the same act may render the wrongdoer delictually as well as
contractually liable.
3. One and the same act cannot found delictual as well as criminal liability.
4. A claim for damages is the secondary remedy in respect of a crime.
Question 8
Veronica’s donkey attacks some of the employees on the farm. Just as the
donkey was about to injure Charles (an employee), he shot and killed it with his
rifle. Veronica institutes an action for damages against Charles. Charles may
raise the following ground of justification:
1. Necessity.
2. Private defence.
3. Provocation.
4. None of the above.
Question 9
Jacob insults Lefa, Julie’s friend. Julie, who is angered by the insults, then
insults Jacob. If Jacob institutes a delictual action for the infringement of his
personality rights, Julie may rely on the following ground of justification:
1. Necessity.
2. Private defence.
3. Provocation.
4. None of the above.
Question 10
Allan, an employee of Calvin, was en route to Rustenburg delivering bread
when he accidentally skipped a red robot and crashed into Mandla’s car. If
Mandla successfully sues Calvin (as the employer) for the damage to his car,
Calvin will incur:
1. Vicarious liability.
2. Joint and several liability.
3. Liability based on negligence.
4. Risk liability.
ASSIGNMENT 2018 SEMESTER 2

Question 1
Roy incites his pedigreed, but vicious, dog to attack Jack. Jack panics and
shoots the dog. If Roy institutes a delictual claim against Jack to recover the
cost of the dog, which defence may Jack rely on?
1. Provocation.
2. Private defence.
3. Necessity.
4. None of the above defences.
Question 2
Titus places a home-made bomb in a field where cattle are grazing. The bomb
explodes damaging crops and injuring the cattle. Which one of the following
forms of fault did Titus have in respect of the damage?
1. Dolus indeterminatus.
2. Dolus determinatus.
3. Luxuria.
4. Gross negligence.
Question 3
Which one of the following benefits is taken into account in reducing the amount
of damages awarded to a plaintiff?
1. Benefits received in terms of life assurance.
2. Insurance money received.
3. Donations or ex gratia benefits.
4. An award from the Compensation Commissioner.
Question 4
Which one of the following is not a requirement for an interdict?
1. There must be no other remedy available to the applicant.
2. Intention or negligence must be present.
3. There must be a “clear right”.
4. There must be an infringement or a threat of an infringement of a clear right.
Question 5
Indicate the correct statement with regard to delictual remedies.
1. The actio iniuriarum is directed at “satisfaction” for the wrongful and
intentional injury to personality.
2. Intention is always a requirement for the actio legis Aquiliae.
3. South African law follows the casuistic approach with regard to delictual
liability.
4. “Invasion of privacy” in practice is considered as a form of damnum iniuria
datum.
Question 6
Indicate the incorrect statement with regard to the law of delict and the
constitution.
1. An infringement of a right may constitute a constitutional wrong and a delict.
2. The requirements for a delict and a constitutional wrong differ materially.
3. Both a constitutional remedy and a delictual remedy are aimed primarily at
compensation.
4. Direct application means that the state must generally respect fundamental
rights and not infringe them.
Question 7
Rocky insults Tammy who in retaliation slaps Rocky across the cheek. If Rocky
institutes a delictual action for the infringement of his personality rights, Tammy
may rely on the following ground of justification:
1. Necessity.
2. Private defence.
3. Provocation.
4. None of the above.
Question 8
Brendon purchases a ticket to a boat ride. The ticket clearly states that the
management of the amusement park will not be held liable for any negligence
on its part. As the ride comes to a stop, the boat jerks due to a mechanical fault
and Brendon bumps his head against the seat rendering him unconscious.
When Brendon awakens, he decides to institute an action for damages against
the management of the amusement park. In order to exclude or limit liability,
the management may rely on:
1. Consent to the risk of injury.
2. Contributory negligence.
3. Official capacity.
4. None of the above.
Question 9
Kabelo plays tennis in his back yard. He foresees the possibility of his ball
breaking a window in his neighbour’s house, but decides that it will not happen.
If the ball indeed breaks the window, Kabelo had the following in respect of the
damage:
1. Dolus indeterminatus.
2. Dolus eventualis.
3. Dolus indirectus.
4. luxuria.
Question 10
Willy’s bull tramples and eats Charlotte’s maize that she planted in her fields.
Which remedy may be available to Charlotte?
1. Actio de pauperie.
2. Actio de pastu.
3. Actio de effusis vel deiectis.
4. Actio de feris.
ASSIGNMENT 2019 SEMESTER 1

Question 1
1. Which one of the following conditions may amount to automatism?
1. Provocation.
2. Self-defence.
3. Necessity.
4. Hypnosis.
Question 2
Choose the correct statement. In Carmichele v Minister of Safety and Security
(Centre for Applied Legal Studies Intervening) 2001 (4) SA 938 (CC) the
Constitutional Court:
1. recognised a claim for Constitutional damages.
2. noted that the law of delict admirably reflected the spirit, purport and object
of the Bill of Rights.
3. declared that the courts of lower instance had erred by applying a pre-
constitutional concept of the boni mores.
4. recognised the concept of a Constitutional delict.
Question 3
Janice threatens Queeneth with a gun and orders her to hand over her
cellphone. Queeneth, in fear of her life, hands over the cellphone. One week
later, Queeneth spots Janice at a nearby shopping centre. Queeneth rushes
home and grabs a knife. She returns to the shopping centre and stabs Janice.
Janice wants to institute a delictual action against Queeneth. Queeneth may
rely on the following ground of justification:
1. necessity.
2. provocation.
3. private defence.
4. none of the above
Question 4
Sibusiso is away on holiday. Frank, his neighbour, is keeping a watchful eye on
his house for him. Frank notices that the house is flooded with water and
realises that a water pipe in the house has burst. Frank breaks down the front
door of Sibusiso’s house in order to turn off the water to prevent further damage.
If Sibusiso wants to institute a delictual action against Frank for breaking down
his front door, Frank may rely on the following ground of justification:
1. statutory authority.
2. necessity.
3. provocation.
4. private defence.
Question 5
Joseph plays tennis in his back yard. He foresees the possibility of his ball
breaking a window in his neighbour’s house, but decides that it will not happen.
If the ball indeed breaks the window, Joseph had the following in respect of the
damage:
1. dolus indeterminatus.
2. dolus eventualis.
3. dolus indirectus.
4. luxuria.
Question 6
Dr Lucas, a well-known obstetrician, delivered Nicole’s daughter, Mandy.
During delivery, the nerves in Mandy’s right shoulder were injured resulting in
the paralysis of her arm. All of this happened because Dr Lucas failed to inform
Nicole of the potential complications inherent in delivering a large baby. Indicate
the most correct statement: Negligence will be determined according to the
standard of:
1. the reasonable doctor.
2. the reasonable person.
3. the reasonable man.
4. the reasonable child
Question 7
Which of the following best describes what is meant by “psychological lesion”?
1. Pure economic loss.
2. Negligent misrepresentation.
3. Emotional shock.
4. Infringement of the right to identity.
Question 8
Which is the odd one out?
1. The adequate causation theory.
2. The sum-formula approach.
3. Direct consequences.
4. Normative foreseeability.
Question 9
In which one of the following situations can it be said that publication of
defamatory words has taken place?
1. Two German tourists visit South Africa. They start arguing in German in front
of some South Africans (who do not understand German) and the one tourist
calls the other a liar and an adulterer.
2. Mr Xolani tells his wife, Mrs Xolani, that Vanessa at his office is having an
affair with her secretary.
3. Mrs Brown tells her husband, Mr Brown, that Theresa, a work colleague of
Mrs Brown, has stolen money from the cash register on numerous occasions.
4. Bob meets Joseph at the local post office and tells Joseph that the manager
of the post office, Mrs Posh, is involved in an adulterous affair with Joseph’s
neighbour Henry.
Question 10
John and Peter bear a grudge against Greg and agree to teach him a lesson
he will not forget. While Greg is in a shopping centre, John and Peter damage
Greg’s motor car by hitting it with hammers. If Greg successfully sues John and
Peter in delict, they will incur:
1. vicarious liability.
2. joint and several liability.
3. liability based on contributory intent.
4. strict liability.
Question 1
John takes David’s big and rather aggressive dog for a walk. John incites the
dog to attack Garth. The dog charges at Garth, but Garth shoots the dog and
kills it. If David institutes a delictual action against Garth, on what ground of
justification may Garth rely?
1. Necessity.
2. Private defence.
3. Provocation.
4. None of the above.
Question 2
Markus, a financial adviser, negligently advises Hans to make a bad
investment. As a result of this, Hans suffers a serious financial setback.
However, there is no damage to Hans’s person or property. Which one of the
following delictual remedies may be available if Hans wishes to recover his
financial loss from Markus?
1. The actio legis Aquiliae.
2. The actio iniuriarum.
3. The action for pain and suffering.
4. None of the above.
Question 3
Shaun steals a big-screen television set from an electronics store. To get to the
television set, he smashes a window made from expensive security glass. In
respect of the damage to the window, Shaun has:
1. Dolus indirectus.
2. Dolus eventualis.
3. Dolus indeterminatus.
4. Luxuria.
Question 4
Clive is employed by Raymond as a driver in Raymond’s courier business. Clive
has the weekend off, and decides to visit Mary on Saturday evening. On his
way to Mary’s apartment, Clive is involved in an accident due to his own
negligence. Clive’s vehicle and the vehicle of Catherine, the other motorist
involved in the accident, are badly damaged. Catherine discovers that Clive’s
financial position is not good. Advise Catherine on the best course of action:
1. Institute an action against Raymond based on vicarious liability.
2. Institute an action against Raymond and Clive as joint wrongdoers.
3. Institute an action against Raymond based on culpa in eligendo.
4. Institute an action against Clive.
Question 5
Which is the odd one out?
1. The adequate causation theory.
2. The sum-formula approach.
3. Direct consequences.
4. Normative foreseeability.
Question 6
Mike’s cow eats and tramples George’s crops. Mike forgot to close the gate
between his and George’s land. Which remedy or remedies may be available
to George?
1. actio de pauperie and actio legis Aquiliae.
2. actio de pastu only.
3. actio de pauperie only.
4. actio de pastu and actio legis Aquiliae.
Question 7
Norman enters into a sexual relationship with Lex’s wife. Which remedy or
remedies may be available to Lex?
1. actio iniuriarum only.
2. action for pain and suffering only.
3. actio iniuriarum and action for pain and suffering.
4. none of the above.
Question 8
Jors allows Maans to fire some practice shots with his new pistol at an apple
on Jors’s head. The second shot makes a hole in Jors’s left ear. Jors institutes
a delictual action against Maans. Maans may rely on following defence:
1. execution of an official command.
2. provocation.
3. consent to the risk of injury.
4. none of the above.
Question 9
John and Peter bear a grudge against Greg and agree to teach him a
lesson he will not forget. While Greg is in a shopping centre, John and
Peter damage Greg’s motor car by hitting it with hammers. If Greg
successfully sues John and Peter in delict, they will incur:
1. vicarious liability.
2. joint and several liability.
3. liability based on contributory intent.
4. strict liability.
Question 10
According to the following case liability of the media for defamation is
based on negligence:
1. Kruger v Coetzee 1966 2 SA 428 (A).
2. Molefe v Mahaeng 1999 1 SA 562 (SCA).
3. National Media Ltd v Bogoshi 1988 4 SA 1196 (SCA).
4. S v Goliath 1972 3 SA 1 (A)
Question 1

1. Which one of the following conditions may amount to automatism?


1. Provocation.
2. Self-defence.
3. Black-out.
4. Necessity.
(1)

The correct alternative is [3]. It is accepted that a black-out may cause a person to act
involuntarily. Alternatives 1, 2 and 4 are generally considered as grounds of justification which
negate the element of wrongfulness. See Neethling, Potgieter and Visser Neethling-Potgieter-
Visser Law of Delict Chapter 2 para 3; Chapter 3 para 7; Study Guide, study units 3, 9-11.

Question 2

Choose the correct statement. In Carmichele v Minister of Safety and Security (Centre for
Applied Legal Studies Intervening) 2001 (4) SA 938 (CC) the Constitutional Court:
1. declared that the courts of lower instance had erred by applying a pre-constitutional concept
of the boni mores.
2. noted that the law of delict admirably reflected the spirit, purport and object of the Bill of
Rights.
3. recognised a claim for Constitutional damages.
4. recognised the concept of a Constitutional delict.

(1)

The correct alternative is [1]. See Neethling, Potgieter and Visser Neethling-Potgieter-Visser
Law of Delict Chapter 3 para 5.2.4 fn 214.
Question 3

Dr Berenice, a well-known obstetrician, delivered Bongi’s son, Kabelo. During delivery, the
nerves in Kabelo’s right shoulder were injured resulting in the paralysis of his arm. All of this
happened because Dr Berenice failed to inform Bongi of the potential complications inherent in
delivering a large baby. Indicate the most correct statement: Negligence will be determined
according to the standard of:
1. the reasonable doctor.
2. the reasonable person.
3. the reasonable man.
4. the reasonable child.
(1)

The correct alternative is [1]. In this scenario the wrongdoer is an expert, therefore the test is
adjusted and raised to that of the reasonable expert, that is, the reasonable doctor. See
Neethling, Potgieter and Visser Neethling-Potgieter-Visser Law of Delict Chapter 4 para 4.5.3;
Study Guide, study unit 16.

Question 4
Musa is away on holiday. Frank, his neighbour, is keeping a watchful eye on his house for him
and undertook to take care of Musa’s cat. Frank notices that the house is on fire and realises
that Musa’s cat is trapped in the burning house. Frank breaks down the front door of Musa’s
house in order to save the cat. If Musa wants to institute a delictual action against Frank for
breaking down his front door, Frank may rely on the following ground of justification:
1. statutory authority.
2. necessity.
3. provocation.
4. private defence.
(1)

The correct alternative is [2]. In this scenario, the defences of private defence, provocation
and statutory authority are not applicable. Necessity exists when the defendant is placed in
such a position that he or she is able to protect an interest (his or her own legally recognisable
interest or that of someone else) only by reasonably violating the interests of another person.
Frank acted out of necessity in protecting the life of the cat. All the requirements for necessity
are present. See Neethling, Potgieter and Visser Neethling-Potgieter-Visser Law of Delict
Chapter 3 para 7.3; Study Guide, study unit 10.
Question 5
Dick is so offended by William’s speech at a political rally, that he throws an apple at William.
William falls off the podium and sustains a broken arm as well as a few broken ribs. He is
admitted to hospital. While he is being treated there, a nurse leaves the windows of the ward
open and William contracts pneumonia. Which one of the following is incorrect?

1. Dick acted wrongfully.


2. Dick had fault.
3. There is a factual causal link between Dick’s conduct and William’s pneumonia.
4. There is a legal causal link between Dick’s conduct and William’s pneumonia.
(1)

The correct alternative is [4]. The emphasis here was on the incorrect statement. Dick did
act wrongfully when he threw the apple and he had fault (intention) when he directed the apple
at William. Dicks conduct has a factual causal link to William’s broken arm, ribs and pneumonia.
To determine factual causation, the conditio sine qua non theory (also known as the ‘but for’
test) is applied. With regard to William contracting pneumonia, Dick’s intentional wrongful
conduct was the factual cause but not the legal cause. The nurse’s conduct (leaving the window
open) may be considered a novus actus interveniens. Therefore, alternative 4 is the incorrect
statement and the correct alternative to this question. See Neethling, Potgieter and Visser
Neethling-Potgieter-Visser Law of Delict Chapter 5 para 2-3; Study Guide, study unit 21-22.

Question 6

The most suitable remedy for the prevention of harm is:


1. the actio legis Aquiliae.
2. the actio de effusis vel deiectis.

3. the interdict.
4. mitigation of loss.
(1)

The correct alternative is [3]. The interdict is directed at the prevention of a wrongful act
(impending wrongful act or to prevent the continuation of a wrongful act that has already
commenced) and is therefore the most suitable remedy. The interdict has a preventative
function and as a result, there is no need for the requirement of fault on the part of the
wrongdoer to be present. The requirements that need to be met for the granting of an interdict
are: there must be an act by the respondent, the act must be wrongful, and no other ordinary
remedy which would prevent the wrongful conduct must be available to the applicant. See
Neethling, Potgieter and Visser Neethling-Potgieter-Visser Law of Delict Chapter 7 para 2;
Study Guide, study unit 25.
Question 7

The principle dictating that all past and prospective loss must be claimed together stemming
from a single cause of action, is known as:
1. the sum-formula approach.
2. the ‘once and for all’ rule.
3. compensating advantages.
4. the concrete approach to damage.

(1)

The correct alternative is [2]. The “once and for all” rule expresses that in all claims for
compensation and satisfaction arising out of a delict, the plaintiff must claim damages for all
damage already sustained and all future damages insofar as the claims are based on a single
cause of action. See Neethling, Potgieter and Visser Neethling-Potgieter-Visser Law of Delict
Chapter 6 para 4.7; Study Guide, study unit 24.

Question 8
Which is the odd one out?
1. The adequate causation theory.
2. Normative foreseeability.
3. Direct consequences.
4. The sum-formula approach.
(1)

The correct alternative is [4]. Alternatives 1, 2 and 3 relate to establishing legal causation
while alternative 4 is an approach used in determining the element of harm/loss/damage. See in
general Study Guide, study units 22-24.
Question 9

Eddie and Robie bear a grudge against Greg and agree to teach him a lesson he will not forget.
While Greg is in a shopping centre, Eddie and Robie damage Greg’s motor car by hitting it with
hammers. If Greg successfully sues Eddie and Robie in delict, they will incur:
1. vicarious liability.
2. joint and several liability.
3. liability based on contributory intent.
4. strict liability.
(1)

The correct alternative is [2]. Both Eddie and Robie act together in causing damage to
Greg’s motor car. They are therefore deemed joint wrongdoers and are held jointly and
severally liable for the damage to Greg’s car. See Neethling, Potgieter and Visser
Neethling-Potgieter-Visser Law of Delict Chapter 8 para 1; Study Guide, study unit 26.

Question 10
In which one of the following situations can it be said that publication of defamatory words has
taken place?
1. Two German tourists visit South Africa. They start arguing in German in front of some South
Africans (who do not understand German) and the one tourist calls the other a liar and an
adulterer.
2. Mr Xolani tells his wife, Mrs Xolani, that Vanessa at his office is having an affair with her
secretary.
3. Bob meets Ricky at the local post office and tells Ricky that the manager of the post office,
Mrs Posh, is involved in an adulterous affair with Ricky’s neighbour Henry.
4. Mrs Brown tells her husband, Mr Brown, that Theresa, a work colleague of Mrs Brown, has
stolen money from the cash register on numerous occasions.
(1)

The correct alternative is [3]. A defamatory statement must be disclosed to a third


person. In alternatives 2 and 4, publication of defamatory words has not taken place
because an exception applies in respect of spouses. Publication of defamatory words has
also not taken place in alternative 1 as disclosure of defamatory words were heard by
outsiders who are not aware of the meaning of the defamatory words. See Neethling,
Potgieter and Visser Neethling-Potgieter- Visser Law of Delict Chapter 10 para 3.2.2.1;
Study Guide, study unit 29.
Question 1
When delictual norms, such as the boni mores test for wrongfulness, are given content
in the light of the basic values of Chapter 2 of the Constitution, this is known as:
1. direct application of the Bill of Rights
2. indirect application of the Bill of Rights
3. vertical application of the Bill of Rights
4. horizontal application of the Bill of Rights
(1)

The correct alternative is [2].

Question 2

Which statement is incorrect?

1. Only an act or omission that has been willed, can give rise to delictual liability.
2. An act or an omission that is irrational or inexplicable may give rise to delictual liability.
3. The behaviour of an animal can never qualify as conduct for the purpose of the law of
delict.
4. A company can act for the purpose of the law of delict.
(1)

The correct answer is [1].

Question 3

A places a baseball bat into B’s hand. Before B can offer resistance, A takes hold of B’s
hand that is holding the baseball bat and forces a hard blow to C’s head. In respect of
the injury to C, what defence can B raise?

1. Necessity.
2. Provocation.
3. Private defence.
4. Sane automatism.
(1)

The correct alternative is [4].


Question 4
In respect of which one of the following remedies is there an exception to the principle that
wrongfulness can only be ascertained after a harmful consequence has been caused?
1. The actio legis Aquiliae.
2. The actio de effusis vel deiectis.
3. The interdict.
4. Action for pain and suffering.
(1)

The correct alternative is [3].

Question 5
Which one of the following statements is correct?
1. Accountability is a prerequisite for fault.
2. A person is accountable if his behaviour is susceptible to control of his will.
3. Intoxication cannot influence accountability.
4. A person is accountable if he directs his will at an unlawful result.
(1)

The correct alternative is [1].

Question 6
S is employed by R as a driver in R’s business. S has the weekend off, and decides to visit M
on Saturday evening. On his way to M’s apartment, S is involved in an accident due to his own
negligence. S’s vehicle and the vehicle of C, the other motorist involved in the accident, are
badly damaged. C discovers that S’s financial position is not good. Advise C on the best
course of action:
1. Institute an action against R based on vicarious liability.
2. Institute an action against R and S as joint wrongdoers.
3. Institute an action against R based on culpa in eligendo.
4. Institute an action against S.
(1)

The correct answer is [4].


PVL3703/201

Question 7

P left his ipad in his car. G wants to steal P’s ipad. G realizes that he would have to break
through the window of P’s car to get to the ipad. In respect of the damage to the window of the
car, G has:

1. Dolus indeterminatus.
2. Dolus eventualis.
3. Dolus indirectus.
4. Luxuria.
(1)

The correct alternative is [3].

Question 8
X insults Y, who in retaliation slaps X across the cheek. If X institutes a delictual action for the
infringement of her personality rights, Y may rely on the following defence:
1. Necessity.
2. Private defence.
3. Provocation.
4. None of the above.
(1)

The correct alternative is [4].

Question 9
If a domestic animal has caused harm to a person and the owner of the animal was not
negligent, the most appropriate delictual remedy to consider is:
1. The actio legis Aquiliae.
2. The actio de pauperie.
3. The actio de pastu.
4. None of the above.
(1)

The correct alternative is [2].

Question 10
What is the criterion for determining factual causation?
1. Actio libera in causa.
2. Novus actus interveniens.
3. Conditio sine qua non.
4. The talem qualem rule.
(1)

The correct alternative is [3].

5
PVL3703/201

2.1 ASSIGNMENT 02 (MULTIPLE CHOICE ASSIGNMENT)

COMMENTARY
ASSIGNMENT 02: MULTIPLE CHOICE QUESTIONS
Unique assignment number: 806561

Question 1
Jacob tells Cyril that Henry, the mayor of Pretoria, has embezzled money. Cyril repeats this
information to Zapiro, a cartoonist for a local newspaper. Zapiro investigates and finds the
information to be true. He draws a cartoon depicting this and publishes it in the newspaper,
The Daily Update, with the knowledge of the editor Piet. Which one of the following options is
the most correct?

1. Zapiro, The Daily Update and Piet will incur liability with the actio iniuriarum.

2. Jacob, Zapiro, The Daily Update and Piet will incur liability with the actio iniuriarum.

3. Jacob, Cyril, Zapiro, The Daily Update and Piet will incur liability with the actio
iniuriarum.

4. No one will incur liability with the actio iniuriarum.


(1)

6
The correct alternative is [4].

Question 2

Mary, a financial adviser, negligently advises Musti to make a bad investment. As a result of
this, Musti suffers a serious financial setback. However, there is no damage to Musti’s person
or property. Which one of the following delictual remedies may be available if Musti wishes to
recover his financial loss from Mary?
1. The actio legis Aquiliae.

2. The actio iniuriarum.

3. The action for pain and suffering.

4. None of the above.

(1)

The correct alternative is [1].

Question 3
Indicate the correct statement:

1. Irrational human behaviour cannot constitute conduct for the purposes of the law of
delict.

2. Human behaviour need not be willed to constitute conduct.

3. The behaviour of an animal can constitute conduct under certain circumstances.

4. The presence of a so-called actio libera in causa confirms the presence of


automatism.
(1)

The correct alternative is [2].

Question 4
Harry’s prize bull breaks through a fence and ventures onto Mark’s farm. The bull is
aggressive and charges at Mark’s employees. The employees clamber into a small tree to
escape the wrath of the bull. The bull begins to bash the trunk of the tree. As the tree
appears to be about to topple, Mark shoots and kills the bull to save his employees. Harry
institutes an action for damages against Mark. Mark may raise the following ground of
justification:

1. Necessity.

2. Private defence.

3. Provocation.

4. Official capacity
(1)
7
PVL3703/201

The correct alternative is [1].

Question 5

Which is the odd one out?

1. The adequate causation theory.

2. The sum-formula approach.

3. Direct consequences.

4. Normative foreseeability
(1)

The correct alternative is [2].

Question 6
Mike’s cow eats and tramples George’s crops. Mike forgot t o close the gate between his
and George’s land. Which remedy or remedies may be available to George?

1. actio de pauperie and actio legis Aquiliae.

2. actio de pastu only.

3. actio de pauperie only.

4. actio de pastu and actio legis Aquiliae.


(1)

The correct answer is [4].

Question 7

Piet clambers over Tumi’s fence with the aim of stealing oranges from Tumi’s orchard. Tumi’s
dog attacks Piet and Piet sustains serious injuries. Piet wants to recover hospital expenses from
Tumi. Which remedy may be available to Piet?

1. Actio de pauperie.

2. Actio de pastu.

3. Actio de feris.

4. None of the above.

(1)

The correct alternative is [4].

7
PVL3703/201
Question 8
Wessel allows Frik to fire some practice shots with his new catapult at an apple on Wessel’s
head. The second shot hits Wessel in his left eye. Wessel institutes a delictual action against
Frik. Frik may rely on the following defence:

1. execution of an official command.

2. provocation.

3. consent to the risk of injury.

4. None of the above


(1)

The correct alternative is [4].

Question 9
Gumba and Paul bear a grudge against Krishaan and agree to teach him a lesson he will not
forget. While Krishaan is shopping at Victoria’s shop, Gumba and Paul damage Krishaan’s
vehicle by hitting it with baseball bats. If Krishaan successfully sues Gumba and Paul in delict,
they will incur:

1. vicarious liability.

2. joint and several liability.

3. liability based on contributory intent.

4. strict liability.
(1)

The correct alternative is [2].

Question 10

According to the following case liability of the media for defamation is based on negligence:

1. Kruger v Coetzee 1966 2 SA 428 (A).

2. Molefe v Mahaeng 1999 1 SA 562 (SCA).

3. National Media Ltd v Bogoshi 1988 4 SA 1196 (SCA).

4. S v Goliath 1972 3 SA 1 (A).


(1)

The correct alternative is [3].

8
PVL3703/201

ANSWERS

Question 1
John takes David’s big and rather aggressive dog for a walk. John incites the dog to
attack Garth. The dog charges at Garth, but Garth shoots the dog and kills it. If David
institutes a delictual action against Garth, on what ground of justification may Garth
rely?
1. Necessity.
2. Private defence.
3. Provocation.
4. None of the above.
(1)

The correct alternative is [1].

Question 2

Markus, a financial adviser, negligently advises Hans to make a bad investment. As a


result of this, Hans suffers a serious financial setback. However, there is no damage to
Hans’s person or property. Which one of the following delictual remedies may be
available if Hans wishes to recover his financial loss from Markus?
1. The actio legis Aquiliae.
2. The actio iniuriarum.
3. The action for pain and suffering.
4. None of the above.
(1)

The correct answer is [1].

9
PVL3703/201

Question 3

Shaun steals a big-screen television set from an electronics store. To get to the
television set, he smashes a window made from expensive security glass. In respect of
the damage to the window, Shaun has:
1. Dolus indirectus.
2. Dolus eventualis.
3. Dolus indeterminatus.
4. Luxuria.
(1)

The correct alternative is [1].

Question 4
Clive is employed by Raymond as a driver in Raymond’s courier business. Clive has
the weekend off, and decides to visit Mary on Saturday evening. On his way to Mary’s
apartment, Clive is involved in an accident due to his own negligence. Clive’s vehicle
and the vehicle of Catherine, the other motorist involved in the accident, are badly
damaged. Catherine discovers that Clive’s financial position is not good. Advise
Catherine on the best course of action:

1. Institute an action against Raymond based on vicarious liability.


2. Institute an action against Raymond and Clive as joint wrongdoers.
3. Institute an action against Raymond based on culpa in eligendo.
4. Institute an action against Clive.
(1)

The correct alternative is [4].

Question 5

Which is the odd one out?


1. The adequate causation theory.
2. The sum-formula approach.
3. Direct consequences.
4. Normative foreseeability.
(1)

The correct alternative is [2].

7
PVL3703/201

Question 6
Mike’s cow eats and tramples George’s crops. Mike forgot to close the gate between
his and George’s land. Which remedy or remedies may be available to George?
1. actio de pauperie and actio legis Aquiliae.
2. actio de pastu only.
3. actio de pauperie only.
4. actio de pastu and actio legis Aquiliae.
(1)

The correct answer is [4].

Question 7

Norman enters into a sexual relationship with Lex’s wife. Which remedy or remedies
may be available to Lex?
1. actio iniuriarum only.
2. action for pain and suffering only.
3. actio iniuriarum and action for pain and suffering.
4. none of the above.
(1)

The correct alternative is [3].

Question 8
Jors allows Maans to fire some practice shots with his new pistol at an apple on Jors’s
head. The second shot makes a hole in Jors’s left ear. Jors institutes a delictual action
against Maans. Maans may rely on following defence:
1. execution of an official command.
2. provocation.
3. consent to the risk of injury.
4. none of the above.
(1)

The correct alternative is [4].

8
PVL3703/201

Question 9
John and Peter bear a grudge against Greg and agree to teach him a lesson he will
not forget. While Greg is in a shopping centre, John and Peter damage Greg’s motor
car by hitting it with hammers. If Greg successfully sues John and Peter in delict, they
will incur:

1. vicarious liability.
2. joint and several liability.
3. liability based on contributory intent.
4. strict liability.
(1)

The correct alternative is [2].

Question 10

According to the following case liability of the media for defamation is based on negligence:
1. Kruger v Coetzee 1966 2 SA 428 (A).
2. Molefe v Mahaeng 1999 1 SA 562 (SCA).
3. National Media Ltd v Bogoshi 1988 4 SA 1196 (SCA).
4. S v Goliath 1972 3 SA 1 (A).
(1)

The correct alternative is [3].

9
SUMMARISED NOTES
LAW OF DELICT – NOTES

Private law: regulates relationships between individuals in a


community.

Role of delict: indicate which interests are recognized by law, under


what circumstances they are protected against infringement and how
a disturbance in the balance is restored.

Definition of delict: a delict is an act of a person, which in a


wrongful and culpable way causes harm to another.

5 requirements:
1. Act
2. Wrongfulness
3. Fault
4. Causation (factual and legal)
5. Damage (damnum iniuria datum)

All must be present before conduct can become a delict.

Difference between delict and a crime:

Delict:
1. Protects private interests (private law)
2. The aggrieved party institutes the action
3. Objective: claim damages as compensation
4. Can’t have attempted delict

Crime:
1. Protects public interest (public law)
2. The state prosecutes
3. Objective: punish the criminal
4. Can have attempted crime

Both are wrongful culpable acts causing damage

Delict and breach of contract:

Delict:
1. Excludes non-fulfillment of a duty to perform (real right)
2. Primary remedy = damages

Breach of contract:
1. Breach = non-fulfillment of a contractual obligation to perform
(personal right)
2. Primary remedy: performance of the contract
Delict, the Constitution and fundamental rights:

The Constitution is supreme and conduct inconsistent with it is


invalid.
Fundamental rights can be limited by the law of general application,
but only to the extent that the limitation is reasonable and justifiable
in an open and democratic society based on human dignity equality
and freedom (Section 36).
Courts must promote the values that underlie this society.
International law must be taken into account.

Direct application: the fundamental rights relevant to or connected


with the law of delict include = right to property, life, freedom, privacy,
dignity, equality, freedom of expression, freedom of religion belief and
opinion, the right to assembly, demonstration, picket, petition,
freedom of association, freedom of trade, occupation and profession.

The state must protect fundamental rights and not infringe them
except unless the requirements for Section 36 have been complied
with.
In case of an infringement that is not justified, anyone who is entitled
to relief can approach a competent court.

Indirect application: all private law rules, principles and those


regulating the law of delict are subject to chapter 2 and applies in
particular to the so-called “open-ended or flexible” delictual principles.
For example, the boni mores test for wrongfulness where policy
considerations and factors such as reasonableness, fairness and
justice may play an important part.
THE ACT (CONDUCT)

Consists of a voluntary human commission or omission:

Elements:

1. Human act: where an animal is used as an instrument a human


act is still present. A juristic person can act through its agents
(company) and be held delictually liable for its actions.

Gijzen: held – delictual liability can occur without there being an


act on the part of the defendant, in the case of land subsistence
cases. Such an act must be wrongful: an act and its consequences
may be separated from each other in time and space, but there
always has to be an act.

2. Voluntary conduct: the act must have been performed voluntarily –


the wrongdoer must have had control over his muscular
movements (i.e. If it is susceptible to control b the will of the
person involved).

One’s voluntary conduct does not have to be willed or desired, as


can be seen in the case of S v Russell: X forgot to warn others that
an electric current had been switched on. Someone was
electrocuted. Russell stated that he wasn’t guilty because he hadn’t
willed or desired the outcome.
Court held: that the test was whether he was able to utter a
warning = GUILTY.

3. Commission and Ommission: conduct can be in the form of a


commission or omission. Liability for an omission is in general
more restricted than liability for a positive act (commission). The
law is hesitant to find that there was a legal duty on someone to
act positively and so to prevent damage to another.

The Defence of Automatism:

The voluntary conduct on the part of the defendant is a requirement


for delictual liability.
The defendant could argue that the conduct complained of doesn’t
satisfy the requirement of voluntariness.

This is where someone acted mechanically = sleep, unconscious,


fainting fit, absolute compulsion (X grabs Y’s hand and stabs P using
Y’s hand), epileptic fit, serious intoxication, black out.
If these are present a person is incapable of controlling his bodily
movement = purely mechanical action and that person raises the
defence of automatism
Molefe: the defendant doesn’t bear the onus to prove that he was in a
state of automatism, its for the plaintiff to prove that the defendant
acted voluntarily.

Dhlamini: X was sleeping on the floor in a room with others when he


had a nightmare; he then stabbed and killed Y with a knife while
under the influence of his dream. He was not convicted of any
crime.

Mkize: X stabbed and killed Y with a knife while X was having an


epileptic fit, he was acquitted of murder.

Du Plessis: X was charged with negligent driving as he injured a


pedestrian. He was 72 years old and experienced a black-out due
to low blood pressure. He was found not guilty.

In respect of sane automatism (where it’s not a consequence of mental


illness) the onus is on the plaintiff to prove that the defendant has
acted voluntarily and therefore not mechanically.

Automatism doesn’t mean that there’s no voluntary act whatsoever by


the defendant which caused the damage, but only that the conduct in
question wasn’t voluntary.

Antecedent liability:

The defence of automatism will not succeed if the defendant


intentionally created the situation in which he acts involuntarily in
order to harm another (actio libera in causa).
The defendant may not successfully rely on the defence of automatism
where he was negligent with regard to his automatic conduct.
This is where the reasonable man would have foreseen the possibility
of causing harm while in a state of automatism.
E.g. drinking while knowing or reasonably foreseeing that you will
later drive a car – knowing you may suffer an epileptic fit but still
driving.

Victor: X was convicted of negligent driving despite the fact he caused


the accident. During an epileptic fit, Victor collided with a pedestrian
and another car as the evidence revealed. He’d been suffering from
epileptic fits for the last 13 years and had insufficient reason to
believe he would not have a fit on that day.
WRONGFULNESS

For liability to follow an act, prejudice must be caused in a wrongful


or unreasonable manner. Without wrongfulness defendant cannot be
held liable.

Determination of wrongfulness is essentially a dual investigation,


looking at:
1. Was there an infringement of a legally protected interest – the
act must have a harmful consequence.
2. If the interest has been prejudiced, legal norms are used to
determine if it occurred in a legally reprehensible manner
(unreasonable manner) – boni mores test.

Act and consequence:

An act is only delictually wrongful when it has as its consequence the


factual infringement of an individual interest.
An act on its own without a harmful consequence can never be held
delictually wrongful – e.g. X races down main street at 160 km in peak
hour traffic, his act isn’t considered wrongful in delict as there has
been no infringement of an interest.

An act and its consequence are always separated by time and space.

Pinchin v Santam Insurance Co Ltd:

A pregnant woman was involved in a car accident caused by the


defendant’s negligence. When the child was born it was found to suffer
from serious brain damage.

Compensation was claimed from the defendant on the child’s behalf.


But at the time of the defendants act the child was in ventre matris
and as a result the child had no legal personality and therefore no
legally protected interest that could be infringed.

The judge – was of the opinion that the child did have an action but it
failed because there was no evidence that the accident caused the
brain injuries. The judge based his view on the nasciturus fiction – an
unborn child is regarded as having been born whenever it’s in the
child’s interests.
Joubert argues: it’s unnecessary to rely on the nasciturus fiction to
find that the child would have an action – because the act and its
consequences are separate both in time and space – the child need
not have legal capacity at the time of the act.
The defendant’s conduct resulted in a harmful consequence much
later.

In RAF V Mtati it was held that the nasciturus fiction does not always
offer a solution in all circumstances.

BONI MORES TEST: BASIC TEST FOR WRONGFULNESS

Boni mores test: The legal convictions of the community (boni mores
test) as the basic test for wrongfulness: The general norm to see if an
infringement of interests is unlawful, is the legal convictions of the
community = boni mores. This is an objective test. (Steenkamp NO)

Basic question: whether according to the legal convictions of the


community and in light of all the circumstances of the case the
defendant infringed the interests of the plaintiff in a reasonable or
unreasonable manner.

The application of the Boni Mores test = the ex post facto weighing up
of the interests of the defendant actually promoted by his act and
those which he infringed. (Tommi Meyer)

The court must weigh the conflicting interests of the defendant and
plaintiff in light of all the circumstances to decide whether the
infringement of the plaintiff’s interests was reasonable or not.
This is done by looking at factors: the foreseeable loss, the costs to
prevent loss, the motive of the defendant, extent of the harm etc.

Subjective factors
The objective nature of the Boni mores test appears from the fact that
subjective factors such as the defendant’s mental disposition,
knowledge, and motive normally don’t play a role in determining
wrongfulness.
To determine wrongfulness it’s the weighing up of interests to
determine whether the defendant acted reasonably or not and
subjective factors are normally irrelevant.

In exceptional circumstances subjective factors do play a role in


determining wrongfulness – a defendant who plants deciduous trees
on the boundary of his property for the sole purpose of injuring his
neighbour = wrongful.
Therefore, an improper motive for the sole purpose of injuring another
will also render reasonable conduct, unreasonable. The fact that the
defendant actually knew or subjectively foresaw that he plaintiff would
suffer damage as a result of his conduct is taken into consideration in
determining wrongfulness in cases of pure economic loss or
omissions.

Application of boni mores test:

Boni Mores test is the basic test for wrongfulness but still functions at
a supplementary level because the convictions of the community
concerning what is reasonable or unreasonable have over time found
expression in common law, statutory norms, grounds of justification
etc. so its seldom necessary to apply the Boni Mores test directly.
There are 2 ways the Boni Mores test is applied as a supplementary
test for wrongfulness:

1. When the wrongfulness of the defendants conduct doesn’t


appear from the violation of a pre-existing norm or the
lawfulness thereof doesn’t appear from the presence of a
recognized ground of justification.
E.g. champion swimmer sees a drowning child and does
nothing, or if a body builder would not be put in danger of
helping an old granny who’s being beaten up by a younger and
smaller person, or for example, the case of Ewels when an off
duty police officer failed to stop an assault.

2. When borderline cases need refinement. In other words, to


determine if the defendant transgressed the limits of private
defence by his conduct, or if consent to violation of legal
protection of interests should be tolerated etc. a good example
from case law is S v Goliath under compulsion from Y and
fearing for his own life helped Y to kill Z – X raised the defence
of necessity.

So look to more precise methods of determining wrongfulness:


a) Infringement of a subjective right
b) Non-compliance with a legal duty to act
DOCTRINE OF SUBJECTIVE RIGHTS:

Tommie Meyer Case: the courts accepted the doctrine of subjective


rights = wrongfulness consists of the infringement of a subjective
right.
Said there was a duel relationship:

• Subject-object relationship: provides the holder of the right with


the power to use, enjoy and alienate the object of his right. The
limits of his rights are determined by the law.

• Subject-subject relationship: the holder of the right can enforce


his powers over a legal object against all other legal subjects

Nature of subjective rights:


1. Real rights: ownership over things
2. Personality rights: good name
3. Personal rights: payment etc
4. Immaterial property rights: intellect/poem/artwork
5. Personal immaterial property rights: earning capacity

Look at whether the right has been infringed: was the holder of the
right was disturbed in the use and enjoyment of his right (if the
subject-object relationship has been disturbed and whether the
infringement complained of took place in a legally reprehensible
manner.
If established the conduct is wrongful.

WONGFULNESS AS A BREACH OF LEGAL DUTY:

First, one has to ask whether, according to the boni mores test, a legal
duty has been breached. In cases of omissions or the causing of pure
economic loss, instead of asking whether a subjective right has been
infringed, it’s better to ask whether the defendant has a legal duty to
prevent the harm.

In Van Eeden v Minister of Safety and Security the court stated that
the test is one of reasonableness and the defendant is under a legal
duty to act positively and if it’s reasonable to expect the defendant to
take positive steps to prevent harm.

Such a duty is often referred to as a duty not to act negligently as


seen in the case of Shabalala v Metrorail.
LIABILITY FOR AN OMMISSION:

General rule: a person doesn’t act wrongfully in terms of the law of


delict where he fails to act positively to prevent harm to another. This
means that there is no absolute duty to prevent loss.

Liability follows only if the omission was in fact wrongful – where a


legal duty rested on the defendant to act positively to prevent harm
from occurring and he failed to comply with that duty.

PRIOR CONDUCT:

A person acts prima facie wrongfully when he creates a new source of


danger by means of a commission and then fails to eliminate that
danger, with the result that harm is caused to another person.
Prior conduct isn’t a prerequisite for the existence of a legal duty – butat
one stage this was the only category where liability was imposed ona
failure to act.

The requirements for legal duty was first introduced in Halliwell case
which stated that prior conduct had to be present before one would be
liable for an omission.

MUNICIPLITY CASES:

Halliwell v JHB municipal council:


The council laid cobblestones adjacent to tramlines in road surface
(experiment) – these wore smooth. H was in his horse cart, they
slipped; injuries:
AD: where road authority constructs, in such a way as to introduce a
new source of danger which would other wise not have existed, it
must take due steps to guard against that danger.
I.E. with the creation of a new danger (prior conduct) legal duty arises
Failure to comply with the legal duty = OMMISSION.
Silva’s Fishing Corporation v Maweza (1957):
The defendant was the owner of a fishing fleet, the engine of one of his
boats failed and the boat drifted out to open sea for 9 days during
which the defendant failed to take steps to rescue it and it was
eventually lost in a storm.
The plaintiff’s husband drowned and she instituted action against the
defendant.

Majority: 3/5: prior conduct – present since the defendant created a


potentially dangerous situation by providing the boat and consenting
to the fishing run from which he would benefit financially.
Therefore a legal duty rests on the defendant to take steps to rescue
the crew.

Minority: STEYN JA – was critical of the Halliwell approach ….. That


liability for omissions was based on prior conduct ONLY.

Regal v African Superslate (1963):


The applicant applied for an interdict to compel the respondent, his
neighbor who lived further up the river, to take steps to prevent slate
being washed down by the river onto the applicants land.
The court decided that prior conduct WASN’T AN INDESPENSIBLE
REQUIREMENT FOR A LEGAL DUTY TO ACT.
It was suggested that you look to see if the loss could be prevented by
reasonable measures.

In this case the steps that would have been necessary to prevent the
washing down of the slate – building of a dam wall – not in relation
therefore there was no duty on the respondent to prevent the loss.

Problem: in this case it was thought that the rejection of prior conduct
was limited to interdicts.

Minister of forestry v Quathlamba (1973):


In this case the AD had to decide whether a landowner was liable for
damage resulting from his failure to control a fire, which, through no
action of his, broke out on his property.
The court 1st had to decide whether the defendant was under a duty to
control the fire – judge held that a mere omission did not found
liability – LIABILITY FOR AN OMMISSION WAS DEPENDENT ON THE
EXISTANCE OF A LEGAL DUTY TO ACT.
The law does in certain circumstances, where there has been no prior
conduct, impose a duty on the landowner to control a fire, which
breaks out on his land.

Confirmed the Regal decision – PRIOR CONDUCT IS NOT THE ONLY


CRITERION FOR ESATBLISHING A LEGAL DUTY.
Minister of police v Ewels (1975) NB
Here the respondent claimed damages from the appellant on the
ground that a policeman in the service of the appellant failed to take
steps to prevent the respondent from being assaulted and injured.

In this judgment, the generally accepted view that wrongfulness is


determined by the legal convictions of the community has now been
applied to omissions.
It’s a more flexible approach – legal duty arises when the legal
convictions of the community demand as much. Failure to comply
with this duty = wrongful omission.

Any doubt that prior conduct isn’t indispensable for the existence of a
legal duty to the municipality cases was removed in:

Van der Merwe Burger v the municipality of Warrenton (1987)


Held: prior conduct as a criterion for establishing a legal duty was
abandoned in Ewels.
So the court applied the legal convictions of the community test = the
municipality should have foreseen the damage and was liable.

Rabie v Kimberly municipality (1991):


A municipality, which had been aware that a traffic light was
malfunctioning but had failed to properly investigate and repair it,
was held liable for damages resulting from an accident at the
intersection.

Cape Town Municipality v Bakkerud


This case confirmed the judgment in EWELS and held that legal
convictions of the community can even, in absence of “prior conduct”,
place a legal duty on the municipality.
CONTROL OF A DANGEROUS OBJECT:

Control over a dangerous (or potentially dangerous) object (fire etc)


can be a factor in determining whether a legal duty rested on a person
to prevent someone being injured by a particular situation.

2 relevant questions:
1. Whether there was actual control – whether control was present
can be adduced from the fact the defendant had actually taken
control over a dangerous situation, from the fact the defendant
was owner of the dangerous object or from statutory provisions
placing control on the defendant.

2. In light of the control, whether a legal duty rested on the


defendant to take steps to prevent damage resulting from his
omission to exercise proper control – the fact that the defendant
had been in control of a dangerous (potentially dangerous)
object isn’t necessarily enough to establish a duty to take
precautionary measures but may be a factor.

Van Eeden – police had a legal duty to prevent a dangerous prisoner


from escaping and raping and robbing the plaintiff.

It’s been held that an occupier of property where dangerous conditions


exist has a legal duty to prevent injury to persons who visit the
premises (even trespassers) – Tsogo Sun Holdings v Qing-He Shan:
Prevented their clients from carrying weapons to prevent harm to
other clients.

A legal duty can also rest on the owner or controller of property to


control a fire on such property (Minister of Forestry v Quathlamba).
The same applies to someone in possession of a firearm or dangerous
animal (S v Fernandez).

If a legal duty exists, injury resulting from the omission to control the
dangerous situation is prima facie wrongful.
To avoid liability, it has to be clear either that the defendant’s
wrongful omission was lawful or he had taken reasonable steps to
prevent injury.
S v Fernandez: F had a baboon in a cage – the cage fell into disrepair
and the baboon escaped – it bit a child who later died from the bite.

The court said


1. that he had a duty to see that the baboon wasn’t outside the
cage
2. he took no steps to ensure the animal wouldn’t get out
3. he should have foreseen the likelihood of the baboon injuring
someone should it get out.

Ablort – Morgan v Whyte bank farms: said that if there was a legal
duty – focus on whether the defendant knew or ought reasonably to
have known about the danger and whether reasonable steps were
taken in the circumstances.
Look at: a) the probability of damage
b) Possible action by the defendant
c) Chances of success
d) Cost of measures
e) Interests of individuals and the community

RULES OF LAW:

Law (common or statute) places an obligation on a person to perform


certain acts. At common law, the owner of a lower land is obliged to
provide lateral support for neighbour’s property.

Statutory law:
1) Minister of police v Ewels: duty on police to prevent a crime.

2) Charmichele v Minister of Safety and Security: Applicant was


viciously assaulted by person awaiting trial but out on bail. Q:
Was there a legal duty on the investigating officers and
prosecutor to protect its citizens? Court a quo and Appeal court
said no. Constitutional court said – when determining whether a
legal duty to act rests on the police, the interests of the parties
and conflicting interests of the community must be weighed. It
said that relevant factors must be taken into account and that
such a case must be weighed against the constitution,
promoting human dignity, equality and freedom and in which
government has positive duties in which to promote and uphold
such values. In terms of Police Act, there is a legal duty placed
upon the police to protect its citizens, and the prosecutor has a
legal duty to furnish the court with relevant information
concerning the question on whether bail should be granted or
not.
Held: The court a quo was wrong in granting absolution from
the instance. The case was sent back to high court for the trial
to continue.
3) Kadir v Minister of Law and Order: The police failed to record
and the necessary information relating to the driver and the
identity of the vehicle at the scene of an accident. Court said
there was a legal duty on the police. On appeal – court
disagreed and said the duty on the police in this case, did not
have the same weight as Ewels. Therefore, policeman cannot be
held personally liable in terms of delictual law.

4) Minister of Safety and Security v Van Duivenboden: X possessed


two firearms, abused alcohol, was aggressive and mistreated his
family. In an argument, X killed his wife and injured his
daughter with his guns. The police knew of X’s behavior and
failed to take away X’s firearms. Y issued claim against the
police. They claimed there was a legal duty on the police to take
away firearms from incompetent people. A single judge rejected
the claim, but on a appeal to three judges who agreed that the
police did have a legal duty to prevent such damages by
screening applicants of firearm licenses.

RELATIONSHIP BETWEEN THE PARTIES:

This special relationship is an indication that one party has a legal


duty towards the other to prevent harm – parent-child. For example, a
policeman/citizen (Ewels); employer/employee (Silva’s Fishing);
Doctor/Patient (Stewart)

OFFICE:

Sometimes the person’s occupation or office he holds places a legal


duty on him to conduct himself in a particular manner in relation to
the public. In Macadamia Finance Ltd v De Wet, the defendants, we
were liquidators, inter alia failed to insure the assets of the first
company under their control, and the defendant’s breach of that duty
rendered them liable for the first plaintiff’s resultant loss.

CONTRACTUAL UNDERTAKING:

A enters into a contract with B in which he undertakes to ensure the


safety of C – lifeguard.
CREATE THE IMPRESSION THAT A 3RD PARTY WILL BE
PROTECTED:

One party acts in reasonable reliance on the impression created by


the other party that the latter will protect the person and property of
the former, a legal duty rests on the party creating the impression to
prevent prejudice to the part acting in reliance on that impression.

Compass Motors Industries (Pty) Ltd v Callguard:

In this case a security firm was in control of certain premises in


performance of a contract to minimize the chances of theft and
damage.
Question: whether the security firm could be held delictually liable by
a 3rd party for the loss of property on the premises.
Court decided with reference to the Boni Mores – the defendant had a
legal duty towards the plaintiff whose car was lawfully on the
premises, but the defendant escaped liability due to the absence of
negligence on his part.

THE GENERAL WRONGFUL CRITICISMS:

Instead of looking at the legal duty to act, where there is no strong


indication, rather look at reasonableness of the failure to act (for
example, the champion swimmer).

BREACH OF A STATUTORY DUTY:

The causing of damage by means of conduct in breach of a statutory


duty is prima facie wrongful.
If the breach infringed the plaintiffs interests in a legally reprehensible
manner = wrongful.

Patz v Greene and co:


A who traded in the vicinity of a mining compound applied for an
interdict against B, who ran a similar trade on claim land at the
entrance of the compound. A based his application on the fact that
trading on claim land was prohibited by statute.
On appeal it was decided, that the infringement of another’s good will,
is unlawful if it’s caused by conduct expressly prohibited by statute.

The plaintiff has to prove the following:


• That the plaintiff had a private law remedy at his disposal
• Statutory duty was for the benefit of the plaintiff
• Nature of the harm falls within the scope of statute
• The defendant transgressed statute
• There was a causal nexus between the defendant’s
transgression and the plaintiff’s harm.
WRONGFULNESS
GROUNDS OF JUSTIFICATION

Grounds of justification are special circumstances in which conduct


that appears to be wrongful is rendered lawful = the violation of the
interest isn’t unreasonable or contra boni mores.

Grounds of justification exclude wrongfulness.

The ONUS is on the defendant to prove that his otherwise wrongful


conduct is justified in the circumstances and therefore according to
the legal convictions of the community his conduct ISNT wrongful.

Grounds of justification include:


1. Defence
2. Necessity
3. Provocation
4. Consent
5. Statutory authority
6. Official capacity
7. Power to discipline

PRIVATE DEFENCE

Definition: Private defence is present when the defendant directs his


actions against another’s actual (or imminently threatening) wrongful
act, to protect his own legally recognized interests or the interests of
another.

Both the attack and the defence must meet requirements for private
defence to apply:

Requirements for the attack:


1. The attack must consist of a human act: it must be a
commission/omission. One can use an animal as an
instrument.

2. The attack must be wrongful: it must threaten a legally


protected interest without justification – against interests which
are worthy of protection = life, bodily integrity, honour, property
etc. One cannot act in defence of a lawful attack – person
cannot rely on defence in resisting a policeman who is in the
process of arresting him lawfully.
One may not act in defence against someone who has the right
to violate ones interests. – 2 people are involved in a duel to
which they both agreed there is no question of defence because
they are both acting unlawfully.
Putative defence doesn’t constitute private defence.
3. The attack must have already commenced or be imminently
threatening – BUT not yet completed:
Magohlwane: Y threatened to kill X with an axe – he took his
food and clothes – X ran home and got a knife – Y threatened
again and X stabbed Y = PD because the attack hadn’t yet
terminated.

Not a requirement:
1) Fault on the part of the aggressor isn’t a requirement –
therefore one may act in defence even against someone who
is incapable of having a blameworthy state of mind for
example, someone who is mentally ill.
2) It’s also not a requirement that the attack be directed at the
defender – it can also be directed at a 3rd party – R attempts
to hit S and X intervenes and strikes R.

Requirements for the defence:

1. The defence must be directed against the aggressor.

2. The Defence must be necessary to protect the threatened right:


the act of defence must be the only reasonable alternative to
protect the threatened interest – if there was a less detrimental
reasonable option, the defender must have used it.
Flee then force.
The opinion is that the defender should rather flee, unless such
flight exposes him to danger (shot in the back)

3. The defence cannot be more harmful than necessary to avert the


attack: the defence must be reasonable – it must not be out of
proportion to the attack. The value of the interests and measure
of defence can differ – but an extreme imbalance is
unacceptable – X slaps Y so Y shoots and kills X
Van Wyk case

Putative defence: X cant rely on PD if he thought he was in


danger when he wasn’t – you determine X’s act objectively (but
he wont be liable for murder because he lacks fault) = we must
consider the facts of the individual cases objectively in the light
of the actual facts and NOT what X thought them to be.
If X exceeds the bounds of PD – his act makes him the
unlawful attacker.
PRIVATE DEFENCE WITH REGARD TO PROPERTY:

The question is whether someone can kill another person to protect


their property and rely on private defence?
Before: Killing someone to protect property can NEVER be justified
using private defence.

Now: In the case of S v Van Wyk, Van Wyk set up shot gun in his shop
to protect his prop against thieves, who had been a problem in past. A
shot from gun killed burglar & VW prosecuted for murder. In this
case, the court had to decide:
1. Whether you can rely on doctrine of defence when 1 has killed or
injured another in order to protect your prop?
2. Whether grounds of defence were exceeded in this case? – Look at
whether steps taken by him were only way of warding off the
attack.

Question: could he succeed with Private Defence?


Judges decided that setting up of gun was indeed only reasonable
possibility if he wanted to protect his prop. VW couldn’t be expected to
sleep in his shop every night and burglar proofing, night watchmen
and watchdogs had proved ineffective. Police couldn’t guard shop and
alarm system would have been useless. VW hadn’t set up gun in way
as to demonstrate any motive to kill. He also put notice warning of
danger on door of shop.

VW didn’t exceed bounds of justification – there must be no less


harmful method of keeping property and where possible one should
issue a warning shot.
This case isn’t general authority for killing in defence of property –
whether one is entitled to kill in defence of prop will depend on facts of
each case.
NECESSITY

Definition: a state of necessity exists when the defendant is placed in


a position by superior force (vis maior) that he is able to protect his
interests or those of someone else, only by reasonably violating the
interests of an innocent 3rd party.

E.g. A acts out of necessity when he knocks out the window of B’s
burning house in order to rescue C, who is trapped inside the house.
A wont be liable to B for the damage to the window because the
breaking of the window is lawful – necessity justifies A’s conduct.

Distinction between necessity and Private Defence:


Private Defence= the defence is directed at the attack by the
wrongdoer.
Necessity = conduct violates the interests of an innocent 3rd party.

Guidelines to determine necessity:

1. An actual state of necessity must exist: it is not however clear


whether the defendant can rely on necessity when he created
the state of necessity. There is authority which supports both
views. However, the view that a person may rely on necessity
even though he created the situation himself enjoys more
support. It is argued that the conduct creating the emergency
and the defensive act should be kept apart. If the conduct
creating the emergency constitutes a crime or causes damage,
the defendant should be held liable for it; nevertheless, such
conduct should not preclude him from acting out of necessity in
order to escape from the emergency.
For example, where a person starts a fire inside a house and
then has to break a door in order to escape. On the basis of this
argument, he should be held liable for the damage caused by
the fire, but not for the damage caused by his escape because
such damage was self induced.
According to this argument, every act committed out of
necessity is lawful, irrespective of whether it was caused by the
defendant or not, but damage caused thereby may still be
actionable because it is (factually and legally) causally
connected to a preceding wrongful act.

2. The possible existence of a state of necessity must be


determined objectively: the test is objective = you look at the
actual facts and not what the defendant thought them to be.
The fact that the defendant was in a state of terror was also not
relevant.
Putative (imagined) necessity isn’t necessity – S v Pretorius:
Appellant was found guilty of exceeding the speed limit – it
appeared that his 2 year old child had taken a number of pain
tablets and the appellant believed that the child’s life was in
danger and wanted to get the child to hospital as quickly as
possible – on appeal the court found he acted out of necessity
and set the conviction aside – In REALITY there was no question
of necessity as the child’s life was never in danger – the only
symptom from the overdose would have been an upset stomach.

3. The state of necessity must be present or imminent: it must not


have terminated or be expected only in the future.

4. The defendant need not only protect his own interests, but can
protect the interests of another: A acts to protect his child.

5. Not only life or physical integrity, but other interests like


property may be protected out of necessity: honour, identity – to
protect B’s physical integrity, A provides C, who intends
assaulting B with false information about B’s whereabouts,
privacy – doctor is compelled to examine an unconscious
person, freedom – during a storm a captain of a ship
temporarily locks passengers below deck to prevent the ship
from going down, and feelings – the doctor convinces a wife who
is seriously ill to undergo an operation, against the wishes of
her husband, may be violated in a state of emergency.

6. A person cant rely on necessity where he’s legally compelled to


endure the danger.

7. The interest, which is sacrificed, must not be more valuable


than the interest, which is protected: the defendant cannot
cause more harm than is necessary.

8. Whether necessity can justify homicide:


English law – R v Dudley and Stephens: 2 people who were
drifting on a small boat after a ship disaster, were convicted of
murder after they cut the throat and ate the weakest of the
passengers to stay alive after 20 days at sea – the court rejected
the plea of necessity as necessity can never justify killing of an
innocent person.
In S v Goliath the AD held that homicide may be justified in a
state of necessity: in this case A under the compulsion from B
and fearing for his own life, helped B to kill C – court said A’s
defence of necessity could be upheld. The law recognizes that a
person's own life is more important to him than the life of
another.

9. The act of necessity must be the only reasonable possible means


of escaping from the danger: the act must be necessary to
protect the threatened right – the defendant must have no other
reasonable means available to him to prevent the infringement
of another’s rights – if the defendant can escape from the
emergency by fleeing, he must flee.
PROVOCATION

Definition: provocation is present when a defendant is provoked or


incited by words or action to cause harm to the plaintiff.

Provocation is a ground of justification, which renders the defendants


conduct lawful, therefore the plaintiff can’t claim damages (B v C).
Provocation is assessed objectively by weighing the proactive conduct
against the reaction to it by using the boni mores test -
(reasonableness).

Difference between private defence and provocation:


Private Defence: the defendant averts the attack which is immanent or
already commenced but not yet terminated = act of defence
Provocation: defendant reacts immediately after the provocative
conduct has already terminated = act of revenge.

R v Van Vuuren: the appellant was prosecuted for assault – he had


grabbed the complainant by the arm after the complainant insulted
the appellant’s wife. The appellant raised private defence as a ground
of justification and this was accepted by the courts – he grabbed the
complainant’s arm to prevent him from continuing his insults.
BUT it was clear that the verbal abuse had already terminated when
the appellant grabbed the complainants arm = provocation
BUT he wouldn’t have been able to rely on provocation because it was
a physical attack against provocative words!!!!

Provocation in the case of physical assault:

General rule: provocation isn’t a complete defence where provocative


words precede a physical attack – this is so even where the words
were gravely insulting and defamatory – X who slaps Y merely because
Y verbally provoked him (Bennet v Minister of Police).
Exception: Parker v Scott – can be a complete defence in certain
circumastances.

BUT: where the provocation takes the form of a physical assault,


provocation can serve as a ground of justification for a counter-
assault of a physical nature.
Provocation will be a complete defence against an action based on
assault if 2 requirements are met:
1. The provocative conduct must be of such a nature that the
reaction to it by means of a physical assault is reasonable =
excusable – you look at this objectively, would the reasonable
person in the position of the defendant have acted the way the
defendant did.
2. The conduct of the provoked defendant must be an immediate and
reasonable retaliation against the body of the plaintiff – so the act
of revenge must not be out of proportion – the different
interests must be of equal value or similar nature. Provocation =
weighing up of interests: here the interests of the person who is
provoked are compared with the interests of the person who
provoked.

Provocation in cases of defamation and insult:

Defamatory or insulting allegations made during an argument in


reaction to provocative conduct, may be justified in certain
circumstances.

REQUIREMENTS:
1. The provocation must be of such a nature that a reasonable
person in the position of the defendant would also have acted by
defaming or insulting the plaintiff = test is objective
2. Defamation or insult in retortion must stay within the
prescribed limits – the limits aren’t exceeded when the
defamation or insult: a) immediately or directly follows the
provocation and b) isn’t out of proportion to the provocative
conduct.

Principle of Compensatio: (2nd requirement) – where 2 people have


defamed or insulted each other in such a manner that the one
instance of defamation or insult isn’t out of proportion to the other. In
other words, the two insults cancel each other out. (Bester v Calitz)
CONSENT

Definition: where a person legally capable of expressing his will gives


consent to injury or harm, the causing of such harm will be lawful.

Consent is a ground of justification: the person suffering harm waives


his right to the extent that he permits the defendant to violate his
interests.

Volenti non fit inuria: the defendant isn’t liable where the injured
person has consented to injury – he who consents cannot be injured.
It describes BOTH forms of consent.

Forms of consent:
1) Consent to injury: the injured party consents to specific harm
– e.g. A consents to B’s removing his appendix – E consents to
the barber F to cutting his hair.
Esterhuizen v Adminstrator Transvaal: Plaintiff submitted to
radium treatment and sustained serious injuries. Court said: if
a person gives consent thinking it will help them or that the
procedure would be without risk, it is not informed consent.
In such a case, the consent MUST be informed consent.

2) Consent to the risk of injury: the injured party consents to the


risk of harm caused by the defendants conduct – a rugby
player accepts the risk that he may be injured in a tackle. =
The injured person will not be able to hold the person who
injured him, delictually liable because he has consented to the
risk of such harm.
Boshoff: Plaintiff was hit on the head with a racket during a
game of squash and sustained injury to his eye. Court held
that the plaintiff cannot claim damages because he consent to
risk of injury and such consent was not contra boni mores.

Voluntary assumption of risk = sometimes referred to as consent to


injury or contributory intent.
Characteristics of consent:

a) Consent to injury is a unilateral act – it need not be known to


the defendant and it can be unilaterally revoked at any stage
preceding the defendant’s conduct.
b) Consent is a legal act, which restricts the injured person’s rights
= the consent must be apparent or evident.
c) Consent must be given either expressly (words) or tacitly
(conduct) – incitement, encouragement or invitation = consent
BUT submission or mere knowledge = no consent.
d) Consent must be given before the prejudicial conduct – approval
given after the act ISNT consent.
e) As a rule the prejudiced person himself must consent – only in
exceptional circumstances can consent be given by someone
else.

Consent is a question of fact – where D thought that consent had been


given, when in fact it hadn’t, no ground of justification exists and he
acted wrongfully. (R v K)

Requirements for valid consent:

1. Consent must be given freely and voluntarily –


R v McCoy: X, an airhostess was remanded for failing to fasten
her seatbelt – so to not loose her job she allowed the boss to
slap her bottom 6 times – the boss was charged with indecent
assault and his defence was that X had consented.
It was held: that her consent wasn’t given freely and voluntarily
due to the fact that she was about to loose her job.
2. The person giving consent must be capable of volition – must be
intellectually mature enough to appreciate the implications of
his acts and he must not be mentally ill or under the influence
of drugs.
3. The consenting person must have full knowledge of the extent of
the prejudice – (Esterhuizen)
R v C - R was asleep and was woken by C who was having sex
with her – she thought it was her husband and later saw it
wasn’t. C was charged with rape – using consent as a defence –
it didn’t constitute valid consent, as it wasn’t based on full
knowledge.
Castell v De Greef – The court said that the test is determined by
evidence led on what the reasonable doctor would’ve told his
patient in the same situation.
In medical procedures a medical practitioner has a duty to
inform a patient of any material risks connected to the
treatment.
The extent of the doctor’s duty should be established.
In casu the court used the reasonable doctor test: look at what
the reasonable doctor would have told the patient in the
circumstances.
On appeal reasonable patient test (material risk test) was
preferred: the doctor’s duty to inform is to be established with
reference to the needs and expectations of the particular
patient. For a patient's consent to be a ground of justification
that excludes wrongfulness – the doctor is obliged to warn the
patient of any material risk:
A risk will be material if:
1. A reasonable person in the patient’s position, if warned of
risk, would have attached significance to it.
2. The doctor is or should reasonably be aware that the
particular patient if warned would attach significance to
it

4. The consenting party must realize or appreciate fully the nature


and extent of the harm – the plaintiff must comprehend and
understand the nature and extent of harm.
5. The consenting party must subjectively consent to the
prejudicial act.
6. The consent must not be contra boni mores.

Exceeding the limitations of consent:

The impairment must fall within the limit of the consent. For example:
Vorster the plaintiff was a passenger in a car that was dicing with
another. The car overturned and the plaintiff was injured. The court
held that the plaintiff did consent, but did not however, consent to
gross negligence of the driver. Therefore, the plaintiff was able to claim
damages.
Burger v Administrator Kaap: A patient gave consent to Dr R to perform
surgery. However, years later the patient discovered that it was in fact
Dr L who performed the surgery. The court held that “the debt arising
from Dr L was different to that of Dr R. Therefore, consent was not
given and exceeded the limitation of consent.

Pactum de non petendo in anticipando:


This is a contractual undertaking not to institute action against the
actor = not hold the actor liable.
There is no doubt that the actor committed a delict, but the
prejudiced person undertakes not to hold the actor liable.
Wrongfulness isn’t excluded – only the action against him is.
The prejudiced person loses the remedy, which he would otherwise
have had at his disposal – he waives his action.

E.g. Jameson’s Minors v CSAR:


The deceased was injured in a train accident caused by the
defendant’s negligence, and subsequently died.
He was a passenger traveling on a free pass issued to him on
condition that the railways wouldn’t be liable in the event of injury
caused through the negligence of the railways.
An action therefore couldn’t be instituted against the railways.
The court held: such an agreement was no defence to the actions
instituted by the deceased’s dependents.
BUT if the deceased had merely been injured he wouldn’t have been
able to institute action against the railway.
OFFICIAL CAPACITY AND OFFICIAL COMMAND

Official capacity: certain public officials (law enforcement and


judicial officers) are authorized by law to perform certain acts.
Should they cause damage in the process, their conduct will be
justified (lawful) and they will not be liable.

Should the official exceed his authority, he acts unreasonable and


wrongfully and cannot be held liable.
Official capacity is exceeded, for example, where the official acts with
malice.

May v Udwin: where a claim was based on alleged defamation


instituted against a magistrate failed.
Held: to justify defamatory remarks the magistrate can rely on official
capacity – BUT this isn’t absolute.

Execution of an official command: infringements of interests in


carrying out a lawful command (where a constable shoots a fleeing
murderer at the order of his officer = not wrongful.

Question: when execution of a wrongful command can constitute a


defence?
S v Banda laid down the following requirements for the defence:
1. The order must be given by a person in position of lawful
authority over the A
2. There must be a duty on the A to obey the order given
3. The A must have done no more harm than was necessary to
carry out the order

According to case law there is no absolute duty on subordinates to


obey the orders of their superiors and this defence should be limited.

To determine whether the order is wrongful, look at the judgment of


the reasonable person.
In cases where a duty to obedience of wrongful orders applies, the
ground of justification is really necessity in the way of compulsion and
not official command.
STATUTORY AUTHORITY

Definition: a person doesn’t act wrongfully if he performs an act


(which would otherwise have been wrongful) while exercising a
statutory authority.

Harmful conduct authorized by statute is reasonable and therefore


lawful.
BUT:
1. Statute must authorize the infringement of the particular
interest
2. The conduct must not exceed the bounds of the authority given
by the statute.

Guidelines:
a) The question whether statute authorizes the infringement of the
interest concerned, depends on the intention of the legislature –
interpretation of statutes:
1. If the statute is directory = the infringement of a private
interest is authorized = injured person cant get
compensation
2. If the statute is permissive and makes no provision for the
payment of damages = presumption that the infringement
isn’t authorized
3. This presumption above falls away if authority is entrusted
to public body acting in public interest.
4. If the authorized act is localized = presumption that the
infringement is authorized
5. If the authorization is permissive and general, not localized
and doesn’t entail and infringement of private interests =
legislature didn’t intend the interests to be infringed.

b) To determine whether the authorized act exceeds the bounds of the


authority the following are taken into account:
1. It must have been impossible for the defendant to exercise the
powers without infringing the plaintiffs interests (onus on
defendant)
2. It must have been impossible to prevent or minimize the
damage by taking reasonable precautions or by using
another method (onus on plaintiff).

If the defendant exceeded the bounds of statutory authority =


wrongful.
Using Force:

The arrestor is the only justified in using deadly force, in other words,
force that is intended or is likely to cause death or grievous bodily
harm to a suspect, in particular circumstances: if he believes on
reasonable grounds that:
a) The force is immediately necessary for the purpose of protecting
the arrestor, any personal lawfully assisting the arrestor or any
other person from imminent or future death or grievous bodily
harm
b) There is substantial risk that the suspect will cause imminent
or future death or grievous bodily harm if the arrest is delayed
c) The offence for which the arrest is sought is in progress and is
of a forcible and serious nature and involves the use of life-
threatening violence or a strong likelihood that it cause grievous
bodily harm.

Requirements to use force:

1) Reasonable grounds
2) Must be necessary
3) protect person’s life or bodily integrity
4) conduct must be immediately threatening or will happen in the
future
POWER TO DISCIPLINE

Common law: parents and people in loco parentis (teachers) have, by


virtue of their authority over children, the power to administer
punishment to them for the purpose of education and correction.

BUT NOW:
Section 10 of the S.A Schools Act prohibits corporal punishment in
public as well as private schools.
It’s submitted that the wording of S10 makes it clear that a parent
cannot delegate his power to someone else.
(R v Janke and Janke)

Objectively viewed, punishment must be exercised moderately and


reasonably – the purpose of punishment must be to correct the child.
Malice or improper motive is an indication of unreasonable conduct,
which is wrongful.

Section 12 of the Constitution: confers the right on everyone not to be


treated or punished in a cruel, inhuman or degrading manner.

According to case law, the following factors must be considered in


determining whether the punishment was moderate and reasonable:
1. The nature and seriousness of the transgression
2. The degree of punishment or force inflicted
3. The physical and mental condition of the person punished
4. The gender and age of the child
5. The physical disposition of the child
6. The means of correction and
7. The purpose and motive of the person inflicting the punishment

There is a presumption that chastisement exercised by virtue of power


to chastise, was given reasonable and without malice.
A person who alleges otherwise must prove that no power to chastise
existed or that in the circumstances the chastisement was exercised
in an unreasonable manner.
ABUSE OF RIGHTS AND NEIGHBOUR LAW

If you exercise your rights over your property in a legally


impermissible manner = abuse of rights. Abuse is therefore the use of
land and the expense of another. This is considered conduct with the
exclusive aim of harming another person. However, if such a person
does not consider the “abuser’s” conduct unreasonable, he cannot
claim damages.

Principles:
1. General rule: the owner of immovable property can use his
property as he sees fit – as long as he acts within the bounds
placed by law on his ownership. (Gien v Gien)
2. Because the owner isn’t completely free to use his property as
he wishes – his interests must be weighed against the interests
of his neighbour.
3. The basic question = reasonableness – whether the defendant
used his property reasonably or unreasonably: conduct with the
aim of harming your neighbour is wrongful – improper motive
renders the act wrongful if it prejudices the neighbour without
benefiting the actor in any way. (Kirch v Glaser)
4. Where the benefit the actor gets from his conduct is slight and
damage to the neighbours property is serious = unreasonable
and wrongful, regardless of his motive. (Regal Hastings v African
Superslate)
5. Where the actor harms the neighbour while advancing his own
reasonable interests, he doesn’t act wrongfully, even if he had
an improper motive = improper motive isn’t enough – BUT if the
sole motive is to harm the neighbour = wrongful.

Nuisance (unreasonable use of land at the expense of another):

Repeated infringement of the plaintiffs property rights = smoke, gas,


odors, leaves, slate, music.
An objective weighing up of interests of the parties is involved.

Factors:
• Type of noise
• Degrees of persistence
• Time of noise
• Locality of noise
FAULT

Fault is a general requirement for delictual liability.

In practice 2 main forms of fault are recognized:


1. Intention (dolus)
2. Negligence (culpa)

Fault can be present ONLY if the person acted wrongfully.

With the actio legis aquiliae and the action for pain and suffering
either intention or negligence is enough for liability.
With the actio inuriarum – intention is required and negligence isn’t
sufficient.

1st it must be established whether the defendant had the capacity to


be held accountable.

ACCOUNTABILITY:

To determine if a person has fault, one must first see if he is


accountable for his actions. A person is accountable if he has the
necessary mental ability to distinguish between right and wrong
and that he can act in accordance with such appreciation – he
must have the required mental ability at the time of the act – if he
lacks accountability at the relevant time = there can be NO fault on
his part.

Accountability is seen as the basis for fault.


People who lack the necessary accountability include:
1. Youth: a) child under 7 (infans) always lacks
accountability (culpae incapax) – there is an
irrebuttable presumption that he isn’t
accountable.
b) There is a rebuttable presumption that a
child over 7 but under 14 (impubes) lacks
accountability = he’s assumed cupae incapax
until the contrary is proven.

2. Mental disease or illness: where a person cant at the given


moment distinguish between right and wrong or where they
can but they cant act in accordance with such appreciation,
they are culpae incapax = NO FAULT and no liability.
3. Intoxication: persons who are under the influence of
intoxicating liquor or drugs MAY be culpae incapax (lack
culpability). BUT the mere consumption of liquor or the use
of drugs may in a given situation be a NEGLIGENT act for
which the defendant MAY be held responsible.
4. Provocation: where a person under provocation loses his
temper and becomes passionately angry he MAY be said to
lack accountability and thus wont be blamed for his
(intentional) conduct

Once it’s been established that the defendant was accountable at the
relevant stage, it must then be determined whether he acted
INTENTIONALLY or NEGLIGENTLY.

INTENT

NB: a person acts intentionally if his will is directed at a result, which


he causes while conscious of the wrongfulness of his conduct.

2 elements of intention:
1. Direction of will
2. Knowledge (consciousness) of wrongfulness.

Direction of will:

Different forms of intention:


a) Direct intent (dolus directus): where the wrongdoer desires a
particular consequence of his conduct = X decides to shoot and
kill Y in order to take Y’s money – the execution of this plan is
accompanied by direct intent because it’s X’s plan that Y
should die.

b) Indirect intent (dolus indirectus): where the wrongdoer directly


intends one consequence but at the same time has knowledge
that another consequence will unavoidably follow. The causing
of the 2nd consequence is accompanied by indirect intent = X
desires to shoot and kill Y who is standing behind a closed
window – the bullet aimed at Y 1st shatters the window and
then kills Y. with regard to Y’s death X had DIRECT INTENT,
but he didn’t want to break the window (he knew it was a
consequence) so INDIRECT INTENT is present in respect of the
window.

c) Dolus Eventualis: the wrongdoer while not desiring a


particular result, foresees the possibility that he may cause the
result and reconciles himself to this fact – Question whether
the wrongdoer actually subjectively foresaw the possibility of
the consequence.
X wants to kill his enemy Y. Z is standing next to Y when X
takes aim. X foresaw the possibility that he could miss and hit
Z with fatal consequences. BUT X decides to proceed. The
bullet misses Y and hits Z = X shot Z intentional even though
he didn’t desire this consequence or foresee it as a necessary
consequence of his conduct – the mere fact that X subjectively
foresaw that he might hit Z is enough to conclude he acted
intentionally.

BUT if X initially foresaw the possibility that he might hit Z but


later for some reason came to the conclusion that it wouldn’t
happen = NO DOLUS EVENTUALIS = in such a case luxuria
(conscious negligence) may be present.

Further important types of intent:

Dolus determinatus (definite intent) – Where a wrongdoer’s will is


directed at a result which he causes while he has a specific person or
object in mind. This type of intention could take the form of dolus
directus, dolus indirectus, or even dolus eventualis.

Dolus indeterminatus (indefinite intent) – where a wrongdoer has no


specific person or object in mind while his will is directed at a specific
result. For example, a person places a bomb in a shopping centre. He
has no idea of exactly how many people or who specifically will be
hurt, but he still has intent regarding the specific consequence.

Consciousness (knowledge) of wrongfulness:


Knowledge of wrongfulness as a requirement of intent indicates that
it’s insufficient for the wrongdoer merely to direct his will at causing a
particular result: he must also know or at least foresee the possibility
that his conduct is wrongful.

Motive and Mistake:


Motive: Motive = reason for someone’s conduct and must not be
confused with intent.

Intent: denotes a willed act which the wrongdoer knows is wrongful

Motive refers to the reason why the person acts in a particular way –
his desires or will. So a person may, despite the fact that in his
opinion he has a good motive, still act with intent (where you kill
another to spare him from suffering).
BUT intent may be absent in a case where a person who has a bad
motive but who believes his conduct is lawful.

Mistake: the problem here is whether intent is present where the


wrongdoer causes a result in a manner different from that foreseen by
him.
Material deviation: intention is absent while intention is assumed to
be present where the deviation isn’t markedly different from the
foreseen causal chain of events
e.g. X wants to shoot and kill Y, he fires a shot in order to hit Y in the
head. The bullet misses Y and hits a rock and then hits Y in the heart
with fatal consequences = intention.
S v Goosen: X, Y and 3 other people participated in a robbery. X drove
the car. X did foresee that Y, who had a loaded gun, could shoot and
kill their victim. During the robbery Y jumped out of the way of a car
driven by the victim and this caused a shot to be discharged
involuntarily. The shot hit and killed the victim.
Court held: that the causing of death by intentional conduct differs
from the causing of death from involuntary conduct = material
difference between the actual causal chain of events and that foreseen
by X = X acquitted.

Masilela: 2 accused throttled the deceased; hit him over the head with
intention to kill him and then, thinking he was dead, set the house
alight.
Court Held: there was a mistake as to the causal chain of events, BUT
the accused still had INTENTION to murder. Therefore – guilty.
NEGLIGENCE

Definition: fault refers to a blameworthy attitude of someone who


acted WRONGFULLY – negligence = someone is blamed for conduct
which is careless, because by giving insufficient attention to his
actions, he failed to adhere to the standard of care legally required of
him.

This is an objective test on the standard of the reasonable person.

A defendant is negligent if the reasonable person in his position would


have acted differently = if the unlawful causing of damage was
REASONABLY FORESEEABLE and PREVENTABLE.

Kruger v Coetzee: Test for negligence:

The reasonable person in the position of the defendant:


1. Would foresee the reasonable possibility of his conduct injuring
another and causing him patrimonial loss.
2. Would take reasonable steps to guard against such occurrence
3. And the defendant failed to take such steps.

Reasonable person characteristic:

The reasonable person is a fictitious person – not exceptionally gifted,


careful but not someone who recklessly takes chances.

Reasonable person has certain minimum knowledge and mental


capacity, which enable him to appreciate the dangerous potential of
certain actions.
Reasonable person knows there are inherent dangers in the use of
guns, explosives, cars, poison etc.

BUT our law shouldn’t completely ignore physical handicaps in


determining the possible negligence of (for example) a blind person =
the question should be how a reasonable person with a particular
handicap would have acted.

THE NEGLIGENCE OF CHILDREN:

Before 1965 there was a tendency for the courts top take into account
the youthfulness of the wrongdoer in determining his negligence = test
for negligence by means of the conduct expected from the
REASONABLE CHILD of the wrongdoers age and intellectual
development.
In 1965 the AD in Jones v Santam- adopted a new approach to
determine negligence with regard to children.

The court implied that the criteria for determining negligence is


always objective in the sense that in all situations the test of the
REASONABLE PERSON is applied.
According to this approach, in determining if a child acted with
negligence:
1. See whether the child concerned met the standard of care
required of a reasonable person
2. Ask whether the child was accountable for his actions

This approach had a critical reception:


a. It would seem that the earlier test of the reasonable child was
more acceptable
b. It’s clear that the court in Jones placed the cart before the horse
by 1st looking into fault and then into accountability.

Logically you must 1st look at accountability and once someone is


found to be accountable only then can you look at fault.

Roxa v Mtshayi: AD accepted this criticism by 1st testing


accountability and then for negligence.

Weber v Santam: the AD had another chance to consider the Jones


case and said that it didn’t materially depart from the common law
and CONFIRMED the approach.

EXPERTS:

It’s considered whether the fact that the wrongdoer had expertise in
regard to the negligent conduct affects the application of the
reasonable person test.

The general test for negligence cannot be applied in considering the


conduct of the defendant where such conduct calls for expertise.
So in the case of an expert (doctor etc) the test for negligence is the test
of the SO- CALLED REASONABLE EXPERT (i.e. the reasonabledoctor).

The reasonable expert is identical to the reasonable person in all


respects, except that a reasonable measure of the relevant expertise is
added (Van Wyk v Lewis).

Durr v ABSA Bank Ltd:


The SCA approved the approach in Van Wyk and emphasized that the
decision of what is reasonable under the circumstances is for the
court to decide.

There may be instances where it would not be negligent for a


layperson to undertake an activity for which special skill is required.
For example, X, a layperson, takes control of an aircraft. X’s control is
not necessarily negligent, even though he has no knowledge or
expertise in piloting an aircraft. A lay person is only negligent if he
does not act as a reasonable lay person. He is therefore not expected
to act or be compared to a reasonable expert. (Mrupe)

CAN NEGLIGENCE AND INTENTION OVERLAP?

Van der Merwe: from the definition of negligence, it would seem


negligence only exists as a consequence of no intention being present.
This means that negligence and intention are mutually exclusive, in
other words, one can’t exist without the other.

S v Ngubane: intention and negligence are not mutually exclusive. The


question was whether someone convicted of murder, could be charged
with culpable homicide too. This is the accepted view.

Boberg: “where dolus is present, so too is culpa”.

When applying this concept to delict: the intentional causing of harm


to another person is contrary to the standard of care which the
reasonable person would have exercised and negligence is thus
simultaneously present.

DIFFERENCE BETWEEN ORDINARY AND GROSS NEGLIGENCE:

In terms of the use of the aquilian remedy, the two types don’t matter.
However, some statutory and contractual provisions limit liability to
“gross negligence” only.
In terms of inuiria, gross negligence can replace intention, if intention
cannot be proven.
Gross negligence is defined by MV Stellas Tingas Transnet Ltd: “To
qualify as gross negligence the conduct in question, although falling
short of dolus eventualis, must involve a departure from the standard
of the reasonable person to such an extent that it may properly be
catagorised as extreme; it must demonstrate, where there is found to
be conscious risk-taking, a complete obtuseness of mind or, where
there is no conscious risk-taking, a total failure to take care. If
something less were required, the distinction between ordinary and
gross negligence would lose its validity.”

NEGLIGENCE AND OMISSIONS:

Negligence as a form of fault is not the same as an omission. An


omission is a form of conduct and can be performed intentionally or
negligently.

Negligence: foreseeability and preventability of damage

Test for negligence rests on 2 pillars:


1. Reasonable foreseeability
2. Reasonable preventability of damage

REASONABLE FORSEEABILITY:
There are 2 views as to the foreseeability test:

1. Abstract (absolute) approach: The question whether someone


acted negligently must be answered by determining whether
harm to others was in GENERAL reasonably foreseeable. It’s not
a requirement for negligence that the extent of damage or a
particular consequence that actually occurred should have been
reasonably foreseeable – it’s enough if damage in GENERAL was
reasonably foreseeable.

2. Concrete (relative) approach: This is based on the premise


that a person’s conduct can only be negligent in respect of a
SPECIFIC CONSEQUENCE or consequences = it’s a prerequisite
for negligence that the occurrence of a particular consequence
must be reasonably foreseeable. The wrongdoer is only negligent
with reference to a specific consequence if that consequence and
not merely damage in general was reasonably foreseeable.

UNISA: says that the concrete approach is preferred for the


following reasons:
The question whether the reasonable person in the position of the
wrongdoer would have acted differently in order to prevent damage
can only be answered in a meaningful way by reference to the
consequences that were reasonably foreseeable and not damage in
general.
(Ablort Morgan v Whyte Bank)
PREVENTABILITY:

Whether the reasonable person would have taken precautionary steps


to prevent damage from occurring.
4 factors highlight the question whether the reasonable person would
take steps to guard against the occurrence of damage:

1. Nature and extent of the risk inherent in the wrongdoers


conduct: the fact that the nature and extent aren’t serious or
the harm-foreseen slight, may have the result that the
reasonable person despite the fact that the harm was
reasonably foreseeable, WOULDN’T have taken steps to prevent
it.

Wassernam v Union Government: a swarm of bees were in the


roof of a police station. While a policeman attempted to locate
the bees he was stung in the lip and died – the question was
whether the defendant was negligent in not driving out the bees.
Held: Reasonable person wouldn’t have taken steps to prevent
the harm from occurring = death wasn’t reasonably foreseeable.

2. Seriousness of the damage, if risk materializes and damage


follows: Where the wrongdoers conduct creates the possibility
of grave and extensive damage occurring, he should take
reasonable steps to prevent such damage, even though there’s
only a slight possibility/chance that the damage will actually
materialize.

Basson case: the defendant was employed by the plaintiff to


erect a roof on top of a silo – during welding, the sparks ignited
bales of stover stacked against the silo.
Held: although the risk of the stover catching alight wasn’t
great, the damage, which could result, was extensive.
The RP would have taken steps to prevent the damage from
occurring.

3. The relative importance and objective of the Wrongdoers


conduct: It may be that the interest served by the conduct is of
such a nature that it’s more important than the risk of harm,
which it involves = Reasonable person wouldn’t have taken
steps to prevent the harm. E.g. Ambulance driver who exceeds
the speed limit in an urban area while taking a seriously ill
patient to the hospital.

4. The cost and difficulty of taking precautionary measures:


Where the risk of harm can be eliminated or reduced without
substantial problems, prejudice or costs, it may be accepted
that the reasonable person would take precautionary measures.
BUT: where the cost and difficulty in taking precautionary
measures is greater than the risk the reasonable person
wouldn’t take steps to reduce the risk.

NB – Grocery store cases


Gordon v Da Mata:
The plaintiff slipped on a cabbage leaf, which was on the floor of
the defendant’s grocery store. – The leaf had fallen onto the floor
while the defendant’s assistant was slashing off the cabbage
leaves.
Held: RP would have taken steps to prevent leaves from falling
on the floor by collecting them = this measure would be simple and
inexpensive.

City of Salisbury v King:


The court had to decide whether it was negligent to leave
slippery vegetable matter that was lying on the market floor.
The plaintiff slipped on a piece of vegetable on the floor of a
large market while the sale of vegetables was in progress.
Held: the mere presence of vegetable matter on the floor of a
market didn’t in itself indicate negligent conduct.
It would be unreasonable, expensive and unrealistic to expect
immediate removal of the vegetable matter as it fell to the floor.
Negligence judged in light of the surrounding circumstances:

It must be self-evident that the negligence of conduct may only be


evaluated in light of all the relevant circumstances of a particular
case.

The following factors must be taken into account:

1. Greater care is required when someone works with something


that is inherently dangerous. (Gun, dynamite, etc).

2. Greater care is expected when a person deals with individuals


who suffer from some disability (children, blind, intoxicated)

3. When a person has to take a decision in a situation of SUDDEN


EMERGENCY and there is insufficient opportunity to consider
all the consequences of his actions – this factor MUST be taken
into account in deciding whether he is negligent = this is called
the DOCTRINE OF SUDDEN EMERGENCY.

THE LAW CANT EXPECT A PERSON WHO HAS TO ACT


SWIFTLY IN A SITUATION OF IMMINENT PERIL TO SHOW THE
SAME JUDGEMENT AND SKILL AS SOMEONE WHO ISNT
ACTING IN SUCH URGENT CIRCUMSTANCES.

Therefore an error of judgment doesn’t necessarily amount to


negligence, because the reasonable person may also make an
error of judgment in certain circumstances.

Three requirements must be satisfied in a case of sudden


emergency for a wrongdoers conduct not to amount to
negligence = to meet the standard required of a reasonable
person:
a) The wrongdoer must have faced a situation of imminent
peril – e.g. a man armed with a panga, slashes at a motorist
who is driving past him – someone who is fighting with
another, picks up a brick and aims it at a motorist, etc =
NOT NEGLIGENT
b) The wrongdoer must not have caused the situation through
his negligence – where a reasonable person would in any
event have prevented the emergency situation – the
wrongdoers conduct will not be excused.
c) The wrongdoer must not have acted in a grossly
unreasonable manner – established whether a reasonable
person in the same circumstances would have made the
same error in judgment as the wrongdoer.
4. A person acts according to the standard of the reasonable
person when he relies on the fact, that another person will also
act in a reasonable way – so a person isn’t negligent when he
fails to take into consideration the negligent conduct of another
driver and is involved in a collision with him. BUT – where the
conduct of another driver is reasonably foreseeable a person
cannot always rely on other road users acting reasonably.

5. Customs and opinions of the community

6. In certain circumstances the appropriate standard of care


required for conduct isn’t entirely left to the discretion of the
court, because there is also a specific statutory provision that
applies.

Brown v Hunt: A petrol attendant (in the employ of the appellant),


spilled some petrol on the floor while filling up a car. The petrol
caught alight and the attendant tried to put the fire out with water,
damaging the respondent’s car. The employer used the doctrine of
sudden emergency because the fire created “imminent peril”. However,
the court said the doctrine of sudden emergency would not work
because the imminent peril would not have existed, had the employee
not spilled the petrol.

Relevance of Negligence:

Example: A is driving his car at an excessive speed and without lights.


He turns the corner and hits B, who was standing in the middle of the
road, and kills him. A did not act as a reasonable person and
therefore should be guilty of negligence (intention if he foresaw the
possibility). However, Van Rensburg states that, even if A did act as a
reasonable person, A would still have hit B. Therefore, A’s negligence
is not relevant to B’s death. A should then not be delictually liable.
Negligence and the duty of care:

Negligence = determined according to the test of the reasonable


person.
BUT on occasion our courts haven’t applied this test and instead
followed English law in applying the DUTY OF CARE DOCTRINE.

1. You see if a duty of care is owed = reasonable person ion the


position of the defendant would have foreseen that his conduct
might damage the plaintiff.

2. Whether there was a breach of the duty of care – courts


consider whether the wrongdoer exercised the standard of care
that the reasonable person would have exercised in order to
prevent the damage = LEGAL DUTY

Proof of negligence:

To succeed in his claim, the onus is on the plaintiff to prove on a


balance of probabilities that the defendant was negligent.
BUT: where there is a statutory presumption of negligence, the onus
rests on the defendant to rebut the presumption of negligence to
escape liability.

Presumption = res ipsa loquitor - creates a presumption of negligence.


Wrongfulness and negligence:

Wrongfulness Negligence
1. The reasonableness of the 1. Reasonable persons conduct is
defendants conduct is determined determined with reference to
by the weighing up of the reasonable foreseeability and
conflicting interests- in the light preventability of the damage
of the legal convictions of the
community – BONI MORES
2. Concerned with the 2. Determination of legal
determination of legally blameworthiness of the defendant
reprehensible conduct for his wrongful conduct – is
determined in light of all the
relevant facts and circumstances
that are actually present
3. Because it concerns the legal 3. Negligence is determined with
reprehensibility of a persons reference to the position in which
conduct, it must be determined in the defendant actually found
light of all the relevancy facts and himself = placing the reasonable
circumstances that are person in the defendants position
ACTUALLY PRESENT.
4. An omission is unreasonable 4. Omission – where the
and thus wrongful – where defendant attempted to comply
according to Boni Mores, a legal with such a duty and his attempt
duty rested on the defendant to coincided with what a reasonable
act and prevent harm and he person would have done, his
neglected to comply with such a wrongful act isn’t accompanied by
duty. negligence and he will escape
liability. Minister of Forestry
CONTRBUTORY FAULT

Contributory fault = conduct of the plaintiff – its relevant in limiting


the extent of the defendants liability and is thus important in practice.

It’s regulated by the Apportionment of Damages Act.

Common law position:

General rule: fault on the art of the plaintiff precluded him from
claiming damages from the defendant, who was also to blame for
causing the damage.

So if two people were at fault, neither could claim damages unless one
was more to blame than the other.

The DOCTRINE OF CONTRIBUTORY NEGLIGENCE was taken over


from English law.
It was developed in Davies v Mann: The plaintiff haltered a donkey in
the road – the defendant driving his wagon, collided with the donkey =
there was negligence on both sides.

In terms of the ALL OR NOTHING RULE, which was in force at the


time, the plaintiff would have been unable to claim damages from the
defendant.

BUT: the court adopted a new approach, since the defendant has the
LAST OPPORTUNITY to avoid the collision; the plaintiff’s negligence
was ignored.

Our court initially accepted that if the negligence of two people


contributed to the causing of a particular result and one or both
suffered damages as a consequence, NEITHER party could institute
action unless the negligence of the one was the DECISIVE CAUSE of
the accident = the negligence of the other party was completely
ignored and he could succeed in full with his claim.

To determine whose negligence was the DESCISIVE CAUSE, look at


who had the last opportunity to avoid the accident.

Also contributory negligence by a deceased was no defence to an


action instituted by his dependants; unless the negligence of the
deceased was the decisive cause of his death (i.e. he had the last
opportunity to avoid it).
If it wasn’t possible to determine whose negligence was the decisive
cause – the dependants could succeed in their claim.
The common law position has now been changed by the
Apportionment of Damages Act.

Apportionment of Damages Act (1956)

Section 1: where any person suffered damage which is caused partly


by his own fault and partly by the fault of another, a claim in respect
of that damage shall not be defeated by reason of the fault of the
claimant BUT the damages covered shall be reduced.

The effect is to abolish the all or nothing rule of common law and to
allow the court to apportion the damage of each party in accordance
with their relative degrees of fault.

The meaning of fault:

In general fault includes both intention and negligence.

The Question: whether a defendant who has intentionally caused


damage to the plaintiff, may raise the defence of contributory
negligence?

At COMMON LAW: the position was that such a defence couldn’t be


sustained where a defendant acted intentionally and it must be
accepted that the statutory position doesn’t change this.

A defendant who intentionally caused harm to the plaintiff will


not be able to ask for a reduction in damages because of
contributory fault.

Distinction between 2 situations:

1. Where the plaintiff intentionally contributed towards his own


loss, while the defendant was merely negligent = plaintiff
forfeited his claim.

2. Where the defendant caused the loss intentionally and the


plaintiffs unreasonable conduct that caused the loss was also
intentional = it appears that the legislature intended to make
provision only for the defence of contributory negligence and
not for contributory intent.

BUT: in ABSA case: the court held that S1 applies when the
form of fault on both the plaintiff and the defendant is
intention.
Court said: that fault includes dolus (intention)
Meaning of apportionment of damages:

Reduction of damages received by the plaintiff, because of his own


fault in respect of the damages he sustained.

Criteria for apportionment of damages:

Reasonable person test for negligence – Section 1 applies only to


damage caused partly by the fault of the plaintiff and partly by that of
the defendant.

South British Insurance Co v Smit – (past position)

Jones v Santam:
In so far as the objective reasonable person test applies, one is dealing
with a deviation from the standard of care, which applies, to all people
in a community.
The court doesn’t take into account that the defendant’s conduct has
contributed to the harm to a greater extent than the plaintiffs
conduct.
The court is satisfied that the negligent acts of both parties are
causally connected to the damage; the question of causation is solved.

The method of determining who should bear which portion of the damage,
involves a comparison of the respective degrees of negligence.

Each party’s degree of negligence is determined by expressing its


deviation from the standard of the reasonable person as a percentage.

Before Jones (in Smit): The AD accepted that once the plaintiff’s
degree of negligence had been established it was unnecessary to
inquire into the extent to which the defendant’s conduct deviated from
the standard of the reasonable person.

E.g. plaintiff was 40% negligent – so automatically the defendant was


60% negligent.

But in Jones: They said that if the plaintiff was 30% negligent it
doesn’t automatically mean that the defendant was 70% liable
To establish the degrees of negligence, the carefulness of the conduct
of each party must be measured separately against the standard of
the reasonable person.

Where the defendant raises the defence of contributory negligence on


the part of the plaintiff, he has the onus to prove the defence on a
balance of probabilities.
Nomeka: The AD confirmed the approach in Smit – that the degree of
the plaintiffs fault automatically determines the fault of the defendant.
It submitted that both Jones and Nomeka apply, but its stated that
the approach in Jones should be confirmed

Fault in regard to the damage or the damage-causing event:


(seat belt cases)

NB question: whether S1 is also applicable where the plaintiff wasn’t


negligent with regard to the damage-causing event BUT where his
negligence increased the damages.

I.e. X failed to wear his seat belt and as a result suffered more
damages than he would have had he been wearing it.

In King v. Pearl Insurance Co Ltd:


Ruled on the fact that the plaintiffs failure to wear a helmet while
driving a scooter which was involved in a collision with a car - didn’t
constitute contributory negligence.
The judge based his decision on the argument that only negligence
with regard to the damage-causing event, as opposed with negligence
in regard to the damage, is taken into consideration for the purposes
of the Act.

This approach was criticized in:

Bowkers Park v SAR & H:


The judge considered the opinion above as unnecessary n the plain
wording of S1 – in his opinion the section leaves no doubt that
contributory negligence relates to fault with regard to the damage
causing event = its always possible that a plaintiffs contributory
negligence with regard to his damage, can lead to a reduction in
damages, even if he cant be held responsible for the actual damage
causing event.
In Victoria case:
Said that failure to wear a seat belt does constitute to contributory
negligence.
The court was of the opinion that the contributory negligence of a
plaintiff who deliberately fails to wear his seat belt will be greater than
a plaintiff who bona fide forgot.
In such a case the contributory negligence of the plaintiff is only
relevant in so far as it leads to an increase in damages.

So S1 applies in respect of that damage for which the plaintiff


is also responsible.

Example:
X suffers loss of R10 000 because of a car accident, which was caused
entirely by Y’s negligence.
It’s proved that X’s damages would only have amounted to R6000, had
there not been contributory negligence on his part, due to his failure
to wear a seatbelt.
So Y is alone responsible for the R6000. (Vorster v AA Mutual
Insurance Association Ltd)

VOLUNTARY ASSUMPTION OF RISK

In regard to contributory fault, voluntary assumption of risk is a


ground that cancels fault BUT isn’t a ground of justification.
Assumption of risk implies that the requirements of the grounds of
justification are absent.

When the plaintiff or injured party are well aware of the risk of danger
but willfully exposes himself to it, he acts intentionally and blame in
the form of contributory intention attaches to him.

His conduct must also be consciously unreasonable = not directed to


achieving a lawful goal. This is because a person does not act
wrongfully in regard to himself, because he strictly speaking does not
have a conscience of wrongfulness in regard to the harm that causes
“consciously unreasonable conduct”.
Where the plaintiff acted with contributory intent, the fault of the
defendant is eliminated.
Although the defendant is also at fault, he isn’t liable towards the
plaintiff because of the fact that the plaintiff himself acted
intentionally.
Lambert v Hefer:
The plaintiff sat in the side car of a bike, knowing the defendant was
sp drunk, that he wasn’t capable of controlling the bike = they had an
accident and the plaintiff got injured while the defendant was killed.
The plaintiff claimed damages from the defendant’s estate.

Contributory intent because the plaintiff was aware of the danger and
possibility of injury BUT decided to expose himself to the risk = the
remedy wasn’t available to her.

Netherlands Insurance co v Van der Vyver:


O was suspected of infidelity by his wife. So she hired a private
investigator (V) to spy on her husband.
V followed O in his car to a spot in the veld; O had a woman in the car
with him. When V approached O’s car O started to drive off.
V jumped on the bonnet to block O’s view and make him stop, but O
accelerated and swerved from side to side finally dislodging V.
V sustained injuries and claimed compensation from the insurer of
O’s car. Court found: O was 50 % negligent and V 50% negligent
So V only got half of his damages.

On APPEAL held that O acted with intention and the court rejected
the defence that V had consented to the risk of injury = failed.
CAUSATION

The causing of damage through conduct = a causal nexus between the


conduct and the damage is required for delict.
The question whether there is a causal nexus is a question of fact and
must be answered in light of the available evidence.

Conditio sine qua non = to determine whether a factual causal nexus


exists between the act and the harmful consequence – if so FACTUAL
CAUSATION.

FACTUAL CAUSATION:

There is no question of delictual liability if it isn’t proved that the


conduct of the defendant caused the damage of the person suffering
the harm.

What is the correct test for determining causation?

Conditio sine qua non theory:

Conditio sine qua non literally means “Condition without which


something does not happen”. The conditio sine qua non was
formulated in (International Shipping Co (Pty) Ltd v Bently).

Also known as the BUT FOR test = according to this an act is the
cause of a result if the act cannot be thought away without the result
disappearing as well.

E.g. in order to determine if X is the cause of Y, eliminate X mentally


and consider whether Y still exists or not.
If Y falls away when X is eliminated, then X is the cause of Y.

Cumulative Causation:

In this case, the conditio sine qua non fails.

For example, if X an Y, both independently and without prior


knowledge, shoot Z and kill him. If we eliminate X’s conduct, Z would
still be dead.

To answer this, medical evidence is needed to prove which bullet (or


both) killed Z. If both bullets killed Z simultaneously, X and Y are both
guilty of murder.
Criticism of the conditio sine qua non theory:

1. The conditio is based on a clumsy, indirect process of thought


that results in circular logic.

2. The conditio fails in cases of so-called cumulative causation -


cumulative causation occurs when more than one act actually
causes a consequence – X and Y simultaneously BUT
independently shoot at Z’s head. If you eliminate X’s shot, Z’s
death doesn’t fall away and visa versa = so the test suggests
that neither X nor Y caused Z’s death.

3. The conditio test isn’t a test of causation because it’s merely an


ex post facto way of expressing a predetermined nexus – X visits
Y to accuse him of adultery with his wife. Y is exceptionally
friendly and offers X a glass of beer. A few minutes after X has
drunk the beer, he suffers convulsions and drops dead – using
the conditio test if you eliminate the beer – you still don’t know
whether he died from poison or a heart attack = its only after
the facts have been established would you know whether the
giving of the beer was the conditio or not.

Conditio sine qua non and causation by omission:

General view: conditio test may be applied when one wishes to


determine whether an omission caused a certain consequence.

S v Van As: police neglected to search for children who had fled into
the night and later died of exposure.
Question: whether their death was caused by the omission to search
for them.
The court attempted to test the causal connection between the
omission and the death by asking whether reasonable search would
have prevented the death.

According to Van Rensburg – the conditio sine qua non test doesn’t
offer a solution in cases of causation by an omission.
The conditio requires you to eliminate something in the mind and not
to add something to given facts.

The argument that the conditio sine qua non test is the true test for
causation in cases of omissions isn’t accepted.
Determination of a factual (causal) nexus:

In view of the fact that conditio sine qua non should be rejected as the
test for causation, what test should then be used.

Van Rensburg – a causal nexus exists where one fact arises out of
another – this must be established according to human experience in
general.

If X stabs Y with a knife and Y is then taken to hospital where as a


result of the negligent conduct of the nurse, he falls off the bed,
fractures his skull and dies – law identifies the initial stabbing and the
negligent conduct of the nurse as the cause of death = causes of
certain consequences can be unlimited.

It’s enough for the purposes of factual causation if defendant’s


conduct has in any way contributed to the damage sustained by the
plaintiff.

Smit v Abrahams: In this case, the plaintiff’s car was damaged by the
defendant. The plaintiff claimed damages and rental for another car.
Botha JA dealt with the question of factual causation and held that
the plaintiff could not purchase another vehicle due to his financial
position. The court came to this conclusion (most importantly),
without referring to the condition sine qua non.
LEGAL CAUSATION:

No legal system holds a wrongdoer liable without some limit for the
endless chain of harmful consequences, which his act may have
caused – means to limit the wrongdoer’s liability.

The question of legal causation arises when determining which


harmful consequence actually caused by the wrongdoer’s wrongful,
curable act should he be held liable for.

In most cases of delict the harm for which the wrongdoer is to be held
liable falls within the limits of his liability, so that it’s unnecessary to
examine legal causation.

Normally legal causation is only a problem where a whole chain of


remote consequences result from the wrongdoers conduct and where
it’s alleged that he shouldn’t be held liable for all the consequences.

The limits of liability should be determined in respect of every


delictual claim and the fact that this determination in most cases
need not be made expressly shouldn’t lead one to believe that legal
causation is relevant only in exceptional cases.

Legal causation as an independent element arises where it appears


that the wrongdoers conduct was wrongful and culpable with
reference to at least certain requirements.

Question: What criteria should be used to determine legal


causation?

THE FLEXIBLE APPROACH:

S v Mokgethi:
A bank teller was shot between the shoulder blades and became a
paraplegic in a wheelchair – he went back to work later but failed to
shift positions in the chair.
As a result he suffered pressure sores and was re-admitted to
hospital.
6 months after the shooting he died as a result of the sores.
The AD held that the wounding of the deceased couldn’t be regarded
as the legal cause of death.
The court held that there is no single criteria for legal causation.
The basic question for the flexible approach was whether there was a
close enough relationship between the wrongdoers conduct and its
consequence, for such consequence to be imputed to the wrongdoer in
view of policy considerations based on reasonableness, justice and
fairness.

This approach was confirmed in Bentley, Smit v Abrahams, and Collett.


AD has expressed itself in favor of the flexible approach.

ADEQUATE CAUSATION:

According to this theory a consequence which has been caused by the


wrongdoer is imputed to him if the consequence is adequately
connected to the conduct.
It’s adequate if according to human experience in the normal course of
events the act has a tendency to bring about that type of consequence.

Ask: was the damage reasonably to be an expected consequence of the


act – did the damage fall within the field of protection.
(S v Daniels)

DIRECT CONSEQUENCES:

According to this theory an actor is liable for all the direct


consequences of his negligent conduct – liability isn’t limited to
foreseeable consequences.
Because this could lead to an exceptional wide liability it’s been
limited to direct physical consequences.

Also it was required that the immediate nature of the nexus between
the cause and the consequence must not be broken by a novus actus
interveniens.

The wide effect of the direct consequences test has been limited by the
foreseeable plaintiff doctrine = an actor doesn’t act negligently to a
plaintiff unless it’s reasonably foreseeable that the particular plaintiff
will be injured.
The actor isn’t liable for an unforeseeable plaintiff, even thought the
harm has flowed directly from the actors conduct and despite the fact
that it’s foreseeable that other people may have been injured.

In 1961 the direct consequences theory was rejected in Overseas


Tankership Ltd in favor of the principle that only foreseeable damage
was recoverable.

Van der Walt argues that the direct consequences theory finds
application with regard to the causing of personal injury where the
wrongdoer is held liable for those consequences, which weren’t
reasonably foreseeable (egg-skull cases).
FAULT (FOR LEGAL CAUSATION)

The wrongdoer is liable only for those consequences in respect of


which he had fault.
Liability must be limited to consequences willed by a person while
aware of their wrongfulness and the wrongful consequences that he
should have foreseen and prevented.

Supporters of the fault-in-relation-to-the-loss approach declare that


legal causation as an independent element of delict is unnecessary –
the question of fault and imputability of loss are disposed off
simultaneously – apply where the concrete approach is followed.

Intention and negligence cannot serve as a satisfactory criterion for


legal causation.

Intention as a criterion for legal causation:

Van der Merwe – wrongdoer is liable for those consequences covered


by his intent and the intended consequence can never be too remote.

Thus: consequences caused intentionally can never fall outside the


limits of liability.

E.g. X knows that his aunt (Y) has nominated him, as her sole heir in
her will. He becomes impatient waiting for her to die a natural death
so that he can get her assets. So he persuades her to take a journey in
her car from Durban to Johannesburg, hoping she might die in an
accident.
Y takes the journey loses control of her car and dies.
There is a factual causal nexus between X’s conduct and Y’s death.

In terms of this theory:

1. Intention in the form of dolus eventualis is present


2. Factual causation is present – But for
3. Applying the fault theory – X foresaw the possibility that his
conduct would lead to her death and reconciled himself to it,
while knowing it was wrongful
4. Therefore – because he has intention, he is liable and there is
no need to look into legal causation
5. Majority of the jurists argue that it’s unreasonable in the
circumstances to hold X liable for Y’s death.
Negligence as the criteria for legal causation

Negligence cannot serve as the test for determining the imputability of


damage. The test for negligence is whether the reasonable man, in the
same position as the wrongdoer would have foreseen and prevented
either injury to another in general (abstract) or the consequences
concerned (concrete).

Boberg: the question of legal causation is answered differently,


depending on whether you use the abstract or concrete approach.

Concrete: X is only negligent if the specific kind of harm suffered was


reasonably foreseeable.
So therefore it’s unnecessary to determine legal causation, since
negligence has already been determined with regard to a specific
consequence.
The concrete test for wrongfulness and negligence, contain all the
elements necessary to keep liability within acceptable limits.

Abstract: X is negligent if any general damage at all was reasonably


foreseeable. Therefore to see if X is liable for the specific consequence
we look to other theories for legal causation.

REASONABLE FORSEEABILITY:

Reasonable foreseeability must not be seen as the single decisive


criteria for establishing liability.

Case law doesn’t offer a clear picture of the content of the


reasonableness criteria.
Liable for damage that was reasonably foreseeable.

Van der Walt: it’s not necessary that all consequences of the
defendants conduct should be foreseen – only the general nature of
harm must have been reasonable foreseeable.
The precise extent of the occurrence need not have been reasonably
foreseeable.

Van Rensburg agrees.

This is the most favorable approach except for the flexible approach.
NOVUS ACTUS INTERVENIENS

= New intervening act


It’s an independent event, which after the wrongdoer’s act has been
concluded, either caused or contributed to the consequences
concerned.

The question now arises as to what extent such an event influences


the possible liability of the wrongdoer.
Where a novus actus interveniens extinguishes the causal connection
between the conduct of the wrongdoer and the consequence, with the
result that the wrongdoers act can no longer be considered to be a
factual cause of the consequence = actor goes free.

It’s more difficult to determine when the new act influenced the result
to such an extent that the result should no longer be imputed to the
actor, although his conduct remains the factual cause of the result.

The novus is to limit the liability of the wrongdoer and plays an


important part in legal causation.
In each case it will have to be determined within the framework of the
relevant imputability test, whether the novus has had the effect of
severing the legal nexus with the result that the consequence
shouldn’t be imputed to the actor.

A novus may be brought about by the (culpable) conduct of the


plaintiff himself, by the culpable conduct of a 3rd party, or by natural
forces like the wind and rain.
An event will qualify as a novus, only if the event WASN’T
REASONABLY FORESEEABLE.

If the intervening cause was foreseeable at the moment of the act,


such an event may not be considered to be a novus which may
influence the imputability of harm to the actor.

Gibson: a distinction should be drawn between the party negligent


prior to the harmful event and any negligence after the event.

Mokgeti: the question was whether the victims own conduct had
broken the causal nexus between the accused's wrongful act and the
result in question.

The court doubted whether it was possible to formulate a general


principle through which it could in all cases be established whether
the victim’s omission would interrupt the required legal causation.
The court suggested a criterion, which may be useful to determine
legal causation in view of the victim’s own conduct:
1. The victim fails to seek medical attention and that’s the cause of
death
2. The wound wasn’t life threatening
3. Where failure to seek medical attention is unreasonable

Alston v Marine Trade Insurance Co Ltd:


The plaintiff suffered a brain injury from a car accident, which was
caused by the negligent driver of a car that was insured by the
defendant. As a result of the brain injury the plaintiff suffered from
manic depression for which he was treated – according to medical
knowledge at the time, there was no reason to think that the
medication he was taking presented any danger.
The plaintiff ate some cheese after taking the medication and suffered
a stroke. It appeared after that the eating of cheese after taking this
medication (parstellin) might lead to death.

Mafesa:
The plaintiff broke his leg in a car accident because of the negligence
of another driver who was insured by the defendant.
He was discharged from hospital and slipped on his crutches and
broke his leg again. His medical expenses increased.

Russel:
Deceased sustain brain injuries causing depression in a car accident.
The deceased then committed suicide. The defendant claimed that the
suicide (which was an informed and voluntary act) was a novus actus
intervenes, breaking the causal chain between the accident and the
deceased’s death. Chetty AJP held that even though the decead’s
death was due to a mental disability and therefore was probably not
intentional, it cannot be a novus actus intervenes. This follows the
English law that a person who is not of sound mental ability, cannot
perform an act.
EGG-SKULL CASES = talem qualem rule

Egg-skull cases arise when the plaintiff, because of one or other


psychological or financial weakness, suffers more serious injury or
loss as a result of the wrongdoers conduct than would have been the
case if the plaintiff didn’t suffer from such a weakness.
Most jurists agree = the wrongdoer should also be liable for the harm,
which may be ascribed to the weaknesses concerned = you take your
victim as you find him.

Van Rensburg – is of the opinion that as a result of the particular


circumstances present, the precise manner in which the consequence
occurs need not be foreseeable with the same degree of probability
which applies in normal cases.

Van der Merwe – finds that the thought “you must take your victim as
you find him” is rejected in so far as the reasonable person wouldn’t
have foreseen the consequences concerned and the injured party
should bear the loss himself.

The most acceptable approach is the flexible criterion for legal


causation as illustrated in Abrahams.
The basic question isn’t whether the damage was a direct
consequence or reasonably foreseeable BUT whether in light of all the
circumstances the damage should be imputed to the wrongdoer.

E.g., Wilson v Birt:


Plaintiff was injured when a poll fell on his head – earlier the plaintiff
was stabbed in the head and had to have a piece of his skull removed.
Therefore the poll caused more damage than it otherwise would.
Court decided: defendant was liable for the full extent of the injury,
despite the fact that the injury may have been attributed by a weak
spot on the plaintiff’s head.
DAMAGE

Damage is the wrongful and culpable act, which has a harmful


consequence – the element of damage is fundamental to a delictual
action for damages.

The compensatory function of delict:

1. Compensation for damage: damage is the monetary equivalent


of damage awarded to a person with the object of eliminating as
fully as possible his past as well as future patrimonial damage
and where applicable non-patrimonial damage – money is
intended to be the equivalent of damage.

2. Satisfaction: if damage is incapable of being compensated


because money cannot be the true equivalent of the impaired
interest, satisfaction becomes relevant. Satisfaction implies
reparation of damage in the form of injury to personality by
effecting retribution for the wrong suffered by the plaintiff and
by satisfying the plaintiff’s and the community’s sense of
justice. It operates by the defendant being ordered to pay a sum
of money to the plaintiff.

Definition of damage:

Damage is the diminution, as a result of a damage-causing event, in


the utility or quality of a patrimonial or personality interest in
satisfying the legally recognized needs of the person involved.

Damage includes patrimonial as well as non-patrimonial loss.

Patrimonial loss:

Definition: is the diminution in the utility of a patrimonial interest in


satisfying the legally recognized needs of the person entitled to such
interest. It can also be seen as the loss or reduction in the value of a
positive asset in someone’s patrimony.

A person’s patrimony (estate):

For example, where an attorney’s negligence regarding execution of a


will has a result that Y is no longer the sole heir as intended by the
testator.
Union Government v Warneke:
Patrimony was defined as a universitas of rights and duties
a. Positive elements: real rights, immaterial property rights and
personal rights. The monetary value of such rights is
determined by the market value of the object in question as well
as any limitation on such right.
b. Negative elements: debt, which someone incurs. Someone’s
patrimony is reduced by the creation of a debt.

Ways in which patrimonial loss is caused:

1. Loss of patrimonial element: property is destroyed, a patrimonial


right in respect thereof is lost and a person’s patrimony
diminishes in value.
2. Reduction in the value of a patrimonial element: where the
object of a patrimonial right is damaged, the utility of such right
is also reduced.
3. The creation or increase of a debt, the delay in getting a benefit
and the creation or acceleration of an expectation of a debt.

Forms of patrimonial loss:


a. Damnum emergens and lucrum cessans:
Damnum emergens – all other forms of damage used to refer to
damage suffered up to the date of the trial.
Lucrum cessans – loss of profit, but also prospective loss.
b. Damage to property and pure economic loss.
c. Direct and consequential loss – direct loss is the immediate
result of a damage-causing event and consequential loss is the
loss which flows from the direct loss
d. General and special damage – general damage is presumed to
flow from an unlawful act and special damage is loss i.r.o.,
which the presumption doesn’t apply and has to be specially
pleaded and proved.

Assessment of patrimonial damage:

Sum formula approach:


This is a comparative test: in terms of this approach damage exists in
the negative difference between the plaintiffs actual patrimonial
position as it exists after the damage-causing event and the
hypothetical patrimonial position that would have existed had the
damage-causing event not occurred = current position compared to a
hypothetical position.

The reason why the hypothetical patrimonial position of the plaintiff is


used is to prove the prospective damage.
A concrete concept of damage:

Case law doesn’t always accept the sum formula approach. The AD in
one occasion held that: the difference between the patrimonial
position of the plaintiff before the unlawful act and the actual position
after.

This is the same as Van der Walt – what must be compared is what is
and the current position shouldn’t be compared with a hypothetical
position which would have existed had the delict not been committed.

Time for assessment of damage:

The date of the commission of the delict is the earliest date on which
all the elements of delict are present.

However if all the elements of delict are present on the date on which
the 1st damage occurred – use that date.

Prospective patrimonial damage (lucrum cessans):

The most important reason why assessment of damage in regard to


prospective loss is relevant is because of the once and for all rule. In
terms of this rule a plaintiff, who claims damages on a specific cause
of action, has only one chance to claim damages for all the damage
suffered as well as all prospective loss.

Definition: prospective damage is damage in the form of patrimonial


loss which will, with a degree of possibility, materialize after the date
of assessment of damages resulting from an earlier damage causing
event.

Nature of future loss:

Prospective damage literally manifests itself in money or otherwise


only in the future. It’s basis is to be found in the impairment of the
plaintiff’s present interests.

Forms of prospective loss:

1. Future expenses on account of the damage-causing event


2. Loss of future income
3. Loss of business or professional profit
4. Loss of prospective support
5. Loss of chance to gain benefit

A claim cannot be instituted merely to recover damages for


prospective loss since the cause of action only exists if some damage
has already been caused.
The once and for all rule:

In claims for compensation and satisfaction arising out of a delict the


plaintiff must claim damages for all the damage already sustained or
expected in the future in so far as its based on a single cause of action

The rule has practical implications:


1. Prescription in regard to the claim for damages commences as
soon as the cause of action arises and the debt in respect of
payment of damages is claimable = all the elements of delict are
present and the plaintiff is aware of or should reasonably be
aware of the identity of the debtor and the facts of the cause of
action.
2. A plaintiff who has sued with or without success for part of his
damage cannot thereafter sue for another part, if both claims
are based on a single cause of action.

Exceptions to the rule:


a. Where an act causes a nuisance, a claim may be instituted
each time the nuisance causes damage
b. In subsistence cases, where a person causes damage through
unlawful excavation, there’s a cause of action every time that
damage occurs and the plaintiff isn’t expected to claim once
and for all.
c. In the case of a continuing wrong which causes damage.

The collateral source rule and compensating advantages (res inter


alios acta):

A damage-causing event often not only causes loss but also results in
the plaintiff receiving some benefit.

Suppose X is negligently injured by Y and has to incur R1000 medical


expenses. However X’s aunt Z feels sorry for him and donates R1000
to him.
What role if any does the receipt of R1000 from Z play in the
assessment of X’s damages and the damages that Y has to pay?
So positive law made the following:

The following benefits which the plaintiff gets on account of his loss
aren’t taken into account in reducing his damages:

1. Benefits in terms of indemnity insurance and non-indemnity


insurance (life insurance)
2. Benefits from a medical fund or sick leave
3. Benefits in terms of the contractual right of an owner-seller of
property
4. Insurance money and pension payable to dependents whose
breadwinner has been killed
5. Donations
6. Discretionary payment of pension benefit to a person who has
been injured
7. Earning capacity of a widow who claims loss of support caused
by the death of her husband.

Benefits which the plaintiff has received or may receive must be taken
into account in reducing his damages:

1. Benefit of an injured person of receiving medical treatment free


of charge in a provincial hospital
2. Marriage prospects of a widow who claims loss of support
3. Savings on income tax due to loss of income
4. The amount which the plaintiff has received from the liability
insurer of the defendant
5. The plaintiffs possible saving on living expenses on account of
his injuries
6. Accelerated benefits from the estate of the deceased
breadwinner.

MITIGATION OF LOSS

The plaintiff cannot recover damages for a loss which is the factual
result of the defendants conduct, but which could have been
prevented if the plaintiff had taken reasonable steps.

A failure to ensure that damage doesn’t accumulate can be seen as a


failure by the plaintiff to take reasonable steps to limit the initial loss
or an omission to prevent further damage.

a. Plaintiff is obliged to take all reasonable steps to limit the


damage caused by the defendant’s delict. A plaintiff who fails to
mitigate his loss cannot recover damages in respect of the loss
he could reasonably have prevented.
b. A plaintiff, who has taken reasonable steps to mitigate his loss,
may also recover damages for loss caused by such reasonable
steps.
c. Where the plaintiff has reduced damages by taking reasonable
steps, the defendant is only liable to compensate for the actual
loss.
d. The onus of proving that the plaintiff didn’t properly fulfill his
duty rests on the defendant.

NON-PATRIMONIAL DAMAGE:

Definition: non-patrimonial damage is the diminution as a result of


a damage-causing event, in the quality of the highly personal interests
of a person in satisfying his legally recognized needs but which doesn’t
effect his patrimony = physical-mental integrity, liberty, reputation,
dignity, privacy, feelings.
DELICTUAL REMEDIES

South African law rests on 3 principles:


1. actio legis Aquiliae
2. actio inuriarum
3. Action for pain and suffering

Actions:

Patrimonial damage (damnum inuria datum): damages for the


wrongful and culpable (negligence/intention) causing of patrimonial
damages are claimed = Actio legis aquiliae.

Injury to personality: satisfaction for the wrongful and intentional


injury to personality = actio inuriarum.

Action for pain and suffering: compensation for injury to personality


as a result of the wrongful and negligent/intentional impairment of a
bodily or physical-mental integrity is claimed.

The interdict:

Delictual actions are directed at compensation for patrimonial damage


or the impairment of personality. There is also an important remedy
that doesn’t aim at compensation, but with which a person can avert
an impending wrongful act or prevent the continuation of a wrongful
act that has already commenced = interdict.

Prohibitory interdict: prohibits the wrongdoer from committing a


wrongful act at all or from continuing with a wrongful act.

Mandatory interdict: requires positive conduct on the part of the


wrongdoer to terminate the continuing wrongfulness of an act that
has already been committed.

The courts state three requirements for granting an interdict:


1. There must be an act by the respondent
2. The act must be wrongful
3. No other remedy must be available to the applicant.

Exclusionary clauses:

Parties to a contract can restrict their liability through exclusionary


clauses.
The precise restriction on the wrongdoer’s liability will depend on the
interpretation of the clause concerned and such interpretation will
influence the question of what remedies the prejudiced party has at
his disposal.
Prescription of Remedies:

According to the Prescription Act, a delictual debt prescribes (and the


delictual action is also extinguished) three is after is originated (note
that in terms of the RAF, the prescription periods will differ between
hit and run accidents, and those accidents where the wrongdoer is
known). Prescription begins the moment all the elements of a delict
are present.

JOINT WRONGDOERS

Damage can be caused by a single wrongdoer but it can also be


caused by more than one wrongdoer = joint wrongdoers

Presently the position is regulated by the Apportionment of Damages


Act 34. The act abolishes the common law distinction between joint
wrongdoers and concurrent wrongdoers (act independently)

Joint wrongdoers are now defined as persons who are jointly and
severally liable in delict for the same damage.

A person may thus only be sued as a joint wrongdoer if he was indeed


delictually liable as against the plaintiff.
If joint wrongdoers comply with this requirement, the Act is applicable
irrespective of whether they acted negligently or intentionally.

Maphosa v Wilke: the defendant negligently drove his truck into a


stationary bus. The driver of the bus was also negligent.
The plaintiff, the defendant’s passenger was injured and he instituted
a claim. On the ground of the bus driver’s negligence, the defendant
joined both the owner (the employer of the bus driver) and the insurer
of the bus as 3rd parties (joint wrongdoers).
The court had to decide whether the last mentioned two parties could
be joint wrongdoers and it found that they could.

In Lloyed-Grey case: it was held that the Act is also applicable where
one joint wrongdoer acts intentionally and the other negligently.

Joint wrongdoers = the plaintiff has the right to sue whichever


wrongdoer he chooses for the full amount of the damages (they are in
solidum liable).

Joint wrongdoers can be sued in the same action. If so the court can
order that the joint wrongdoers shall be jointly and severally liable
and that payment by one of them shall absolve the others from any
liability to the plaintiff.
If the court is satisfied that all the joint wrongdoers are before it, it
may apportion the damages among them on the basis of their relative
degrees of fault and may give judgment against every wrongdoer for
his part in the damages. A joint wrongdoer has a right of recourse
against the other if he was made to pay the full amount of the
damages.
SPECIFIC FORMS OF PATRIMONIAL LOSS

Psychological lesions (emotional shock)

A psychological lesion is described as any recognizable harmful


infringement of the brain and nervous system of a person.
The existence of the lesion should be proved by supporting psychiatric
evidence.
The injury can be sustained in various ways: nervous shock, fright or
other mental suffering.

Bester v Commercial Union:


The AD said that the impairment of personality and patrimonial loss
resulting from psychiatric injury or emotional shock caused
wrongfully and intentional or negligently, founds the action for pain
and suffering and the actio legis aquiliae.

Prior to Bester the SA law of delict lacked clear principles in this field.
This resulted in two restrictions on liability for emotional shock:
1. The shock must have originated from a physical injury.
2. The aggrieved party himself must have been in personal danger
of being physically injured.

The 1st concerns wrongfulness and the 2nd negligence or legal


causation.

Wrongfulness: the requirement of physical harm – infringement to the


right to physical integrity, which is wrongful – was rejected in Bester.
According to Bester the brain and nervous system are as much a part
of the physical body as an arm and leg. As a result a physical injury
isn’t absolutely necessary to found liability.

Negligence and legal causation: the requirement of personal danger


was also rejected in Bester and replaced by reasonable foreseeability
of harm (with regard to emotional shock).

In order to establish negligence, the reasonable foreseeability and


preventability of the psychological lesion must be ascertained.
However where the emotional shock is a further consequence of the
wrongdoer’s already established negligent act, the question of legal
causation is at hand, namely whether the wrongdoer’s negligent act
can be regarded as the legal cause of the psychological lesion.
Barnard v Santam Bpk:
The plaintiff sustained serious nervous shock when she heard of her
son’s death in a collision caused by the negligence of the driver of 1 of
the cars.
The supreme court of appeal incorrectly determined the driver’s
negligence with regard to the mother’s shock as well.

In the application of the foreseeability test, it doesn’t make a


difference whether the test serves to establish negligence or legal
causation- this must be determined in every case, taking in the
circumstances that are relevant.

The following factors play a role in this regard:


1. The fact that the psychological lesion resulted from physical
injury
2. The fact that the plaintiff was in personal danger of being
physically injured
3. The fact the plaintiff was informed of the death or injury of a
close relative
4. The fact the plaintiff personally witnessed the death or injury of
a close relative.

Once the court has found that serious emotional shock was
reasonably foreseeable, the wrongdoer is then liable for any
detrimental physical or mental consequence – regardless of whether
they were foreseeable as well (talem qualem rule applies – you take
your victim as you find him).
INJURY OR DEATH OF ANOTHER

Requirements for loss of support:


1. The deceased must have been under a duty to support the
dependent
2. Dependent must have had a right to such support

Action for non dependents:


❖ Executor: for funeral and medical expenses
❖ Heirs and family for funeral expenses
❖ Persons having a duty of support
❖ Master and domestic servant

PURE ECONOMIC LOSS:


The Aquilian action is available to claim damages for pure economic
loss.

1. Pure economic loss may comprise patrimonial loss that doesn’t


result from damage to property or impairment to personality.
Greenfield engineering works: where A suffered financial loss
because a cheque that was sent to him by B was stolen and as a
result of the fact that the cheque hadn’t been crossed, the thief was
able to open a bank account withdraw the money and disappear
with it.
2. Pure economic loss may also refer to financial loss that does flow
from the damage to property or impairment to personality, but
which doesn’t involve the plaintiff’s property or person.
Coronation Brick: A negligently damaged cables that provided
electricity to B’s factory and B suffers loss of production.
3. Pure economic loss includes patrimonial loss that’s the result of
damage to property or injury to personality but where the
defendant didn’t cause the loss or injury.
Kadir v Minister of law and order: as a result of the negligence of A,
B’s motorcar left the road and he was injured. 2 constables who
arrived on the scene failed to take any particulars concerning the
identity of A and as a result of the omission B couldn’t claim from
the MMF for his damage and suffered pure economic loss.

To found liability for pure economic loss, the qualification is that the
wrongdoers conduct must comply with the general delictual
requirements.

Other specific forms of damnum iniuria datum:


1. Injury or death of another
2. Negligent misrepresentation
3. Interference with a contractual relationship
4. Unlawful competition
5. Manufacturers liability
SPECIFIC FORMS OF PERSONALITY INFRINGEMENT
(inuria)

For example, the right to fama or good name

Defamation:

Definition: is the intentional infringement of another person’s right to his


good name. Defamation is the wrongful, intentional publication or words
or behavior concerning another person, which has the effect of injuring
his status, good name or reputation.

Elements of inuria:
1. Act = publication of words or behavior
2. Injury to personality = the defamatory effect of the words or
behavior
3. Wrongfulness = the infringement of the personality right to good
name
4. Intent

Note: It is not an element that deformation be false. True words can


also be actionable in certain circumstances.

Elements:

Publication (act):

Since good name, respect or status which a person enjoys in society


relates to the opinion of others concerning him and defamation
consists of the infringement of his good name, its self-evident that
defamation will only arise if the defamatory statement or behavior has
been publicized to a 3rd person.
Without such publication the opinion of others with regard to the
person involved cannot be lowered.
Publication is necessary for defamation.

This requirement is satisfied if the words or conduct are made known


or disclosed to at least one person other than the plaintiff himself.
Important qualifications:
1. Courts don’t consider the disclosure of defamatory words or
behavior to an outsider who’s unaware of the defamatory
character or meaning thereof in relation to the plaintiff as
publication.
2. The communication of defamatory words concerning a 3rd party
by one spouse to the other, doesn’t constitute publication
according to the decision in Bowles.

Once publication is established the plaintiff must prove that the


defendant was responsible for the publication – as a general rule the
publication is attributed to the defendant if he was aware or could
reasonably have expected that an outsider would take cognisance of
the defamation.

Not only the person from whom the defamatory remark originated but
also any other person that repeats, confirms or draws attention to it is
responsible.

Defamatory effect: wrongfulness:

Wrongfulness with regard to the defamation lies in the infringement of


a person’s right to his good name.
When determining wrongfulness, the question whether the good name
of the person involved has in fact been infringed is irrelevant.
The only relevant question is that in the opinion of the reasonable
person with normal intelligence and development, the reputation of
the person concerned has been injured.

The following principles apply in respect of this test:


1. The reasonable person is fictional, normal, well-balanced right
thinking person.
2. Reasonable person is a member of society in general and not
only of a certain group.
3. The reaction of the reasonable person is dependent on the
circumstances of a particular case
4. Verbal abuse is in most cases not defamatory because it
normally doesn’t injure a persons good name
5. Words are prima facie of according to the ordinary meaning
defamatory or not.
6. If the words have an ambiguous meaning – the 1 defamatory
and the other not – then the meaning most favorable to the
defendant must be followed.
Grounds of justification:

A plaintiff, who proves that a publication is defamatory and that it


refers to him, provides only prima facie proof of wrongfulness.
A presumption of wrongfulness then arises which places the onus on
the defendant to rebut it; he can do this by proving a ground of
justification:

1. Privilege: This exists when someone has a right, duty or interest


to make specific defamatory assertions and the people to whom
the assertions are publicized have a corresponding right, duty
or interest to learn of the assertions.
Privilege grants a person the legal right to injure another’s good
name and in doing so sets aside the prima facie wrongfulness of his
conduct.

Absolute privilege: defendant is protected absolutely, in the


sense that liability for defamation is excluded.

Relative privilege: defendant enjoys conditional protection; this


protection falls away as soon as the plaintiff proves the
defendant exceeds the bounds of privilege.

1. Discharge of a duty or furtherance of an interest: person


has a legal, moral or social duty or legal interest in
making defamatory assertions to another, who has a duty
or interest to learn of the assertions. A father and his
daughter with regard to the character of her fiancé.
2. Judicial or quasi-judicial proceedings – medical and
dental council
3. Privileged reports

2. Truth and public interest: The prima facie wrongfulness of the


defendants conduct will be cancelled if he proves that the
defamatory remarks were true and in public’s interest. The
defendant only has to prove that the remarks are substantially
true.

3. Fair comment: The prima facie wrongfulness of the defamatory


publication may be set aside if the defendant proves the
defamation forms part of a fair comment on facts that are true
and in the public interest

4 Requirements:
1. The defamation must amount to comment
2. The comment must be fair
3. The facts on which the comment is based must be true
4. These facts must be in public’s interest.
Grounds Excluding Fault:

1) Animus inuriandi (negligence): This is accepted as a requirement


for defamation – negligence is therefore insufficient to hold the
wrongdoer liable.
National Media ltd v Bogoshi: the court introduced negligence as
the basis of liability of the press for defamation.
2) Mistake: person is unaware of the wrongfulness of his
defamatory publication, because he bona fide believes that his
conduct is lawful. Consciousness of wrongfulness, an essential
element of intent, is absent and therefore intent is also absent
as a result of his mistake.
3) Jest: if the defendant proved that he published the defamatory
words in jest, he should be able to rebut the presumption of
animus = reason that his will wasn’t directed at the
infringement of the plaintiffs good name. The courts don’t follow
this approach = courts regard the reasonable bystander should
regard the words as a joke.

Rights relating to dignitas (other forms of rights)

1. The right to dignity


2. Right to privacy
3. Right to identity
4. Right to feelings
5. Breach of Promise (adultery/abduction/enticement/
harbouring)
FORMS OF LIABILITY WITHOUT FAULT
(Formulated by an English case: Rylands v Fletcher)

Damage caused by animals

1) Actio de pauperie:

The prejudiced person may claim damages from the owner of a


domestic animal, which has caused damage.
Fault on the part of the owner ISNT a requirement for liability.

The following requirements must be met:


1. The defendant must be the owner of the animal when the
damage is inflicted.
2. The animal must be a domestic animal
3. The animal must act contrary to its own nature when
inflicting damage – as a rule an animal doesn’t act contrary
to its own nature if it’s reacting to external stimuli –
defences: vis major, culpable conduct on the part of the
prejudiced person and provocation.
4. The prejudiced person or his property must be lawfully
present at the location when the damage is inflicted.

2) Actio de Pastu:

Damages are claimed from the owner of an animal, which caused loss,
by eating plants.
Strict liability on the part of the owner.

Requirements:
1. The defendant must be the owner of the animal when the
damage is caused
2. Animal must cause damage by eating plants
3. The animal must be of its own volition when causing the
damage.

3) Actio de feris:

In terms of an edict of the aediles curules, the bringing of wild animals


onto or into a public place was prohibited. If a person broke this rule
and the animal caused damage to someone, the offender (who does
NOT need to be the owner) was liable. Problem: it is unclear whether
this action is still part of our law.

4) Vicarious liability:

Strict liability of one person for the delict of another.


The former is indirectly vicariously liable for the damage caused by
the latter.
Employer – employee:

Where the employee acting within the scope of his employment


commits a delict, his employer is fully liable for the damage.

3 requirements:
1. There must be an employer-employee relationship at the time
the delict was committed
2. The employee must commit a delict
3. The employee must act within the scope of his employment
when the delict is committed

The employee acts within the scope, if he acts in the execution or


fulfillment of his duties in terms of the employment contract.
He acts outside the scope where he disengages himself completely
from his employment and promotes his own interests.

Rabie: an act done by a servant for his own interests, although


occasioned by his employment may fall outside the scope of his
employment and that in deciding whether an act of the servant does
so fall, some reference must be made to the servant’s intention.
The test is subjective.

– If there is a close link between the servants act for his own interest
and the business, the master may yet be liable = objective test.

The commission of a delict during the performance of a forbidden act


should be seen – if the forbidden act is connected to the general
character of the employers work and thus falls within the scope of his
employment, the employer would be vicariously liable.

Motorcar owner – motorcar driver:

Where a motorcar owner allows someone else to drive his car and the
driver negligently causes an accident, the owner is fully liable for the loss
provided that the following 3 requirements are met:

1. The owner must request the driver to drive the car or supervise
his driving
2. The vehicle must be driven in the interests of the owner
3. The owner must retain the right of control over the manner in
which the vehicle is driven.
PVL 3703
EXAM QUESTIONS

1. Name FOUR differences between a delict and a crime. (4)

DELICT CRIME
1. Protects private interests (private law) 1. Protects public interests (public law)
2. Aggrieved party institutes action 2. State prosecutes
3. Objective = claim damages as 3. Objective = punish criminal
compensation 4. Can have attempted crime
4. No attempted delict

2. Write brief notes on the similarities and differences between breach of contract and
delict. (4)

DELICT BREACH OF CONTRACT


1. Excludes non-fulfilment of a duty to 1. Breach = Non-fulfilment of a
perform (real right) contractual obligation to perform
2. Primary remedy = damages (personal right)
2. Primary remedy = performance of the
contract.

3. Name the THREE most important delictual actions and explain how they differ from
each other in respect of :

(a) the type of harm for which action is instituted and


(b) the form of fault usually required for the institution of each action. (9)
DELICTUAL ACTION TYPE OF HARM FORM OF FAULT
REQUIRED
Actio legis Aquiliae Damages for wrongful Intention or negligence
causing of patrimonial
loss(damnum iniuria datum)
Actio iniuriarum Satisfaction for wrongful Intention
injury to personality
Action for pain and Compensation for wrongful Intention or negligence
suffering impairment of a bodily or
physical-mental integrity

4. List TEN fundamental rights entrenched in chapter 2 of the Constitution which are
relevant to the law of delict. (5)

1. Religion
2. Speech
3. Life
4. Privacy
5. Property
6. Security
7. Education
8. Adequate housing
9. Good name
10. Dignity

7. Write brief notes on the indirect application of the Bill of Rights to the law of
delict. (5)

Is all private law rules, principles and those regulating law of delict – subject to Chapter 2 of the
Constitution (namely fundamental rights e.g. right to religion, life, dignity, etc.)
Indirect application is about delictual issues e.g. infringement of one of fundamental rights.
1st use indirect application before use direct application.
CONDUCT
1. Define conduct. (2)

Conduct is a voluntary human act or omission :


• Human – where an animal is used as an instrument a human act is still present. A juristic
person can act through its agents (e.g. company)and be held delictually liable for its actions.
• Voluntary conduct – act must have been performed voluntarily i.e. wrongdoer must have
had control over his muscular movements.

2. Write notes on conduct or act as an element of delict. Include a brief discussion on the
most important defence excluding this element. (10)

Conduct is a voluntary human act or omission. With a human act, a person may use an animal as
an instrument. Juristic persons act through their organs and may be held delictually liable.
Conduct must be voluntary, this means that the conduct is susceptible to the control by the will
of the person involved, and that the person has the mental ability to control his muscles and
movements. This does not mean that he willed or desired his conduct or that the conduct is
rational or explicable.
Conduct can consist of a commissio or an omissio. Of importance is the fact that liability for an
omission is more restricted than a positive act. The law is hesitant to find a legal duty to act
positively.
Defence excluding conduct
Defendant may argue that the conduct complained of was not voluntary i.e. it was mechanical –
he then raises the defence of automatism.
Following conditions may cause a person to act involuntarily: absolute compulsion; sleep;
unconsciousness; a fainting fit; epileptic fit; serious intoxication; blackout; reflex movements; heart
attack; hypnosis; emotional pressure or metal disease.
The situation in which the person acts involuntarily must not have been created intentionally to
harm another.
Defendant has the onus to prove his defence on a balance of probabilities.
In Molefe v Mohaeng the court decided differently – SCA decided that the defence of automatism
does not have any effect on the burden of proof. Appellant must prove that respondent acted
negligently and that includes proof that the negligent conduct on which it is based was indeed
voluntary conduct.

3. X is involved in an accident whilst driving his car. When he regains consciousness, he


has no recollection of how the accident took place. He is hospitalised and during
treatment for head injuries, the doctors determine that he suffered an epileptic fit at the
time of the accident. The car of Y, the other person involved in the accident, is badly
damaged.

Can it be said that it was an act on the part of X that damaged Y’s car ?
Will it make a difference to your answer if X had been receiving treatment for epilepsy
before the accident, but had failed to take his medicine for several days before the
accident took place ? Discuss. (10)

Conduct is a voluntary human act or omission :


• Human – where an animal is used as an instrument a human act is still present. A juristic
person can act through its agents (e.g. company) and be held delictually liable for its
actions.
• Voluntary conduct – act must have been performed voluntarily i.e. wrongdoer must have
had control over his muscular movements.
Defence of automatism :
Voluntary conduct on the part of defendant is a requirement for delictual liability.
Defendant could argue that the conduct complained of does not satisfy requirement of
voluntariness.

This is where someone acted mechanically (incapable of controlling his bodily movements) :

• Sleep (Dhlamini)
• Unconscious
• Fainting fit
• Absolute compulsion (X grabs Ys hand and stabs P using Y’s hand)
• Serious intoxication
• Black out
• Epileptic fit (Victor)

If these are present a person is incapable of controlling his bodily movements = purely mechanical
action and that person raises the defence of automatism – which must be proved ona balance of
probabilities by the defendant (onus on the defence).
Not an act on the part of X because he was acting out in a mechanical fashion and was not
capable of controlling his bodily movements.

Failure to take epilepsy medication


Yes, it would make a big difference because defence won’t succeed if defendant intentionally
created the situation in which he acts voluntarily in order to harm another (action libera in causa).
Defendant may not successfully rely on this defence if he was negligent in his automatic conduct
– reasonable person would have foreseen the possibility of causing harm while in a state of
automatism.
Thus X will be delictually liable for the damage of Y.
4. Peter and Sue go to a party. At the party Peter drinks a whole bottle of rum. Sue
notices that he drinks too much but still accepts a ride home from him. Sue forgets to
buckle up. On the way home, Peter loses control of the car and hits a tree. Sue suffers
neck & back injuries. She wants to recover medical expenses from Peter. Peter raises
a number of defences, one of them being automatism.

Will he succeed in this defence ? Refer to case law and legislation (10)

Conduct is a voluntary human act or omission. With a human act, a person may use an animal as
an instrument. Juristic persons act through their organs and may be held delictually liable.
Conduct must be voluntary, this means that the conduct is susceptible to the control by the will
of the person involved, and that the person has the mental ability to control his muscles and
movements. This does not mean that he willed or desired his conduct or that the conduct is
rational or explicable.
Conduct can consist of a commissio or an omissio. Of importance is the fact that liability for an
omission is more restricted than a positive act. The law is hesitant to find a legal duty to act
positively.
Defence excluding conduct
Defendant may argue that the conduct complained of was not voluntary i.e. it was mechanical –
he then raises the defence of automatism.
Following conditions may cause a person to act involuntarily: absolute compulsion; sleep;
unconsciousness; a fainting fit; epileptic fit; serious intoxication; blackout; reflex movements; heart
attack; hypnosis; emotional pressure or metal disease.
The situation in which the person acts involuntarily must not have been created intentionally to
harm another.

In respect of sane automatism defence, (not mental illness) onus is on plaintiff to prove defendant
acted voluntarily and therefore not mechanically. Sane automatism doesn’t mean there’s no
voluntary act whatsoever by the defendant which caused damage, but only that conduct in
question wasn’t voluntary.
Defendant may not successfully rely on this defence if he was negligent in his automatic conduct
– reasonable person would have foreseen the possibility of causing harm while in a state of
automatism.
Peter won’t succeed in this defence – he can’t raise it due to the fact that he acted voluntarily. He
acted voluntarily in the fact that he knew that he was going to drive a car after the party, but drank
anyway.

5. John and Mary go to a party. In the course of the evening, John consumes a whole bottle
of rum. Mary notices that John drinks too much, but she still accepts an offer from him to
drive her home. On the way to Mary’s house, John loses control over the car and collides
at high speed with a tree. Mary sustains back injuries for which she is hospitalised for a
period of 2 weeks. Mary wishes to recover her medical expenses from John.
Answer the following questions with reference to the above set of facts :

(a) What type of loss did Mary suffer ? (½)

Patrimonial loss.
(You could have also stated that Mary suffered non-patrimonial loss in the form of pain and
suffering but because the question states that she wishes to recover her medical expenses
from John, patrimonial loss was more correct)

(b) What delictual action can Mary therefore consider to institute against John ?(½)

Actio legis Aquiliae


(if you stated in (a) that Mary suffered non-patrimonial loss, your answer here must be the
action for pain and suffering)

(c) John avers that he didn’t act voluntarily for the purposes of delictual liability.
Identify his defence and briefly discuss the merits thereof. (4)

Automatism is the defence that the conduct complained of was done involuntarily. Conduct
is involuntary if a person acts mechanically, that is, he or she is incapable of controlling his
or her bodily movements. Serious intoxication may eliminate the voluntariness of conduct,
but the situation in which the person acts involuntarily must not have been created
intentionally to harm another, (action libera in causa). The defendant may also not
successfully rely on the defence of automatism where he was negligent with regard to his
automatic conduct. This will be the case where the reasonable man would have foreseen
the possibility of causing harm while in a state of automatism, for instance, where he drinks
alcohol whiles knowing or reasonably foreseeing that he will later drive a motor vehicle.
In the given set of facts, John consumes an entire bottle of rum. He was probably so drunk
that he was incapable of controlling his bodily movements and therefore acted involuntarily.
The reasonable man would, however, have foreseen that he might cause harm if he drinks
a bottle of rum while knowing or reasonably foreseeing that he will later drive a motor
vehicle, and John should also have foreseen this

(d) Assume that the court finds that John has acted. John now avers that Mary has
consented to the risk of injury. Discuss the merits of this defence. (6)

Consent to injury is a unilateral legal act which restricts the injured person’s rights – it may
be given expressly or tacitly and must be given before the prejudicial conduct. Prejudiced
person himself must consent.
Consent as a ground of justification exempts the wrongfulness on the part of the wrongdoer.
The consent must be given freely or voluntarily, and the person giving consentmust also be
capable of volition. This means that he must be mentally advanced enoughto realise the
implications of his conduct and have full knowledge of the extent of the possible harm. The
consenting party must also realise or fully appreciate what the nature or full extent of the
harm will be and must subjectively consent to the full extent of the prejudicial act. The
consent must be permitted by the legal order. Consent to bodily injury is normally contra
bones mores, unless the contrary is evident.
Nothing in the facts of this case indicates that Mary’s consent to the risk of her injuries
would be reasonable, and it will therefore be regarded as contra bones mores and will not
be accepted as a valid defence.

(e) Assume that the defence of consent to the risk of injury is rejected by the court. John
now avers that he isn’t accountable. Discuss the merits of his defence.(3)

A person is accountable (culpae capax) if he has the necessary mental ability to distinguish
between right and wrong and if he or she can also act in accordance with suchappreciation.
Intoxication may cause a person t lack the necessary mental capacity and not be
accountable. If John had been accountable at the stage when he started consuming
alcohol, his conduct may, however, still be regarded as negligent.

(f) Assume that John was indeed accountable. Did he act intentionally ? (3)

An accountable person acts intentionally if his will is directed at a result which he causes
while conscious of the wrongfulness of his conduct. In the question John did not act
intentionally because his will was not directed at the result he caused and he might also not
have been conscious of the wrongfulness of his conduct.

(g) Assume that John didn’t act intentionally. Did he act negligently ? Discuss briefly
with reference to case law. (4)

In Kruger v Coetzee the reasonable man test for negligence is stated: a person is negligent
if the reasonable man in his position, would foresee the reasonable possibility ofhis conduct
injuring another in his person or property and causing him patrimonial loss and would take
reasonable steps to guard against such occurrence and the defendant failed to take such
steps. In the question, John acted negligently.

(h) Assume that John did act negligently. John avers that Mary had contributory intent
in respect of her damage. Discuss the merits of this assertion and indicate what the
effect of successful reliance thereon would be. (8)

Mary’s conduct can only be regarded as intentional if it can be proven that she foresaw the
possibility that she could be injured, but nevertheless recklessly got into the car with John
(dolus eventualis), well aware of the possible wrongfulness of her conduct.
If it could be proven that Mary intentionally contributed to her own damages, the
Apportionment of Damages Act 1956 would be relevant – this Act only refers to ‘fault’.
In general, the term ‘fault’ refers to both intent and negligence. It would appear that the
legislature intended to make provision for only the defence of contributory negligence on
the part of the plaintiff and not for the defence of contributory intent, due to inter alia the
wording used in the long title and the heading of section 1.

(i) Assume that John’s defence of contributory intent fails. John now avers that since
Mary hadn’t fastened her seatbelt, she was contributory negligent in respect of her
own injuries. Discuss the merits of this defence with reference to King v Pearl
Insurance Co. Ltd and other relevant cases and indicate what the effect of
successful reliance thereon would be. (7)
In King v Pearl the court held that the plaintiff’s failure to wear a crash-helmet
while driving a scooter did not constitute contributory negligence.
Only negligence with regard to the damage-causing event, as opposed to the
damage itself, is considered for purposes for the Apportionment of Damages Act
– this approach was criticized in Bowkers.
In Vitoria the court held that the failure to wear a seat belt does constitute a
contributory negligence for purposes of the Act.
Effect of a successful reliance on a defence of contributory negligence is that an
apportionment of damages, in terms of the Apportionment of Damages Act must
be made in relation to each party’s relative degree of fault.
Contributory negligence of Mary is only relevant in so far as it has led to an increase
in her damage.
Therefore section 1(1) (a) applies only in respect of the damage for which Mary is
indeed also responsible. If Mary, for example suffered R10 000 damage ifshe had
indeed fastened her seatbelt, but R15 000 if she had not fastened it, only R5 000
will be subject to apportionment.

WRONGFULNESS

1. Write notes on the general criterion for wrongfulness, as well as two doctrines which
may be considered applications thereof. (10)

The general norm to determine whether a particular infringement is unlawful is the legal
convictions of the community or the boni mores. The boni mores test is an objective test based
on the criterion of reasonableness. The basic question is whether, according to the legal
convictions of the community and in the light of all the circumstances of the case, the defendant
infringed the interests of the plaintiff in a reasonable or unreasonable manner.
The doctrines that are practical applications thereof are :

(a) The infringement of a subjective right


Fundamental premise of this doctrine is that the infringement of such a right is wrongful. A
dual investigation is necessary:
• Has the subject-object relationship in fact been disturbed? (factual infringement)
• Was the infringement in a legally reprehensible way? (norm-violation)
A subjective right is therefore infringed when the relationship between the holder of a
right and the object of the right has been infringed in a legally reprehensible manner.
(b) Wrongfulness as a breach of a legal duty
Sometimes the infringement of a subjective right is not appropriate to determine
wrongfulness, e.g. where no clearly defined right exists for instance in misrepresentation
cases, cases of omission or pure economic loss. In instances like this, wrongfulness is
determined by asking whether the defendant has a legal duty to prevent loss. In general
there is no duty to prevent loss. The question is whether according to the boni mores or
legal convictions of the community there was a legal duty to act positively or to avoid pure
economic loss.

2. X forgets to warn Y that a cable is carrying an electric current. Y touches the cable
and sustains serious burns. Against Y’s actions for damages, X raises a defence
that he didn’t will or desire to harm Y and that he therefore didn’t act. Discuss the
merits of this defence. (2)

Conduct can be in the form of a commission or omission.


Liability for an omission is in general more restricted than liability for a positive act (commission)
– law is hesitant to find that there was a legal duty on someone to act positively and so to prevent
damage to another. But this must be distinguished from the case where a person fails to take
precautions against the occurrence of damage and his failure isn’t part of a positive act.
S v Russell has similar facts and accused was found guilty.
Therefore X should be held liable.

3. “Prior conduct is always a requirement in deciding whether an omission has


takenplace …”

Do you agree ? Discuss with reference to case law. (10)

Conduct is a voluntary human act or omission.


Liability for an omission – general rule : a person doesn’t act wrongfully for the purpose of law of
delict where he fails to act positively to prevent harm to another. There’s no absolute duty to
prevent loss.
Liability follows only if an omission was in fact wrong – where a legal duty rested on the defendant
to act positively to prevent harm from occurring and he failed to comply with that duty. Prior
conduct – a person acts prima facie wrongfully where he creates a new source of danger by
means of a commission and then fails to eliminate that danger, with the result that harm is caused
to another person.

Prior conduct isn’t a prerequisite for the existence of a legal duty – but at one stage this was the
only category where liability was imposed on a failure to act.
Municipality cases – yes, prior conduct is a requirement for liability for an omission only if it’s
reasonable.
Minister van Polisie v Ewels (policemen failed to prevent assault) – result in this case is that
you don’t need prior conduct and this was confirmed in Van der Merwe v Burger

4. Mrs. M, deep in thought, walks on a sidewalk. Too late she notices the presence of
a huge hole in the middle of the sidewalk. She falls into the hole and fractures a hip.
It transpires that the hole was dug by municipal workers to implement a new
sewerage system. Mrs. M wants to recover damages from the municipality. The only
precautionarymeasure taken by the municipality to protect the public was to cordon
off the hole with asingle red and white ribbon.

Apply the following questions with reference to this set of facts :

(a) The representative of the municipality avers that the municipality didn’t act for the
purposes of law, since no municipal officer willed Mrs. M’s injuries. Briefly discuss
the merits of this statement. (2)

Conduct can be in the form of a commission or omission. Liability for an omission is in


general more restricted than liability for a positive act (commission).
Law is hesitant to find that there was a legal duty on someone to act positively and so to
prevent damage to another. But this must be distinguished from the case where a person
fails to take precautions against the occurrence of damage and his failure isn’t part of a
positive act.
Municipality had a legal duty in terms of the legal convictions of the community to prevent
this accident from happening.

5. X is the proud owner of a horse. Y poisons the horse and it dies. How
does oneestablish that Y’s conduct was wrongful ? Explain with reference to :

(a) the general criterion used

General test used is the boni mores test (legal convictions of the community).

(b) a practical application of the mentioned criterion and

More precise methods of determining wrongfulness :


• Infringement of a subjective right
• Non-compliance with a legal duty to act

(c) the two stages in which the enquiry will proceed. (5)

1st stage = prior conduct


2nd stage = control of a dangerous object

6. In a certain town, the paved sidewalks are in a poor condition due to erosion.
Several holes and furrows have developed in the sidewalks. The municipality
neglects to repair the sidewalks, despite requests to this effect by several of the
residents. One day, Mrs M, an aged resident, inadvertently steps into a hole in a
sidewalk, falls, and suffers serious injuries for which she is hospitalised for a month.
Mrs M wishes to recover damages from the municipality in a delictual action.

Discuss only whether the conduct of the municipality was wrongful. Refer in your answer
to relevant case law. (10)

The conduct of the municipality takes the form of an omission.


Since Halliwell decision, courts maintained that an omission could not be wrongful on its
own, but had to be preceded by positive conduct creating a new source of danger – rule
was known as omissio-per-commissionem rule or the prior-conduct rule.
Rule was applied in several cases in which municipalities were the defendants – the so-
called municipality cases.However, rule was criticised for its unsatisfactory application by
the courts. In Regal v African Superslate, AD held that prior conduct was not necessary
for an omission to be wrongful. However, the case concerned an application for an
interdict, and not an action for damages.
In Minister van Polisie v Ewels the AD declared that prior conduct was not a
prerequisite for an omission to be wrongful. The question whether the alleged
wrongdoer had a legal duty to act, and whether such a legal duty existed or not, had to
be determined with reference to the legal convictions of the community. Prior conduct
could be a factor pointing towards the existence of a legal duty, but it was not an absolute
requisite for the existence of a legal duty.

In Van der Merwe, the judgment in the Ewels case was applied to a municipality.
Therefore, in applying the discussed principles and case law to the given facts, a
legal duty rested on the municipality to maintain the sidewalks in a safe condition.
Accordingly, the omission of the municipality was wrongful.

7. How is the wrongfulness of an omission determined ? Discuss with reference to


case law on the so-called omissio per commissionem rule. (10)

General rule regarding liability for an omission


A person doesn’t act wrongfully for the purpose of law of delict where he fails to act positively to
prevent harm to another. There is no absolute duty to prevent loss.
Liability follows only if an omission was in fact wrong – where a legal duty rested on the defendant
to act positively to prevent harm from occurring and he failed to comply with that duty. Prior
conduct
A person acts prima facie wrongfully where he creates a new source of danger by means of a
commission and then fails to eliminate that danger, with the result that harm is caused to another
person.
Prior conduct isn’t a prerequisite for the existence of a legal duty – but at one stage this was the
only category where liability was imposed on a failure to act.
Municipality cases
Ewels (policemen failed to prevent assault) – result in this case is that you don’t need prior
conduct. Confirmed in Van der Merwe v Burger.
8. Define ‘private defence’. (2)

When the defendant directs his actions against another’s actual (or immediately threatening)
wrongful act, to protect his own legally recognised interests or the interests of another.

9. R, a robber is fleeing the police. As the police dog starts to breathe down his neck,
he runs into a zoo and seeks refuge in a large enclosure with a wide trench around
it. It transpires that the enclosure houses a large gorilla, not at all happy with R’s
intrusion onits territory. The gorilla charges at R, who fires 2 shots with his pistol,
causing the gorilla to collapse, seriously injured. The gorilla is hospitalised in a
private clinic in the city where it is treated for its wounds. The zoo authorities want
to recover from R the expenses in respect of this medical treatment.

Advise R whether he can successfully raise the following grounds of justification and
substantiate your answer in each case:

(a) Private defence (1)

No, defence must be against a human attack

(b) Necessity (4)

A state of necessity will exist if R can prove that he was laced in such a position by superior
force (gorilla), that he was able to protect his interests (his life) only be reasonably violating
the interests of an innocent 3rd party (zoo authorities, as owner of gorilla)
There is authority for both the view that a state of necessity created by the defendant (by
jumping into gorilla’s den) excludes a plea of necessity and for the view that a person may
rely on necessity even if he himself created the state of necessity
Van der Merwe and Olivier = damage caused by a preceding wrongful act (as in this set of
facts) may still be actionable, although the act committed out of necessity (shooting the
gorilla) may be lawful.
R can also argue that his right to life (protected in Constitution) must surely be worth more
than the zoo authorities’ right to ownership over an animal

(c) Assume for the purposes of the rest of your answer that R’s conduct was wrongful.
Advise R whether he can avoid liability by relying on the doctrine of sudden
emergency. (5)

Doctrine of sudden emergency =when a person has to take a decision in a situation of


sudden emergency and there is insufficient opportunity to consider all the consequences
of his actions – this factor MUST be taken into account in deciding whether he is negligent.
Error of judgement does not necessarily amount to negligence because the reasonable
person may also make an error of judgement in certain circumstances.
Requirementswhich must be met in this doctrine for a defendant’s action not to amount to
negligence:
1. R (wrongdoer) must have faced a situation of imminent peril (attack by gorilla)
2. R mustn’t have caused the situation through his negligence or imprudence
3. R mustn’t have acted in a grossly unreasonable manner (by shooting the gorilla) –
must be established whether a reasonable person in the same circumstances would
have made the same error in judgment as the wrongdoer.
Here R’s conduct was negligent.
R created the situation through his own negligence – thus, according to the requirements of
this doctrine his actions will not be excused (Brown v Hunt)

10. Define ‘necessity’. (3)

A state of necessity exist when the defendant is placed in a position by superior force (vis major)
that he is able to protect his interests or those of someone else, only by reasonably violating the
interests of an innocent 3rd party.

11. X offers to take Y’s well-trained watch dog for a walk. X sees that his sworn enemy,
Z, isstrolling in a park. X instructs the dog to attack Z and the dog immediately
charges at Z with blood-curdling growls. Z draws his pistol and kills the dog. Y
wishes to recover the value of the dog from Z in a delictual action.

Of necessity and private defence, which one of these two grounds of justification is
available to Z ? Explain. (5)

Z can rely on necessity as a ground of justification because he was protecting his own interest
i.e. his life.
Necessity = a state of necessity exist when the defendant is placed in a position by superior force
(vis major) that he is able to protect his interests or those of someone else, only by reasonably
violating the interests of an innocent 3rd party.
Guidelines to determine necessity:
• An actual state of necessity must exist
• Possible state of necessity must be determined objectively
• State of necessity must be present or imminent
• Defendant need not only protect his own interests but can protect the interest of another
• Not only life or physical integrity but other interests like property may be protected out of
necessity
• Person can’t rely on necessity where he’s legally compelled to endure the danger.
• Interest which is sacrificed mustn’t be more valuable than the interest protected.
• The act of necessity must be only reasonable possible means of escaping from the
danger.
• Whether necessity can justify homicide ?
R v Dudley = no, S v Goliath = yes, Van der Merwe = yes.
12. X, brandishing a hunting knife, tells Y that if Y does not help him to kill Z, X will kill
Y. Y hits X over the head with a blunt object. X suffers severe concussion. What
ground of justification can Y raise if X institutes a delictual action against Y ?
Substantiate your answer. (10)

The facts are of such a nature that we must consider two grounds of justification, namely
private defence and necessity. Private defence is present when the defendant directs his
actions against another person's actual or imminently threatening wrongful act in order
to protect his own legitimate interests or such interests of someone else. ✓
Private defence will be present if the following requirements are met:
1. Defence must be directed against the aggressor; ✓
2. Defence must be necessary to protect the threatened right – and this implies
that there must be no reasonable alternative to act of defence; ✓and
3. Act of defence must not be more harmful than is necessary to ward off the attack.
✓ Requirement (3) implies that: there must be a measure of proportionality between the
attack and the defending act, although absolute proportionality is not required; ✓the
value of the protected interest and the sacrificed interest may differ; ✓ the interests need
not be similar in character; ✓ and the means of defence employed by the defender
need not be similar to those of the attacker ✓.

Necessity, on the other hand, exists when the defendant is placed in such a position by
a superior force that he is able to protect his interests or those of someone else only by
reasonably violating the interests of an innocent third person.✓
The most important difference between private defence and necessity is the fact that
private defence is directed at an attack by a wrongdoer; whereas when acting out
necessity the interests of an innocent third party are prejudiced. ✓In other words, if the
plaintiff was an attacker, private defence may be applicable, whereas if the plaintiff was
an innocent third person, necessity may be applicable.
From the above it is clear that private defence is the appropriate ground of justification in
our setof facts. Y harmed the interests of X, and X was certainly no innocent third person.
On the contrary, X had directed wrongful attacks or imminently threatening attacks
against both Z and
Y. ✓If Y had assisted X to harm Z, and Z somehow survived and instituted a delictual
claim against Y, it would have been appropriate to consider whether necessity was
applicable (compare the facts of S v Goliath 1972 (3) SA 1 (A)
13. Name SIX requirements for valid consent. (6)

1. Consent must be given freely and voluntarily


2. Person giving consent must be capable of volition – must be intellectually mature enough
to appreciate the implications of his acts and must not be mentally ill or under the influence
of the drugs.
3. Consenting person must have full knowledge of the extent of the prejudice.
4. Consenting party must realise or appreciate fully the nature and extent of harm
5. Consenting party must subjectively consent to the prejudicial act.
6. Consent must not be contra boni mores.

14. X and Y are sworn enemies. One day they unexpectedly bump into each other. X
whips out a water pistol, directs it at Y and says, “Die scum”. Y, firmly under the
impression that his life is in danger, produces his real gun and wounds X in the
forearm with a quickshot. X wishes to recover from Y the medical expenses for
treatment of his wound.

Is there any ground of justification available to Y ? Discuss briefly. (4)

Test under these circumstances = objective.


Attack must be really imminent or present and subjective belief is irrelevant.
Private defence is not available, and putative defence is not a ground of justification.
FAULT
1. Must the test for negligence be adapted when applied to a child wrongdoer? Discuss
critically the development of our case law in this regard. (7)

Before 1965
There was a tendency for the courts to take into account the youthfulness of the
wrongdoer in determining his negligence – test for negligence by means of the conduct
expected from the reasonable child of the wrongdoer’s age and intellectual development.
In 1965
Jones v Santam:
AD adopted a new approach to determine negligence with respect to children. Court
implied thatcriteria for determining negligence is always objective in the sense that in all
situations the test of reasonable person is applied.
According to this approach in determining if a child acted with negligence:
1st see whether the child concerned met the standard of care required of a reasonable
person
2 nd ask whether the child was accountable for his actions.
Criticism on this approach: seems that earlier test of the reasonable child was more
acceptable. Clear that the court in Jones placed the cart before the horse by 1 st looking
into fault and then into accountability.
Roxa v Mtshayi – AD accepted this criticism by 1st testing accountability and2ndfor
negligence.

2. When is a child accountable ? Discuss. (4)

Test for accountability = if person has the necessary mental ability to distinguish between right
and wrong and can act in accordance with such an appreciation.
A child’s accountability falls under persons who lacks the necessary accountability:
Youth:
• 0 – 7 : always lack accountability –irrebuttable presumption that he not accountable.
• 7 – 14 :rebuttable presumption that he lacks accountability – he’s assumed unaccountable
until the contrary is proven.
• 14 – UP : accountable on a balance of probabilities.
Thus a child will be accountable if he is over the age of 14.

3. Write critical notes on the way in which our courts have determined the negligence of a
child before and after 1965. (10)

Before 1965
Courts tended to consider the youthfulness of a wrongdoer in determining negligence –
they tested for negligence by means of the conduct expected of the reasonable child of
the wrongdoer’s age and intellectual development.
After 1965
AD in Jones v Santam adopted a new approach – criterion for negligence is always
objective, therefore it is the test of the reasonable man under all circumstances.
According to this approach, it must 1st be determined whether the child met the required
standard of care of the reasonable man. 2nd it must be asked whether the child was
culpae capax (accountable).
When answering 1st question, the childhood of the wrongdoer is relevant.
When answering 2nd question (concerning the wrongdoer’s accountability) – subjective
qualities like the child’s intellect, maturity, experience, insight, etc. must be taken into
account to determine whether the child could differentiate between right and wrong and
act in accordance with that insight.
New approach has been criticised as follows:
• Earlier approach of the test of the reasonable child is more acceptable because one
cannot realistically measure a child against an adult’s standard.
• It must 1stbe determined whether a person is accountable before testing for
negligence.
In a subsequent decision in Roxa, the AD followed the correct order by testing first the
accountability and then for negligence.
However, in Weber, the AD once again confirmed the decision in the Jones case. In the
Weber case it was held that the accountability of a child should be determined with
reference to the specific situation in which he found himself, and that if this approach was
implemented with insight, most of the criticism against the Jones case would disappear.

4. X is a passenger in Y’s motorcar. Due to Y’s negligent driving, they are involved in a
collision. X had not fastened her seatbelt. She is hospitalised for multiple injuries and
her medical expenses amount to R10 000. If she had fastened her seat belt, her injurie
would have amounted to R6 000. X wishes to recover her damage from Y.

Which defence is available to Y ? Discuss with reference to case law. (10)

Consent to risk of injury


Y can perhaps aver that X tacitly consented to risk of injury.
However, this defence can be disqualified immediately as it is contra bonos mores to
consent to (risk of) serious bodily injury.

Contributory Fault
Common Law: general rule = fault on part of plaintiff precluded him from claiming
damages from defendant, who was also to blame for causing the damages.
Doctrine of contributory negligence was created in Davies v Mann: plaintiff haltered
donkey in road – defendant driving wagon collided with donkey = negligence on both
sides.
In terms of ‘all or nothing’ rule which was enforced at that time, plaintiff would have been
unable to claim damages from defendant BUT court adopted new approach : since
defendant had last opportunity to avoid collision, plaintiff’s negligence ignored.
Court accepted if negligence of two people contributed to causing of particular result and
one or both suffered damages – neither could claim unless negligence of one was
decisive cause (who had last opportunity) of accident = negligence of other party ignored
and he could claim full damages.
If reasonable person placed in position of X, the reasonable person would have foreseen
the possibility of heightened damage and would have taken preventative steps by
fastening her seatbelt.
Common law position changed by Apportionment of Damages Act.

Apportionment of Damages Act (1956):


Fault includes both negligence and intention.
Defendant who negligently caused plaintiff harm could not ask for reduction in
damages because of contributory fault.
Distinction between four situations:
• Defendant negligence – plaintiff intention = no claim.
• Defendant intention – plaintiff intention = no claim.
• Defendant intention – plaintiff negligence = full claim.
• Defendant negligence – plaintiff negligence = apportion the damages (Jones
Case).Criteria for apportionment of damages:
Reasonable person in the position of the defendant:
1. Would foresee the reasonable possibility of his conduct injuring another &
causing him patrimonial loss.
2. Would take reasonable steps to guard against such
occurrence And the defendant failed to take such steps.

Section 1 applies only to damages caused partly by fault of plaintiff and partly of
defendant. Reasonable person test applies – dealing with a deviation from the
standard of care which applies to all people in community.

Court does not take into account that defendant’s conduct has contributed to harm
to a greater extent than plaintiff’s conduct.
If court is satisfied that negligence of both parties is causally connected to damage,
question of causation is solved.

Method of determining who should bear which portion of damage involves a comparison
of the respective degrees of negligence :
• Before Jones case:
AD accepted that once plaintiff’s degree of negligence had been established, it’s
unnecessary to establish defendant’s.
e.g. if plaintiff = 40%, then defendant = automatically 60%.
• Jones case:
AD said if Plaintiff was 30% negligent, it doesn’t automatically means defendant was
70% liable. To establish degree of negligence, carefulness of conduct of each party
must be measured separately against standards of reasonable person.
If defendant raises defence of contributory negligence on part of plaintiff, he (defendant)
has the onus to prove on a balance of probabilities.

Fault in respect to damage or the damage-causing event :


Question is whether section 1 applies where plaintiff wasn’t negligent with regards to
damage- causing event but where his negligence increased the damages?
King case : no helmet didn’t constitute contributory negligence. Didn’t cause damage-
causing event = full damages.
Vitoria case : no seatbelt = contributory negligence. It increased damages = no claim.
Uijs case: no seatbelt because refused to wear = contributory negligence. Increased
damages
= no claim.
So here both X & Y were negligent. X didn’t wear seatbelt and thus increased damages.
Y can rely on contributory negligence. Thus X can only claim R6000 for damages.

(10)

7. X and Y are involved in a car accident; they are both driving their own cars. The accident
occurred on a straight stretch of road with 2 lanes. The accident was a head on collision.
X had been traveling in an easterly direction and Y in a westerly direction. However at the
moment of impact both vehicles are in the lane moving in the westerly direction.

(a) Was X negligent? Discuss with reference to case law. (5)

Negligence judged in the light of the surrounding circumstances: it must be self-


evident that the negligence of conduct may only be evaluated in light of all the
relevant circumstances of a particular case.
In the facts there were no witnesses to the collision thus presumption = res ipsa
loquitur – creates a presumption of negligence:
Contended that this doctrine creates a presumption of negligence, but current
position is that there’s no shift of the onus of proof in such a case. Defendant may
still submit evidence to show that what happened was not due to any negligence on
his part.
Here X is negligent because of this doctrine because his car is in the wrong lane.

(b) Would it make any difference to your answer if X had swerved to the wrong side of
the road because a well built man had suddenly appeared on the sidewalk and
struck at X’s car with a panga? Discuss with reference to case law. (5)

Yes, it would make a difference to my answer :


Here the defence of X is that of doctrine of sudden emergency and is a ground of
justification for negligence.
Doctrine of sudden emergency = when a person has to take a decision in a situation
of sudden emergency and there is insufficient opportunity to consider all the
consequences of his actions – this factor MUST be taken into account in deciding
whether he is negligent.
Requirements which must be met in this doctrine for a defendant’s action not to
amount to negligence – name and discuss.
Here, X’s conduct didnot amount to negligence, because reasonable person would
have taken the same steps as X.

8. X drives his car late at night. Suddenly a well-built man appears in front of him and strikes
out at the car with a panga. X gets a fright and swerves; he collides with Y’s parked car.
In an action for damages by Y, X avers that in view of the circumstances of the case, he
didn’t act negligently.

(a) Describe the test for negligence and mention the most well known case in which the
test was formulated. (4)

Test for negligence was created in Kruger v Coetzee = reasonable person in


the position of defendant :
1. Would foresee the reasonable possibility of his conduct injuring another &
causing him patrimonial loss.
2. Would take reasonable steps to guard against such
occurrence And defendant failed to take such steps.

(b) Discuss the defence raised by X and the requirements for a successful reliance
thereon. (6)

Defence of X = doctrine of sudden emergency – is a ground of justification for


negligence. Doctrine of sudden emergency = when a person has to take a
decision in a situation of sudden emergency and there is insufficient opportunity
to consider all the consequences of his actions – this factor must be taken into
account in deciding whether he is negligent. Requirementswhich must be met in
this doctrine for a defendant's action not to amount to negligence:
1. The wrongdoer must have faced a situation of imminent peril. (=not
negligence)

2. The wrongdoer mustn’t have caused the situation through his negligence –
where a reasonable person would in any event have prevented the emergency
situation – wrongdoer’s conduct won’t be excused (Brown v Hunt).

3. The wrongdoer mustn’t have acted in a grossly unreasonable manner –


established whether a reasonable person in the same circumstances would
have made the same error in judgment as the wrongdoer.
Here, X’s conduct didn’t amount to negligence, because the reasonable person
would have taken the same steps as X.

9. Imagine one claimant and one defendant in a set of facts. Assume that the defendant’s
conduct complies with all the elements of delict and that the claimants conduct with regard
to his damage is also blameworthy.

Briefly discuss in each one of the following situations the effect of contributory fault on
an action for damages instituted by the claimant against the defendant.
(a) Where both the claimant and the defendant are negligent. (4)

If plaintiff and defendant acted negligently, their damages must be apportioned.


Jones case: AD said if plaintiff is 30% negligent, it doesn’t automatically mean the
defendant is 70% liable.
To establish the degrees of negligence, the carefulness of the conduct of each
party must be measured separately against standards of reasonable person

(b) Where the claimants for of fault is contributory negligence and the defendant acted
intentionally. (1)

Plaintiff will be able to claim full damages.

(c) Where the claimants form of fault is contributory intent and the defendant acted
negligently. (4)

Where plaintiff intentionally contributed towards his own loss, while defendant was
merely negligent – plaintiff forfeited his claim.
Vitoria case : plaintiff didn’t wear a seatbelt and increased her damages = no claim.

(d) Where both the claimant and the defendant acted intentionally with regard to the
formers damage. (1)

Where plaintiff acted with intention and defendant acted with intention = no claim
awarded.

10. X stops at petrol station to have tank of his car filled up. While waiting for a pump
attendant, he sees another person drive off, throwing a cigarette butt out of window. An
explosion takes place and the next moment there is a fire on the driveway, close to one
of the petrol pumps. X realises that the cigarette butt has ignited some petrol that has spilt
there.
Having a quick look around, X sees a hosepipe usually used to wash cars. He opens
the tap widely and sprays water on fire. But the water doesn’t extinguish the fire. Burning
petrol starts to float on top of the water and is driven by the force of water stream from
the hosepipe under a car belonging to Y, another customer at the filling station. Y’s car
catches fire and by the time the pump assistants have managed to extinguish fire with
foam fire extinguishers, Y’s car is badly damaged.

Was X’s conduct wrongful ? Discuss with reference to the test for wrongfulness and a
possible defence X may raise. (10)

Test for wrongfulness


To determine wrongfulness = dual investigation:
1. Was there an infringement of a legally protected interest ?– the act must have
harmful consequences.
2. If the interest has been prejudiced, legal norms are used to determine if it
occurred in a legally reprehensible manner (unreasonable manner) – boni mores
test.

Here X’s conduct was wrongful because it infringed Y’s real right to his property and
the act occurred in an unreasonable manner.
Defence
X may raise the defence of necessity as a ground of justification and may succeed
in this defence.
Necessity = when the defendant is placed in a position by superior force (vis major) that
he is able to protect his interests or those of someone else, only by reasonably violating
the interests of an innocent 3rd party.
Guidelines to determine necessity:
• An actual state of necessity must exist.
• The possible existence of a state of necessity must be determined objectively.
• The state of necessity must be present or imminent.
• Defendant need not only protect his own interests, but can protect the interests of
another.
• Not only life or physical integrity but other interests like prop may be protected
out of necessity.
• A person cannot rely on necessity where he is legally compelled to endure danger
• The interests which are sacrificed must be more valuable than the interest which is
protected.
• The act of necessity must be the only reasonable possible means of escaping
from the danger.
• Whether necessity can justify homicide : R v Dudley = no; S v Goliath = yes;
Van der Merwe = yes
CAUSATION
1. Explain the difference between the two forms of causation and describe briefly the tests
employed to determine whether the mentioned two forms of causation are present in a
given case. Refer to relevant case law. (10)

Causation = causing of damage through conduct – a causal nexus between


conduct and damage is required for delict.
Factual causation
No question of delictual liability if it is proved that the conduct of defendant caused the
damage of the person suffering the harm.
• Conditio sine qua non theory
Also known as the BUT FOR test – according to this, an act is the cause of a
result if the act can’t be thought away without the result disappearing as well.

Criticism of the conditio sine qua no theory:


1. Conditio is based on a clumsy, indirect process of thought that results in
circular logic.
2. Conditio fails in cases of so-called cumulative causation (occurs when more
than 1 act actually causes a consequence) – where both X & Y shoot Z,
eliminate X & death still exists & vice versa.
3. Conditio test isn’t a test of causation because it’s merely an ex post facto way
of expressing a predetermined nexus.

Conditio sine qua non and omissions


General view =conditio test may be applied when one wishes to determine whether
an omission caused a certain consequence.
S v Van As: Police neglected to search for children who had fled into the night and
later died of exposure.
Question was whether their death was caused by the omission to search for them.
Court attempted to test the causal connection between the omission & the death by
asking, whether reasonable search would have prevented the death (add something
instead of eliminating).
According to Van Rensburg : conditio sine qua non test doesn’t offer a solution in
cases of causation by an omission. Conditio requires you to eliminate something in
the mind and not to add something to the facts. Conditio test is not accepted for
omissions.

Determination of a factual (causal) nexus :


Van Rensburg: a causal nexus exists where 1 fact arises out of another – this must be
established according to human experience in general.
Enough for the purposes of factual causation if defendant’s conduct has in any way
contributed to the damage sustained by the plaintiff.

Legal causation
No legal system hold a wrongdoer liable without some limit for the endless chain of
harmful consequences, which his act may have caused – means to limit the wrongdoer’s
liability.

• Question of legal causation arises when determining which harmful consequence


actually caused by wrongdoer’s wrongful, culpable act should be held liable for.
• In most cases of delict, harm for which the wrongdoer is to be held liable falls within
the limits of his liability, so that it’s unnecessary to examine legal causation.
• Normally legal causation is only a problem where a whole chain of remote
consequences result from the wrongdoer’s conduct and where it’s alleged that he
should be held liable for all the consequences.
• Limit of liability should be determined in respect of every delictual claim and fact that
this determination in most cases need not be made expressly shouldn’t lead one to
believe that legal causation is relevant only in exceptional cases.
Alston v Marine Trade Insurance : plaintiff suffered brain injuries from car accident due
to negligence of driver insured by defendant. As result from brain injuries, plaintiff suffered
from manic depression for which he was treated. According to medical knowledge at time,
there was no danger thought of by taking the medicines. Plaintiff ate cheese after taking
medicines and had a stroke. Appeared that after taking medicine, eating cheese can lead
to death.
Mafesa : plaintiff broke his leg in car accident because of negligence of driver who was
insured by defendant. He was discharged from hospital & slipped on his crutches & broke
leg again. Hismedical expenses increased.

Question is what criteria should be used to determine legal causation ?


• Flexible approach
S v Mokgethi: held there is no single criteria for legal causation.
Basic question for flexible approach was whether there was a close enough
relationship between wrongdoer’s conduct & its consequences, for such
consequences to be imputed to wrongdoer in view of policy considerations based
on reasonableness, justice and fairness.
Approach confirmed in Bentley, Smith v Abrahams and Collett.

• Adequate causation
It’s adequate if according to human experience in the normal course of events the
act hasthe tendency to bring about that type of consequence.

• Direct consequence
An actor is liable for all the direct consequences of his negligence conduct –
liability isn’t limited to foreseeable consequences.

• Theory of fault
Wrongdoer is liable only for those consequences in respect of which he had fault.
Liability must be ltd to consequences willed by a person while aware of their
wrongfulness and wrongful consequences that he should have foreseen and
prevented.

In terms of this theory:


➢ Intention in the form of dolus eventualis is present.
➢ Factual causation is present
➢ Applying the fault theory – X foresaw the possibility would lead to death and
reconciled himself to it, while knowing it was wrongful.
➢ Therefore – because he has intention, he is liable and there is no need to
look into legal causation
➢ Majority of jurists argue that it’s unreasonable in the circumstances to hold
X liable for Y’s death.

Brown v Hoffman : defendant punched plaintiff in face few times at political


meeting. Plaintiff suffered severe head injuries and claimed for:
1. Infringed dignity
2. Injuries to head, face & brain
3. Medical expenses
4. Pain & Suffering
5. Loss of amenities of life
Hoffman relied on theory of fault to try and limit extent of his liability. Argued on
behalf of defendant that liability was limited to extent of defendant’s intention.
Defendant hadn’t intended causing serious injuries it was argued that he couldn’t be
held liable for damages falling outside his intention and compensation for general
damages like pain and suffering couldn’t be claimed.
Negligence as criteria for legal causation – cannot serve as test for determining of
damage.
Test for negligence = whether reasonable in position of wrongdoer would have
foreseen and prevented situation.
Reasonable foreseeability therefore liable for hospital expenses..

• Novus actus interveniens


New intervening act – an independent event which, after the wrongdoer’s act has
been concluded, either caused or contributed to the consequences concerned.
Where novus actus extinguished causal nexus between conduct of wrongdoer’s act
and consequences, with result that wrongdoer’s act can no longer be considered
factual cause of consequences = actor goes free.
More difficult to determine when new act influenced the result to such an extent that
result should no longer be imputed to actor, although conduct remains factual cause
of result = flexible approach to legal causation.
Novus actus is to limit liability of wrongdoer and plays an important role in legal
causation. In each case will have to be determined within framework of the relevant
imputability test whether novus actus has had the effect of severing the legal nexus
with result that consequences shouldn’t be imputed to actor.
Novus actus might be brought about by conduct of plaintiff himself, by culpable
conduct of 3rd party or by vis major.
An event will qualify as a novus actus only if event wasn’t reasonably foreseeable.
Mokgethi :question was whether victim’s own conduct had broken causal nexus
between accused’s wrongful act and result in question.
Court suggested a criterion, which may be useful to determine legal causation in
view of this victim’s own conduct:
1. Victim fails to seek medical attention and that is cause of death.
2. Wound wasn’t life threatening
3. Where failure to seek medical attention in unreasonable.

4. Mrs. M, deep in thought, walks on the side walk. Too late she notices the presence of a
huge hole in the middle of the sidewalk. She falls into the hole and fractures her hip. It
transpires that the hole was dug by municipal workers to implement a new sewerage
system. Mrs.M wants to recover damages from the municipality. The only precautionary
measure taken by the municipality to protect public was to cordon off the hole with a single
red and white ribbon.

Assume Mrs.M’s hospital costs amount to R10000. This is not the end of her misery. After
being discharged, she slips while walking on her crutches and falls and breaks her arm.
Her medical costs for her broken arm amount to R2000. Mrs. M also wishes to claim this
further amount from the municipality but the municipality denies that it caused her second
set of damages of R2000. Discuss with reference to case law. (10)

Municipality = legal causation of Mrs.M’s damages and is liable for


R10000 for her hip injury. But R2 000 for broken arm constitutes a novus
actus interveniens.
This accident wasn’t reasonably foreseeable thus the municipality is not
liable to R2 000, only for R10 000.
5. "X's conduct is a condition without which Y's damage would never have arisen."
Discuss this statement critically. (4)

Conditio sine qua non theory: also known as the BUT FOR test = an act is the cause of
a result if the act can’t be thought away without the result disappearing as well.
Falls under factual causation.
6. What is the criterion to determine legal causation ? Discuss. (6)

Flexibility approach
S v Mokgethi : held there is no single criteria for legal causation.
Basic question for flexible approach was whether there was a close enough relationship
between wrongdoer’s conduct & its consequences, for such consequences to be imputed
to wrongdoer in view of policy considerations based on reasonableness, justice and
fairness.
Bank teller shot in back by robber. Now paraplegic in wheelchair. Went back to work few
months later. Doctor told him to shift his bum. He failed to do this and died of pressure
sores 6 months after the shooting. AD held that wounding of deceased couldn’t be
regarded as legal causation of death.
Approach confirmed in Bentley, Smith v Abrahams &
Collett. Adequate causation
It’s adequate if according to human experience in the normal course of events the act
has the tendency to bring about that type of consequence.
Direct consequences
An actor is liable for all the direct consequences of his negligence conduct – liability isn’t
limited to foreseeable consequences.
Theory of fault
Wrongdoer is liable only for those consequences in respect of which he had fault.
Liability must be limited to consequences willed by a person while aware of their
wrongfulness and wrongful consequences that he should have foreseen and prevented.

7. Discuss in detail the importance of the decision in S v Mokgethi in respect of factual


and legal causation. Do not discuss the facts of the case. (10)

S v Mokgethi is an AD case involving factual and legal causation. Although it is a


criminal case, the principles set out by Van Heerden are also of importance to the law of
delict.
Approach set out in Mokgethi has subsequently been accepted by the Appeal Court in
a delict case – International Shipping Co.
The Mokgethi case can be considered as authority for the following views:
• Factual and legal causation must be distinguished from each other
• Wrongfulness and fault cannot function as criteria for legal causation
• In the determination of legal causation, none of the existing criteria for legal
causation can be applied satisfactorily in all cases. Therefore, a flexible approach
should rather be adopted. The key question is whether a close enough relationship
exists between the defendant’s act and its consequence for the consequence to be
imputed to the defendant, taking considerations of policy into account on grounds of
reasonableness, fairness and justice. The existing tests of legal causation can play
a subsidiary role in the determination of legal causation based on the flexible
criterion.
• Considerations that are at issue in determining legal causation can differ from one
area of law to another. Consequently, the same factual situation might lead to for
example, delictual liability but not necessarily to, for example, criminal liability.
• In some cases the victim’s intervening act can sever the legal causal connection
between the factual cause and result.

8. Explain the importance of the decision in S v Mokgethi 1990 (1) SA 32 (A) for the
element of causation. (10)

Mokgethi is factual cause of deceased’s back injury, because ‘but for’ him shooting
deceased, deceased would not have been in a wheel chair nor had to shift his bum to
prevent getting pressure sores.
Legal causation : no legal system hold a wrongdoer liable without some limit for the
endless chain of harmful consequences, which his act may have caused – means to limit
the wrongdoer’s liability.
Mokgethi was legal cause for deceased having to be in a wheelchair.
But Mokgethi was not legal cause for deceased’s death – between the shooting and the
dying a ‘new intervening act’ occurred.
Deceased didn’t shift his position and died of pressure sores on his bum this
constituted a
novus actus.

DAMAGE
1. Define ‘damage’. (2)

Damage is the diminution (reduction), as a result of a damage-causing event, in the utility or


quality of a patrimonial or personality interest in satisfying the legally recognised needs of the
person involved.

2. What is meant with the ‘plaintiff’s duty to mitigate loss’ ? Explain in one or two
sentences. (2)

It is a principle of the law of delict that a plaintiff may not recover damages for a loss which
is a factual result of the defendant’s conduct, but which could have been prevented if the
plaintiff had taken reasonable steps.

3. Name two methods by which patrimonial loss and the extent thereof are determined,
and explain the difference between the two methods. (10)

Sum-formula approach or the comparative approach


Loss or damage is the negative difference between the injured person’s financial position
or patrimony and is hypothetical financial position or patrimony in which he would have
been if the damage-causing vent did not take place.
Concrete approach
Damage is the difference between the patrimonial position before the delict and
thereafter. The position after the delict is compared with the position before the delict.

The difference between these two approaches = the sum-formula approach uses a
hypothetical position that would have existed had the delict not been committed,
whereas such a hypothetical position is absent in the case of the concrete approach.

4. What criterion is used to determine whether an act, causing pure economic loss is
wrongful ? Explain in one short sentence. (1)

The criterion used is breach of a legal duty.

5. What is meant by the ‘sum-formula approach’ ? Explain briefly. (2)

Comparative test: in terms of this approach damage exists in the negative difference
between the plaintiff’s actual patrimonial position as it exists after the damage-causing
event & the hypothetical patrimonial position that would have existed had the damage-
causing event not occurred = current position compared to a hypothetical position

6. What is meant by the ‘once-and-for-all rule’ ? (3)

In claims for compensation and satisfaction arising out of a delict, plaintiff must claim
damages for all the damages already sustained or expected in the future in so far as it’s
based on a single cause of action

7. What is meant by the ‘duty to mitigate’ ? (1)

Plaintiff can’t recover damages for a loss which is the factual result of the defendant’s
conduct, but which could have been prevented if the plaintiff had taken reasonable steps.
A failure to ensure that damages don’t accumulate can be seen as a failure by the
plaintiff to take reasonable steps to limit the initial loss or an omission to prevent further
damages.
In other words, the plaintiff must try to keep the damages low during the waiting period.

8. Define prospective damage. (3)

Patrimonial damage = damage in the form of patrimonial loss which will, with a degree of
possibility, materialise after the date of assessment of damages resulting from an earlier
damage causing event.

9. Mrs. M, deep in thought, walks on the side walk. Too late she notices the presence of a
huge hole in the middle of the sidewalk. She falls into the hole and fractures her hip. It
transpires that the hole was dug by municipal workers to implement a new sewerage
system. Mrs.M wants to recover damages from the municipality. The only precautionary
measure taken by the municipality to protect public was to cordon off the hole with a single
red and white ribbon.
Assume that Mrs. M is informed by her doctor that due to her hip injury, a possibility
exists of her having to undergo a further operation sometime in the future. At what stage
must Mrs.M claim the compensation from the municipality for such an operation ? (3)

Once and for all rule = in claims for compensation & satisfaction arising out of a delict the
Plaintiff must claim damages for all the damages already sustained or expected in the
future in so far as it’s based on a single cause of action.
Mrs. M must claim the compensation now because she won’t be able to claim
it later. Prescription for the claim of damages = 3years

10. It has been suggested that our law should adopt a concrete approach to damages,
except in cases of prospective loss, liability for misrepresentation and loss of profit.

Answer the following questions :

(a) What is the concrete approach to damages? (1)

Difference between the patrimonial position of the plaintiff before the unlawful act
and the actual position after.

(b) What approach to damages should be accepted in the case of prospective loss,
liability for a misrepresentation and loss of profit ? Give the name of this approach
and explain. (2)

Prospective patrimonial damage = damage in the form of patrimonial loss which


will, with a degree of possibility, materialise after the date of assessment of damages
resulting from an earlier damage causing event.
Approach to be used = once and for all rule
In claims for compensation & satisfaction arising out of a delict the plaintiff must
claim damages for all the damages already sustained or expected in the future in so
far as it’s based on a single cause of action.

14. Name the TWO forms of damages and give an example of each. (2)

Patrimonial damages e.g. loss of income.


Non-patrimonial damages e.g. infringement of dignity.
DELICTUAL REMEDIES
1. Name THREE requirements for an interdict. (3)

There must be an act by the


respondent The act must be wrongful
No other remedy must be available.

2. Which remedy will a person institute to avert an impending wrongful act? (1)

Interdict

JOINT WRONGDOERS
1. Due to the negligent conduct of both X and Y, Z’s garden shed burns down. Z sues X in
delict for the full amount of his damage. X approaches you for legal advice. Advise X,
referring to relevant legislation. (5)

Joint wrongdoers = persons who are jointly and severally liable in delict for the same
damages.

SPECIFIC FORMS OF DAMNUM


INIURIA DATUM
1. Explain the concept ‘pure economic loss’ with reference to case law. (5)

Aquilian action is available to claim damages for pure economic loss.

Pure economic loss may compromise patrimonial loss that doesn’t result from damage
to prop or impairment to personality.
Greenfield Engineering Works : where X suffered financial loss because a cheque that
was sent to him by B was stolen and as a result of the fact that the cheque hadn’t been
crossed, the thief was able to open a bank account, withdraw the money and disappear
with it.
Pure economic loss may also refer to financial loss that does flow from the damage to
property or impairment to personality, but which doesn’t involve the Plaintiff’s property or
person.
Coronation Brick : X’s negligence damaged cables that provided electricity to B’s factory
and B suffered loss of production.

Pure economic loss including patrimonial loss that’s the result of damage to property
or injuryto personality but where the defendant didn’t cause the loss or injury.
Kadir v Minister of Law &Order:as a result of the negligence of X, B’s car left the road
and he was injured. Two constables who arrived on the scene failed to take any
particulars concerning the identity of X and as a result of the omission, B couldn’t claim
from the MMF for his damage and suffered pure economic loss.

2. What criterion is used to determine whether an act, causing pure economic loss is
wrongful ? Explain in one short sentence. (1)

The criterion used is breach of a legal duty.

3. Write notes on the way in which Bester v Commercial Union influenced delictual liability
for causing emotional shock. (6)

Bester v Commercial Union : AD said that the impairment of personality and patrimonial
loss resulting from psychiatric injury or emotional shock caused wrongfully and
intentionally or negligently, founds the action for the pain and suffering and the actio legis
Aquiliae.
Before Bester
SA law of delict lacked clear principles in this field. This resulted in restrictions on liability
for emotional shock:
1. The shock must have originated from a physical injury– wrongfulness.
2. The aggrieved party himself must have been in personal danger of being physically
injured – negligence or legal causation.
These were rejected in Bester case.

After Bester
Following factors play a role in this regard:
• The fact that the psychological lesion resulted from physical injury.
• The fact that the plaintiff was in personal danger of being physically injured.
• The fact that the plaintiff was informed of the death or injury of a close relative
(jokingly).
• The fact that the plaintiff personally witnessed the death or injury of a close relative.

Once the court has found that serious emotional shock was reasonably foreseeable, the
wrongdoer is then liable for any detrimental physical or mental consequences –
regardless of whether they were foreseeable as well (talem qualem rule applies – you
take your victim as you find him).

4. State FOUR factors that play a role in determining whether emotional shock was
reasonably foreseeable by the defendant. (4)
Fact that the psychological lesion resulted from physical injury.
Fact that the plaintiff was in personal danger of being physically injured.
Fact that the plaintiff was informed of the death or injury of a close relative (jokingly).
Fact that the plaintiff personally witnessed the death or injury of a close relative.

5. Briefly explain the so-called ‘thin skull’ (talem qualem) rule. (2)

A defendant cannot escape liability by proving that the plaintiff was particularly
susceptible tothe prejudicial consequences of the shock and that the consequences were
therefore not reasonably foreseeable.

6. Two brothers walk across the street. A car skips the red robot and instantly kills one of
them. As a result of witnessing his brother’s death, the surviving brother suffers severe
psychological disturbances and has to go for treatment to a therapist.

(a) Which TWO aspects must the surviving brother prove in order to claim for the
psychological disturbances suffered by him? (2)

Wrongfulness
Intention or negligence.

(b) Name the case which provides authority for your answer above. (1)

Bester v Commercial Union.

(c) With which remedy can the surviving brother claim for the injuries suffered ? (1)

Actio legis Aquiliae.

(d) Which form of fault is required for the remedy in (c) above ? (1)

Negligence.

(e) Must the surviving brother prove that he suffered physical injuries in order to claim
for psychological disturbances suffered by him ? (2)

No, the requirement of physical harm – infringement to the right to physical integrity,
which is wrongful, was rejected in Bester. According to Bester, the brain and
nervous system are as much a part of the physical body as an arm and a leg. As
result, a physical injury isnot absolutely necessary to found liability.

7. Name the case that is the locus classicus in SA law for the recognition that personality
injury and patrimonial loss resulting from emotional shock are actionable.
Bester v Commercial Union Versekeringsmaatskappy

8. Why did the court in the abovementioned case reject the requirement that the plaintiff
should have suffered physical harm before he can institute a claim based on emotional
shock ? Describe the court’s reasoning in one sentence. (1)

According to the court, the brain and nervous system are as much a part of the physical
body as an arm or a leg.

9. With what new requirement did the court substitute the ‘physical injury’ requirement ?
Describe briefly. (1)

To be actionable, the harm caused by the shock must be reasonably serious.

10. The court also rejected the requirement that the plaintiff should have been in personal
danger before he/she could claim for emotional shock. With what requirement did the
court replace the ‘personal danger’ requirement ? Describe briefly. (1)

The yardstick of reasonable foresees ability of harm, that is, was the emotional shock
reasonably foreseeable.

DAMNUM INIURIA DATUM


1. Name FIVE specific forms of damnum iniuria datum. (2½)

Injury or death of another


Negligence
misrepresentation
Interference with a contractual
relationship Unlawful compensation
Manufacturer’s liability
SPECIFIC FORMS OF INIURIA
1. When does privilege exist? (2)

When someone has a right, duty or interest to make specific defamatory assertions and
the people to whom the assertions are publicised have a corresponding right, duty or
interest to learn of the assertions.

2. Distinguish between relative and absolute privilege and give an example of each. (3)

Relative privilege
Defendant enjoys conditional protection – protection falls away as soon as the plaintiff
proves the defendant exceeds the bounds of privilege.

Absolute privilege
Defendant is protected absolutely, in the sense that liability for defamation is excluded.
e.g. parliament

3. Briefly discuss the grounds excluding fault in the case of defamation with reference to
case law. (3)

National Media v Bogoshi – court introduced negligence as the basis of liability of the
press for defamation.
Grounds excluding fault
1. Mistake: persons are unaware of the wrongfulness of his defamatory publication
because he bona fide believes that his conduct is lawful.
2. Jest: if the defendant proved that he published the defamatory words in jest, he
should be able to rebut the presumption of animus (intention).

4. What is the general criterion which is used to determine whether a negligent


misstatement is wrongful ? (1)

The only relevant question is if in the opinion of the reasonable person with normal
intelligence and development the reputation of the person concerned has been injured.

5. Identify the types of personality infringement that B might have suffered in each of the
following situations : (3)

(a) A calls B a liar when they are alone.

Right to dignity and feelings.

(b) A, B’s doctor, tells a friend that B has AIDS

Right to privacy and good name.


(c) A is a 2nd handcar dealer. He places an advert in the newspaper with the photo
of a car he wants to sell. B, a well known soccer player, who happened to be at the
dealer to look at the cars when the photo was taken, also appears in the photo.

Right to privacy.

6. What is the importance of National Media Ltd v Bogoshi for the law of
defamation? (10)

Animus iniuriandi (intention)


This is accepted as a requirement for defamation – negligence is therefore insufficient
to hold the wrongdoer liable.
National Media v Bogoshi : court introduced negligence as the basis of liability of the
press for defamation.
Grounds excluding fault:
• Mistake: person is unaware of the wrongfulness of his defamatory publication
because he bona fide believes that his conduct is lawful.
• Jest: if the defendant proved that he published the defamatory words in jest, he
should be able to rebut the presumption of animus (intention).

7. In which TWO ways can privacy be infringed? (2)

When you reveal something private of the person to a 3 rd


party. When you are a peeping tom.

8. Identify the grounds of justification that a defendant in a defamation case may use in
each of the following situations and substantiate your answer :

(a) T is found guilty of a terrible crime. During his judgment the Magistrate calls him a
‘scumbag’ and a ‘lowlife’. T wants to sue the magistrate for defamation. (2)

Magistrate can use the ground of justification known as fair


comment. Requirements :
1. Defamation must amount to comment.
2. Comment must be fair.
3. Facts on which the comment is based must be true.
4. These facts must be in public’s interest.

9. Identify the grounds of justification that may be available to a defendant in a defamation


case in each of the following situations :

(a) A and B are members of the town council. Both are running for the office of mayor.
At a town meeting arranged by A, B reveals that A was expelled from university
many years ago because he was found guilty of dishonesty during an exam. A wants
to sue B for defamation. (1)

Here B can use the grounds of justification known as truth and public interest.
The prima facie wrongfulness of the B’s conduct will be cancelled if he proves that
the remarks were true and in the public’s interest. B only has to prove that the
remarks are substantially true.

10. X, Y’s father finds out that Y’s fiancé, V, was in jail before he met her. X tells his
daughter this. V accuses X of defaming him.

Which personality interest of V was infringed ? (1)

His right to his good name.

11. Write brief notes on the determining of wrongfulness in a defamation case. (4)

Wrongfulness with respect to the defamation lies in the infringement of a person’s right to
his good name. When determining wrongfulness the question is whether the good name
of the person involved has in fact been infringed is irrelevant.
The only relevant question is if, in the opinion of the reasonable person with normal
intelligence and development, the reputation of the person concerned has been injured.

Following principles apply in respect of this test:

Reasonable person is a fictional, normal, well balanced right thinking person.


Verbal is in most cases is not defamatory because it normally doesn’t injure a person’s
good name.

Words are prima facie of according to the ordinary meaning defamatory or not.

12. Which personality interests are infringed by the following delicts respectively ?

(a) Defamation (½)


(b) Insult (½)
13. Does the publication of defamatory words, according to the courts, take place in the
following circumstances ?

(a) Two Japanese tourists in SA quarrel and in front of a group of South Africans, the
one calls the other a liar and a thief in their mother tongue. (1)

No, because courts don’t consider the disclosure of defamatory words or behaviour
to an outsider who’s unaware of the defamatory character or meaning thereof in
relation to the plaintiff as publication.

(b) Mr. X tells his wife that Z, who works in his office with him, stole some
money. (1)

No, because the communication of defamatory words concerning a 3rd party by one
spouse to another doesn’t constitute publication.

FORMS OF LIABILITY
WITHOUT FAULT
1. Name the THREE requirements for a successful actio de pastu. (3)

Defendant must be the owner of the animal when the damage is


caused Animal must cause damage by eating plants
Animal must act of its own volition when causing damage

2. Define vicarious liability.

Strict liability of one person for the delict of another. The former is indirectly vicariously
liable for the damage caused by the latter.

3. Name THREE requirements for the vicarious liability of an employer. (3)

There must be an employer – employee relationship at the time when the delict is
committed. Employee must commit a delict.
Employee must act within the scope of his employment when the delict is committed.

4. B plans to visit her friend C, who is looking after A’s house. C tells B that there is a vicious
dog on the premises, but promises to lock it up before B arrives. B arrives at the appointed
time. As she enters the premises, the dog appears from behind the house and bites her
leg. Answer the following questions:

(a) What action is available to B in order to recover her damage ? (1)


Actio de pauperie.

(b) Against whom does she institute the action mentioned above ? (1)

Against the owner A.

5. Name FOUR defences that can be raised against the actio de pauperie. (4)

Vis major
Culpable or provocative conduct on part of prejudiced person
Culpable conduct on part of outsider
Provocation by another animal
Voluntary assumption of risk

GENERAL
1. X takes his usual afternoon walk on the footpath on the mountain slope behind the
picturesque town in which he lives. Out of the corner of his eye he notices a huge rock
rolling down the mountain slope in his direction. He gets a fright and swiftly jumps away
in an attempt to avoid being crushed by the rock. In the process he kicks to pieces an
almost completed painting that Y, a famous artist, has been peacefully working on.
However, the rock crosses the footpath harmlessly, several metres away from the spot
where X would have been had he not left the path. Y is very upset about the destruction
of his painting, which he was planning to sell for at least R1 0 000, and wishes to institute
a delictual claim against X.

Are you of the opinion that Y will be able to hold X delictually liable? Discuss. (20)

Because the question does not confine itself to a specific delictual element, it may be
necessary to consider all five elements of delictual liability.
(Note that, for reasons of space and time, examination questions will usually be more
limited in their scope, e.g. asking whether X acted wrongfully, or whether he had been
negligent, etc.)

Act
An act or conduct is defined as a voluntary, human act or omission. It is clear that an act
is present in the given set of facts. The only question is whether this act was voluntary.
Voluntariness means that the act must have been susceptible to control of the will of the
person involved. This does not mean that the person must have willed or desired his
conduct; but simply that he must have been able to control the act. If the act was not
susceptible of control by the will, the defendant can evade liability by raising the defence
of automatism. In the case under discussion, it is highly improbable that X would convince
the court that he acted involuntarily. Impulsive and spontaneous acts do not really amount
to reflex movements and are usually regarded as voluntary. We can conclude that X acted
for the purposes of the law of delict.

Wrongfulness
Now that we have established that X had acted, we must investigate whether his act was
wrongful. The basic criterion of wrongfulness is the boni mores. Two more concrete
applications of the boni mores are respectively the infringement of a subjective right, and
breach of a legal duty, as tests for wrongfulness. A painting is an object of subjective
rights. Apart from the right of ownership (a real right) to the physical canvas and paint,
the painter also has copyright (an immaterial property right) in respect of the artwork as
intangible product of the human mind and endeavour. Infringement of a subjective right
consists of two elements. First a factual disturbance of the subject-object relationship
must have taken place. This means that the alleged wrongdoer must in fact have
interfered with the powers of the legal subject to use, enjoy and dispose of the object of
his right. In the given case, this has indeed happened: As a result of X's conduct the
painter is unable to use, enjoy and dispose of his painting as before. Secondly, this
disturbance must have been caused in violation of a norm, that is, in an unreasonable
or legally impermissible manner. This is established with reference to the boni mores,
the legal convictions of the community. A factual disturbance of the subject-object
relationship will, in the absence of a ground of justification, usually violate a norm.
Therefore one can conclude that unless X can successfully raise a ground of justification,
his conduct was wrongful.

Is any ground of justification available to X ? Necessity exists when the defendant is


placed in such a position by superior force that he is able to protect his own interests (or
those of someone else) only by reasonably violating the interests of an innocent third
party. Whether necessity exists, must be determined objectively. The circumstances
which actually prevailed and the consequences that actually ensued must be considered.
The question is whether a state of necessity actually existed, not whether the defendant
believed it to exist. In the given set of facts it transpires that the rock would not have
fallen on the path where X had been walking. Therefore he was not objectively in danger
even though he subjectively believed he was. His reliance on necessity will therefore fail,
and we can conclude that he acted wrongfully.

Fault
Having now determined that X acted wrongfully, this does not necessarily imply that he
will be liable. In order to complete the answer we need to consider further delictual
elements. Next in line after wrongfulness is fault: Fault may be present in two forms: intent
and negligence. A prerequisite for both forms of fault is accountability. If the alleged
wrongdoer was unaccountable, he cannot be at fault. A person is accountable if he has
the mental abilities to distinguish between right and wrong and to act in accordance with
this appreciation. Factors like youth, mental disease, extreme intoxication, etc. may render
a person unaccountable. There is no indication in the given facts that any such factor is
present, and we may therefore accept that X was accountable and could therefore be at
fault.

Did X act with intent ? A person acts intentionally if his will is directed at a result which
he causes while conscious of the wrongfulness of his conduct. It is very unlikely that X
directed his will at destroying Y's painting. Furthermore, it is even more unlikely that X
was aware of the wrongfulness of his conduct. He probably had no time to consider this,
and even if he did, he probably thought he was entitled to get away from the rock as
quickly as possible to save his own life. The test for intent is subjective (unlike the
objective test for wrongfulness). What X was thinking (namely that his life was in danger)
is thus more important when we consider intent than the objective reality (namely that he
was, in fact, in no danger). Therefore, X did not act with intent.

If X did not act intentionally, he may still have been negligent, and thus negligence is our
next inquiry. According to Kruger v Coetzee a person is negligent if the reasonable
person in his position would have foreseen damage arising from his conduct and would
have taken steps to prevent that harm, and the defendant (or alleged wrongdoer) failed
to take such steps. Negligence is determined with reference to the circumstances of
the particular case.

Determining negligence in a situation like the one in the question is usually referred to
as an application of the so-called "doctrine of sudden emergency". The principle
accepted here is that the law cannot expect a person who has to act swiftly in a situation
of imminent peril to show the same judgment and skill as a person who is not acting
under such circumstances. An error of judgment made in such a situation will therefore
not necessarily amount to negligence.

Our case law contains three requirements which must be satisfied before conduct in a
situation of sudden emergency will not be regarded as negligent :
1. The wrongdoer must have faced a situation of imminent peril.
2. The wrongdoer must not have caused the perilous situation through his own
negligence or imprudence.
3. The wrongdoer must not have acted absolutely unreasonably.

The principle is that although the reasonable person may make an error of judgment, he
or she will not lose his or her head or act completely foolishly. The application of the
"doctrine of sudden emergency" is therefore not an independent test for negligence, but
simply an application of the reasonable person test in specific circumstances, namely a
situation of sudden danger. Applied to the facts in the question, it appears that the above-
mentioned requirements have been met, and the reasonable person in X's position would
probably have acted in the same manner. X was therefore not negligent. X will therefore
not be liable for V's damage.

You might also like